Sunteți pe pagina 1din 612

www.crackjee.

xyz
EBD_7764
EBD_7764
EBD_7764
EBD_7764
www.crackjee.xyz
EBD_7764
EBD_7764
EBD_7764
EBD_7764
EBD_7764
EBD_7764
EBD_7764
EBD_7764
EBD_7764
EBD_7764
EBD_7764
EBD_7764
EBD_7764
EBD_7764
EBD_7764
EBD_7764
EBD_7764
EBD_7764
EBD_7764
EBD_7764
EBD_7764
EBD_7764
EBD_7764
EBD_7764
JEE MAIN SOLVED PAPER–2018
PHYSICS
m T
1. The density of a material in the shape of a cube m
m2
2
is determined by measuring three sides of the
cube and its mass. If the relative errors in
measuring the mass and length are respectively T
1.5% and 1%, the maximum error in determining
the density is: m1
(1) 2.5% (2) 3.5%
(3) 4.5% (4) 6% m1g
2. All the graphs below are intended to represent (1) 18.3 kg (2) 27.3 kg
the same motion. One of them does it incorrectly. (3) 43.3 kg (4) 10.3 kg
Pick it up. 4. A particle is moving in a circular path of radius a
velocity under the action of an attractive potential
k
position U= - . Its total energy is:
(1) 2r 2
k k
(1) - 2 (2)
distance 4a 2a 2
3 k
(3) zero (4) -
time 2 a2
(2)
5. In a collinear collision, a particle with an initial
speed n0 strikes a stationary particle of the
same mass. If the final total kinetic energy is
position 50% greater than the original kinetic energy, the
magnitude of the relative velocity between the
two particles, after collision, is:
(3) time
n0
(1) (2) 2n0
4

velocity n0 n0
(3) (4)
2 2
6. Seven identical circular planar disks, each of
(4) time mass M and radius R are welded symmetrically
as shown. The moment of inertia of the
arrangement about the axis normal to the plane
and passing through the point P is:
3. Two masses m1 = 5 kg and m2 = 10 kg, connected
by an inextensible string over a frictionless
P
pulley, are moving as shown in the figure. The
coefficient of friction of horizontal surface is 0.15. O
The minimum weight m that should be put on
top of m2 to stop the motion is:
EBD_7764
2018-2 JEE Main Solved Paper–2018

19 55 10. Two moles of an ideal monoatomic gas occupies


(1) MR 2 (2) MR 2 a volume V at 27°C. The gas expands
2 2
adiabatically to a volume 2 V. Calculate (a) the
73 181 final temperature of the gas and (b) change in its
(3) MR 2 (4) MR2
2 2 internal energy.
7. From a uniform circular disc of radius R and mass (1) (a) 189 K (b) 2.7 kJ
R (2) (a)195 K (b) –2.7 kJ
9 M, a small disc of radius is removed as (3) (a)189 K (b) –2.7 kJ
3
shown in the figure. The moment of inertia of (4) (a)195 K (b) 2.7 kJ
the remaining disc about an axis perpendicular 11. The mass of a hydrogen molecule is 3.32×10–27
to the plane of the disc and passing, through kg. If 1023 hydrogen molecules strike, per
centre of disc is : second, a fixed wall of area 2 cm2 at an angle of
45° to the normal, and rebound elastically with a
speed of 103 m/s, then the pressure on the wall
2R is nearly:
3 (1) 2.35 × 103 N/m2 (2) 4.70 × 103 N/m2
(3) 2.35 × 102 N/m2 (4) 4.70 × 102 N/m2
R
12. A silver atom in a solid oscillates in simple
harmonic motion in some direction with a
frequency of 10 12 /sec. What is the force
40 constant of the bonds connecting one atom with
(1) 4 MR2 (2) MR2 the other? (Mole wt. of silver = 108 and Avagadro
9
number = 6.02 ×1023 gm mole –1)
37 (1) 6.4 N/m (2) 7.1 N/m
(3) 10 MR2 (4) MR2
9 (3) 2.2 N/m (4) 5.5 N/m
8. A particle is moving with a uniform speed in a 13. A granite rod of 60 cm length is clamped at its
circular orbit of radius R in a central force middle point and is set into longitudinal
inversely proportional to the nth power of R. If vibrations. The density of granite is 2.7 × 103
the period of rotation of the particle is T, then: kg/m3 and its Young's modulus is 9.27×1010 Pa.
(1) T µ R3/2 for any n.(2) T µ R n /2+1 What will be the fundamental frequency of the
longitudinal vibrations?
(3) T µ R (n +1)/2 (4) T µ R n/2
(1) 5 kHz (2) 2.5 kHz
9. A solid sphere of radius r made of a soft material
(3) 10 kHz (4) 7.5 kHz
of bulk modulus K is surrounded by a liquid in a
cylindrical container. A massless piston of area 14. Three concentric metal shells A, B and C of
a floats on the surface of the liquid, covering respective radii a, b and c (a < b < c) have surface
entire cross-section of cylindrical container. charge densities +s, –s and +s respectively.
When a mass m is placed on the surface of the The potential of shell B is:
piston to compress the liquid, the fractional
s é a2 - b2 ù s é a 2 - b2 ù
æ dr ö (1) ê +c ú (2) ê +cú
decrement in the radius of the sphere çè ÷ø ,is : Î0 ëê a ûú Î0 ëê b ûú
r
Ka Ka s é b2 - c2 ù s é b 2 - c2 ù
(1) (2) (3) ê +a ú (4) ê +a ú
mg 3mg Î0 ëê b Î0 ëê c
ûú ûú
mg mg
(3) (4)
3Ka Ka
JEE Main Solved Paper–2018 2018-3

15. A parallel plate capacitor of capacitance 90 pF is (1) 2 (2) 3


connected to a battery of emf 20V. If a dielectric
1
5 (3) 2 (4)
material of dielectric constant k = is inserted 2
3
20. For an RLC circuit driven with voltage of
between the plates, the magnitude of the
induced charge will be: 1
amplitude vm and frequency w0 = the
(1) 1.2 n C (2) 0.3 n C LC
(3) 2.4 n C (4) 0.9 n C current exhibits resonance. The quality factor, Q
16. In an a.c. circuit, the instantaneous e.m.f. and is given by:
current are given by w0 L w0 R
e = 100 sin 30 t (1) (2)
R L
æ pö R CR
i = 20 sin ç 30 t - ÷ (3) (4)
è 4ø (w0C) w0
In one cycle of a.c., the average power consumed 21. An EM wave from air enters a medium. The
by the circuit and the wattless current are, ur é æ z öù
respectively: electric fields are E1 = E01 xµcos ê2pv ç - t ÷ ú in
ë è c øû
1000 air and
(1) 50W, 10A (2) W, 10A
2 ur
E 2 = E02 xµcos [ k(2z - ct)] in medium, where the
50 wave number k and frequency v refer to their
(3) W, 0 (4) 50W, 0
2 values in air. The medium is nonmagnetic. If Îr1
17. Two batteries with e.m.f. 12 V and 13 V are
connected in parallel across a load resistor of and Îr2 refer to relative permittivities of air and
10 W . The internal resistances of the two batteries medium respectively, which of the following
options is correct?
are 1 W and 2 W respectively. The voltage across
the load lies between: Îr Îr
1 =4 1
(1) 11.6 V and 11.7 V (2) 11.5 V and 11.6 V (1) (2) =2
Îr Îr
2 2
(3) 11.4 V and 11.5 V (4) 11.7 V and 11.8 V
18. An electron, a proton and an alpha particle Îr Îr
1 1 1 1
having the same kinetic energy are moving in
(3) = (4) =
circular orbits of radii re, rp, ra respectively in a Îr 4 Îr 2
2 2
uniform magnetic field B. The relation between
22. Unpolarized light of intensity I passes through
re, rp, ra is : an ideal polarizer A. Another indentical polarizer
(1) re > rp = ra (2) re < rp = ra B is placed behind A. The intensity of light
(3) re < rp < ra (4) re < ra < rp
I
19. The dipole moment of a circular loop carrying a beyond B is found to be . Now another
2
current I, is m and the magnetic field at the centre
identical polarizer C is placed between A and B.
of the loop is B1. When the dipole moment is
doubled by keeping the current constant, the I
The intensity beyond B is now found to be .
magnetic field at the centre of the loop is B2. 8
The angle between polarizer A and C is:
B
The ratio 1 is: (1) 0° (2) 30°
B2 (3) 45° (4) 60°
EBD_7764
2018-4 JEE Main Solved Paper–2018
23. The angular width of the central maximum in a (1) 0 (2) 15 mA
single slit diffraction pattern is 60°. The width (3) mA (4) mA
of the slit is 1 m. The slit is illuminated by 28. A telephonic communication service is working
monochromatic plane waves. If another slit of
at carrier frequency of 10 GHz. Only10% of it is
same width is made near it, Young's fringes can
utilized for transmission. How many telephonic
be observed on a screen placed at a distance 50
channels can be transmitted simultaneously if
cm from the slits. If the observed fringe width is
each channel requires a bandwidth of 5 kHz?
1 cm, what is slit separation distance?
(i.e. distance between the centres of each slit.) (1) 2 × 103 (2) 2 × 104
(3) 2 × 10 5 (4) 2 × 106
(1) 25 m (2) 50 m
(3) 75 m (4) 100 m 29. In a potentiometer experiment, it is found that
24. An electron from various excited states of no current passes through the galvanometer
hydrogen atom emit radiation to come to the when the terminals of the cell are connected
ground state. Let n, g be the de Broglie across 52 cm of the potentiometer wire. If the
wavelength of the electron in the n th state and cell is shunted by a resistance of 5 , a balance
the ground state respectively. Let n be the is found when the cell is connected across 40
cm of the wire. Find the internal resistance of the
wavelength of the emitted photon in the
cell.
transition from the nth state to the ground state.
For large n, (A, B are constants) (1) 1 (2) 1.5
(3) 2 (4) 2.5
B
(1) n A+ (2) A+B 30. On interchanging the resistances, the balance
2 n n
n point of a meter bridge shifts to the left by 10 cm.
The resistance of their series combination is 1k .
(3) 2 2 (4) 2
n A+B n n How much was the resistance on the left slot
25. If the series limit frequency of the Lyman series before interchanging the resistances?
is v1, then the series limit frequency of the P- (1) (2)
fund series is : (3) (4)
(1) 25 L (2) 16 L
(3) L/16 (4) L/25 CHEMISTRY
26. It is found that if a neutron suffers an elastic 31. The ratio of mass percent of C and H of an
collinear collision with deuterium at rest, organic compound (CXHYOZ) is 6 : 1. If one
fractional loss of its energy is pd; while for its molecule of the above compound (CXHYOZ)
similar collision with carbon nucleus at rest, contains half as much oxygen as required to burn
fractional loss of energy is Pc. The values of Pd one molecule of compound CXHY completely to
and Pc are respectively: CO 2 and H2O. The empirical formula of
(1) 89, 28 (2) 28, 89 compound CXHYOZ is :
(1) C3H6O3 (2) C2H4O
(3) (4) (3) C3H4O2 (4) C2H4O3
27. The reading of the ammeter for a silicon diode in 32. Which type of ‘defect’ has the presence of
the given circuit is : cations in the interstitial sites?
200 (1) Schottky defect
(2) Vacancy defect
(3) Frenkel defect
(4) Metal deficiency defect

3V
JEE Main Solved Paper–2018 2018-5
33. According to molecular orbital theory, which of 39. At 518°C, the rate of decomposition of a sample
the following will not be a viable molecule? of gaseous acetaldehyde, initially at a pressure
of 363 Torr, was 1.00 Torr s–1 when 5% had
(1) He22+ (2) He+2
reacted and 0.5 Torr s–1 when 33% had reacted.
(3) H2– (4) H 2– The order of the reaction is :
2
(1) 2 (2) 3
34. Which of the following lines correctly show the
(3) 1 (4) 0
temperature dependence of equilibrium constant,
40. How long (approximate) should water be
K, for an exothermic reaction?
electrolysed by passing through 100 amperes
ln K A current so that the oxygen released can
completely burn 27.66 g of diborane?
B 1 (Atomic weight of B = 10.8 u)
(0, 0) T(K) (1) 6.4 hours (2) 0.8 hours
++
++ C (3) 3.2 hours (4) 1.6 hours
++
++ 41. The recommended concentration of fluoride ion
D
in drinking water is up to 1ppm as fluoride ion is
(1) A and B (2) B and C required to make teeth enamel harder by
(3) C and D (4) A and D converting [3Ca3(PO4)2. Ca(OH)2] to :
35. The combustion of benzene (l) gives CO2 (g) (1) [CaF2]
and H2O (l). Given that heat of combustion of (2) [3(CaF2).Ca(OH)2]
benzene at constant volume is –3263.9 kJ mol–1 (3) [3Ca3(PO4)2.CaF2]
at 25°C; heat of combustion (in kJ mol–1) of (4) [3{(Ca(OH)2}.CaF2]
benzene at constant pressure will be : 42. Which of the following compounds contain(s)
(R= 8.314 JK–1 mol–1) no covalent bond(s)?
(1) 4152.6 (2) –452.46 KCl, PH3, O2, B2H6, H2SO4
(3) 3260 (4) –3267.6 (1) KCl, B2H6, PH3 (2) KCl, H2SO4
36. For 1 molal aqueous solution of the following (3) KCl (4) KCl, B2H6
compounds, which one will show the highest 43. Which of the following are Lewis acids?
freezing point? (1) PH3 and BCl3 (2) AlCl3 and SiCl4
(1) [Co(H2O)6]Cl3 (3) PH3 and SiCl4 (4) BCl3 and AlCl3
(2) [Co(H2O)5Cl]Cl2.H2O
(3) [Co(H2O)4Cl2]Cl. 2H2O 44. Total number of lone pair of electrons in I3– ion
(4) [Co(H2O)3Cl3].3H2O is :
37. An aqueous solution contains 0.10 MH2S and (1) 3 (2) 6
0.20 M HCl. If the equilibrium constants for the (3) 9 (4) 12
formation of HS– from H2S is 1.0 × 10–7 and that 45. Which of the following salts is the most basic in
of S2– from HS– ions is 1.2 × 10–13 then the aqueous solution?
concentration of S2– ions in aqueous solution is : (1) Al(CN)3 (2) CH3COOK
(1) 5 × 10–8 (2) 3 × 10–20 (3) FeCl3 (4) Pb(CH3COO)2
(3) 6 × 10–21 (4) 5 × 10–19 46. Hydrogen peroxide oxidises [Fe(CN)6]4– to
38. An aqueous solution contains an unknown [Fe(CN) 6]3– in acidic medium but reduces
concentration of Ba2+. When 50 mL of a 1 M [Fe(CN)6]3– to [Fe(CN)6]4– in alkaline medium.
solution of Na2SO4 is added, BaSO4 just begins The other products formed are respectively:
to precipitate. The final volume is 500 mL. The (1) (H2O + O2) and H2O
solubility product of BaSO4 is 1 × 10–10. What is (2) (H2O + O2) and (H2O + OH–)
the original concentration of Ba2+ ? (3) H2O and (H2O + O2)
(1) 5 × 10–9 M (2) 2 × 10–9 M (4) H2O and (H2O + OH–)
(3) 1.1 × 10–9 M (4) 1.0 × 10–10 M
EBD_7764
2018-6 JEE Main Solved Paper–2018
47. The oxidation states of Cr in [Cr(H2O)6]Cl3, 54. Phenol on treatment with CO2 in the presence
[Cr(C6H6)2], and K2[Cr(CN)2(O)2(O)2(NH3)] of NaOH followed by acidification produces
respectively are : compound X as the major product. X on treatment
(1) +3, +4, and +6 (2) +3, +2, and +4 with (CH3CO)2O in the presence of catalytic
(3) +3, 0, and +6 (4) +3, 0, and + 4 amount of H2SO4 produces :
48. The compound that does not produce nitrogen
gas by the thermal decomposition is :
(1) Ba(N3)2 (2) (NH4)2Cr2O7
(3) NH4NO2 (4) (NH4)2SO4
49. When metal ‘M’ is treated with NaOH, a white
(1) (2)
gelatinous precipitate ‘X’ is obtained, which is
soluble in excess of NaOH. Compound ‘X’ when
heated strongly gives an oxide which is used in
chromatography as an adsorbent. The metal ‘M’
is :
(1) Zn (2) Ca
(3) Al (4) Fe
50. Consider the following reaction and statements: (3) (4)
[Co(NH3)4Br2]+ + Br– ® [Co(NH3)3Br3] + NH3
(I) Two isomers are produced if the reactant
complex ion is a cis-isomer.
55. An alkali is titrated against an acid with methyl
(II) Two isomers are produced if the reactant
orange as indicator, which of the following in a
complex ion is a trans -isomer
correct combination?
(III) Only one isomer is produced if the reactant
Base Acid End point
complex ion is a trans -isomer
(1) Weak Strong Colourless to pink
(IV) Only one isomer is produced if the reactant
(2) Strong Strong Pinkish red to
complex ion is a cis-isomer.
yellow
The correct statements are:
(3) Weak Strong Yellow to Pinkish
(1) (I) and (II) (2) (I) and (III)
red
(3) (III) and (IV) (4) (II) and (IV)
(4) Strong Strong Pink to colourless
51. Glucose on prolonged heating with HI gives :
56. The predominant form of histamine present in
(1) n-Hexane (2) 1- Hexene
human blood is (pKa, Histidine – 6.0)
(3) Hexanoic acid (4) 6-iodohexanal
52. The trans-alkenes are formed by the reduction H
N
of alkynes with: (1)
(1) H2-Pd/C, BaSO4 (2) NaBH4 N
(3) Na/liq. NH3 (4) Sn - HCl
53. Which of the following compounds will be H
N +
suitable for Kjeldahl’s method for nitrogen +
(2)
estimation? N
H
NH2
(1) (2) H
N
(3) +
– N
NO2 N+2 Cl H
(3) (4)
H
N +
(4)
N
JEE Main Solved Paper–2018 2018-7

57. Phenol reacts with methyl chloroformate in the presence of NaOH to form product A. A reacts with Br 2 to
form product B. A and B are respectively :

(1) and

(2) and

(3) and

(4) and

58. The increasing order of basicity of the following


compounds is
(a) (2)

(b)

(c) (3)

(d)
(1) (a) < (b) < (c) < (d)
(4)
(2) (b) < (a) < (c) < (d)
(3) (b) < (a) < (d) < (c)
(4) (d) < (b) < (a) < (c) 60. The major product of the following reaction is :
59. The major product formed in the following
reaction is :

(1) (2)

(1)
(3) (4)
EBD_7764
2018-8 JEE Main Solved Paper–2018

MATHEMATICS 66. Let g(x) = cos x 2 ,f (x) = x , and a, b (a < b)


61. The integral be the roots of the quadratic equation

sin 2 x cos 2 x 18x 2 - 9 px + p 2 = 0 . Then the area (in sq.


ò (sin5 x + cos3x sin 2 x + sin 3x cos2 x + cos5x)2 dx units) bounded by the curve y = (gof)(x) and
is equal to : the lines x = a, x = b and y = 0 , is :
-1 1
(1) + C (2) +C 1 1
3(1 + tan 3 x) 1 + cot 3 x (1) ( 3 + 1) (2) ( 3 - 2)
2 2

-1 1
(3) +C (4) +C 1 1
1 + cot 3 x 3(1 + tan3 x) (3) ( 2 - 1) (4) ( 3 - 1)
2 2
(where C is a constant of integration) 67. The sum of the co-efficients of all odd degree
62. Tangents are drawn to the hyperbola terms in the expansion of
4x 2 - y2 = 36 at the points P and Q. If these
(x + x3 - 1)5 + (x - x3 - 1)5 ,(x > 1) is :
tangents intersect at the point T(0, 3) then the
area (in sq. units) of DPTQ is : (1) 0 (2) 1
(1) 54 3 (2) 60 3 (3) 2 (4) – 1
(3) 36 5 (4) 45 5 68. Let a1,a 2 ,a 3 ,..., a 49 be in A.P. such that
63. Tangent and normal are drawn at P(16, 16) on the
12
parabola y2 = 16x , which intersect the axis of å a 4k +1 = 416 and a 9 + a 43 = 66. If
k =0
the parabola at A and B, respectively. If C is the
centre of the circle through the points P, A and B a12 + a 22 + ... + a17
2
= 140m , then m is equal to :
and ÐCPB = q , then a value of tan q is :
(1) 68 (2) 34
(1) 2 (2) 3
(3) 33 (4) 66
4 1
(3) (4) 9 9
3 2
r 69. If å (xi - 5) = 9 and å (x i - 5)2 = 45 , then the
64. Let u be a vector coplanar with the vectors i =1 i=1
r r r standard deviation of the 9 items x1, x2, ..., x9 is :
a = 2iˆ + 3jˆ - kˆ and b = ˆj + kˆ . If u is perpen-
(1) 4 (2) 2
r r r r
dicular to a and u × b - 24 , then | u |2 is equal (3) 3 (4) 9
to : 70. PQR is a triangular park with PQ = PR = 200 m. A
(1) 315 (2) 256 T.V. tower stands at the mid-point of QR. If the
(3) 84 (4) 336 angles of elevation of the top of the tower at P, Q
and R are respectively 45°, 30° and 30°, then the
65. If a, b Î C are the distinct roots, of the equation
height of the tower (in m) is :
x 2 - x + 1 = 0 , then a101 + b107 is equal to : (1) 50 (2) 100 3
(1) 0 (2) 1
(3) 50 2 (4) 100
(3) 2 (4) – 1
JEE Main Solved Paper–2018 2018-9
71. Two sets A and B are as under : 76. A bag contains 4 red and 6 black balls. A ball is
A = {(a, b) Î R ´ R :| a - 5 | < 1 and | b - 5 | < 1}; drawn at random from the bag, its colour is ob-
served and this ball along with two additional
2 2
B = {(a,b) Î R ´ R : 4(a - 6) + 9(b - 5) £ 36}. balls of the same colour are returned to the bag.
Then : If now a ball is drawn at random from the bag,
(1) A Ì B then the probability that this drawn ball is red,
(2) A Ç B = f (an empty set) is :
(3) neither A Ì B nor B Ì A 2 1
(1) (2)
(4) B Ì A 5 5
72. From 6 different novels and 3 different dictio-
naries, 4 novels and 1 dictionary are to be se- 3 3
(3) (4)
lected and arranged in a row on a shelf so that 4 10
the dictionary is always in the middle. The num- 77. The length of the projection of the line segment
ber of such arrangements is : joining the points (5, –1, 4) and (4, –1, 3) on the
(1) less than 500
plane, x + y + z = 7 is:
(2) at least 500 but less than 750
(3) at least 750 but less than 1000 2 1
(1) (2)
(4) at least 1000 3 3
1 1 2 2
73. Let f (x) = x 2 + and g(x) = x - , (3)
3
(4)
3
x2 x
78. If sum of all the solutions of the equation
f (x)
x Î R - {-1,0,1} . If h(x) = , then the local æ æp ö æp ö 1ö
g(x) 8cos x × ç cos ç + x ÷ × cos ç - x ÷ - ÷ - 1 in
è è6 ø è6 ø 2ø
minimum value of h(x) is :
(1) – 3 (2) -2 2 [0, p] is kp, then k is equal to :
(3) 2 2 (4) 3 13 8
(1) (2)
9 9
74. For each t Î R , let [t] be the greatest integer
less than or equal to t. Then 20 2
(3) (4)
æé 1 ù é 2 ù é15 ù ö 9 3
lim x ç ê ú + ê ú + ... + ê ú ÷
è
x ®0 + ë û ë û
x x ë x ûø 79. A straight the through a fixed point (2, 3)
intersects the coordinate axes at distinct points
(1) is equal to 15.
(2) is equal to 120. P and Q. If O is the origin and the rectangle OPRQ
(3) does not exist (in R). is completed, then the locus of R is :
(4) is equal to 0. (1) 2x + 3y = xy (2) 3x + 2y = xy

p (3) 3x + 2y = 6xy (4) 3x + 2y = 6


2
sin 2 x 80. Let A be the sum of the first 20 terms and B be
75. The value of ò x
dx is : the sum of the first 40 terms of the series
p 1+ 2
-
2 12 + 2 × 22 + 32 + 2.4 2 + 52 + 2.62 + ...
p
(1) (2) 4p If B - 2A = 100 l , then l is equal to :
2
(1) 248 (2) 464
p p (3) 496 (4) 232
(3) (4)
4 8
EBD_7764
2018-10 JEE Main Solved Paper–2018

81. If the curves y2 = 6x,9x 2 + by 2 = 16 intersect 87. Let S = {x Î R : x ³ 0 and


each other at right angles, then the value of b is : 2 | x - 3 | + x ( x - 6) + 6 = 0 . Then S :
7 (1) contains exactly one element.
(1) (2) 4
2 (2) contains exactly two elements.
9
(3) contains exactly four elements.
(3) (4) 6 (4) is an empty set.
2
82. Let the orthocentre and centroid of a triangle be 88. If the tangent at (1, 7) to the curve x 2 = y - 6
A(–3, 5) and B(3, 3) respectively. If C is the
circumcentre of this triangle, then the radius of touches the circle x 2 + y2 + 16x + 12y + c = 0
the circle having line segment AC as diameter, is : then the value of c is :
5 (1) 185 (2) 85
(1) 2 10 (2) 3 (3) 95 (4) 195
2
89. Let y - y(x) be the solution of the differential
3 5
(3) (4) 10 dy
2 equation sin x + y cos x = 4x, x Î (0, p) . If
dx
83. Let S = { t Î R : f (x) =| x - p | (e|x| - 1)sin | x | is
æ pö æ pö
not differentiable at t}. Then the set S is equal to : y ç ÷ = 0 , then y ç ÷ is equal to :
(1) {0} (2) {p} è2ø è6ø
(3) {0, p} (4) f (an empty set)
-8 2 8
x - 4 2x 2x (1) p (2) - p2
9 3 9
84. If 2x x - 4 2x = (A+ Bx)(x - A) 2 ,
4 4
2x 2x x-4 (3) - p2 (4) p2
9 9 3
then the ordered pair (A, B) is equal to :
90. If L1 is the line of intersection of the planes
(1) (– 4, 3) (2) (– 4, 5)
(3) (4, 5) (4) (– 4, – 5) 2x - 2y + 3z - 2 = 0, x - y + z + 1 = 0 and L2 is
85. The Boolean expression the line of intersection of the planes
~ (pÚ q) Ú (~ p Ù q) is equivalent to : x + 2y - z - 3 = 0, 3x - y + 2z - 1 = 0 , then the
(1) p (2) q distance of the origin from the plane, containing
(3) ~q (4) ~p the lines L1 and L2, is :
86. If the system of linear equations
x + ky + 3z = 0 1 1
(1) (2)
3x + ky – 2z = 0 3 2 2 2
2x + 4y – 3z = 0
1 1
xz (3) (4)
has a non-zero solution (x, y, z), then is equal 2 4 2
y2
to :
(1) 10 (2) – 30
(3) 30 (4) – 10
JEE Main Solved Paper–2018 2018-11

Hints and Solutions


PHYSICS 100
= m + 10 \ m = 23.3kg ; close to
Mass (M) M 3
1. (3) Density (d) = =
Volume (V) L3 option (2)

Dd DM 3DL ¶u K
= + 4. (3) F=- rˆ = 3 rˆ
\ Error in density, ¶r r
d M L
= 1.5% + 3(1%) = 4.5% Since particle is moving in circular path
2. (2) Graphs in option (3) position-time and mv 2 K K
option (1) velocity-position ar e F= = Þ mv 2 =
3
corresponding to velocity-time graph
r r r2
option (4) and its distance-time graph is as 1 K
given below. Hence distance-time graph \ K.E. = mv2 = 2
2 2r
option (2) is incorrect.
Total energy = P.E. + K.E.
distance K K
=- 2
+ = Zero
2r 2r 2
K
(Q P.E. = - given)
2r 2
time
5. (2)
Before Collision After Collision
V0 V1 V2
m m Þ m m
3 (2) Given : m1 = 5kg; m2 = 10kg; m = 0.15
FBD for m1, m1g – T = m1a Stationary
Þ 50 – T = 5 × a
1 2 1 2 3æ1 ö
and, T – 0.15 (m + 10) g = (10 + m)a mv + mv = ç mv02 ÷
For rest a = 0 2 1 2 2 2è2 ø
or, 50 = 0.15 (m + 10) 10 3
Þ v12 + v 22 = v20 ....(i)
2
N From momentum conservation
m
T mv0 = m(v1 + v2) ....(ii)
m2
Squarring both sides,
m(m+m2)g (m+m2)g T (v1 + v2)2 = v02
m1 Þ v12 + v22 + 2v1v2 = v02
m1g = 50N v20
2v1v2 = -
2
3
Þ 5= (m + 10) 3 2 v2
20 (v1 - v 2 ) 2 = v 21 + v 22 - 2v1v2 = v0 + 0
2 2
EBD_7764
2018-12 JEE Main Solved Paper–2018
Solving we get relative velocity between the Moment of inertia (I1) of the complete disc
two particles 1
v1 - v2 = 2v0 = 9 MR 2 about an axis passing through
2
6. (4) Using parallel axes theorem, moment of inertia O and perpendicular to the plane of the disc.
about ‘O’ M.I. of the cut out portion about an axis
Io = Icm + md2 passing through O' and perpendicular to
the plane of disc
7MR 2 55MR 2
= + 6(M ´ (2R)2 ) = 2
2 2 1 æ Rö
= ´M ´ç ÷
2 è 3ø

2R \ M.I. (I2) of the cut out portion about


2R 2R an axis passing thr ough O and
O perpendicular to the plane of disc
2R é1 æ Rö
2
æ 2R ö ù
2
2R = ê ´M ´ç ÷ +M ´ç ÷ ú
êë 2 è 3ø è 3 ø ú
2R û
[Using perpendicular axis theorem]
Again, moment of inertia about point P, \ The total M.I. of the system about an
axis passing through O and perpendicular
Ip = Io + md 2
to the plane of the disc is
I = I1 + I2
55MR 2 181
= + 7M(3R) 2 = MR 2 =
2 2
7. (1) Let s be the mass per unit area. 1 é1 æ Rö
2
æ 2R ö ù
2
9 MR 2 - ê ´ M ´ ç ÷ + M ´ ç ÷ ú
2 êë 2 è 3ø è 3 ø ú
û
R/ 3
O'
9MR 2 9MR 2 (9 - 1)MR 2
= - = = 4 MR 2
2 18 2
2R/3
1
O
8. (3) mw2 R = Force µ
Rn

mv2
(Force = )
R

1 1
The total mass of the disc Þ w2 µ Þ wµ n +1
= s × p R2 = 9M R n +1
R 2
The mass of the circular disc cut
2 2p
æ Rö p R2 Time period T =
= s´pç ÷ = s´ =M w
è 3ø 9
n +1
Let us consider the above system as a
complete disc of mass 9M and a negative Time period, T µ R 2

mass M super imposed on it.


JEE Main Solved Paper–2018 2018-13

volumetric stress P ˆ P ˆ P ˆ P ˆ
9. (3) Bulk modulus, K = DP = J+ J+ i- i
volumetricstrain 2 2 2 2
mg 2P ˆ
K= DP = J = IH molecule
æ dV ö 2
aç ÷
è V ø
2P ˆ
dV mg Þ Iwall = - J
Þ = ....(i) 2
V Ka Pressure, P
4 3
volume of sphere , V = pR F
=
2P
n (Q n = no.of particles)
3 =
A A
dV 3dr
Fractional change in volume = 2 ´ 3.32 ´ 10 -27 ´ 103 ´ 1023
V r =
...(ii) 2 ´ 10 -4
3dr mg =2.35 × 103N/m2
Using eq. (i) & (ii) = 12. (2) As we know, frequency in SHM
r Ka
1 k
dr mg f= = 1012
\ = (fractionaldecrement in radius) 2p m
r 3Ka
10. (3) In an adiabatic process where m = mass of one atom
TVg–1 = Constant Mass of one atom of silver,

or, T1V1g–1 = T2V2g–1 108


´ 10-3 kg
5
(
= 6.02 ´1023
)
For monoatomic gas g =
3
1 k
(300)V2/3 = T2(2V)2/3 ´ 6.02 ´ 1023 = 1012
2p 108 ´ 10-3
300
Þ T2 = Solving we get, spring constant, K =
(2) 2/3 7.1N/m
T2 = 189 K (final temperature) 13. (1) In solids, Velocity of wave
f
Change in internal energy DU = n R DT Y 9.27 ´ 1010
2 V= =
r 2.7 ´ 103
æ 3 öæ 25 ö
= 2 ç ÷ç ÷ (-111) = -2.7 kJ v = 5.85 × 103 m/sec
è 2 øè 3 ø Since rod is clamped at middle fundamental
11. (1) Change in momentum wave shape is as follow
j^ l
= L Þ l = 2L
2
P ^J
2
P A A
P^ N
45° 45° 2 i
i^ l/2
P ^i
– P J^ 2 l = 1.2m (Q L = 60 cm = 0.6m (given)
2 Using v = fl
P
EBD_7764
2018-14 JEE Main Solved Paper–2018

v 5.85 ´103 1000


Þ f= = Pavg = watt
l 1.2 2
= 4.88 × 103 Hz ; 5 KHz Wattless current I = Irms sin q
14. (2) Potential outside the shell,
I0 20
KQ = sin q = sin 45° = 10A
Voutside = 2 2
r
where r is distance of point from the 12V 1W
17. (2) T U
centre of shell
v–12 2W
KQ S P
Potential inside the shell, Vinside =
R v–13
where ‘R” is radius of the shell R Q
V 10W 0
s
C Using Kirchhoff’s law at P we get
–s
B
s V - 12 V - 13 V - 0
+ + =0
a A 1 2 10
b c [Let potential at P, Q, U = 0 and at R = V
V V V 12 13 0
Þ + + = + +
1 2 10 1 2 10
Kq A Kq B KqC
VB = + + 10 + 5 + 1 24 + 13
rb rb rc Þ V=
10 2
1 é s4pa 2 s4pb 2 s4pc 2 ù
VB = ê - + ú æ 16 ö 37
4p Î0 ëê b b c ûú Þ Vç ÷ =
è 10 ø 2

s é a 2 - b2 ù 37 ´10 370
VB = ê + cú ÞV= = = 11.56 volt
Î0 16 ´ 2 32
êë b úû
18. (2) As we know, radius of circular path in
15. (1) Charge on Capacitor, Qi = CV magnetic field
After inserting dielectric of dielectric
constant = K Qf = (kC) V 2Km
r=
Induced charges on dielectric qB
Qind = Qf - Qi = KCV - CV
2Km e
æ5 ö For electron, re = ....(i)
= (K - 1)CV = ç - 1 ÷ ´ 90pF ´ 2V = 1.2nc eB
è3 ø
16. (2) As we know, average power Pavg = Vrms Irms 2Km p
cosq For proton, rp = ....(ii)
eB
æ V öæ I ö æ 100 ö æ 20 ö For a particle,
= ç 0 ÷ ç 0 ÷ cos q = ç ÷ç ÷ cos 45°
è 2 øè 2 ø è 2 øè 2 ø 2Km a 2K4m p 2Km p
ra = = = ...(iii)
(Q q = 45°) qa B 2eB eB
\ r e < rp = ra (Q me < mp)
JEE Main Solved Paper–2018 2018-15

19. (3) Magnetic field at the centre of loop, After introducing polariser C between A
and B,
m0 I
B1 = Polariser A Polariser C Polariser B
2R
Dipole moment of circular loop is m = IA
m1 = I.A = I.pR2 {R = Radius of the loop}
If moment is doubled (keeping current I I/2 I/2Cos q
2
I/2Cos4 q
constant)
R becomes 2R
I I 1
cos4 q = Þ cos4 q =
( )
2 2
m 2 = I.p 2R = 2.IpR = 2m1 2 8 4

m0 I 1
B2 = Þ cos q = or, q = 45°
2 ( 2R ) 2
2l
23. (1) Angular width of central maxima =
m0 I d
B1 2R
\ = = 2 d l´D
B2 m0 I or, l = ; Fringe width, b =
2 ( 2R ) 2 d'

d 50 ´ 10-2 10 -6 ´ 50 ´10 -2
10-2 = ´ =
w0 w L 2 d' 2´d'
20. (1) Quality factor Q = = 0
2Dw R Therefore, slit separation distance, d’ =
1 25mm
v= 24. (1) Wavelength of emitted photon from nth
21. (3) Velocity of EM wave is given by mÎ
state to the ground state,
w 1 æ1 1 ö
Velocity in air = k = C = RZ 2 ç - ÷
Ln è12
n2 ø
C -1
Velocity in medium = 2 1 æ 1 ö
Ln = 1-
2 ç ÷
Here, m1 = m2 = 1 as medium is non-magnetic RZ è n 2 ø
Since n is very large, using binomial
1
theorem
Îr1 C Îr1 1
\ = =2 Þ = 1 æ 1 ö
1 æCö Îr2 Ln = 1+
4 2 ç ÷
ç ÷ RZ è n 2 ø
Îr2 è2ø
1 1 æ 1 ö
22. (3) Axis of transmission of A & B are parallel. Ln = + ç ÷
2
RZ RZ 2 è n 2 ø
Polariser A Polariser B As we know,

2pr æ n 2h2 ö 1
ln = = 2p ç ÷ µn
n ç 4p2 mZe2 ÷ n
è ø
I I/2 I/2
B
Ln » A +
ln2
EBD_7764
2018-16 JEE Main Solved Paper–2018

25. (4) hnL = E ¥ - E1 ...(i) Current,

hnf = E ¥ - E5 ...(ii) V - DV 3 - 0.7 2.3


I= = = = 11.5mA
R 200 200
2
z2 E5 æ 1 ö 1 28. (3) If n = no. of channels
Eµ Þ =ç ÷ =
n 2 E1 è 5 ø 25 10% of 10 GHz = n × 5 KHz
10
hn L E1 or, ´ 10 ´ 109 = n ´ 5 ´ 103
Eqn (i) / (ii) Þ =
hn f E 5
100
Þ n = 2 × 105
n L 25 n 29. (2) Using formula, internal resistance,
Þ = Þ nf = L
nf 1 25 æ l -l ö
r =ç 1 2 ÷s
26. (1) For collision of neutron with deuterium: è l2 ø
v v1 v2
m 2m m 2m æ 52 - 40 ö
=ç ÷ ´ 5 = 1.5W
Applying conservation of momentum : è 40 ø
mv + 0 = mv1 + 2mv2 .....(i) 30. (3) R1 + R2 = 1000
v2 – v1 = v .....(ii) Þ R2 = 1000 – R1
Q Collision is elastic, e = 1
v R1 R2 = 1000 – R1
From eqn (i) and eqn (ii) v1 = -
3
G
1 1
mv2 - mv12
2 2 8 (l )
Pd = = = 0.89 100 – l
1 2 9 On balancing condition
mv
2 R1(100 – l) = (1000 – R1)l ...(i)
Now, For collision of neutron with carbon On Interchanging resistance balance point
nucleus shifts left by 10 cm
v v1 v2
m 12m m 12m R2=1000 – R1 R1
Applying Conservation of momentum
mv + 0 = mv1+ 12mv2 ....(iii) G
v = v2 – v1 ....(iv)
From eqn (iii) and eqn (iv) (l – 10) (100 – l + 10)
11 =(110 – l)
v1 = - v
13 On balancing condition
(1000 – R1) (110 – l) = R1 (l – 10)
2 or, R1 (l – 10) = (1000 – R1) (110 – l) ...(ii)
1 1 æ 11 ö
mv2 - m ç v ÷ Dividing eqn (i) by (ii)
2 2 è 13 ø 48
Pc = = » 0.28
1 169 100 - l l
mv2 =
2 l - 10 110 - l
27. (3) Clearly from fig. given in question, Silicon Þ (100 – l) (110 – l) = l(l – 10)
diode is in forward bias. Þ 11000 – 100l – 110l + l2 = l2 – 10l
\ Potential barrier across diode Þ 11000 = 200l
or, l = 55
DV = 0.7 volts
JEE Main Solved Paper–2018 2018-17
Putting the value of ‘l’ in eqn (i)
R1 (100 – 55) = (1000 – R1) 55 2-2
H 22 - (Z = 4) s s* =0
Þ R1 (45) = (1000 – R1) 55 1s 2 1s 2 2
Þ R1 (9) = (1000 – R1) 11
Þ 20 R1 = 11000 2-0
He 22+ (Z = 2) s =1
1s 2 2
\ R1 = 550KW
CHEMISTRY Molecule having zero bond order will not be a
viable molecule.
31. (4) 34. (1) From thermodynamics

Relative Relative Simplest whole -DH° DS°


Element ln K = +
mass mole number ratio RT R
6 for exothermic reaction,
C 6 = 0.5 1
12 DH = –ve
1
H 1 =1 2 – DH°
1 slope = = + ve
R
So, X = 1, Y = 2 So from graph, line should be A & B.
Equation for combustion of CXHY 35. (4)
æ Yö Y
CX H Y + ç X + ÷ O2 ¾¾
® XCO 2 + H 2O 15
è 4ø 2 C 6 H 6 ( l) + O 2 ( g ) ¾¾
® 6CO 2 ( g ) + 3H 2 O ( l )
2
æ Yö
Oxygen atoms required = 2 çè X + ÷ø 15 3
4 Dn g = 6 - =-
As mentioned, 2 2
æ Yö DH = DU + Dn g RT
2 ç X + ÷ = 2Z
è 4ø
æ 2ö æ 3ö
Þ çè1 + ÷ø = Z Þ Z = 1.5 = -3263.9 + ç - ÷ ´ 8.314 ´ 298 ´ 10-3
4 è 2ø
\ molecule can be written as
= – 3263.9 + (–3.71)
CXHYOZ
= – 3267.6 kJ mol–1
C1H2O3/2
36. (4) Number of particles (i)
Þ C2H4O3
(1) [Co(H2O)6]Cl3 1
32. (3) In Frenkel defect some of ion (usually cation
due to their small size) missing from their (2) [Co(H2O)5Cl]Cl2.H2O 3
normal position and occupies position in (3) [Co(H2O)4Cl2]Cl.2H2O 2
interstitial position.
(4) [Co(H2O)3Cl3].3H2O 4
33. (4) Electronic configuration Bond order
DTf µ i ; where DTf = (Tf - Tf¢ )
2 -1
He +2 (Z = 3) s 2 s* 1 = 0.5 Remember, the greater the no. of particles,
1s 1s 2
the lower will be the freezing point.
2 -1 Compound (d) will have the highest freezing
H-2 (Z = 3) s s* = 0.5
1s 2 1s1 2 point due to least no of particles.
EBD_7764
2018-18 JEE Main Solved Paper–2018
37. (2) In presence of external H+, r1 = 1 torr s–1, when 5% reacted
r2 = 0.5 torr s–1, when 33 % reacted
ˆˆ† 2H + + S2 - , K a . K a = K eq
H 2S ‡ˆˆ 1 2
(a - x1 ) = 0.95(unreacted)
2
éH + ù éS2- ù (a - x 2 ) = 0.67(unreacted)
ë û ë û = 1 ´ 10-7 ´ 1.2 ´ 10-13
\
[ H2S] r1 é (a - x1 ) ù
m
1 æ 0.95 ö
m
=ê ú ; =ç ÷
r2 ë (a - x 2 ) û 0.5 è 0.67 ø
[ 0.2]2 éëS2- ùû
= 1.2 ´ 10 -20 2 = (1.41)m Þ 2 = ( 2)m
[ ]
0.1
Þm=2
éS2- ù -20
= 3 ´ 10 40. (3)
ë û B2 H 6 + 3O 2 ¾¾
® B2 O 3 + 3H 2 O
27.66 g of B2H6 (1 mole) requires 3 moles
38. (3) of oxygen (O2) for complete burning.
1M + Final
Na2SO 4 Ba2+ ¾® Solution Required O2 (3 moles) is obtained by
50 mL 450 mL 500 mL electrolysis of 6 moles of H2O
On electrolysis : 6H 2 O ¾¾
® 6H 2 + 3O 2
Concentration of SO 2–
4 in BaSO4 solution
Number of Faradays = 12 = Amount of
M1V1 = M2V2 charge
1 × 50 = M2 × 500 12 × 96500 = i × t
1 12 × 96500 = 100 × t
M2 =
10 12 ´ 96500
t= second
For just precipitation 100
Ionic product = Ksp
12 ´ 96500
[Ba2+] [SO42–] = Ksp(BaSO4) = hour
100 ´ 3600
1 = 3.2 hours
[Ba2+] × = 10–10
10
[Ba2+] = 10–9 M in 500 mL solution 41. (3) [3Ca 3 (PO 4 )2 .Ca(OH) 2 ]+ 2F – ¾¾
®
Hydroxyapatite (drinking
water
Thus [Ba2+] in original solution (450 mL) upto1ppm)
Þ M1 × 450 = 10–9 × 500
[ where M1 = molarity of original solution]
[3Ca 3(PO 4 ) 2 .CaF2 ] + 2OH –
500 Fluorapatite
M1 = ´ 10 –9 = 1.11 ´ 10-9 M (Harder teeth
450 enamel)

39. (1) CH 3CHO ¾¾


® CH 4 + CO
42. (3) KCl is an ionic compound while other (PH3,
Generally r µ (a – x)m O2, B2H6, H2SO4) are covalent compounds.
JEE Main Solved Paper–2018 2018-19

43. (2, 4) BCl3 and AlCl3, both have vacant p-orbital 47. (3) [Cr (H2O)6] Cl3
and incomplete octet thus they behave as Þ x + 6 × 0 + (– 1) × 3 = 0
Lewis acids. Þ x = +3
SiCl4 can accept lone pair of electron in d- [Cr (C6H6)2]
orbital of silicon hence it can act as Lewis
x+2×0= 0;x=0
acid.
K2 [Cr (CN)2 (O)2 (O2) (NH3)]
Although the most suitable answer is (c).
However, both options (c) and (a) can be 2 × 1 + x + 2 × (– 1) + 2 × (– 2) + (– 2) + 0 = 0
considered as correct answers. x=+6
e.g. hydrolysis of SiCl4 48. (4)
D
Cl Cl (1) Ba(N 3 ) 2 ¾¾® Ba + 3N 2
H
D
Si + H2O ¾® Si O (2) (NH 4 )2 Cr2 O7 ¾¾® Cr2 O3 + N 2 + 4H 2O
Cl Cl H
Cl Cl Cl Cl D
(3) NH 4 NO 2 ¾¾® N 2 + 2H 2 O
D
Cl (4) (NH 4 ) 2 SO 4 ¾¾® 2NH 3 + H 2 SO 4
NH3 is evolved in reaction (c).
Cl — Si — OH + HCl
49. (3)
Cl NaOH
Al + 3H 2 O ¾¾¾¾
® Al(OH)3 ¯ + 3 / 2H 2 (g)
i.e., option (a) AlCl 3 and SiCl 4 is also (X)
correct.
White gelatinous ppt.
44. (3)
I
excess of NaOH
Al(OH)3 ¾¾¾¾¾¾® Na[Al(OH) 4 ]
I Soluble

D
I 2Al(OH)3 ¾¾
® Al2O3 + 3H 2O
(X )
\ Total number of lone pair of electrons is 9. Al2O3 is used as adsorbent in chromatography.
45. (2) CH 3 COOK is a salt of weak acid Thus, metal ‘M’ is Al.
(CH3COOH) and strong base (KOH).
FeCl3 is a salt of weak base [Fe(OH)3] and
50. (2) Br
strong acid (HCl).
H3N Br
Pb (CH3COO)2 is a salt of weak base
Pb(OH)2 and weak acid (CH3COOH)
Al(CN)3 is a salt of weak base [Al (OH)3]
and weak acid (HCN).
H3N NH3
1 NH3
46. (3) [Fe(CN)6]4– + H2O2 + H+ ¾® [Fe(CN)6]3–
2 cis-isomer
+ H2O +Br –
1 –
[Fe(CN)6]3– + H O + OH
2 2 2 1
¾® [Fe(CN)6]4– + H2O + O2
2
EBD_7764
2018-20 JEE Main Solved Paper–2018

Br Br
H3N Br H3N Br

+
H3N Br H3N NH3
NH3 Br
fac– mer–
(2 isomers)

Br Br
H3N NH3 H3N NH3
Br –
¾+ ¾ ®

H3N NH3 H3N Br


Br Br
trans mer (1 isomer)

51. (1) 55. (3) pH range for methyl orange is


CHO ¬¾¾¾¾¾ 3.9 - 4.5 ¾¾¾¾
®
Pinkish red Yellow
HI, D
(CH — OH)4 ¾® CH3CH2CH2CH2CH2CH3 Generally, weak bases have pH greater than
n-Hexane 7. When methyl orange is added to a weak
CH2 — OH
base solution, solution becomes yellow.
52. (3)
Na /liq.NH
¾¾¾¾¾¾ 3
® This solution is then titrated by a strong
Birch reduction
acid and at the end point pH will be less
53. (1) Kjeldahl’s method is not applicable for
than 3.1.
compounds containing nitrogen in nitro
and azo groups and nitrogen in ring, as N \ Solution becomes pinkish red.
of these compounds does not change to 56. (4) Structure of histamine
ammonium sulphate under these H
conditions. N
54. (1) more basic

OH OH N
COOH Blood is slightly basic in nature (7.35 pH).
CO2, NaOH At this pH, terminal NH2 will get protonated
¾¾¾¾¾® due to more basic nature.
Acidification
\ Predominant structure of histamine is
(X)
(Major) H
O N
O—C—CH3
COOH N
(CH CO) O
¾ ¾3 ¾ ¾
2
®
H2SO4
Acetyl salicylic acid
(Aspirin)
JEE Main Solved Paper–2018 2018-21

O

OH O O O — C — O — CH3
57. (3) – Cl — C — O — CH3
OH
¾ ® ¾¾¾¾¾¾®
(A)

O
O — C — O — CH3

Br
¾¾¾¾
2
¾®

Br
(B)

+
NH2 NH3
58. (3) (a) Protonation
¾¾¾¾®
(1° and sp3 )
+
NH NH2
Protonation
(b) ¾¾¾¾ ®
(sp2 )
+
NH2 NH2 NH2
Protonation
¾¾¾¾® ¾¾®
(c) +
NH NH2 NH2

[Equivalent resonance]

¾Protonation
¾¾¾® +
(d) NHCH3 NH2 CH3
(2° and sp3)
Hence, correct order of basicity will be : b < a < d < c.
59. (4)

¾ ¾®
(–C2H 5I)

– CH 3OH

+
EBD_7764
2018-22 JEE Main Solved Paper–2018
60. (2) CH 3 O–
is a strong base and strong
x(0) y(3)
nucleophile, so favourable condition is \ Equation of PQ is : - =1
SN2/E2. 9 36
The given alkyl halide is 2° and b carbons Þ y = –12
are 4° and 2°, so sufficiently hindered, thus
E2 dominates over SN2.
Also, polarity of CH3OH (solvent) is not Y
as high as H2O, so E1 is also dominated by
E2.
x 2 y2
- =1
4° 9 36
Br (0, 3)
b X' X
a CH O

¾¾®
3
E2
b H (Major product)
Q P
(2°) R

Y'
MATHEMATICS
61. (1) Let I
sin2 xcos2 x 1
=ò dx \ Area of DPQT = ´ TR ´ PQ
(sin5x+ cos3xsin2x+ sin3xcos2 x + cos5x)2 2

Q P º (3 5, -12) \ TR = 3 + 12 = 15,
2 2
sin x cos x \ Area of
=ò dx
2 2 3 3 2
[(sin x + cos x)(sin x + cos x)]
1
DPQT = ´15 ´ 6 5 = 45 5 sq. units
2
sin 2 x cos 2 x
=ò dx 63. (1) Equation of tnagent at P(16, 16) is given
(sin 3 x + cos3 x)2
as:
x – 2y + 16 = 0
tan 2 x × sec 2 x
=ò dx
(1 + tan 3x)2 Y
)
, 16
Now, put (1 + tan3x) = t 16
P(
Þ 3 tan2x sec2x dx = dt t
No

en
ng
rm

-1 Ta
al

1 dt 1 X' X
3 ò t2
\ I= =- + C = +C A B(24, 0)
3t 3(1 + tan3 x) (–16, 0)
62. (4) Here equation of hyperbola is

x2 y2 Y'
- =1
9 36
Now, PQ is the chord of contant 4
Slope of PC (m1) =
3
JEE Main Solved Paper–2018 2018-23

Slope of PB (m2) = –2 12
68. (2) Q å a 4k +1 = 416
4 k =0
+2
m1 - m 2
Hence, tan q = = 3 13
1 + m1 × m 2 1 - 4 × 2 Þ [2a1 + 48d] = 416
2
3
Þ a1 + 24d = 32 ...(1)
Þ tan q = 2 Now, a9 + a43 = 66 Þ 2a1 + 50d = 66
...(2)
r r r
64. (4) Q u, a & b are coplanar From eq. (1) & (2) we get; d = 1 and a1 = 8
r r r r r r r r r 17 17
\ u = l(a´ b) ´ a = l{a 2 .b - (a .b)a} Also, å a 2r = å [8 + (r - 1)1]2 = 140 m
r =1 r =1
= l{-4iˆ + 8jˆ + 16k}
ˆ = l'{- iˆ+ 2 ˆj+ 4k}.
ˆ
17
rr Þ å (r + 7)2 = 140 m
Also, u.b = 24 Þ l' = 4
r =1
r
\ u = -4iˆ + 8jˆ + 16kˆ 17

r2
Þ å (r 2 + 14r + 49) = 140 m
Þ u = 336 r =1

65. (2) a, b are roots of x2 – x + 1 = 0 æ 17 ´18 ´ 35 ö æ 17 ´18 ö


Þ ç ÷ + 14 ç ÷ + (49 ´17) = 140
è 6 ø è 2 ø
\ a = -w and b = -w2
Þ m = 34
where w is cube root of unity
9 9
\ a101 + b107 = (–w)101 + (–w)107
= –[w2 + w] = –[–1] = 1
69. (2) Given å (x i - 5) = 9 Þ å xi = 54 ...(i)
i =1 i =1
66. (4) Here, 18x2 – 9px + p2 = 0
9
Þ (3x – p) (6x – p) = 0
Also, å (x i - 5)2 = 45
p p i=1
Þ a= ,b=
6 3 9 9
Also, gof(x) = cosx Þ å x i2 - 10å xi + 9(25) = 45 ...(ii)
i =1 i =1
p /3 From (i) and (ii) we get,
\ Req. area = ò cos xdx = 3 - 1
9
p /6 2
å xi2 = 360
67. (3) Since we know that, i =1
(x + a)5 + (x – a)5 2
= 2[5C0 x5 + 5C2 x3× a2 + 5C4x×a4] Since, variance =
å xi 2 - æ å x i ö
9 ç 9 ÷
è ø
(x + ) + (x - )
5 5
\ x3 - 1 x3 - 1
2
360 æ 54 ö
= 2[5C0 x5 + 5C2 x3(x3 – 1) + 5C4x(x3 – 1)2] = - ç ÷ = 40 – 36 = 4
9 è9ø
Þ 2[x5 + 10x6 – 10x3 + 5x7 – 10x4 + 5x]
\ Sum of coefficients of odd degree terms = 2. \ Standard deviation = Variance = 2
EBD_7764
2018-24 JEE Main Solved Paper–2018
70. (4) 72. (4) \ Required number of ways =
P 6
C4 ´ 3 C1 ´ 4!
= 15 × 3 × 24 = 1080
200 45°
200
1
x2 +
N
73. (3) Here, h(x) = x2 = æ x - 1 ö + 2
çè ÷
1 xø 1
90° x- x-
30° h 30° x x
Q R
M
Let height of tower MN = h
1
In DQMN we have When x - <0
x
MN
tan 30° =
QM 1 2
\ x- + £ -2 2
x 1
\ QM = 3h = MR ...(1) x-
x
Now in DMNP
MN = PM ...(2) Hence, -2 2 will be local maximum value
In DPMQ we have : of h(x).
MP = (200)2 - ( 3 h)2 1
When x - >0
\ From (2), we get : x

(200)2 - ( 3 h)2 = h Þ h = 100m 1 2


\ x- + ³2 2
71. (1) A ={(a,b) Î R ´ R :| a - 5 |< 1,| b - 5 |< 1} x x- 1
x
Let a – 5, = x, b – 5 = y
Set A contains all points inside Hence, 2 2 will be local minimum value of
| x |< 1,| y |< 1 h(x).
2 2
B = {(a,b) ÎR ´ R: 4(a - 6) + 9(B- 5) £ 36} 74.
æ 1 15 ö
(2) Since, lim x ç é ù + é ù + ... + é ù ÷
2
ê ú ê ú êë x úû ø
Set B contains all points inside or on x ® 0+ è ë x û ë x û

(x - 1)2 y 2 æ 1 + 2 + 3 + ... + 15 ö
+ =1 = lim+ x ç
è ÷ø -
9 4 x®0 x
Y
æì 1 ü ì 2 ü ì15 ü ö
çè í x ý + í x ý + ... + í x ý ÷ø
î þ î þ î þ
(–1, 1) (0, 1)
(0, 1)
ìrü ìrü
X¢ (–1, 0) (1, 0) X Q 0 £ í ý <1 Þ 0 £ xí ý < x
îxþ îxþ
(1, –1)
(–1, –1) (0, –1)
æ 1 + 2 + 3 + ... + 15 ö 15 ´ 16
\ lim+ x çè ÷ø = = 120
x ®0 x 2

\ (±1, ±1) lies inside the ellipse.
Hence, A Ì B .
JEE Main Solved Paper–2018 2018-25

p /2
sin 2 x 77. (3) C B (5, –1, 4)
75. (3) Let, I = ò x
dx ...(i)
-p /2 1 + 2

b b
Using, ò f (x)dx = ò f (a + b - x)dx, we get : A (4, –1, 3)
A¢ B¢
a a

p /2
sin 2 x
I= ò -x
dx ...(ii)
($i + $j + k$ ) =
-p /2 1 + 2
( )
AC = AB.AC = $i + k$ .
3
2
3
Adding (i) and (ii), we get;
Now,
p /2
4 2
2I = ò sin 2 x dx Þ A'B' = BC = AB2 - AC2 = 2 -
3
=
3
-p /2

2
x /2 \ Length of projection <
2 × ò sin 2 x dx
2I = 3
0 p
(1) Q 8cos x æç cos 2 - sin 2 x - ö÷ = 1
1
78.
è 6 2ø
p p
Þ 2I =
2´ Þ I =
4 4 æ3 1 ö
Þ 8cos x ç - - sin 2 x ÷ = 1
76. (1) Let Rt be the even of drawing red ball in tth è4 2 ø
draw and Bt be the event of drawing black æ1 ö
ball in tth draw. Þ 8cos x ç - (1 - cos 2 x) ÷ = 1
è4 ø
Now, in the given bag there are 4 red and 6
æ1 ö
black balls. Þ 8cos x ççç ,1 ∗ cos 2 x÷÷÷ < 1
è4 ø
4 6
\ P(R1 ) = and P(B1 ) = æ 3ö
10 10 Þ 8cos x çççcos2 x , ÷÷÷ < 1
è 4ø
æR ö 6 æR ö 4 æ 4cos3 x , 3cos x ö÷
And, P ç 2 ÷ = and P ç 2 ÷ = ç ÷÷ < 1
è R1 ø 12 è B1 ø 12 Þ 8çç ÷
çè 4 ø÷
Now, required probability
Þ 2(4cos3 x , 3cos x) < 1
æR ö æR ö 1
= P(R1) ´ P ç 2 ÷ + P(B1 ) ´ P ç 2 ÷ Þ 2cos3x = 1 Þ cos3x =
è R1 ø è B1 ø 2
p
æ 4 6ö æ 6 4ö 2 \ 3x = 2np ± , n Î1
= ç ´ ÷+ç ´ ÷ = 3
è 10 12 ø è 10 12 ø 5
2np p
Þ x= ±
3 9
EBD_7764
2018-26 JEE Main Solved Paper–2018

p 2p p 2p p Y
In x Î [0, p]: x = , + , - , only
9 3 9 3 9 y2 = 6x
Sum of all the solutions of the equation
P(x1, y1)
æ1 2 1 2 1ö 13
= ç + + + - ÷p = p
è 9 3 9 3 9 ø 9 X
79. (2) Equation of PQ is
x y
+ =1 ...(1)
h k
9x2 + by2 = 16
Since, point of intersection is on both the
curves, then
(0, k)Q R(h, k) y12 = 6x1 ...(i)

and 9x12 + by12 = 16 ...(ii)


(2, 3) Now, find the slope of tangent to both the
curves at the point of intersection P(x1, y1)
O For slope of curves:
P(h, 0)
curve (i):
æ dy ö 3
Since, (1) passes through the fixed point çè ÷ø = m1 =
dx (x y1
1 ,y1 )
2 3
(2, 3) Then, + =1 curve (ii):
h k
æ dy ö 9x
Then, the locus of R is
2 3
+ = 1 or 3x + 2y and çè dx ÷ø = m2 = - 1
x y (x1 ,y1 ) by1
= xy. Since, both the curves intersect each other
80. (1) Here, B – 2A = at right angle then,
40 20 40 20 27x1 x1
m1m 2 = -1 Þ = 1 Þ b = 27
å an - 2å a n = å a n - 2å an by12 y12
n =1 n =1 n =21 n =1
B – 2A = (212 + 2.222 + 232 + 2.242 + ..... + 402) 1 9
– (12 + 2.22 + 32 + 2.42 ..... + 202) \ from equation (i), b = 27 ´ =
6 2
= 20 [22 + 2.24 + 26 + 2.28 + ..... + 60] 82. (2) Since Orthocentre of the triangle is A(–3, 5)
and centriod of the triangle is B(3, 3), then
é + 24 + 26....60) + (24 + 28 + .... + 60) ù AB =
= 20 ê(22
14442444 3 144 42444 3ú 40 = 2 10
ë 20 terms 10 terms û
A B C
é 20 10 ù Centroid divides orthocentre and
20 ê (22 + 60) + (24 + 60) ú
ë2 2 û circumcentre of the triangle in ratio 2 : 1
= 10[20.82 + 10.84] \ AB : BC = 2 : 1
= 100[164 + 84] = 100.248 2
81. (3) Let curve intersect each other at point Now, AB = AC
3
P(x1, y1)
JEE Main Solved Paper–2018 2018-27

3 3 x-4 2x 2x
Þ AC = AB = (2 10) Þ AC = 3 10
2 2 2x x-4 2x = (Bx - 4)(x + 4) 2
\ Radius of circle with AC as diametre
2x 2x x-4
AC 3 5 Now take x common from both the sides
= = 10 = 3
2 2 2
83. (4) f (x) =| x - p | (e|x| - 1)sin | x |
4
1- 2x 2x
Check differentiability of f(x) at x = p and x = 0 x
at x = p : 4 4 4
2x 1- 2x = (B - )(1 + ) 2
\ x x x
| p + h- p | (e|x +h| - 1)sin | p + h | -0
R.H.D. = lim 4
h®0 h 2x 2x 1-
x
| p - h - p | (e|p-h| - 1)sin | p - h | -0
L.H.D = lim 1
h ®0 -h Now take x ® ¥ , then ®0
=0 x
Q RHD = LHD 1 2 2
Therefore, function is differentiable at x = p
Þ 2 1 2 = BÞ B =5
at x = 0:
2 2 1
| h - p | (e|h| - 1) sin | h | -0 \ ordered pair (A, B) is (–4, 5)
R.H.D. = lim =0
h ®0 h
85. (4) : (pÚ q) Ú (: p Ù q)
|-h|
| -h - p | (e - 1)sin | - h | -0 ( : p Ù : q) Ú (: p Ù q)
L.H.D. = lim =0
h ®0 -h Þ : pÙ (: q Ú q)
\ RHD = LHD
Þ : pÙ t º : p
Therefore, function is differentiable.
at x = 0. 86. (1) For non zero solution of the system of
linear equations;
Since, the function f(x) is differentiable at all the
points including p and 0. 1 k 3
i.e., f(x) is every where differentiable . 3 k -2 = 0
Therefore, there is no element in the set S.
2 4 -3
Þ S = f (an empty set)
x -4 2x 2x Þ k = 11
Now equations become
84. (2) Here, 2x x-4 2x = (A+Bx) (x–A)2 x + 11y + 3z = 0 ...(1)
2x 2x x -4 3x + 11y – 2z = 0 ...(2)
2x + 4y – 3z = 0 ...(3)
Put
Adding equations (1) & (3) we get
-4 0 0 3x + 15y = 0
x =0Þ 0 -4 0 = A3 Þ A3 = (-4)3 Þ x = –5y
Now put x = –5y in equation (1), we get
0 0 -4 –5y + 11y + 3z = 0
Þ A = -4 Þ z = –2y
Þ xz ( -5 y)( -2 y)
\ = = 10
2
y y2
EBD_7764
2018-28 JEE Main Solved Paper–2018

87. (2) Case-I: x Î[0,9]


2x 2
Þ y(x) = + c ...(2)
2(3 - x ) + x - 6 x + 6 = 0 sin x

Þ x - 8 x + 12 = 0 Þ x = 4, 2 æp ö
Q eq. (2) passes through ç ,0 ÷
Þ x = 16, 4 è2 ø
Since x Î[0,9] p2
\ x=4 Þ0= +C
2
Case-II: x Î[9, ¥]
p2
2( x - 3) + x - 6 x + 6 = 0 ÞC= -
2
Now, put the value of C in (1)
Þ x - 4 x = 0 Þ x = 16,0
Since x Î[9, ¥] p2
Then, ysin x = 2x 2 - is the solution
\ x = 16 2
Hence, x = 4 & 16
æpö æ p p2 ö
2
88. (3) Equation of tangent at (1, 7) to x2 = y – 6 8p2
is \ y ç 6 ÷ = çç 2. 36 - 2 ÷÷ 2 = - 9
è ø è ø
2x – y + 5 = 0.
90. (1) Equation of plane passing through the
line of intersection of first two planes is:
(–8, –6) 2x–y+5=0 (2 x - 2y + 3z - 2) + l (x - y+ z + 1) = 0
O or
P
x(l + 2) - y(2 + l ) + z(l + 3) + (l - 2) = 0
...(i)
is having infinite number of solution with
x + 2y – z – 3 = 0 and 3x – y + 2z – 1 = 0,
Now, perpendicular from centre O(–8, –6) then
to
2x – y + 5 = 0 should be equal to radius of
the circle (l + 2) -(l + 2) (l + 3)
\ -16 + 6 + 5 1 2 -1 = 0
= 64 + 36 - C
5 3 -1 2
Þ 5 = 100 - c Þ l=5
Þ c = 95 Now put l = 5 in (i), we get
89. (2) Consider the given differential equation 7x –7y + 8z + 3 = 0
the Now perpendicular distance from (0, 0, 0)
sinxdy + ycosxdx = 4xdx to the place containing L1 and L2 =
Þ d(y.sinx) = 4xdx
3 1
Integrate both sides =
Þ y.sinx = 2x2 + C ...(1) 162 3 2
www.crackjee.xyz

Topic-wise Solved Papers Physics

Physical World, Units


and Measurements 1
1. Identify the pair whose dimensions are equal (c) [MT–1 C–1] (d) [MT–2 C–1]
[2002] 7. A body of mass m = 3.513 kg is moving along
(a) torque and work the x-axis with a speed of 5.00 ms–1 . The
(b) stress and energy magnitude of its momentum is recorded as
(c) force and stress [2008]
(d) force and work (a) 17.6 kg ms–1 (b) 17.565 kg ms–1
(c) 17.56 kg ms–1 (d) 17.57 kg ms–1
1
2. Dimensions of , where symbols have their 8. Two full turns of the circular scale of a screw
mo e o gauge cover a distance of 1mm on its main scale.
usual meaning, are [2003] The total number of divisions on the circular
scale is 50. Further, it is found that the screw
(a) [L-1T] (b) [L-2 T 2 ] gauge has a zero error of – 0.03 mm. While
measuring the diameter of a thin wire, a student
(c) [L2 T -2 ] (d) [LT -1 ]
notes the main scale reading of 3 mm and the
3. The physical quantities not having same number of circular scale divisions in line with
dimensions are [2003] the main scale as 35. The diameter of the wire is
(a) torque and work (a) 3.32 mm (b) 3.73 mm [2008]
(b) momentum and planck’s constant (c) 3.67 mm (d) 3.38 mm
(c) stress and young’s modulus 9. In an experiment the angles are required to be
measured using an instrument, 29 divisions of
(d) speed and (m o e o ) -1 / 2 the main scale exactly coincide with the 30
4. Which one of the following represents the divisions of the vernier scale. If the smallest
correct dimensions of the coefficient of division of the main scale is half- a degree
viscosity? [2004] (= 0.5°), then the least count of the instrument is:
é MLT -1 ù [2009]
(a) é ML-1T -1 ù (b)
ë û ë û (a) half minute (b) one degree
(c) half degree (d) one minute
é ML-1T -2 ù -2 -2
(c) ë û (d) éë ML T ùû 10. The respective number of significant figures for
the numbers 23.023, 0.0003 and 2.1 × 10–3 are
5. Out of the following pair , which one does NOT [2010]
have identical dimensions ? [2005] (a) 5, 1, 2 (b) 5, 1, 5
(a) Impulse and momentum (c) 5, 5, 2 (d) 4, 4, 2
(b) Angular momentum and planck’s constant 11. A screw gauge gives the following reading when
(c) Work and torque used to measure the diameter of a wire.
(d) Moment of inertia and moment of a force Main scale reading : 0 mm
6. The dimensions of magnetic field in M, L, T and Circular scale reading : 52 divisions
C (coulomb) is given as [2008]
Given that 1mm on main scale corresponds to
(a) [MLT–1 C–1] (b) [MT2 C–2]
EBD_7764
www.crackjee.xyz
P-2 Physics
100 divisions of the circular scale. The diameter scale and main scale has 10 divisions
of wire from the above data is [2011] in 1 cm.
(c) A screw gauge having 100 divisions in the
(a) 0.052 cm (b) 0.026 cm
circular scale and pitch as 1 mm.
(c) 0.005 cm (d) 0.52 cm (d) A screw gauge having 50 divisions in the
12. Resistance of a given wire is obtained by circular scale and pitch as 1 mm.
measuring the current flowing in it and the 17. The period of oscillation of a simple pendulum
voltage difference applied across it. If the
percentage errors in the measurement of the L
is T = 2p . Measured value of L is 20.0 cm
current and the voltage difference are 3% each, g
then error in the value of resistance of the wire known to 1 mm accuracy and time for 100
is [2012] oscillations of the pendulum is found to be 90 s
(a) 6% (b) zero using a wrist watch of 1s resolution. The
(c) 1% (d) 3% accuracy in the determination of g is :
(a) 1% (b) 5%
13. A spectrometer gives the following reading when (c) 2% (d) 3%
used to measure the angle of a prism.
18. A student measures the time period of 100
Main scale reading : 58.5 degree oscillations of a simple pendulum four times. The
Vernier scale reading : 09 divisions data set is 90 s, 91 s, 95 s, and 92 s. If the minimum
Given that 1 division on main scale corresponds division in the measuring clock is 1 s, then the
to 0.5 degree. Total divisions on the Vernier scale reported mean time should be: [2016]
is 30 and match with 29 divisions of the main (a) 92 ± 1.8 s (b) 92 ± 3s
scale. The angle of the prism from the above (c) 92 ± 1.5 s (d) 92 ± 5.0 s
data is [2012] 19. A screw gauge with a pitch of 0.5 mm and a
(a) 58.59 degree (b) 58.77 degree circular scale with 50 divisions is used to
(c) 58.65 degree (d) 59 degree measure the thickness of a thin sheet of
Aluminium. Before starting the measurement, it
14. Let [Î0] denote the dimensional formula of the
is found that wen the two jaws of the screw
permittivity of vacuum. If M = mass, L = length,
gauge are brought in contact, the 45th division
T = time and A = electric current, then: [2013]
coincides with the main scale line and the zero
(a) Î0 = [M–1 L–3 T2 A]
of the main scale is barely visible. What is the
(b) Î0 = [M–1 L–3 T4 A2]
thickness of the sheet if the main scale reading
(c) Î0 = [M1 L2 T1 A2]
(d) Î0 = [M1 L2 T1 A] is 0.5 mm and the 25th division coincides with
15. The current voltage relation of a diode is given the main scale line? [2016]
by I = (e1000V/T – 1)mA, where the applied (a) 0.70 mm (b) 0.50 mm
voltage V is in volts and the temperature T is in (c) 0.75 mm (d) 0.80 mm
degree kelvin. If a student makes an error 20. The following observations were taken for
measuring ±0.01V while measuring the current determining surface tensiton T of water by
of 5 mA at 300 K, what will be the error in the capillary method :
value of current in mA? [2014] Diameter of capilary, D = 1.25 × 10–2 m
(a) 0.2 mA (b) 0.02 mA
rise of water, h = 1.45 × 10–2 m
(c) 0.5 mA (d) 0.05 mA
16. A student measured the length of a rod and Using g = 9.80 m/s2 and the simplified relation
wrote it as 3.50 cm. Which instrument did he rhg
T= ´103 N/m, the possible error in surface
use to measure it? [2014] 2
(a) A meter scale. tension is closest to : [2017]
(b) A vernier calliper where the 10 divisions in (a) 2. 4 % (b) 10 %
vernier scale matches with 9 division in main (c) 0.15% (d) 1.5%
www.crackjee.xyz
Physical World, Units and Measurements P-3

Answer Key
1 2 3 4 5 6 7 8 9 10 11 12 13 14 15
(a) (c) (b) (a) (d) (c) (a) (d) (d) (a) (a) (a) (c) (b) (a)
16 17 18 19 20
(b) (d) (c) (d) (d)

r r
1. (a) W = F × s = Fs cos q 8. (d) Least count of screw gauge
-2 2 -2 0.5
= [ MLT ][ L] = [ML T ] ; = mm = 0.01mm
r r r 50
t = r ´ F Þ t = rF sin q \ Reading = [Main scale reading + circular
scale reading × L.C] – (zero error)
= [ L ] [MLT -2 ] = [ ML2T -2 ] = [3 + 35 × 0.01] – (– 0.03) = 3.38 mm
2. (c) We know that the velocity of light in 9. (d) 30 Divisions of vernier scale coincide with
vacuum is given by 29 divisions of main scales
1 29
c= Therefore 1 V.S.D = MSD
mo eo 30
Least count = 1 MSD – 1VSD
1 29
\ m e = c21 = L2T–2 = 1 MSD - MSD
o o 30
3. (b) Momentum = mv = [MLT–1] 1
= MSD
Planck’s constant, 30
E [ ML2T –2 ] 1
h= = = [ML2T -1 ] = ´ 0.5° = 1 minute.
–1 30
v [T ] 10. (a) Number of significant figures in 23.023 = 5
4. (a) From Stokes law, Number of significant figures in 0.0003 = 1
F Number of significant figures in 2.1 × 10–3 = 2
F = 6ph r v Þ h = 6p r v So, the radiation belongs to X-rays part of
the spectrum.
[ MLT -2 ] 1
\h = Þ h = [ ML-1T -1 ] 11. (a) L.C. = mm
[ L][ LT -1 ] 100
Diameter of wire = MSR + CSR × L.C.
5. (d) Moment of Inertia, I = Mr 2 1
[I] = [ ML2 ] = 0+ × 52 = 0.52 mm = 0.052 cm
100
r uur uur
Moment of force, t = r ´ F V V ± DV
12. (a) R = Þ R ± DR =
r I I ± DI
é t ù = [ L][ MLT -2 ] = [ML2T -2 ]
ë û æ ö
6. (c) We know that F = q v B
æ DR ö V çç 1 ± DV / V ÷÷
F MLT -2 Rç1 ± ÷= DI
\ B= = = [MT -1C -1 ] è R ø I ç 1 ± ÷
qv C ´ LT -1 è I ø
7. (a) Momentum, p = m × v
æ DR ö æ DV ö + æ DI ö
= (3.513) × (5.00) = 17.565 kg m/s ç ÷ =ç ÷ ç ÷ = (3 + 3)% = 6%
= 17.6 (Rounding off to get three significant è R ø è V ø è I ø
figures)
EBD_7764
www.crackjee.xyz
P-4 Physics
13. (c) Q Reading of Vernier = Main scale reading
2 L
+ Vernier scale reading × least count. 17. (d) As, g = 4p
Main scale reading = 58.5 T2
Vernier scale reading = 09 division Dg DL DT
least count of Vernier = 0.5°/30 So, ´ 100 = ´ 100 + 2 ´ 100
g L T
0.5°
Thus, R = 58.5° + 9 × 0.1 1
30 = ´ 100 + 2 ´ ´ 100 = 2.72 ; 3%
20 90
R = 58.65°
14. (b) As we know, | DT1 | + | DT2 | + | DT3 | + | DT4 |
18. (c) DT =
4
1 q1q 2 q1q 2
F= Þ e0 = 2 +1+ 3 + 0
4pe 0 R 2 4pFR 2 = = 1.5
4
C2 [AT]2 As the resolution of measuring clock is 1.5
Hence, e0 = = therefore the mean time should be 92 ± 1.5
N.m 2 MLT -2 .L2
= [M–1 L–3 T4 A2] 0.5
19. (d) L.C. = = 0.01 mm
15. (a) The current voltage relation of diode is 50
Zero error = 5 × 0.01 = 0.05 mm (Negative)
I = (e1000 V /T - 1) mA (given) Reading = (0.5 + 25 × 0.01) + 0.05 = 0.80 mm
When, I = 5mA, e1000 V /T = 6mA rhg
20. (d) Surface tension, T = ´ 103
2
1000
Also, dI = (e1000 V /T
)´ Relative error in surface tension,
T
(By exponential function)
DT Dr Dh
1000 = + +0
= (6 mA) ´ ´ (0.01) = 0.2 mA T r h
300
16. (b) Measured length of rod = 3.50 cm (Q g, 2 and 103 are constant)
For vernier scale with 1 Main Scale Division
Percentage error
= 1 mm
9 Main Scale Division = 10 Vernier Scale
DT æ 10 –2 ´ 0.01 10 –2 ´ 0.01ö
Division, 100 ´ =ç + ÷ 100
Least count = 1 MSD –1 VSD = 0.1 mm T è 1.25 ´ 10 –2 1.45 ´ 10 –2 ø
= (0.8 + 0.689)
= (1.489) = 1.489% @ 1.5%
www.crackjee.xyz

Motion in a
Straight Line 2
1. If a body looses half of its velocity on penetrating 7h
(b) meters from the ground
3 cm in a wooden block, then how much will it 9
penetrate more before coming to rest? [2002] h
(c) meters from the ground
(a) 1 cm (b) 2 cm 9
(c) 3 cm (d) 4 cm. 17 h
(d) meters from the ground
2. Speeds of two identical cars are u and 4u at the 18
specific instant. The ratio of the respective 6. An automobile travelling with a speed of 60
distances in which the two cars are stopped from km/h, can brake to stop within a distance of 20m.
that instant is [2002] If the car is going twice as fast i.e., 120 km/h, the
(a) 1 : 1 (b) 1 : 4 stopping distance will be [2004]
(a) 60 m (b) 40 m
(c) 1 : 8 (d) 1 : 16
(c) 20 m (d) 80 m
3. From a building two balls A and B are thrown
7. A car, starting from rest, accelerates at the rate f
such that A is thrown upwards and B downwards through a distance S, then continues at constant
(both vertically). If vA and vB are their respective speed for time t and then decelerates at the rate
velocities on reaching the ground, then[2002]
f
(a) vB > vA to come to rest. If the total distance traversed
2
(b) vA = vB
is 15 S , then [2005]
(c) vA > vB
1
(d) their velocities depend on their masses. (a) S = ft 2 (b) S = f t
4. A car, moving with a speed of 50 km/hr, can be 6
stopped by brakes after at least 6 m. If the same 1 2 1 2
(c) S = ft (d) S = ft
car is moving at a speed of 100 km/hr, the minimum 4 72
stopping distance is [2003] 8. A particle is moving eastwards with a velocity
(a) 12 m (b) 18 m of 5 ms–1. In 10 seconds the velocity changes
(c) 24 m (d) 6 m to 5 ms–1 northwards. The average acceleration
5. A ball is released from the top of a tower of in this time is [2005]
height h meters. It takes T seconds to reach the 1 -2
(a) ms towards north
T 2
ground. What is the position of the ball at 1
3 (b) ms - 2 towards north-east
second [2004] 2
1
8h (c) ms - 2 towards north-west
(a) meters from the ground 2
9
(d) zero
EBD_7764
www.crackjee.xyz
P-6 Physics

9. The relation between time t and distance x is (x1 – x2)


t = ax2 + bx where a and b are constants. The
acceleration is [2005]
(a) –2bv 3 (b) –2abv 2
(d) O
t
(c) 2av2 (d) –2av 3
10. A particle located at x = 0 at time t = 0, starts 13. Consider a rubber ball freely falling from a height
moving along with the positive x-direction with h = 4.9 m onto a horizontal elastic plate. Assume
a velocity 'v' that varies as v = a x . The that the duration of collision is negligible and
displacement of the particle varies with time as the collision with the plate is totally elastic.
Then the velocity as a function of time and the
[2006]
height as a function of time will be : [2009]
(a) t 2 (b) t
v
(c) t 1/2 (d) t 3
+v1
11. The velocity of a particle is v = v0 + gt + ft2. If its
position is x = 0 at t = 0, then its displacement (a) O t
after unit time (t = 1) is [2007] –v1
(a) v0 + g /2 + f (b) v0 + 2g + 3f
y
(c) v0 + g /2 + f/3 (d) v0 + g + f
12. A body is at rest at x = 0. At t = 0, it starts h
moving in the positive x-direction with a
constant acceleration. At the same instant
another body passes through x = 0 moving in t
the positive x-direction with a constant speed. v
The position of the first body is given by x1(t)
+v1
after time ‘t’; and that of the second body by
x2(t) after the same time interval. Which of the t
following graphs correctly describes (x1 – x2) (b) O t1 2t1 4t1
as a function of time ‘t’? [2008] –v1
(x1 – x2)
y

h
t
(a) t
O
t
(x1 – x2)

(c) O t
t
t1 2t1
(b) O

(x1 – x2) y

(c) t
O
t
www.crackjee.xyz
Motion in a Straight Line P-7

15. From a tower of height H, a particle is thrown


v
v1 vertically upwards with a speed u. The time taken
by the particle, to hit the ground, is n times that
taken by it to reach the highest point of its path.
(d) O t The relation between H, u and n is: [2014]
(a) 2gH = n2u2
(b) gH = (n – 2)2 u2d
y (c) 2gH = nu2 (n – 2)
h (d) gH = (n – 2)u2
16. A body is thrown vertically upwards. Which
one of the following graphs correctly represent
t
the velocity vs time? [2017]

14. An object, moving with a speed of 6.25 m/s, is


decelerated at a rate given by (a) (b)
dv
= -2.5 v where v is the instantaneous
dt
speed. The time taken by the object, to come to
rest, would be: [2011]
(a) 2 s (b) 4 s (c) (d)
(c) 8 s (d) 1 s

Answer Key
1 2 3 4 5 6 7 8 9 10 11 12 13 14 15
(a) (d) (b) (c) (a) (d) (d) (c) (d) (a) (c) (b) (b) (a) (c)
16
(a)

1. (a) Activity A to B -3
u Þa = u2
u1 = u ; v1 = , s1 = 0.03 m, a1 = ? 4 ´ 0.06
2 Activity B to C: Assuming the same
v12 - u12 = 2a1s1 ...(i) retardation
-3
u u/2 speed = 0 u2 = u /2 ; v2 = 0 ; s2 = ? ; a2 = u2
4 ´ 0.06
A 3 cm B C v22 - u 22 = 2a2 ´ s2 ...(ii)

2 u2 æ -3 u 2 ö
æ uö \ 0- = 2ç
\ ç ÷ - u 2 = 2 ´ a ´ 0.03 ÷ ´ s2
è 2ø 4 è 4 ´ 0.06 ø
u2 3 2 1
Þ - u 2 = 0.06a Þ - u = 0.06a Þ s2 = m = 1 cm
4 4 100
EBD_7764
w w w . c r a c k j e e .
P-8 Physics
Alternatively, dividing (i) and (ii), v = 0, s = s, a = a \ v 2 - u 2 = 2as
v12 - u12 2a ´ s1 2
= 2 æ 5ö
v22 - u22 2a ´ s2 Þ 0 - ç100 ´ ÷ = 2as
è 18 ø
2 2
æ uö 2 æ 5ö
çè ÷ø - u Þ - ç100 ´ ÷ = 2as … (ii)
2 0.03 è 18 ø
Þ = Þ s2 = 1 cm. Dividing (i) and (ii) we get
2 s2
æ uö
0-ç ÷ 100 ´ 100 2 ´ a ´ s
è 2ø = Þ s = 24m
50 ´ 50 2´a´6
2. (d) For car 1
u1 = u, v1 = 0, a1 = – a, s1 = s1 1 2
5. (a) We have s = ut + gt ,
2
\ v12 - u12 = 2a1s1 Þ – u2 = – 2as1
1 2
Þ u2 = 2as1 ...(i) or h = gT (Q u = 0)
For car 2 2
u2 = 4u, v1 = 0, a2 = – a, s2 = s2 now for T/3 second, vertical distance
moved is given by
\ v22 - u22 = 2a2 s2 Þ – (4u)2 = 2(–a) s2 2
1 æTö 1 gT 2 h
Þ 16 u2 = 2as2 ...(ii) h' = g ç ÷ Þ h' = ´ =
Dividing (i) and (ii), 2 è 3ø 2 9 9
h
u2 2as1 1 s \ position of ball from ground = h -
= Þ = 1 9
2 2as2 16 s2
16u 8h
=
9
3. (b) Ball A is thrown upwards
from the building. During 5 50
u 6. (d) Speed, u = 60 ´ m/s = m/s
its downward journey A 18 3
u
when it comes back to 5 100
the point of throw, its d = 20m, u' = 120 ´ = m/s
18 3
speed is equal to the h Let declaration be a then (0) – u2 = –2ad
2
speed of throw. So, for or u2 = 2ad … (1)
the journey of both the B and (0)2 – u'2 = –2ad'
balls from point
A to B . or u '2 = 2 ad ' …(2)
We can apply v2 – u2 = 2gh. (2) divided by (1) gives,
As u, g, h are same for both the balls, d'
4 = Þ d ' = 4 ´ 20 = 80m
vA = vB d
5 1 2
4. (c) Case-1 : u = 50 ´ m / s, 7. (d) Distance from A to B = S = ft1
18 2
v = 0,s = 6m, a = a Distance from B to C = ( ft1 ) t
v 2 - u 2 = 2as u2 ( ft1 )2
2 Distance from C to D = =
æ 5ö 2a 2( f / 2)
Þ 02 - ç 50 ´ ÷ = 2 ´ a ´ 6
è 18 ø = ft12 = 2 S
2
æ 5ö A f B C f /2 D
Þ - ç 50 ´ ÷ = 2 ´ a ´ 6 ....(i)
è 18 ø t1 t 2t 1
5
Case-2 : u = 100 km/hr = 100 ´ m/sec 15 S
18
www.crackjee.xyz
Motion in a Straight Line P-9

Þ S + f t1t + 2 S = 15 S 1
2ax + b = .
Þ f t1t = 12 S ............. (i) v
Again differentiating,
1 2
f t1 = S ............ (ii) dx 1 dv
2 2a + 0 = - 2
dt v dt
t
Dividing (i) by (ii), we get t1 = dv 3 æ dx = vö
6 Þ = f = - 2av çèQ ÷ø
2 dt dt
1 ætö f t2
Þ S= fç ÷ = dx dx
2 è 6ø 72 10. (a) v=a x , =a xÞ = a dt
dt x
8. (c) Average acceleration
uur x
dx
t
x
=
change in velocity D v
= ò x = a ò dt ; éê 2 x ùú = a[t ]t0
time interval t 0 0
ë 1 û0
v2 a2 2
N Þ 2 x = at Þ x = t
4
D v = v 2 + (- v 1 ) 11. (c) We know that, v =
dx
Þ dx = v dt
dt
x t
90° Integrating, ò dx = ò v dt
W E 0 0
- v1 v1 t
or x = ò (v0 + gt + ft 2 ) dt
0
t
é gt 2 ft 3 ù
uur uur Suur ur ur = êv0 t + + ú
v1 = 5iˆ, v2 = 5 ˆj , D v = (v 2 - v 1 ) ëê 2 3 úû
0
= v12 + v22 + 2v1v2 cos 90 gt 2 ft 3
or, x = v0 t + +
2 3
= 5 2 + 52 + 0
g f
[As | v1 | = | v2 | = 5 m/s] At t = 1, x = v0 + + .
2 3
12. (b) For the body starting from rest
= 5 2 m/s
uur
Dv 5 2 1 1 2 1 2
Avg. acc. = = = m / s2 x1 = 0 + at Þ x1 = at
t 10 2 2 2
5 x1 – x2
tan q = = -1
-5
which means q is in the second quadrant.
(towards north-west)
9. (d) t = ax 2 + bx ; Diff. with respect to time (t)
d d dx dx
(t ) = a ( x 2 ) + b = a.2 x + b.v. v/a
t
dt dt dt dt
1 = 2axv + bv = v (2ax + b)(v = velocity)
For the body moving with constant speed
x2 = vt
EBD_7764
www.crackjee.xyz
P-10 Physics
1 2 graph (b).
\ x1 - x2 = at - vt
2 dv
14. (a) = -2.5 v
at t = 0, x1–x2 = 0 dt
v dv
For t < ; the slope is negative Þ = – 2.5 dt
a v
v Integrating,
For t = ; the slope is zero 0 -½ t
a ò6.25 v dv = -2.5ò dt
0
v 0
For t > ; the slope is positive é v +½ ù
= -2.5 [ t ]0
t
a Þ ê (½) ú
These characteristics are represented by ëê ûú 6.25
graph (b). Þ – 2(6.25)½ = – 2.5t
13. (b) For downward motion v = –gt Þ t = 2 sec
The velocity of the rubber ball increases in 15. (c) Speed on reaching ground
downward direction and we get a straight u
line between v and t with a negative slope. v= u 2 + 2 gh
H
1 2
Also applying y - y0 = ut + at Now, v = u + at
2
1 2 1 2 Þ u 2 + 2 gh = -u + gt
We get y - h = - gt Þ y = h - gt
2 2 Time taken to reach highest point is
The graph between y and t is a parabola u
with y = h at t = 0. As time increases y t= ,
g
decreases.
For upward motion. u + u 2 + 2 gH nu
The ball suffer elastic collision with the Þ t= = (from
g g
horizontal elastic plate therefore the
question)
direction of velocity is reversed and the
magnitude remains the same. Þ 2gH = n(n –2)u2
Here v = u – gt where u is the velocity just 16. (a) For a body thrown vertically upwards
acceleration remains constant (a = – g) and
after collision.
velocity at anytime t is given by V = u – gt
As t increases, v decreases. We get a straight During rise velocity decreases linearly and
line between v and t with negative slope. during fall velocity increases linearly and
1 2 direction is opposite to each other.
Also y = ut - gt Hence graph (a) correctly depicts velocity
2
All these characteristics are represented by versus time.
www.crackjee.xyz

Motion in a Plane 3
1. A boy playing on the roof of a 10 m high building 5. A ball is thrown from a point with a speed 'v0' at
throws a ball with a speed of 10m/s at an angle an elevation angle of q. From the same point
of 30º with the horizontal. How far from the and at the same instant, a person starts running
throwing point will the ball be at the height of 10 ' v0 '
m from the ground ? [2003] with a constant speed to catch the ball.
2
Will the person be able to catch the ball? If yes,
1 3
[ g = 10m/s2 , sin 30o = , cos 30o = ] what should be the angle of projection q ?[2004]
2 2
(a) No (b) Yes, 30°
(a) 5.20m (b) 4.33m
(c) Yes, 60° (d) Yes, 45°
(c) 2.60m (d) 8.66m
2. The co-ordinates of a moving particle at any 6. A particle has an initial velocity of 3iˆ + 4 ˆj and
time ‘t’are given by x = at3 and y = bt3. The an acceleration of 0.4iˆ + 0.3 ˆj . Its speed after
speed of the particle at time ‘t’ is given by 10 s is : [2009]
[2003]
(a) 7 2 units (b) 7 units
(a) 2 2 (b) 2 2 2 (c) 8.5 units (d) 10 units
3t a + b 3t a +b
r
7. A particle is moving with velocity n = k ( yiˆ + xjˆ) ,
(c) t 2 a 2 + b2 (d) a 2 + b2 where k is a constant. The general equation for
3. A projectile can have the same range ‘R’ for two its path is [2010]
angles of projection. If ‘T1’ and ‘T2’ to be time of (a) y = x2 + constant
flights in the two cases, then the product of the (b) y2 = x + constant
two time of flights is directly proportional to. (c) xy = constant
[2004] (d) y2 = x2 + constant
1 8. A point P moves in counter-clockwise direction
(a) R (b)
R on a circular path as shown in the figure. The
1 movement of 'P' is such that it sweeps out a
(c) (d) R 2
R2 length s = t3 + 5, where s is in metres and t is in
4. Which of the following statements is FALSE for seconds. The radius of the path is 20 m. The
a particle moving in a circle with a constant acceleration of 'P' when t = 2 s is nearly.[2010]
angular speed ? [2004] y
(a) The acceleration vector points to the centre
of the circle B
(b) The acceleration vector is tangent to the P(x,y)
circle
(c) The velocity vector is tangent to the circle m
20
(d) The velocity and acceleration vectors are
perpendicular to each other. x
O A
EBD_7764
www.crackjee.xyz
P-12 Physics
(a) 13m/s2 (b) 12 m/s2 13. Two stones are thrown up simultaneously from
(c) 7.2 ms2 (d) 14m/s2 the edge of a cliff 240 m high with initial speed of
9. For a particle in uniform circular motion, the 10 m/s and 40 m/s respectively. Which of the
r following graph best represents the time
acceleration a at a point P(R,q) on the circle of
radius R is (Here q is measured from the x-axis ) variation of relative position of the second stone
[2010] with respect to the first ? [2015]
n2 n2 (Assume stones do not rebound after hitting
(a) - cos q iˆ + sin q ˆj the ground and neglect air resistance, take
R R
g = 10 m/s2)
n2 n2
(b) - sin q iˆ + cos q ˆj (The figures are schematic and not drawn to
R R scale)
n2 n2
(c) - cos q iˆ - sin q ˆj
R R (y2 – y1) m
2 2 (a) 240
n ˆ n ˆ
(d) i+ j
R R
10. A water fountain on the ground sprinkles water
all around it. If the speed of water coming out of t(s)
8 12
the fountain is v, the total area around the
fountain that gets wet is : [2011] (y2 – y1 ) m
v4 p v4 (b) 240
(a) p (b)
g2 2 g2
v2 v2
(c) p (d) p
g2 g t(s)
8 12
11. A boy can throw a stone up to a maximum height
of 10 m. The maximum horizontal distance that (y2 – y1 ) m
the boy can throw the same stone up to will be (c) 240
[2012]
(a) 20 2 m (b) 10 m
(c) 10 2 m (d) 20 m t(s)
t® 8 12
12. A projectile is given an initial velocity of
(y2 – y1 ) m
(iˆ + 2 ˆj ) m/s, where iˆ is along the ground and (d) 240
ĵ is along the vertical. If g = 10 m/s2, the
equation of its trajectory is : [2013]
(a) y = x – 5x 2 (b) y = 2x – 5x2
2 t(s)
(c) 4y = 2x – 5x (d) 4y = 2x – 25x2 12

Answer Key
1 2 3 4 5 6 7 8 9 10 11 12 13
(d) (b) (a) (b) (c) (a) (a) (d) (c) (a) (d) (b) (b)
www.crackjee.xyz
Motion in a Plane P-13

1. (d) From the figure it is clear that range is r


required
7. (a) v = k ( y iˆ + x ˆj )
x-component of v = ky
u 2 sin 2q (10)2 sin(2 ´ 30°) dx
R= = =5 3 = ky
g 10 Þ ...(1)
dt
u y-component of v = kx
30° dy
Range R Þ = kx ...(2)
dt
dy x
From (1) and (2) , dx = y
10m

10m

Tower
Þ ydy = xdx Þ y2 = x2 + constant
8. (d) s = t3 + 5
2. (b) x = at3 and y = bt3 ds
= 3t 2
Þ velocity, v =
dx dy dt
vx = = 3at 2 and v y = = 3bt 2
dt dt dv
Tangential acceleration at = = 6t
\ v = v 2x + v2y = 9a 2t 4 + 9b2t 4 dt
v 2 9t 4
2 2 2 Radial acceleration ac = =
= 3t a + b R R
3. (a) The angle for which the ranges are same is At t = 2s, at = 6 × 2 = 12 m/s2
complementary. 9 ´ 16
Let one angle be q, then other is 90° – q ac = = 7.2 m/s2
20
2u sin q 2u cos q \ Resultant acceleration
T1 = , T2 =
g g = at2 + ac2
2
4u sin q cos q
T1T2 = =2R = (12) 2 + (7.2) 2 = 144 + 51.84
g
= 195.84 = 14 m/s2
u 2 sin 2 q r
(Q R = ) 9. (c) Clearly a = ac cos q(-iˆ) +ac sin q(- ˆj )
g
Hence it is proportional to R. -v 2 v2
4. (b) Only option (b) is false since acceleration = cos q iˆ - sin q ˆj
vector is always radial (i.e., towards the R R
center) for uniform circular motion. Y
5. (c) Yes, the person can catch the ball when
horizontal velocity is equal to the horizontal P( R, q)
component of ball’s velocity, the motion
of ball will be only in vertical direction with R
q
respect to person for that, X
O
vo
= vo cos q or q = 60°
2
r r
6. (a) Given u = 3iˆ + 4 ˆj , a = 0.4iˆ + 0.3 ˆj , t = 10 s 10. (a) Total area around fountain
r r r
v = u + at = 3iˆ + 4 ˆj + (0.4iˆ + 0.3 ˆj) ´ 10 2
A = pRmax
= 7iˆ + 7 ˆj v 2 sin 2q v 2 sin 90° v 2
Where Rmax = = =
r g g g
\ | v |= 72 + 72 = 7 2 units
EBD_7764
www.crackjee.xyz
P-14 Physics
v4
r
12. (b) From equation, v = iˆ + 2 ˆj
\ A= p 2
g Þ x=t … (i)
2 2 2 2
u sin q u sin q 1
11. (d) R = ,H= y = 2t - (10t 2 ) … (ii)
g 2g 2
From (i) and (ii), y = 2x – 5x2
Hmax at 2q = 90°
13. (b) y1 = 10t – 5t2 ; y2 = 40t – 5t2
u2 for y1 = – 240m, t = 8s
Hmax =
2g \ y2 – y1 = 30t for t < 8s.
u2 for t > 8s,
= 10 Þ u2 = 10 g ´ 2
2g 1
y2 – y1 = 240 – 40t – gt2
u 2 sin 2q u2 2
R= Þ Rmax =
g g
10 ´ g ´ 2
Rmax = = 20 metre
g
www.crackjee.xyz
Laws of Motion P-15

Laws of Motion 4
1. A lift is moving down with acceleration a. A man 7. Three identical blocks of masses m = 2 kg are
in the lift drops a ball inside the lift. The drawn by a force F = 10. 2 N with an acceleration
acceleration of the ball as observed by the man of 0. 6 ms-2 on a frictionless surface, then what
in the lift and a man standing stationary on the is the tension (in N) in the string between the
ground are respectively [2002] blocks B and C? [2002]
(a) g, g (b) g – a, g – a
C B A F
(c) g – a, g (d) a, g
2. When forces F1, F2, F3 are acting on a particle
of mass m such that F2 and F3 are mutually (a) 9.2 (b) 3.4
perpendicular, then the particle remains (c) 4 (d) 9.8
stationary. If the force F1 is now removed then 8. One end of a massless rope, which passes over a
the acceleration of the particle is [2002] massless and frictionless pulley P is tied to a hook
(a) F1/m (b) F2F3 /mF1 C while the other end is free. Maximum tension
(c) (F2 - F3)/m (d) F2 /m. that the rope can bear is 360 N. With what value
3. The minimum velocity (in ms-1) with which a car of maximum safe acceleration (in ms-2) can a man
driver must traverse a flat curve of radius 150 m of 60 kg climb on the rope? [2002]
and coefficient of friction 0.6 to avoid skidding is P
(a) 60 (b) 30 [2002]
(c) 15 (d) 25 C
4. A solid sphere, a hollow sphere and a ring are
released from top of an inclined plane
(frictionless) so that they slide down the plane.
Then maximum acceleration down the plane is (a) 16 (b) 6
for (no rolling) [2002] (c) 4 (d) 8
(a) solid sphere (b) hollow sphere
9. A spring balance is attached to the ceiling of a
(c) ring (d) all same. lift. A man hangs his bag on the spring and the
5. Two forces are such that the sum of their spring reads 49 N, when the lift is stationary. If
magnitudes is 18 N and their resultant is 12 N the lift moves downward with an acceleration
which is perpendicular to the smaller force. Then
the magnitudes of the forces are [2002] of 5 m / s 2 , the reading of the spring balance
(a) 12 N, 6 N (b) 13 N, 5 N will be [2003]
(c) 10 N, 8 N (d) 16N, 2N. (a) 24 N (b) 74 N
6. A light string passing over a smooth light pulley (c) 15 N (d) 49 N
connects two blocks of masses m1 and m 2 10. Three forces start acting simultaneously on a
(vertically). If the acceleration of the system is r
particle moving with velocity, v . These forces
g/8, then the ratio of the masses is [2002] are represented in magnitude and direction by
(a) 8 : 1 (b) 9 : 7 the three sides of a triangle ABC. The particle
(c) 4 : 3 (d) 5 : 3.
EBD_7764
www.crackjee.xyz
P-16 Physics
will now move with velocity [2003] pulley. What is the acceleration of the masses
r C when left free to move ? ( g = 9.8m / s2 ) [2004]
(a) less than v
r
(b) greater than v
r (a) 5 m / s2
(c) v in the direction of
the largest force BC (b) 9.8 m / s 2
A
(d) vr , remaining unchanged
B
11. A horizontal force of 10 N is necessary to just (c) 0.2 m / s 2
hold a block stationary against a wall. The
coefficient of friction between the block and the (d) 4.8 m / s 2
wall is 0.2. The weight of the block is [2003]
17. A block rests on a rough inclined plane making
(a) 20 N an angle of 30° with the horizontal. The
(b) 50 N coefficient of static friction between the block
and the plane is 0.8. If the frictional force on the
(c) 100 N 10N block is 10 N, the mass of the block (in kg) is
(d) 2 N (take g = 10 m / s 2 ) [2004]
12. A marble block of mass 2 kg lying on ice when (a) 1.6 (b) 4.0
given a velocity of 6 m/s is stopped by friction (c) 2.0 (d) 2.5
in 10 s. Then the coefficient of friction is [2003] 18. A smooth block is released at rest on a 45° incline
(a) 0.02 (b) 0.03 and then slides a distance ‘d’. The time taken to
(c) 0.04 (d) 0.06 slide is ‘n’ times as much to slide on rough incline
13. A block of mass M is pulled along a horizontal than on a smooth incline. The coefficient of
frictionless surface by a rope of mass m. If a friction is [2005]
force P is applied at the free end of the rope, the
1 1
force exerted by the rope on the block is[2003] (a) mk = 1 – 2 (b) mk = 1-
Pm Pm n n2
(a) (b)
M +m M -m
1 1
PM (c) ms = 1 - (d) ms = 1-
(c) P (d)
M +m n 2
n2
14. A light spring balance hangs from the hook of 19. A parachutist after bailing out falls 50 m without
the other light spring balance and a block of friction. When parachute opens, it decelerates
mass M kg hangs from the former one. Then the at 2 m/s2 . He reaches the ground with a speed
true statement about the scale reading is [2003] of 3 m/s. At what height, did he bail out ?
(a) both the scales read M kg each [2005]
(b) the scale of the lower one reads M kg and (a) 182 m (b) 91 m
of the upper one zero (c) 111 m (d) 293 m
(c) the reading of the two scales can be 20. An annular ring with inner and outer radii R1
anything but the sum of the reading will be
M kg and R2 is rolling without slipping with a uniform
(d) both the scales read M/2 kg each angular speed. The ratio of the forces
experienced by the two particles situated on the
15. A rocket with a lift-off mass 3.5 ´ 10 4 kg is
blasted upwards with an initial acceleration of F1
inner and outer parts of the ring , F is[2005]
10m/s2. Then the initial thrust of the blast is 2
(a) 3.5 ´ 10 5 N (b) 7.0 ´ 10 5 N [2003] 2 R2
æ R1 ö
(c) 14.0 ´ 10 N5 (d) 1.75 ´ 10 N 5 (a) çè R ÷ø (b) R1
2
16. Two masses m1 = 5g and m2 = 4.8 kg tied to R1
a string are hanging over a light frictionless (c) R2 (d) 1
www.crackjee.xyz
Laws of Motion P-17
21. The upper half of an inclined plane with horizontal plane. Initially the blocks are at rest and
inclination f is perfectly smooth while the lower the spring is unstretched. Then a constant force F
half is rough. A body starting from rest at the starts acting on the block of mass M to pull it. Find
top will again come to rest at the bottom if the the force of the block of mass m. [2007]
coefficient of friction for the lower half is given MF mF
(a) (b)
by [2005] (m + M ) M
(a) 2 cos f (b) 2 sin f mF
(c) ( M + m) F (d)
(c) tan f (d) 2 tan f m (m + M )
22. A particle of mass 0.3 kg subject to a force 28. Two fixed frictionless inclined planes making an
F = – kx with k = 15 N/m . What will be its initial angle 30° and 60° with the vertical are shown in
acceleration if it is released from a point 20 cm the figure. Two blocks A and B are placed on the
away from the origin ? [2005] two planes. What is the relative vertical
acceleration of A with respect to B ? [2010]
(a) 15 m / s 2 (b) 3 m / s 2
A
(c) 10 m / s 2 (d) 5 m / s 2 B
23. A block is kept on a frictionless inclined surface
with angle of inclination ‘ a ’ . The incline is given
an acceleration ‘a’ to keep the block stationary.
Then a is equal to [2005]
60° 30°
(a) 4.9 ms–2 in horizontal direction
(b) 9.8 ms–2 in vertical direction
(c) Zero
(d) 4.9 ms–2 in vertical direction
a 29. The minimum force required to start pushing a
a body up rough (frictional coefficient m) inclined
(a) g cosec a (b) g / tan a plane is F1 while the minimum force needed to
(c) g tan a (d) g prevent it from sliding down is F2. If the inclined
24. Consider a car moving on a straight road with a plane makes an angle q from the horizontal such
speed of 100 m/s . The distance at which car can F
that tan q = 2m then the ratio 1 is [2011 RS]
be stopped is [ m k = 0.5 ] [2005] F2
(a) 1000 m (b) 800 m (a) 1 (b) 2
(c) 400 m (d) 100 m (c) 3 (d) 4
25. A ball of mass 0.2 kg is thrown vertically upwards 30. If a spring of stiffness ‘k’ is cut into parts ‘A’
by applying a force by hand. If the hand moves
0.2 m while applying the force and the ball goes and ‘B’ of length l A : l B = 2 : 3, then the
upto 2 m height further, find the magnitude of stiffness of spring ‘A’ is given by [2011 RS]
the force. (Consider g = 10 m/s2). [2006]
3k 2k
(a) 4 N (b) 16 N (a) (b)
(c) 20 N (d) 22 N 5 5
26. A player caught a cricket ball of mass 150 g 5k
moving at a rate of 20 m/s. If the catching process (c) k (d)
is completed in 0.1s, the force of the blow exerted 2
by the ball on the hand of the player is equal to 31. A particle of mass m is at rest at the origin at time
[2006] t = 0. It is subjected to a force F(t) = F0e–bt in the
(a) 150 N (b) 3 N x direction. Its speed v(t) is depicted by which
(c) 30 N (d) 300 N of the following curves? [2012]
27. A block of mass m is connected to another block
of mass M by a spring (massless) of spring
constant k. The block are kept on a smooth
EBD_7764
www.crackjee.xyz
P-18 Physics
F0 33. A block of mass m is placed on a surface with a
mb
x3
(a) v(t)
vertical cross section given by y = . If the
6
t coefficient of friction is 0.5, the maximum height
F0 above the ground at which the block can be
mb placed without slipping is: [2014]
(b)
v(t) 1 2
(a) m (b) m
t 6 3
F0 1 1
mb (c) m (d) m
3 2
(c) v(t) 34. Given in the figure are two blocks A and B of
t weight 20 N and 100 N, respectively. These are
F0 being pressed against a wall by a force F as
mb shown. If the coefficient of friction between the
blocks is 0.1 and between block B and the wall is
(d) v(t) 0.15, the frictional force applied by the wall on
t block B is: [2015]
32. A mass ‘m’ is supported by a massless string
wound around a uniform hollow cylinder of mass
m and radius R. If the string does not slip on the
cylinder, with what acceleration will the mass F
A B
fall or release? [2014]
2g
(a)
3

g
(b) R
2
m
5g
(c) (a) 120 N (b) 150 N
6 (c) 100 N (d) 80 N
(d) g
m

Answer Key
1 2 3 4 5 6 7 8 9 10 11 12 13 14 15
(c) (a) (b) (d) (b) (b) (b) (c) (a) (d) (d) (d) (d) (a) (b)
16 17 18 19 20 21 22 23 24 25 26 27 28 29 30
(c) (c) (b) (d) (c) (d) (c) (c) (a) (d) (c) (d) (d) (c) (d)
31 32 33 34
(c) (b) (a) (a)
www.crackjee.xyz
Laws of Motion P-19

6. (b) For mass m1


1. (c) • For the man standing in the left, the m1g – T = m1a
acceleration of the ball
r r r For mass m2
abm = ab - am Þ abm = g – a T–m2g = m2a
Where 'a' is the acceleration of the
mass (because the acceleration of the
lift is 'a' )
• For the man standing on the ground,
the acceleration of the ball
r r r
abm = ab - am Þ abm = g – 0 = g T T
2. (a) When F1 , F2 and F3 are acting on a particle a m2
then the particle remains stationary. This m1 a
means that the resultant of F1 , F2 and F3 is m2g
zero. When F1 is removed, F2 and F3 will
remain. But the resultant of F2 and F3 m1g
Adding the equations we get
should be equal and opposite to F1. i.e. |F2
+ F3| = |F1| (m - m2 ) g
a= 1
| F2 + F3 | F1 m1 + m2
\ a= Þ a= g
m m
3. (b) For negotiating a circular curve on a levelled Here a =
8
road, the maximum velocity of the car is
m1
vmax = mrg -1
1 m2 m1 m
Here m = 0.6, r = 150 m, g = 9.8 \ = Þ +1 = 8 1 - 8
8 m1 m2 m2
\ vmax = 0.6´150´9.8 ; 30m / s +1
m2
4. (d) This is a case of sliding (the plane being
frictionless) and therefore the acceleration m1 9
of all the bodies is same (g sin q). Þ =
m2 7
7. (b) F = ( m + m + m) ´ a
g sin q
10.2
\a = m / s2
6
10.2
q \ T2 = ma = 2 ´ = 3.4N
6
5. (b) Let the two forces be F1 and F2 and let F2
F
< F1 . R is the resultant force. T2
Given F1 + F2 = 18 ...(i) C B A
T2 T1 T1
From the figure F22 + 2
R = F12 8. (c) mg – T = ma
F12 - F22 = R 2 \ a=g-
T
= 10 -
360
= 4m / s 2
\ F12 - F22 = 144 ...(ii) m 60
Only option (b) follows equation (i) and T
(ii).
F1
a
®

F2 R
mg
mg
F1
EBD_7764
www.crackjee.xyz
P-20 Physics
9. (a) For the bag accelerating down we get P = (m + M) a
mg – T = ma P
\ a=
m+M
Taking the block as a system,
we get T = Ma
T MP
\ T=
a m+M
mg 14. (a) The Earth pulls the block by a force Mg.
The block in turn exerts a force Mg on the
spring of spring balance S1 which therefore
49 shows a reading of M kgf.
\ T = m( g – a ) = (10 – 5) = 24.5 N The spring S1 is massless. Therefore, it
10 exerts a force of Mg on the spring of spring
10. (d) As shown in the figure, the three forces balance S2 which shows the reading of M
are represented by the sides of a triangle kgf.
taken in the same order. Therefore the
r r
resultant force is zero. Fnet = ma.
Therefore, acceleration is also zero i.e., s2 Mkgf
velocity remains unchanged.
11. (d) For the block to remain stationary with the
wall
f=W s1 Mkgf
f = mN
M
Mg
10N 10N 10N
15. (b) As shown in the figure F – mg = ma
W Thrust (F)
mN = W
0.2 × 10 = W Þ W = 2 N

12. (d) u = 6 m/s, v = 0, t = 10s, a = ?


v = u + at Þ 0 = 6 + a ´ 10
-6 a
Þ a= = -0.6m / s2
10
mg
mg
f = mN N
\ F = m ( g + a) = 3.5 × 104 ( 10+10)
The retardation is due to the frictional force
= 7 × 105 N
\ f = ma Þ mN = ma Þ mmg = ma
æ m - m2 ö
a 0.6 16. (c) Acceleration a = ç 1 g
Þm= = = 0.06 è m1 + m2 ÷ø
g 10
13. (d) Taking the rope and the block as a system (5 - 4.8) ´ 9.8
a = m / s2 = 0.2 m/s2
(5 + 4.8)
M m
T P
www.crackjee.xyz
Laws of Motion P-21
17. (c) fs
N
50 m
mg
v
30° a = - 2 m / s2
mg sinq = f s ( for body to be at rest)
Þ m ´ 10 ´ sin 30° = 10
3m / s
Þ m ´ 5 = 10 Þ m = 2.0 kg
The velocity at ground,
18. (b) g sin q - mg cos q
v = 3m/s
d q v2 - u2 32 - 980
n
g si d S= = » 243 m
45° 45° 2´2 4
Initially he has fallen 50 m.
smooth rough
When surface is When surface is \ Total height from where
he bailed out = 243 + 50 = 293 m
smooth rough
20. (c) a2
1
d = ( g sin q)t12 , R2
2
R1 v 2 = wR 2
1 2
d = ( g sin q - mg cos q) t2 a1
2 v1 = wR 1
2d 2d
t1 = , t2 =
g sin q g sin q - mg cos q
According to question, t2 = nt1 v12 w 2 R12
a1 = = = w 2 R1
2d 2d R1 R1
n =
g sin q g sin q - mg cos q v22
a2 = = w 2 R2
m , as applicable here, is coefficient of R2
kinetic friction as the block moves over the Taking particle masses equal
inclined plane.
F1 ma1 a1 R1
1 = = =
n= F2 ma2 a2 R2
1- m k
æ 1 ö The force experienced by any particle is
ççQ cos 45° = sin 45° = ÷÷ only along radial direction.
è 2ø Force experienced by the particle, F = mw2R
1 1 F R
n2 = or 1- m k = 2 \ 1 = 1
1 - mk n F2 R2
1 21. (d) Acceleration of block while sliding down
or m k = 1 - 2 upper half
n = g sin f ;
19. (d) Initial velocity of parachute
after bailing out, retardation of block while sliding down
lower half
u= 2gh = – ( g sin f - mg cos f)
u= 2 ´ 9.8 ´ 50 = 14 5 For the block to come to rest at the bottom,
acceleration in I half = retardation in II half.
EBD_7764
www.crackjee.xyz
P-22 Physics
g sin f = -( g sin f - mg cos f) Whand + Wgravity = DK
Þ m = 2 tan f
Þ F (0.2) + (0.2)(10)(2.2) = 0 Þ F = 22 N
According to work-energy theorem, 26. (c) m(v - u ) 0.15(0 - 20)
W = DK = 0 F= = = 30 N
t 0.1
(Since initial and final speeds are zero) 27. (d) Writing free body-diagrams for m & M,
\ Workdone by friction + Work done M
by gravity = 0 m
K
l F
i.e., -( µ mg cos f ) + mg l sin f = 0
2 N N
µ a
or cos f = sin f or µ = 2 tan f T T
2 m M
F
22. (c) Mass (m) = 0.3 kg Þ F = m.a = – 15 x
15 -150 mg Mg
a=– x= x = - 50 x we get T = ma and F – T = Ma
0.3 3
where T is force due to spring
a = – 50 × 0.2 = 10 m / s 2 Þ F – ma = Ma or,, F = Ma + ma
23. (c) From free body diagram, F
\ a= .
mag cos M +m
N Now, force acting on the block of mass m is
a a æ F ö = mF
ma = m ç .
a è M + m ÷ø m + M
a 28. (d) mg sin q = ma
mg cosa
+ ma sina mg mg sin a \ a = g sin q
where a is along the inclined plane
For block to remain stationary, \ vertical component of acceleration is g
mg sin a = ma cos a Þ a = g tan a sin2 q
\ relative vertical acceleration of A with
24. (a) v 2 - u 2 = 2 as or respect to B is
02 - u 2 = 2( -m k g ) s g
g (sin 2 60 - sin 2 30] = = 4.9 m/s2
1 2
-1002 = 2 ´ - ´ 10 ´ s in vertical direction
2 29. (c) N1 F1
s = 1000 m
25. (d) Let the velocity of the ball just when it
leaves the hand is u then applying,
v2 – u2 = 2as for upward journey
Þ -u 2 = 2( -10) ´ 2 Þ u 2 = 40 mg sin q
f1 mg cos q
Again applying v2 – u2 = 2as for the upward mg
journey of the ball, when the ball is in the q
hands of the thrower, N2
v2 – u2 = 2as
2
F

f2
Þ 40 - 0 = 2 (a) 0.2 Þ a = 100 m/s2
\ F = ma = 0.2 ´ 100 = 20 N
Þ N - mg = 20 Þ N = 20 + 2 = 22 N mg sin q
mg cos q
q mg
www.crackjee.xyz
Laws of Motion P-23
For the upward motion of the body or T = ma
mg sin q + f1 = F1 Þ mg – ma = ma
or, F1 = mg sin q + mmg cos q g
For the downward motion of the body, Þ a=
mg sin q – f 2 = F2 2
33. (a) At limiting equilibrium,
or F2 = mg sin q – mmg cos q m = tanq
F1 sin q + m cos q
\ =
F2 sin q - m cos q
tan q + m 2m + m 3m m
Þ = = =3
tan q - m 2m - m m y
q
2l æ 3l ö
30. (d) Here, l A = , l B = ç ÷
5 è 5ø
k l = k Al A = k B l B
æ 2l ö
kl = kA ç ÷
è 5ø
5k dy x 2
kA = tanq = m = = (from question)
2 dx 2
31. (c) Given that F(t) = F0 e -bt Q Coefficient of friction m = 0.5
dv x2
Þ m = F0 e -bt \ 0.5 =
dt 2
dv F0 -bt Þ x=+1
= e
dt m
v t x3 1
F Now, y = = m
ò dv = m0 ò e -bt dt 6 6
0 0 f1 f2

ée -bt ù t
v=
F0
ê ú =
F0 é
êë -b úû 0 mb
m ë ( )
- e - bt - e -0 ù
û 34. (a) F
A B
N

F
Þ v = 0 é1 - e -bt ù
mb ë û 20N
f1
100N
32. (b) From figure, Assuming both the blocks are stationary
N= F
f1 = 20 N
a f2 = 100 + 20 = 120N
R f

T
m a
mg
Acceleration a = Ra …(i) 120N
Considering the two blocks as one system
and mg – T = ma …(ii) and due to equilibrium f = 120N
From equation (i) and (ii)
æ aö
T × R = mR2a = mR2 çè ÷ø
R
EBD_7764
www.crackjee.xyz
P-24 Physics

Work, Energy
and Power 5
1. Consider the following two statements :[2003] (c) 7.2 J (d) 1200 J
A. Linear momentum of a system of particles r r r r
7. A force F = (5i + 3 j + 2k ) N is applied over a
is zero
particle which displaces it from its origin to the
B. Kinetic energy of a system of particles is zero. r r r
Then point r = (2i - j )m. The work done on the
(a) A does not imply B and B does not imply A particle in joules is [2004]
(b) A implies B but B does not imply A (a) +10 (b) +7
(c) A does not imply B but B implies A
(d) A implies B and B implies A (c) –7 (d) +13
2. A wire suspended vertically from one of its ends 8. A body of mass ‘m’, accelerates uniformly from
is stretched by attaching a weight of 200N to rest to ‘v1’ in time ‘t1’. The instantaneous power
the lower end. The weight stretches the wire by delivered to the body as a function of time ‘t’ is
1 mm. Then the elastic energy stored in the wire [2004]
is [2003] mv1t 2 mv12t
(a) 0.2 J (b) 10 J (a) (b)
(c) 20 J (d) 0.1 J t1 t12
3. A spring of spring constant 5 × 103 N/m is mv1t mv12t
stretched initially by 5cm from the unstretched (c) t1 (d)
t1
position. Then the work required to stretch it
further by another 5 cm is [2003] 9. A particle is acted upon by a force of constant
(a) 12.50 N-m (b) 18.75 N-m magnitude which is always perpendicular to the
(c) 25.00 N-m (d) 6.25 N-m velocity of the particle, the motion of the particles
4. A body is moved along a straight line by a takes place in a plane. It follows that [2004]
machine delivering a constant power. The (a) its kinetic energy is constant
distance moved by the body in time ‘t’ is (b) its acceleration is constant
proportional to [2003]
(c) its velocity is constant
(a) t 3/4 (b) t 3/2
(c) t 1/4 (d) t 1/2 (d) it moves in a straight line
5. A particle moves in a straight line with retardation 10. The block of mass M moving on the frictionless
proportional to its displacement. Its loss of horizontal surface collides with the spring of
kinetic energy for any displacement x is spring constant k and compresses it by length
proportional to [2004] L. The maximum momentum of the block after
(a) x (b) ex collision is [2005]
(c) x2 (d) loge x M
6. A uniform chain of length 2 m is kept on a table
such that a length of 60 cm hangs freely from
the edge of the table. The total mass of the chain
is 4 kg. What is the work done in pulling the kL2
entire chain on the table ? [2004] (a) (b) Mk L
2M
(a) 12 J (b) 3.6 J ML2
(c) (d) zero
k
www.crackjee.xyz
Work, Energy & Power P-25
11. A spherical ball of mass 20 kg is stationary at (c) 1.25 J (d) 0.5 J
the top of a hill of height 100 m. It rolls down a 16. The potential energy of a 1 kg particle free to
smooth surface to the ground, then climbs up move along the x-axis is given by
another hill of height 30 m and finally rolls down
to a horizontal base at a height of 20 m above æ x 4 x2 ö
V ( x) = ç - ÷ J.
the ground. The velocity attained by the ball is è 4 2ø
[2005]
(a) 20 m/s (b) 40 m/s The total mechanical energy of the particle is 2
J. Then, the maximum speed (in m/s) is [2006]
(c) 10 30 m/s (d) 10 m/s
3
12. A body of mass m is accelerated uniformly from (a) (b) 2
rest to a speed v in a time T. The instantaneous 2
power delivered to the body as a function of 1
time is given by [2005] (c) (d) 2
2
mv 2 mv 2 17. A 2 kg block slides on a horizontal floor with a
(a) 2
.t 2 (b) 2
.t
T T speed of 4m/s. It strikes a uncompressed spring,
and compresses it till the block is motionless. The
1 mv 2 2 1 mv 2 kinetic friction force is 15N and spring constant
(c) .t (d) .t is 10,000 N/m. The spring compresses by [2007]
2 T2 2 T2
(a) 8.5 cm (b) 5.5 cm
13. A mass ‘m’ moves with a velocity ‘v’ and collides (c) 2.5 cm (d) 11.0 cm
inelastically with another identical mass . After 18. An athlete in the olympic games covers a
distance of 100 m in 10 s. His kinetic energy can
collision the lst mass moves with velocity
v
be estimated to be in the range [2008]
3
(a) 200 J - 500 J (b) 2 × 105 J - 3 × 105 J
in a direction perpendicular to the initial direction (c) 20, 000 J - 50,000 J (d) 2,000 J - 5, 000 J
of motion. Find the speed of the 2 nd mass after 19. A block of mass 0.50 kg is moving with a speed of
collision. [2005] 2.00 ms–1 on a smooth surface. It strikes another
mass of 1.00 kg and then they move together as
m m v a single body. The energy loss during the
3
A before Aafter collision is [2008]
collision collision (a) 0.16 J (b) 1.00 J
(a) 3v (b) v (c) 0.67 J (d) 0.34 J
20. The potential energy function for the force
v 2 between two atoms in a diatomic molecule is
(c) (d) v
3 3 a b
approximately given by U(x) = 12 - 6 , where
14. A bomb of mass 16kg at rest explodes into two x x
pieces of masses 4 kg and 12 kg. The velolcity a and b are constants and x is the distance
of the 12 kg mass is 4 ms–1. The kinetic energy between the atoms. If the dissociation energy
of the other mass is [2006] of the molecule is
(a) 144 J (b) 288 J
(c) 192 J (d) 96 J D = éëU ( x = ¥) - U at equilibrium ùû , D is [2010]
15. A particle of mass 100g is thrown vertically
upwards with a speed of 5 m/s. The work done b2 b2
(a) (b)
by the force of gravity during the time the particle 2a 12a
goes up is [2006]
(a) –0.5 J (b) –1.25 J b2 b2
(c) (d)
4a 6a
EBD_7764
www.crackjee.xyz
P-26 Physics
21. Statement -1: Two particles moving in the same
direction do not lose all their energy in a
completely inelastic collision. [2010]
(a) aL2 + bL3 (b)
1
2
(aL2 + bL3 )
Statement -2 : Principle of conservation of 1 æ aL2 bL3 ö
aL2 bL3 ç + ÷
momentum holds true for all kinds of collisions. (c) + (d)
2 3 2 çè 2 3 ÷ø
(a) Statement -1 is true, Statement -2 is true ;
Statement -2 is the correct explanation of 25. A particle of mass m moving in the x direction
Statement -1. with speed 2v is hit by another particle of mass
(b) Statement -1 is true, Statement -2 is true; 2m moving in the y direction with speed v. If the
Statement -2 is not the correct explanation collision is perfectly inelastic, the percentage
of Statement -1 loss in the energy during the collision is close
(c) Statement -1 is false, Statement -2 is true. to : [2015]
(d) Statement -1 is true, Statement -2 is false. (a) 56% (b) 62%
(c) 44% (d) 50%
22. At time t = 0 a particle starts moving along the 26. A person trying to lose weight by burning fat
x-axis. If its kinetic energy increases uniformly lifts a mass of 10 kg upto a height of 1 m 1000
with time ‘t’, the net force acting on it must be times. Assume that the potential energy lost each
proportional to [2011 RS] time he lowers the mass is dissipated. How much
(a) constant (b) t fat will he use up considering the work done
1 only when the weight is lifted up? Fat supplies
(c) (d) t
t 3.8 × 107 J of energy per kg which is converted
23. This question has statement I and statement II. to mechanical energy with a 20% efficiency rate.
Of the four choices given after the statements, Take g = 9.8 ms–2 : [2016]
choose the one that best describes the two (a) 44 ´ 104 J (b) 49 ´ 104 J
statements. (c) 45 ´ 104 J (d) 46 ´ 104 J
Statement - I: Apoint particle of mass m moving 27. A point particle of mass m, moves long the
with speed u collides with stationary point uniformly rough track PQR as shown in the
particle of mass M. If the maximum energy loss figure. The coefficient of friction, between the
æ 1 2ö particle and the rough track equals m. The particle
possible is given as f çè mv ÷ø then f = is released, from rest from the point P and it comes
2
to rest at a point R. The energies, lost by the
æ m ö ball, over the parts, PQ and QR, of the track, are
ç ÷. equal to each other, and no energy is lost when
èM + mø
Statement - II: Maximum energy loss occurs particle changes direction from PQ to QR. The
when the particles get stuck together as a result value of the coefficient of friction m and the
of the collision. [2013] distance x (= QR), are, respectively close to :
P [2016]
(a) Statement - I is true, Statment - II is true,
Statement - II is the correct explanation of
Statement - I.
(b) Statement-I is true, Statment - II is true, h = 2m
Statement - II is not the correct explanation
of Statement - II. 30° R
(c) Statement - I is true, Statment - II is false. Q
(d) Statement - I is false, Statment - II is true. Horizontal surface
24. When a rubber-band is stretched by a distance
x, it exerts restoring force of magnitude F = ax + (a) 0.29 and 3.5 m (b) 0.29 and 6.5 m
bx2 where a and b are constants. The work done (c) 0.2 and 6.5 m (d) 0.2 and 3.5 m
in stretching the unstretched rubber-band by L 28. A body of mass m = 10–2 kg is moving in a medium
is: [2014] and experiences a frictional force F = –kv2. Its
www.crackjee.xyz
Work, Energy & Power P-27
intial speed is v0 = 10 ms–1. If, after 10 s, its energy 29. A time dependent force F = 6t acts on a particle
1 2 of mass 1 kg. If the particle starts from rest, the
is mv0 , the value of k will be: [2017] work done by the force during the first 1 secand
8
(a) 10–4 kg m–1 (b) 10–1 kg m–1 s–1 will be [2017]
(c) 10–3 kg m–1 (d) 10–3 kg s–1 (a) 9 J (b) 18 J
(c) 4.5 J (d) 22 J

Answer Key
1 2 3 4 5 6 7 8 9 10 11 12 13 14 15
(c) (d) (b) (b) (c) (b) (b) (b) (a) (b) (b) (b) (d) (b) (b)
16 17 18 19 20 21 22 23 24 25 26 27 28 29
(a) (b) (d) (c) (d) (a) (c) (d) (c) (a) (b) (a) (a) (c)

1. (c) Kinetic energy of a system of particle is zero v t


only when the speed of each particles is zero. 1 2
\ mò vdv = c ò dt \ mv = ct
And if speed of each particle is zero, the 2
0 0
linear momentum of the system of particle
has to be zero. 2c 1 2
\ v= ´t
Also the linear momentum of the system may m
be zero even when the particles are moving.
This is because linear momentum is a vector dx 2c 1 2 dx
\ = ´t where v =
quantity. In this case the kinetic energy of dt m dt
the system of particles will not be zero. x t
\ A does not imply B but B implies A. 2c 1

2. (d) The elastic potential energy


\ ò dx = m
´ ò t 2 dt
0 0
1 3
= ´ Force ´ extension 2c 2t 2 3
2 x= ´ Þ xµt 2
m 3
1
= ´ 200 ´ 0.001 = 0.1 J 5. (c) Given : retardation µ displacement
2 i.e., a = - x
3. (b) k = 5 ´ 103 N/m dv
But a = v
1
(
W = k x22 - x12
2
) dx
v2 x
vdv
1 \ = -x Þ ò v dv = - ò xdx
= ´ 5 ´ 103 é(0.1)2 - (0.05)2 ù dx
v1 0
2 ë û
5000
=
2
´ 0.15 ´ 0.05 = 18.75 Nm (v 2
2 )
- v12 = -
x2
2
4. (b) We know that F × v = Power
1 æ - x2 ö
\ F ´ v = c where c = constant Þ
1
2
(
m v22 - v12 = m ç
2 è 2 ø
) ÷
dv æ mdv ö
\m ´v =c çè\ F = ma = ÷
dt dt ø \ Loss in kinetic energy, \ DK µ x 2
EBD_7764
www.crackjee.xyz
P-28 Physics
6. (b) Mass of over hanging chain Using conservation of energy,
4 æ1 2 ö
m¢ = ´ (0.6)kg m (10 × 100) = m çè v + 10 ´ 20÷ø
2 2
Let at the surface PE = 0
C.M.of hanging part = 0.3 m below the table 1 2
or v = 800 or v = 1600 = 40 m/s
2
4
U i = - m ¢gx = - ´ 0.6 ´ 10 ´ 0.30
2
Loss in potential energy = gain in kinetic
DU = m ' gx = 3.6J = Workdone in putting energy
the entire chain on the table.
1
7. (b) Workdone in displacing the particle, m ´ g ´ 80 = mv 2
rr 2
W = F . x = (5iˆ + 3 ˆj + 2kˆ).(2iˆ - ˆj ) 1 2
= 10 – 3 = 7 joules 10 ´ 80 = v
2
8. (b) Let acceleration of body be a v2 = 1600 or v = 40 m/s
v1 12. (b) u = 0; v = u + aT; v = aT
\ v1 = 0 + at1 Þ a = Instantaneous power = F × v = m. a.
t1
at = m. a2 . t
vt
\ v = at Þ v = 1
t1 v2
\ Instantaneous power = m t
rr r r T2
Pinst = F .v = (ma ).v
2 v
æ mv ö æ v t ö æv ö = (v2 )y
= ç 1÷ ç 1 ÷ = m 1 t 3
è t1 ø è t1 ø çè t ÷ø 13. (d)
1
9. (a) Work done by such force is always zero since
force is acting in a direction perpendicular to
velocity. u1 = v u2 = 0
\ from work-energy theorem = DK = 0 m m
K remains constant. In x-direction : mv + 0 = m (0) + m(v2)x
1 1 æ v ö
10. (b) Mv 2 = k L2 In y-direction : 0 + 0 = m ç ÷ + m(v2 ) y
2 2 è 3ø
k is
Þv= .L
M v
Þ (v2 ) y = and (v2)x = v
M 3
2
æ v ö
\ v2 = ç + v2
Momentum = M × v è 3 ÷ø

k v2 4 2v
=M× .L = kM . L Þ v2 = + v2 = v =
M 3 3 3
11. (b)
In x-direction, mv = mv1 cosq ...(1)
where v1 is the velocity of second mass
100
30 20 mv
In y-direction, 0= - mv1 sin q
mgH 1 3
mv 2 + mgh
2
www.crackjee.xyz
Work, Energy & Power P-29

mv 1 9
or m1v1 sin q = ...(2) \ K.E.(max.) = 2 + =
3 4 4
1 2
v/ 3 K.E.max . = mvmax .
2
v 1 2 9 3
m v Þ ´ 1 ´ vmax . = Þ vmax. =
v1 cosq 2 4 2
q 17. (b) Let the block compress the spring by x before
v1 stopping.
kinetic energy of the block = (P.E of
v1 sinq compressed spring) + work done against
Squaring and adding eqns. (1) and (2) friction.
1 1
v2 2 ´ 2 ´ (4)2 = ´ 10,000 ´ x 2 + 15 ´ x
v12 = v2 + Þ v1 = v 2 2
3 3 10,000 x2 + 30x – 32 = 0
14. (b) Let the velocity and mass of 4 kg piece be v1
and m1 and that of 12 kg piece be v2 and m2. Þ 5000 x 2 + 15 x - 16 = 0
16 kg
-15 ± (15)2 - 4 ´ (5000)(-16)
Initial momentum \ x=
=0
2 ´ 5000
Situation 1 = 0.055m = 5.5cm.
18. (d) The average speed of the athelete
4 kg = m1 m2 = 12 kg Final momentum
v1 v2 = m2v2 – m1v1 100
v= = 10 m/s
Situation 2 10
Applying conservation of linear momentum 1 2
m2v2 = m1v1 \ K.E. = mv
2
12 ´ 4 If mass is 40 kg then,
Þ v1 = = 12 ms -1
4 1
1 1 K.E. = ´ 40 ´ (10) 2 = 2000 J
\ K .E.1 = m1v12 = ´ 4 ´ 144 = 288 J 2
2 2 If mass is 100 kg then,
1 1 1
15. (b) K.E = mv 2 = ´ 0.1 ´ 25 = 1.25 J K.E. = ´ 100 ´ (10) 2 = 5000 J
2 2 2
æ1 ö 19. (c) Initial kinetic energy of the system
W = - mgh = - ç mv 2 ÷ = -1.25 J
è2 ø 1 1
K.Ei = mu 2 + M (0)2
é 1 ù 2 2
2
êëQ mgh = 2 mv by energy conservation úû 1
= ´ 0.5 ´ 2 ´ 2 + 0 = 1J
16. (a) Velocity is maximum when K.E. is maximum 2
For minimum. P.E., For collision, applying conservation of linear
dV momentum
= 0 Þ x 3 - x = 0 Þ x = ±1 m × u = (m + M) × v
dx
2
1 1 1 \ 0.5 ´ 2 = (0.5 + 1) ´ v Þ v = m / s
Þ Min. P.E. = - =- J 3
4 2 4 Final kinetic energy of the system is
K.E.(max.) + P.E.(min.) = 2 (Given)
1
K.E f = (m + M )v 2
2
EBD_7764
www.crackjee.xyz
P-30 Physics
1 2 2 1 Statement II is a case of perfectly inelastic
=(0.5 + 1) ´ ´ = J collision.
2 3 3 3
\ Energy loss during collision By comparing the equation given in
statement I with above equation, we get
æ 1ö
= ç1 - ÷ J = 0.67J æ M ö æ m ö
è 3ø f =ç
è m + M ÷ø instead of çè M + m ÷ø
dU ( x )
20. (d) At equilibrium : =0 Hence statement I is wrong and statement
dx II is correct.
1
-12a -6b 24. (c) Work done in stretching the rubber-band
= æ 2a ö 6
Þ Þ x=ç ÷ by a distance dx is
x11 x5 è b ø dW = F dx = (ax + bx2)dx
a b b2 Integrating both sides,
\ U at equilibrium = 2
- = - and
æ 2a ö æ 2a ö 4a L L
ç ÷ ç ÷
è b ø aL2 bL3
è b ø W = ò axdx + ò bx 2 dx = +
2 3
U ( x= ¥ ) = 0 0 0

æ b2 ö b2 Y
\ D = 0 -ç- ÷ = pf = 3 m V
è 4a ø 4a
21. (a) In completely inelastic collision, all energy m
is not lost (so, statement -1 is true) and the 2v pi 45°
25. (a) X
principle of conservation of momentum
holds good for all kinds of collisions (so,
statement -2 is true) . Statement -2 explains v
statement -1 correctly because applying the 2m
principle of conservation of momentum, we
can get the common velocity and hence Initial momentum of the system
the kinetic energy of the combined body. pi = [m(2V)2 ´ m(2V)2 ]
22. (c) K.E. µ t or K. E. = ct
1 2 = 2m ´ 2V
Þ mv = ct Final momentum of the system = 3mV
2
By the law of conservation of momentum
p2
Þ = ct (Q p = mv) 2 2v
2m 2 2mv = 3mV Þ = Vcombined
3
Þ p = 2ctm Loss in energy
1 1 1 1
Þ F= 2 cm ´ DE = m1V12 + m2V22 - (m1 + m2 )Vcombined
2
2 t 2 2 2
1 4 5
Þ F µ DE = 3mv2 - mv2 = mv2 = 55.55%
t 3 3
P2 P2 Percentage loss in energy during the collision
23. (d) Maximum energy loss = - ; 56%
2m 2(m + M)
W mgh ´ 1000 10 ´ 9.8 ´ 1 ´ 1000
é P2 1 2 ù 26. (b) n = = =
êQ K.E. = = mv ú input input input
ëê 2m 2 ûú 98000
Input = = 49 × 104J
2
P é M ù 1 ì M ü 0.2
= ê ú = mv 2 í ý
2m ë (m + M) û 2 îm + M þ
www.crackjee.xyz
Work, Energy & Power P-31

49 ´ 10 4 5
dV
10
Fat used =
3.8 ´ 10 7 = 12.89 × 10–3kg. ò 2
= -100 K ò dt
10 V 0
27. (a) Loss in P.E. = Work done against friction
from p ® Q 1 1
+ work done against friction from Q ® R - = 100K (10) or, K = 10–4 kgm–1
5 10
mgh = m(mgcosq) PQ + mmg (QR)
h = m cos q × PQ + m(QR) dV
29. (c) Using, F = ma = m
2 = m × × + mx dt
2 = m + mx ..... (i) [sin 30° = ] dV
Also work done P ® Q = work done Q ® R 6t = 1. [Q m = 1 kg given]
dt
\ m = mx
v 1
28. (a) Let Vf is the final speed of the body. ét 2 ù
ò dV = ò 6t dt V = 6 ê ú = 3 ms
2
–1

From questions, 0 ë û0
1 1 V0 [Q t = 1 sec given]
mV f2 = mV02 Þ Vf = = 5 m/ s From work-energy theorem,
2 8 2

æ dV ö 2 dV W = DKE =
1
2
( ) 1
m V 2 - u 2 = ´ 1 ´ 9 = 4.5 J
2
F = mç ÷ = -kV \ (10–2) = –kV2
è dt ø dt
EBD_7764
www.crackjee.xyz
P-32 Physics

System of Particles
and Rotational Motion 6
1. Initial angular velocity of a circular disc of mass 6. A particle performing uniform circular motion has
M is w1. Then two small spheres of mass m are angular frequency is doubled & its kinetic energy
attached gently to diametrically opposite points halved, then the new angular momentum is [2003]
on the edge of the disc. What is the final angular
velocity of the disc? [2002] (a) L (b) 2 L
4
æ M + mö æ M + mö L
(a) çè ÷w (b) çè ÷w (c) 4 L (d)
M ø 1 m ø 1 r 2
æ M ö æ M ö 7. Let F be the force acting on a particle having
÷w r r
(c) çè
M + 4m ø 1
(d) çè ÷ w1.
M + 2mø position vector r , and T be the torque of this
force about the origin.
r rThen [2003]
2. Two identical particles move towards each other r r
(a) r .T = 0 and F .T ¹ 0
with velocity 2v and v respectively. The velocity r r r r
of centre of mass is [2002] (b) r .T ¹ 0 and F .T = 0
r r r r
(a) v (b) v/3 (c) r .Tr ¹ 0 and Fr .Tr ¹ 0
(c) v/2 (d) zero r
(d) r .T = 0 and F .T = 0
3. Moment of inertia of a circular wire of mass M 8. A solid sphere is rotating in free space. If the
and radius R about its diameter is [2002] radius of the sphere is increased keeping mass
(a) MR2/2 (b) MR2 same, which one of the following will not be
(c) 2MR2 (d) MR2/4. affected ? [2004]
4. A particle of mass m moves along line PC with (a) Angular velocity
velocity v as shown. What is the angular (b) Angular momentum
momentum of the particle about P? [2002] (c) Moment of inertia
(d) Rotational kinetic energy
C 9. One solid sphere A and another hollow sphere B
L are of same mass and same outer radii. Their
moment of inertia about their diameters are
P respectively IA and IB such that [2004]
l (a) I A < IB (b) I A > IB
O
(a) mvL (b) mvl IA dA
(c) mvr (d) zero. (c) IA = IB (d) =
I B dB
5. A circular disc X of radius R is made from an iron
plate of thickness t, and another disc Y of radius where dA and dB are their densities.
10. A body A of mass M while falling vertically
t
4R is made from an iron plate of thickness . downwards under gravity breaks into two parts;
4 1 2
Then the relation between the moment of inertia a body B of mass M and a body C of mass
I X and IY is [2003] 3 3
M. The centre of mass of bodies B and C taken
(a) ΙY = 32 Ι X (b) ΙY = 16 Ι X together shifts compared to that of body A
towards [2005]
(c) ΙY = Ι X (d) ΙY = 64 Ι X
www.crackjee.xyz
System of Particles and Rotational Motion P-33
(a) does not shift
(b) depends on height of breaking 15. A force of – Fkˆ acts on O, the origin of the
(c) body B coordinate system. The torque about the point
(d) body C (1, –1) is [2006]
Z
11. The moment of inertia of a uniform semicircular
disc of mass M and radius r about a line
perpendicular to the plane of the disc through
the centre is [2005]
2 1 O
(a) Mr 2 (b) Mr Y
5 4
1
(c) Mr 2 (d) Mr 2 X
2
12. A ‘T’ shaped object with dimensions shown in (a) F (iˆ - ˆj ) (b) - F (iˆ + ˆj )
the figure, is lying on a smooth floor. A force
uur (c) F (iˆ + ˆj ) (d) - F (iˆ - ˆj )
‘ F ’ is applied at the point P parallel to AB, such 16. A thin circular ring of mass m and radius R is
that the object has only the translational motion rotating about its axis with a constant angular
without rotation. Find the location of P with velocity w. Two objects each of mass M are
respect to C. [2005] attached gently to the opposite ends of a
l diameter of the ring. The ring now rotates with
A B
an angular velocity w' = [2006]
w (m + 2 M ) w(m - 2 M )
P (a) (b)
2l m (m + 2 M )
F
wm wm
(c) (d)
(m + M ) (m + 2M )
C 17. A circular disc of radius R is removed from a
3 2 bigger circular disc of radius 2R such that the
(a) l (b)l
2 3 circumferences of the discs coincide. The centre
4 of mass of the new disc is a / R form the centre
(c) l (d) l of the bigger disc. The value of a is [2007]
3
13. Consider a two particle system with particles (a) 1/4 (b) 1/3
having masses m1 and m2. If the first particle is (c) 1/2 (d) 1/6
pushed towards the centre of mass through a 18. A round uniform body of radius R, mass M and
distance d, by what distance should the second moment of inertia I rolls down (without slipping)
particle is moved, so as to keep the centre of an inclined plane making an angle q with the
mass at the same position? [2006] horizontal. Then its acceleration is [2007]
m2 m1
(a) m d (b) m + m d g sin q g sin q
1 1 2 (a) 2
(b)
m1 1 - MR / I 1 + I / MR 2
(c) m d (d) d g sin q g sin q
2 (c) (d)
2
14. Four point masses, each of value m, are placed 1 + MR / I 1 - I / MR 2
at the corners of a square ABCD of side l. The 19. Angular momentum of the particle rotating with
moment of inertia of this system about an axis a central force is constant due to [2007]
passing through A and parallel to BD is [2006] (a) constant torque
(a) 2ml 2 (b) 3ml2 (b) constant force
(c) constant linear momentum
(c) 3ml 2 (d) ml 2 (d) zero torque
EBD_7764
www.crackjee.xyz
P-34 Physics
20. For the given uniform square lamina ABCD, 23. A thin uniform rod of length l and mass m is swinging
whose centre is O, [2007] freely about a horizontal axis passing through its
D F C end. Its maximum angular speed is w. Its centre of
mass rises to a maximum height of [2009]
1 lw 1 l 2w 2
O (a) (b)
6 g 2 g

B 1 l 2w 2 1 l 2w2
A E (c) (d)
6 g 3 g
(a) I AC = 2 I EF (b) 2 I AC = I EF 24. A mass m hangs with the help of a string
(c) I AD = 3I EF (d) I AC = I EF wrapped around a pulley on a frictionless
21. A thin rod of length ‘L’ is lying along the x-axis bearing. The pulley has mass m and radius R.
with its ends at x = 0 and x = L. Its linear density Assuming pulley to be a perfect uniform circular
n disc, the acceleration of the mass m, if the string
æ xö
(mass/length) varies with x as k ç ÷ , where n does not slip on the pulley, is: [2011]
è Lø
can be zero or any positive number. If the 2
(a) g (b) g
position xCM of the centre of mass of the rod is 3
plotted against ‘n’, which of the following graphs g 3
best approximates the dependence of xCM on n? (c) (d) g
3 2
[2008]
xCM 25. A thin horizontal circular disc is rotating about a
vertical axis passing through its centre. An insect
L is at rest at a point near the rim of the disc. The
L insect now moves along a diameter of the disc
(a)
2 to reach its other end. During the journey of the
n
O insect, the angular speed of the disc. [2011]
xCM
(a) continuously decreases
(b) continuously increases
L (c) first increases and then decreases
(b) 2
n (d) remains unchanged
O 26. A pulley of radius 2 m is rotated about its axis by
xCM
a force F = (20t – 5t2 ) newton (where t is
L measured in seconds) applied tangentially. If the
L moment of inertia of the pulley about its axis of
(c) 2
n rotation is 10 kg-m2 the number of rotations made
O by the pulley before its direction of motion is
xCM
reversed, is: [2011]
L
L (a) more than 3 but less than 6
(d) 2 (b) more than 6 but less than 9
n
O (c) more than 9 (d) less than 3
22. Consider a uniform square plate of side ‘a’ and 27. A loop of radius r and mass m rotating with an
mass ‘M’. The moment of inertia of this plate
about an axis perpendicular to its plane and angular velocity w0 is placed on a rough horizontal
passing through one of its corners is [2008] surface. The initial velocity of the centre of the
loop is zero.What will be the velocity of the centre
5 1
(a) Ma 2 (b) Ma 2 of the loop when it ceases to slip ? [2013]
6 12
rw0 rw0
7 2 (a) (b)
(c) Ma 2 (d) Ma 2 4 3
12 3
rw0
(c) (d) rw0
2
www.crackjee.xyz
System of Particles and Rotational Motion P-35
28. A bob of mass m attached to an inextensible moving from B to C.
string of length l is suspended from a vertical ur mv
support. The bob rotates in a horizontal circle with (b) L = Rk$ when the particle is moving
an angular speedw rad/s about the vertical. About 2
from D to A.
the point of suspension: [2014]
ur mv $
(a) angular momentum is conserved.
(c) L = – Rk when the particle is moving
(b) angular momentum changes in magnitude 2
but not in direction. from A to B.
(c) angular momentum changes in direction but ur éR ù
not in magnitude. (d) L = mv êê - a ú k$ when the particle is
ë 2 úû
(d) angular momentum changes both in
moving from C to D.
direction and magnitude. 32. A roller is made by joining together two cones
29. Distance of the centre of mass of a solid uniform at their vertices O. It is kept on two rails AB and
cone from its vertex is z0. If the radius of its base is CD, which are placed asymmetrically (see
R and its height is h then z0 is equal to : [2015] figure), with its axis perpendicular to CD and its
5h 3h 2 centre O at the centre of line joining AB and Cd
(a) (b) (see figure). It is given a light push so that it
8 8R starts rolling with its centre O moving parallel to
h2 3h CD in the direction shown. As it moves, the roller
(c) (d) will tend to : [2016]
4R 4
30. From a solid sphere of mass M and radius R a B D
cube of maximum possible volume is cut. Moment
of inertia of cube about an axis passing through
its center and perpendicular to one of its faces is:
4MR 2 4MR 2
(a) (b) [2015] O
9 3p 3 3p
MR 2 MR 2
(c) (d)
32 2p 16 2p C
31. A particle of mass m is moving along the side of A
a square of side 'a', with a uniform speed v in the (a) go straight.
x-y plane as shown in the figure : [2016] (b) turn left and right alternately.
(c) turn left.
y (d) turn right.
33. The moment of inertia of a uniform cylinder of
D a C length l and radius R about its perpendicular
V bisector is I. What is the ratio l /R such that the
a V V a moment of inertia is minimum ? [2017]
A V
a B 3
V (a) 1 (b)
2
45° R
a
O 3 3
(c) (d)
Which of the following statements is false for 2 2
ur
the angular momentum L about the origin? 34. A slender uniform rod of mass M and length l
is pivoted at one end so that it can rotate in a
ur éR ù
(a) L = mv ê + a ú k$ when the particle is vertical plane (see figure). There is negligible
ê
ë 2 úû friction at the pivot. The free end is held
EBD_7764
www.crackjee.xyz
P-36 Physics
vertically above the pivot and then released. The 3g 2g
angular acceleration of the rod when it makes an (a) cos q (b) cos q
2l 3l
angle q with the vertical is [2017]
3g 2g
(c) sin q (d) sin q
2l 2l

Answer Key
1 2 3 4 5 6 7 8 9 10 11 12 13 14 15
(c) (c) (a) (d) (d) (a) (d) (b) (a) (a) (c) (d) (c) (c) (c)
16 17 18 19 20 21 22 23 24 25 26 27 28 29 30
(d) (b) (b) (d) (d) (a) (d) (c) (b) (c) (a) (c) (c) (d) (a)
31 32 33 34
(a) (c) (c) (c)

1. (c) When two small spheres of mass m are 3. (a) M. I of a circular wire about an axis nn' passing
attached gently, the external torque, about through the centre of the circle and
the axis of rotation, is zero and therefore perpendicular to the plane of the circle = MR2
the angular momentum about the axis of
Z
rotation is constant. n Y
I
\ I1w1 = I 2 w 2 Þ w 2 = 1 w1
I2
1 2
Here I1 = MR X
2
1 2 2
n'
and I 2 = MR + 2mR As shown in the figure, X-axis and Y-axis
2
lie in the plane of the ring . Then by
1
MR 2 perpendicular axis theorem
2 M
\ w2 = 1 ´ w1 = w1 IX + IY = IZ
M + 4m Þ 2 IX = MR2 [Q IX = IY (by symmetry)
MR 2 + 2mR 2
2 and IZ = MR2]
2. (c) The velocity of centre of mass of two
particle system is given by 1
\ IX = MR 2
m v + m2 v2 2
vc = 1 1 4. (d) Angular momentum (L)
m1 + m2 = (linear momentum) × (perpendicular
m m distance of the line of action of
v momentum from the axis of rotation)
2v = mv × r [Here r = 0 because the line of
m(2v ) + m(-v ) v = mv × 0 action of momentum passes
= =
m+m 2 =0 through the axis of rotation]
www.crackjee.xyz
System of Particles and Rotational Motion P-37

mass( M ) \ I A < IB
5. (d) We know that density (d ) = 10. (a) Does not shift as no external force acts.
volume(V )
The centre of mass of the system continues
\ M = d ´ V = d ´ ( pR 2 ´ t ) . its original path. It is only the internal forces
The moment of inertia of a disc is given by which comes into play while breaking.
1 11. (c) The disc may be assumed as combination
I = MR 2 of two semi circular parts.
2
Let I be the moment of inertia of the uniform
1 pd
\ I = ( d ´ pR 2 ´ t ) R 2 = t ´ R4 semicircular disc
2 2
2Mr 2 Mr 2
I t R4 t ´ R4 1 Þ 2I = ÞI=
\ X = X X = = 2 2
IY tY RY4 t 64
´ (4 R )4 l
4
12. (d) A B
1 2 L (0, 2 l)
6. (a) K .E . Iw , but L = Iw Þ I =
2 w
1L 1
\ K .E. = ´ w 2 = Lw
2w 2 F P (0, l) 2l
K .E L´w K .E L´w
\ = Þ = y
K .E ' L ' ´ w ' K .E L '´ 2w
2
L (0, 0)
\ L' = C
4 uur
r r r
7. (d) We know that t = r ´ F To have linear motion, the force F has to
be applied at centre of mass.
i.e. the point ‘P’has to be at the centre of
F

r
mass
t
m1 y1 + m2 y2 m ´ 2 l + 2m ´ l 4l
y= = =
m1 + m2 3m 3

13. (c) Initially,

m1 ( - x1 ) + m2 x2
r r 0= Þ m1 x1 = m2 x2 ...(1)
The angle between t and r is 90° and the m1 + m2
r r Finally,
angle between t and F is also 90°. We The centre of mass is at the origin
also know that the dot product of two
vectors which have an angle of 90° between x1– d x2– d ¢
them is zero. Therefore (d) is the correct d d¢
option. m1 O m2
8. (b) Angular momentum will remain the same
since external torque is zero. m1 (d - x1 ) + m2 ( x2 - d ')
9. (a) The moment of inertia of solid sphere A \0 =
m1 + m2
2 2
about its diameter I A = MR . Þ 0 = m1d - m1 x1 + m2 x2 - m2 d '
5
The moment of inertia of a hollow sphere B m1
Þd'= d [From (1).]
2 2 m2
about its diameter I B = MR .
3
EBD_7764
www.crackjee.xyz
P-38 Physics
14. (c)
n
D l C
xc.m =
( 4psR ) ´ 0 + ( -psR ) R
2 2

2
O 4psR 2 - psR 2
l/
A B -psR 2 ´ R
\ xc.m =
3psR2
n' R 1
\ xc.m = - Þa=
Inn' = M.I due to the point mass at B + 3 3
M.I due to the point mass at D + 18. (b) This is a standard formula and should be
M.I due to the point mass at C. memorized.
2 g sin q
æ l ö a=
I nn ' = 2 ´ m ç + m( 2l) 2 I
è 2 ÷ø 1+
MR 2
= ml 2 + 2ml 2 = 3ml 2 uur
ur ur uur uur d Lc
15. (c) Torque t = r ´ F = (iˆ - ˆj ) ´ ( - Fkˆ) 19. (d) We know that t c =
dt
uur
= F [- iˆ ´ kˆ + ˆj ´ kˆ] where tc torque about the center of mass
= F ( ˆj + iˆ) = F ( iˆ + ˆj) uur
of the body and Lc = Angular momentum
éSince kˆ ´ iˆ = ˆj and ˆj ´ kˆ = iˆ ù about the center of mass of the body.
ë û
16. (d) Applyin g conservation of an gular Central forces act along the center of mass.
Therefore torque about center of mass is
momentum I'w' = Iw
zero.
(mR 2 + 2MR 2 )w ' = mR 2 w uur uur
When t c = 0 then Lc = constt.
é m ù 20. (d) By the theorem of perpendicular axes,
Þ w' = wê
ë m + 2 M úû Iz = Ix + Iy or, Iz = 2 Iy
17. (b) Let the mass per unit area be s.
Then the mass of the complete disc (Q Ix = Iy by symmetry of the figure)
Z
= s [p(2 R) 2 ] = 4psR 2 Y
D F C
2R

X
O R O

A E B
The mass of the r emoved disc
= s (pR 2 ) = psR 2 Iz
\ I EF = ...(i)
Let us consider the above situation to be a 2
complete disc of radius 2R on which a disc Again, bythe same theorem Iz = IAC + IBD = 2 IAC
of radius R of negative mass is (\ IAC = IBD by symmetry of the figure)
superimposed. Let O be the origin. Then
Iz
the above figure can be redrawn keeping \ I AC = ...(ii)
in mind the concept of centre of mass as : 2
2 R From (i) and (ii), we get, IEF = IAC.
4ps R 21. (a) · When n = 0, x = k where k is a constant.
O –ps R
2 This means that the linear mass density is
constant. In this case the centre of mass
www.crackjee.xyz
System of Particles and Rotational Motion P-39
will be at the midelle of the rod ie at L/2.
Therefore (c) is ruled out DB 2a a
Also, DO = = =
· n is positive and as its value increases, 2 2 2
the rate of increase of linear mass density According to parallel axis theorem
with increase in x increases. This shows that 2
the centre of mass will shift towards that æ a ö Ma 2 Ma 2
Imm ' = I nn ' + M ç ÷ = +
end of the rod where n = L as the value of n è 2ø 6 2
increases. Therefore graph (b) is ruled out.
n Ma 2 + 3Ma 2 2
æ xö = = Ma 2
· The linear mass density l = k çè ÷ø 6 3
L
x 23. (c) O
Here £1
L
With increase in the value of n, the centre
of mass shift towards the end x = L such
that first the shifting is at a higher rate with C. M
increase in the value of n and then the rate h
decreases with the value of n. C. M Reference
These characteristics are represented by level for P.E.
graph (a).
L L L n
æ xö
ò xdm ò x (l dx) ò k çè L÷ø .xdx The moment of inertia of the rod about O is
xCM = 0
= 0
= 0 1 2
L L L n ml . The maximum angular speed of the
æxö 3
ò dm ò l dx ò k çè L ÷ø dx rod is when the rod is instantaneously
vertical. The energy of the rod in this
0 0 0
L 1
é x n+ 2 ù condition is I w 2 where I is the moment of
kê 2
nú inertia of the rod about O. When the rod is in
êë (n + 2) L úû 0 L (n + 1) its extreme portion, its angular velocity is zero
= =
L n+2 momentarily. In this case, the energy of the
é k x n +1 ù rod is mgh where h is the maximum height to
ê nú which the centre of mass (C.M) rises
êë (n + 1) L úû 0
1 2 1 æ 1 2ö 2
For n = 0 , xCM =
L
; n = 1, \ mgh = I w = 2 çè 3 ml ÷ø w
2
2
l 2 w2
2L 3L Þ h=
xCM = ; n = 2, xCM = ;.... 6g
3 4 24. (b) For translational motion,
1 Ma 2 mg – T = ma .....(1)
22. (d) Inn' = M (a 2 + a 2 ) = For rotational motion,
12 6
n m a
T.R = I a = I ....(2)
R
A m R
D
T

m
O
mg
Solving (1) & (2),

B
C

n m 1
EBD_7764
w w w . c r a c k j e
P-40 Physics
mg mg 2 mg 2 g 1 3 3
a= = 2
= = + ´
æ I ö mR 3m 3
çè m + ÷ m+ 2 5 2
R2 ø 2 =
2R 3 3 1
25. (c) As insect moves along a diameter, the - ´
2 5 2
effective mass and hence the M.I. first
decreases then increases so from principle 1æ 3ö 1 8
of conservation of angular momentum, ç1 + ÷ ´
2è 5ø
angular speed, first increases then = = 2 5
decreases. 3æ 1ö 3´4
26. (a) F = 20t – 5t2 ç1 - ÷
5 è 5ø 10
FR
\a = = 4t - t 2 8
I
= 10 = 8 = 2
dw
Þ = 4t - t 2 3´4 3´4 3
dt
10
w t
Þ ò d w = ò 4t - t dt
2
( ) ycm =
ò ydm
0 0 29. (d)
t3
ò dm
Þ w = 2t 2 - (as w = 0 at t = 0, 6s) y a
3
h r h
q 6 æ 2 t3 ö 2
ò dq = ò ç 2t - ÷ dt

ò pr dyr ´ y 3h
è R
0 0 =0 =
Þ q = 36 rad 1 2 4
pR hr
36 3
Þ n= <6
2
2p
30. (a) Here a = R
r 3
27. (c) o o 4 3
pR
M
Now, = 3 3
From conservation of angular momentum M¢ a
about any fix point on the surface,
4 3
mr 2 w 0 = 2mr 2 w pR
3
= 3 3
= p. a
w0 r æ 2 ö 2
Þ w = w0 / 2 Þ v =
2
[Q v = rw ] çè R ÷ø
3
28. (c) Torque working on the bob of mass m is,
t = mg ´ l sin q . (Direction parallel to 2M
M¢ =
plane of rotation of particle) 3p
Moment of inertia of the cube about the
M¢ a 2
q given axis, I =
l 6
l
2
2M æ 2 ö
m ´ç R÷
mg 3p è 3 ø 4MR 2
ur = =
As t is perpendicular to L , direction of L 6 9 3p
changes but magnitude remains same.
www.crackjee.xyz
System of Particles and Rotational Motion P-41

31. (a) We know that L = mvr^ m é 2 l2 ù


I= êR + ú
y 4ë 3û

D V C
a a
V
a a V
l
a
A V B
R/ 2 m é V l2 ù dl m é -v 2l ù
= ê + ú Þ = + =0
4 ë pl 3 û dl 4 êë pl 2 3 úû
O a X
R v 2l 2 pl 3
= Þv=
2 pl 2 3 3
In none of the cases, the perpendicular 2pl 3 l2 3 l 3
pR 2 l = Þ 2 = or, =
æ R ö 3 R 2 R 2
distance r^ is çè - a÷
2 ø 34. (c) Torque at angle q
32. (c) As shown in the diagram, the normal
reaction of AB on roller will shift towards l
t = Mg sin q.
O. 2
This will lead to tending of the system of
cones to turn left. w,a
B D
Q
Q

O
Also t = la
l
\ l a = Mg sin q
2
A C
Ml 2 l é Ml 2 ù
33. (c) As we know, moment of inertia of a solid .a = Mg sin q êQ I rod = ú
3 2 ë 3 û
cylinder about an axis which is
perpendicular bisector la sin q 3 g sin q
Þ =g \ a=
mR 2 ml 2 3 2 2l
I= +
4 12
EBD_7764
www.crackjee.xyz
P-42 Physics

Gravitation 7
1. The kinetic energy needed to project a body of 7. The escape velocity for a body projected vertically
mass m from the earth surface (radius R) to infinity upwards from the surface of earth is 11 km/s. If
is [2002] the body is projected at an angle of 45°with the
(a) mgR/2 (b) 2mgR vertical, the escape velocity will be [2003]
(c) mgR (d) mgR/4. (a) 11 2 km / s (b) 22 km/s
2. If suddenly the gravitational force of attraction
11
between Earth and a satellite revolving around (c) 11 km/s (d) km / s
it becomes zero, then the satellite will [2002] 2
(a) continue to move in its orbit with same 8. A satellite of mass m revolves around the earth of
velocity radius R at a height x from its surface. If g is the
acceleration due to gravity on the surface of the
(b) move tangentially to the original orbit in earth, the orbital speed of the satellite is [2004]
the same velocity
gR 2 gR
(c) become stationary in its orbit (a) (b)
(d) move towards the earth R+ x R-x
3. Energy required to move a body of mass m from æ gR 2 ö 1/ 2
an orbit of radius 2R to 3R is [2002] (c) gx (d) ç ÷
2 2 è R + xø
(a) GMm/12R (b) GMm/3R
(c) GMm/8R (d) GMm/6R. 9. The time period of an earth satellite in circular
orbit is independent of [2004]
4. The escape velocity of a body depends upon (a) both the mass and radius of the orbit
mass as [2002] (b) radius of its orbit
(a) m0 (b) m1 (c) the mass of the satellite
(c) m2 (d) m3 (d) neither the mass of the satellite nor the
5. The time period of a satellite of earth is 5 hours. radius of its orbit.
If the separation between the earth and the 10. If ‘g’ is the acceleration due to gravity on the
satellite is increased to 4 times the previous earth’s surface, the gain in the potential energy
value, the new time period will become [2003] of an object of mass ‘m’ raised from the surface
(a) 10 hours (b) 80 hours of the earth to a height equal to the radius ‘R' of
(c) 40 hours (d) 20 hours the earth is [2004]
6. Two spherical bodies of mass M and 5M & radii R 1 1
(a) mgR (b) mgR
& 2R respectively are released in free space with 4 2
initial separation between their centres equal to (c) 2 mgR (d) mgR
12 R. If they attract each other due to gravitational 11. Suppose the gravitational force varies inversely
force only, then the distance covered by the as the nth power of distance. Then the time period
smaller body just before collision is [2003] of a planet in circular orbit of radius ‘R’ around the
(a) 2.5 R (b) 4.5 R sun will be proportional to [2004]
æ n -1ö
(c) 7.5 R (d) 1.5 R
(a) Rn (b) èç 2 ø÷
R
æ n +1ö æ n- 2ö
çè ÷
(c) R 2 ø
(d) èç 2 ø÷
R
www.crackjee.xyz
Gravitation P-43
12. The change in the value of ‘g’ at a height ‘h’ above 17. The height at which the acceleration due to
the surface of the earth is the same as at a depth ‘d’ g
below the surface of earth. When both ‘d’ and ‘h’ gravity becomes (where g = the acceleration
9
are much smaller than the radius of earth, then due to gravity on the surface of the earth) in
which one of the following is correct? [2005] terms of R, the radius of the earth, is [2009]
3h h R
(a) d = (b) d = (a) (b) R / 2
2 2 2
(c) d = h (d) d =2 h
13. A particle of mass 10 g is kept on the surface of (c) 2R (d) 2 R
a uniform sphere of mass 100 kg and radius 10 18. Two bodies of masses m and 4 m are placed at a
cm. Find the work to be done against the distance r. The gravitational potential at a point
gravitational force between them to take the on the line joining them where the gravitational
particle far away from the sphere [2005] field is zero is: [2011]
4Gm 6Gm
(you may take G = 6.67× 10 -11 Nm 2 / kg 2 ) (a) - (b) -
r r
(a) 3.33 × 10 -10 J (b) 13.34 × 10 -10 J 9Gm
(c) - (d) zero
r
(c) 6.67 × 10 -10 J (d) 6.67 × 10 -9 J
19. Two particles of equal mass ‘m’ go around a circle
14. Average density of the earth [2005]
of radius R under the action of their mutual
(a) is a complex function of g
gravitational attraction. The speed of each particle
(b) does not depend on g
with respect to their centre of mass is [2011 RS]
(c) is inversely proportional to g
(d) is directly proportional to g Gm Gm
(a) (b)
15. A planet in a distant solar system is 10 times 4R 3R
more massive than the earth and its radius is 10 Gm Gm
times smaller. Given that the escape velocity from (c) (d)
2R R
the earth is 11 km s–1, the escape velocity from 20. The mass of a spaceship is 1000 kg. It is to be
the surface of the planet would be [2008] launched from the earth's surface out into free
(a) 1.1 km s–1 (b) 11 km s–1 space. The value of g and R (radius of earth) are
(c) 110 km s–1 (d) 0.11 km s–1 10 m/s2 and 6400 km respectively. The required
16. This question contains Statement-1 and energy for this work will be [2012]
Statement-2. Of the four choices given after the (a) 6.4 × 1011Joules (b) 6.4 × 108 Joules
statements, choose the one that best describes (c) 6.4 × 109 Joules (d) 6.4 × 1010 Joules
the two statements. [2008] 21. What is the minimum energy required to launch
Statement-1 : For a mass M kept at the centre of a satellite of mass m from the surface of a planet
a cube of side ‘a’, the flux of gravitational field of mass M and radius R in a circular orbit at an
passing through its sides 4 p GM. and altitude of 2R? [2013]
Statement-2: If the direction of a field due to a
point source is radial and its dependence on the 5GmM 2GmM
(a) (b)
1 6R 3R
distance ‘r’ from the source is given as 2 , its
r GmM GmM
flux through a closed surface depends only on (c) (d)
the strength of the source enclosed by the 2R 2R
surface and not on the size or shape of the 22. Four particles, each of mass M and equidistant
surface. from each other, move along a circle of radius R
(a) Statement -1 is false, Statement-2 is true under the action of their mutual gravitational
(b) Statement -1 is true, Statement-2 is true; attraction. The speed of each particle is:[2014]
Statement -2 is a correct explanation for
GM GM
Statement-1 (a) (b) 2 2
(c) Statement -1 is true, Statement-2 is true; R R
Statement -2 is not a correct explanation
for Statement-1
(d) Statement -1 is true, Statement-2 is false
(c)
GM
R
( )
1 + 2 2 (d)
1 GM
2 R
(
1+ 2 2 )
EBD_7764
www.crackjee.xyz
P-44 Physics
23. From a solid sphere of mass M and radius R, a
spherical portion of radius R/2 is removed, as (a) gR / 2 (b) gR ( 2 -1 )
shown in the figure. Taking gravitational
potential V = 0 at r = ¥, the potential at the centre (c) 2gR (d) gR
of the cavity thus formed is : [2015] 25. The variation of acceleration due to gravity g
(G = gravitational constant) with distance d from centre of the earth is best
represented by (R = Earth's radius): [2017]
g

(a) (b)
d
-2GM -2GM O R
(a) (b)
3R R g
-GM -GM
(c) (d)
2R R (c) (d)
24. A satellite is revolving in a circular orbit at a d
height 'h' from the earth's surface (radius of earth
O R
R; h < < R). The minimum increase in its orbital
velocity required, so that the satellite could
escape from the earth's gravitational field, is close
to : (Neglect the effect of atmosphere.) [2016]
Answer Key
1 2 3 4 5 6 7 8 9 10 11 12 13 14 15
(c) (c) (d) (a) (c) (c) (c) (d) (c) (b) (c) (d) (c) (d) (c)
16 17 18 19 20 21 22 23 24 25
(b) (d) (c) (a) (d) (a) (d) (d) (b) (b)

1 -2GMm + 3GMm GMm


1. (c) K. E = m ve2 where ve = escape velocity = =
2 6R 6R
= 2gR 2GM
4. (a) Escape velocity, ve = 2 gR =
R
1
\ K.E = m ´ 2 gR = mgR Þ ve µ m0
2
2. (c) Due to inertia of motion it will move Where M, R are the mass and radius of the
tangentially to the original orbit in the same planet respectively. In this expression the
mass of the body (m) is not present
velocity. showing that the escape velocity is
3. (d) Energy required = (Potential energy of the independent of the mass.
Earth -mass system when mass is at 5. (c) According to Kepler’s law of planetary
distance 3R) – (Potential energy of the Earth motion T2 µ R3
-mass system when mass is at distance 2R) 3 3
æR ö 2
é 4R ù 2
-GMm æ -GMm ö -GMm GMm \ T2 = T1 ç 2 ÷ = 5´ ê ú
-ç + èR ø ëRû
= è 2R ÷ø = 3R
1
3R 2R 3
= 5 ´ 2 = 40 hours
www.crackjee.xyz
Gravitation P-45
6. (c) The gravitational force acting on both the
masses is the same. We know that 7. (c) ve = 2 gR
Force = mass × acceleration. The escape velocity is independent of the
9R angle at which the body is projected.
8. (d) Gravitational force provides the necessary
xM x5M centripetal force.
R 2R
mv 2 GmM GM
12R \ = also g = 2
( R + x) ( R + x) 2
R
1
For same force, acceleration µ
mass mv 2 æ GM ö R 2 n!
\ = mç
\
a5 M
=
M
=
1 ( R + x) è R 2 ÷ø ( R + x )2 r !( n - r ) !
aM 5M 5 .....(i)
Let t be the time taken for the two masses mv 2 R2
to collide and x5M, xM be the distance \ = mg
( R + x) ( R + x) 2
travelled by the mass 5M and M
respectively. 1/ 2
For mass 5M gR 2 æ gR 2 ö
2
u = 0, S = x5M , t = t , a = a5M
\v = Þ v=ç ÷
R+ x è R + xø
1
S = ut + at 2 mv 2 GmM
2 9. (c) We have, =
1 R + x ( R + x )2
\ x5 M = a5 M t 2 ....(ii)
2 x = height of satellite from earth surface
For mass M m = mass of satellite
u = 0, s = xM, t = t, a = aM
GM GM
1 2 Þ v2 = or v =
\ s = ut + at ( R + x) R+x
2
1 2 2p ( R + x ) 2 p ( R + x )
Þ xM = aM t … (iii) T= =
2 v GM
Dividing (ii) by (iii) R+x
1
x5 M a5 M t 2 which is independent of mass of satellite
= 2 a 1
= 5M = GmM
xM 1
a t 2 a M 5 [From (i)] 10. (b) At earth surface, P.E. of system is –
2 M R
At a distance R from the earth's surface,
\ 5 x5M = xM ....(iv)
From the figure it is clear that GmM
P.E .of system is -
x5M+ xM= 9R ....(v) 2R
Where O is the point where the two
spheres collide. -GmM GmM GmM
\ DU = + ; DU =
From (iv) and (v) 2R R 2R
xM GM GM
+ xM = 9 R Now = g; \ = gR
5 2 R
R
\ 6 xM = 45R
1
45 \ DU = mgR
\ xM = R = 7.5 R 2
6
EBD_7764
www.crackjee.xyz
P-46 Physics
where, M = mass enclosed in the closed
11. (c) F = KR–n = MRw2 Þ w 2 = KR - ( n +1) surface
- ( n +1) r 1
or w = KR 2 This relationship is valid when | E g | µ 2 .
r
- (n +1)
2p g' R2
µR 2 17. (d) We know that =
T g ( R + h)2
+ ( n +1) 2
g /9 é R ù
\T µ \ =ê
ë R + h úû
R 2 g
12. (d) Variation of g with altitude is, R 1
\ =
é 2h ù R+h 3
gh = g ê1 - ú ;
ë Rû \ h = 2R
variation of g with depth is, 18. (c) Let the gravitational field at P, distant x
from mass m, be zero.
é dù Gm 4Gm
gd = g ê1 - ú =
ë Rû \ 2
x (r - x )2
Equating g h and gd , we get d = 2h 1 2
Þ = \ r – x = 2x
13. (c) Workdone, x r-x
r
é -GMm ù x=
W = DU = U f - U i = 0 - ê 3
ë R úû m P 4m
x
6.67 ´10 -11 ´ 100 10 r
W= ´
0.1 1000 Gravitational potential at P,
= 6.67 × 10–10 J V =-
Gm 4Gm
- =-
9Gm
r 2r r
4
G ´ r ´ pR 3 3 3
GM Gr ´ V 3 19. (a) Here, centripetal force will be given by the
14. (d) g = 2 = Þg= 2
R R 2 R gravitational force between the two
particles.
4
g = rpG. R where r ® average R
3
density
m m
æ 3g ö
r =ç
è 4pGR ÷ø
Þ r is directly proportional to g.
Gm 2
2GM p = mw 2 R Þ Gm = w 2
( 2R) 2
4 R3
(ve ) p Rp Mp Re
15. (c) = = ´ = Gm
(ve )e 2GM e Me Rp Þ w=
4 R3
Re If the velocity of the two particles with
respect to the centre of gravity is v then
10M e Re
= ´ = 10 v = wR
Me R e /10
Gm Gm
\ (ve ) p = 10 ´ (ve )e = 10 ´ 11 = 110 km / s v= 3
´R=
4R 4R
16. (b) Gravitational flux through a closed surface 20. (d) The work done to launch the spaceship
is given by
uuur r ¥ ur uur ¥
GMm
ò Eg × dS = -4p GM W = - ò F.dr = - ò 2 dr
R R r
www.crackjee.xyz
Gravitation P-47

GMm 2
GM é 1 1 ù
W =+ … (i) Þ + 2
R R êë 4 ú = Mv

The force of attraction of the earth on the
spaceship, when it was on the earth's Gm æ 2 + 4 ö 1 Gm
surface \ v= = (1 + 2 2)
R çè 4 2 ÷ø 2 R
GMm
F=
R2 23. (d) Due to complete solid sphere, potential at
point P
GMm GM
Þ mg = Þ g= 2 … (ii)
-GM é 2 æ R ö
2 2ù
R R
Vsphere = ê3R - ç ÷ ú
The required energy for this work is given by 2R 3 êë è2ø úû
= U - W = mgR
= 1000 × 10 × 6400 × 103 -GM æ 11R 2 ö GM
= ç ÷ = -11
= 6.4 × 1010 Joule 3 ç 4 ÷ 8R
2R è ø
21. (a) As we know, Solid
-GMm sphere
Gravitational potential energy =
r
and orbital velocity, v0 = GM / R + h
P
1 GMm 1 GM GMm
Ef = mv02 - = m - Cavity
2 3R 2 3R 3R
GMm æ 1 ö - GMm
= ç - 1÷ =
3R è 2 ø 6R Due to cavity part potential at point P
-GMm GM
Ei = +K 3 8 3GM
R Vcavity = - =-
Ei = E f 2 R 8R
Therefore minimum required energy, 2
So potential at the centre of cavity
5GMm
K= = Vsphere - Vcavity
6R
Mv 2 11GM æ 3 GM ö -GM
22. (d) 2 F cos 45° + F ' = (From figure) =- -ç- ÷=
R 8R è 8 R ø R
GM 2 GM 2
Where F = and F ' = 24. (b) For h << R, the orbital velocity is gR
( 2 R) 2 4R 2
Escape velocity = 2gR
F
M M \ The minimum increase in its orbital
F' velocity
F
R = 2gR – gR = gR ( 2 – 1)
o 25. (b) Variation of acceleration due to gravity, g
with distance 'd ' from centre of the earth
Gm
If d < R, g = 2 .d i.e., g µ d (straight line)
M M R
Gm
2 ´ GM 2 GM 2 Mv 2 If d = R, gs = 2
Þ + = R
2( R 2)2 4 R2 R Gm 1
If d > R, g = 2 i.e., g µ 2
d d
EBD_7764
www.crackjee.xyz
P-48 Physics

Mechanical Properties
of Solids 8
1. A spring of force constant 800 N/m has an weights W each are hung at the two ends, the
extension of 5 cm. The work done in extending it elongation of the wire will be (in mm) [2006]
from 5 cm to 15 cm is [2002] (a) l (b) 2l
(a) 16 J (b) 8 J
(c) zero (d) l/2
(c) 32 J (d) 24 J
2. A wire fixed at the upper end stretches by length 5. Two wires are made of the same material and have
l by applying a force F. The work done in the same volume. However wire 1 has cross-
stretching is [2004] sectional area A and wire 2 has cross-sectional
area 3A. If the length of wire 1 increases by Dx on
(a) 2Fl (b) Fl applying force F, how much force is needed to
F Fl stretch wire 2 by the same amount? [2009]
(c) (d) (a) 4 F (b) 6 F
2l 2
(c) 9 F (d) F
3. If ‘S’ is stress and ‘Y’ is young’s modulus of 6. The pressure that has to be applied to the ends of a
material of a wire, the energy stored in the wire steel wire of length 10 cm to keep its length constant
per unit volume is [2005] when its temperature is raised by 100ºC is:
2
S
(a) (b) 2S 2Y (For steel Young’s modulus is 2 ´ 1011 Nm -2 and
2Y coefficient of thermal expansion is
S 2Y
(c) (d) 1.1 ´ 10 -5 K -1 ) [2014]
2Y S2
(a) 2.2 ´ 10 Pa
8
(b) 2.2 ´ 10 Pa
9
4. A wire elongates by l mm when a load W is hanged
from it. If the wire goes over a pulley and two (c) 2.2 ´ 10 Pa
7
(d) 2.2 ´ 106 Pa

Answer Key
1 2 3 4 5 6
(b) (d) (a) (a) (c) (a)

1. (b) Small amount of work done in extending 800 é


the spring by dx is = (0.15)2 - (0.05)2 ù
2 ë û
dW = k x dx = 400 [(0.15 + 0.05)(0.15 – 0.05)]
0.15 = 400 × 0.2 × 0.1 = 8 J
\W= k ò x dx 2. (d) Work done by constant force in displacing
the object by a distance l.
0.05
= change in potential energy
www.crackjee.xyz
Mechanical Properties of Solids P-49

1
= × stress × strain ×volume
2 l
5. (c)
1 F l Fl A Y
= ´ ´ ´ A´ L =
2 A L 2
Wire (1)
3. (a) Energy stored per unit volume,
1
E= ´ stress ´ strain
2 3A Y
We know that,
stress stress
Y= or strain = l/3
strain Y
Wire (2)
1 stress 1 S2
E= ´ stress ´ = .
2 Y 2 Y As shown in the figure, the wires will have the
4. (a) Case (i) same Young’s modulus (same material) and
the length of the wire of area of cross-section
3A will be l/3 (same volume as wire 1).
For wire 1,
T F/A
Y= ...(i)
D x/l
T T For wire 2 ,
W F '/ 3 A
Y= ...(ii)
Dx /( l / 3)
W W F l F' l
From (i) and (ii) , ´ = ´
At equilibrium, T = W A Dx 3 A 3Dx
Þ F ' = 9F
W/A
Y= .....(1)
l/L stress
6. (a) Young's modulus Y =
Case (ii) At equilibrium T = W strain
stress = Y ´ strain
W/A W/A
\Y = ÞY = Stress in steel wire = Applied pressure
l/2 l/L
Pressure = stress = Y × strain
L/2
DL
Þ Elongation is the same. Str ain = = α D T (As length is
L
constant)
= 2 × 1011 × 1.1 × 10–5 × 100 = 2.2 × 108 Pa
EBD_7764
www.crackjee.xyz
P-50 Physics

Mechanical Properties
of Fluids 9
1. A cylinder of height 20 m is completely filled 5. If the terminal speed of a sphere of gold (den-
with water. The velocity of efflux of water sity = 19.5 kg/m3) is 0.2 m/s in a viscous liquid
(in ms-1) through a small hole on the side wall of (density = 1.5 kg/m3), find the terminal speed of
the cylinder near its bottom is [2002] a sphere of silver (density = 10.5 kg/m3) of the
(a) 10 (b) 20 same size in the same liquid [2006]
(c) 25.5 (d) 5 (a) 0.4 m/s (b) 0.133 m/s
(c) 0.1 m/s (d) 0.2 m/s
2. Spherical balls of radius ‘R’ are falling in a
6. A spherical solid ball of volume V is made of a
viscous fluid of viscosity ‘h’ with a velocity ‘v’. material of density r1. It is falling through a liquid
The retarding viscous force acting on the of density r1 (r2< r1). Assume that the liquid
spherical ball is [2004] applies a viscous force on the ball that is
(a) inversely proportional to both radius ‘R’ proportional to the square of its speed
and velocity ‘v’ v, i.e., Fviscous = –kv2 (k > 0). The terminal speed
(b) directly proportional to both radius ‘R’ and of the ball is [2008]
velocity ‘v’ Vg (r1 – r2 ) Vg r1
(c) directly proportional to ‘R’ but inversely (a) (b)
k k
proportional to ‘v’
(d) inversely proportional to ‘R’ but directly Vg r1 Vg (r1 – r2 )
(c) (d)
proportional to velocity ‘v’ k k
3. If two soap bubbles of different radii are 7. A jar is filled with two non-mixing liquids 1 and 2
having densities r1 and, r2 respectively. A solid
connected by a tube [2004]
ball, made of a material of density r3 , is dropped
(a) air flows from the smaller bubble to the in the jar. It comes to equilibrium in the position
bigger shown in the figure.Which of the following is
(b) air flows from bigger bubble to the smaller true for r1, r2and r3? [2008]
bubble till the sizes are interchanged r1

(c) air flows from the bigger bubble to the


smaller bubble till the sizes become equal r3
(d) there is no flow of air.
4. A 20 cm long capillary tube is dipped in water.
The water rises up to 8 cm. If the entire (a) r3 < r1 < r2 (b) r1 > r3 > r2
arrangement is put in a freely falling elevator the (c) r1 < r2 < r3 (d) r1 < r3 < r2
length of water column in the capillary tube will 8. A capillary tube (A) is dipped in water. Another
be [2005] identical tube (B) is dipped in a soap-water
(a) 10 cm (b) 8 cm solution. Which of the following shows the
relative nature of the liquid columns in the two
(c) 20 cm (d) 4 cm tubes? [2008]
www.crackjee.xyz
Mechanical Properties of Fluids P-51

A B

water oil
(a)

(c) (d)
oil
water
A B

10. Work done in increasing the size of a soap


bubble from a radius of 3 cm to 5 cm is nearly
(b) (Surface tension of soap solution = 0.03 Nm–1)
[2011]
(a) 0.2 p mJ (b) 2p mJ
(c) 0.4p mJ (d) 4p mJ
A B
11. Water is flowing continuously from a tap having
an internal diameter 8 × 10–3 m. The water
velocity as it leaves the tap is 0.4 ms–1. The
(c) diameter of the water stream at a distance
2 × 10–1 m below the tap is close to: [2011]
–3
(a) 7.5 × 10 m –3
(b) 9.6 × 10 m
–3
(c) 3.6 × 10 m (d) 5.0 × 10–3 m
A B 12. Two mercury drops (each of radius ‘r’) merge to
form bigger drop. The surface energy of the
bigger drop, if T is the surface tension, is :
(d) [2011 RS]
2 2
(a) 4pr T (b) 2pr T
(c) 28 / 3 pr 2T (d) 25/ 3 pr 2T
9. A ball is made of a material of density r where
13. If a ball of steel (density r = 7.8 g cm–3) attains
roil < r < rwater with roil and rwater represe- a terminal velocity of 10 cm s–1 when falling in
nting the densities of oil and water, respectively. water (Coefficient of viscosity hwater = 8.5 ×
The oil and water are immiscible. If the above
10–4 Pa.s), then, its terminal velocity in glycerine
ball is in equilibrium in a mixture of this oil and
water, which of the following pictures represents (r = 1.2 g cm–3, h = 13.2 Pa.s) would be, nearly
its equilibrium position ? [2010] [2011 RS]
(a) 6.25 × 10–4 cm s–1
(b) 6.45 × 10–4 cm s–1
(c) 1.5 × 10–5 cm s–1
water oil (d) 1.6 × 10–5 cm s–1
14. A thin liquid film formed between a U-shaped wire
and a light slider supports a weight of 1.5 × 10–2
(a) (b) N (see figure). The length of the slider is 30 cm
oil and its weight is negligible. The surface tension
water
of the liquid film is [2012]
EBD_7764
www.crackjee.xyz
P-52 Physics
water is T, value of r just before bubbles detach
is: (density of water is rw) [2014]
FILM

W
(a) 0.0125 Nm–1 (b) 0.1 Nm–1
(c) 0.05 Nm –1 (d) 0.025 Nm–1
15. There is a circular tube in a vertical plane. Two R
liquids which do not mix and of densities d1 and
d2 are filled in the tube. Each liquid subtends
90º angle at centre. Radius joining their interface 2r
d1 rw g rw g
makes an angle a with vertical. Ratio (a) R2 (b) R2
d2 3T 6T
is: [2014] rw g 3rw g
(c) R2 (d) R2
T T
17. An open glass tube is immersed in mercury in
such a way that a length of 8 cm extends above
the mercury level. The open end of the tube is
a d2 then closed and sealed and the tube is raised
vertically up by additional 46 cm. What will be
length of the air column above mercury in the
tube now?
d1 (Atmospheric pressure = 76 cm of Hg) [2014]
(a) 16 cm (b) 22 cm
1 + sin a 1 + cos a
(a) (b) (c) 38 cm (d) 6 cm
1 - sin a 1 - cos a
18. A man grows into a giant such that his linear
1 + tan a 1 + sin a dimensions increase by a factor of 9. Assuming
(c) (d) that his density remains same, the stress in the
1 - tan a 1 - cos a
leg will change by a factor of [2017]
16. On heating water, bubbles being formed at the
bottom of the vessel detach and rise. Take the 1
(a) 81 (b)
bubbles to be spheres of radius R and making a 81
circular contact of radius r with the bottom of 1
(c) 9 (d)
the vessel. If r << R and the surface tension of 9

Answer Key
1 2 3 4 5 6 7 8 9 10 11 12 13 14 15
(b) (b) (a) (c) (c) (a) (d) (c) (b) (c) (c) (c) (a) (d) (c)
16 17 18
(a) (a) (c)
www.crackjee.xyz
Mechanical Properties of Fluids P-53

1. (b) The velocity of efflux is given liquid. Therefore we can conclude that r1 < r2
v = 2gh Also r3 < r2 otherwise the ball would have
sink to the bottom of the jar.
Where h is the height of the free surface of
liquid from the hole Also r3 > r1 otherwise the ball would have
floated in liquid 1. From the above
\ v = 2 ´10 ´ 20 = 20 m / s discussion we conclude that r1 < r3 < r2.
2. (b) From Stoke's law, 8. (c) In case of water, the meniscus shape is
viscous force F = 6phrv concave upwards. Also according to
hence F is directly proportional to radius 2T cos q
& velocity. ascent formula h =
3. (a) Let pressure outside be P0. rrg
2T The surface tension (T) of soap solution
\ P1 ( in smaller bubble ) = P0 + is less than water. Therefore rise of soap
r solution in the capillary tube is less as
2T compared to water. As in the case of water,
P2 ( in bigger bubble ) = P0 + ( R > r)
R the meniscus shape of soap solution is also
\ P1 > P2 concave upwards.
hence air moves from smaller bubble to bigger 9. (b) Oil will float on water so, (2) or (4) is the
bubble. correct option. But density of ball is more
4. (c) Water fills the tube entirely in gravityless than that of oil,, hence it will sink in oil.
condition i.e., 20 cm. 10. (c) W = 2T DV
2 W = 2T4p[(52) – (3)2] × 10–4
2r ( d1 - d 2 ) g
5. (c) Terminal velocity, vT = = 2 × 0.03 × 4p [25 – 9] × 10–4 J
9h = 0.4p × 10–3 J
vT = 0.4p mJ
2 (10.5 - 1.5) 9 11. (c) From Bernoulli's theorem,
= Þ vT = 0.2 ´
0.2 (19.5 - 1.5) 2 18 1 1
\ vT = 0.1 m/s P0 + rv12rgh = P0 + rv22 + 0
2 2 2
6. (a) The condition for terminal speed (vt) is
Weight = Buoyant force + Viscous force v2 = v12 + 2 gh = 0.16 + 2 ´ 10 ´ 0.2
Fv B=Vr2 g = 2.03 m/s
From equation of continuity
A2v2 = A1v1
D22 D2
p ´ v2 = p 1 v1
4 4

v1
Þ D2 = D1 = 3.55 × 10–3 m
v2
W=V r 1g 12. (c) Sum of volumes of 2 smaller drops
= Volume of the bigger drop
\ V r1 g = V r2 g + kvt2 4 4
2. pr 3 = pR 3 Þ R = 21/ 3 r
Vg ( r1 - r 2 ) 3 3
\ vt = 2
k Surface energy = T .4pR
7. (d) From the figure it is clear that liquid 1 floats on
liquid 2. The lighter liquid floats over heavier = T 4p 22 / 3 r 2 = T .28 / 3 pr 2 .
EBD_7764
w w w . c r a c k j e e . x y
P-54 Physics

13. (a) V rg = 6phrv + V rl g


Þ Vg ( r - rl ) = 6phrv

(
'
)
Also Vg r- ri = 6ph ' rv ' R
q

(r - rl' )
\ v 'h' = ´ vh rq
(r - r l ) T×dl
Force due to excess pressure = upthrust
(r - rl' ) vh
Þ v' = ´ 2T
(r - rl ) h ' Access pressure in air bubble =
R
(7.8 - 1.2) 10 ´ 8.5 ´ 10-4
= ´ 2T 4p R 3
(7.8 - 1) 13.2 (p r 2 ) = rw g
R 3T
\ v ' = 6.25 ´ 10-4 cm/s
14. (d) The surface tension of the liquid film is 2 2 R 4 rw g 2 2rw g
Þr = Þ r=R
F 3T 3T
given as T = , where F is the force, and
2l
l = length of the slider. 17. (a) (54–x)
F = 2lT 54 cm P
At equilibrium, F = W 8 cm x
2Tl = mg
mg 1.5 ´ 10 -2 1.5
T= = - 2
=
2 l 2 ´ 30 ´ 10 60
= 0.025 Nm–1 Hg
15. (c) Pressure at interface A must be same from
both the sides to be in equilibrium.
Length of the air column above mercury in
the tube is,
P + x = P0
R d2 Rsina Þ P = (76 – x)
a
Þ 8 × A × 76 = (76 – x) × A × (54 – x)
a Rcosa R \ x = 38
Rsin a – Rsin a Thus, length of air column
A = 54 – 38 = 16 cm.
d1
18. (c) As linear dimension increases by a factor of 9
\ ( R cos a + R sin a )d 2 g vf
\ = 93
= ( R cos a - R sin a )d1 g vi
d1 cos a + sin a 1 + tan a Q Density remains same
Þ d = cos a - sin a = 1 - tan a So, mass µ Volume
2 mf ( Area ) f
16. (a) When the bubble gets detached, = 93 Þ = 92
mi ( Area ) i
Buoyant force = force due to surface
tension force (mass ) ´ g
Stress (s) = =
area area
s 2 æ m f ö æ Ai ö 9 3
= ç ÷ = =9
s1 çè mi ÷ø è A f ø 92
www.crackjee.xyz

Thermal Properties
of Matter 10
1. Heat given to a body which raises its temperature 7. According to Newton’s law of cooling, the rate
by 1°C is [2002] of cooling of a body is proportional to (Dq)n,
(a) water equivalent where Dq is the difference of the temperature of
(b) thermal capacity the body and the surroundings, and n is equal to
(c) specific heat [2003]
(d) temperature gradient (a) two (b) three
2. Infrared radiation is detected by [2002] (c) four (d) one
(a) spectrometer (b) pyrometer 8. If the temperature of the sun were to increase
(c) nanometer (d) photometer from T to 2T and its radius from R to 2R, then the
3. Which of the following is more close to a black ratio of the radiant energy received on earth to
body? [2002] what it was previously will be [2004]
(a) black board paint (a) 32 (b) 16
(b) green leaves (c) 4 (d) 64
(c) black holes 9. The temperature of the two outer surfaces of a
(d) red roses composite slab, consisting of two materials
4. If mass-energy equivalence is taken into having coefficients of thermal conductivity K
account, when water is cooled to form ice, the and 2K and thickness x and 4x, respectively, are
mass of water should [2002] T2 and T1 (T2 > T1 ) . The rate of heat transfer
(a) increase
(b) remain unchanged through the slab, in a steady state is
(c) decrease æ A(T2 - T1 ) K ö
(d) first increase then decrease çè ÷ø f , with f equal to [2004]
x
5. Two spheres of the same material have radii 1 m
and 4 m and temperatures 4000 K and 2000 K
respectively. The ratio of the energy radiated x 4x

per second by the first sphere to that by the


second is [2002]
(a) 1 : 1 (b) 16 : 1 2K T1
K
(c) 4 : 1 (d) 1 : 9.
6. The earth radiates in the infra-red region of the
spectrum. The spectrum is correctly given by
[2003]
(a) Rayleigh Jeans law 2 1
(a) (b)
(b) Planck’s law of radiation 3 2
(c) Stefan’s law of radiation 1
(d) Wien’s law (c) 1 (d)
3
EBD_7764
www.crackjee.xyz
P-56 Physics
10. The figure shows a system of two concentric T1 l1 l2 T2
spheres of radii r1 and r2 are kept at temperatures
T1 and T2, respectively. The radial rate of flow of
heat in a substance between the two concentric K1 K2
spheres is proportional to [2005] ( K1l1T1 + K 2 l2T2 )
(a)
( K1l1 + K 2 l2 )
r1 ( K 2l2T1 + K1l1T2 )
(b)
T1 ( K1l1 + K 2 l2 )
r2 T2 ( K 2l1T1 + K1l2T2 )
(c)
( K 2 l1 + K1l2 )
ær ö (r2 - r1 )
(a) ln ç 2 ÷ (b) ( K1l2T1 + K 2l1T2 )
è r1 ø (r1 r2 ) (d)
( K1l2 + K 2 l1 )
r1 r2
(c) ( r2 - r1 ) (d) 14. A long metallic bar is carrying heat from one of
(r2 - r1 ) its ends to the other end under steady–state.
11. Assuming the Sun to be a spherical body of The variation of temperature q along the length
radius R at a temperature of TK, evaluate the x of the bar from its hot end is best described by
total radiant powerd incident of Earth at a which of the following figures? [2009]
distance r from the Sun [2006]
q q
T4 T4
(a) 4pr02 R 2s 2 (b) pr02 R 2s 2
r r (a) (b)
4 4
T T x x
(c) r02 R2 s 2 (d)
2
R s 2
4pr r
where r0 is the radius of the Earth and s is q q
Stefan's constant.
12. Two rigid boxes containing different ideal gases
are placed on a table. Box A contains one mole (c) (d)
of nitrogen at temperature T0, while Box contains x x
æ 7ö
one mole of helium at temperature çè ÷ø T0 . The 15. 100g of water is heated from 30°C to 50°C.
3
boxes are then put into thermal contact with each Ignoring the slight expansion of the water, the
other, and heat flows between them until the change in its internal energy is (specific heat of
gases reach a common final temperature (ignore water is 4184 J/kg/K): [2011]
the heat capacity of boxes). Then, the final (a) 8.4 kJ (b) 84 kJ
temperature of the gases, Tf in terms of T0 is (c) 2.1 kJ (d) 4.2 kJ
[2006] 16. The specific heat capacity of a metal at low
3 7
(a) T f = T0 (b) T f = T0 temperature (T) is given as
7 3
3 5 3
æ T ö
(c) T f = T0 (d) T f = T0 C p (kJK -1kg -1 ) = 32 ç
2 2 è 400 ÷ø
13. One end of a thermally insulated rod is kept at a
A 100 gram vessel of this metal is to be cooled
temperature T1 and the other at T2. The rod is
from 20ºK to 4ºK by a special refrigerator
composed of two sections of length l1 and l2
operating at room temperature (27°C). The
and thermal conductivities K 1 and K 2
amount of work required to cool the vessel is
respectively. The temperature at the interface of
[2011 RS]
the two section is [2007]
www.crackjee.xyz
Thermal Properties of Matter P-57
(a) greater than 0.148 kJ
(b) between 0.148 kJ and 0.028 kJ T T
(c) less than 0.028 kJ q0
(d) equal to 0.002 kJ (a) (b)
17. A wooden wheel of radius R is made of two O t O t
semicircular part (see figure). The two parts are
held together by a ring made of a metal strip of T
T
cross sectional area S and length L. L is slightly
less than 2pR. To fit the ring on the wheel, it is q0 q0
(c) (d)
heated so that its temperature rises by DT and it
just steps over the wheel. As it cools down to O t O t
surrounding temperature, it presses the 20. Three rods of Copper, Brass and Steel are welded
semicircular parts together. If the coefficient of together to form a Y shaped structure. Area of
linear expansion of the metal is a, and its cross - section of each rod = 4 cm2. End of
Young's modulus is Y, the force that one part of copper rod is maintained at 100ºC where as ends
the wheel applies on the other part is : [2012] of brass and steel are kept at 0ºC. Lengths of the
copper, brass and steel rods are 46, 13 and 12
cms respectively. The rods are thermally
R insulated from surroundings excepts at ends.
Thermal conductivities of copper, brass and
steel are 0.92, 0.26 and 0.12 CGS units
respectively. Rate of heat flow through copper
rod is: [2014]
(a) 2pSY aDT (b) SY aDT (a) 1.2 cal/s (b) 2.4 cal/s
(c) pSY aDT (d) 2SY aDT (c) 4.8 cal/s (d) 6.0 cal/s
18. A liquid in a beaker has temperature q(t) at time 21. Consider a spherical shell of radius R at
t and q0 is temperature of surroundings, then temperature T. The black body radiation inside it
according to Newton's law of cooling the correct can be considered as an ideal gas of photons with
graph between loge(q – q0) and t is : [2012] U
internal energy per unit volume u = µ T4 and
V
loge (q – q0)
loge (q – q 0)

1æUö
pressure p = ç ÷ . If the shell now undergoes
3è V ø
an adiabatic expansion the relation between T
(a) (b) and R is : [2015]
t 0 1 1
0 t (a) T µ (b) T µ 3
R R
(c) T µ e–R (d) T µ e–3R
loge (q – q0)

loge (q – q0)

22. A pendulum clock loses 12 s a day if the


temperature is 40°C and gains 4 s a day if the
(c) (d) temperature is 20°C. The temperature at which
0 t 0 the clock will show correct time, and the co-
t
efficient of linear expansion (a) of the metal of
19. If a piece of metal is heated to temperature q and the pendulum shaft are respectively : [2016]
then allowed to cool in a room which is at
(a) 30°C; a = 1.85 × 10–3/°C
temperature q 0 , the graph between the
temperature T of the metal and time t will be (b) 55°C; a = 1.85 × 10–2/°C
closest to [2013] (c) 25°C; a = 1.85 × 10–5/°C
(d) 60°C; a = 1.85 × 10–4/°C
EBD_7764
www.crackjee.xyz
P-58 Physics
23. A copper ball of mass 100 gm is at a temperature 24. An external pressure P is applied on a cube at
T. It is dropped in a copper calorimeter of mass 0oC so that it is equally compressed from all
100 gm, filled with 170 gm of water at room sides. K is the bulk modulus of the material of
temperature. Subsequently, the temperature of the cube and a is its coefficient of linear
the system is found to be 75°C. T is given by expansion. Suppose we want to bring the cube
(Given : room temperature = 30° C, specific heat to its original size by heating. The temperature
of copper = 0.1 cal/gm°C [2017] should be raised by : [2017]
(1) 1250°C (2) 825°C 3a
(1) (2) 3PKa
(3) 800°C (4) 885° C PK
P P
(3) (4)
3a K aK
Answer Key
1 2 3 4 5 6 7 8 9 10 11 12 13 14 15
(b) (b) (a) (c) (a) (d) (d) (d) (d) (d) (b) (c) (d) (a) (a)
16 17 18 19 20 21 22 23 24
(d) (d) (a) (c) (c) (a) (c) (d) (c)

1. (b) Heat required for raising the temperature 2 2 4


8. (d) E = s AT 4 ; A µ R \ E µ R T
of the whole body by 1ºC is called the
thermal capacity of the body. E2 R22 T24
\ =
2. (b) Pyrometer is used to detect infra-red E1 R12 T14
radiation. put R2 = 2R, R1 = R ; T2 = 2T, T1 = T
3. (a) Black board paint is quite approximately
equal to black bodies. E2 (2R )2 (2T )4
Þ = = 64
4. (c) When water is cooled to form ice, energy E1 R 2T 4
is released from water in the form of heat. 9. (d) The thermal resistance is given by
As energy is equivalent to mass, therefore, x 4x x 2 x 3x
when water is cooled to ice, its mass + = + =
KA 2KA KA KA KA
decreases. dQ DT (T2 - T1 ) KA
5. (a) The energy radiated per second is given by \ = =
dt 3x 3x
E = esT4 A
KA
For same material e is same. s is stefan's 1 ì A(T2 - T1 ) K ü
constant = í ý 1
3î x þ \f =
F1 T14 A1 T14 4pr12 3
\ = = 10. (d)
F2 T24 A2 T24 4pr22 T - dT

dr
(4000)4 ´12 1
= = ·
4 2 1
(2000) ´ 4 T1
r1
r
6. (d) Wein’s law correctly explains the spectrum
7. (d) According to Newton’s law of cooling
dQ T2
- µ (Dq) r2
dt
www.crackjee.xyz
Thermal Properties of Matter P-59
Consider a shell of thickness (dr) and of 13. (d) Let T be the temperature of the interface.
radiius (r) and let the temperature of inner As the two sections are in series, the rate
and outer surfaces of this shell be T and of flow of heat in them will be equal.
(T – dT) respectively. T1 T2
1 2
dQ
= rate of flow of heat through it
dt
K1 K2
KA[(T - dT ) - T ] - KAdT
= = K1 A(T1 - T ) K 2 A(T - T2 )
dr dr \ = ,
dT l1 l2
2
= -4pKr (Q A = 4pr 2 ) where A is the area of cross-section.
dr
To measure the radial rate of heat flow, or, K1 A(T1 - T )l 2 = K 2 A(T - T2 )l1
integration technique is used, since the area
of the surface through which heat will flow or, K1T1l 2 - K1T l 2 = K 2T l1 - K 2T2 l1
is not constant. or, ( K 2 l1 + K1l 2 )T = K1T1l 2 + K2T2 l1

æ dQ ö
r2
1
T2 K1T1l 2 + K 2T2 l1
Then, ç ò r2 dr = -4 pK ò dT \T =
è dt ÷ø K 2 l1 + K1l 2
r1 T1
K1l 2T1 + K 2 l1T2
dQ é 1 1 ù = .
ê - ú = -4pK [T2 - T 1 ] K1l 2 + K 2 l1
dt ë r1 r2 û 14. (a) The heat flow rate is given by
dQ -4pKr1r2 (T2 - T1 ) dQ kA(q1 - q)
or = =
dt (r2 - r1 ) dt x
dQ r r x dQ x dQ
\ µ 1 2 Þ q -q = Þ q = q1 -
1 kA dt
dt (r2 - r1 ) kA dt
11. (b) Total power radiated by Sun = sT4 × 4pR2 where q1 is the temperature of hot end and
The intensity of power at earth's surface q is temperature at a distance x from hot
end.
sT 4 ´ 4pR2 The above equation can be graphically
=
4pr 2 represented by option (a).
Total power received by Earth 15. (a) DU = DQ = mcDT
= 100 × 10–3 × 4184 (50 – 30) » 8.4 kJ
sT 4 R 2 16. (d) Required work = energy released
= (pr02 )
ò
2
r Here, Q = mc dT
12. (c) Heat lost by He = Heat gained by N2
4
æ T3 ö
n1Cv1 DT1 = n2Cv2 DT2 =ò 0.1 ´ 32 ´ ç
è 4003 ø
÷ dT » 0.002 kJ.
20
3 é7 ù 5 é
R T - Tf = R T f - T0 ùû Therefore, required work = 0.002 kJ
2 ëê 3 0 ûú 2 ë 17. (d) The Young modulus is given as
7T0 - 3T f = 5T f - 5T0 F/S
Y=
DL / L
12 Here it is given as
Þ 12T0 = 8T f Þ T f = T0
8 F
Y= ´ 2 pR {L = 2pR}
3 S2 pDR
Þ T f = T0 .
2 FR
or Y = …(i)
S.DR
EBD_7764
w w w . c r a c k j e
P-60 Physics
The coefficient of linear expansion is given 1æUö
DR 21. (a) As, P = ç ÷
as a = 3èVø
RDT
U
DR R 1 But = KT 4
Þ = a.DT Þ = … (ii) V
R DR aDT
From equation (i) and (ii) 1 4
So, P = KT
F 3
Y= Þ F = Y.S.aDT
S.aDT uRT 1
\ The ring is pressing the wheel from or = KT 4 [As PV = u RT]
V 3
both sides, Thus
Fnet = 2F = 2YSaDT 4 3 3
pR T = constant
18. (a) Newton's law of cooling 3
dq 1
= - k(q - q0 ) Therefore, Tµ
dt R
dq 1
Þ = - kdt Time lost/gained per day =
(q - q0 ) 22. (c) µ Dq ´ 86400
2
Integrating second
Þ log(q - q0 ) = -kt + c
Which represents an equation of straight 1
12 = a (40 – q) ´ 86400 .... (i)
line. 2
Thus the option (a) is correct. 1
19. (c) According to Newton’s law of cooling, the 4 = a (q – 20) ´ 86400 ....(ii)
2
temperature goes on decreasing with time
non-linearly. 40 – q
On dividing we get, 3 =
20. (c) Rate of heat flow is given by, q – 20
3q – 60 = 40 – q
KA(q1 - q 2 ) 4q = 100 Þ q = 25°C
Q=
l
23. (d) According to principle of calorimetry,
Where, K = coefficient of thermal
conductivity Heat lost = Heat gain
l = length of rod and A = area of cross-
section of rod 100 × 0.1( – 75) = 100 × 0.1 × 45 + 170 × 1 × 45

100°C 10 – 750 = 450 + 7650


10 = 1200 + 7650 = 8850
Copper
T = 885°C
T 24. (c) As we know, Bulk modulus
B Brass
Steel
0°C DP DV P
0°C K= Þ =
If the junction temperature is T, then æ -DV ö V K
çè ÷
QCopper = QBrass + QSteel V ø a
V = V0 (1 + gDt)
0.92 ´ 4(100 - T ) 0.26 ´ 4 ´ (T - 0)
= + DV
46 13 = gDt
V0
0.12 ´ 4 ´ (T - 0)
12 P P P
\ = gDt Þ Dt = =
Þ 200 – 2T = 2T + T K gK 3aK
Þ T = 40°C
0.92 ´ 4 ´ 60
\ QCopper = = 4.8 cal/s
46
www.crackjee.xyz
Thermodynamics P-61

Thermodynamics 11
1. Which statement is incorrect? [2002] 7. Which of the following statements is correct
(a) All reversible cycles have same efficiency for any thermodynamic system ? [2004]
(b) Reversible cycle has more efficiency than (a) The change in entropy can never be zero
an irreversible one (b) Internal energy and entropy are state
(c) Carnot cycle is a reversible one functions
(d) Carnot cycle has the maximum efficiency (c) The internal energy changes in all
in all cycles processes
2. Even Carnot engine cannot give 100% efficiency (d) The work done in an adiabatic process is
because we cannot [2002] always zero.
(a) prevent radiation 8. Two thermally insulated vessels 1 and 2 are filled
(b) find ideal sources with air at temperatures (T1, T2), volume (V1, V2),
(c) reach absolute zero temperature and pressure (P1, P2) respectively. If the valve
(d) eliminate friction joining the two vessels is opened, the
3. “Heat cannot by itself flow from a body at lower temperature inside the vessel at equilibrium will
temperature to a body at higher temperature” is be [2004]
a statement or consequence of [2003] (a) T1T2(P1V1 + P2V2)/(P1V1T2 + P2V2T1)
(a) second law of thermodynamics
(b) (T1 + T2)/2
(b) conservation of momentum
(c) conservation of mass (c) T1 + T2
(d) first law of thermodynamics (d) T1T2(P1V1 + P2V2)/(P1V1T1 + P2V2T2)
4. During an adiabatic process, the pressure of a 9. Which of the following is incorrect regarding
gas is found to be proportional to the cube of its the first law of thermodynamics? [2005]
absolute temperature. The ratio CP/CV for the (a) It is a restatement of the principle of
gas is [2003] conservation of energy
4 (b) It is not applicable to any cyclic process
(a) (b) 2 (c) It introduces the concept of the entropy
3
5 3 (d) It introduces the concept of the internal
(c) (d) energy
3 2
5. Which of the following parameters does not 10. A system goes from A to B via two processes I
characterize the thermodynamic state of matter? and II as shown in figure. If DU1 and DU2 are the
[2003] changes in internal energies in the processes I
(a) Temperature (b) Pressure and II respectively, then [2005]
(c) Work (d) Volume p II
6. A Carnot engine takes 3 × 106 cal of heat from a
reservoir at 627°C, and gives it to a sink at 27°C. A B
The work done by the engine is [2003] I
(a) 4.2 × 106 J (b) 8.4 × 106 J
(c) 16.8 × 10 J6 (d) zero v
EBD_7764
www.crackjee.xyz
P-62 Physics
(a) relation between DU1 and DU2 can not be 15. An insulated container of gas has two chambers
determined separated by an insulating partition. One of the
(b) DU1 = DU2 chambers has volume V1 and contains ideal gas
(c) DU2 = DU1 DU 2 < DU1 at pressure P1 and temperature T1. The other
(d) DU2 > DU1 chamber has volume V2 and contains ideal gas
11. The temperature-entropy diagram of a reversible at pressure P2 and temperature T2. If the partition
engine cycle is given in the figure. Its efficiency is removed without doing any work on the gas,
is [2005] the final equilibrium temperature of the gas in
T the container will be [2008]

(a) 1 1 + P2V2 )
T1T2 ( PV
2T0
1 1 2 + P2V2T1
PV T
T0
1 1T1 + P2V2T2
PV
(b)
1 1 + P2V2
PV
S
S0 2S0
1 1T2 + P2V2T1
PV
1 1 (c)
(a) (b) 1 1 + P2V2
PV
4 2
2 1 1 + P2V2 )
T1T2 ( PV
1 (d)
(c)
3
(d) 1 1 1 + P2V2T2
PV T
3
12. The work of 146 kJ is performed in order to Directions for questions 16 to 18 : Questions are
compress one kilo mole of gas adiabatically and based on the following paragraph.
in this process the temperature of the gas Two moles of helium gas are taken over the cycle
increases by 7°C. The gas is ABCDA, as shown in the P-T diagram. [2009]
(R = 8.3 J mol–1 K–1) [2006]
(a) diatomic 5 A B
(b) triatomic 2 × 10
(c) a mixture of monoatomic and diatomic
(d) monoatomic P (Pa)
13. When a system is taken from state i to state f
along the path iaf, it is found that Q =50 cal and 5
1 × 10 D C
W = 20 cal. Along the path ibf Q = 36 cal. W along T
the path ibf is [2007] 300K 500K
a f 16. Assuming the gas to be ideal the work done on
the gas in taking it from A to B is
(a) 300 R (b) 400 R
i b (c) 500 R (d) 200 R
(a) 14 cal (b) 6 cal 17. The work done on the gas in taking it from D to
(c) 16 cal (d) 66 cal A is
14. A Carnot engine, having an efficiency of h = 1/
(a) + 414 R (b) – 690 R
10 as heat engine, is used as a refrigerator. If the
work done on the system is 10 J, the amount of (c) + 690 R (d) – 414 R
energy absorbed from the reservoir at lower 18. The net work done on the gas in the cycle
temperature is [2007] ABCDA is
(a) 100 J (b) 99 J (a) 276 R (b) 1076 R
(c) 90 J (d) 1 J
(c) 1904 R (d) zero
www.crackjee.xyz
Thermodynamics P-63
19. A diatomic ideal gas is used in a Carnot engine The above p-v diagram represents the
as the working substance. If during the adiabatic thermodynamic cycle of an engine, operating
expansion part of the cycle the volume of the with an ideal monatomic gas. The amount of heat,
gas increases from V to 32 V, the efficiency of extracted from the source in a single cycle is
the engine is [2010] [2013]
(a) 0.5 (b) 0.75
(c) 0.99 (d) 0.25 æ 13 ö
(a) p 0 v 0 (b) ç ÷ p0 v0
20. A Carnot engine operating between è2ø

temperatures T1 and T2 has efficiency


1
. When æ 11 ö
6 (c) ç ÷ p0 v0 (d) 4p0v0
è2ø
T2 is lowered by 62 K its efficiency increases to 24. One mole of a diatomic ideal gas undergoes a
1 cyclic process ABC as shown in figure. The
. Then T1 and T2 are, respectively: [2011] process BC is adiabatic. The temperatures at A,
3
(a) 372 K and 310 K (b) 330 K and 268 K B and C are 400 K, 800 K and 600 K respectively.
(c) 310 K and 248 K (d) 372 K and 310 K Choose the correct statement: [2014]
21. Helium gas goes through a cycle ABCDA
(consisting of two isochoric and isobaric lines)
as shown in figure. The efficiency of this cycle
B
is nearly : (Assume the gas to be close to ideal 800 K
gas) [2012]
(a) 15.4 % (b) 9.1 %
(c) 10.5% (d) 12.5 % P

2P0 B C 600 k
A C
400 K
P0 D
A V
(a) The change in internal energy in whole
V0 2V0 cyclic process is 250 R.
22. A Carnot engine, whose efficiency is 40%, takes (b) The change in internal energy in the process
in heat from a source maintained at a temperature CA is 700 R.
of 500K. It is desired to have an engine of (c) The change in internal energy in the
efficiency 60%. Then, the intake temperature for process AB is -350 R.
the same exhaust (sink) temperature must be : (d) The change in internal energy in the
[2012] process BC is – 500 R.
(a) efficiency of Carnot engine cannot be made 25. A solid body of constant heat capacity 1 J/°C is
larger than 50% being heated by keeping it in contact with
(b) 1200 K reservoirs in two ways : [2015]
(c) 750 K (i) Sequentially keeping in contact with 2
(d) 600 K reservoirs such that each reservoir supplies
23. p same amount of heat.
A B (ii) Sequentially keeping in contact with 8
2p0
reservoirs such that each reservoir supplies
p0 same amount of heat.
D C In both the cases body is brought from initial
temperature 100°C to final temperature 200°C.
v0 2v0 v
EBD_7764
www.crackjee.xyz
P-64 Physics
Entropy change of the body in the two cases 9P0 V0 9P0 V0
respectively is : (a) (b)
2nR nR
(a) ln2, 2ln2 (b) 2ln2, 8ln2
(c) ln2, 4ln2 (d) ln2, ln2 9P0 V0 3P0 V0
(c) (d)
26. 'n' moles of an ideal gas undergoes a process 2nR 2nR
A ® B as shown in the figure. The maximum 27. An ideal gas undergoes a quasi static, reversible
temperature of the gas during the process will process in which its molar heat capacity C
be : [2016] remains constant. If during this process the
P relation of pressure P and volume V is given by
A PVn = constant, then n is given by (Here CP and
2P0 CV are molar specific heat at constant pressure
and constant volume, respectively) : [2016]

P0 CP - C C - CV
B (a) n= (b) n=
C - CV C - CP

CP C – CP
(c) n= (d) n=
V0 2V0 V CV C – CV

Answer Key
1 2 3 4 5 6 7 8 9 10 11 12 13 14 15
(a) (c) (a) (d) (c) (b) (b) (a) (b, c) (b) (d) (a) (b) (c) (a)
16 17 18 19 20 21 22 23 24 25 26 27
(b) (a) (a) (b) (d) (a) (c) (b) (d) (d) (c) (d)

1. (a) All reversible engines working for the same g


temperature of source and sink have same 1-g ....(ii)
Þ PT = constt.
efficiencies. If the temperatures are
different, the efficiency is different. g
From (i) an d (ii) = -3
2. (c) In Carnot's cycle we assume frictionless 1- g
piston, absolute insulation and ideal source 3
and sink (reservoirs). The efficiency of Þ g = -3 + 3g Þ g=
2
T2 5. (c) Work is a path function. The remaining
carnot's cycle is given by h = 1 - T three parameters are state function.
1
For h = 1 or 100 %, T2 = 0 K. T2 (273 + 27)
The temperature of 0 K (absolute zero) can 6. (b) h = 1 - T = 1 - (273 + 627)
1
not be obtained . 300 1 2
3. (a) This is a statement of second law of =1- = 1- =
thermodynamics 900 3 3
W
4. (d) P µ T 3 Þ PT -3 = constant ....(i) But h =
Q
But for an adiabatic process, the pressure W 2 2 2
temperature relationship is given by \ = Þ W = ´ Q = ´ 3 ´ 106
P1–g Tg = constant Q 3 3 3
www.crackjee.xyz
Thermodynamics P-65

= 2 ´ 106 cal Q2 TS 1
= 1- = 1- 0 0 =
Q1 3 3
= 2 ´ 106 ´ 4.2 J = 8.4 ´ 106 J T S
2 0 0
7. (b) Internal energy and entropy are state
function, they do not depend upon path nRDT 1000 ´ 8.3 ´ 7
12. (a) W= Þ -146000 =
taken. 1- g 1- g
8. (a) Here Q = 0 and W = 0. Therefore, from first 58.1 58.1
law of thermodynamics DU = Q + W = 0 or 1 - g = - Þ g = 1+ = 1.4
146 146
\ Internal energy of the system with Hence the gas is diatomic.
partition = Internal energy of the system 13. (b) For path iaf, Q = 50 cal, W = 20 cal
without partition.
a f
n1Cv T1 + n2 Cv T2 = (n1 + n2 )Cv T
n1T1 + n2T2
\ T=
n1 + n2 i b
PV PV By first law of thermodynamics,
1 1
But n1 = and n2 = 2 2 DU = Q - W = 50 – 20 = 30 cal.
RT1 RT2
For path ibf
PV
1 1 PV Q = 36 cal
´ T1 + 2 2 ´ T2
RT1 RT2 W =?
\\ T = or, W = Q – DU
PV
1 1 + 2V2
P
RT1 RT2 (Since, the change in internal energy does not
depend on the path, therefore DU = 30 cal)
1 1 + P2V2 )
T1T2 ( PV \ W = Q – DU = 36 – 30 = 6 cal.
=
1 1T2 + P2V2T1
PV 14. (c) The efficiency (h) of a Carnot engine and
9. (b, c)First law is applicable to a cyclic process. the coefficient of performance (b) of a
Concept of entropy is introduced by the refrigerator are related as
second law. 1- h 1
10. (b) Change in internal energy do not depend b= Here, h =
h 10
upon the path followed by the process. It
only depends on initial and final states i.e., 1
1-
DU1 = DU2 10 = 9.
\ b=
1 3 æ 1ö
11. (d) Q1 = T0 S 0 + T0 S0 = T0 S 0 çè ÷ø
2 2 10
Q2 = T0(2S0 – S0) = T0S0 Also, Coefficient of performance (b) is given
and Q3 = 0 Q
by b = 2 , where Q 2 is the energy
W Q1 - Q2 W
h= = absorbed from the reservoir.
Q1 Q1
or, 9 = Q2 \ Q2 = 90 J.
T 10
15. (a) Here Q = 0 and W = 0. Therefore from first
law of thermodynamics DU = Q + W = 0
2T0 \ Internal energy of the system with
Q1
Q3 partition = Internal energy of the system
T0 without partition.
Q2 n1Cv T1 + n2 Cv T2 = (n1 + n2 )Cv T
S
S0 2S 0 n T + n2 T2
\T = 1 1
n1 + n2
EBD_7764
w w w . c r a c k j e e . x
P-66 Physics
PV
1 1 PV 7
But n1 = and n2 = 2 2 For diatomic gas, g =
RT1 RT2 5
PV PV 2
1 1
´ T1 + 2 2 ´ T2 \ g -1 =
RT1 RT2 5
\T= 2
PV
1 1 + 2V2
P
\ T = (32) 5 .T Þ T1 = 4T2
RT1 RT2 1 2
T2
1 1 + P2V2 )
T1T2 ( PV
Now, efficiency = 1 - T
=
1 1T2 + P2V2T1
PV 1
16. (b) A to B is an isobaric process. The work T2 1 3
done = 1- = 1 - = = 0.75.
4T2 4 4
W = nR (T2 - T1 )
20. (d) Efficiency of engine
= 2R (500 - 300) = 400 R T2
h1 = 1 -
17. (a) Work done by the system intheisothermal T1
process
T
2 5
P Þ T =6 ....(i)
DA is W = 2.303 nRT log10 D 1
PA
T2 - 62 1
Again, h2 = 1 - = ....(ii)
= 2.303 ´ 2 R ´ 300 T1 3

1 ´ 105 Solving (i) and (ii), we get,


log10 – 414R. 5
2 ´ 105 T1 = 372 K and T2 = × 372 = 310 K
Therefore, work done on the gas is + 414 R. 6
18. (a) The net work in the cycle ABCDA is output work
21. (a) The efficiency h =
W = W AB + WBC + WCD + WDA input work
Input work = Work done in going A to B +
PB
= 400R + 2.303nRT log + (-400R) - 414R workdone in going B to C and the work
PC done in going C to D.
n n
2 ´ 105 Wi = (P0V0 ) + (2P0V0 ) + 2P0V0
= 2.303 ´ 2R ´ 500log - 414 R 2 2
1 ´ 105 where n = degree of freedom
= 693.2 R – 414 R = 279.2 R
which is 3 for mono-atomic gases like He
19. (b)
æ3 3 ö
= ç + .2 + 2 ÷ P0 V0
P è2 2 ø
T1
(V, T1)
æ 3 + 10 ö 13
=ç ÷ P0 V0 = P0 V0
è 2 ø 2
(32 V, T2)
and W0 = P0V0
T2 P0V0 2
h= =
V 13 13
P0V0
g -1
2
We have, TV = constant Efficiency in %
g -1
Þ T1V = T2 (32V )g -1 2 200
g -1
h= ´ 100 = ; 15.4%
Þ T1 = (32) .T2 13 13
www.crackjee.xyz
Thermodynamics P-67
22. (c) The efficiency of the engine is given as
- P0
æ T ö P = V V + 3P
h = ç 1 - 2 ÷ ´ 100 0
è T1 ø
For first case -P0
[slope = , c = 3P0]
T1 = 500 K; h = 40 V0

æ T2 ö PV0 + P0V = 3P0V0 ...(i)


40 = ç 1 - ÷ ´ 100 But pv = nRT
è 500 ø
40 T2 nRT
\p= ...(ii)
Þ = 1- v
100 500
T2 60 nRT
Þ = Þ T2 = 300 K From (i) & (ii) V0 + P0V = 3P0V0
500 100 v
For second case : \ nRT V0 + P0V2 = 3P0V0 ...(iii)

æ dT
60 300 ö 300 40 For temperature to be maximum =0
= ç1- ÷
T = dv
100 è T 2 ø 2 100
Differentiating e.q. (iii) by 'v' we get
100 ´ 300
Þ T2 = Þ T2 = 750 K dT
40 nRV0 + P0(2v) = 3P0V0
dv
23. (b) Heat is extracted from the source in path
DA and AB is dT
\ nRV0 = 3P0V0 – 2 P0V
3 æ P V ö 5 æ 2P V ö dv
DQ = R ç 0 0 ÷ + R ç 0 0 ÷
2 è R ø 2 è R ø
dT 3P0 V0 - 2P0 V
3 5 æ 13ö = nRV0
=0
Þ P0V0 + 2 P0V0 = ç ÷ P0V0 dv
2 2 è 2ø
24. (d) In cyclic process, change in total internal 3V0 3P
V= \ p= 0 [From (i)]
energy is zero. 2 2
DUcyclic = 0
9Po Vo
5R \ Tmax = [From (iii)]
DUBC = nCv DT = 1 ´ DT 4nR
2
Where, Cv = molar specific heat at constant 27. (d) For a polytropic process
volume. R R
For BC, DT = –200 K C = Cv + \ C - Cv =
\ DUBC = –500R 1- n 1- n
25. (d) The entropy change of the body in the two R R
cases is same as entropy is a state function. \ 1- n = \ 1- =n
26. (c) The equation for the line is C - Cv C - Cv
P C - Cv - R C - C v - Cp + Cv
\ n= =
3Po C - Cv C - Cv

c C - Cp
q = (Q C p - C v = R )
2Po C - Cv
Po
q
Po
Vo
V
Vo 2Vo
EBD_7764
www.crackjee.xyz
P-68 Physics

Kinetic Theory
12
1. Cooking gas containers are kept in a lorry ( g - 1) Mv 2 K gM 2v
moving with uniform speed. The temperature of (a) (b) K
2 gR 2R
the gas molecules inside will [2002] ( g - 1) ( g - 1)
(a) increase (c) Mv 2 K (d) Mv 2 K
2R 2( g + 1) R
(b) decrease 7. Consider an ideal gas confined in an isolated
(c) remain same closed chamber. As the gas undergoes an
(d) decrease for some, while increase for others adiabatic expansion, the average time of collision
2. At what temperature is the r.m.s velocity of a between molecules increases as Vq, where V is
hydrogen molecule equal to that of an oxygen the volume of the gas. The value of q is :
molecule at 47°C? [2002]
(a) 80 K (b) –73 K æ Cp ö
(c) 3 K (d) 20 K. çg = ÷ [2015]
è Cv ø
3. A gaseous mixture consists of 16 g of helium
C g +1 g -1
and 16 g of oxygen. The ratio p of the mixture (a) (b)
Cv 2 2
is [2005] 3g + 5 3g - 5
(c) (d)
(a) 1.62 (b) 1.59 6 6
(c) 1.54 (d) 1.4 8. The temperature of an open room of volume 30
4. The speed of sound in oxygen (O2) at a certain m3 increases from 17°C to 27°C due to sunshine.
temperature is 460 ms–1. The speed of sound in The atmospheric pressure in the room remains
helium (He) at the same temperature will be 1 × 105 Pa. If ni and nf are the number of molecules
(assume both gases to be ideal) [2008] in the room before and after heating, then nf – ni
(a) 1421 ms–1 (b) 500 ms–1 will be : [2017]
(c) 650 ms–1 (d) 330 ms–1 25 25
5. One kg of a diatomic gas is at a pressure of (a) 2.5 ´ 10 (b) -2.5 ´ 10
8 × 104N/m2. The density of the gas is 4kg/m3. (c) -1.61 ´ 1023 (d) 1.38 ´ 1023
What is the energy of the gas due to its thermal
9. Cp and Cv are specific heats at constant pressure
motion? [2009]
and constant volume respectively. It is observed
(a) 5 × 104 J (b) 6 × 104 J that [2017]
(c) 7 × 104 J (d) 3 × 104 J Cp – Cv = a for hydrogen gas
6. A thermally insulated vessel contains an ideal
Cp – Cv = b for nitrogen gas
gas of molecular mass M and ratio of specific
heats g. It is moving with speed v and it's The correct relation between a and b is :
suddenly brought to rest. Assuming no heat is (a) a = 14 b (b) a = 28 b
lost to the surroundings, its temperature 1
increases by: [2011] (c) a= b (d) a=b
14
www.crackjee.xyz
Kinetic Theory P-69

Answer Key
1 2 3 4 5 6 7 8 9
(c) (d) (a) (a) (a) (c) (a) (b) (a)

1. (c) Since P and V are not changing, so vO2 g O2 M He


temperature remain same. \ = ´
vHe M O2 g He
8RT
2. (d) vrms =
pM 1.4 4
TH 2 TO2 = ´ = 0.3237
= 32 1.67
For vrms to be equal M M O2
H2 vO 2 460
Here M H 2 = 2; M O2 = 32 ; \ vHe = = = 1421 m / s
0.3237 0.3237
TO 2 = 47 + 273 = 320 K mass 1
5. (a) Volume = = m3
TH 2 density 4
320
\ = Þ TH 2 = 20 K
2 32 5 5 1
K.E = PV = ´ 8 ´ 104 ´ = 5 ´ 10 4 J
n1Cv1 + n2Cv2 2 2 4
3. (a) For mixture of gas, Cv = Alternatively:
n1 + n2
5 5m 5 m PM
K.E = nRT = RT = ´
3 1 5 5 2 2M 2M d
4 ´ R + ´ R 6R + R
2 2 2 = 4 [Q PM = dRT ]
=
æ 1ö 9 5 mP 5 1 ´ 8 ´ 104
çè 4 + ÷ø 2 = = ´ = 5 ´ 104 J
2 2 d 2 4
6. (c) Here, work done is zero.
29 R ´ 23 29 R
= = and So, loss in kinetic energy = change in
9´4 18 internal energy of gas
5R 1 7 R 1 2 R
4´ + ´ mv = nCv DT = n DT
n1C p1 + n2 C p2 2 2 2 2 g -1
Cp = =
(n1 + n2 ) æ 1ö 1 2 m R
mv = DT
çè 4 + ÷ø 2 M g -1
2
Mv 2 ( g - 1)
7 \ DT = K
10 R + R 2R
= 4 = 47 R
9 18 1
7. (a) t=
2 æ N ö 3RT
2pd2 ç ÷
èVø M
Cp 47 R 18
Þ
\ = ´ = 1.62 V
Cv 18 29 R tµ
T
4. (a) The speed of sound in a gas is given by
As, TVg–1 = K So, t µ Vg + 1/2
gRT
v= g +1
M Therefore, q =
2
EBD_7764
www.crackjee.xyz
P-70 Physics
8. (b) Given: Temperature Ti = 17 + 273 = 290 K 1 ´ 105 ´ 30 æ 1 1 ö
= ´ 6.023 ´ 10 23 ç - ÷
Temperature Tf = 27 + 273 = 300 K 8.314 è 300 290 ø
Atmospheric pressure, P0 = 1 × 105 Pa = – 2.5 × 1025
Volume of room, V0 = 30 m3
9. (a) As we know, Cp – Cv = R where Cp and Cv
Difference in number of molecules,
nf – ni = ? are molar specific heat capacities
R
Using ideal gas equation, PV = nRT(N0), or, Cp – Cv =
M
N0 = Avogadro's number
R
PV For hydrogen (M = 2) Cp – Cv = a =
Þ n= (N ) 2
RT 0
R
P0V0 æ 1 1 ö For nitrogen (M = 28) Cp – Cv = b =
- ÷N 28
\ nf – ni = ç
R è T f Ti ø 0 a
\ = 14 or, a = 14b
b
www.crackjee.xyz

Oscillations
13
1. In a simple harmonic oscillator, at the mean velocities, during oscillation, are equal, the ratio
position [2002] of amplitude of A and B is [2003]
(a) kinetic energy is minimum, potential energy
k1 k2
is maximum (a) (b)
(b) both kinetic and potential energies are k2 k1
maximum
(c) kinetic energy is maximum, potential energy k2 k1
(c) (d) k2
is minimum k1
(d) both kinetic and potential energies are 6. The length of a simple pendulum executing
minimum simple harmonic motion is increased by 21%.
2. If a spring has time period T, and is cut into n The percentage increase in the time period of
equal parts, then the time period of each part the pendulum of increased length is [2003]
will be [2002] (a) 11% (b) 21%
(a) T n (b) T / n (c) 42% (d) 10%
(c) nT (d) T 7. The displacement of a particle varies according
3. A child swinging on a swing in sitting position, to the relation x = 4(cos pt + sin pt). The
stands up, then the time period if the swing will amplitude of the particle is [2003]
[2002] (a) – 4 (b) 4
(a) increase (c) 4 2 (d) 8
(b) decrease 8. A body executes simple harmonic motion. The
(c) remains same potential energy (P.E), the kinetic energy (K.E)
(d) increases if the child is long and decreases and total energy (T.E) are measured as a function
if the child is short of displacement x. Which of the following
4. A mass M is suspended from a spring of negligible statements is true ? [2003]
mass. The spring is pulled a little and then released so (a) K.E. is maximum when x = 0
that the mass executes SHM of time period T. If the (b) T.E is zero when x = 0
(c) K.E is maximum when x is maximum
mass is increased by m, the time period
(d) P.E is maximum when x = 0
5T m 9. The bob of a simple pendulum executes simple
becomes .Then the ratio of is [2003]
3 M harmonic motion in water with a period t, while
3 25 the period of oscillation of the bob is t0 in air.
(a) (b) Neglecting frictional force of water and given that
5 9
16 5 the density of the bob is (4/3) × 1000 kg/m3.
(c) (d) Which relationship between t and t0 is true?
9 3
5. Two particles A and B of equal masses are [2004]
suspended from two massless springs of spring (a) t = 2t0 (b) t = t0/2
constants k1 and k2, respectively. If the maximum (c) t = t0 (d) t = 4t0
EBD_7764
www.crackjee.xyz
P-72 Physics
10. A particle at the end of a spring executes S.H.M (b) a periodic, but not simple harmonic motion
with a period t1. while the corresponding period 2p
with a period
for another spring is t2. If the period of oscillation w
with the two springs in series is T then [2004] p
(c) a simple harmonic motion with a period
(a) T -1 = t1-1 + t 2-1 (b) T 2 = t12 + t 22 w
(c) T = t1 + t2 (d) T -2 = t1-2 + t 2-2 (d) a simple harmonic motion with a period
11. The total energy of a particle, executing simple 2p
harmonic motion is [2004]
2 w
(a) independent of x (b) µ x 16. The bob of a simple pendulum is a spherical
1/ 2
(c) µ x (d) µ x hollow ball filled with water. A plugged hole near
where x is the displacement from the mean the bottom of the oscillating bob gets suddenly
position. unplugged. During observation, till water is
12. A particle of mass m is attached to a spring (of coming out, the time period of oscillation would
spring constant k) and has a natural angular [2005]
frequency w0. An external force F(t) proportional (a) first decrease and then increase to the
to cos wt (w ¹ w 0 ) is applied to the oscillator.. original value
(b) first increase and then decrease to the
The displacement of the oscillator will be original value
proportional to [2004] (c) increase towards a saturation value
1 1
(d) remain unchanged
(a) 2 2 (b) 2 2
m (w 0 + w ) m (w 0 - w )
m m 17. If a simple harmonic motion is represented by
(c) 2 2 (d) 2 2 2
w -w (w + w ) d x
0 0 + ax = 0 , its time period is [2005]
13. In forced oscillation of a particle the amplitude dt 2
is maximum for a frequency w1 of the force while 2p 2p
(a) (b)
the energy is maximum for a frequency w2 of a a
the force; then [2004] (c) 2p a (d) 2pa
(a) w1 < w2 when damping is small and 18. The maximum velocity of a particle, executing
simple harmonic motion with an amplitude 7 mm,
w1 > w2 when damping is large
is 4.4 m/s. The period of oscillation is [2006]
(b) w1 > w2 (a) 0.01 s (b) 10 s
(c) w1 = w2 (c) 0.1 s (d) 100 s
(d) w1 < w 2 19. Starting from the origin a body oscillates simple
14. Two simple harmonic motions are represented harmonically with a period of 2 s. After what
time will its kinetic energy be 75% of the total
by the equations y1 = 0.1 sin æç100pt + p ö÷ and energy? [2006]
è 3ø
1 1
y 2 = 0.1 cos pt . The phase difference of the (a) s (b) s
6 4
velocity of particle 1 with respect to the velocity
of particle 2 is [2005] 1 1
(c) s (d) s
p -p 3 12
(a) (b)
3 6 20. Two springs, of force constants k1 and k2 are
p -p connected to a mass m as shown. The frequency
(c) (d) of oscillation of the mass is f. If both k1 and k2
6 3
2 are made four times their original values, the
15. The function sin (wt ) represents [2005] frequency of oscillation becomes [2007]
(a) a periodic, but not simple harmonic motion
p
with a period
w
www.crackjee.xyz
Oscillations P-73
m æ A1 ö
k1 k2
of çè A ÷ø is: [2011]
2
1
(a) 2 f (b) f /2 M +m æ M ö2
(c) f /4 (d) 4 f (a) (b) çè ÷
M M + mø
21. A particle of mass m executes simple harmonic 1
motion with amplitude a and frequency n. The æ M + mö 2 M
(c) çè ÷ (d)
average kinetic energy during its motion from M ø M +m
the position of equilibrium to the end is[2007] 27. A wooden cube (density of wood ‘d’) of side ‘ l ’
(a) 2p 2 ma 2 n2 (b) p 2 ma 2 n2 floats in a liquid of density ‘r’ with its upper and
1 lower surfaces horizontal. If the cube is pushed
2 2
(c) ma n (d) 4p 2 ma 2 n2 slightly down and released, it performs simple
4 harmonic motion of period ‘T’ [2011 RS]
22. The displacement of an object attached to a
spring and executing simple harmonic motion is ld lr
given by x = 2 × 10–2 cos pt metre.The time at (a) 2p (b) 2p
rg dg
which the maximum speed first occurs is[2007]
(a) 0.25 s (b) 0.5 s ld lr
(c) 0.75 s (d) 0.125 s (c) 2p (d) 2p
23. A point mass oscillates along the x-axis (r - d ) g (r - d ) g
according to the law x = x0 cos(wt - p / 4) . If the 28. If a simple pendulum has significant amplitude (up
to a factor of 1/e of original) only in the period
acceleration of the particle is written as between t = 0s to t = t s, then t may be called the
a = A cos(wt + d ) ,then [2007] average life of the pendulum. When the spherical
2 bob of the pendulum suffers a retardation (due to
(a) A = x0 w , d = 3p / 4 viscous drag) proportional to its velocity with b as
(b) A = x0, d = -p / 4 the constant of proportionality, the average life
time of the pendulum in second is (assuming
(c) A = x0 w 2 , d = p / 4 damping is small) [2012]
2 0.693
(d) A = x0 w , d = -p / 4 (a) (b) b
b
24. If x, v and a denote the displacement, the velocity
and the acceleration of a particle executing simple 1 2
(c) (d)
harmonic motion of time period T, then, which of b b
the following does not change with time?[2009] 29. This question has Statement 1 and Statement 2. Of
(a) aT/x (b) aT + 2pv the four choices given after the Statements, choose
(c) aT/v (d) a2T2 + 4p2v2 the one that best describes the two Statements.
25. Two particles are executing simple harmonic If two springs S1 and S2 of force constants k1
motion of the same amplitude A and frequency and k2 respectively, are stretched by the same
w along the x-axis. Their mean position is force, it is found that more work is done on
separated by distance X0(X0 > A). If the maximum spring S1 than on spring S2.
separation between them is (X0 + A), the phase Statement 1 : If stretched by the same amount
difference between their motion is: [2011] work done on S1
p p Statement 2 : k1 < k2 [2012]
(a) (b) (a) Statement 1 is false, Statement 2 is true.
3 4
p p (b) Statement 1 is true, Statement 2 is false.
(c) (d) (c) Statement 1 is true, Statement 2 is true,
6 2
26. A mass M, attached to a horizontal spring, Statement 2 is the correct explanation for
executes S.H.M. with amplitude A1. When the Statement 1
mass M passes through its mean position then (d) Statement 1 is true, Statement 2 is true,
a smaller mass m is placed over it and both of Statement 2 is not the correct explanation
them move together with amplitude A2. The ratio for Statement 1
EBD_7764
www.crackjee.xyz
P-74 Physics
30. The amplitude of a damped oscillator decreases of the following represents these correctly?
to 0.9 times its original magnitude in 5s. In (graphs are schematic and not drawn to scale)
another 10s it will decrease to a times its original [2015]
magnitude, where a equals [2013]
(a) 0.7 (b) 0.81 E KE
(c) 0.729 (d) 0.6
31. An ideal gas enclosed in a vertical cylindrical
container supports a freely moving piston of (a) d
mass M. The piston and the cylinder have equal
cross sectional area A. When the piston is in
equilibrium, the volume of the gas is V0 and its
pressure is P0. The piston is slightly displaced PE
from the equilibrium position and released.
Assuming that the system is completely isolated
E
from its surrounding, the piston executes a PE
simple harmonic motion with frequency [2013]
1 AgP0 1 V0 MP0 (b) KE
(a) (b)
2p V0 M 2p A 2 g

1 A 2 gP0 1 MV0 E
(c) (d)
2p MV0 2p AgP0 KE
32. A particle moves with simple harmonic motion
in a straight line. In first t s, after starting from PE
(c)
rest it travels a distance a, and in next t s it d
travels 2a, in same direction, then: [2014]
(a) amplitude of motion is 3a E PE
(b) time period of oscillations is 8t
(c) amplitude of motion is 4a
(d) time period of oscillations is 6t KE
(d) d
33. A pendulum made of a uniform wire of cross
sectional area A has time period T. When an
additional mass M is added to its bob, the time 35. A particle performs simple harmonic mition with
period changes to TM. If the Young's modulus amplitude A. Its speed is trebled at the instant
1 2A
of the material of the wire is Y then is equal that it is at a distance from equilibrium
Y 3
to : position. The new amplitude of the motion is :
(g = gravitational acceleration) [2015] [2016]
é æ T ö2 ù A é æ T ö ù A
2
7A
(a) ê1 - ç M ÷ ú (b) ê1 - ç T ÷ ú Mg (a) A 3 (b)
3
ë è T ø û Mg êë è M ø úû
A
éæ T ö2 ù A éæ T ö2 ù Mg (c) 41 (d) 3A
M
(c) êç ÷ - 1ú (d) êç M ÷ - 1ú 3
ëè T ø û Mg ëè T ø û A 36. A particle is executing simple harmonic motion
34. For a simple pendulum, a graph is plotted with a time period T. At time t = 0, it is at its
between its kinetic energy (KE) and potential position of equilibrium. The kinetic energy-time
energy (PE) against its displacement d. Which one graph of the particle will look like: [2017]
www.crackjee.xyz
Oscillations P-75

(c)
(a)

(d)
(b)

Answer Key
1 2 3 4 5 6 7 8 9 10 11 12 13 14 15
(c) (b) (b) (c) (c) (d) (c) (a) (a) (b) (a) (b) (c) (b) (c)
16 17 18 19 20 21 22 23 24 25 26 27 28 29 30
(b) (a) (a) (a) (a) (b) (b) (a) (a) (a) (c) (a) (d) (b) (c)
31 32 33 34 35 36
(c) (d) (c) (d) (b) (b)

1. (c) The kinetic energy (K. E.) and potential


energy (U) of a simple harmonic oscillator point of suspension
is given by,
1 2 2 1 2 l'
K.E = k ( A - x ) ; U = kx l
2 2
Where A = amplitude and k = mw2
x = displacement from the mean position
At the mean position x = 0
1 2
\ K.E. = kA = Maximum and U = 0 Case (ii) child standing Case (i) child sitting
2
2. (b) Let the spring constant of the original M
4. (c) T = 2p
m k
spring be k. Then its time period T = 2p
k M + m 5T
where m is the mass of oscillating body. T ' = 2p =
When the spring is cut into n equal parts, k 3
the spring constant of one part becomes M +m 5 M
nk. Therefore the new time period, \ 2p = ´ 2p
k 3 k
m T
T ' = 2p = 25
nk n M +m = ´M
9
l
3. (b) The time period T = 2p where l = m 25 m 25 16
g 1+ = Þ = -1 =
M 9 M 9 9
distance between the point of suspension 5. (c) Maximum velocity during SHM = Aw
and the centre of mass of the child. But k = mw2
As shown in the figure, l ¢ < l k
\ w=
\ T ¢ < T i.e., the period decreases. m
EBD_7764
www.crackjee.xyz
P-76 Physics
k l
\ Maximum velocity = A \ t = 2p
m g/4
Here the maximum velocity is same and m t = 2t0
is also same
m
A1 k2 10. (b) For first spring, t1 = 2p ,
\ A1 k1 = A2 k 2 \ = k1
A2 k1
m
l 1.21l For second spring, t 2 = 2p
6. (d) T = 2p and T ' = 2p k2
g g when springs are in series then,
(Q l ' = l + 21% of l) k1k 2
keff =
T '- T kl + k 2
% increase = ´ 100
T
m ( kl + k 2 )
1.21l - l \ T = 2p
=
l
´ 100 = ( )
1.21 - 1 ´ 100 k1k 2
2 2
= (1.1 - 1) ´ 100 = 10% \ T = 2p
m m
+ = 2p
t2
+ 1
t
k 2 k1 2 2
7. (c) x = 4(cos pt + sin pt ) (2p) (2p)
æ sin pt cos pt ö 2 2 2
= 2 ´ 4ç + Þ T = t1 + t 2
è 2 ÷
2 ø where x is the displacement from the mean
position
= 4 2(sin p t cos 45° + cos p t sin 45°) 11. (a) At any instant the total energy is
x = 4 2 sin(p t + 45°) 1 2
kA = constant, where A0 = amplitude
on comparing it with x = A sin(wt + f) 2 0
hence total energy is independent of x.
we get A = 4 2 12. (b) Equation of displacement is given by
1 x = A sin(wt + f)
8. (a) K.E. = mw 2 (a 2 - x 2 )
2 F0
When x = 0, K.E is maximum and is equal to where A =
2 2 2
1 m (w 0 - w )
mw 2 a 2 .
2 F0
=
l l m (w 0 2 - w 2 )
9. (a) t = 2p ; t 0 = 2p
g eff g Here damping effect is considered to be
Buoyant zero
force 1000 Vg 1
\x µ
2 2
m (w 0 - w )
4 13. (c) Since energy µ ( Amplitude) 2 , the
´ 1000 Vg maximum for both of them occurs at the
3
same frequency
Weight
\ w1 = w2
æ4 ö 1000
Net force = çè - 1÷ø ´ 1000 Vg = Vg dy1 pö
3 3 æ
14. (b) v1 = = 0.1 ´ 100p cos ç100pt + ÷
1000 Vg g dt è 3ø
g eff = =
4 4 dy2 æ pö
3 ´ ´ 1000 V v2 = = - 0.1p sin pt = 0.1p cos ç pt + ÷
3 dt è 2ø
www.crackjee.xyz
Oscillations P-77

p p 2 p - 3p 19. (a) K.E. of a body undergoing SHM is given by,


\ Phase diff. = f1 - f 2 = - =
3 2 6 1 2 2 2
K .E. = ma w cos wt
p 2
=–
6 1 2 2
15. (c) Clearly sin2 wt is a periodic function as sin T .E. = ma w
2
p Given K.E. = 0.75 T.E.
wt is periodic with period
w 2 p
Þ 0.75 = cos wt Þ wt =
6
p p´2 1
Þt= Þt= Þt= s
6´w 6 ´ 2p 6
0 p/w 2p/w 3p/w 20. (a) The two springs are in parallel.
2 \ Effective spring constant,
d y
For SHM 2
µ -y k = k1 + k2
dt Now, frequency of oscillation is given by
dy 1 k
= 2w sin wt cos wt = w sin 2wt f =
dt 2p m
2
d y 2
= 2w cos 2wt which is not 1 k1 + k 2
dt 2 or, f = ....(i)
proportional to –y. Hence, it is not in SHM. 2p m
16. (b) Centre of mass of combination of liquid and When both k1 and k2 are made four times
hollow portion (at position l ), first goes their original values, the new frequency is
given by
down ( to l + D l) and when total water is
drained out, centre of mass regain its 1 4 k1 + 4k 2
original position (to l ), f '=
2p m
l
T = 2p 1 4(k1 + 4k 2 ) æ 1 k1 + k 2 ö
g = = 2ç ÷
\ ‘T ’ first increases and then decreases 2p m è 2p m ø
to original value. 21. (b) The kinetic energy of a particle executing
S.H.M. is given by
1
K = ma2 w2 sin2wt
2
where, m = mass of particle
a = amplitude
c w = angular frequency
t = time
2 1
d x 2
Now, average K.E. = < K > = < mw2 a2
17. (a) = -ax = -w x 2
dt 2 sin2 wt >
2p 2p 1
Þ w = a or T = = = mw2a2 <sin2 wt >
w a 2
18. (a) Maximum velocity, 1 æ 1ö æ 2 1ö
= mw2a2 çè ÷ø çè Q < sin q > = ÷ø
vmax = a w 2 2 2
2p 1 2 2 1
vmax = a ´ = mw a = ma2 (2pn) 2 (Q w = 2pn)
T 4 4
-3
2pa 2 ´ 3.14 ´ 7 ´ 10 or, < K > = p ma n
2 2 2
ÞT = = » 0.01 s
vmax 4.4
EBD_7764
www.crackjee.xyz
P-78 Physics
22. (b) Here, x = 2 × 10–2 cos p t 27. (a)
Speed is given by
dx
v= = 2 × 10–2 p sin p t
dt d
For the first time, the speed to be maximum, l
p
sin p t = 1 or, sin p t = sin
2
p 1
Þ pt = or,, t = = 0.5 sec.
2 2 l0 r
23. (a) Here,
x = x0 cos (wt – p / 4 )
dx æ pö
\ Velocity, v = = - x0 w sin ç wt - ÷
dt è 4ø At equilibrium
Acceleration, Fb = mg
dv 2 æ pö
a= = - x0w cos ç wt - ÷ rAl 0 g = dAlg ....(i)
dt è 4ø
2 é æ p ö ù
= x0w cos ê p + ç wt - ÷ ú
ë è 4øû
æ 3 p ö
= x0 w 2 cos ç wt + ÷ ...(1) d
è 4ø
Acceleration, a = A cos (wt + d) ...(2)
Comparing the two equations, we get
3p
A = x0w2 and d = .
4
24. (a) For an SHM, the acceleration a = -w2 x l0 + x
2 a
where w is a constant. Therefore, is a
x
constant. The time period T is also
aT Restoring force,
constant. Therefore, is a constant. F = mg – Fb'
x2
25. (a) Let, x1 = A sin w t and x = A sin (w t + f) F = mg - rA ( l 0 + x ) g
æ fö f
x2 – x1 = 2A cos çè wt + 2 ÷ø sin 2 dAla = dAlg - rAl 0 g - rgAx
rg
As the maximum separation between the a=- x
particles is A, dl
f p Therefore, wooden cube performs S.H.M.
\ 2 A sin = A Þ f=
2 3 rg ld
26. (c) The net force becomes zero atthe mean \ w= Þ T = 2p
dl rg
point.
Therefore, linear momentum must be 28. (d) The equation of motion for the pendulum,
conserved. suffering retardation
\ Mv1 = (M + m)v2 F = – kx - bv
k k 2
MA1
M
= ( M + m) A2
m+M Þ m d x + kx + b dk = 0
dt 2 dt
æ k ö 2
\ çV = A M ÷ d x k
+ x+
b dx
=0
è ø Þ
dt 2 m m dt
Q A1 M = A2 M + m
d2 x b dx k
A1 m+ M Þ 2
+ + x =0 … (1)
\ A = dt m dt m
2 M
www.crackjee.xyz
Oscillations P-79
lt
Let x = e is the solution of the equation (1) P0 x0
g

dx d x 2 P=
= le l t Þ = l 2 e lt ( x0 - x )g
dt 2
dt
Substituting in the equation (1) Piston
b k
l 2 e lt + l e lt + e l t = 0
m m
x
b k
l2 + l + = 0
m m x0 Cylinder
containing
b b2 k ideal gas
-± -4
m m 2 m -b ± b 2 - 4km Let piston is displaced by distance x
l= =
2 2m æ P xg ö
Solving the equation (1) for x, we have 0 0
Mg - ç ÷ A = Frestoring
-b
çè ( x - x )g ÷ø
t 0
x = e 2m
æ x0g ö
2 k +b
2 P0 A ç1 - ÷ = Frestoring
w = w0 - l where w0 = , l = çè ( x - x )g ÷ø
m 2 0
1 2 [ x0 - x » x0 ]
The average life = =
l b
gP0 Ax
1 2 F=-
29. (b) Q w = kx x0
2 \ Frequency with which piston executes
1 1 2 SHM.
w1 = k1x 2 ; w2 = k2 x
2 2
Since w1 > w2 Thus (k1 > k2) 1 gP0 A 1 gP0 A2
f =
bt 2p x0 M = 2p MV0
-
2m 32. (d) In simple harmonic motion, starting from
30. (c) Q A = A 0e
rest,
(where, A0 = maximum amplitude) At t = 0 , x = A
According to the questions, after 5 x = Acoswt ..... (i)
second, When t = t , x = A – a
-
b(5) When t = 2 t , x = A –3a
0.9A 0 = A 0e … (i) 2m From equation (i)
After 10 more second, A – a = Acosw t ......(ii)
A – 3a = A cos2w t ....(iii)
b(15)
-
…(ii) As cos2w t = 2 cos2 w t – 1...(iv)
A = A0 e 2m From equation (ii), (iii) and (iv)
From eqns (i) and (ii) 2
A = 0.729 A0 \ a = 0.729 A - 3a æ A-a ö
= 2ç ÷ -1
Mg A è A ø
31. (c) = P0
A A - 3a 2 A2 + 2a 2 - 4 Aa - A2
Þ =
P0V0 g = PV g A A2
Mg = P0A … (1) 2 2 2
Þ A – 3aA = A + 2a – 4Aa
g g Þ 2a2 = aA
P0 Ax0 = PA( x0 - x) Þ A = 2a
EBD_7764
www.crackjee.xyz
P-80 Physics
a 1 2
Þ = 2 æ 2A ö
A 2 Initially V= w A -ç ÷
Now, A – a = A coswt è 3 ø
A- a 2
Þ cos wt = 2 æ 2A ö
A Finally 3v = w A' - ç ÷
è 3 ø
1 2p p Where A'= final amplitude (Given at x =
Þ cos wt = or t=
2 T 3 2A
Þ T= 6t , velocity to trebled)
3
l 2
33. (c) As we know, time period, T = 2p æ 2A ö
g A' 2 - ç ÷
3 è 3 ø
When additional mass M is added then On dividing we get =
1 æ 2A ö
2
l + Dl A2 - ç ÷
TM = 2p è 3 ø
g

l + Dl 2 é 2 4A 2 ù 4A 2
TM = æ TM ö l + Dl 9 ê A - 9 ú = A'2 –
l or ç ÷ =
T
è T ø l ëê ûú 9
2 7A
æ TM ö Mg \ A' =
or, ç =1+ 3
è T ÷ø Ay 36. (b) For a particle executing SHM
é Mgl ù At mean position; t = 0, wt = 0, y = 0, V = Vmax =
êQ Dl = Ay ú aw
ë û
1
\ K.E. = KEmax = mw2a2
1 éæ T ö
2 ù
A 2
\ = êç M ÷ - 1ú
y ëêè T ø Mg T p
ûú At extreme position : t = , wt = , y = A, V =
4 2
1 Vmin = 0
34. (d) K.E = k ( A2 - d 2 )
2 \ K.E. = KEmin = 0
1
1 2 Kinetic energy in SHM, KE = mw2(a2 – y2)
and P.E. = kd 2
2
At mean position d = 0. At extrement 1
= mw2a2cos2wt
positions d = A 2
Hence graph (2) correctly depicts kinetic energy
2 2
35. (b) We know that V = w A - x time graph.
www.crackjee.xyz
Waves P-81

waves 14
1. Length of a string tied to two rigid supports is æ pö
40 cm. Maximum length (wavelength in cm) of a y = 10 - 4 sinç 600t - 2x + ÷ metres
è 3ø
stationary wave produced on it is [2002]
where x is expressed in metres and t in seconds.
(a) 20 (b) 80
The speed of the wave - motion, in ms–1 , is
(c) 40 (d) 120
[2003]
2. Tube A has both ends open while tube B has
(a) 300 (b) 600
one end closed, otherwise they are identical. The
(c) 1200 (d) 200
ratio of fundamental frequency of tube A and B
7. A metal wire of linear mass density of 9.8 g/m is
is [2002]
stretched with a tension of 10 kg-wt between
(a) 1 : 2 (b) 1 : 4
two rigid supports 1 metre apart. The wire passes
(c) 2 : 1 (d) 4 : 1.
at its middle point between the poles of a
3. A tuning fork arrangement (pair) produces 4
permanent magnet, and it vibrates in resonance
beats/sec with one fork of frequency 288 cps. A
when carrying an alternating current of
little wax is placed on the unknown fork and it
frequency n. The frequency n of the alternating
then produces 2 beats/sec. The frequency of
source is [2003]
the unknown fork is [2002]
(a) 50 Hz (b) 100 Hz
(a) 286 cps (b) 292 cps
(c) 200Hz (d) 25Hz
(c) 294 cps (d) 288 cps
8. A tuning fork of known frequency 256 Hz makes
4. A wave y = a sin(wt–kx) on a string meets with
5 beats per second with the vibrating string of a
another wave producing a node at x = 0. Then
piano. The beat frequency decreases to 2 beats
the equation of the unknown wave is [2002]
per second when the tension in the piano string
(a) y = a sin( w t + kx) is slightly increased. The frequency of the piano
(b) y = –a sin( w t + kx) string before increasing the tension was [2003]
(c) y = a sin( w t – kx) (a) (256 + 2) Hz (b) (256 – 2) Hz
(d) y = –a sin( w t – kx) (c) (256 – 5) Hz (d) (256 + 5) Hz
5. When temperature increases, the frequency of 9. The displacement y of a particle in a medium can
a tuning fork [2002] be expressed as,
(a) increases pö
æ
(b) decreases y = 10-6 sin ç100t + 20 x + ÷ m where t is in
è 4ø
(c) remains same
second and x in meter. The speed of the wave is
(d) increases or decreases depending on the [2004]
material (a) 20 m/s (b) 5 m/s
6. The displacement y of a wave travelling in the (c) 2000 m/s (d) 5p m/s
x -direction is given by
EBD_7764
www.crackjee.xyz
P-82 Physics
10. When two tuning forks ar e sounded measures the column length to be x cm for the
simultaneously, 4 beats per second are heard. second resonance. Then [2008]
Now, some tape is attached on the prong of the (a) 18 > x (b) x > 54
fork 2. When the tuning forks are sounded again, (c) 54 > x > 36 (d) 36 > x > 18
6 beats per second are heard. If the frequency of 16. A wave travelling along the x-axis is described
fork 1 is 200 Hz, then what was the original by the equation y(x, t) = 0.005 cos (a x – bt). If
frequency of fork 2? [2005] the wavelength and the time period of the wave
are 0.08 m and 2.0s, respectively, then a and b in
(a) 202 Hz (b) 200 Hz
appropriate units are [2008]
(c) 204 Hz (d) 196 Hz
(a) a = 25.00 p , b = p
11. An observer moves towards a stationary source
0.08 2.0
of sound, with a velocity one-fifth of the velocity (b) a = ,b =
p p
of sound. What is the percentage increase in
0.04 1.0
the apparent frequency ? [2005] (c) a = ,b =
p p
(a) 0.5% (b) zero p
(c) 20 % (d) 5 % (d) a = 12.50p, b =
2.0
12. A whistle producing sound waves of 17. Three sound waves of equal amplitudes have
frequencies 9500 HZ and above is approaching frequencies (n –1), n, (n + 1). They superpose to
a stationary person with speed v ms–1 . The give beats. The number of beats produced per
velocity of sound in air is 300 ms–1. If the person second will be : [2009]
can hear frequencies upto a maximum of 10,000 (a) 3 (b) 2
HZ, the maximum value of v upto which he can (c) 1 (d) 4
hear whistle is [2006] 18. A motor cycle starts from rest and accelerates
15 along a straight path at 2m/s2. At the starting
(a) 15 2 ms -1 (b) ms -1 point of the motor cycle there is a stationary
2
(c) 15 ms -1 (d) 30 ms-1 electric siren. How far has the motor cycle gone
when the driver hears the frequency of the siren
13. A string is stretched between fixed points
at 94% of its value when the motor cycle was at
separated by 75.0 cm. It is observed to have
rest? (Speed of sound = 330 ms–1) [2009]
resonant frequencies of 420 Hz and 315 Hz. There
(a) 98 m (b) 147 m
are no other resonant frequencies between these
(c) 196 m (d) 49 m
two. Then, the lowest resonant frequency for
19. The equation of a wave on a string of linear
this string is [2006] mass density 0.04 kg m–1 is given by
(a) 105 Hz (b) 1.05 Hz
é æ t x öù
(c) 1050 Hz (d) 10.5 Hz y = 0.02(m) sin ê2p ç - ÷ú .
14. A sound absorber attenuates the sound level ë è 0.04(s ) 0.50(m) ø û
The tension in the string is [2010]
by 20 dB. The intensity decreases by a factor of
(a) 4.0 N (b) 12.5 N
[2007]
(c) 0.5 N (d) 6.25 N
(a) 100 (b) 1000 20. The transverse displacement y (x, t) of a wave
(c) 10000 (d) 10
15. While measuring the speed of sound by on a string is given by y( x, t ) = e (
- ax 2 + bt 2 + 2 ab ) xt ).
performing a resonance column experiment, a This represents a: [2011]
student gets the first resonance condition at a (a) wave moving in – x direction with speed
column length of 18 cm during winter. Repeating b
the same experiment during summer, she a
www.crackjee.xyz
Waves P-83

(b) standing wave of frequency b 24. A sonometer wire of length 1.5 m is made of
steel. The tension in it produces an elastic strain
1
(c) standing wave of frequency of 1%. What is the fundamental frequency of
b
steel if density and elasticity of steel are 7.7 ×
a
(d) wave moving in + x direction speed 103 kg/m3 and 2.2 × 1011 N/m2 respectively?
b
[2013]
21. A travelling wave represented by
(a) 188.5 Hz (b) 178.2 Hz
y = A sin (wt – kx) is superimposed on another
(c) 200.5 Hz (d) 770 Hz
wave represented by y = A sin (wt + kx). The
25. A pipe of length 85 cm is closed from one end.
resultant is [2011 RS]
Find the number of possible natural oscillations
(a) A wave travelling along + x direction
of air column in the pipe whose frequencies lie
(b) A wave travelling along – x direction
below 1250 Hz. The velocity of sound in air is
(c) A standing wave having nodes at
340 m/s. [2014]
nl
x= , n = 0,1, 2.... (a) 12 (b) 8
2
(d) A standing wave having nodes at (c) 6 (d) 4
æ 1ö l 26. A train is moving on a straight track with speed
x = ç n + ÷ ; n = 0,1, 2.... 20 ms–1. It is blowing its whistle at the frequency
è 2ø 2 of 1000 Hz. The percentage change in the
22. Statement - 1 : Two longitudinal waves given frequency heard by a person standing near the
by equations : y1 ( x, t ) = 2a sin(wt - kx ) and track as the train passes him is (speed of sound
y2 ( x, t ) = a sin (2wt - 2kx) will have equal = 320 ms–1) close to : [2015]
intensity. [2011 RS] (a) 18% (b) 24%
Statement - 2 : Intensity of waves of given (c) 6% (d) 12%
frequency in same medium is proportional to 27. A pipe open at both ends has a fundamental
square of amplitude only. frequency f in air. The pipe is dipped vertically in
(a) Statement-1 is true, statement-2 is false. water so that half of it is in water. The fundamental
(b) Statement-1 is true, statement-2 is true, frequency of the air column is now : [2016]
statement-2 is the correct explanation of (a) 2f (b) f
statement-1
f 3f
(c) Statement-1 is true, statement-2 is true, (c) (d)
2 4
statement-2 is not the correct explanation
of statement-1 28. A uniform string of length 20 m is suspended
from a rigid support. A short wave pulse is
(d) Statement-1 is false, statement-2 is true.
introduced at its lowest end. It starts moving up
23. A cylindrical tube, open at both ends, has a
the string. The time taken to reach the supports
fundamental frequency f in air. The tube is
is : [2016]
dipped vertically in water so that half of it is in –2
(take g = 10 ms )
water. The fundamental frequency of the air-
column is now : [2012] (a) 2 2s (b) 2s
(a) f (b) f / 2
(c) 2p 2 s (d) 2 s
(c) 3 f /4 (d) 2 f

Answer Key
1 2 3 4 5 6 7 8 9 10 11 12 13 14 15
(b) (c) (b) (b) (b) (a) (a) (c) (b) (d) (c) (c) (a) (a) (b)
16 17 18 19 20 21 22 23 24 25 26 27 28
(a) (b) (a) (d) (a) (d) (a) (a) (b) (c) (d) (b) (a)
EBD_7764
www.crackjee.xyz
P-84 Physics

1. (b) This will happen for fundamental mode of and the frequency of known fork. This
vibration as shown in the figure. would produce high beat frequency.
l 4. (b) To form a node there should be
Here, = 40 \ l = 80 cm superposition of this wave with the
2
reflected wave. The reflected wave should
S1 S2 travel in opposite direction with a phase
change of p. The equation of the reflected
40 cm wave will be
y = a sin (wt + kx + p)
S1 and S2 are rigid support
2. (c) The fundamental frequency for closed Þ y = – a sin (wt + kx)
organ pipe is given by 5. (b) The frequency of a tuning fork is given by
the expression
v é lù
uc =
4l êëQl = 4 úû m2 k Y
f=
Where l = length of the tube and v is the 4 3 pl 2 r
velocity of sound in air.
As temperature increases, l increases and
l = l /4
therefore f decreases.
æ pö
The fundamental frequency for open organ 6. (a) y = 10-4 sin ç 600t - 2x + ÷
pipe is given by è 3ø
v é lù But y = A sin ( wt - kx + f)
u0 =
2l
êëQl = 2 úû
On comparing we get w = 600; k=2
l Also velocity of wave is given by
l=
2
w 600
v= = = 300 ms -1
k 2
u0 v 4l 2 7. (a) For a string vibrating between two rigid
\ u = 2l ´ v = 1 support, the fundamental frequency is
c
given by
3. (b) A tuning fork produces 4 beats/sec with
another tuning fork of frequency 288 cps. v 1 T
From this information we can conclude that n= =
2l 2l m
the frequency of unknown fork is 288 + 4
cps or 288 – 4 cps i.e. 292 cps or 284 cps.
When a little wax is placed on the unknown
fork, it produces 2 beats/sec. When a little l
wax is placed on the unknown fork, its l=
frequency decreases and simultaneously 2
the beat frequency decreases confirming 1 10 ´ 9.8
that the frequency of the unknown fork is = = 50 Hz
292 cps. 2 ´ 1 9.8 ´ 10-3

NOTE Had the frequency of unknown As the string is vibrating in resonance to


a.c of frequency n, therefore both the
fork been 284 cps, then on placing wax its
frequencies are same.
frequency would have decreased thereby
increasing the gap between its frequency
www.crackjee.xyz
Waves P-85
8. (c) A tuning fork of frequency 256 Hz makes 5 n=1
beats/second with the vibrating string of a Therefore lowest resonant frequency
piano. Therefore, the frequency of the = 105 Hz.
vibrating string of piano is (256 ± 5) Hz. i.e.,
either 261Hz or 251 Hz. When the tension æI ö
in the piano string increases, its frequency 14. (a) We have, L1 = 10 log ç 1 ÷ ;
è I0 ø
will increases. Now since the beat
frequency decreases, we can conclude that æI ö
the frequency of piano string is 251Hz L2 = 10 log ç 2 ÷
è I0 ø
9. (b) From equation given,
æI ö æI ö
w = 100 and k = 20 \ L1 – L2 = 10 log ç 1 ÷ - 10log ç 2 ÷
w 100 è I0 ø è I0 ø
v= = = 5m / s
k 20 æI I ö
10. (d) Frequency of fork 1 = 200 Hz = n 0 or, DL = 10 log ç 1 ´ 0 ÷
No. of beats heard when fork 2 is è I0 I 2 ø
sounded with fork 1
æI ö
= Dn = 4 or, DL = 10log ç 1 ÷
è I2 ø
Now we know that if on loading (attach-
ing tape) an unknown fork, the beat
frequency increases (from 4 to 6 in this æI ö æI ö
or, 20 = 10 log ç 1 ÷ or, 2 = log ç 1 ÷
case) then the frequency of the unknown è I2 ø è I2 ø
fork 2 is given by,
n = n0 – Dn = 200 – 4 = 196 Hz I1 I
or, = 102 or, I2 = 1 .
I2 100
é vù
é v + v0 ù êv + 5 ú é6ù Þ Intensity decreases by a factor 100.
11. (c) n ' = n ê ú = nê ú = nê ú
ë v û ê v ú ë5û 15. (b) For first resonant length
êë úû
n' 6 n' - n 6 - 5 v v
= ; = ´ 100 = 20% n= = (in winter)
n 5 n 5 4l1 4 ´ 18
é v ù For second resonant length
12. (c) n' = n ê ú
ë v - vs û 3v ' 3v '
n' = = (in summer)
é 300 ù 4l 2 4x
Þ 10000 = 9500 ê ú
ë 300 - v û v 3v'
\ =
Þ 300 - v = 300 ´ 0.95 4 ×18 4× x
v'
Þ v = 300 - 285 = 15 ms -1 \ x = 3 ´ 18 ´
v
v v'
13. (a) Given nv = 315 and (n + 1) = 420 \ x = 54 ´ cm
2l 2l v
v' > v because velocity of light is greater in
n + 1 420
Þ = Þn =3 summer as compared to winter (v µ T )
n 315
\ x > 54cm
v v
Hence 3 ´ = 315 Þ = 105 Hz 16 (a) y(x, t) = 0.005 cos (ax - bt) (Given)
2l 2l
The lowest resonant frequency is when Comparing it with the standard equation
of wave
EBD_7764
www.crackjee.xyz
P-86 Physics
y(x, t) = a cos (kx - wt) we get Velocity on a string is given by
k = a and w = b T
v=
2p 2p m
But k = and w =
l T
\ T = v2 ´ m = (12.5)2 × 0.04 = 6.25 N
2p 2p
Þ = a and =b 20. (a) Given wave equation is y(x,t)
l T
Given that l = 0.08 m and T = 2.0s = e(- ax +bt 2 + 2 ab xt )
2

2p 2p
\ a= = 25p and b = =p -[( ax )2 + ( b t ) 2 + 2 a x . b t ]
0.08 2 =e
17. (b) Maximum number of beats -( ax + bt ) 2
=e
= ( n + 1) – ( n – 1) = 2
2
æ ö
2 b ÷
u=0 a = 2m/s vm - çè x + tø
18. (a) = e a
Electric s Motor It is a function of type y = f (x + vt)
siren cycle
b
Þ Speed of wave =
v 2m - u 2 = 2as a
21. (d) y = A sin (wt – kx) + A sin (wt + kx)
\ v2m = 2 ´ 2 ´ s y = 2A sin wt cos kx
For standing wave nodes
\ vm = 2 s
cos kx = 0
According to Doppler’s effect
2p p
é v - vm ù .x = (2n + 1)
n ' =n ê ú l 2
ë v û

\ x=
( 2n + 1) l , n = 0,1, 2,3,...........
é 330 - 2 s ù
0.94n = n ê ú 4
ë 330 û 22. (a) Since, I µ A2w 2
Þ s = 98.01 m
I1 µ (2a)2 w 2
é æ t ö x ù
19. (d) y = 0.02(m)sin ê2p ç ÷ - ú I 2 µ a 2 (2w ) 2
ë è 0.04( s) ø 0.50(m) û
I1 = I 2
Comparing this equation with the standard
wave equation Intensity depends on frequency also.
23. (a) The fundamental frequency of open tube
y = a sin(wt - kx ) v
we get + n0 = 2l … (i)
0
2p where l is the length of the tube
w=
0.04 v = speed of sound
That of closed tube
1 v
Þ n= = 25 Hz nc =
0.04 … (ii)
4lc
2p l0
k= Þ l = 0.5 m According to the problem lc =
0.50 2
\ velocity, v = nl = 25 × 0.5 m/s v v
= 12.5 m/s Thus nc = Þ nc = … (iii)
l0 / 2 2l
www.crackjee.xyz
Waves P-87
From equations (i) and (iii)
æ f2 ö æ 300 ö
n 0 = nc çè f - 1÷ø ´ 100 = çè 340 - 1÷ø ´ 100 ; 12%
1
Thus, nc = f ( Q n0 = f is given)
24. (b) Fundamental frequency, f
27. (b)
v 1 T 1 T
f = = =
2l 2l m 2l Ar l l

é T mù
êQ v = and m = ú (a) (b)
ë m lû
The fundamental frequency in case (a) is
Tl T Y Dl
Also, Y = Þ = v
ADl A l f=
2l
1 gDl The fundamental frequency in case (b) is
Þ f = ....(i)
2l lr v u
f'= = =f
Dl 4(l / 2) 2l
l = 1.5 m, = 0.01, 28. (a) We know that velocity in string is given by
l
r = 7.7 × 103 kg/m3 (given) T
g = 2.2 × 1011 N/m2 (given) v= ...(i)
m
Dl
Putting the value of l, , r and g in eqn. m mass of string
l where m = =
(i) we get, l length of string
2 103 m
f = ´ The tension T = ´ x´g
7 3 or f » 178.2 Hz
..(ii)
l
From (1) and (2)
25. (c) Length of pipe = 85 cm = 0.85m
Frequency of oscillations of air column in
closed organ pipe is given by,
l
(2n - 1)u T
f =
4L
x
(2n - 1)u
f = £ 1250
4L dx
= gx
(2n - 1) ´ 340 dt
Þ £ 1250
0.85 ´ 4
Þ 2n – 1 < 12.5 » 6 x -1/2 dx = g dt
l l
é v ù 320 -1/2
26. (d) f1 = f ê
ë v - v
ú= f´
sû 300
Hz \ òx dx - g ò dt 2 l
0 0
é v ù 320 l 20
f2 = f ê ú= f ´ Hz = g´t \ t = 2 =2 =2 2
ë v + vsû 340 g 10
EBD_7764
www.crackjee.xyz
P-88 Physics

Electric Charges
and Fields 15
1. A charged particle q is placed at the centre O of 5. Two spherical conductors B and C having equal
cube of length L (A B C D E F G H). Another radii and carrying equal charges on them repel
same charge q is placed at a distance L from O. each other with a force F when kept apart at
Then the electric flux through ABCD is [2002] some distance. A third spherical conductor
E F
having same radius as that B but uncharged is
D
c
brought in contact with B, then brought in
O contact with C and finally removed away from
q q
H both. The new force of repulsion between B and
G
C is [2004]
A
B (a) F/8 (b) 3 F/4
L
(c) F/4 (d) 3 F/8
(a) q /4 p Î0 L (b) zero
6. Four charges equal to -Q are placed at the four
(c) q/2 p Î0 L (d) q/3 p Î0 L corners of a square and a charge q is at its centre.
2. If a charge q is placed at the centre of the line If the system is in equilibrium the value of q is
joining two equal charges Q such that the system [2004]
is in equilibrium then the value of q is [2002]
Q Q
(a) Q/2 (b) –Q/2 (a) - (1 + 2 2) (b) (1 + 2 2)
(c) Q/4 (d) –Q/4 2 4
3. If the electric flux entering and leaving an enclosed
Q Q
surface respectively is f1 and f 2 , the electric (c) - (1 + 2 2) (d) (1 + 2 2)
charge inside the surface will be [2003] 4 2
(a) (f2 - f1 )e o (b) (f1 + f2 ) / e o 7. A charged oil drop is suspended in a uniform
(c) (f2 - f1) / e o (d) (f1 + f2 )e o field of 3×104 v/m so that it neither falls nor rises.
4. Three charges –q1 , +q2 and –q3 are place as The charge on the drop will be (Take the mass of
shown in the figure. The x - component of the the charge = 9.9×10–15 kg and g = 10 m/s2)
force on –q1 is proportional to [2003] [2004]
Y (a) 1.6×10–18 C (b) 3.2×10–18 C
-q 3 (c) 3.3×10–18 C (d) 4.8×10–18 C
8. Two point charges + 8q and – 2q are located at
x = 0 and x = L respectively. The location of a
a
b point on the x axis at which the net electric field
due to these two point charges is zero is
-q 1 +q 2 X [2005]
q2 q3 q2 q3
(a) 2
- 2
cos q (b) + sin q L
a2
2 (a) (b) 2 L
b a b
4
q2 q3 q2 q3
(c) + cos q (d) - sin q (c) 4 L (d) 8 L
b2 a2 b2 a2
www.crackjee.xyz
Electric Charges and Fields P-89
9. A charged ball B hangs from a silk thread S,
E(r) E(r)
which makes an angle q with a large charged
conducting sheet P, as shown in the figure. The
(a) (b) r
surface charge density s of the sheet is r O R
O R
proportional to [2005]
E(r)
E(r)
P
q
S (c) R
r (d) r
O
O R

14. A charge Q is placed at each of the opposite


B corners of a square. A charge q is placed at each
(a) cot q (b) cos q of the other two corners. If the net electrical force
(c) tan q (d) sin q on Q is zero, then Q/q equals: [2009]
10. An electric dipole is placed at an angle of 30° to (a) –1 (b) 1
a non-uniform electric field. The dipole will 1
experience [2006] (c) - (d) -2 2
2
(a) a translational force only in the direction
Q
of the field 15. Let r (r ) = r be th e charge density
(b) a translational force only in a direction p R4
normal to the direction of the field distribution for a solid sphere of radius R and
(c) a torque as well as a translational force total charge Q. For a point ‘P’ inside the sphere
at distance r1 from the centre of the sphere, the
(d) a torque only
magnitude of electric field is : [2009]
11. Two spherical conductors A and B of radii 1 mm
and 2 mm are separated by a distance of 5 cm Q Qr12
(a) (b)
and are uniformly charged. If the spheres are 4p Î0 r12 4p Î0 R4
connected by a conducting wire then in
Qr12
equilibrium condition, the ratio of the magnitude (c) (d) 0
of the electric fields at the surfaces of spheres A 3p Î0 R4
and B is [2006] 16. A thin semi-circular ring of radius r has a positive
(a) 4 : 1 (b) 1 : 2 charge q distributed uniformly over it. The net
ur
(c) 2 : 1 (d) 1 : 4 field E at the centre O is [2010]
12. If gE and gM are the accelerations due to gravity j
on the surfaces of the earth and the moon
respectively and if Millikan’s oil drop experiment
could be performed on the two surfaces, one
will find the ratio [2007] i
electronic charge on the moon O
to be q ˆj q
electronic charge on the earth - ˆj
(a) 2 2 (b)
(a) g M / g E (b) 1 4p e 0 r 4p e 0 r 2
2

(c) 0 (d) g E / g M q q
- ˆj ˆj
13. A thin spherical shell of radus R has charge Q (c) (d)
2p e 0 r 2
2p e 0 r 2
2 2
spread uniformly over its surface. Which of the
17. Let there be a spherically symmetric charge
following graphs most closely represents the
distribution with charge density varying as
electric field E(r) produced by the shell in the
æ5 rö
range 0 £ r < ¥, where r is the distance from the r(r ) = r0 ç - ÷ upto r = R , and r(r ) = 0
è 4 Rø
centre of the shell? [2008]
EBD_7764
www.crackjee.xyz
P-90 Physics
for r > R , where r is the distance from the origin. 22. A long cylindrical shell carries positive surface
The electric field at a distance r(r < R) from the charge s in the upper half and negative surface
origin is given by [2010] charge - s in the lower half. The electric field
r0 r æ 5 r ö 4pr0 r æ 5 r ö lines around the cylinder will look like figure
(a) ç - ÷ (b) 3e çè 3 - R ÷ø given in : (figures are schematic and not drawn
4e 0 è 3 R ø 0
r0 r æ 5 r ö r0 r æ 5 r ö to scale) [2015]
(c) ç - ÷ (d) 3ε çè 4 - R ÷ø
4ε0 è 4 R ø 0
18. Two identical charged spheres suspended from (a) (b)
a common point by two massless strings of
length l are initially a distance d(d << l) apart
because of their mutual repulsion. The charge
begins to leak from both the spheres at a (c) (d) `
constant rate. As a result charges approach each
other with a velocity v. Then as a function of
distance x between them, [2011] 23. The region between two concentric spheres of
(a) v µ x–1 (b) v µ x½ radii 'a' and 'b', respectively (see figure), have
(c) v µ x (d) v µ x–½ A
19. The potential inside a charged spherical ball is volume charge density r= , where A is a
r
given by f = ar2 + b where r is the distance from constant and r is the distance from the centre.
the centre and a, b are constants. Then the charge At the centre of the spheres is a point charge Q.
density inside the ball is: [2011] The value of A such that the electric field in the
(a) –6ae0r (b) –24pae0 region between the spheres will be constant, is:
(c) –6ae0 (d) –24pae0r [2016]
20. In a uniformly charged sphere of total charge Q
and radius R, the electric field E is plotted as
function of distance from the centre, The graph
which would correspond to the above will be:
[2012]
a

E E Q
b
(a) (b)
Rr Rr 2Q 2Q
(a) p a 2 - b 2 (b)
E E ( ) pa 2

Q Q
(c) (d)
(c) (d) 2p b2 - a 2
R r Rr
2pa 2 ( )
ur
21. Two charges, each equal to q, are kept at x = – a 24. An electric dipole has a fixed dipole moment p ,
and x = a on the x-axis. A particle of mass m and which makes angle q with respect to x-axis. When
uur
q subjected to an electric field E1 = Eiˆ , it
charge q0 = is placed at the origin. If charge ur
2 experiences a torque T1 = t iˆ . When subjected
q0 is given a small displacement (y <<a) along uur
the y-axis, the net force acting on the particle is to another electric field E2 = 3E1 ˆj it
uur ur
proportional to [2013] experiences torque T2 = -T1 . The angle q is :
(a) y (b) –y [2017]
1 1 (a) 60° (b) 90°
(c) (d) – (c) 30° (d) 45°
y y
www.crackjee.xyz
Electric Charges and Fields P-91

Ans we r K e y
1 2 3 4 5 6 7 8 9 10 11 12 13 14 15
(b) (d) (a) (b) (d) (b) (c) (b) (c) (c) (c) (b) (a) (d) (b)
16 17 18 19 20 21 22 23 24
(c) (a) (d) (c) (c) (a) (c) (c) (a)

1. (b) Both the charges are identical and placed q2 q3


symmetrically about ABCD. The flux \ Fx µ 2
+ sin q
crossing ABCD due to each charge is b a2
1é q ù
ê ú
6 ë Î0 û but in opposite directions.
Therefore the resultant is zero. F12
C F13 sin q
D q

q
.
q
.

B F13
F13cos q
A
2. (d) For equilibrium of charge Q C
B
Q´Q Qq 5. (d) ×
k +k =0 r r
(2x) 2 x2 QB QC
x x F µ
x2
Q q Q x is distance between the spheres. After
Q Q
Þ q=- first operation charge on B is halved i.e
4 2
3. (a) The flux entering an enclosed surface is Q
taken as negative and the flux leaving the and charge on third sphere becomes .
2
surface is taken as positive, by convention. Now it is touched to C, charge then equally
Therefore the net flux leaving the enclosed distributes them selves to make potential
surface = f2 - f1 same, hence charge on C becomes
\ the charge enclosed in the surface by æ Q ö 1 3Q
çè Q + ÷ø = .
Gauss’s law is q =Î0 (f 2 - f1 ) 2 2 4
4. (b) Force on charge q1 due to q2 is æ 3Q ö æ Q ö
qq çè ÷ø çè ÷ø
F12 = k 1 22 QC' QB' 4 2 3 Q2
\ Fnew µ = =
b 2
x x 2
8 x2
Force on charge q1 due to q3 is
3
qq or Fnew = F
F13 = k 1 23 8
a 6. (b) Net field at A should be zero
The X - component of the force (Fx) on
2 E1 + E2 = E3
q1 is F12 + F13 sin q
q1q2 q1q2 kQ ´ 2 kQ kq
\ Fx = k +k sin q \ 2
+ 2
= 2
b2 a2 a ( 2 a) æ a ö
çè ÷

EBD_7764
www.crackjee.xyz
P-92 Physics
E3 –Q –Q B +Q1 +Q2
E1 11. (c)
A
E2 r1 r2
2 E1 A –Q –Q A
E B
After connection, V1 = V2
Q 2 Q Q
Þ + = 2q Þ q = (2 2 + 1) Q Q Q Q
1 2 4 Þk 1 =k 2 Þ 1 = 2
r1 r2 r1 r2
7. (c) At equilibrium, electric force on drop
balances weight of drop. The ratio of electric fields
Q
mg k 21
. qE = mg Þ q = E1 r E Q r2
E = 1 Þ 1 = 21 ´ 2
E2 Q E2 r1 Q2
9.9 ´ 10-15 ´ 10 k 22
q= = 3.3 ´ 10 -18 C r2
3 ´ 104
E1 r1 ´ r22 E r 2
- K 2q K 8q 1 4 Þ = Þ 1 = 2 =
8. (b) + =0 Þ = E2 r12 ´ r2 E2 r1 1
2 2 2
( x - L) x ( x - L) x2
Since the distance between the spheres is
1 2 large as compared to their diameters, the
or = Þ x = 2 x - 2 L or x = 2L
x-L x induced effects may be ignored.
9. (c) 12. (b) Electronic charge does not depend on
P
acceleration due to gravity as it is a
T cos q universal constant.
q T So, electronic charge on earth
s = electronic charge on moon
F = Eq = q \ Required ratio = 1.
e0K
13. (a) The electric field inside a thin spherical
T sin q shell of radius R has charge Q spread
uniformly over its surface is zero.
mg Q ++ + +
+
+

R
+

+ + +

s
+++

T sin q = .q .... (i) Q


e0 K E=k 2
T cos q = mg r
++

.... (ii)
+ +

E=0
Dividing (i) by (ii),
+

+ + +
++
sq
tan q = Outside the shell the electric field is
e 0 K . mg
Q
\ s µ tan q E = k . These characteristics are
10. (c) r2
+q represented by graph (a).
14. (d) Let F be the force between Q and Q. The
F1 force between q and Q should be attractive
E1 for net force on Q to be zero. Let F ¢ be the
force between Q and q . The resultant of
F2 F ¢ and F ¢ is R. For equilibrium
E2 Q q
–q
The electric field will be different at the
location of the two charges. Therefore the l R F¢
two forces will be unequal. This will result
in a force as well as torque.
q Q

F
www.crackjee.xyz
Electric Charges and Fields P-93
r r q
R+ F = 0 2 F ' = -F = (rd q) (Qdl = rd q)
pr
Qq Q2 Q æqö
2´k 2
= -k 2 Þ = -2 2 = ç ÷ dq
l ( 2 l) q èpø
15. (b) Electric field at O due to dq is
R 1 dq 1 q
dE = . = 4p Î . 2 d q
4p Î0 r 2 0 pr
The component dE cos q will be counter
balanced by another element on left portion.
x dx Hence resultant field at O is the resultant of the
component dE sinq only.
p
q
\ E = ò dE sin q = ò 4 p2 r 2 Î sin qd q
0 0
Let us consider a spherical shell of q
thickness dx and radius x. The volume of
=
2 2
[ - cos q]p0
4p r Î0
this spherical shell = 4px 2 dx . The charge q q
enclosed within shell = (+1 + 1) = 2 2
2 2
4p r Î0 2p r Î0
é Q.x ù 2 4Q 3 The direction of E is towards negative y-axis.
= ê 4 ú[4px dx] = 4 x dx r q
ë pR û R \ E=- 2 2 ˆj
The charge enclosed in a sphere of radius 2p r Î0
r1 is 17. (a) Let us consider a spherical shell of radius x
r1 r1 and thickness dx.
4Q 4Q é x 4 ù Q 4 Charge on this shell
ò
3
= x dx = ê ú = 4 r1
R4 4 4
R êë úû R æ5 x ö 2
0 0 dq = r.4px 2 dx = r0 ç 4 - R ÷ .4px dx
\ The electric field at point P inside the è ø
sphere at a distance r1 from the centre of \ Total charge in the spherical region from
centre to r (r < R ) is
the sphere is
r
æ5 xö
é Q 4ù q = ò dq = 4 pr0 ò ç - ÷ x 2 dx
r
1 êë R 4 1 úû 0
è4 Rø
E=
4p Î0 r12

1 Q 2 dx
= r1
4p Î0 R 4 x
16. (c) Let us consider a differential element dl.
charge on this element.
j
+
+

+ dl é 5 r3 1 r 4 ù
+

- . ú = pr0r 3 æç - ö÷
5 r
+ dq
+
= 4 pr0ê .
ëê 4 3 R 4 ûú è3 Rø
+ dE q
+
cos q
i \ Electric field at r,
O 1 q
dE E= .
4p Î0 r 2
dE sin q
1 pr0 r 3 æ 5 r ö r r æ5 r ö
æ q ö
dq = ç ÷ dl = . 2 ç - ÷= 0 ç - ÷
4pÎ0 r è 3 R ø 4 Î0 è 3 R ø
è pr ø
EBD_7764
www.crackjee.xyz
P-94 Physics
18. (d) At any instant r
E on the surface of the charged
T cosq = mg ....(i) sphere
r 1 q r 1
T sin q = Fe ....(ii) Es = i.e., Es µ nˆ …(ii)
4p Î0 R 2
R2
sin q Fe r
Þ cos q = mg E on any point away from the uniformly
charged sphere is given
Þ Fe = mg tan q r 1 q
E= nˆ
kq 2 4p Î0 r 2
Þ = mg tan q
x2 r 1
Þ q2 µ x2 tan q E µ 2 nˆ … (iii)
r
x Q R is the radius of the sphere, which is
sin q = r
2l constant, thus E is maximum and
For small q, sin q » tan q constant at the surface of the sphere. But
\ q2 µ x3 decreases on moving away from the
surface of the uniformally charged sphere.
y
l q l
Tcosq
T
q Tsinq F F
Fe 21. (a) x
x q a a q
mg
Þ F sin q F sin q Þ Fnet
dq 2 dx
Þq µ x
dt dt 2Fcos q
dq = 2F cosq
\ = const.
dt æ qö
\q µ x2.v 2kq ç ÷
è 2ø y
Fnet = .
( )
3 2
Þ x ax 2 .v y2 + a2 y + a2
2
2
Þv µ x–1/2 [Q q2 µ x3]
19. (c) Electric field æ qö
2kq ç ÷ y
df è 2ø kq2 y
E=- = – 2ar ....(i) Fnet = Þ
dr ( y 2 + a 2 )3/ 2 a3
By Gauss's theorem So, F µ y
1 q 22. (c) Field lines originate perpendicular from
E= ....(ii)
4pe 0 r 2 positive charge and terminate
From (i) and (ii), perpendicular at negative charge. Further
Q = –8 pe0ar3 this system can be treated as an electric
dipole.
Þdq = – 24pe0ar2 dr 23. (c) Applying Gauss's law
dq
Charge density, r = = – 6e0a uur uur
4pr 2dr Q
20.
r
(c) E inside the charged sphere Ñò S E. ds = Î0
Q + 2par 2 - 2pAa 2
R
P \ E × 4pr2 =
Î0
O P Q dr
r=
dv
r
Ein = 0 …(i)
www.crackjee.xyz
Electric Charges and Fields P-95
24. (a) T = PE sinq Torque experienced by the
ur ur ur
r dr dipole in an electric field, T = P ´ E
ur
Q a Gaussiam p = p cosq iˆ + psinq ĵ
surface ur r
b E1 = Ei
Q = r4pr2 ur ur ur
T 1 = p ´ E1 = (p cos q iˆ + p sin q ĵ ) × E ( iˆ )
.
A
ò r 4pr
2 t k̂ = pE sinq (– k̂ ) ...(i)
Q= dr =2pA [r2 – a2]
ur
a E 2 = 3 E1 j ˆ
é Q - 2pAa 2 ù ur
1 T 2 = p cos qiˆ + p sin qˆj ) ´ 3 E1 ˆj
E= ê + 2pA ú
4p Î0 ëê r 2
ûú tkˆ = 3 pE cos qkˆ
1 ...(ii)
For E to be independent of 'r' From eqns. (i) and (ii)
Q – 2pAa2 = 0
pE sinq = 3 pE cosq
Q
\ A= tanq = 3 \ q = 60°
2pa 2
EBD_7764
www.crackjee.xyz
P-96 Physics

Electrostatic Potential
and Capacitance 16
1. On moving a charge of 20 coulomb by 2 cm, 2 J 6. The work done in placing a charge of 8 × 10–18
of work is done, then the potential difference coulomb on a condenser of capacity 100
between the points is [2002] micro-farad is [2003]
(a) 0.1 V (b) 8 V (a) 16 ´ 10 -32 joule
(c) 2 V (d) 0.5 V. (b) 3.1´10 -26 joule
2. If there are n capacitors in parallel connected to (c) 4 ´10-10 joule
V volt source, then the energy stored is equal to (d) 32 ´ 10-32 joule
[2002] 7. Two thin wire rings each having a radius R are
1 placed at a distance d apart with their axes
(a) CV (b) nCV2
2 coinciding. The charges on the two rings are +q
1 and -q. The potential difference between the
(c) CV2 (d) CV2 centres of the two rings is [2005]
2n
3. Capacitance (in F) of a spherical conductor with é1 ù
q 1
radius 1 m is [2002] (a) ê - ú
-10 -6 2p Î0 ëê R R 2 + d 2 úû
(a) 1.1´ 10 (b) 10
(c) 9 ´ 10 -9 (d) 10 -3 qR
(b)
4. A sheet of aluminium foil of negligible thickness 4p Î0 d 2
is introduced between the plates of a capacitor.
q é1 1 ù
The capacitance of the capacitor [2003] (c) ê - ú
(a) decreases 4p Î0 ëê R R 2 + d 2 úû
(b) remains unchanged (d) zero
(c) becomes infinite 8. A parallel plate capacitor is made by stacking n
(d) increases equally spaced plates connected alternatively.
5. A thin spherical conducting shell of radius R If the capacitance between any two adjacent
has a charge q. Another charge Q is placed at plates is ‘C’ then the resultant capacitance is
the centre of the shell. The electrostatic potential [2005]
R (a) (n + 1) C (b) (n – 1) C
at a point P , a distance from the centre of the (c) nC (d) C
2
shell is [2003] 9. A fully charged capacitor has a capacitance ‘C’.
2Q 2Q 2q It is discharged through a small coil of resistance
(a) 4pe R (b) 4pe R - 4pe R wire embedded in a thermally insulated block of
o o o
specific heat capacity ‘s’ and mass ‘m’. If the
2Q q (q + Q)2 temperature of the block is raised by ‘DT’, the
(c) +
4pe o R 4pe o R (d) 4pe o R potential difference ‘V’ across the capacitance
is [2005]
www.crackjee.xyz
Electrostatic Potential and Capacitance P-97
ur
mC DT 2mC DT (c) both E and V change
(a) (b) ur
s s (d) E and V remain unchanged
2ms DT ms DT 13. The potential at a point x (measured in m m) due
(c) (d)
C C to some charges situated on the x-axis is given
10. Two insulating plates are both uniformly charged by V(x) = 20/(x2 – 4) volt. The electric field E
in such a way that the potential difference at x = 4 m m is given by [2007]
between them is V2 – V1 = 20 V. (i.e., plate 2 is at (a) (10/9) volt/ m m and in the +ve x direction
a higher potential). The plates are separated by (b) (5/3) volt/ m m and in the –ve x direction
d = 0.1 m and can be treated as infinitely large. (c) (5/3) volt/ m m and in the +ve x direction
An electron is released from rest on the inner (d) (10/9) volt/ m m and in the –ve x direction
surface of plate 1. What is its speed when it hits 14. A parallel plate condenser with a dielectric of
plate 2? (e = 1.6 × 10–19 C, me = 9.11 × 10–31 kg) dielectric constant K between the plates has a
[2006] capacity C and is charged to a potential V volt.
The dielectric slab is slowly removed from
Y between the plates and then reinserted. The net
work done by the system in this process is [2007]
0.1 m 1
(a) zero (b) ( K - 1) CV 2
X 2
2
(c) CV ( K - 1) (d) ( K - 1) CV 2
K
15. A parallel plate capacitor with air between the
1 2 plates has capacitance of 9 pF. The separation
(a) 2.65 × 106 m/s (b) 7.02 × 1012 m/s between its plates is ‘d’. The space between the
(c) 1.87 × 106 m/s (d) 32 × 10–19 m/s plates is now filled with two dielectrics. One of
11. An electric charge 10–3 m C is placed at the origin the dielectrics has dielectric constant k1 = 3 and
(0, 0) of X – Y co-ordinate system. Two points A d
thickness while the other one has dielectric
and B are situated at ( 2, 2) and (2, 0) 3
respectively. The potential difference between 2d
constant k2 = 6 and thickness . Capacitance
the points A and B will be [2007] 3
(a) 4.5 volts (b) 9 volts of the capacitor is now [2008]
(c) Zero (d) 2 volt (a) 1.8 pF (b) 45 pF
12. Charges are placed on the vertices of a square (c) 40.5 pF (d) 20.25 pF
ur
as shown. Let E be the electric field and V the 16. Two points P and Q are maintained at the
potential at the centre. If the charges on A and B potentials of 10 V and – 4 V, respectively. The
are interchanged with those on D and C work done in moving 100 electrons from P to Q
respectively, then [2007] is: [2009]
q q (a) 9.60 × 10 J –17 (b) –2.24 × 10 J –16

A B (c) 2.24 × 10–16 J (d) –9.60× 10–17 J


17. Two positive charges of magnitude ‘q’ are
placed, at the ends of a side (side 1) of a square
of side ‘2a’. Two negative charges of the same
D C magnitude are kept at the other corners. Starting
-q -q from rest, if a charge Q moves from the middle of
ur
(a) E changes, V remains unchanged side 1 to the centre of square, its kinetic energy
ur at the centre of square is [2011 RS]
(b) E remains unchanged, V changes
EBD_7764
www.crackjee.xyz
P-98 Physics
(a) zero dielectric of dialectric constant 2.2 between them.
1 2 qQ æ 1 ö When the electric field in the dielectric is
(b) 1+ 3 × 104 V/m the charge density of the positive
4 pe 0 a çè ÷

plate will be close to: [2014]
1 2 qQ æ 2 ö (a) 6 ´ 10-7 C m 2 (b) 3 ´10-7 C m2
(c) ç 1- ÷
4 pe 0 a è 5ø
(c) 3 ´ 104 C m 2 (d) 6 ´ 104 C m 2
1 2 qQ æ 1 ö
(d) 1- 21. In the given circuit, charge Q2 on the 2µF
4 pe 0 a çè ÷
5ø capacitor changes as C is varied from 1µF to
18. This questions has statement-1 and statement- 3µF. Q2 as a function of 'C' is given properly by:
2. Of the four choices given after the statements, (figures are drawn schematically and are not
choose the one that best describes the two to scale) [2015]
statements.
An insulating solid sphere of radius R has a 1µF
uniformly positive charge density r. As a result C
of this uniform charge distribution there is a finite 2µF
value of electric potential at the centre of the
sphere, at the surface of the sphere and also at a
point outside the sphere. The electric potential E
at infinite is zero. [2012] Charge
Statement -1 When a charge q is taken from the
centre to the surface of the sphere its potential Q2
qr (a)
energy changes by .
3e0 1µF 3µF
C

Statement -2 The electric field at a distance r


Charge
rr
(r <R) from the centre of the sphere is .
3e0 Q2
(a) Statement 1 is true, Statement 2 is true; (b)
Statement 2 is not the correct explanation C
1µF 3µF
of statement 1.
(b) Statement 1 is true Statement 2 is false. Charge
(c) Statement 1 is false Statement 2 is true.
Q2
(d) Statement 1 is true, Statement 2 is true,
Statement 2 is the correct explanation of (c)
Statement 1 1µF 3µF
C
r
19. Assume that an electric field E = 30x 2 ˆi exists
Charge
in space. Then the potential difference VA - VO ,
Q2
where VO is the potential at the origin and VA
(d)
the potential at x = 2 m is: [2014] C
(a) 120 J/C (b) –120 J/C 1µF 3µF
(c) –80 J/C (d) 80 J/C
22. A uniformly charged solid sphere of radius R
20. A parallel plate capacitor is made of two circular has potential V0 (measured with respect to ¥)
plates separated by a distance 5 mm and with a on its surface. For this sphere the equipotential
www.crackjee.xyz
Electrostatic Potential and Capacitance P-99

3V0 5V0 3V0 3mF


surfaces with potentials , , and
2 4 4 4mF
V0 9mF
have radius R1, R2, R3 and R4 respectively..
4
Then [2015]
2mF
(a) R1 = 0 and R2 < (R4 – R3)
+ –
(b) 2R < R4
(c) R1 = 0 and R2 > (R4 – R3) 8V
(a) 420 N/C (b) 480 N/C
(d) R1 ¹ 0 and (R2 – R1) > (R4 – R3) (c) 240 N/C (d) 360 N/C
24. A capacitance of 2mF is required in an electrical
23. A combination of capacitors is set up as shown circuit across a potential difference of 1.0 kV. A
in the figure. The magnitude of the electric field, large number of 1mF capacitors are available
due to a point charge Q (having a charge equal which can withstand a potential difference of
to the sum of the charges on the 4 mF and 9 mF not more than 300 V. The minimum number of
capacitors), at a point distance 30 m from it, would capacitors required to achieve this is [2017]
equal : [2016] (a) 24 (b) 32
(c) 2 (d) 16
Answer Key
1 2 3 4 5 6 7 8 9 10 11 12 13 14 15
(a) (b) (a) (b) (c) (d) (a) (b) (c) (a) (c) (a) (a) (a) (c)
16 17 18 19 20 21 22 23 24
(c) (d) (c) (c) (a) (d) (a) (a) (b)

WAB of V between the plates.


1. (a) We know that = VB - V A
q So energy stored in each capacitor
1
2J
= 0.1J/C = 0.1V = CV 2 .
\ VB - V A = 2
20C
\ Energy stored in n capacitor
2. (b) The equivalent capacitance of n identical
capacitors of capacitance C is equal to nC. é1 2ù
Energy stored in this capacitor = ê CV ú ´ n
ë2 û
1 1 3. (a) For an isolated sphere, the capacitance is
E= (nC )V 2 = nCV 2 given by
2 2
1
C C = 4p Î0 r = ´ 1 = 1.1 ´ 10-10 F
C 9 ´ 109
n times
C 4. (b) The capacitance of a parallel plate capacitor
in which a metal plate of thickness t is
V inserted is given by
V eo A eo A
Alternatively C= . Here t ® 0 \ C =
d -t d
Each capacitor has a potential difference
EBD_7764
www.crackjee.xyz
P-100 Physics
5. (c) Electric potential due to charge Q placed éq q ù
1 q q
at the centre of the spherical shell at point = ê + - - ú
4 pe 0 êë R R R2 + d 2 R 2 + d 2 úû
P is
éq
1 q ù
= ê - ú
R/2 2pe 0
ëê R R 2 + d 2 ûú
Q P
8. (b) As n plates are joined, it means (n – 1)
R q combination joined in parallel.
\ resultant capacitance = (n – 1) C
1 Q 1 2Q 9. (c) Applying conservation of energy,
V1 = =
4pe o R / 2 4pe o R 1 2m. s. Dt
CV 2 = m. s Dt ; V =
Electric potential due to charge q on the 2 C
surface of the spherical shell at any point 1
10. (a) eV = mv 2
inside the shell is 2

V2 =
1 q 2eV 2 ´ 1.6 ´ 10-19 ´ 20
4pe o R Þv= =
m 9.1 ´ 10 -31
\ The net electric potential at point P is
1 2Q 1 q = 2.65 ´10 6 m / s
V = V1 + V2 = + Y
4pe o R 4pe o R
6. (d) The work done is stored as the potential
energy. The potential energy stored in a 11. (c) A(Ö2,Ö2)
capacitor is given by ®
r1
( )
-18 2
1 Q 2 1 8 ´ 10
U= = ´ O
2 C 2 100 ´ 10 -6 X
(0,0) ®
r2 B (2,0)
–32
= 32 ´ 10
( )
J
7. (a) q q The distance of point A 2, 2 from the
origin,
ur 2 2
OA = | r1 | = ( 2) + ( 2) = 4 = 2units.
The distance of point B(2, 0) from the
R R origin,
1 2 uur 2 2
OB = | r2 | = (2) + (0) = 2 units.
d
At (1) using, potential 1 Q
Now, potential at A, VA = .
(V1 ) = Vself + Vdue to (2) 4p Î0 (OA)

éq ù 1 Q
1 q Potential at B, VB = .
Þ V1 = ê - ú 4p Î0 (OB)
4pe 0êë R R + d2
2 úû \ Potential difference between the points
At (2) using potential A and B is given by
(V2 ) = Vself + Vdue to (1) 1 Q 1 Q
VA – VB = . - .
é- q ù 4p Î0 OA 4p Î0 OB
1 q
Þ V2 = ê + ú Q æ 1 1 ö Q æ 1 1ö
4pe 0 êë R úû = çè - ÷ø = ç - ÷
R + d2
2
4p Î0 OA OB 4p Î0 è 2 2 ø
DV = V1 - V2 Q
= ´ 0 = 0.
4p Î0
www.crackjee.xyz
Electrostatic Potential and Capacitance P-101
12. (a) As shown in the figure, the resultant K is the dielectric constant.
electric fields before and after Again, when the dielectric slab is removed
interchanging the charges will have the slowly its energy increases to initial
same magnitude, but opposite directions. potential energy. Thus, work done is zero.
Also, the potential will be same in both
cases as it is a scalar quantity.
q q 15. (c)
A B

The given capacitance is equal to two


capacitances connected in series where

® k1Î0 A 3k1 Î0 A
E C1 = =
d /3 d
D C
3 ´ 3Î0 A 9 Î0 A
-q -q = =
d d
-q -q and
A B k Î A 3k2 Î0 A
® C1 = 2 0 =
E 2d / 3 2d
3 ´6 Î0 A 9 Î0 A
= =
2d d
The equivalent capacitance Ceq is
1 1 1
D C = +
Ceq C1 C2
q q
d d 2d
20 = + =
13. (a) Here, V(x) = 2 volt 9 Î0 A 9 Î0 A 9 Î0 A
x -4 9 Î0 A 9
d æ 20 ö \ Ceq = = ´ 9 pF = 40.5 pF
dV 2 d 2
We know that E = - =- ç
dx dx è x 2 - 4 ÷ø
WPQ
40 x 16. (c) = (VQ - VP )
or, E = + q
( x 2 - 4) 2
At x = 4 mm , Þ WPQ = q (VQ - VP )
40 ´ 4 160 10
E= + =+ = + volt / mm. = (– 100 × 1.6 × 10–19) (– 4 – 10)
(42 - 4) 2 144 9
r = +2.24 × 10–16J
Positive sign indicates that E is in +ve x- 17. (d) Potential at point A,
direction. 2 kq 2 kq
14. (a) The potential energy of a charged VA = -
a a 5
Q2 kq
capacitor is given by U = . \ (potential due to each q = and
2C a
If a dielectric slab is inserted between the - kq
potential due to each – q = )
Q2 a 5
plates, the energy is given by , where
2KC
EBD_7764
www.crackjee.xyz
P-102 Physics
2a R 2 rq
q q DU =
A
6 Î0
19. (c) Potential difference between any two
points in an electric field is given by,
2a ur uur
B dV = – E.dx
VA 2

–q –q ò dV = - ò 30 x 2 dx
VO 0
Potential at point. B,
VB = 0 VA - VO = -[10 x3 ]20 = -80 J / C
( Q Point B is equidistant from all the four 20. (a) Electric field in presence of dielectric
charges) between the two plates of a parallel plate
\ Using work energy theorem, capaciator is given by,
(WAB ) electric = Q (VA - VB )
E=
s
K e0
2kqQ é 1 ù
= ê1- ú Then, charge density
a ë 5û
s = K e0 E
æ 1 ö 2Qq é 1 ù = 2.2 × 8.85 × 10–12 × 3 × 104
=ç ÷ ê1- ú
è 4pe 0 ø a ë 5û » 6 × 10–7 C/m2
18. (c) The potential energy at the centre of the s.s
= (Let surface area of plates = s)
sphere v
3 KQq s.s
Uc = =
2 R s æ ld ö
ln ç1 + ÷
The potential energy at the surface of the l è K0 ø
sphere d 1
= sl. (Q in vacuum e0 =1)
KqQ d æ ld ö
Us = ln ç1 + ÷
R è K0 ø
Now change in the energy ld æ sö
DU = Uc - U s
c= . C0 çè here, C0 = ÷ø
æ ld ö d
ln ç 1 +
è K 0 ÷ø
KQq é 3 ù Q1 1mF
= -1
R êë 2 úû 21. (d)
Q
KQq
= C
2R
4 3 Q2 2mF
Where Q = r.V = r. pR
3
3
2K pR rq
DU = 2 2
3 R From figure, Q2 = Q = Q
3 2 +1 3
2 1 pR rq
DU = ´
3 4p Î0 R æC ´ 3ö
Q = E çè ÷
C + 3ø
www.crackjee.xyz
Electrostatic Potential and Capacitance P-103
3µF
2 æ 3CE ö 2CE 23. (a) 4µF C1 = 4µF 12µF = CP
\ Q2 = ç ÷ =
3 èC +3ø C +3
9µF
Þ
Therefore graph d correctly dipicts. 2µF
8v 8v
Charge é æ 12 ö ù
Charge on C1 is q1 = ê ç
è ÷ø ´ 8ú ´ 4 =
ë 4 + 12 û
24mc

4
The voltage across CP is VP = ×8 = 2v
4 + 12
C
1mF 3mF \ Voltage across 9mF is also 2V
Kq \ Charge on 9mF capacitor = 9 × 2 = 18mC
22. (a) We know, V0 = = Vsurface \ Total charge on 4 mF and 9mF = 42mc
R
Kq
(3R 2 – r 2 ) [For r < R] KQ 42 ´ 10-6
Now, Vi = \E = = 9 × 109 ×
2R 3 r2 30 ´ 30
At the centre of sphare r = 0. Here
= 420 NC–1
3
V = V0 24. (b) To get a capacitance of 2mF arrangement
2
Kq of capacitors of capacitance 1mF as shown
5 Kq
Now, = (3R 2 – r 2 ) in figure 8 capacitors of 1mF in parallel with
4 R 2R 3 four such branches in series i.e., 32 such
R capacitors are required.
R2 =
2
3 Kq Kq
= 3
4 R R
1 Kq Kq
=
4 R R4 m m m m
R4 = 4R
Also, R1 = 0 and R2 < (R4 – R3)
1 1 1 1 1
= + + + \ Ceq = 2 mF
Ceq 8 8 8 8
EBD_7764
www.crackjee.xyz
P-104 Physics

Current Electricity
17
1. If an ammeter is to be used in place of a voltmeter, 6. The thermo e.m.f. of a thermo-couple is 25mV/°C
then we must connect with the ammeter a [2002] at room temperature. A galvanometer of 40 ohm
(a) low resistance in parallel resistance, capable of detecting current as low
(b) high resistance in parallel as 10–5 A, is connected with the thermo couple.
(c) high resistance in series The smallest temperature difference that can be
(d) low resistance in series.
detected by this system is [2003]
2. A wire when connected to 220 V mains supply
(a) 16°C (b) 12°C
has power dissipation P1. Now the wire is cut
into two equal pieces which are connected in (c) 8°C 8 o C (d) 20°C
parallel to the same supply. Power dissipation in 7. The negative Zn pole of a Daniell cell, sending a
this case is P2. Then P2 : P1 is [2002] constant current through a circuit, decreases in
(a) 1 (b) 4 mass by 0.13g in 30 minutes. If the
(c) 2 (d) 3 electeochemical equivalent of Zn and Cu are
3. If in the circuit, power dissipation is 150 W, then 32.5 and 31.5 respectively, the increase in the
R is [2002] mass of the positive Cu pole in this time is
R [2003]
(a) 0.180 g (b) 0.141g
(c) 0.126 g (d) 0.242 g
2W
8. An ammeter reads upto 1 ampere. Its internal
15 V resistance is 0.81ohm. To increase the range to
(a) 2W (b) 6W 10 A the value of the required shunt is [2003]
(c) 5W (d) 4W (a) 0.03 W (b) 0.3W
4. The mass of product liberated on anode in an (c) 0.9W (d) 0.09W
electrochemical cell depends on [2002] 9. A 3 volt battery with negligible internal
(a) (It)1/2 (b) It resistance is connected in a circuit as shown in
(c) I/t (d) I2t the figure. The current I, in the circuit will be
(where t is the time period for which the current [2003]
is passed).
5. The length of a wire of a potentiometer is 100
cm, and the e. m.f. of its standard cell is E volt. It
is employed to measure the e.m.f. of a battery
whose internal resistance is 0.5W. If the balance 3W
point is obtained at l = 30 cm from the positive 3W
3V
end, the e.m.f. of the battery is [2003]
30 E 30 E
(a) (b) 100 - 0.5
100.5 ( )
3W
30 ( E - 0.5i ) 30 E
(c) (d) (a) 1 A (b) 1.5 A
100 100
(c) 2 A (d) 1/3 A
where i is the current in the potentiometer wire.
www.crackjee.xyz
Current Electricity P-105
10. A 220 volt, 1000 watt bulb is connected across (c) metals with low temperature coefficient of
a 110 volt mains supply. The power consumed resistivity
will be [2003] (d) semiconducting materials having low
(a) 750 watt (b) 500 watt temperature coefficient of resistivity
(c) 250 watt (d) 1000 watt 17. Time taken by a 836 W heater to heat one litre of
11. The length of a given cylindrical wire is increased water from 10°C to 40°C is [2004]
by 100%. Due to the consequent decrease in (a) 150 s (b) 100 s
diameter the change in the resistance of the wire (c) 50 s (d) 200 s
will be [2003] 18. The thermo emf of a thermocouple varies with
(a) 200% (b) 100%
(c) 50% (d) 300% the temperature q of the hot junction as
12. The total current supplied to the circuit by the E = aq + bq2 in volts where the ratio a/b is 700°C.
battery is [2004] If the cold junction is kept at 0°C, then the neutral
temperature is [2004]
2W (a) 1400°C
6V
6W 3W (b) 350°C
(c) 700°C
1.5W (d) No neutral temperature is possible for this
termocouple.
(a) 4 A (b) 2 A 19. The electrochemical equivalent of a metal is
(c) 1 A (d) 6 A 3.35109–7 kg per Coulomb. The mass of the metal
13. The resistance of the series combination of two liberated at the cathode when a 3A current is
resistances is S. when they are joined in parallel
passed for 2 seconds will be [2004]
the total resistance is P. If S = nP then the
(a) 6.6×1057kg (b) 9.9×10–7 kg
minimum possible value of n is [2004]
(a) 2 (b) 3 (c) 19.8×10–7 kg (d) 1.1×10–7 kg
(c) 4 (d) 1 20. Two thin, long, parallel wires, separated by a
14. An electric current is passed through a circuit distance ‘d’ carry a current of ‘i’ A in the same
containing two wires of the same material, direction. They will [2005]
connected in parallel. If the lengths and radii (a) repel each other with a force of
4 2 m 0i 2 /(2pd )
arein the ratio of and , then the ratio of the (b) attract each other with a force of
3 3
current passing through the wires will be [2004] m 0i 2 /(2pd )
(a) 8/9 (b) 1/3 (c) repel each other with a force of
(c) 3 (d) 2 m 0 i 2 /(2pd 2 )
15. In a meter bridge experiment null point is obtained (d) attract each other with a force of
at 20 cm. from one end of the wire when resistance m 0 i 2 /(2pd 2 )
X is balanced against another resistance Y. If X < 21. A heater coil is cut into two equal parts and only
Y, then where will be the new position of the null one part is now used in the heater. The heat
point from the same end, if one decides to balance generated will now be [2005]
a resistance of 4 X against Y [2004] (a) four times (b) doubled
(a) 40 cm (b) 80 cm (c) halved (d) one fourth
(c) 50 cm (d) 70 cm
22. In the circuit , the galvanometer G shows zero
16. The thermistors are usually made of [2004]
deflection. If the batteries A and B have
(a) metal oxides with high temperature
negligible internal resistance, the value of the
coefficient of resistivity
(b) metals with high temperature coefficient of resistor R will be - [2005]
resistivity
EBD_7764
www.crackjee.xyz
P-106 Physics
500 W (a) 0.5W (b) 1W
G
(c) 2W (d) 4W
2V 27. The resistance of hot tungsten filament is about
12V B R A 10 times the cold resistance. What will be the
resistance of 100 W and 200 V lamp when not in
use ? [2005]
(a) 100 W (b) 200W
(a) 20 W (b) 40 W
(c) 1000 W (d) 500 W
23. A moving coil galvanometer has 150 equal (c) 200 W (d) 400 W
divisions. Its current sensitivity is 10-divisions per 28. An energy source will supply a constant current
milliampere and voltage sensitivity is 2 divisions into the load if its internal resistance is [2005]
per millivolt. In order that each division reads 1 (a) very large as compared to the load
volt, the resistance in ohms needed to be resistance
connected in series with the coil will be - [2005] (b) equal to the resistance of the load
(a) 105 (b) 103 (c) non-zero but less than the resistance of
(c) 9995 (d) 99995
the load
24. Two sources of equal emf are connected to an
(d) zero
external resistance R. The internal resistance of
29. The Kirchhoff's first law (Si = 0) and second law
the two sources are R1and R2 (R1 > R1). If the
potential difference across the source having (SiR = SE), where the symbols have their usual
internal resistance R 2 is zero, then [2005] meanings, are respectively based on [2006]
(a) R = R2 - R1 (a) conservation of charge, conservation of
(b) R = R2 ´ ( R1 + R2 ) /( R2 - R1 ) momentum
(c) R = R1R2 /( R2 - R1 ) (b) conservation of energy, conservation of
(d) R = R1R2 /( R1 - R2 ) charge
(c) conservation of momentum, conservation
25. Two voltameters, one of copper and another of
of charge
silver, are joined in parallel. When a total charge
q flows through the voltameters, equal amount (d) conservation of charge, conservatrion of
of metals are deposited. If the electrochemical energy
equivalents of copper and silver are Z1 and Z 2 30. A material 'B' has twice the specific resistance of
respectively the charge which flows through the 'A'. A circular wire made of 'B' has twice the
silver voltameter is [2005] diameter of a wire made of 'A'. then for the two
q q wires to have the same resistance, the ratio lB/lA
(a) (b) of their respective lengths must be [2006]
Z Z
1+ 2 1+ 1 1
Z1 Z2 (a) 1 (b)
2
1
Z2 Z1 (c) (d) 2
(c) q (d) q 4
Z1 Z2 31. A thermocouple is made from two metals,
26. In a potentiometer experiment the balancing with Antimony and Bismuth. If one junction of the
a cell is at length 240 cm. On shunting the cell couple is kept hot and the other is kept cold,
with a resistance of 2W, the balancing length then, an electric current will [2006]
becomes 120 cm. The internal resistance of the (a) flow from Antimony to Bismuth at the hot
junction
cell is [2005] (b) flow from Bismuth to Antimony at the cold
junction
www.crackjee.xyz
Current Electricity P-107
(c) now flow through the thermocouple stored in the capacitor and the work done by
(d) flow from Antimony to Bismuth at the cold the battery will be [2007]
junction (a) 1/2 (b) 1
32. The current I drawn from the 5 volt source will be (c) 2 (d) 1/4
[2006] 37. The resistance of a wire is 5 ohm at 50°C and 6
10W ohm at 100°C. The resistance of the wire at 0°C
will be [2007]
5W 10W 20W
(a) 3 ohm (b) 2 ohm
(c) 1 ohm (d) 4 ohm
I 38. Shown in the figure below is a meter-bridge set
10W
up with null deflection in the galvanometer.
+–
55W R
5 volt
(a) 0.33 A (b) 0.5 A
(c) 0.67 A (d) 0.17 A G
33. The resistance of a bulb filmanet is 100W at a
temperature of 100°C. If its temperature 20 cm
coefficient of resistance be 0.005 per °C, its
resistance will become 200 W at a temperature of
[2006]
(a) 300°C (b) 400°C The value of the unknown resistor R is [2008]
(c) 500°C (d) 200°C (a) 13.75 W (b) 220 W
34. In a Wheatstone's bridge, three resistances P, Q
(c) 110 W (d) 55 W
and R connected in the three arms and the fourth
DIRECTIONS : Question No. 39 and 40 are based on
arm is formed by two resistances S1 and S2
the following paragraph.
connected in parallel. The condition for the
bridge to be balanced will be [2006] Consider a block of conducting material of
resistivity ‘r’ shown in the figure. Current ‘I’
P 2R P R ( S1 + S2 ) enters at ‘A’ and leaves from ‘D’. We apply
(a) = (b) =
Q S1 + S2 Q S1 S2 superposition principle to find voltage ‘DV’
developed between ‘B’ and ‘C’. The calculation
P R ( S1 + S2 ) P R is done in the following steps:
(c) = =
Q 2S1S2 (d) Q S1 + S2 (i) Take current ‘I’ entering from ‘A’ and
35. An electric bulb is rated 220 volt - 100 watt. The assume it to spread over a hemispherical
power consumed by it when operated on 110 surface in the block.
volt will be [2006] (ii) Calculate field E(r) at distance ‘r’ from A by
(a) 75 watt (b) 40 watt using Ohm’s law E = r j, where j is the
(c) 25 watt (d) 50 watt current per unit area at ‘r’.
36. A battery is used to charge a parallel plate (iii) From the ‘r’ dependence of E(r), obtain the
capacitor till the potential difference between potential V(r) at r.
the plates becomes equal to the electromotive (iv) Repeat (i), (ii) and (iii) for current ‘I’ leaving
force of the battery. The ratio of the energy ‘D’ and superpose results for ‘A’ and ‘D’.
EBD_7764
www.crackjee.xyz
P-108 Physics
one-fourth its initial value. Then the ratio t1/ t2
I DV I
will be [2010]
1
(a) 1 (b)
2
a b a 1
A B C D (c) (d) 2
4
43. Two conductors have the same resistance at 0°C
but their temperature coefficients of resistance
are a1 and a2. The respective temperature
coefficients of their series and parallel
39. DV measured between B and C is [2008] combinations are nearly [2010]
rI rI rI rI
(a) – (b) – a1 + a 2
pa p(a + b) a (a + b) (a) , a1 + a 2
2
rI rI rI
(c) – (d)
2pa 2p(a + b) 2p(a - b) a1 + a 2
(b) a1 + a 2 ,
40. For current entering at A, the electric field at a 2
distance ‘r’ from A is [2008]
a1a 2
rI rI (c) a1 + a 2 ,
(a) (b) a1 + a 2
8pr 2 r2
rI rI a1 + a 2 a1 + a 2
(d) ,
(c) 2 (d) 2 2 2
2pr 4pr
41. A 5V battery with internal resistance 2W and a 44. If a wire is stretched to make it 0.1% longer, its
2V battery with internal resistance 1W are resistance will : [2011]
connected to a 10W resistor as shown in the (a) increase by 0.2%
figure. [2008] (b) decrease by 0.2%
(c) decrease by 0.05%
P2
(d) increase by 0.05%
45. If 400 W of resistance is made by adding four
100 W resistances of tolerance 5%, then the
tolerance of the combination is [2011 RS]
5V (a) 5% (b) 10%
2V
2W 10W 1W (c) 15% (b) 20%
46. The current in the primary circuit of a
potentiometer is 0.2 A. The specific resistance
and cross-section of the potentiometer wire are
4 × 10–7 ohm metre and 8 × 10–7 m2, respectively.
The potential gradient will be equal to[2011 RS]
The current in the 10W resistor is
(a) 0.27 A P2 to P1 (b) 0.03 A P1 to P2 (a) 1 V /m (b) 0.5 V/m
(c) 0.03 A P2 to P1 (d) 0.27 A P1 to P2 (c) 0.1 V/m (d) 0.2 V/m
42. Let C be the capacitance of a capacitor 47. Two electric bulbs rated 25W – 220 V and 100W
discharging through a resistor R. Suppose t1 is – 220V are connected in series to a 440 V supply.
the time taken for the energy stored in the Which of the bulbs will fuse? [2012]
capacitor to reduce to half its initial value and t2 (a) Both (b) 100 W
is the time taken for the charge to reduce to (c) 25 W (d) Neither
www.crackjee.xyz
Current Electricity P-109
48. The supply voltage to room is 120V. The 6V
resistance of the lead wires is 6W. A 60 W bulb is P 2W
already switched on. What is the decrease of
voltage across the bulb, when a 240 W heater is 1W 9V
switched on in parallel to the bulb? [2013]
(a) zero (b) 2.9 Volt 3W W 3W
(c) 13.3 Volt (d) 10.04 Volt (a) 0.13 A, from Q to P
49. This questions has Statement I and Statement (b) 0.13 A, from P to Q
II. Of the four choices given after the Statements, (c) 1.3A from P to Q
choose the one that best describes into two (d) 0A
Statements. 53. The temperature dependence of resistances of
Statement-I : Higher the range, greater is the Cu and undoped Si in the temperature range
resistance of ammeter. 300-400 K, is best described by : [2016]
Statement-II : To increase the range of ammeter, (a) Linear increase for Cu, exponential
additional shunt needs to be used across it. decrease of Si.
[2013] (b) Linear decrease for Cu, linear decrease for
(a) Statement-I is true, Statement-II is true, Si.
Statement-II is the correct explanation of (c) Linear increase for Cu, linear increase for
Statement-I. Si.
(b) Statement-I is true, Statement-II is true, (d) Linear increase for Cu, exponential increase
Statement-II is not the correct explanation for Si.
of Statement-I. 54. Which of the following statements is false ?
(c) Statement-I is true, Statement-II is false. [2017]
(a) A rheostat can be used as a potential
(d) Statement-I is false, Statement-II is true.
divider
50. In a large building, there are 15 bulbs of 40 W, 5 (b) Kirchhoff's second law represents energy
bulbs of 100 W, 5 fans of 80 W and 1 heater of 1 conservation
kW. The voltage of electric mains is 220 V. The (c) Wheatstone bridge is the most sensitive
minimum capacity of the main fuse of the when all the four resistances are of the
building will be: [2014] same order of magnitude
(a) 8 A (b) 10 A (d) In a balanced wheatstone bridge if the cell
and the galvanometer are exchanged, the
(c) 12 A (d) 14 A
null point is disturbed.
51. When 5V potential difference is applied across
a wire of length 0.1 m, the drift speed of electrons 55. [2017]
is 2.5 × 10–4 ms–1. If the electron density in the
wire is 8 × 1028 m–3, the resistivity of the material
is close to : [2015]
(a) 1.6 × 10–6 Wm (b) 1.6 × 10–5 Wm
In the above circuit the current in each resistance
(c) 1.6 × 10–8 Wm (d) 1.6 × 10–7 Wm is
52. In the circuit shown, the current in the 1W (a) 0.5A (b) 0 A
resistor is : [2015] (c) 1 A (d) 0.25 A
EBD_7764
www.crackjee.xyz
P-110 Physics
Answer Key
1 2 3 4 5 6 7 8 9 10 11 12 13 14 15
(c) (b) (b) (b) (d) (a) (c) (d) (b) (c) (d) (a) (c) (b) (c)
16 17 18 19 20 21 22 23 24 25 26 27 28 29 30
(a) (a) (d) (c) (b) (b) (a) (c) (a) (a) (c) (b) (d) (d) (d)
31 32 33 34 35 36 37 38 39 40 41 42 43 44 45
(d) (b) (b) (b) (c) (a) (d) (b) (a) (c) (c) (c) (d) (a) (a)
46 47 48 49 50 51 52 53 54 55
(c) (c) (d) (d) (c) (b) (a) (a) (d) (b)

1. (c) To convert a galvanometer into a voltmeter 15 ´ 15


we connect a high resistance in series with \ 15 0 = R
the galvanometer. eq
The same procedure needs to be done if
ammeter is to be used as a voltmeter. 15 3
\ Req = =
10 2
V2
2. (b) Case 1 P1 = 2R 3
R Þ = Þ 4 R = 6 + 3R Þ R = 6W
R 2+ R 2
4. (b) According to Faraday's first law of
electrolysis
m = ZIt Þ m µ It
V 5. (d) From the principle of potentiometer, V µ l
Case 2
The wire is cut into two equal pieces. V l
Þ = ; where
Therefore, the resistance of the individual E L
R V = emf of battery, E = emf of standard cell.
wire is . These are connected in parallel L = length of potentiometer wire
2
R/2 R El 30E
\ Req = = V= =
2 4 L 100
R/2 R/4 E
R/2
=
i
V V i
V2 æ V2 ö
P =
\ 2 R/4 = 4 ç ÷ = 4P1 r
è R ø
E'
2´R
3. (b) The equivalent resistance is Req = NOTE In this arrangement, the internal
2+ R
V2 resistance of the battery E does not play
\ Power dissipation P = any role as current is not passing through
Re q the battery.
www.crackjee.xyz
Current Electricity P-111
6. (a) Let q be the smallest temperature 11. (d) The total volume remains the same before
difference that can be detected by the and after stretching.
thermocouple, then Therefore A ´ l = A ' ´ l '
I × R = (25 × 10–6) q
Here l ' = 2l
where I is the smallest current which can
be detected by the galvanometer of A´l A´l A
\ A' = = =
resistance R. l' 2l 2
Here I = 10–5 A, R = 40 W Percentage change in resistance
\ 10–5 × 40 = 25 × 10 – 6 × q l' l
R f - Ri r -
\ q = 16°C.
= ´ 100 = A ' A ´ 100
7. (c) According to Faraday’s first law of Ri l
electrolysis r
A
m = Z×q
For same q, m µ Z éæ l ' A ö ù
= êç ´ ÷ - 1ú ´ 100
mCu ZCu ëè A ' l ø û
\ m =Z é æ 2l A ö ù
Zn Zn
= êç ´ ÷ - 1ú ´ 100
êëè 2 l ø úû
A
Z Cu
Þ mCu = ´ mZn
Z Zn = 300%
6V
31.5
= ´ 0.13 = 0.126 g
32.5 2W
8. (d) ig × G = (i – ig) S 1.5W
12. (a)
ig ´ G 1 ´ 0.81
\ S= = = 0.09W 6W 3W
i - ig 10 - 1
6V 6V
3 ´ 6 18
9. (b) Rp = = = 2W
3+ 6 9 3/2W 3/2W 3W
V 3
\ V = IR ÞI = = = 1.5 A 3W 3W
R 2
6
3W 3W hence Req = 3/2; \I = = 4A
3V 3/ 2
13. (c) R1 R2
Resistance of the series combination,
3W S = R1 + R2
Resistance of the parallel combination,
I
R1 R2
3V 6W 3V P=
3W 2W R1 + R2
n( R1R2 )
S = nP Þ R1 + R2 =
2 ( R1 + R2 )
Vrated (220)2
10. (c) We know that R = =
Prated 1000 Þ ( R1 + R2 )2 = nR1 R2
When this bulb is connected to 110 volt Minimum value of n is 4 for that
mains supply we get
( R1 + R2 )2 = 4 R1 R2 Þ ( R1 - R2 )2 = 0
V 2 (110) 2 ´ 1000 1000
P= = = = 250W
R (220)2 4
EBD_7764
www.crackjee.xyz
P-112 Physics
R1 i1 F m 0 i1 i2 m 0 i 2
= =
l 2 pd 2 pd
R2 i2 (attractive as current is in the same direction)
14. (b)
V 2t
V 21. (b) H =
R
rl1 rl
R1 = ; R2 = 2 R
pr12 pr22 Resistance of half the coil =
2
i1R1 = i2R2 (same potential difference) \ As R reduces to half, ‘H’ will be doubled.
i1 R2 l 2 r12 3 4 1 500 W
\ = = ´ = ´ = A
i2 R1 l1 r22 4 9 3 22. (a)
i
R1 l1 2V
= R
15. (c) R2 l 2 where l2 = 100 – l1 12V

In the first case X = 20


Y 80 10 1
In the second case 12 – 2 = (500W)i Þ i = =
4X l 500 50
= Þ l = 50 12 1
Y 100 - l Again, i = =
16. (a) Thermistors are usually made of 500 + R 50
metaloxides with high temperature
coefficient of resistivity. Þ 500 + R = 600
Þ R = 100 W
17. (a) DQ = mC p ´ DT 23. (c) Resistance of Galvanometer,
= 1 × 4180 × (40 – 10) = 4180 × 30
( \ DQ = heat supplied in time t for heating Current sensitivity 10
G= Þ G= = 5W
1L water from 10°C to 40°C) Voltage sensitivity 2
4180 ´ 30 Here ig = Full scale deflection current =
also DQ = 836 ´ t Þ t = = 150s
836 150
18. (d) Neutral temperature is the temperature of = 15 mA
a hot junction at which E is maximum. 10
dE V = voltage to be measured = 150 volts
Þ = 0 or a + 2bq = 0 (such that each division reads 1 volt)
dq
-a 150
Þq = = -350 Þ R= - 5 = 9995W
2b 15 ´10 -3
d2E 24. (a)
Þ 2
= 2b R1 R2
I
dq
hence no q is possible for E to be maximum
no neutral temperature is possible.
19. (c) The mass liberated m and electrochemical
equivalent of a metal Z, are related as m = Zit
Þ m = 3.3 ´ 10 -7 ´ 3 ´ 2 = 19.8 ´ 10 -7 kg
20. (b) i i R
F
2e
I=
R + R1 + R 2
Potential difference across second cell
www.crackjee.xyz
Current Electricity P-113

= V = e - iR 2 = 0 30. (d) rB = 2rA


dB = 2dA
2e r B l B r Al A
e – .R2 = 0
R + R1 + R 2 RB = RA Þ A = A
B A
R + R1 + R 2 - 2R 2 = 0
l B r A d B2 r 4d 2
R + R1 - R 2 = 0 \ = ´ = A ´ A =2
l A rB d A2 2r A d A2
\ R = R 2 - R1
31. (d) At cold junction, current flows from
25. (a) Mass deposited
Antimony to Bismuth (because current
m = Zq
flows from metal occurring later in the series
1 Þ Z1 = q2 to metal occurring earlier in the
Þ Zµ .... (i) thermoelectric series).
q Z 2 q1
32. (b) The network of resistors is a balanced
Also q = q1 + q2 .... (ii) wheatstone bridge. The equivalent circuit
is
q q
Þ = 1 +1 10W 20W
q2 q2
30W
(Dividing (ii) by q2 )
q
Þ q2 = q
.... (iii) 15W
1+ 1 5W 10W
q2
5V 5V
From equation (i) and (iii),
q 10W
q2 =
Z
1+ 2
Z1
26. (c) The internal resistance of the cell,
5V
æl -l ö 240 - 120 15 ´ 30
r= ç 1 2÷ ´R = ´ 2 = 2W Req = = 10 W
è l2 ø 120 15 + 30
V 5
V2 Þ I= = = 0.5 A
27. (b) P = Vi = R 10
R 33. (b) R1 = R0 [1 + a × 100] = 100 ....(1)
V 2 200 ´ 200 R2 = R0 [1 + a × T] = 200 ....(2)
Rhot = = = 400 W On dividing we get
P 100
400 200 1 + aT 1 + 0.005 T
Rcold = = 40 W = Þ2=
100 1 + 100a 1 + 100 ´ 0.005
10
Þ T = 400°C
E
28. (d) I = , Internal resistance (r) is NOTE We may use this expression as
R+r
an approximation because the difference
E
zero, I = = constant. in the answers is appreciable. For accurate
R results one should use
29. (d) NOTE Kirchhoff's first law is based on R = R0eaDT
conservation of charge and Kirchhoff's P R SS
34. (b) = where S = 1 2
second law is based on conservation of Q S S1 + S 2
energy. 35. (c) The resistance of the bulb is
V 2 (220) 2
R= =
P 100
EBD_7764
www.crackjee.xyz
P-114 Physics
The power consumed when operated at rI é 1 ù
a
rI rI
110 V is =- ê - ú = -
2 p ë r û a + b 2 pa 2 p ( a + b )
(110)2 100
P= = = 25 W On applying superposition as mentioned
2 4
(220) /100 we get
36. (a) Required ratio ' rI rI
1 DVBC = 2 ´ DVBC = -
2 pa p ( a + b )
Energy stored in capacitor 2 CV
= = rI
Workdone by the battery Ce 2 40. (c) As shown in Answer (a) E =
2pr 2
where C = Capacitance of capacitor
41. (c) Applying Kirchoff's loop law in AB P2P1A
V = Potential difference, , we get
e = emf of battery
-2i + 5 - 10 i1 = 0 .....(i)
1 2
Ce 1
( Q V = e)
i P2 i–i1
= 2 = B C
Ce 2 2 i1
37. (d) We know that 10W 2V
Rt = R0 (1 + at ), 5V 1W
where Rt is the resistance of the wire at t 2W
ºC,
R0 is the resistance of the wire at 0ºC A P1 D
and a is the temperature coefficient of Again applying Kirchoff's loop law in P2
resistance. CDP1P2 we get, 10 i1 + 2 – i + i1= 0 ....(ii)
Þ R50 = R0 (1 + 50 a ) ... (i)
é 5 - 10i1 ù
R100 = R0 (1 + 100 a ) ... (ii) From (i) and (ii) 11 i1 + 2 - ê ú=0
ë 2 û
From (i), R50 – R0 = 50 aR0 ... (iii)
1
From (ii), R100 – R0 = 100 aR0 ... (iv) Þ i1 = A from P2 to P1
32
Dividing (iii) by (iv), we get
q2
R50 - R0 1 42. (c) Initial energy of capacitor, E1 = 1
= 2C
R100 - R0 2 Final energy of capacitor,
Here, R50 = 5W and R100 = 6W 2
æ q1 ö
5 - R0 1 1 q12
\ = E2 = E1 = =ç 2÷
6 - R0 2 2 4C ç ÷
è 2C ø
or, 6 – R0 = 10 – 2 R0 or, R0 = 4W . 1
38. (b) According to the condition of balancing \ t1 = time for the charge to reduce to
2
55 R of its initial value
= Þ R = 220W
20 80 and t2 = time for the charge to reduce to
39. (a) Let j be the current density. 1
I of its initial value
Then j ´ 2pr 2 = I Þ j = 4
2 pr 2 We have, q2 = q1e-t / CR
rI
\ E = rj = æ q2 ö t
2pr 2 Þ ln ç q ÷ = - CR
a r uur a
è 1ø
rI
Now, DVBC '
= - ò E. dr = - ò dr æ 1 ö -t1
a + b 2 pr
2 \ ln ç ÷= ...(1)
a +b è 2 ø CR
www.crackjee.xyz
Current Electricity P-115
46. (c) Potential gradient
æ 1 ö -t2
and ln ç ÷ = ...(2) V IR I æ rl ö I r
è 4 ø CR x= = = ç ÷=
l l lè A ø A
æ 1 ö
ln ç ÷
t 0.2 ´ 4 ´ 10-70.8
By (1) and (2) , 1 = è 2 ø x= -7
=
= 0.1 V/m
8 ´ 10 8
ln æç ö÷
t2 1
47. (c) The current upto which bulb rated 25W –
è4ø 220V, will not fuse
ln æ ö
1 W1 25
1 çè 2 ÷ø 1 I1 = V = 220 Amp
= = 1
2 æ
2ln ç ÷
1ö 4
W2 100
è2ø Similarly, I2 = V = 220 Amp
43. (d) R1 = R0 [1 + a1D t ] ; 2
The current flowing through the circuit
R2 = R0 [1 + a2 D t ] B1 B2
In Series, R = R1 + R2
= R0 [ 2 + (a1 + a 2 )D t ]
é æ a1 + a 2 ö ù R1 r2
= 2R0 ê1 + çè ÷Dt
ë 2 ø úû
a1 + a 2
\ a eq = 440V
2
1 1 1 440
= + I=
In Parallel , Reff
R R1 R2
1 1 Reff = R1 + R2
= R 1 + a D t + R 1+ a D t
0[ 1 ] 0[ 2 ] V12 (220)2 V 2 (220)2
R1 = = ; R2 = 2 =
1 P1 25 P 100
Þ
R0
(1 + a eq D t ) 440
2 I=
(220) 2 (220) 2
1 1 +
= + 25 100
R0 (1 + a1D t ) R0 (1 + a 2 D t )
440
2(1 - a eq Dt ) = (1 - a1Dt )(1 - a 2 Dt ) =
2é 1 1 ù ; I = 40 Amp
(220) ê +
a1 + a 2 ë 25 100 úû 220
\ a eq =
2
25 ö æ 40 ö < I æ = 100 A ö
44. (a) Resistance of wire Q I1 æç = A÷ < I ç = A ÷ 2 ç 200 ÷
è 220 ø è 220 ø è ø
rl rl 2
R= = (where Al = C) Thus the bulb rated 25 W–220 will fuse.
A C
\ Fractional change in resistance
DR Dl 6W
=2
R l Bulb
\ Resistance will increase by 0.2% (Lead)
45. (a) 48. (d)
R R R R
R = 100 ± 5
Þ 4R = 400 ± 20
Thus, tolerance of combination is also 5%. 120 V
Power of bulb = 60 W (given)
EBD_7764
www.crackjee.xyz
P-116 Physics
120 ´ 120 6V
Resistance of bulb = = 240W P 2W
60
é V2 ù
êQ P = ú 9V
ë R û
1W
Power of heater = 240W (given)
120 ´120 3W q 4W
Resistance of heater = = 60W
240 6 = 3 I1 + I1 – I2 ; 4I1 – I2 = 6 ...(1)
Voltage across bulb before heater is – 9 + 2I2 – (I1 – I2) + 3I2 = 0
switched on,
– I1 + 6I2 = 9 ...(2)
240
V1 = ´ 120 = 117.73 volt On solving (1) and (2)
246 I1 = 0.13A
Voltage across bulb after heater is
switched on, Direction Q to P, since I1 > I2.
48 r P
V2 = ´120 = 106.66 volt 53. (a)
54
Hence decrease in voltage
V1 – V2 = 117.073 – 106.66 = 10.04 Volt
(approximately) T
49. (d) Statements I is false and Statement II is true T
Metal (for limited Semiconductor
IgG range of temperature)
–Eg

For ammeter, shunt resistance, S =


KT B

I – Ig 54. (d) There is no change in null point, if the cell


Therefore for I to increase, S should and the galvanometer are exchanged in a
decrease, So additional S can be connected balanced wheatstone bridge.
across it.
50. (c) Total power consumed by electrical
appliances in the building, Ptotal = 2500W
Watt = Volt × ampere
Þ 2500 = V × I Þ 2500 = 220 I
2500
Þ I= = 11.36 » 12A
220
(Minimum capacity of main fuse)
l
51. (b) V = IR = (neAvd )r
A R1 R2
V
On balancing condition =
R3 R4
\ r=
Vd lne
After exchange
Here V = potential difference
l = length of wire
n = no. of electrons per unit volume of
conductor.
e = no. of electrons
Placing the value of above parameters we get
resistivity
5
r= -19
8 ´ 10 28
´ 1.6 ´ 10 ´ 2.5 ´ 10-4 ´ 0.1
= 1.6 × 10–5Wm
52. (a) From KVL R1 R3
On balancing condition = `
– 6 + 3I1 + 1 (Ii – I2) = 0 R2 R
55. (b) The potential difference in each loop is zero.
\ No current will flow or current in each
resistance is Zero.
www.crackjee.xyz

Moving Charges
and Magnetism 18
1. If a current is passed through a spring then the 5. The time period of a charged particle undergoing
spring will [2002] a circular motion in a uniform magnetic field is
(a) expand (b) compress independent of its [2002]
(c) remains same (d) none of these (a) speed (b) mass
2. If in a circular coil A of radius R, current I is (c) charge (d) magnetic induction
flowing and in another coil B of radius 2R a 6. A particle of mass M and charge Q moving with
current 2I is flowing, then the ratio of the r
velocity v describe a circular path of radius R
magnetic fields BA and BB, produced by them
when subjected to a uniform transverse magnetic
will be [2002]
field of induction B. The work done by the field
(a) 1 (b) 2
when the particle completes one full circle is
(c) 1/2 (d) 4
[2003]
3. If an electron and a proton having same momenta
enter perpendicular to a magnetic field, then æ Mv 2 ö
[2002] (a) ç ÷ 2pR (b) zero
(a) curved path of electron and proton will be è R ø
same (ignoring the sense of revolution) (c) B Q 2p R (d) B Qv2p R
7. -18
A particle of charge - 16 ´ 10 coulomb moving
(b) they will move undeflected
(c) curved path of electron is more curved than with velocity 10 ms -1 along the x-axis enters a
that of the proton region where a magnetic field of induction B is
(d) path of proton is more curved. along the y-axis, and an electric field of
4. Wires 1 and 2 carrying currents i 1 and i 2 magnitude 10 4 V / m is along the negative
respectively are inclined at an angle θ to each z-axis. If the charged particle continues moving
other. What is the force on a small element dl of along the x-axis, the magnitude of B is [2003]
wire 2 at a distance of r from wire 1 (as shown in (a) 103 Wb/m 2 (b) 105 Wb/m 2
figure) due to the magnetic field of wire 1? (c) 1016 Wb/m 2 (d) 10-3 Wb/m 2
8. A current i ampere flows along an infinitely long
[2002]
straight thin walled tube, then the magnetic
1 2
induction at any point inside the tube is[2004]
i1
m 0 2i
r i2 (a) . tesla (b) zero
4p r
q dl 2i
(c) infinite (d) tesla
r
m0 m0 9. A long wire carries a steady current. It is bent
(a) i1i2 dl tan q (b) i1i2 dl sin q into a circle of one turn and the magnetic field at
2pr 2pr
the centre of the coil is B. It is then bent into a
m0 m0 circular loop of n turns. The magnetic field at
(c) i1i2 dl cos q (d) i1i2 dl sin q
2pr 4 pr the centre of the coil will be [2004]
EBD_7764
w w w . c r a c k j e e . x y z
P-118 Physics
(a) 2n B (b) n2 B 15. In a region, steady and uniform electric and
(c) nB (d) 2 n2 B magnetic fields are present. These two fields are
10. The magnetic field due to a current carrying parallel to each other. A charged particle is
circular loop of radius 3 cm at a point on the axis released from rest in this region. The path of the
at a distance of 4 cm from the centre is 54 mT. particle will be a [2006]
What will be its value at the centre of loop?
(a) helix (b) straight line
[2004]
(c) ellipse (d) circle
(a) 125 mT (b) 150 mT
(c) 250 mT (d) 75 mT 16. A long solenoid has 200 turns per cm and carries
11. Two long conductors, separated by a distance a current i. The magnetic field at its centre is
d carry current I1 and I2 in the same direction. 6.28 × 10–2 Weber/m2. Another long solenoid
They exert a force F on each other. Now the i
has 100 turns per cm and it carries a current .
current in one of them is increased to two times 3
and its direction is reversed. The distance is also The value of the magnetic field at its centre is
increased to 3d. The new value of the force [2006]
between them is [2004] (a) 1.05 × 10–2 Weber/m2
2F F
(a) - (b) (b) 1.05 × 10–5 Weber/m2
3 3
F (c) 1.05 × 10–3 Weber/m2
(c) –2 F (d) -
3 (d) 1.05 × 10–4 Weber/m2
12. Two concentric coils each of radius equal to 17. A long straight wire of radius a carries a steady
2 p cm are placed at right angles to each other. 3 current i. The current is uniformly distributed
ampere and 4 ampere are the currents flowing in across its cross section. The ratio of the
each coil respectively. The magnetic induction magnetic field at a/2 and 2a is [2007]
in Weber/m2 at the centre of the coils will be
( m0 = 4p ´ 10-7 Wb / A.m ) [2005]
(a) 1/2
(c) 4
(b) 1/4
(d) 1
-5
(a) 10 (b) 12 ´ 10 -5 18. A current I flows along the length of an infinitely
-5
(c) 7 ´ 10 (d) 5 ´ 10 -5 long, straight, thin walled pipe. Then [2007]
13. A charged particle of mass m and charge q travels (a) the magnetic field at all points inside the
on a circular path of radius r that is perpendicular pipe is the same, but not zero
to a magnetic field B. The time taken by the
(b) the magnetic field is zero only on the axis
particle to complete one revolution is [2005]
of the pipe
2pq 2 B 2 pmq
(a) (b) (c) the magnetic field is different at different
m B
points inside the pipe
2pm 2pqB
(c) (d) (d) the magnetic field at any point inside the
qB m
14. A uniform electric field and a uniform magnetic pipe is zero
field are acting along the same direction in a 19. A charged particle with charge q enters a region
certain region. If an electron is projected along of constant, uniform and mutually orthogonal
ur ur r
the direction of the fields with a certain velocity fields E urand B uwith
r a velocity v perpendicular
then [2005] to both E and B , and comes out without any
r
(a) its velocity will increase change in magnitude or direction of v . Then
(b) Its velocity will decrease [2007]
(c) it will turn towards left of direction of r ur ur r ur ur
motion (a) v = B ´ E / E 2 (b) v = E ´ B / B 2
(d) it will turn towards right of direction of motion r ur ur r ur ur
(c) v = B ´ E / B 2 (d) v = E ´ B / E 2
www.crackjee.xyz
Moving Charges and Magnetism P-119
20. A charged particle moves through a magnetic B
field perpendicular to its direction. Then[2007] a A
(a) kinetic energy changes but the momentum
is constant I1 30° I
(b) the momentum changes but the kinetic O
energy is constant
D
(c) both momentum and kinetic energy of the
b C
particle are not constant
(d) both momentum and kinetic energy of the 23. The magnitude of the magnetic field (B) due to
particle are constant the loop ABCD at the origin (O) is :
21. Two identical conducting wires AOB and COD m o I (b - a )
are placed at right angles to each other. The wire (a)
24 ab
AOB carries an electric current I1 and COD
mo I é b - a ù
carries a current I2. The magnetic field on a point (b)
lying at a distance d from O, in a direction 4p êë ab úû
perpendicular to the plane of the wires AOB and mo I
COD, will be given by [2007] (c) [2(b - a) + p / 3(a + b)]
4p
m0 (d) zero
(a) ( I12 + I 2 2 ) 24. Due to the presence of the current I1 at the
2 pd 1 origin:
m 0 æ I1 + I 2 ö 2
(b) ç ÷ (a) The forces on AD and BC are zero.
2p è d ø (b) The magnitude of the net force on the loop
1
(c)
m0
(
2pd 1
)
I 2 + I 22 2 I I
is given by 1 mo [2(b - a) + p / 3(a + b] .
4p
m0 (c) The magnitude of the net force on the loop
(d)
2pd
( I1 + I 2 )
m o II1
22. A horizontal overhead powerline is at height of is given by (b - a).
24ab
4m from the ground and carries a current of 100A (d) The forces on AB and DC are zero.
from east to west. The magnetic field directly 25. Two long parallel wires are at a distance 2d apart.
below it on the ground is (m0 = 4p×10 –7 Tm A–1) They carry steady equal currents flowing out of
[2008] the plane of the paper as shown. The variation
(a) 2.5 × 10–7 T southward of the magnetic field B along the line XX' is given
(b) 5 × 10–6 T northward by [2010]
(c) 5 × 10–6 T southward B
(d) 2.5 × 10–7 T northward
Directions : Question numbers 23 and 24 are based
on the following paragraph. (a) X X¢
A current loop ABCD is held fixed on the plane of the
paper as shown in the figure. The arcs BC (radius = b) d d
and DA (radius = a) of the loop are joined by two
B
straight wires AB and CD. A steady current I is flowing
in the loop. Angle made by AB and CD at the origin O
is 30°. Another straight thin wire with steady current (b) X X¢
I1 flowing out of the plane of the paper is kept at the
origin. [2009] d d
EBD_7764
www.crackjee.xyz
P-120 Physics

B (a) ra = rp = rd (b) ra = rp < rd


(c) ra > rd > rp (d) ra = rd > rp
(c) X X¢ 30. A circular loop of radius 0.3 cm lies parallel to a
much bigger circular loop of radius 20 cm. The
d d centre of the small loop is on the axis of the
bigger loop. The distance between their centres
is 15 cm. If a current of 2.0 A flows through the
B
smaller loop, then the flux linked with bigger loop
is [2013]
(d) X X¢ (a) 9.1 × 10–11 weber
(b) 6 × 10–11 weber
d d
(c) 3.3 × 10–11 weber
(d) 6.6 × 10–9 weber
26. A current I flows in an infinitely long wire with 31. A conductor lies along the z-axis at
cross section in the form of a semi-circular ring -1.5 £ z < 1.5 m and carries a fixed current of
of radius R. The magnitude of the magnetic 10.0 A in -â z direction (see figure). For a field
induction along its axis is: [2011] r
B = 3.0 ´10-4 e -0.2x aˆ y T, fin d the power
m0 I m0 I
(a) (b) required to move the conductor at constant
2p 2 R 2pR
speed to x = 2.0 m, y = 0 m in 5 ´10-3 s. Assume
m0 I m0 I
(c) (d) parallel motion along the x-axis. [2014]
4p R p2 R
27. An electric charge +q moves with velocity z
r
v = 3iˆ + 4 ˆj + kˆ in an electromagnetic field 1.5
ur ur
given by E = 3i$ + $j + 2k$ and B = iˆ + ˆj - 3kˆ
I

The y - component of the force experienced by +


q is : [2011 RS] B y
(a) 11 q (b) 5 q 2.0
(c) 3 q (d) 2 q
28. A thin circular disc of radius R is uniformly x
charged with density s > 0 per unit area. The –1.5
disc rotates about its axis with a uniform angular
speed w.The magnetic moment of the disc is
(a) 1.57 W (b) 2.97 W
[2011 RS]
(c) 14.85 W (d) 29.7 W
pR 4 32. A rectangular loop of sides 10 cm and 5 cm
(a) pR 4 sw (b) sw
2 carrying a current 1 of 12 A is placed in different
orientations as shown in the figures below :
pR 4
(c) sw (d) 2pR 4 sw [2015]
4 z
29. Proton, deuteron and alpha particle of same
kinetic energy are moving in circular trajectories I
B
in a constant magnetic field. The radii of proton,
I I
deuteron and alpha particle are respectively rp, (A) y
rd and ra. Which one of the following relation is I
correct? [2012] x
www.crackjee.xyz
Moving Charges and Magnetism P-121
uur uur
z (a) F1 is radially inwards and F2 = 0
uur uur
B (b) F1 is radially outwards and F2 = 0
uur uur
(B) I (c) F1 = F2 = 0
I y uur uur
I (d) F1 is radially inwards and F2 is radially
x I outwards
z 34. Two identical wires A and B, each of length 'l',
carry the same current I. Wire A is bent into a
I B circle of radius R and wire B is bent to form a
I square of side 'a'. If BA and BB are the values of
(C) I y magnetic field at the centres of the circle and
I BA
x square respectively, then the ratio is:
BB
z [2016]
p2 p2
B (a) (b)
16 8 2
(D) I
I y p 2 p2
I (c) (d)
x I 8 16 2
35. A galvanometer having a coil resistance of 100
If there is a uniform magnetic field of 0.3 T in the W gives a full scale deflection, when a currect of
positive z direction, in which orientations the 1 mA is passed through it. The value of the
loop would be in (i) stable equilibrium and (ii) resistance, which can convert this galvanometer
unstable equilibrium ? into ammeter giving a full scale deflection for a
(a) (B) and (D), respectively current of 10 A, is : [2016]
(b) (B) and (C), respectively (a) 0.1 W (b) 3W
(c) (A) and (B), respectively (c) 0.01W (d) 2W
(d) (A) and (C), respectively 36. When a current of 5 mA is passed through a
33. Two coaxial solenoids of different radius carry galvanometer having a coil of resistance 15W, it
uur
current I in the same direction. F1 be the shows full scale deflection. The value of the
magnetic force on the inner solenoid due to the resistance to be put in series with th e
uur galvanometer to convert it into to voltmeter of
outer one and F2 be the magnetic force on the range 0 - 10 V is [2017]
outer solenoid due to the inner one. Then : (a) 2.535× 103 W (b) 4.005 × 103 W
[2015] (c) 1.985 × 103 W (d) 2.045 × 103 W

Answer Key
1 2 3 4 5 6 7 8 9 10 11 12 13 14 15
(b) (a) (a) (c) (a) (b) (a) (b) (b) (c) (a) (d) (c) (b) (b)
16 17 18 19 20 21 22 23 24 25 26 27 28 29 30
(a) (d) (d) (b) (b) (c) (c) (a) (d) (a) (d) (a) (c) (b) (a)
31 32 33 34 35 36
(b) (a) (c) (b) (c) (c)
EBD_7764
www.crackjee.xyz
P-122 Physics

1. (b) When current is passed through a spring The force exerted due to this magnetic field
then current flows parallel in the adjacent on current element i2 dl is
turns. dF = i2 dl B sin 90°
NOTE When two wires are placed \ dF = i2 dl
parallel to each other and current flows in é m 0 i1 cos q ù m0
the same direction, the wires attract each ê 2 p r ú = 2 pr i1 i2 dl cos q
ë û
other.
5. (a) The time period of a charged particle (m, q)
Similarly, here the various turns attract each
other and the spring will compress. moving in a magnetic field (B) is T = 2pm
qB
The time period does not depend on the
2. (a) We know that the magnetic field produced speed of the particle.
by a current carrying circular coil of radius 6. (b) The workdone, dW = Fds cosq
m0 I The angle between force and
r at its centre is B = ´ 2p displacement is 90° . Therefore work
4p r done is zero.
m I × × ×
Here B A = 0 ´ 2p
4p R
m0 2I BA
and BB = ´ 2p Þ =1 × × ×
4p 2 R BB
3. (a) When a charged particle enters F
perpendicular to a magnetic field, then it
moves in a circular path of radius. × ×
S
p 7. (a) The situation is shown in the figure.
r=
qB FE= Force due to electric field
where q = Charge of the particle FB = Force due to magnetic field
p = Momentum of the particle It is given that the charged particle remains
B = Magnetic field moving along X-axis (i.e. undeviated).
Here p, q and B are constant for electron Therefore FB = FE
and proton, therefore the radius will be same. Þ qvB = qE
4. (c) Magnetic field due to current in wire 1 at
point P distant r from the wire is E 104
ÞB= = = 103 weber/m2
m i
B = 0 1 [ cos q + cos q] v
Y
10
4p r
Z'

q i2
i1 B
r P
dl E
FB
q X
v
E
m 0 i1 cos q
B= (directed perpendicular to
2p r Z
the plane of paper, inwards)
www.crackjee.xyz
Moving Charges and Magnetism P-123
8. (b) Using Ampere’s law at a distance r from 2
axis, B is same from symmetry. m0 i2 m0 ´ 4 ´ 10
r uur B2 = =
-2 4p
2(2p ´ 10 )
ò B.dl = m0i i.e., B ´ 2pr = m 0i
Here i is zero, for r < R, whereas R is the m
B= B12 + B22 = 0 . 5 × 102
radius 4p
\ B=0
9. (b) Magentic field at the centre of a circular Þ B = 10 - 7 ´ 5 ´ 10 2
coil of radius R carrying current i is Þ B = 5 × 10–5 Wb / m2
m i
13. (c) Equating magnetic force to centripetal
B= 0 force,
2R
mv 2
Given : n ´ (2pr ') = 2pR = qvB sin 90º
Þ nr ' = R ...(1) r
Time to complete one revolution,
n.m 0i 2pr 2pm
B' = ...(2) =
2r ' T=
v qB
nm 0 i.n 14. (b) Due to electric field, it experiences force
From (1) and (2), B ' = = n2 B
2pR and accelerates i.e. its velocity decreases.
10. (c) The magnetic field at a point on the axis of a 15. (b) The charged particle will move along the
circular loop at a distance x from centre is, lines of electric field (and magnetic field).
m 0i a 2 Magnetic field will exert no force. The force
B= by electric field will be along the lines of
2( x 2 + a 2 )3 / 2 uniform electric field. Hence the particle will
m i move in a straight line.
B' = 0 B2 m0 n2i2
2a =
16. (a) B1 m 0 n1i1
B.( x 2 + a2 )3/ 2 i
\ B' = 100 ´
a3 B2 3
Þ =
54(53 ) 6.28 ´ 10 -2 200 ´ i
Put x = 4 & a = 3 Þ B ' = = 250 µT
3´3´ 3 6.28 ´ 10 -2
11. (a) Force between two long conductor carrying Þ B2 = = 1.05 ´ 10 -2 Wb/m 2
current, 6
m 2I I 17. (d) Here, current is uniformly distributed
F = 0 1 2 ´l across the cross-section of the wire,
4p d therefore, current enclosed in the amperean
µ 2(2 I1 ) I 2
F'=- 0 l æ aö
4p 3d path formed at a distance r1 ç = ÷
è 2ø
F ' -2
\ =
F 3
12. (d) (1)
a/2
P1 P2

(2)

The magnetic field due to circular coil, B1 æ p r2 ö


2 = ç 12 ÷ ´ I , where I is total current
µ i m 0 i1 m ´ 3 ´ 10 èpa ø
= 01 = = 0
2r -2 4p
2 (2 p ´ 10 ) \ Magnetic field at P1 is
EBD_7764
www.crackjee.xyz
P-124 Physics
m0 ´ current enclosed P
B1 = A D
Path
B1 B2
æ p r2 ö
m 0 ´ ç 12 ÷ ´ I
è pa ø m ´ I r1 I1 d I2
Þ B1 = = 0
2p r1 2p a 2
Now, magnetic field at point P2, O
m0 I m I
B2 = . = 0 .
2p (2a) 4pa
B1 m 0 Ir1 4pa C B
\ Required ratio = = ´
B2 2pa 2 m 0 I
\ Resultant field, B = B12 + B22
a
2 r1 2 ´ 2 But B1 =
m 0 I1 m I
and B2 = 0 2
= = = 1. 2 pd 2 pd
a a
18. (d) There is no current inside the pipe. 2
ur uur
Therefore oò B . d l = µo I
æ m ö
\ B= ç 0 ÷
è 2pd ø (I
2
1 + I 22 )
I=0\ B=0 m0 2
( )
1/ 2
r r or, B = I1 + I 22
19. (b) Here, E and B are perpendicular to each 2 pd
r 22. (c) The magnetic field is
other and the velocity v does not change;
therefore m0 2I 2 ´ 100
B= = 10 -7 ´ = 5 × 10–6 T
E 4p r 4
qE = qvB Þ v =
B W N
Also,
r r
E´B E B sin q
=
2 100A
B B2
E B sin 90° E r
= 2
= = |v| =v
B B
20. (b) When a charged particle enters a magnetic 4m
field at a direction perpendicular to the E
direction of motion, the path of the motion S
is circular. In circular motion the direction Ground
of velocity changes at every point (the B
magnitude remains constant).
Therefore, the tangential momentum will According to right hand palm rule, the
change at every point. But kinetic energy magnetic field is directed towards south.
23. (a) The magnetic field at O due to current in
1
will remain constant as it is given by mv 2 DA is
2
and v2 is the square of the magnitude of mo I p
B1 = ´ (dir ected vertically
velocity which does not change. 4p a 6
21. (c) Clearly, the magnetic fields at a point P, upwards)
equidistant from AOB and COD will have The magnetic field at O due to current in
directions perpendicular to each other, as BC is
they are placed normal to each other.
www.crackjee.xyz
Moving Charges and Magnetism P-125

mo I p = q éë3i$ + $j + 2kˆ + $i ( -12 - 1)


B2 = ´ (dir ected vertically
4p b 6
downwards) - $j ( -9 - 1) + k ( 3 - 4 ) ùû
The magnetic field due to current AB and
CD at O is zero. = q éë3iˆ + $j + 2k$ - 13i$ + 10$j - k$ ùû
Therefore the net magnetic field is
B = B1 - B2 (directed vertically upwards) = q éë -10i$ + 11j$ + k$ ùû
mo I p m o I p Fy = 11qjˆ
= - ´
4 p a 6 4p b 6 Thus, the y component of the force.
m I æ 1 1ö m I q Magnetic dipole moment
= o ç - ÷ = o (b - a) 28. (c) =
24 è a b ø 24ab 2m Angular momentum
r r r
24. (d) F = I ( l ´ B)
The force on AD and BC due to current I1
is zero. This is because the directions of
uur r
current element I d l and magnetic field B a
are parallel.
25. (a) The magnetic field varies inversely with the
distance for a long conductor. That is,
1
Bµ \ Magnetic dipole moment (M)
d
so, graph in option (a) is the correct one.
q æ mR 2 ö 1 4
dq M = .ç ÷ .w = s.pR w.
26. (d) Current in a small element, dI = I 2m è 2 ø 4
p
Magnetic field due to the element
mv2 mv
m 2dI 29. (b) = qvB Þ r =
dB = 0 r qB
4p R
The component dB cos q, of the field is mp vp
cancelled by another opposite component. Þ rp = ;
Therefore, qpB
md v d m v
rd = ; ra = a a
qd B qa B

ma = 4mp , md = 2mp
qa = 2qp , q d = qp
dB
p From the problem
m0 I m0I
Bnet = ò dB sin q = ò sin qd q = 1
2p 2 R 0 p2 R Ep = Ed = Ea = mp vp 2
2
27. (a) Lorentz force acting on the particle
ur ur r ur 1 1
= md vd 2 = ma va2
F = q éë E + v × B ùû 2 2

é ˆi ˆj kˆ ù Þ vp2 = 2vd2 = 4mv22


ê ú
= q ê 3i$ + $j + 2k$ + 3 4 1 ú Thus we have, ra = rp < rd
ê ú
ê 1 1 -3 ú 30. (a) As we know, Magnetic flux, f = B. A
ë û
EBD_7764
www.crackjee.xyz
P-126 Physics
r r
m0 (2)(20 ´ 10 -2 ) 2 33. (c) F1 = F2 = 0
´ p(0.3 ´ 10 -2 ) 2
2[(0.2)2 + (0.15) 2 ] because of action and reaction pair

On solving
= 9.216 × 10–11 = 9.2 × 10–11 weber 34. (b) Case (a) :
31. (b) Work done in moving the conductor is,
2 µ0 I µ I
W = ò Fdx bA = ´ 2p = 0 ´ 2p
0 4p R 4p l / 2p
2 (2pR = l)
= ò 3.0 ´ 10 -4 e -0.2 x ´ 10 ´ 3dx
0 µ0 I
= ´ (2p) 2
4p l
Case (b) :
l=3m
I = 10 A
z 45°
a BB
x a/2
2
= 9 ´ 10 -3 e-0.2 x dx
ò
0 µ0 I
-3 BB = 4 × [sin 45° + sin 45°]
9 ´ 10 4p a / 2
= [-e -0.2 ´ 2 + 1]
0.2 µ0 I 2 µ0 I 64 m0 I
= 4´ ´ ´ = ´ = 32 2
-4 -0.2 x (By exponential 4p l / 8 2 4p l 2 4pl
B = 3.0 ´ 10 e
function) [4a = l]
35. (c) Ig G = ( I – Ig)s
9 ´ 10-3 \ 10–3 × 100 = (10 – 10–3) × S
= ´ [1 - e -0.4 ]
0.2 \ S » 0.01W
= 9 × 10–3 × (0.33) = 2.97 × 10–3J 36. (c) Given : Current through the galvanometer,
Power required to move the conductor is, ig = 5 × 10–3 A
W Galvanometer resistance, G = 15W
P= Let resistance R to be put in series with the
t
galvanometer to convert it into a voltmeter.
2.97 ´ 10-3 V = ig (R + G)
P= = 2.97 W
(0.2) ´ 5 ´ 10-3 10 = 5 × 10–3 (R + 15)
r r \ R = 2000 – 15 = 1985 = 1.985 × 103 W
32. (a) For stable equilibrium M || B
r r
For unstable equilibrium M || (–B)
www.crackjee.xyz
Moving Charges and Magnetism P-127

Magnetism and
Matter 19
1. A thin rectangular magnet suspended freely has (c) a ferromagnetic material becomes
a period of oscillation equal to T. Now it is broken diamagnetic
into two equal halves (each having half of the (d) a paramagnetic material becomes
original length) and one piece is made to ferromagnetic
oscillate freely in the same field. If its period of 5. The length of a magnet is large compared to its
width and breadth. The time period of its
T' oscillation in a vibration magnetometer is 2s. The
oscillation is T ' , the ratio is [2003]
T magnet is cut along its length into three equal
1 parts and these parts are then placed on each
(a) (b) 1
other with their like poles together. The time
2 2 2 period of this combination will be [2004]
1
(c) 2 (d) 2
4 (a) 2 3 s (b) s
2. A magnetic needle lying parallel to a magnetic 3
field requiers W units of work to turn it through 2
(c) 2 s (d) s
3
600 . The torque needed to maintain the needle
6. The materials suitable for making electromagnets
in this position will be [2003] should have [2004]
3W (a) high retentivity and low coercivity
(a) (b) W
(b) low retentivity and low coercivity
(c) high retentivity and high coercivity
(c) 3 (d) 2 W
W (d) low retentivity and high coercivity
2 7. A magnetic needle is kept in a non-uniform
3. The magnetic lines of force inside a bar magnet
magnetic field. It experiences [2005]
[2003]
(a) neither a force nor a torque
(a) are from north-pole to south-pole of the
(b) a torque but not a force
magnet (c) a force but not a torque
(b) do not exist (d) a force and a torque
(c) depend upon the area of cross-section of 8. Needles N 1 , N 2 and N 3 are made of a
the bar magnet ferromagnetic, a paramagnetic and a diamagnetic
(d) are from south-pole to north-pole of the substance respectively. A magnet when brought
Magnet close to them will [2006]
4. Curie temperature is the temperature above (a) attract N1 and N2 strongly but repel N3
which [2003] (b) attract N1 strongly, N2 weakly and repel N3
(a) a ferromagnetic material becomes weakly
paramagnetic (c) attract N1 strongly, but repel N2 and N3
(b) a paramagnetic material becomes weakly
diamagnetic (d) attract all three of them
EBD_7764
www.crackjee.xyz
P-128 Physics
9. Relative permittivity and permeability of a The current required to be passed in a solenoid
material er and mr, respectively. Which of the of length 10 cm and number of turns 100, so that
following values of these quantities are allowed the magnet gets demagnetized when inside the
for a diamagnetic material? [2008] solenoid, is: [2014]
(a) er = 0.5, mr = 1.5 (a) 30 mA (b) 60 mA
(b) er = 1.5, mr = 0.5 (c) 3 A (d) 6 A
(c) er = 0.5, mr = 0.5 12. Hysteresis loops for two magnetic materials A
(d) er = 1.5, mr = 1.5 and B are given below : [2016]
10. Two short bar magnets of length 1 cm each have D B
magnetic moments 1.20 Am2 and 1.00 Am2
respectively. They are placed on a horizontal
table parallel to each other with their N poles
H H
pointing towards the South. They have a
common magnetic equator and are separated by
a distance of 20.0 cm. The value of the resultant
horizontal magnetic induction at the mid-point (A) (B)
O of the line joining their centres is close to
(Horizontal component of earth.s magnetic These materials are used to make magnets for
induction is 3.6× 10.5Wb/m2) [2013] elecric generators, transformer core and
electromagnet core. Then it is proper to use :
(a) 3.6 × 10.5 Wb/m2
(a) A for transformers and B for electric
(b) 2.56 × 10.4 Wb/m2 generators.
(c) 3.50 × 10.4 Wb/m2 (b) B for electromagnets and transformers.
(d) 5.80 × 10.4 Wb/m2 (c) A for electric generators and trasformers.
11. The coercivity of a small magnet where the (d) A for electromagnets and B for electric
ferromagnet gets demagnetized is 3 × 103 Am–1. generators

A ns we r K e y
1 2 3 4 5 6 7 8 9 10 11 12
(b) (a) (d) (a) (b) (b) (d) (b) (b) (b) (c) (b)

1. (b) The time period of a rectangular magnet Magnet is cut into two identical pieces
oscillating in earth’s magnetic field is given such that each piece has half the original
I I'
by T = 2p length. Then T ' = 2p
mBH m ' BH
where I = Moment of inertia of the 2
rectangular magnet 1 æ M ö æ lö I m
m = Magnetic moment where I ' = çè ÷ø çè ÷ø = and m ' =
12 2 2 8 2
BH = Horizontal component of the earth’s
magnetic field
Case 1 T' I' m I /8 m 1 1
\ = ´ = ´ = =
I T m' I m/2 I 4 2
T = 2p 1
where I = M l2
mBH 12 2. (a) W = MB (cos q1 - cos q2 )
Case 2
= MB (cos 0° - cos 60°)
www.crackjee.xyz
Moving Charges and Magnetism P-129

1 MB 8. (b) Ferromagnetic substance has magnetic


= MB(1 - ) = domains whereas paramagnetic substances
2 2
have magnetic dipoles which get attracted
MB
\ t = MB sin q = MB sin 60° = 3 = 3W to a magnetic field. Diamagnetic substances
2 do not have magnetic dipole but in the
3. (d) As shown in the figure, the magnetic lines presence of external magnetic field due to
of force are directed from south to north their orbital motion of electrons these
inside a bar magnet. substances are repelled.
9. (b) For a diamagnetic material, the value of µr
is less than one. For any material, the value
of Îr is always greater than 1.
Solving we get, B = 5 × 10–8 tesla
N S 10. (b) Given : M1 = 1.20 Am2
N
BH
B1
4. (a) The temperature above which a S B2 S
ferromagnetic substance becomes O
paramagnetic is called Curie’s temperature. N N
r r
I I
5. (b) T = 2p = 2p where
M ´B MB S
1
I= ml 2 20
12 M2 = 1.00 Am2 ; r = cm = 0.1m
When the magnet is cut into three pieces 2
the pole strength will remain the same and Bnet = B1 + B2 + BH
1 æ mö æ l ö I m 0 ( M1 + M 2 )
M.I. (I¢) = çè ÷ø çè ÷ø ´ 3 = Bnet = + BH
12 3 3 9 4p r3
We have, Magnetic moment (M) 10 -7 (1.2 + 1)
= Pole strength (m) × l = + 3.6 ´ 10 -5 = 2.56 ´ 10 -4 wb/m2
\ New magnetic moment, (0.1)3
11. (c) Magnetic field in solenoid B = m0n i
æ lö
M ' = m ´ ç ÷ ´ 3 = ml = M B
è 3ø Þ m = ni
0
T 2 (Where n = number of turns per unit length)
\T'= = s.
9 3 B Ni
= 3 100i
Þ
m0 L Þ 3 ´ 10 =
6. (b) Electromagnet should be amenable to
magnetisation & demagnetization. 10 ´ 10-2
\ retentivity should be low & coercivity Þ i = 3A
should be low. 12. (b) Graph [A] is for material used for making
7. (d) A magnetic needle kept in non uniform permanent magnets (high coercivity)
agnetic field experience a force and torque Graph [B] is for making electromagnets and
due to unequal forces acting on poles. transformers.
EBD_7764
www.crackjee.xyz
P-130 Physics

Electromagnetic
Induction 20
1. A conducting square loop of side L and 5. One conducting U tube can slide inside another
resistance R moves in its plane with a uniform as shown in figure, maintaining electrical
velocity v perpendicular to one of its sides. A contacts between the tubes. The magnetic field
magnetic induction B constant in time and space, B is perpendicular to the plane of the figure . If
pointing perpendicular and into the plane at the each tube moves towards the other at a constant
loop exists everywhere with half the loop outside speed v, then the emf induced in the circuit in
the field, as shown in figure. The induced emf is terms of B, l and v where l is the width of each
[2002] tube, will be [2005]
A B
v
L V v X

C
(a) – Blv (b) Blv
(a) zero (b) RvB (c) 2 Blv (d) zero
(c) vBL/R (d) vBL 6. The self inductance of the motor of an electric
2. Two coils are placed close to each other. The fan is 10 H. In order to impart maximum power at
mutual inductance of the pair of coils depends 50 Hz, it should be connected to a capacitance
upon [2003] of [2005]
(a) the rates at which currents are changing in (a) 8 mF (b) 4 mF
the two coils (c) 2 mF (d) 1 mF
7. In an AC generator, a coil with N turns, all of the
(b) relative position and orientation of the two
same area A and total resistance R, rotates with
coils
frequency w in a magnetic field B. The maximum
(c) the materials of the wires of the coils
value of emf generated in the coil is [2006]
(d) the currents in the two coils (a) N.A.B.R.w (b) N.A.B
3. When the current changes from +2 A to –2A in (c) N.A.B.R. (d) N.A.B.w
0.05 second, an e.m.f. of 8 V is induced in a coil. 8. The flux linked with a coil at any instant 't' is
The coefficient of self -induction of the coil is given by f = 10t2 – 50t + 250. The induced emf at
[2003] t = 3s is [2006]
(a) 0.2 H (b) 0.4 H (a) –190 V (b) –10 V
(c) 0.8 H (d) 0.1 H (c) 10 V (d) 190 V
4. A metal conductor of length 1 m rotates vertically 9. Two coaxial solenoids are made by winding thin
about one of its ends at angular velocity 5 radians insulated wire over a pipe of cross-sectional area
per second. If the horizontal component of earth’s A = 10 cm2 and length = 20 cm. If one of the
magnetic field is 0.2×10–4T, then the e.m.f. solenoid has 300 turns and the other 400 turns,
developed between the two ends of the conductor their mutual inductance is [2008]
is [2004] (m0 = 4p × 10 –7 Tm A–1)
(a) 5 mV (b) 50 mV (a) 2.4p × 10–5 H (b) 4.8p × 10–4 H
–5 (d) 2.4p × 10–4 H
(c) 5 mV (d) 50mV (c) 4.8p × 10 H
www.crackjee.xyz
Electromagnetic Induction P-131
10. A boat is moving due east in a region where the
earth's magnetic field is 5.0 × 10–5 NA–1 m–1 due
north and horizontal. The boat carries a vertical B
aerial 2 m long. If the speed of the boat is 1.50 (a) (b) B
ms–1, the magnitude of the induced emf in the R
wire of aerial is: [2011] R
(a) 0.75 mV (b) 0.50 mV
(c) 0.15 mV (d) 1mV
11. A horizontal straight wire 20 m long extending B B
from east to west falling with a speed of 5.0 m/s, (c) (d)
at right angles to the horizontal component of R R
the earth’s magnetic field 0.30 × 10–4 Wb/m2.
14. A metallic rod of length ‘l’ is tied to a string of
The instantaneous value of the e.m.f. induced in
length 2l and made to rotate with angular speed
the wire will be [2011 RS]
w on a horizontal table with one end of the string
(a) 3 mV (b) 4.5 mV fixed. If there is a vertical magnetic field ‘B’ in
(c) 1.5 mV (d) 6.0 mV the region, the e.m.f. induced across the ends of
12. A coil is suspended in a uniform magnetic field, the rod is [2013]
with the plane of the coil parallel to the magnetic
lines of force. When a current is passed through
the coil it starts oscillating; It is very difficult to
stop. But if an aluminium plate is placed near to
the coil, it stops. This is due to : [2012]
(a) developement of air current when the plate
is placed
(b) induction of electrical charge on the plate 2 Bwl2 3Bwl 2
(c) shielding of magnetic lines of force as (a) (b)
2 2
aluminium is a paramagnetic material.
(d) electromagnetic induction in the aluminium 4 Bwl2 5Bwl 2
(c) (d)
plate giving rise to electromagnetic 2 2
damping. 15. In a coil of resistance 100W, a current is induced
13. A charge Q is uniformly distributed over the by changing the magnetic flux through it as
surface of non-conducting disc of radius R. The shown in the figure. The magnitude of change
disc rotates about an axis perpendicular to its in flux through the coil is [2017]
plane and passing through its centre with an
angular velocity w. As a result of this rotation a
magnetic field of induction B is obtained at the
centre of the disc. If we keep both the amount of
charge placed on the disc and its angular
velocity to be constant and vary the radius of
the disc then the variation of the magnetic
induction at the centre of the disc will be (a) 250 Wb (b) 275 Wb
represented by the figure : [2012] (c) 200 Wb (d) 225 Wb

Answer Key
1 2 3 4 5 6 7 8 9 10 11 12 13 14 15
(d) (b) (d) (b) (c) (d) (d) (b) (d) (c) (a) (d) (a) (d) (a)
EBD_7764
www.crackjee.xyz
P-132 Physics

1. (d) The induced emf is 8. (b) f = 10t2 – 50t + 250


r r
-d f d ( B. A) - d ( BA cos 0º ) df
e= =- = e=- = - (20t - 50)
dt dt dt dt
× × × × et = 3 = –10 V
m0 N1 N2 A
9. (d) M =
× × × × l
4p ´ 10-7 ´ 300 ´ 400 ´ 100 ´ 10 -4
× l
V
× × × =
0.2
× × × × m N N A
M= 0 1 2
X l
× × × × = 2.4p × 10–4 H
10. (c) Induced emf = vBH l = 1.5 × 5 × 10–5 × 2
dA d (l ´ x ) = 15 × 10–5
\ e = –B = -B
dt dt = 0.15 mV
dx 11. (a)
\ e = - Bl = - Blv W E
dt eind = Bvl
2. (b) Mutual inductance depends on the relative
= 0.3 × 10–4 × 5 × 20
position and orientation of the two coils.
= 3 × 10–3 V = 3 mV.
Df -D ( LI ) DI 12. (d) Because of the Lenz's law of conservation
3. (d) e=- = = -L
Dt Dt Dt of energy.
13. (a) The magnetic field due to a disc is given as
DI 4
\ | e |= L Þ 8= L ´ m wQ 1
Dt 0.05 B= 0 i.e., B µ
2 pR R
8 ´ 0.05 14. (d) Here, induced e.m.f.
Þ L= = 0.1H
4 w l
2l
4. (b) l = 1m, w = 5 rad/s, B = 0.2 ´ 10 -4 T dx
x
-4
Bwl 0.2 ´ 10 ´ 5 ´ 1 3l
e= = = 50mV [(3l)2 – (2l)2 ]
2 2 e= ò (wx) Bdx = Bw 2
5. (c) Relative velocity = v + v = 2v 2l
\ emf. = B.l (2v) 5 Bl 2 w
6. (d) For maximum power, XL = XC, which =
yields 2
15. (a) According to Faraday's law of
1 1 df
C= = electromagnetic induction, e=
(2 pn) 2 L 4 p 2 ´ 50 ´ 50 ´ 10 dt
Also, e = iR
\ C = 0.1 ´ 10-5 F = 1mF df
ur ur \ iR = Þ ò d f = R ò idt
df d ( N B. A) dt
7. (d) e = - =- Magnitude of change in flux (df) = R ×
dt dt area under current vs time graph
d 1 1
= -N ( BA cos wt ) = NBAw sin wt or, df = 100 ´ ´ ´ 10 = 250 Wb
dt 2 2
Þ e max = NBAw
www.crackjee.xyz

Alternating Current 21
1. The power factor of an AC circuit having 6. Alternating current can not be measured by D.C.
resistance (R) and inductance (L) connected in ammeter because [2004]
series and an angular velocity w is [2002] (a) Average value of current for complete cycle
(a) R/ w L (b) R/(R2 + w 2L2)1/2 is zero
(c) w L/R (d) R/(R2 – w 2L2)1/2 (b) A.C. Changes direction
(c) A.C. can not pass through D.C. Ammeter
2. The inductance between A and D is [2002]
(d) D.C. Ammeter will get damaged.
7. In an LCR series a.c. circuit, the voltage across
each of the components, L, C and R is 50V. The
A 3H 3H 3H D voltage across the LC combination will be[2004]
(a) 100 V (b) 50 2 V
(a) 3.66 H (b) 9 H (c) 50 V (d) 0 V (zero)
(c) 0.66 H (d) 1 H. 8. In a LCR circuit capacitance is changed from C
3. In a transformer, number of turns in the primary to 2 C. For the resonant frequency to remain
coil are 140 and that in the secondary coil are unchanged, the inductance should be changed
280. If current in primary coil is 4 A, then that in from L to [2004]
the secondary coil is [2002] (a) L/2 (b) 2 L
(a) 4 A (b) 2 A (c) 4 L (d) L/4
9. The phase difference between the alternating
(c) 6 A (d) 10 A.
p
4. In an oscillating LC circuit the maximum charge current and emf is . Which of the following
2
on the capacitor is Q. The charge on the cannot be the constituent of the circuit? [2005]
capacitor when the energy is stored equally (a) R, L (b) C alone
between the electric and magnetic field is (c) L alone (d) L, C
[2003] 10. A circuit has a resistance of 12 ohm and an
Q impedance of 15 ohm. The power factor of the
Q
(a) (b) circuit will be [2005]
2 3 (a) 0.4 (b) 0.8
Q (c) 0.125 (d) 1.25
(c) (d) Q
2 11. A coil of inductance 300 mH and resistance 2 W
5. The core of any transformer is laminated so as is connected to a source of voltage 2V. The
to [2003]
current reaches half of its steady state value in
(a) reduce the energy loss due to eddy currents
(b) make it light weight [2005]
(c) make it robust and strong (a) 0.1 s (b) 0.05 s
(d) increase the secondary voltage (c) 0.3 s (d) 0.15 s
EBD_7764
www.crackjee.xyz
P-134 Physics
12. In a series resonant LCR circuit, the voltage the figure. The internal resistance of the battery
across R is 100 volts and R = 1 kW with C = 2mF. is negligible. The switch S is closed at t = 0. The
The resonant frequency w is 200 rad/s. At potential drop across L as a function of time is
resonance the voltage across L is [2006] [2009]
(a) 2.5 × 10–2 V
(c) 250 V
(b) 40 V
(d) 4 × 10–3 V
(a)
12 -3t
t
e V (b) 6 1 - e (
-t / 0.2
V )
13. An inductor (L = 100 mH), a resistor (R = 100 W) (c) 12e–5t V (d) 6e–5t V
and a battery (E = 100 V) are initially connected 17. In the circuit shown below, the key K is closed
in series as shown in the figure. After a long at t = 0. The current through the battery is [2010]
time the battery is disconnected after short K
V
circuiting the points A and B. The current in the
circuit 1 ms after the short circuit is [2006]
L L R1

R2
R
VR1R2 V
A B (a) at t = 0 and R at t = ¥
R12 + R22 2
E
(a) 1/eA (b) eA V V ( R1 + R2 )
(c) 0.1 A (d) 1 A (b) at t = 0 and at t = ¥
R2 R1 R2
14. In an a.c. circuit the voltage applied is E = E0 sin
wt. The resulting current in the circuit is V VR1 R2
(c) at t = 0 and at t = ¥
pö R2 R12 + R22
æ
I = I 0 sin ç wt - ÷ . The power consumption
è 2ø V ( R1 + R2 ) V
in the circuit is given by [2007] (d) at t = 0 and at t = ¥
R1 R2 R2
E I
(a) P = 2 E0 I0 (b) P = 0 0 18. In a series LCR circuit R = 200W and the voltage
2
E0 I 0 and the frequency of the main supply is 220V
(c) P = zero (d) P = and 50 Hz respectively. On taking out the
2
15. An ideal coil of 10H is connected in series with capacitance from the circuit the current lags
a resistance of 5W and a battery of 5V. 2second behind the voltage by 30°. On taking out the
after the connection is made, the current flowing inductor from the circuit the current leads the
in ampere in the circuit is [2007] voltage by 30°. The power dissipated in the LCR
(a) (1 – e–1) (b) (1 – e) circuit is [2010]
(c) e (d) e–1 (a) 305 W (b) 210 W
16. E (c) Zero W (d) 242 W
L 19. A fully charged capacitor C with initial charge q0
R1 is connected to a coil of self inductance L at t = 0.
The time at which the energy is stored equally
between the electric and the magnetic fields is:
R2 [2011]
p
S (a) LC (b) 2p LC
4
An inductor of inductance L = 400 mH and (c) LC (d) p LC
resistors of resistance R1 = 2W and R2 = 2W are 20. A resistor ‘R’ and 2µF capacitor in series is
connected to a battery of emf 12 V as shown in connected through a switch to 200 V direct
www.crackjee.xyz
Alternating Current P-135
supply. Across the capacitor is a neon bulb that A C R
lights up at 120 V. Calculate the value of R to
make the bulb light up 5 s after the switch has
been closed. (log10 2.5 = 0.4) [2011] L
(a) 1.7 × 105 W (b) 2.7 × 106 W
B
(c) 3.3 × 107 W (d) 1.3 × 104 W
21. Combination of two identical capacitors, a
resistor R and a dc voltage source of voltage 6V e
is used in an experiment on a (C-R) circuit. It is (a) (b) 1
1- e
found that for a parallel combination of the 1- e
capacitor the time in which the voltage of the (c) –1 (d)
e
fully charged combination reduces to half its
24. An inductor (L = 0.03 H) and a resistor (R = 0.15
original voltage is 10 second. For series
kW) are connected in series to a battery of 15V
combination the time for needed for reducing
EMF in a circuit shown below. The key K1 has
the voltage of the fully charged series been kept closed for a long time. Then at t = 0,
combination by half is [2011 RS] K 1 is opened and key K 2 is closed
(a) 10 second (b) 5 second simultaneously. At t = l ms, the current in the
(c) 2.5 second (d) 20 second
22. In an LCR circuit as shown below both switches circuit will be : ( e5 @ 150 ) [2015]
are open initially. Now switch S1 is closed, S2
0.03 H 0.15 kW
kept open. (q is charge on the capacitor and t =
RC is Capacitive time constant). Which of the
following statement is correct ? [2013] K2
V

15V K1
R
S1 (a) 6.7 mA (b) 0.67 mA
(c) 100 mA (d) 67 mA
C S2 25. An LCR circuit is equivalent to a damped
pendulum. In an LCR circuit the capacitor is
L charged to Q0 and then connected to the L and
R as shown below : [2015]
(a) Work done by the battery is half of the
energy dissipated in the resistor L
R
(b) At, t = t, q = CV/2
(c) At, t = 2t, q = CV (1 – e–2)
(d) At, t = 2 t, q = CV (1 – e–1) C
23. In the circuit shown here, the point ‘C’ is kept
connected to point ‘A’ till the current flowing If a student plots graphs of the square of
through the circuit becomes constant. Afterward,
suddenly, point ‘C’ is disconnected from point
( )
maximum charge Q 2Max on the capacitor with
time(t) for two different values L1 and L2 (L1 >
‘A’ and connected to point ‘B’ at time t = 0. Ratio
L2) of L then which of the following represents
of the voltage across resistance and the inductor
this graph correctly ? (plots are schematic and
at t = L/R will be equal to: [2014]
not drawn to scale)
EBD_7764
www.crackjee.xyz
P-136 Physics
26. An arc lamp requires a direct current of 10 A at
L1 80 V to function. If it is connected to a 220 V
2
QMax (rms), 50 Hz AC supply, the series inductor
(a) L2 needed for it to work is close to : [2016]
t (a) 0.044 H (b) 0.065 H
(c) 80 H (d) 0.08 H
27. In the given circuit diagram when the current
2 reaches steady state in the circuit, the charge
QMax Q 0 (For both L1 and L2)
(b) on the capacitor of capacitance C will be : [2017]
t

2 L1
QMax L2
(c)
t

L2 r2 r1
2 (a) CE (b) CE
QMax L (r + r2 ) (r1 + r)
(d) 1
r1
t (c) CE (d) CE
(r2 + r)

Answer Key
1 2 3 4 5 6 7 8 9 10 11 12 13 14 15
(b) (d) (b) (c) (a) (a) (d) (a) (a) (b) (a) (c) (a) (c) (a)
16 17 18 19 20 21 22 23 24 25 26 27
(c) (c) (d) (a) (b) (c) (c) (c) (b) (c) (b) (a)

1. (b) The impedance triangle for resistance (R) 2. (d) These three inductors are connected in
and inductor (L) connected in series is parallel. The equivalent inductance Lp is
shown in the figure. given by
2 1 1 1 1 1 1 1 3
2
L = + + = + + = =1
L p L1 L2 L3 3 3 3 3
2 +w
R XL= w L
\ Lp = 1
f 3. (b) Np = 140, Ns = 280, Ip = 4A, Is = ?
R Is N p
R For a transformer I = N
p s
Power factor cosf =
R2 + w 2 L2
I s 140
Þ = Þ Is = 2 A
4 280
www.crackjee.xyz
Alternating Current P-137
4. (c) When the capacitor is completely charged, -3
L 300 ´ 10
the total energy in the LC circuit is with the Þ t= log 2 = ´ 0.69
capacitor and that energy is R 2
Þ t = 0.1 sec.
1 Q2
E= V 100
2 C 12. (c) Across resistor, I = = = 0.1 A
When half energy is with the capacitor in R 1000
At resonance,
the form of electric field between the plates
of the capacitor we get
1 1
E 1 Q '2 X L = XC = = = 2500
= where Q ' is the charge on wC 200 ´ 2 ´ 10-6
2 2 C Voltage across L is
one plate of the capacitor
I X L = 0.1 ´ 2500 = 250 V
1 1 Q 2 1 Q '2 Q 13. (a) Initially, when steady state is achieved,
\ ´ = Þ Q' =
2 2 C 2 C 2 E
i=
5. (a) Laminated core provide less area of cross- R
section for the current to flow. Because of Let E is short circuited at t = 0. Then
this, resistance of the core increases and E
current decreases thereby decreasing the At t = 0, i0 =
eddy current losses. R
Let during decay of current at any time the
6. (a) D.C. ammeter measure average current in
AC current, average current is zero for current flowing is - L di - iR = 0
complete cycle. Hence reading will be zero. dt
7. (d) Since the phase difference between L & C i t
di R di R
is p, Þ
i
= - dt Þ
L ò i ò L
= - dt
\ net voltage difference across i0 0
LC = 50 – 50 = 0 R
i R - t
8. (a) For resonant frequency to remain same Þ log e = - t Þ i = i0 e L
LC should be const. LC = const i0 L
L -100 ´10-3
Þ LC = L' × 2C Þ L ' = R
E - t 100 100 ´10 -3 1
2 Þi= e L = e =
9. (a) Phase difference for R–L circuit lies R 100 e
æ pö 14. (c) We know that power consumed in a.c.
between ç 0, ÷ circuit is given by, P = Erms.Irms cos f
è 2ø
Here, E = E0 sin wt
R 12 4
10. (b) Power factor = cos f = = = = 0.8 æ pö
Z 15 5 I = I0 sin ç wt - ÷
11. (a) The charging of inductance given by, è 2ø
which implies that the phase difference,
æ - ö
Rt
p
i = i0 ç1 - e L ÷ f=
çè ÷ø 2
p
-
Rt
-
Rt \ P = Erms .I rms .cos =0
i0 1 2
= i0 (1 - e L ) Þ e L =
2 2 æ p ö
Taking log on both the sides, çè Q cos = 0÷ø
2
Rt
- = log1 - log 2
L
EBD_7764
www.crackjee.xyz
P-138 Physics
æ - tö
R 2
2 æ 1 ö
15. (a) I = I o ç1 - e L ÷ Now, Z = R + ç - wL ÷
çè ÷ø è wC ø
(When current is in growth in LR circuit) æ 200 200 ö
2
= (200) 2 + ç - ÷ = 200 W
æ - ´2 ö
5
Eæ - tö
R
5 è 3 3ø
= ç1 - e L ÷ = ç 1 - e 10 ÷
R çè ÷ø 5 çè ÷ø Power dissipated = Vrms I rms cos f
–1
= (1 – e ) Vrms R æ Rö
16. (c) Growth in current in LR2 branch when
= Vrms . . çQ cos f = ÷
Z Z è Zø
switch is closed is given by
V 2rms R (220)2 ´ 200
E = =
i= [1 - e - R2t / L ] Z2 (200)2
R2
R2t
220 ´ 220
di E R2 - R2t / L E - = = 242 W
Þ = . .e = e L 200
dt R2 L L 1 2
19. (a) Energy stored in magnetic field = Li
Hence, potential drop across 2
æ E -R t / L ö -R t / L 1 q2
L = çè e 2 ÷ø L = Ee 2 Energy stored in electric field =
2 C
L
-
2t 1 2 1 q2
-3 \ Li =
= 12e 400´10 = 12e–5tV 2 2 C
17. (c) At t = 0 , no current will flow through L and 1
R1 Also q = q0 cos wt and w =
LC
V p
\ Current through battery = R On solving t = LC
2 4
At t = ¥ , 20. (b) We have, V = V0 (1 – e–t/RC)
RR Þ 120 = 200(1 – e–t/RC)
effective resistance, Reff = 1 2 Þ t = RC in (2.5)
R1 + R2 Þ R = 2.71 × 106 W
V 21. (c) Time constant for parallel combination
\ Current through battery = R = 2RC
eff
Time constant for series combination
V ( R1 + R2 )
= RC
R1R2 =
2
18. (d) When capacitance is taken out, the circuit In first case :
is LR.
t
wL - 1 V0
\ tan f = V = V0 e 2 RC = ...(1)
R 2
1 200 In second case :
Þ wL = R tan f = 200 ´ = t2
3 3 - V0
( RC / 2) ....(2)
Again, when inductor is taken out, the V = V0 e =
circuit is CR. 2
From (1) and (2)
1
\ tan f = t1 t2
w CR =
2 RC ( RC / 2)
1 1 200
Þ = R tan f = 200 ´ = t 10
wc 3 3 Þ t2 = 1 = = 2.5 sec.
4 4
www.crackjee.xyz
Alternating Current P-139
22. (c) Charge on he capacitor at any time t is
given by q = CV (1– et/t) d 2q R dq q
2
++ =0
at t = 2t dt L dt Lc
q = CV (1 – e–2) From damped harmonic oscillator, the
23. (c) Applying Kirchhoff's law of voltage in
dt
closed loop amplitude is given by A = Ao e -
V 2m
–VR –VC = 0 Þ R = -1
VC Double differential equation
2
VR d x b dx k
+ + =x 0
A C R dt 2 m dt m
Rt Rt
- -
2 2 L
L VL = Q oe
Qmax 2L Þ Q max
= Qoe

Hence damping will be faster for lesser self


B
inductance.
26. (b) Here
15 ´ 100 i=
e
=
e
=
e
24. (b) I(0) = = 0.1A
0.15 ´ 10 3 R 2 + X 2L R 2 + w2 L2 R 2 + 4p2 v 2 L2

I (¥) = 0 220
–t
10 =
I (t) = [I (0) – I (¥)] 64 + 4p2 (50) 2 L
e /R
L + i( ¥ )
–t R V 80
I(t) = 0.1 e L / R = 0.1 e L [Q R = = =8]
I 10
0.15 ´1000
On solving we get
L = 0.065 H
I(t) = 0.1 e 0.03 = 0.67mA
27. (a) In steady state, flow of current through
25. (c) From KVL at any time t capacitor will be zero.
Current through the circuit,
di
R L E
+ dt – i=
i r + r2

+ –
q c
Potential difference through capacitor
q di Q æ E ö
- iR - L = 0 Vc = = E - ir = E - ç r
è r + r2 ÷ø
c dt C
dq q dq Ld 2q
i=- Þ + R+ 2 =0 r2
dt c dt dt \ Q = CE
r + r2
EBD_7764
www.crackjee.xyz
P-140 Physics

Electromagnetic
Waves 22
1. Electromagnetic waves are transverse in nature (a) Electric energy density is double of the
is evident by [2002] magnetic energy density.
(a) polarization (b) interference (b) Electric energy density is half of the
(c) reflection (d) diffraction magnetic energy density.
2. An electromagnetic wave of frequency v = 3.0 (c) Electric energy density is equal to the
MHz passes from vacuum into a dielectric magnetic energy density.
medium with permittivity Î = 4.0. Then [2004]
(d) Both electric and magnetic energy densities
(a) wave length is halved and frequency
are zero.
remains unchanged
6. Match List - I (Electromagnetic wave type) with
(b) wave length is doubled and frequency
List - II (Its association/application) and select
becomes half
the correct option from the choices given below
(c) wave length is doubled and the frequency
the lists: [2014]
remains unchanged
(d) wave length and frequency both remain List 1 List 2
unchanged. 1. Infrared waves (i) To treat muscular
3. An electromagnetic wave in vacuum has the strain
r r 2. Radio waves (ii) For broadcasting
electric and magnetic field E and B , which are
always perpendicular to each other. The 3. X-rays (iii) To detect fracture of
r bones
direction of polarization is given by X and that
r 4. Ultraviolet rays (iv) Absorbed by the
of wave propagation by k . Then [2012] ozone layer of the
r r r r r
(a) X || B and k || B ´ E atmosphere
r r r r r 1 2 3 4
(b) X || E and k || E ´ B
r r r r r (a) (iv) (iii) (ii) (i)
(c) X || B and k || E ´ B (b) (i) (ii) (iv) (iii)
r r r r r (c) (iii) (ii) (i) (iv)
(d) X || E and k || B ´ E
(d) (i) (ii) (iii) (iv)
4. The magnetic field in a travelling electromagnetic 7. Arrange the following electromagnetic radiations
wave has a peak value of 20 nT. The peak value per quantum in the order of increasing energy :
of electric field strength is : [2013] [2016]
(a) 3 V/m (b) 6 V/m A : Blue light B : Yellow light
(c) 9 V/m (d) 12 V/m C : X-ray D : Radiowave.
5. During the propagation of electromagnetic (a) C, A, B, D (b) B, A, D, C
waves in a medium: [2014] (c) D, B, A, C (d) A, B, D, C
www.crackjee.xyz
Electromagnetic Waves P-141

Answer Key
1 2 3 4 5 6 7
(a) (a) (b) (b) (c) (d) (c)

1. (a) The phenomenon of polarisation is shown 1


only by transverse waves. 5. (c) E0 = CB0 and C =
m0 e 0
2. (a) Frequency remains constant during
refraction 1
Electric energy density = e 0 E0 2 = m E
1 c 2
vmed = = 1 Bo 2
µ0 Î0 ´4 2 Magnetic energy density = = mB
2 m0
l med vmed c / 2 1 Thus, mE = mB
= = =
l air vair c 2 Energy is equally divided between electric
\ wavelength is halved and frequency and magnetic field
remains unchanged 6. (d)
3. (b) Q The E.M. wave are transverse in nature (1) Infrared rays are used to treat muscular
strain because these are heat rays.
i.e.,
r r (2) Radio waves are used for broadcasting
k ´E r
= =H …(i) because these waves have very long
mw r wavelength ranging from few centimeters
r
where H = B to few hundred kilometers
r r m (3) X-rays are used to detect fracture of bones
k ´H r
and = -E … (ii) because they have high penetrating power
r we r r r but they can't penetrate through denser
k is ^ H and k is also ^ to E
r r r r r medium like dones.
or In other words X || E and k || E ´ B (4) Ultraviolet rays are absorbed by ozone of
4. (b) From question, the atmosphere.
B0 = 20 nT = 20 × 10–9T E, Decreases
(Q velocity of light in vacuum C = 3 × 108 7. (c) g-rays X-rays uv-rays Visible rays IR rays Radio
ms–1) VIBGYOR Microwaves waves
r r r
E0 = B0 ´ C Radio wave < yellow light < blue light < X-
r r r rays
| E 0 |=| B | . | C |= 20 ´ 10 -9 ´ 3 ´ 108
(Increasing order of energy)
= 6 V/m.
EBD_7764
www.crackjee.xyz
P-142 Physics

Ray Optics and


Optical Instruments 23
1. An astronomical telescope has a large aperture 7. A light ray is incident perpendicularly to one
to [2002] face of a 90° prism and is totally internally
(a) reduce spherical aberration reflected at the glass-air interface. If the angle of
(b) have high resolution reflection is 45°, we conclude that the refractive
(c) increase span of observation index n [2004]
(d) have low dispersion
2. If two mirrors are kept at 60° to each other, then
the number of images formed by them is [2002]
(a) 5 (b) 6 45°
(c) 7 (d) 8 45°
3. Which of the following is used in optical fibres?
[2002]
45°
(a) total internal reflection
(b) scattering
(c) diffraction 1
(a) n> (b) n> 2
(d) refraction. 2
4. Consider telecommunication through optical 1
(c) n< (d) n < 2
fibres. Which of the following statements is not 2
true? [2003] 8. A plano convex lens of refractive index 1.5 and
(a) Optical fibres can be of graded refractive
radius of curvature 30 cm, is silvered at the curved
index
surface. Now this lens has been used to form the
(b) Optical fibres are subject to
electromagnetic interference from outside image of an object. At what distance from this
(c) Optical fibres have extremely low lens an object be placed in order to have a real
transmission loss image of size of the object [2004]
(d) Optical fibres may have homogeneous core (a) 60 cm (b) 30 cm
with a suitable cladding. (c) 20 cm (d) 80 cm
5. The image formed by an objective of a 9. A fish looking up through the water sees the
compound microscope is [2003] outside world contained in a circular horizon. If
(a) virtual and diminished 4
(b) real and diminished the refractive index of water is and the fish is
3
(c) real and enlarged 12 cm below the surface, the radius of this circle
(d) virtual and enlarged in cm is [2005]
6. To get three images of a single object, one should 36
have two plane mirrors at an angle of [2003] (a) (b) 36 7
7
(a) 60º (b) 90º
(c) 4 5 (d) 36 5
(c) 120º (d) 30º
www.crackjee.xyz
Ray Optics and Optical Instruments P-143

10. A thin glass (refractive index 1.5) lens has optical


power of – 5 D in air. Its optical power in a liquid
medium with refractive index 1.6 will be [2005] q
(a) – 1D (b) 1 D
(c) – 25 D (d) 25 D The incident angle q for which the light ray
11. The refractive index of a glass is 1.520 for red grazes along the wall of the rod is : [2009]
light and 1.525 for blue light. Let D1 and D2 be æ 2 ö
angles of minimum deviation for red and blue (a) sin -1 ( 3/2 ) (b) sin -1 ç
è 3ø
÷
light respectively in a prism of this glass. Then,
[2006] æ 1 ö
(c) sin -1 ç ÷ (d) sin -1 (1/ 2 )
(a) D1 < D2 è 3ø
(b) D1 = D2 16. In an optics experiment, with the position of the
(c) D1 can be less than or greater than D2 object fixed, a student varies the position of a
depending upon the angle of prism convex lens and for each position, the screen is
(d) D1 > D2 adjusted to get a clear image of the object. A
12. Two lenses of power –15 D and +5 D are in graph between the object distance u and the
contact with each other. The focal length of the image distance v, from the lens, is plotted using
combination is [2007] the same scale for the two axes. A straight line
(a) + 10 cm (b) – 20 cm passing through the origin and making an angle
(c) – 10 cm (d) + 20 cm of 45° with the x-axis meets the experimental
13. A student measures the focal length of a convex curve at P. The coordinates of P will be [2009]
lens by putting an object pin at a distance ‘u’ æ f fö
from the lens and measuring the distance ‘v’ of (a) çè 2 , 2 ÷ø (b) ( f, f )
the image pin. The graph between ‘u’ and ‘v’
plotted by the studen t should look (c) ( 4 f, 4 f ) (d) ( 2 f, 2 f )
like [2008] 17. Let the x-z plane be the boundary between two
transparent media. Medium 1 in z ³ 0 has a
v(cm) v(cm)
refractive index of 2 and medium 2 with z < 0
has a refractive index of 3 . A ray of light in
(a) (b)
medium 1 given by the vector
O u(cm) O u(cm) r
A= 6 3iˆ + 8 3 ˆj - 10 kˆ is incident on the plane of
v(cm) separation. The angle of refraction in medium 2
v(cm)
is: [2011]
(c) (d) (a) 45° (b) 60°
O u(cm) O u(cm) (c) 75° (d) 30°
18. A car is fitted with a convex side-view mirror of
14. An experment is performed to find the refractive focal length 20 cm. A second car 2.8 m behind
index of glass using a travelling microscope. In the first car is overtaking the first car at a relative
this experiment distances are measured by[2008] speed of 15 m/s. The speed of the image of the
(a) a vernier scale provided on the microscope second car as seen in the mirror of the first one
(b) a standard laboratory scale is : [2011]
(c) a meter scale provided on the microscope 1
(a) m/s (b) 10 m/s
(d) a screw gauge provided on the microscope 15
1
15. A transparent solid cylindrical rod has a (c) 15 m/s (d) m/s
10
2 19. A beaker contains water up to a height h1 and
refractive index of . It is surrounded by air..
3 kerosene of height h2 above water so that the
A light ray is incident at the mid-point of one total height of (water + kerosene) is (h1 + h2).
end of the rod as shown in the figure.
EBD_7764
w w w . c r
P-144 Physics
Refractive index of water is m1 and that of
kerosene is m2. The apparent shift in the position d d
of the bottom of the beaker when viewed from
above is [2011 RS] (c) (d)
o o
i i
æ 1 ö æ 1ö
(a) çè 1 + m ÷ø h1 - çè1 + m ÷ø h2 24. A thin convex lens made from crown glass
1 2

æ ö æ 1 ö æ 3ö
(b) ç 1 -
1 çm = ÷ has focal length f. When it is measured
è m ÷ø h1 + çè1 - m ÷ø h2 è 2ø
1 2
in two different liquids having refractive indices
æ 1 ö æ 1 ö
(c) ç 1 + ÷ h2 - ç1 + ÷ h1 4 5
è m1ø è m 2 ø and , it has the focal lengths f1 and f2
3 3
æ 1ö æ 1 ö respectively. The correct relation between the
(d) ç 1 - ÷ h2 + ç1 - ÷ h1
è m1 ø è m2 ø focal lengths is: [2014]
(a) f1 = f2 < f
20. When monochromatic red light is used instead
(b) f1 > f and f2 becomes negative
of blue light in a convex lens, its focal length will
[2011 RS] (c) f2 > f and f1 becomes negative
(a) increase (d) f1 and f2 both become negative
(b) decrease 25. Monochromatic light is incident on a glass prism
(c) remain same of angle A. If the refractive index of the material
(d) does not depend on colour of light of the prism is µ, a ray, incident at an angle q, on
21. An object at 2.4 m in front of a lens forms a the face AB would get transmitted through the
sharp image on a film 12 cm behind the lens. A face AC of the prism provided : [2015]
glass plate 1cm thick, of refractive index 1.50 is
interposed between lens and film with its plane
A
faces parallel to film. At what distan ce
(from lens) should object shifted to be in sharp q
focus of film? [2012]
(a) 7.2 m (b) 2.4 m
(c) 3.2 m (d) 5.6 m B C
22. Diameter of a plano-convex lens is 6 cm and
thickness at the centre is 3 mm. If speed of light é æ æ 1 öù
(a) q > cos -1 êµsin ç A + sin -1 ç ÷ ú
in material of lens is 2 × 108 m/s, the focal length è µ ø ûú
ëê è
of the lens is [2013]
(a) 15 cm (b) 20 cm é æ æ 1 öù
(c) 30 cm (d) 10 cm (b) q < cos-1 êµsin ç A + sin -1 ç ÷ ú
êë è è µ ø úû
23. The graph between angle of deviation (d) and
angle of incidence (i) for a triangular prism is
é æ æ 1 öù
represented by [2013] (c) q > sin -1 êµsin ç A - sin -1 ç ÷ ú
ëê è è µ ø ûú
d d

(a) (b) é æ æ 1 öù
(d) q < sin -1 êµsin ç A - sin -1 ç ÷ ú
o o
i ëê è è µ ø ûú
i
www.crackjee.xyz
Ray Optics and Optical Instruments P-145

26. An observer looks at a distant tree of height 10 28. A diverging lens with magnitude of focal length
m with a telescope of magnifying power of 20. 25 cm is placed at a distance of 15 cm from a
To the observer the tree appears : [2016] converging lens of magnitude of focal length 20
(a) 20 times taller (b) 20 times nearer cm. A beam of parallel light falls on the diverging
(c) 10 times taller (d) 10 times nearer lens. The final image formed is : [2017]
27. In an experiment for determination of refractive
(a) real and at a distance of 40 cm from the
index of glass of a prism by i – d, plot it was
divergent lens
found thata ray incident at angle 35°, suffers a
deviation of 40° and that it emerges at angle 79°. (b) real and at a distance of 6 cm from the
In that case which of the following is closest to convergent lens
the maximum possible value of the refractive (c) real and at a distance of 40 cm from
index? [2016] convergent lens
(a) 1.7 (b) 1.8 (d) virtual and at a distance of 40 cm from
(c) 1.5 (d) 1.6 convergent lens.
Answer Key
1 2 3 4 5 6 7 8 9 10 11 12 13 14 15
(b) (a) (a) (b) (c) (b) (b) (c) (a) (b) (a) (c) (c) (a) (c)
16 17 18 19 20 21 22 23 24 25 26 27 28
(d) (a) (a) (b) (a) (d) (c) (c) (b) (c) (b) (c) (c)

1. (b) The resolving power of a telescope 4. (b) Optical fibres form a dielectric wave guide
D and are free from electromagnetic
R.P = interference or radio frequency interference.
1.22 l
5. (c) A real, inverted and enlarged image of the
where D = diameter of the objective lens object is formed by the objective lens of a
l = wavelength of light.
Clearly, larger the aperture, larger is the compound microscope.
value of D, more is the resolving power or 360 360
resolution. 6. (b) When q = 90° then = =4
2. (a) When two plane mirrors are inclined at each q 90
other at an angle q then the number of the is an even number. The number of images
images of a point object placed between formed is given by
360° 360 360
the plane mirrors is - 1, n= -1 = -1 = 4 - 1 = 3
q q 90
360° 7. (b) The incident angle is 45°.
if is even Incident angle > critical angle, i > ic
q
360°
\Number of images formed = -1 = 5 \ sin i > sin ic or sin 45 > sin ic
60º
3. (a) In an optical fibre, light is sent through the 1
fibre without any loss by the phenomenon of sin ic =
n
total internal reflection as shown in the figure. 1 1 1
\ sin 45° > or > Þn> 2
n2 n 2 n
n1 8. (c) The focal length(F) of the final mirror is
n1>n2
1 2 1
= +
F fl f m
EBD_7764
www.crackjee.xyz
P-146 Physics

1 æ 1 1ö 1 8
Here = ( µ - 1) ç - ÷ Þ f m = -8 ´ f a = -8 ´ - =
fl è R1 R2 ø 5 5
m 1.6
Pm = = ´ 5 = 1D
é1 1 ù 1 fm 8
= (1.5 - 1) ê - =
ë a -30 úû 60 11. (a) For a thin prism, D = (m – 1) A
1 1 1 1 Since lb < lr Þ mr < mb Þ D1 < D2
\ = 2´ + =
F 60 30 / 2 10 12. (c) Power of combination is given by
\ F = 10 cm P = P1 + P2 = (– 15 + 5) D = – 10 D.
The combination acts as a converging 1
mirror. For the object to be of the same size 1 1
Now, P = Þ f = = metre
of mirror, f P -10
u = 2F = 20 cm
æ1 ö
1 3 \ f = - ç ´ 100÷ cm = -10 cm.
9. (a) sin qc = = è 10 ø
m 4 13. (c) This graph suggest that when
3 3 R u = – f, v = + µ
or tan qc = = =
16 - 9 7 12 v (cm)

qc qc
12 cm f

–f u (cm)

When the object is moved further away


from the lens, v decreases but remains
36 positive. When u is at – µ , v = f.
Þ R= cm This is how image formation takes place
7 for different positions of the object in case
1 æ 1.5 ö æ 1 1ö of a convex lens.
10. (b) =ç - 1÷ ç - ÷ .... (i) 14. (a) To find the refractive index of glass using a
fa è 1 ø è R1 R2 ø travelling microscope, a vernier scale is
provided on the microscope
1 æ mg öæ 1 1ö
=ç - 1÷ ç - ÷ Q
f m è m m ø è R1 R2 ø
n
1 æ 1.5 ö æ 1 1ö 90 – a
=ç - 1÷ ç - ÷ .... (ii) a
fm è 1.6 ø è R1 R2 ø 15. (c)
q P
æ ö
f m ç 1.5 - 1 ÷
Dividing (i) by (ii), = =–8 Applying Snell’s law at Q
f a ç 1.5 ÷
ç - 1÷ sin 90° 1
è 1.6 ø
n= =
1 1 sin(90º -a ) cos a
Pa = - 5 = Þ fa = -
fa 5 1
\ cos a =
n
www.crackjee.xyz
Ray Optics and Optical Instruments P-147

1 n2 - 1 2 sin i = 3sin r
\ sin a = 1 - cos 2 a = 1 - = ...(1) Ð r = 45°
n2 n
Applying Snell’s Law at P
sin q 2
n= Þ sin q = n ´ sin a = n - 1 ;
sin a
from (1)
2
æ 2 ö 4 1
\ sin q = ç ÷ - 1 = -1 =
è 3ø 3 3
æ 1 ö
or q = sin -1 ç ÷
è 3ø

16. (d) Experimental 18. (a) From mirror formula


|v| curve
1 1 1 dv v 2 æ du ö
+ = so, =- 2ç ÷
v u f dt u è dt ø
Straight 2
line dv æ f ö du
Þ = -ç
dt è u - f ÷ø dt
(2f, 2f) dv 1
P Þ = m/s
dt 15
45° 19. (b)
|u|
m2 Kerosene h2
1 1 1
For a convex lens - =
v u f
m1 Water h1
when u = -a , v = + f
when u = - f , v = +a
é 1ù
Then u = -2 f , v = 2 f Apparent shift due to water = h1 ê1 - ú
ë m1 û
f Apparent shift due to kerosene
Also v =
f é 1 ù
1+ = h2 ê1 - ú
u
ë m 2û
As |u| increases, v decreases for |u| > f. The Thus, total apparent shift :
graph between |v| and |u| is shown in the
figure. A straight line passing through the æ 1ö æ 1 ö
= h1 ç1 - ÷ + h2 ç1 - ÷
origin and making an angle of 45°with the è m1 ø è m2 ø
x-axis meets the experimental curve at
P (2f, 2f ). 20. (a) We know that m R < m B
17. (a) Angle of incidence is given by 1 æ1 1ö
and = (m - 1) ç - ÷
f è R1 R2 ø
cos (p–i) =
(6 3iˆ + 8 3 ˆj - 10kˆ .kˆ ) 1 1
20 Þ > Þ fR > fB .
1 fB fR
– cos i = – 21. (d) The focal length of the lens
2
Ð i = 60 ° 1 1 1 1 1 20 + 1 21
= - = + = =
From Snell's law, f v u 12 240 240 240
EBD_7764
www.crackjee.xyz
P-148 Physics
240 Þ f2 = (–) ve
f = cm
21 25. (c) When r2 = C, ÐN2Rc = 90°
æ 1ö Where C = critical angle
Shift = t ç 1 - ÷
è m ø 1
As sin C = = sin r2
æ 1 ö 1 v
1ç 1 - ÷ = 1´
è 3/2 ø 3 A

1 35
Now v' = 12 - = cm N1
3 3
Now the object distance u. q N2
1 3 21 1 é 3 21 ù Q r1
= - = - r2
u 35 240 5 êë 7 48 úû
R
P
1 1 é 48 - 49 ù
=
u 5 êë 7 ´ 16 úû B C
u = –7 ×16 × 5 = – 560 cm = – 5.6 m Applying snell's law at ‘R’
Velocity of light in vacuum µ sin r2 = 1 sin90° ...(i)
22. (c) \ n =
Velocity of light in medium Applying snell's law at ‘Q’
3 1 × sin q = µ sin r1 ...(ii)
\ n= But r1 = A – r2
2
So, sin q = µ sin (A – r 2)
sin q = µ sin A cos r2 – cos A ...(iii)
R = 3cm [using (i)]
From (1)
3mm
2 1
cos r2 = 1 – sin r2 = 1– ...(iv)
µ2
By eq. (iii) and (iv)
32 + (R – 3mm)2 = R2
1
Þ 32 + R2 – 2R(3mm) + (3mm)2 = R2 sin q = µsin A 1 - - cos A
Þ R » 15 cm µ2
on further solving we can show for ray not
1 æ 3 öæ 1 ö to transmitted through face AC
= ç –1 ÷ç ÷ Þ f = 30 cm
f è 2 øè 15 ø é –1 æ 1 ö ù
23. (c) For the prism as the angle of incidence (i) q = sin–1 ê m sin(A – sin çè µ ÷ø ú
increases, the angle of deviation (d) first ë û
decreases goes to minimum value and then So, for transmission through face AC
increases.
é –1 æ 1 ö ù
24. (b) By Lens maker's formula for convex lens q > sin–1 ê m sin(A – sin çè µ ÷ø ú
ë û
1 æ m öæ 2 ö
=ç - 1÷ ç ÷ 26. (b) A telescope magnifies by making the object
f è mL øè R ø appearing closer.
27. (c) We know that i + e – A = d
4
for, m L1 = , f1 = 4 R 35° + 79° – A = 40° \ A = 74°
3
5
for m L 2 = , f 2 = -5 R
3
www.crackjee.xyz
Ray Optics and Optical Instruments P-149

æ A + dm ö æ 74 + d m ö 15 cm
sin ç ÷ sin ç ÷
è 2 ø= è 2 ø
But m =
sinA / 2 74
sin
2
5 æ d ö
= sin ç 37° + m ÷
3 è 2 ø
5 f = –25 cm f = 20 cm
m max can be . That is m max is less than
3 The image formed by diverging lens is used as
5 an object for converging lens,
= 1.67
3 So for converging lens u = –25 – 15 = –40 cm, f
But dm will be less than 40° so = 20 cm
5 5 \ Final image formed by converging lens
m< sin 57° < sin 60° Þ m = 1.5
3 3 1 1 1
- =
28. (c) As parallel beam incident on diverging lens V -40 20
will form image at focus. or, V = 40 cm from converging lens real and
\ v = –25 cm inverted.
EBD_7764
www.crackjee.xyz
P-150 Physics

Wave Optics
24
1. To demonstrate the phenomenon of interference, 7. In a Young’s double slit experiment the intensity
we require two sources which emit radiation l
[2003] at a point where the path difference is (l being
6
(a) of nearly the same frequency the wavelength of light used) is I. If I0 denotes
(b) of the same frequency
I
(c) of different wavelengths the maximum intensity, is equal to [2007]
(d) of the same frequency and having a definite I0
phase relationship 3 1
2. The angle of incidence at which reflected light is (a) (b)
4 2
totally polarized for reflection from air to glass
(refractive index n), is [2004] 3 1
(c) (d)
(a) tan–1(l/n) (b) sin –1(l/n) 2 2
(c) sin –1(n) (d) tan–1(n) 8. In an experiment, electrons are made to pass
3. The maximum number of possible interference through a narrow slit of width ‘d’ comparable to
maxima for slit-separation equal to twice the their wavelength. They are detected on a screen at
wavelength in Young’s double-slit experiment is a distance ‘D’ from the slit (see figure).
[2004]
(a) three (b) five
(c) infinite (d) zero d
4. A Young’s double slit experiment uses a y=0
monochromatic source. The shape of the
interference fringes formed on a screen is [2005] D
(a) circle (b) hyperbola Which of the following graphs can be expected
(c) parabola (d) straight line to represent the number of electrons ‘N’ detected
5. If I 0 is the intensity of the principal maximum in as a function of the detector position ‘y’(y = 0
the single slit diffraction pattern, then what will corresponds to the middle of the slit) [2008]
be its intensity when the slit width is doubled? y
[2005]

(a) 4 I0 (b) 2 I0 (a) N d


I0
(c) (d) I0
2
6. When an unpolarized light of intensity I0 is y
incident on a polarizing sheet, the intensity of
the light which does not get transmitted is [2005]
1 1 (b) N d
(a) I0 (b) I0
4 2
(c) I0 (d) zero
www.crackjee.xyz
Wave Optics P-151
y 13. This question has a paragraph followed by two
statements, Statement – 1 and Statement – 2. Of
the given four alternatives after the statements,
(c) N d choose the one that describes the statements.
A thin air film is formed by putting the convex
surface of a plane-convex lens over a plane glass
y plate. With monochromatic light, this film gives
an interference pattern due to light reflected from
the top (convex) surface and the bottom (glass
(d) N d plate) surface of the film.
Statement – 1 : When light reflects from the air-
glass plate interface, the reflected wave suffers
9. A mixture of light, consisting of wavelength 590 a phase change of p.
nm and an unknown wavelength, illuminates Statement – 2 : The centre of the interference
Young’s double slit and gives rise to two pattern is dark. [2011]
overlapping interference patterns on the screen. (a) Statement – 1 is true, Statement – 2 is true,
The central maximum of both lights coincide. Statement – 2 is the correct explanation of
Further, it is observed that the third bright fringe Statement – 1.
of known light coincides with the 4th bright fringe (b) Statement – 1 is true, Statement – 2 is true,
of the unknown light. From this data, the Statement – 2 is not the correct explanation
wavelength of the unknown light is: [2009] of Statement – 1.
(a) 885.0 nm (b) 442.5 nm (c) Statement – 1 is false, Statement – 2 is true.
(c) 776.8 nm (d) 393.4 nm (d) Statement – 1 is true, Statement – 2 is false.
Directions : Questions number 10-12 are based on the 14. At two points P and Q on screen in Young’s
following paragraph. double slit experiment, waves from slits S1 and
An initially parallel cylindrical beam travels in a medium l
of refractive index m(I) = m0 + m2 I, where m0 and m2 are S 2 have a path difference of 0 and ,
4
positive constants and I is the intensity of the light respectively. The ratio of intensities at P and Q
beam. The intensity of the beam is decreasing with will be : [2011 RS]
increasing radius
10. As the beam enters the medium , it will [2010] (a) 2 : 1 (b) 2 :1
(a) diverge (c) 4 : 1 (d) 3 : 2
(b) converge 15. In a Young’s double slit experiment, the two slits
(c) diverge near the axis and converge near the act as coherent sources of wave of equal
periphery amplitude A and wavelength l. In another
(d) travel as a cylindrical beam experiment with the same arrangement the two
11. The initial shape of the wavefront of the beam is slits are made to act as incoherent sources of
[2010] waves of same amplitude and wavelength. If the
(a) convex intensity at the middle point of the screen in the
(b) concave first case is I1 and in the second case is I2, then
(c) convex near the axis and concave near the I1
periphery the ratio is [2011 RS]
I2
(d) planar
12. The speed of light in the medium is [2010] (a) 2 (b) 1
(a) minimum on the axis of the beam (c) 0.5 (d) 4
(b) the same everywhere in the beam 16. Statement - 1: On viewing the clear blue portion
(c) directly proportional to the intensity I of the sky through a Calcite Crystal, the intensity
(d) maximum on the axis of the beam of transmitted light varies as the crystal is rotated.
EBD_7764
www.crackjee.xyz
P-152 Physics
Statement - 2: The light coming from the sky is 20. Two beams, A and B, of plane polarized light
polarized due to scattering of sun light by with mutually perpendicular planes of polarization
particles in the atmosphere. The scattering is are seen through a polaroid. From the position
largest for blue light. [2011 RS] when the beam A has maximum intensity (and
(a) Statement -1 is true, statement-2 is false. beam B has zero intensity), a rotation of polaroid
(b) Statement-1 is true, statement-2 is true, through 30° makes the two beams appear equally
statement-2 is the correct explanation of bright. If the initial intensities of the two beams
statement-1 I
(c) Statement-1 is true, statement-2 is true, are IA and IB respectively, then A equals:
IB
statement-2 is not the correct explanation
of statement-1 3
(a) 3 (b) [2014]
(d) Statement-1 is false, statement-2 is true. 2
17. In Young's double slit experiment, one of the slit 1
(c) 1 (d)
is wider than other, so that amplitude of the light 3
from one slit is double of that other slit. If Im be 21. Assuming human pupil to have a radius of 0.25
the maximum intensity, the resultant intensity I cm and a comfortable viewing distance of 25 cm,
when they interfere at phase difference f is given the minimum separation between two objects that
by : [2012] human eye can resolve at 500 nm wavelength is :
Im (a) 100 µm (b) 300 µm [2015]
(a) (4 + 5 cos f)
9 (c) 1 µm (d) 30 µm
22. The box of a pin hole camera, of length L, has a
Im æ 2 fö
(b) ç 1 + 2 cos ÷ hole of radius a. It is assumed that when the hole
3 è 2ø
is illuminated by a parallel beam of light of
Im æ 2 fö wavelength l the spread of the spot (obtained
(c) ç 1 + 4 cos ÷
5 è 2ø on the opposite wall of the camera) is the sum of
its geometrical spread and the spread due to
Im æ 2 fö
(d) ç 1 + 8 cos ÷ diffraction. The spot would then have its
9 è 2ø
minimum size (say bmin) when : [2016]
18. Abeam of unpolarised light of intensity I0 is
passed through a polaroidAand then through (a) a = lL and bmin = 4lL
another polaroid B which is oriented so that its
principal plane makes an angle of l2
(b) a= and bmin = 4lL
45° relative to that of A. The intensity of the L
emergent light is [2013] l2 æ 2l2 ö
ç ÷
(a) I0 (b) I0/2 (c) a = L and bmin = ç L ÷
(c) I0/4 (d) I0/8 è ø
19. Two coherent point sources S 1 and S2 are æ 2l2 ö
ç ÷
separated by a small distance 'd' as shown. The (d) a = ll and bmin = ç L ÷
fringes obtained on the screen will be [2013] è ø
23. In a Young's double slit experiment, slits are
separated by 0.5 mm, and the screen is placed
150 cm away. A beam of light consisting of two
d
wavelengths, 650 nm and 520 nm, is used to
S1 S2 obtain interference fringes on the screen. The
Screen least distance from the common central maximum
D to the point where the bright fringes due to both
the wavelengths coincide is : [2017]
(a) points (b) straight lines (a) 9.75 mm (b) 15. 6 mm
(c) semi-circles (d) concentric circles (c) 1.56 mm (d) 7.8 mm
www.crackjee.xyz
Wave Optics P-153

Answer Key
1 2 3 4 5 6 7 8 9 10 11 12 13 14 15
(d) (d) (b) (d) (a) (b) (a) (d) (b) (b) (d) (a) (b) (a) (a)
16 17 18 19 20 21 22 23
(b) (d) (c) (d) (d) (d) (a) (d)

1. (d) For the phenomenon of interference we


require two sources of light of same difference is
frequency and having a definite phase 2p ´ l / 6 p
relationship (a phase relationship that does =
l 3
not change with time)
2. (d) The angle of incidence for total polarization p
\ Intensity I = I1 + I 2 + 2 I1 I 2 cos
3
is given by tan q = n Þ q = tan -1 n
Where n is the refractive index of the glass. \ I = I1 + I 2 + I1 I 2
3. (b) For constructive interference d sin q = nl when I1= I2 = I' (say) then I = 3I'

( )
2
n Imax = I1 + I 2
given d = 2 l Þ sin q =
2
=( I ' ) = ( 2 I ' ) = 4I '
n = 0, 1, – 1, 2, – 2 hence five maxima are 2 2
I'+
possible
4. (d) The shape of interference fringes formed I 3
on a screen in case of a monochromatic \ =
Imax 4
source is a straight line. Remember for
double hole experiment a hyperbola is
generated. The intensity of light at any point of the
2 screen where the phase difference due to light
æ sin f ö p
5. (a) I = I 0 çç ÷÷ and f = ( b sin q ) coming from the two slits is f is given by
è f ø l
æ fö
When the slit width is doubled, the I = Io cos2 ç ÷ where I is the maximum
amplitude of the wave at the centre of the è 2ø 0

screen is doubled, so the intensity at the intensity.


centre is increased by a factor 4. NOTE This formula is applicable when I1= I2. Here
6. (b) I = I 0 cos2 q f =p /3
I0 2
Intensity of polarized light = I p æ 3ö 3
2 \ = cos2 = ç ÷ =
I0 6 è 2 ø 4
Þ Intensity of untransmitted light
8. (d) The electron beam will be diffracted and the
I I maxima is obtained at y = 0. Also the
= I0 - 0 = 0
2 2 distance between the first minima on both
7. (a) For path difference of l , the phase side will be greater than d.
9. (b) Third bright fringe of known light coincides
difference is 2 p
with the 4th bright fringe of the unknown light.
l
For path difference of , the phase 3(590) D 4lD
6 \ =
d d
EBD_7764
w w w . c r a c k
P-154 Physics

3 15. (a) For coherent sources :


Þ l= ´ 590 = 442.5 nm I1 = 4 I 0
4
10. (b) In the medium, the refractive index will For incoherent sources
decrease from the axis towards the periphery I 2 = 2I 0 \ I1 = 2
of the beam. I2 1
Therefore, the beam will move as one move
16. (b) When viewed through a polaroid which
from the axis to the periphery and hence
is rotated then the light from a clear blue
the beam will converge.
portion of the sky shows a rise and fall of
intensity.
Decreasing m Incident sunlight
Axis (unpolarised)

Sun
11. (d) Initially the parallel beam is cylindrical .
Therefore, the wavefront will be planar. Scattered light
12. (a) The speed of light (c) in a medium of (polarised)
refractive index (m) is given by
c
m = 0 , where c0 is the speed of light in
c
vacuum
c0 c0 to observer
\c= = m + m (I ) 17. (d) Let a1 = a, I1 = a1 2 = a2
m 0 2
As I is decreasing with increasing radius, it a2 = 2a, I2 = a22 = 4a2
is maximum on the axis of the beam. I2 = 4I1
Therefore, c is minimum on the axis of the Ir = a12 + a22 + 2a1a2 cos f
beam.
13. (b) A phase change of p rad appears when the = I1 + I 2 + 2 I1 I2 cos f
ray reflects at the glass-air interface. Also,
the centre of the interference pattern is dark. Ir = I1 + 4 I1 + 2 4 I12 cos f
14. (a) Path difference at p
Dx1 = 0 Þ Ir = 5I1 + 4 I1 cos f … (1)
\ Phase difference at P Now, I max = ( a1 + a2 ) 2 = ( a + 2a ) 2 = 9 a 2
Df1 = 0°
I
Intensity at p Imax = 9I1 Þ I1 = max
I1 = I 0 + I 0 + 2 I 0 cos 0° = 4 I 0
9
Substituting in equation (1)
Path difference at Q
5I 4I
l Ir = max + max cos f
Dx2 = 9 9
4
Imax
\ Phase difference at Q Ir = [ 5 + 4 cos f]
2p l æ p ö 9
DfD2 = . =ç ÷ I é f ù
l 4 è 2ø Ir = max ê 5 + 8 cos2 - 4 ú
Intensity at Q. 9 ë 2 û
p I fù
I 2 = I 0 + I 0 + 2 I0 cos = 2 I 0 é
2 Ir = max ê1 + 8 cos 2 ú
9 ë 2û
I1 4 I0 2
Thus, = =
I 2 2 I0 1
www.crackjee.xyz
Wave Optics P-155

18. (c) Relation between intensities 22. (a) Given geometrical spread = a
45° B l lL
I0 (I0/2) Diffraction spread = ´L=
a a
(unpolarised) IR lL
The sum b = a +
a
A For b to be minimum
æI ö I 1 I db d æ lL ö
Ir = ç 0 ÷ cos 2 (45°) = 0 ´ = 0 =0 ça + ÷=0
è 2ø 2 2 4 da da è a ø
19. (d) It will be concentric circles. a = lL
20. (d) According to malus law, intensity of
emerging beam is given by, b min = lL + lL = 2 lL = 4 lL
I = I0cos2q 23. (d) For common maxima, n1l1 = n2l2
Now, IA' = I A cos230º
n1 l 2 520 ´10-9 4
2
I B ' = I B cos 60º Þ = = =
n2 l1 650 ´ 10-9 5
As I A ' = I B ' For l1
3 1 IA 1 n1l1 D
Þ I A ´ = IB ´ ; I = 3 y= , l1 = 650 nm `
4 4 B d
0.25 1
21. (d) sin q = = 4 ´ 650 ´ 10 -9 ´ 1.5
25 100 y= or, y = 7.8 mn
0.5 ´ 10 -3

0.25cm q

25cm
1.22 l
Resolving power = = 30 mm.
2 m sin q
EBD_7764
www.crackjee.xyz
P-156 Physics

Dual Nature of
Radiation and Matter 25
1. Sodium and copper have work functions 2.3 eV incident radiation gives a straight line whose
and 4.5 eV respectively. Then the ratio of the slope [2004]
wavelengths is nearest to [2002] (a) depends both on the intensity of the
(a) 1 : 2 (b) 4 : 1 radiation and the metal used
(c) 2 : 1 (d) 1 : 4 (b) depends on the intensity of the radiation
2. Formation of covalent bonds in compounds (c) depends on the nature of the metal used
exhibits [2002] (d) is the same for the all metals and independent
(a) wave nature of electron of the intensity of the radiation
(b) particle nature of electron 6. The work function of a substance is 4.0 eV. The
(c) both wave and particle nature of electron longest wavelength of light that can cause
(d) none of these photoelectron emission from this substance is
3. Two identical photocathodes receive light of approximately [2004]
frequencies f1 and f2. If the velocites of the photo (a) 310 nm (b) 400 nm
electrons (of mass m ) coming out are (c) 540 nm (d) 220 nm
respectively v1 and v2, then [2003] 7. A photocell is illuminated by a small bright source
2 2 2h placed 1 m away. When the same source of light
(a) v1 - v2 = ( f - f2 )
m 1 1
is placed m away, the number of electrons
1/ 2 2
é 2h ù emitted by photocathode would [2005]
(b) v1 + v2 = ê ( f1 + f 2 )ú
ëm û (a) increase by a factor of 4
2 2 2h (b) decrease by a factor of 4
(c) v1 + v2 = (f + f )
m 1 2 (c) increase by a factor of 2
é 2h ù
1/ 2 (d) decrease by a factor of 2
(d) v1 - v2 = ê ( f1 - f 2 )ú 8. If the kinetic energy of a free electron doubles,
ëm û
4. A radiation of energy E falls normally on a it’s deBroglie wavelength changes by the factor
perfectly reflecting surface. The momentum [2005]
transferred to the surface is [2004] 1
(a) 2 (b)
(a) Ec (b) 2 E / c 2
(c) E / c (d) E / c 2 1
(c) 2 (d)
5. According to Einstein’s photoelectric equation, 2
the plot of the kinetic energy of the emitted photo 9. The threshold frequency for a metallic surface
electrons from a metal vs the frequency, of the corresponds to an energy of 6.2 eV and the
www.crackjee.xyz
Dual Nature of Radiation and Matter P-157

stopping potential for a radiation incident on Directions: Question No. 13 and 14 are based on the
this surface is 5 V. The incident radiation lies in following paragraph.
[2006] Wave property of electrons implies that they will show
(a) ultra-violet region diffraction effects. Davisson and Germer demonstrated
this by diffracting electrons from crystals. The law
(b) infra-red region
governing the diffraction from a crystal is obtained
(c) visible region by requiring that electron waves reflected from the
(d) X-ray region planes of atoms in a crystal interfere constructively
10. The time taken by a photoelectron to come out (see figure).
after the photon strikes is approximately [2006]
(a) 10–4 s (b) 10–10 s
(c) 10–16 s (d) 10–1 s Inco m g
11. The anode voltage of a photocell is kept fixed. Electr ing i Outgoin s
ons c tro n
Ele
The wavelength l of the light falling on the
cathode is gradually changed. The plate current d
I of the photocell varies as follows [2006]
Crystal plane
13. Electrons accelerated by potential V are
(a) I diffracted from a crystal. If d = 1Å and i = 30°, V
should be about [2008]
(h = 6.6 × 10 – 34 Js, me = 9.1 × 10–31 kg, e = 1.6
O l × 10 – 19 C)
(a) 2000 V (b) 50 V
(c) 500 V (d) 1000 V
(b) I 14. If a strong diffraction peak is observed when
electrons are incident at an angle ‘i’ from the
normal to the crystal planes with distance ‘d’
O l between them (see figure), de Broglie wavelength
ldB of electrons can be calculated by the
relationship ( n is an integer) [2008]
I (a) d sin i = nldB (b) 2d cos i = nldB
(c)
(c) 2d sin i = nldB (d) d cos i = nldB

O l 15. The surface of a metal is illuminted with the light


of 400 nm. The kinetic energy of the ejected
photoelectrons was found to be 1.68 eV. The
work function of the metal is : [2009]
I
(d) (hc = 1240 eV.nm)
(a) 1.41 eV (b) 1.51 eV
O l (c) 1.68 eV (d) 3.09 eV
12. Photon of frequency n has a momentum Question (16–18) has Statement – 1 and
associated with it. If c is the velocity of light, the Statement – 2. Of the four choices given after
momentum is [2007] the statements, choose the one that best
(a) hn / c (b) n /c describes these two statements. [2011]
(c) h n c (d) hn / c2
EBD_7764
www.crackjee.xyz
P-158 Physics
(a) Statement -1 is true, Statement -2 is true ;
Statement -2 is the correct explanation of I
Statement -1.
(b) Statement -1 is true, Statement -2 is true; (a)
Statement -2 is not the correct explanation
of Statement -1 O l
(c) Statement -1 is false, Statement -2 is true.
(d) Statement -1 is true, Statement -2 is false. I
16. Statement -1 : When ultraviolet light is incident
on a photocell, its stopping potential is V0 and
(b)
the maximum kinetic energy of the
photoelectrons is Kmax .When the ultraviolet O l
light is replaced by X-rays, both V0 and Kmax
increase. I
Statement -2 : Photoelectrons are emitted with
speeds ranging from zero to a maximum value
because of the range of frequencies present in (c)
the incident light. [2010] O l
17. Statement – 1: A metallic surface is irradiated by
a monochromatic light of frequency v > v0 (the I
threshold frequency). The maximum kinetic
energy and the stopping potential are Kmax and
V0 respectively. If the frequency incident on the (d)
surface is doubled, both the Kmax and V0 are O l
also doubled.
20. The radiation corresponding to 3 ® 2 transition
Statement – 2 : The maximum kinetic energy and
of hydrogen atom falls on a metal surface to
the stopping potential of photoelectrons emitted
produce photoelectrons. These electrons are
from a surface are linearly dependent on the made to enter a magnetic field of 3 × 10–4 T. If
frequency of incident light. the radius of the largest circular path followed
18. Statement 1: Davisson-Germer experiment by these electrons is 10.0 mm, the work function
established the wave nature of electrons. of the metal is close to: [2014]
Statement 2 : If electrons have wave nature, (a) 1.8 eV (b) 1.1 eV
they can interfere and show diffraction.[2012] (c) 0.8 eV (d) 1.6 eV
(a) Statement 1 is false, Statement 2 is true. 21. Match List - I (Fundamental Experiment) with
(b) Statement 1 is true, Statement 2 is false List - II (its conclusion) and select the correct
(c) Statement 1 is true, Statement 2 is true, option from the choices given below the list:
Statement 2 is the correct explanation of [2015]
statement 1 Lis t-I Lis t-II
(d) Statement 1 is true, Statement 2 is true, A . Fran ck-Hertz (i) Particle n atu re of
Statement 2 is not the correct explanation Exp eriment ligh t
of Statement 1 B. Pho to -electric (ii) Dis crete en erg y
19. The anode voltage of a photocell is kept fixed. experimen t levels of atom
The wavelength l of the light falling on the C. Davis on -Germer (iii) W av e natu re o f
experimen t electro n
cathode is gradually changed. The plate current
(iv ) Structu re o f ato m
I of the photocell varies as follows : [2013]
www.crackjee.xyz
Dual Nature of Radiation and Matter P-159

(a) (A)-(ii); (B)-(i); (C)-(iii)


(b) (A)-(iv); (B)-(iii); (C)-(ii)
(c) (A)-(i); (B)-(iv); (C)-(iii) (a)
(d) (A)-(ii); (B)-(iv); (C)-(iii)
22. Radiation of wavelength l, is incident on a
photocell. The fastest emitted electron has speed
3l
v. If the wavelength is changed to , the
4
speed of the fastest emitted electron will be: (b)
[2016]
1 1
æ 4 ö2 æ 3 ö2
(a) vç
ç ÷ ÷ (b) vç
ç ÷ ÷
è3ø è4ø
1 1
æ 4 ö2 æ 4 ö2 (c)
(c) > v çç ÷÷ (d) < v çç ÷÷
è3ø è3ø
23. An electron beam is accelerated by a potential
difference V to hit a metallic target to produce
X-rays. It produces continuous as well as
characteristic X-rays.If lmin is the smallest
possible wavelength of X-ray in the spectrum, (d)
the variation of log lmin with log V is correctly
represented in : [2017]

Answer Key
1 2 3 4 5 6 7 8 9 10 11 12 13 14 15
(c) (a) (a) (b) (d) (a) (a) (d) (a) (b) (b) (a) (b) (b) (a)
16 17 18 19 20 21 22 23
(d) (c) (c) (d) (b) (a) (c) (c)

1. (c) We know that work function is the energy 1 2


required and energy E = hu hf1 - W = mv ....(i)
2 1
E Na h u Na l Cu For another photo cathode
\ = =
E Cu h uCu l Na 1 2
hf 2 - W = mv ....(ii)
é 1 ù 2 2
êëQ u µ l for light úû Subtracting (ii) from (i) we get
1 1
l Na E Cu 4.5 2
\ l = E = 2.3 » 1
( hf1 - W ) – ( hf 2 - W ) = 2 mv12 - 2 mv22
Cu Na
m 2 2
2. (a) Formation of covalent bond is best \ h( f1 - f 2 ) = (v1 - v2 )
explained by molecular orbital theory. 2
3. (a) For one photocathode
EBD_7764
www.crackjee.xyz
P-160 Physics

2 2h
2 hc
\ v1 - v 2 = ( f - f2 ) Þl=
m 1 f + eV0
E -34 8
4. (b) Momentum of photon = 6.6 ´ 10 ´ 3 ´ 10 -7
c = -19
m » 10
2E 1.6 ´ 10 (6.2 + 5)
Change in momentum = This range lies in ultra violet range.
c
= momentum transferred to the surface 10. (b) The order of time is nano second.
(the photon will reflect with same 11. (b) As l decreases, y increases and hence the
magnitude of momentum in opposite speed of photoelectron increases. The
direction) chances of photo electron to meet the
5. (d) From Equation K .E = hn - f anode increases and hence photo electric
slope of graph of K.E & n is h (Plank's current increases.
constant) 12. (a) Energy of a photon of frequency n is given
which is same for all metals
6. (a) For the longest wavelength to emit photo by E = h n .
electron Also, E = mc2, mc2 = hn
hc hc hn hn
=fÞl = Þ mc = Þ p=
l f c c
6.63 ´ 10-34 ´ 3 ´ 108 13. (b) The path difference between the rays APB
Þl= = 310 nm and CQD is
-16
40 ´ 1.6 ´ 10
2 Dx = MQ + QN = d cos i + d cos i
I ær ö
I1 1 Dx = 2d cos i
7. (a) I µ 2; = ç 2÷ =
r I 2 è r1 ø 4 A B
I 2 ® 4 times I1
C D
When intensity becomes 4 times, no. of
photoelectrons emitted would increase by i
4 times, since number of electrons emitted
per second is directly proportional to P
d N
intensity. M i
8. (d) de-Broglie wavelength, Q
h h We know that for constructive interference
l= =
p 2. m.(K.E) the path difference is nl

1 \ nl = 2d cos i
\ lµ
K. E Also by de-broglie concept
If K.E is doubled, wavelength becomes h h h
l l= = =
p 2mK.E 2meV
2
nh
9. (a) f = 6.2 eV = 6.2 ´ 1.6 ´10 -19 J \ = 2d cos i
2meV
V = 5 volt
2
hc h
- f = eV0 Here n =1 : V =
l 8med 2 cos 2 i
www.crackjee.xyz
Dual Nature of Radiation and Matter P-161

(6.6 ´ 10 -34 )2 B2 r 2 e
=
8 ´ 9.1´ 10 -31
´1.6 ´10 -19
´ (10 -10 2 2
) ´ cos 30 Þ V= = 0.8V
2m
= 50 V For transition between 3 to 2.
14. (b) 2d cos i = nldB
æ 1 1 ö 13.6 ´ 5
15. (a) l = 400 nm, hc = 1240 eV.nm, K.E. =1.68 eV E = 13.6 ç - ÷ = = 1.88eV
è 4 9ø 36
We know that,
Work function = 1.88 eV – 0.8 eV = 1.08 eV
hc hc » 1.1eV
- W = K .E Þ W = - K .E
l l 21. (a) Frank-Hertz experiment - Discrete energy
1240 levels of atom Photoelectric effect - Particle
ÞW= - 1.68 = 3.1 – 1.68 = 1.42 eV nature of light
400
Davison - Germer experiment - wave nature
16. (d) We know that
of electron.
eV0 = K max = hn - f 1 2
22. (c) hn02 – hn0 = mv
where, f is the work function . 2
Hence, as n increases (note that frequency 4 1
\ hn 0 - hn 0 = mv ' 2
of X-rays is greater than that of U.V. rays), 3 2
both V0 and Kmax increase. So statement -
4 4
1 is correct n - n0 n - n0
v '2
3
17. (c) By Einstein photoelectric equation, \ = \ v' = v 3
Kmax = eV0 = hv – hv0 v2 n - n0 n - n0
When v is doubled, Kmax and V0 become 4
more than double. \ v' > v
3
18. (c)
19. (d) As l is increased, there will be a value of l hc
23. (c) In X-ray tube, l min =
above which photoelectrons will be cease eV
to come out so photocurrent will become æ hc ö
In l min = In ç ÷ - InV
zero. Hence (d) is correct answer. è eø
20. (b) Radius of circular path followed by electron Clearly, log lmin versus log V graph slope
is given by, is negative hence option (c) correctly
mu 1 2m depicts.
r= 2meV
= = V
qB eB B e
EBD_7764
www.crackjee.xyz
P-162 Physics

Atoms 26
1. If 13.6 eV energy is required to ionize the the most energy? [2005]
hydrogen atom, then the energy required to
n=4
remove an electron from n = 2 is [2002] n=3
(a) 10.2 eV (b) 0 eV
(c) 3.4 eV (d) 6.8 eV n=2
2. Which of the following atoms has the lowest
ionization potential ? [2003]
14 133 n =1
(a) N (b) Cs I II III IV
7 55
40 16 (a) IV (b) III
(c) Ar (d) O
18 8 (c) II (d) I
3. The wavelengths involved in the spectrum of 7. Which of the following transitions in hydrogen
deuterium ( )
2
1D are slightly different from that atoms emit photons of highest frequency?
[2007]
of hydrogen spectrum, because [2003] (a) n = 1 to n = 2 (b) n = 2 to n = 6
(a) the size of the two nuclei are different (c) n = 6 to n = 2 (d) n = 2 to n = 1
(b) the nuclear forces are different in the two
8. Suppose an electron is attracted towards the
cases k
(c) the masses of the two nuclei are different origin by a force where ‘k’ is a constant and
r
(d) the attraction between the electron and the ‘r’ is the distance of the electron from the origin.
nucleus is differernt in the two cases By applying Bohr model to this system, the
4. If the binding energy of the electron in a hydrogen radius of the nth orbital of the electron is found
atom is 13.6eV, the energy required to remove the to be ‘rn’ and the kinetic energy of the electron
electron from the first excited state of Li + + is to be ‘Tn’. Then which of the following is true?
[2003] [2008]
(a) 30.6 eV (b) 13.6 eV
1 2
(c) 3.4 eV (d) 122.4 eV (a) Tn µ , rn µ n
2
5. The manifestation of band structure in solids is n
due to [2004] (b) Tn independent of n, rn µ n
(a) Bohr’s correspondence principle
1
(b) Pauli’s exclusion principle (c) Tn µ , rn µ n
(c) Heisenberg’s uncertainty principle n
(d) Boltzmann’s law 1
(d) Tn µ , rn µ n 2
6. The diagram shows the energy levels for an n
electron in a certain atom. Which transition
shown represents the emission of a photon with
www.crackjee.xyz
Atoms P-163

9. The transition from the state n = 4 to n = 3 in a 14. As an electron makes a transition from an excited
hydrogen like atom results in ultraviolet state to the ground state of a hydrogen - like
radiation. Infrared radiation will be obtained in atom/ion : [2015]
the transition from : [2009] (a) kinetic energy decreases, potential energy
(a) 3 ® 2 (b) 4 ® 2 increases but total energy remains same
(c) 5 ® 4 (d) 2 ® 1 (b) kinetic energy and total energy decrease
10. Energy required for the electron excitation in but potential energy increases
Li++ from the first to the third Bohr orbit is : (c) its kinetic energy increases but potential
[2011]
energy and total energy decrease
(a) 36.3 eV (b) 108.8 eV
(d) kinetic energy, potential energy and total
(c) 122.4 eV (d) 12.1 eV
energy decrease
11. Hydrogen atom is excited from ground state to
another state with principal quantum number 15. A particle A of mass m and initial velocity v
equal to 4. Then the number of spectral lines in m
the emission spectra will be : [2012] collides with a particle B of mass which is at
2
(a) 2 (b) 3
rest. The collision is head on, and elastic. The
(c) 5 (d) 6
ratio of the de-Broglie wavelengths lA to lB
12. In a hydrogen like atom electron make transition
after the collision is [2017]
from an energy level with quantum number n to
another with quantum number (n – 1). If n>>1, lA 2 lA 1
(a) = (b) l = 2
the frequency of radiation emitted is proportional lB 3 B
to : [2013] lA 1 lA
1 1 (c) = (d) l = 2
(a) (b) lB 3 B
n n2 16. Some energy levels of a molecule are shown in
1 1 the figure. The ratio of the wavelengths
(c) (d)
n 3
n3 r = l1/l2, is given by [2017]
2
13. Hydrogen ( 1 H1 ), Deuterium ( 1 H2 ), singly
ionised Helium (2He4)+, and doubly ionised
lithium (3Li6)++ all have one electron around the
nucleus. Consider an electron transition from n
= 2 to n = 1. If the wavelengths of emitted
radiation are l1, l2, l3 and l4 respectively then
approximately which one of the following is 1
3
correct? [2014] (a) r= (b) r=
4 3
(a) 4l1 = 2l 2 = 2l3 = l 4
4 2
(b) l1 = 2l 2 = 2l3 = l 4 (c) r= (d) r=
3 3
(c) l1 = l 2 = 4l3 = 9l 4
(d) l1 = 2l 2 = 3l3 = 4l 4

Answer Key
1 2 3 4 5 6 7 8 9 10 11 12 13 14 15
(c) (b) (c) (a) (b) (b) (d) (b) (c) (b) (d) (d) (c) (c) (d)
16
(b)
EBD_7764
www.crackjee.xyz
P-164 Physics

1. (c) The energy of nth orbit of hydrogen is


é ù
given by which ê 1 - 1 ú is maximum. Here n2
13.6 êë n12 n2 2 úû
En = - eV /atom
n2 is the higher energy level.
-13.6 é 1 1 ù
For n = 2, En = = -3.4 eV Clearly, ê - ú is maximum for the
4
êë n12 n2 2 úû
Therefore the energy required to remove
electron from n = 2 is + 3.4 eV. third transition, i.e. 2 ® 1 . I transition
2. (b) The ionisation potential increases from left represents the absorption of energy.
to right in a period and decreases from top 7. (d) We have to find the frequency of emitted
to bottom in a group. photons. For emission of photons the
Therefore ceasium will have the lowest transition must take place from a higher
ionisation potential. energy level to a lower energy level which
3. (c) The wavelength of spectrum is given by are given only in options (c) and (d).
Frequency is given by
1 æ 1 1ö
= Rz 2 ç 2 - 2 ÷ æ 1 1ö
l çè n n2 ÷ø hn = -13.6 ç - ÷
1 çè n 2 n 2 ÷ø
2 1
7 For transition from n = 6 to n = 2,
1.097 ´ 10
where R =
1+
m -13.6 æ 1 1 ö 2 æ 13.6 ö
n1 = ç - 2÷ = ´ç ÷
M h è6 2
2 ø 9 è h ø
where m = mass of electron For transition from n = 2 to n = 1,
M = mass of nucleus.
-13.6 æ 1 1 ö 3 æ 13.6 ö
For different M, R is different and therefore n2 = - = ´ç ÷.
l is different h çè 22 12 ÷ø 4 è h ø

13.6 \ n1 > n2
4. (a) 2
En = - Z eV/atom
n 2
8. (b) When F = k = centripetal force, then
r
For lithium ion Z = 3 ; n = 2 ( for first 2
excited state) k mv
=
r r
13.6 Þ mv 2 = constat Þ kinetic energy is
En = - 2
´ 32 = -30.6 eV
2 constant Þ T is independent of n.
5. (b) Pauli’s exclusion principle. 9. (c) It is given that transition from the state n
=4 to n = 3 in a hydrogen like atom result in
é 1 1 ù ultraviolet radiation. For infrared radiation
6. (b) E = Rhc ê 2 - 2 ú the energy gap should be less. The only
êë n1 n2 úû
option is 5 ® 4 .
E will be maximum for the transition for
www.crackjee.xyz
Atoms P-165

Increasing
Energy ze 2 k ze 2
n=5 14. (c) U = –K ; T.E = –
r 2 r
n=4
k ze 2
n=3 K.E = . Here r decreases
2 r
n=2
m
n=1 15. (d) From question, mA = M; mB =
2
10. (b) Energy of excitation,
uA = V uB = 0
æ 1 1ö
D E = 13.6 p2 çè h - h ÷ø eV Let after collision velocity of A = V1 and
1 2
velocity of B = V2
æ1 1ö Applying law of conservation of momentum,
Þ DE = 13.6 (3)2 çè - ÷ = 108.8 eV
12
32 ø
æ mö
11. (d) For ground state, the principal quantum mu = mv1 + ç ÷ v2
è2ø
no. (n) = 1. There is a 3rd excited state for
or, 24= 2v1 + v2 ....(i)
principal quantum number.
By law of collision
Pincipal quantum no. (n)

4 3rd excited
state v2 - v1
e=
Energy states

u-0
3 2nd excited or, u = v2 – v1 ....(ii)
state
[Q collision is elastic, e = 1]
2 1st excited
state using eqns (i) and (ii)
1 ground state 4 4
Possible number of spectral lines v1 = and v 2 = u
1444444 424444444 3 3 3
The possible number of the spectral lines h
is given de-Broglie wavelength l =
n(n - 1) 4(4 - 1) p
= = =6
2 2 m 4
´ u
12. (d) DE = hv l A PB 2 3 =2
\ = =
l B PA 4
DE é 1 1 ù k(2n - 1) m´
n= =kê 2
- 2ú = 2 3
h ë (n - 1) n û n (n - 1)2
hc
16. (b) From energy level diagram, using DE =
2k 1 l
» 3 or n µ 3
n n hc
For wavelength l1 DE = – E – (–2E) =
l1
1 é 1 1 ù
13. (c) Wave number = RZ 2 ê - ú hc
l êë n1 n2 úû
2 \ l1 =
E
æ 4E ö hc
Þ lµ
1 For wavelength l2 DE = – E – ç - ÷ =
è 3 ø l2
Z2
hc l 1
By question n = 1 and n 1 = 2 \ l2 = \r= 1 =
æ Eö l2 3
Then, l1 = l2 = 4l3 = 9l4 çè ÷ø
3
EBD_7764
www.crackjee.xyz
P-166 Physics

Nuclei 27
1. At a specific instant emission of radioactive (a) Protons (b) Neutrinoes
compound is deflected in a magnetic field. The (c) Helium nuclei (d) Electrons
compound can emit 7. In the nuclear fusion reaction
(i) electrons (ii) protons 2 3 4
1 H + 1 H ® 2 He + n
(iii) He2+ (iv) neutrons
given that the repulsive potential energy
The emission at instant can be [2002]
(a) i, ii, iii (b) i, ii, iii, iv between the two nuclei is ~ 7.7 ´ 10 -14 J , the
(c) iv (d) ii, iii temperature at which the gases must be heated
2. If N0 is the original mass of the substance of to initiate the reaction is nearly
half-life period t1/2 = 5 years, then the amount of [Boltzmann’s Constant k = 1.38 ´ 10 -23 J/K ]
substance left after 15 years is [2002] [2003]
(a) N0/8 (b) N0/16 (a) 10 7 K (b) 10 5 K
(c) N0/2 (d) N0/4
238 (c) 10 3 K (d) 10 9 K
3. When a U nucleus originally at rest, decays
8. A nucleus disintegrated into two nuclear parts
by emitting an alpha particle having a speed ‘u’, which have their velocities in the ratio of 2 : 1.
the recoil speed of the residual nucleus is [2003] The ratio of their nuclear sizes will be [2004]
(a) 4u (b) - 4u (a) 3½ : 1 (b) 1:21/3
238 234 1/3
(c) 2 :1 (d) 1:3½
4u
(c) (d) - 4u 9. The binding energy per nucleon of deuteron

4.
234 238
A radioactive sample at any instant has its ( H) and helium nucleus ( He) is 1.1 MeV
2
1
4
2
disintegration rate 5000 disintegrations per and 7 MeV respectively. If two deuteron nuclei
minute. After 5 minutes, the rate is 1250 react to form a single helium nucleus, then the
disintegrations per minute. Then, the decay energy released is [2004]
constant (per minute) is [2003] (a) 23.6 MeV (b) 26.9 MeV
(a) 0.4 ln 2 (b) 0.2 ln 2 (c) 13.9 MeV (d) 19.2 MeV
27
10. If radius of the 13 Al nucleus is estimated to be
(c) 0.1 ln 2 (d) 0.8 ln 2
5. A nucleus with Z= 92 emits the following in a 3.6 fermi then the radius of 125
52 Te nucleus be
sequence: nearly [2005]
(a) 8 fermi (b) 6 fermi
a, b - , b - a, a, a, a, a, b - , b - , a, b + , b + , a
(c) 5 fermi (d) 4 fermi
Then Z of the resulting nucleus is [2003] 7
11. Starting with a sample of pure 66 Cu , of it
(a) 76 (b) 78 8
decays into Zn in 15 minutes. The corresponding
(c) 82 (d) 74
half life is [2005]
6. Which of the following cannot be emitted by
radioactive substances during their decay? (a) 15 minutes (b) 10 minutes
1
[2003] (c) 7 minutes (d) 5 minutes
2
www.crackjee.xyz
Nuclei P-167

12. The intensity of gamma radiation from a given respectively, then in the reaction
source is I. On passing through 36 mm of lead, it p + 73 Li ¾¾
® 2 42 He
I
is reduced to . The thickness of lead which will energy of proton must be [2006]
8
I (a) 28.24 MeV (b) 17.28 MeV
reduce the intensity to will be [2005] (c) 1.46 MeV (d) 39.2 MeV
2
(a) 9 mm (b) 6mm 17. The 'rad' is the correct unit used to report the
(c) 12mm (d) 18mm measurement of [2006]
13. A nuclear transformation is denoted by X (n, a ) (a) the ability of a beam of gamma ray photons
7 . to produce ions in a target
3 Li Which of the following is the nucleus of
(b) the energy delivered by radiation to a target
element X ? [2005] (c) the biological effect of radiation
(a) 10
(b) 12
5 B C6 (d) the rate of decay of a radioactive source
(c) 11 (d) 9 18. If M O is the mass of an oxygen isotope
4 Be 5B
17
8 O ,MP and MN are the masses of a proton
14. 7
When 3Li nuclei are bombarded by protons,
and the resultant nuclei are 4Be8, the emitted and a neutron respectively, the nuclear binding
particles will be [2006] energy of the isotope is [2007]
(a) alpha particles (b) beta particles (a) (MO –17MN)c2
(c) gamma photons (d) neutrons (b) (MO – 8MP)c2
15. The energy spectrum of b-particles [number N(E) (c) (MO– 8MP –9MN)c2
as a function of b-energy E] emitted from a (d) MOc 2
radioactive source is [2006] 19. In gamma ray emission from a nucleus [2007]
(a) only the proton number changes
(b) both the neutron number and the proton
(a) N(E)
number change
E
(c) there is no change in the proton number
E0
and the neutron number
(d) only the neutron number changes
20. The half-life period of a radio-active element X
(b) N(E)
is same as the mean life time of another radio-
active element Y. Initially they have the same
E
E0 number of atoms. Then [2007]
(a) X and Y decay at same rate always
(b) X will decay faster than Y
(c) N(E)
(c) Y will decay faster than X
(d) X and Y have same decay rate initially
E
E0 21. This question contains Statement-1 and
statement-2. Of the four choices given after the
statements, choose the one that best describes
(d) N(E) the two statements. [2008]
Statement-1:
E
E0 Energy is released when heavy nuclei undergo
fission or light nuclei undergo fusion and
16. If the binding energy per nucleon in 73 Li and Statement-2 :
4
2 He nuclei are 5.60 MeV and 7.06 MeV For heavy nuclei, binding energy per nucleon
EBD_7764
www.crackjee.xyz
P-168 Physics
increases with increasing Z while for light nuclei (c) E2 > E1 (d) E1 = 2 E2
it decreases with increasing Z.
(a) Statement-1 is false, Statement-2 is true 24. The speed of daughter nuclei is [2010]
(b) Statement-1 is true, Statement-2 is true;
Statement-2 is a correct explanation for Dm 2Dm
(a) c (b) c
Statement-1 M + Dm M
(c) Statement-1 is true, Statement-2 is true;
Statement-2 is not a correct explanation for Dm Dm
(c) c (d) c
Statement-1 M M + Dm
(d) Statement-1 is true, Statement-2 is false
25. A radioactive nucleus (initial mass number A and
atomic number Z emits 3 a - particles and 2
22. positrons. The ratio of number of neutrons to
that of protons in the final nucleus will be [2010]
A- Z -8 A- Z -4
(a) (b)
Z -4 Z -8
A - Z - 12 A- Z -4
(c) (d)
Z -4 Z -2
26. The half life of a radioactive substance is 20
minutes. The approximate time interval (t2 – t1)
2
between the time t2 when of it had decayed
3
The above is a plot of binding energy per 1
nucleon Eb, against the nuclear mass M; A, B, C, and time t 1 when of it had decayed
3
D, E, F correspond to different nuclei. Consider
four reactions : [2009] is : [2011]
(i) A + B ® C + e (a) 14 min (b) 20 min
(ii) C ® A + B + e (c) 28 min (d) 7 min
27. After absorbing a slowly moving neutron of mass
(iii) D + E ® F + e and
mN (momentum » 0) a nucleus of mass M breaks
(iv) F® D + E + e,
into two nuclei of masses m1 and 5m1 (6m1 = M
where e is the energy released? In which + mN) respectively. If the de Broglie wavelength
reactions is e positive? of the nucleus with mass m1 is l, the de Broglie
(a) (i) and (iii) (b) (ii) and (iv) wavelength of the nucleus will be [2011]
(c) (ii) and (iii) (d) (i) and (iv) (a) 5l (b) l / 5
DIRECTIONS: Questions number 23-24 are based on (c) l (d) 25l
the following paragraph. 28. Statement - 1 : A nucleus having energy E1
A nucleus of mass M + Dm is at rest and decays decays by b– emission to daughter nucleus
M having energy E2, but the b– rays are emitted
into two daughter nuclei of equal mass each. with a continuous energy spectrum having end
2
Speed of light is c. point energy E1 – E2.
Statement - 2 : To conserve energy and
23. The binding energy per nucleon for the parent momentum in b– decay at least three particles must
nucleus is E1 and that for the daughter nuclei is take part in the transformation. [2011 RS]
E2. Then [2010] (a) Statement-1 is correct but statement-2 is
(a) E2 = 2E1 (b) E1 > E2 not correct.
www.crackjee.xyz
Nuclei P-169

(b) Statement-1 and statement-2 both are 30. Half-lives of two radioactive elements A and B
correct and statement-2 is the correct are 20 minutes and 40 minutes, respectively.
explanation of statement-1. Initially, the samples have equal number of
(c) Statement-1 is correct, statement-2 is nuclei. After 80 minutes, the ratio of decayed
correct and statement-2 is not the correct number of A and B nuclei will be : [2016]
explanation of statement-1 (a) 1 : 4 (b) 5 : 4
(d) Statement-1 is incorrect, statement-2 is (c) 1 : 16 (d) 4 : 1
correct. 31. A radioactive nucleus A with a half life T, decays
29. Assume that a neutron breaks into a proton and into a nucleus B. At t = 0, there is no nucleus B.
an electron. The energy released during this At sometime t, the ratio of the number of B to that
process is : (mass of neutron = 1.6725 × 10–27 of A is 0.3. Then, t is given by [2017]
kg, mass of proton = 1.6725 × 10–27 kg, mass of T
(a) t = T log (1.3) (b) t =
electron = 9 × 10–31 kg). [2012] log(1.3)
(a) 0.51 MeV (b) 7.10 MeV log 2 log1.3
(c) t =T (d) t =T
(c) 6.30 MeV (d) 5.4 MeV log1.3 log 2

Answer Key
1 2 3 4 5 6 7 8 9 10 11 12 13 14 15
(a) (a) (c) (a) (b) (a) (d) (b) (a) (b) (d) (c) (a) (c) (c)
16 17 18 19 20 21 22 23 24 25 26 27 28 29 30
(b) (c) (c) (c) (c) (d) (d) (c) (b) (b) (b) (c) (b) (a) (b)
31
(d)

1. (a) Charged particles are deflected in magnetic 3. (c) Here, conservation of linear momentum can
field. be applied
2. (a) After every half-life, the mass of the
substance reduces to half its initial value. 4
234 He
N0 N0 / 2 2
5 years 5 years Th
N 0 ¾¾¾¾
® ¾¾¾¾
® 90
2 2
N0 5years N0 / 4 N0
= ¾¾¾¾
® =
4 2 8 238 × 0 = 4 u + 234 v
4
\ v=- u
15 234
Number of half lives n = =3
5 r 4
We know that \ speed =| v |= u
234
n 3
æ 1ö æ 1ö N
N
N= 0ç ÷ = N 0 ç ÷ = 0
è 2ø è 2ø 8
EBD_7764
www.crackjee.xyz
P-170 Physics

A 1 5000 1/ 3
1 10. (b) R = R0 ( A)
4. (a) l = log e o = log e
t A 5 1250 1/ 3 1/ 3
R1 æA ö æ 27 ö 3
\ =ç 1÷ =ç =
è 125 ÷ø
2
= log e 2 = 0.4 log e 2 R2 è A2 ø 5
5
5. (b) The number of a-particles released = 8 5
R2 = ´ 3.6 = 6 fermi
Therefore the atomic number should 3
decrease by 16 7
11. (d) of Cu decays in 15 minutes.
The number of b–-particles released = 4 8
Therefore the atomic number should 3
7 1 æ1ö
increase by 4. \ Cu undecayed = N = 1 – = =ç ÷
8 8 è2ø
Also the number of b+ particles released is \ No. of half lifes = 3
2, which should decrease the atomic
t 15
number by 2. n= or 3 =
T T
Therefore the final atomic number
15
= Z –16 + 4 – 2 = Z –14 Þ T = half life period = = 5 minutes
3
= 92 – 14 = 78
6. (a) The radioactive substances emit a -
N = N0 (1 – e–lt)
particles (Helium nucleus), b-particles
(electrons) and neutrinoes. N0 - N 1
Þ = e -lt \ =e
-lt
7. (d) The average kinetic energy per molecule N0 8
3 3 ´ 0.693
= kT 3 ln 2 = lt or l = = 0.1386
2 15
Half-lifeperiod,
This kinetic energy should be able to
0.693 0.693
provide the repulsive potential energy t½ = = = 5 minutes
l 0.1386
3 -14
\ kT = 7.7 ´ 10 12. (c) Intensity I = I 0 . e - md ,
2
-14 Applying logarithm on both sides,
2 ´ 7.7 ´ 10 9
ÞT = -23
= 3.7 ´ 10 æ I ö
3 ´ 1.38 ´ 10 - md = log ç ÷
8. (b) From conservation of momentum è I0 ø

m1v1 = m2 v2 æ I /8ö
- m ´ 36 = logç ÷ ..........(i)
è I ø
æ m ö æv ö v
Þ ç 1 ÷ = ç 2 ÷ given 1 = 2 æ I / 2ö
è m2 ø è v1 ø v2 -m ´ d = log ç ...........(ii)
è I ÷ø
3
m1 1 r1 1 æ r1 ö æ 1 ö 1/ 3 Dividing (i) by (ii),
Þ = Þ 3
= Þ ç r ÷ = çè 2 ÷ø
m2 2 r2 2 è 2ø æ 1ö æ 1ö
log ç ÷ 3log ç ÷
9. (a) The chemical reaction of process is 36 è 8ø è 2ø 36
= = = 3 or d =
d æ 1ö æ 1ö 3
212 H ®24 He log ç ÷ log ç ÷
è 2ø è 2ø
Energy released = 4 × (7) – 4(1.1) = 23.6 MeV = 12 mm
www.crackjee.xyz
Nuclei P-171

A æ dN ö
13. (a) ZX + 0 n1 ¾¾
® 3 Li7 + 2 He 4 -ç = l y N0
è dt ÷ø y
On comparison,
Y will decay faster than X.
A = 7 + 4 – 1 = 10, z = 3 + 2 – 0 = 5 21. (d) We know that energy is released when
It is boron 5B10 heavy nuclei undergo fission or light nuclei
7
undergo fusion. Therefore statement (1) is
14. (c) 3 Li + 11p ¾
¾® 8
4 Be + 0
0g correct.
15. (c) The range of energy of b-particles is from The second statement is false because for
zero to some maximum value. heavy nuclei the binding energy per
16. (b) Let E be the energy of proton, then nucleon decreases with increasing Z and
E + 7 ´ 5.6 = 2 ´ [4 ´ 7.06] for light nuclei, B.E/nucleon increases with
increasing Z.
Þ E = 56.48 - 39.2 = 17.28MeV 22. (d) For A + B ® C + e, e is positive. This is
17. (c) The risk posed to a human being by any because Eb for C is greater than the Eb for
radiation exposure depends partly upon A and B.
the absorbed dose, the amount of energy
absorbed per gram of tissue. Absorbed Again for F ® D + E + e, e is positive. This
dose is expressed in rad. A rad is equal to is because Eb for D and E is greater than
100 ergs of energy absorbed by 1 gram of Eb for F.
tissue. The more modern, internationally 23. (c) In nuclear fission, the binding energy per
adopted unit is the gray (named after the nucleon of daughter nuclei is always
English medical physicist L. H. Gray); one greater than the parent nucleus.
gray equals 100 rad. 24. (b) By conservation of energy,
18. (c) Binding energy 2.M 2 1 2M 2
= [ZMP + (A – Z)MN – M]c2 ( M + Dm) c2 = c + . v ,
2 2 2
= [8MP + (17 – 8)MN – M]c2 where v is the speed of the daughter nuclei
= [8MP + 9MN – M]c2
2 M 2
= [8MP + 9MN – Mo]c2 Þ Dmc = v
2
19. (c) There is no change in the proton number
and the neutron number as the g-emission 2 Dm
takes place as a result of excitation or de- \v=c
M
excitation of nuclei. g-rays have no charge 25. (b) As a result of emission of 1 a-particle, the
or mass. mass number decreases by 4 units and
20. (c) According to question, atomic number decreases by 2 units. And
Half life of X, T1/2 = tav , average life of Y by the emission of 1 positron the atomic
number decreases by 1 unit but mass
0.693 1 number remains constant.
Þ =
lX lY \ Mass number of final nucleus = A – 12
Atomic number of final nucleus = Z – 8
Þ l X = (0.693). l Y
\ Number of neutrons = (A – 12) – (Z – 8)
\ lX < lY . =A–Z –4
Now, the rate of decay is given by Number of protons = Z – 8

æ dN ö A-Z -4
-ç = l X N0 \ Required ratio =
è dt ÷ø x Z -8
EBD_7764
www.crackjee.xyz
P-172 Physics
26. (b) Number of undecayed atom after time t2 ; = – 9 × 10–31 kg
N0 The energy released during the process
= N 0 e -lt2 ...(i) E = Dmc2
3
E = 9 × 10–31× 9 × 1016 = 81 × 10–15 Joules
Number of undecayed atom after time t1;
2N0 81 ´ 10 -15
= N 0 e -lt1 ...(ii) E= = 0.511MeV
3 1.6 ´ 10 –19
-lt 1 30. (b) For At½ = 20 min, t = 80 min, number of half
From (i), e 2 = lifes n = 4
3
No
æ 1ö \ Nuclei remaining = 4 . Therefore nuclei
Þ–lt2 = loge çè ÷ø ...(iii) 2
3 decayed
2 No
From (ii) – e -lt2 =
3 = N0 - 4
2
æ 2ö For Bt½ = 40 min., t = 80 min, number of half
Þ–lt1 = loge çè 3 ÷ø ...(iv) lifes n = 2
No
Solving (iii) and (iv), we get \ Nuclei remaining = 2 . Therefore
t2 – t1 = 20 min 2
nuclei decayed
27. (c) pi = 0
No
p f = p1 + p 2 = N0 - 2
2
pi = p f No 1
No - 4 1-
2 16
0 = p1 + p 2 \Required ratio = No = 1 =
No - 2 1-
2 4
p1 = - p 2
15 4 5
´ =
h 16 3 4
l1 =
p1 31. (d) Let initially there are total N0 number of
nuclei
h
l2 = NB
p2 At time t = 0.3(given)
NA
l1 = l 2 Þ NB = 0.3NA
l1 = l 2 = l. N0 = NA + NB = NA + 0.3NA
28. (b) Statement-1: Energy of b-particle from 0 N0
to maximum so E1 - E2 is the continuous \ NA =
1.3
energy spectrum. As we know Nt = N0e – lt
Statement-2: For energy conservation and
momentum conservation at least three N0
or, = N0e – lt
particles, daughter nucleus, b–1 and anti- 1.3
neutron are required. 1
1 = e–lt Þ ln(1.3) = lt
29. (a) 0n ® 11H + -1e 0
¾¾ +n+Q 1.3
The mass defect during the process
ln(1.3) ln (1.3) ln(1.3) T
Dm = mn - mH - me = 1.6725 × 10–27 or, t = Þ t= = ln(2)
l ln(2)
– (1.6725 × 10–27+ 9 × 10–31kg) T
www.crackjee.xyz

28
Semiconductor Electronics :
Materials, Devices and
Simple Circuits
1. At absolute zero, Si acts as [2002] (a) crystal sturcture
(a) non-metal (b) metal (b) variation of the number of charge carriers
(c) insulator (d) none of these with temperature
(c) type of bonding
2. By increasing the temperature, the specific
(d) variation of scattering mechanism with
resistance of a conductor and a semiconductor
temperature
[2002]
7. In the middle of the depletion layer of a reverse-
(a) increases for both
biased p-n junction, the [2003]
(b) decreases for both
(a) electric field is zero
(c) increases, decreases
(b) potential is maximum
(d) decreases, increases
(c) electric field is maximum
3. The energy band gap is maximum in [2002]
(d) potential is zero
(a) metals (b) superconductors
8. When npn transistor is used as an amplifier
(c) insulators (d) semiconductors.
[2004]
4. The part of a transistor which is most heavily
(a) electrons move from collector to base
doped to produce large number of majority
(b) holes move from emitter to base
carriers is [2002]
(c) electrons move from base to collector
(a) emitter
(d) holes move from base to emitter
(b) base
9. For a transistor amplifier in common emitter
(c) collector
configuration for load impedance of 1kW
(d) can be any of the above three. (h fe = 50 and hoe = 25) the current gain is
5. A strip of copper and another of germanium are
[2004]
cooled from room temperature to 80K. The
(a) – 24.8 (b) – 15.7
resistance of [2003]
(c) – 5.2 (d) – 48.78
(a) each of these decreases
10. A piece of copper and another of germanium are
(b) copper strip increases an d that of
cooled from room temperature to 77K, the
germanium decreases
resistance of [2004]
(c) copper strip decreases an d that of
(a) copper increases and germanium decreases
germanium increases
(b) each of them decreases
(d) each of these increases
(c) each of them increases
6. The difference in the variation of resistance with
temeperature in a metal and a semiconductor (d) copper decreases and germanium increases
arises essentially due to the difference in the 11. When p-n junction diode is forward biased then
[2003] [2004]
EBD_7764
www.crackjee.xyz
P-174 Physics

(a) both the depletion region and barrier height current of 5.60 mA. The value of the base current
are reduced amplification factor (b) will be [2006]
(b) the depletion region is widened and barrier (a) 49 (b) 50
height is reduced (c) 51 (d) 48
(c) the depletion region is reduced and barrier 17. A solid which is not transparent to visible light
height is increased and whose conductivity increases with
(d) Both the depletion region and barrier height temperature is formed by [2006]
are increased (a) Ionic bonding
12. The electrical conductivity of a semiconductor (b) Covalent bonding
increases when electromagnetic radiation of (c) Vander Waals bonding
wavelength shorter than 2480 nm is incident on (d) Metallic bonding
it. The band gap in (eV) for the semiconductor is 18. If the ratio of the concentration of electrons to
7
[2005] that of holes in a semiconductor is and the
5
(a) 2.5 eV (b) 1.1 eV 7
ratio of currents is , then what is the ratio of
(c) 0.7 eV (d) 0.5 eV 4
13. In a common base amplifier, the phase difference their drift velocities? [2006]
5 4
between the input signal voltage and output (a) (b)
8 5
voltage is [2005]
5 4
p (c) (d)
(a) p (b) 4 7
4 19. The circuit has two oppositively connected ideal
p diodes in parallel. What is the current flowing in
(c) (d) 0
2 the circuit? [2006]
14. In a full wave rectifier circuit operating from 50 4W
Hz mains frequency, the fundamental frequency
D1 D2
in the ripple would be [2005]
(a) 25 Hz (b) 50 Hz 12V
(c) 70.7 Hz (d) 100 Hz 3W 2W
15. If the lattice constant of this semiconductor is
decreased, then which of the following is correct? (a) 1.71 A (b) 2.00 A
[2006]
(c) 2.31 A (d) 1.33 A
conduction 20. In the following, which one of the diodes reverse
Ec
band width biased? [2006]
band gap Eg +10 V
valence
band width Ev
(a) R
(a) All Ec, Eg, Ev increase +5 V
(b) Ec and Ev increase, but Eg decreases
(b)
(c) Ec and Ev decrease, but Eg increases –10 V
(d) All Ec, Eg, Ev decrease R
16. In a common base mode of a transistor, the
–5 V
collector current is 5.488 mA for an emitter
www.crackjee.xyz
Semiconductor Electronics : Materials, Devices and Simple Circuits P-175

conduction is found between P and Q. By


(c) connecting the common (negative) terminal of
R the multimeter to R and the other (positive)
terminal to P or Q, some resistance is seen on
–10 V the multimeter. Which of the following is true
for the transistor? [2008]
+5 V (a) It is an npn transistor with R as base
(b) It is a pnp transistor with R as base
R (c) It is a pnp transistor with R as emitter
R (d) It is an npn transistor with R as collector
(d)
24. In the circuit below, A and B represent two inputs
and C represents the output. [2008]
A
21. If in a p-n junction diode, a square input signal
of 10 V is applied as shown [2007] C
5V
B
RL

-5V The circuit represents


Then the output signal across RL will be
(a) NOR gate (b) AND gate
+5V 10 V
(c) NAND gate (d) OR gate
(a) (b)
25. The logic circuit shown below has the input
waveforms ‘A’ and ‘B’ as shown. Pick out the
correct output waveform. [2009]
(c) (d)
-10 V -5V A
22. Carbon, silicon and germanium have four Y
valence electrons each. At room temperature B
which one of the following statements is most
Input A
appropriate ? [2007]
(a) The number of free electrons for
Input B
conduction is significant only in Si and Ge
but small in C.
(b) The number of free conduction electrons Output is
is significant in C but small in Si and Ge. (a)
(c) The number of free conduction electrons
is negligibly small in all the three.
(d) The number of free electrons for
(b)
conduction is significant in all the three.
23. A working transistor with its three legs marked
P, Q and R is tested using a multimeter. No
EBD_7764
www.crackjee.xyz
P-176 Physics
(c)
A

(d)
B

26. A p-n junction (D) shown in the figure can act (a) OR gate (b) NOT gate
as a rectifier. An alternating current source (V) is (c) XOR gate (d) NAND gate
connected in the circuit. [2009] 28. The output of an OR gate is connected to both
the inputs of a NAND gate. The combination
D will serve as a: [2011 RS]
R (a) NOT gate (b) NOR gate
(c) AND gate (d) OR gate
29. Truth table for system of four NAND gates as
V ~ shown in figure is : [2012]

A
The current (I) in the resistor (R) can be shown
by : Y

(a) B

A B Y A B Y
I 0 0 0 0 0 0
0 1 1 0 1 0
(b) (a) (b)
1 0 1 1 0 1
1 1 0 1 1 1
t
A B Y A B Y
0 0 1 0 0 1
0 1 1 0 1 0
(c) (c) (d)
1 0 0 1 0 1
1 1 0 1 1 1
I 30. The I-V characteristic of an LED is [2013]

(d)
t
I
27. The combination of gates shown below yields (a)
[2010]
O V
www.crackjee.xyz
Semiconductor Electronics : Materials, Devices and Simple Circuits P-177

34. If p, q, r, s are inputs to a gate and x is its output,


B then, as per the following time graph, the gate is :
G [2016]
Y
R
(b) p
O V q

r
I
s
(c)
x
O V
(a) OR (b) NAND
V O (c) NOT (d) AND
35. Identify the semiconductor devices whose
R characteristics are given below, in the order (p),
Y I
(d) G (q), (r), (s) : [2016]
B I I

31. The forward biased diode connection is:[2014]

(a)
+2V –2V V V

–3V –3V
(b)
(p) (q)
2V 4V
(c) I Resistance
dark
–2V +2V
(d)
V
32. A red LED emits light at 0.1 watt uniformly Intensity
around it. The amplitude of the electric field of Illuminated of light
the light at a distance of 1 m from the diode is :
[2015] (r) (s)
(a) Solar cell, Light dependent resistance,
(a) 5.48 V/m (b) 7.75 V/m
Zener diode, simple diode
(c) 1.73 V/m (d) 2.45 V/m (b) Zener diode, Solar cell, simple diode, Light
33. For a common emitter configuration, if a and dependent resistance
b have their usual meanings, the incorrect (c) Simple diode, Zener diode, Solar cell, Light
dependent resistance
relationship between a and b is : [2016]
(d) Zener diode, Simple diode, Light dependent
b b2 resistance, Solar cell
(a) a= (b) a= 36. In a common emitter amplifier circuit using an n-
1 +b 1 +b2
p-n transistor, the phase difference between the
1 1 b input and the output voltages will be : [2017]
(c) a = b+ 1 (d) a= 1 -b (a) 135° (b) 180°
(c) 45° (d) 90°
EBD_7764
www.crackjee.xyz
P-178 Physics

Answer Key
1 2 3 4 5 6 7 8 9 10 11 12 13 14 15
(c) (c) (c) (a) (c) (b) (c) (d) (d) (d) (a) (d) (d) (d) (c)
16 17 18 19 20 21 22 23 24 25 26 27 28 29 30
(a) (b) (c) (b) (d) (a) (a) (b) (d) (d) (b) (a) (b) (a) (a)
31 32 33 34 35 36
(a) (d) (b) (a) (c) (b)

1. (c) Pure silicon, at absolute zero, will contain 3. (c) The energy band gap is maximum in
all the electrons in bounded state. The insulators. Because of this the conduction
band of insulators is empty.
conduction band will be empty. So there
will be no free electrons (in conduction 4. (a) Emitter sends the majority charge carrriers
towards the collector. Therefore emitter is
band) and holes (in valence band) due to
most heavily doped.
thermal agitation. Pure silicon will act as
insulator. 5. (c) The resistance of metal (like Cu) decreases
with decrease in temperature whereas the
2. (c) Specific resistance is resistivity which is
resistance of a semi-conductor (like Ge)
given by
increases with decrease in temperature.
m 6. (b) When the temperature increases, certain
r=
ne 2 t bounded electrons become free which tend
where n = no. of free electrons per unit to promote conductivity. Simultaneously
volume number of collisions between electrons and
and t = average relaxation time positive kernels increases
For a conductor with rise in temperature n 7. (c) It can be seen from the following graph -
increases and t decreases. But the decrease Electric field
in t is more dominant than increase in n
resulting an increase in the value of r .
For a semiconductor with rise in p n
temperature, n increases and t decreases. Distance
But the increase in n is more dominant than
decrease in t resulting in a decrease in the
value of r. 8. (d) Holes move from base to emmitter.
9. (d) In common emitter configuration current
r 2 = r1 (1 + aDT ) gain
For conductor a is positive - hf e
Ai =
\ r2 > r 1 for D T positive i.e., increase in 1 + boe RL
temperature. -50
For semi conductor a is negative =
1 + 25 ´ 10-6 ´ 1 ´ 103
\ r2 < r 1 for D T positive. = – 48.78
www.crackjee.xyz
Semiconductor Electronics : Materials, Devices and Simple Circuits P-179

10. (d) Copper is a conductor, so its resistance (covalent bonding) and insulators (ionic
decreases on decreasing temperature as bonding) increases with increase in
thermal agitation decreases,; whereas temperature while that of metals (metallic
germanium is semiconductor therefore on bonding) decreases.
decreasing temper ature resistance
increases. Ie ne eAve 7 7 ve v 5
18. (c) = Þ = ´ Þ e =
11. (a) Both the depletion region and barrier height Ih nh eAvh 4 5 vh vh 4
is reduced. 19. (b) D 2 is forward biased whereas D 1 is
12. (d) Band gap = energy of photon of wavelength reversed biased.
2480 nm. So,
So effective resistance of the circuit
hc R = 4 + 2 = 6W
DE =
l
12
æ 6.63 ´10 - 34 ´ 3 ´108 ö 1
\i = =2A
=ç ÷´ eV 6
ç 2480 ´10 -9 ÷ -19
è ø 1.6 ´10 20. (d) p-side connected to low potential and n-
= 0.5 eV side is connected to high potential.
13. (d) Zero; In common base amplifier circuit, 21. (a) The current will flow through RL when the
input and output voltage are in the same diode is forward biased.
phase. 22. (a) Si and Ge are semiconductors but C is an
insulator. Also, the conductivity of Si and
14. (d) Input frequency, f = 50 Hz Þ T = 1 Ge is more than C because the valence
50 electrons of Si, Ge and C lie in third, fourth
and second orbit respectively.
T 1
For full wave rectifier, T1 = = 23. (b) It is a p-n-p transistor with R as base. None
2 100
of the option is correct.
Þ f1 = 100 Hz.
24. (d) A
15. (c) A crystal structure is composed of a unit
cell, a set of atoms arranged in a particular
way; which is periodically repeated in three C
dimensions on a lattice. The spacing
between unit cells in various directions is B
called its lattice parameters or constants. The truth table for the above logic gate is :
Increasing these lattice constants will
increase or widen the band-gap (Eg), which A B C
means more energy would be required by 1 1 1
electrons to reach the conduction band
1 0 1
from the valence band. Automatically Ec
and Ev decreases. 0 1 1
16. (a) IC = 5.488 mA, Ie = 5.6 mA 0 0 0

5.488 a This truth table follows the boolean algebra


a= ,b= = 49 C = A + B which is for OR gate
5. 6 1- a
17. (b) Van der Waal's bonding is attributed to the 25. (d) Here Y = ( A + B ) = A.B = A × B . Thus, it is
attractive forces between molecules of a an AND gate for which truth table is
liquid. The conductivity of semiconductors
EBD_7764
www.crackjee.xyz
P-180 Physics

31. (a) P n
A B Y
0 0 0 For forward bias, p-side must be at higher
0 1 0 potential than n-side. DV = (+ )Ve
1 0 0 1
32. (d) Using Uav = e0 E 2
1 1 1 2
26. (b) We know that a single p-n junction diode P
connected to an a-c source acts as a half But U av =
4 pr 2 ´ c
wave rectifier [Forward biased in one half
P 1
cycle and reverse biased in the other half \ 2
= e0 E 2 ´ c
cycle]. 4 pr 2
27. (a) The final boolean expression is, 2P 2 ´ 0.1 ´ 9 ´ 109
E 20 = =
X = ( A . B ) = A + B = A + B Þ OR gate 4 pr 2 e 0 c 1 ´ 3 ´ 108

28. (b) ( A + B) = NOR gate \ E0 = 6 = 2.45V/m


Ic Ic
When both inputs of NAND gate are 33. (b) We know that a = and b =
connected, it behaves as NOT gate Ie I b
OR + NOT = NOR. Also Ie = Ib + Ic
A
Ic
B (A+B) Ic I b
(A+B) \ a= = b =
I b + Ic I 1+ b
29. (a) 1+ c
Ib
A Option (b) and (d) are therefore correct.
Y = A.AB B.AB

Y2 = A.AB 34. (a) In case of an 'OR' gate the input is zero


when all inputs are zero. If any one input is
' 1', then the output is '1'.
Y1 = AB 35. (c) Graph (p) is for a simple diode.
Graph (q) is showing the V Break down
B
used for zener diode.
Y3 = B.AB
Graph (r) is for solar cell which shows cut-
By expanding this Boolen expression off voltage and open circuit current.
Graph (s) shows the variation of resistance
Y = A.B + B.A
h and hence current with intensity of light.
Thus the truth table for this expression
36. (b) In common emitter configuration for n-p-n
should be (a).
30. (a) For same value of current higher value of transistor input and output signals are 180°
voltage is required for higher frequency out of phase i.e., phase difference between
hence (1) is correct answer. output and input voltage is 180°.
www.crackjee.xyz

Communication
Systems 29
1. This question has Statement – 1 and Statement 4. A diode detector is used to detect an
– 2. Of the four choices given after the amplitudemodulated wave of 60% modulation
statements, choose the one that best describes by using a condenser of capacity 250 picofarad
the two statements. [2011] in parallel with a load resistance 100 kilo ohm.
Statement – 1 : Sky wave signals are used for Find the maximum modulated frequency which
long distance radio communication. These could be detected by it. [2013]
signals are in general, less stable than ground D
wave signals.
Statement – 2 : The state of ionosphere varies
from hour to hour, day to day and season to Signal C R
season.
(a) Statement–1 is true, Statement–2 is true, (a) 10.62 MHz (b) 10.62 kHz
Statement–2 is the correct explanation of (c) 5.31 MHz (d) 5.31 kHz
Statement–1.
(b) Statement–1 is true, Statement–2 is true, 5. A signal of 5 kHz frequency is amplitude modulated
on a carrier wave of frequency 2 MHz. The
Statement–2 is not the correct explanation
frequencies of the resultant signal is/are : [2015]
of Statement – 1.
(a) 2005 kHz, 2000 kHz and 1995 kHz
(c) Statement – 1 is false, Statement – 2 is true.
(d) Statement – 1 is true, Statement – 2 is false. (b) 2000 kHz and 1995 kHz
(c) 2 MHz only
2. Which of the following four alternatives is not
correct ? We need modulation : [2011 RS] (d) 2005 kHz and 1995 kHz
(a) to reduce the time lag between transmission 6. Choose the correct statement :
and reception of the information signal (a) In frequency modulation the amplitude of
(b) to reduce the size of antenna the high frequency carrier wave is made to
vary in proportion to the amplitude of the
(c) to reduce the fractional band width, that is
audio signal.
the ratio of the signal band width to the
centre frequency (b) In frequency modulation the amplitude of
the high frequency carrier wave is made to
(d) to increase the selectivity
vary in proportion to the frequency of the
3. A radar has a power of 1kW and is operating at audio signal.
a frequency of 10 GHz. It is located on a mountain
(c) In amplitude modulation the amplitude of
top of height 500 m. The maximum distance upto
the high frequency carrier wave is made to
which it can detect object located on the surface
of the earth vary in proportion to the amplitude of the
audio signal.
(Radius of earth = 6.4 × 106m) is : [2012]
(d) In amplitude modulation the frequency of
(a) 80 km (b) 16 km the high frequency carrier wave is made to
(c) 40 km (d) 64 km vary in proportion to the amplitude of the
audio signal. [2016]
EBD_7764
www.crackjee.xyz
P-182 Physics
7. In amplitude modulation, sinusoidal carrier of the following frequencies is not contained in
frequency used is denoted by wc and the signal the modulated wave ? [2017]
frequency is denoted by wm. The bandwidth (a) wm + wc (b) wc – wm
(Dwm) of the signal is such that Dwm < wc. Which
(c) wm (d) wc

Answer Key
1 2 3 4 5 6 7
(b) (a) (a) (b) (a) (c) (c)

1. (b) For long distance communication, sky 4. (b) Given : Resistance R = 100 kilo ohm
wave signals are used. = 100 × 103 W
Also, the state of ionosphere varies every time. Capacitance C = 250 picofarad
So, both statements are correct. = 250 × 10–12F
2. (a) Low frequencies cannot be transmitted to t = RC = 100 × 103 × 250 × 10–12 sec
long distances. Therefore, they are super = 2.5 × 107 × 10–12 sec
imposed on a high frequency carrier signal
= 2.5 × 10–5 sec
by a process known as modulation.
The higher frequency whcih can be
Speed of electro-magnetic waves will not detected with tolerable distortion is
change due to modulation. So there will be
1 1
time lag between transmission and f = = Hz
2pma RC 2 p ´ 0.6 ´ 2.5 ´ 10 -5
reception of the information signal.
3. (a) Let d is the maximum distance, upto which 100 ´ 10 4 4
= Hz = ´ 10 4 Hz
it can detect the objects 25 ´ 1.2p 1.2 p
C = 10.61 KHz
This condition is obtained by applying the
h condition that rate of decay of capacitor
d voltage must be equal or less than the rate
B of decay modulated singnal voltage for
proper detection of mdoulated signal.
A q R 5. (a) Amplitude modulated wave consists of
R three frequencies are wc + wm, w,wc – wm
O i.e. 2005 KHz, 2000KHz, 1995 KHz
6. (c) In amplitude modulation, the amplitude of
the high frequency carrier wave made to
vary in proportional to the amplitude of
From DAOC audio signal.

OC 2 = AC 2 + AO2 Audio signal

(h + R)2 = d 2 + R 2 Carrier wave


Þ d 2 = ( h + R )2 - R 2
Amplitude modulated wave
2 2
d = (h + R) - R ; d = h + 2hR 2
7. (c) Modulated carrier wave contains
d = 5002 + 2 ´ 6.4 ´ 10 6 = 80 km frequency wc and wc ± wm
www.crackjee.xyz

Topic-wise Solved Papers Chemistry

Some Basic Concepts of


Chemistry 1
1. In a compound C, H and N atoms are present in 7. To neutralise completely 20 mL of 0.1 M aqueous
9 : 1 : 3.5 by weight. Molecular weight of solution of phosphorous acid (H3 PO3), the
compound is 108. Molecular formula of value of 0.1 M aqueous KOH solution required
compound is [2002] is [2004]
(a) C2H6N2 (b) C3H4N (a) 40 mL (b) 20 mL
(c) C6H8N2 (d) C9H12N3. (c) 10 mL (d) 60 mL
2. With increase of temperature, which of these 8. The ammonia evolved from the treatment of 0.30
changes? [2002] g of an organic compound for the estimation of
(a) molality nitrogen was passed in 100 mL of 0.1 M
(b) weight fraction of solute sulphuric acid. The excess of acid required 20
(c) molarity
mL of 0.5 M sodium hydroxide solution for
(d) mole fraction.
complete neutralization. The organic compound
3. Number of atoms in 558.5 gram Fe (at. wt. of
is [2004]
Fe = 55.85 g mol–1) is [2002]
(a) urea (b) benzamide
(a) twice that in 60 g carbon
(b) 6.023 ´ 1022 (c) acetamide (d) thiourea
(c) half that in 8 g He 9. Two solutions of a substance (non electrolyte)
(d) 558.5 ´ 6.023 ´ 1023 are mixed in the following manner. 480 ml of 1.5
4. What volume of hydrogen gas, at 273 K and 1 M first solution + 520 ml of 1.2 M second
atm. pressure will be consumed in obtaining 21.6 solution. What is the molarity of the final
g of elemental boron (atomic mass = 10.8) from mixture ? [2005]
the reduction of boron trichloride by hydrogen? (a) 2.70 M (b) 1.344 M
[2003] (c) 1.50 M (d) 1.20 M
(a) 67.2 L (b) 44.8 L 10. If we consider that 1/6, in place of 1/12, mass of
(c) 22.4 L (d) 89.6 L carbon atom is taken to be the relative atomic
5. 25ml of a solution of barium hydroxide on mass unit, the mass of one mole of the substance
titration with a 0.1 molar solution of hydrochloric will [2005]
acid gave a litre value of 35ml. The molarity of (a) be a function of the molecular mass of the
barium hydroxide solution was substance
(a) 0.14 (b) 0.28 [2003] (b) remain unchanged
(c) 0.35 (d) 0.07 (c) increase two fold
6. 6.02 × 1020 molecules of urea are present in 100 (d) decrease twice
ml of its solution. The concentration of urea 11. How many moles of magnesium phosphate,
solution is [2004] Mg3(PO4)2 will contain 0.25 mole of oxygen
(a) 0.02 M (b) 0.01 M atoms? [2006]
(c) 0.001 M (d) 0.1 M (a) 1.25 × 10–2 (b) 2.5 × 10–2
(Avogadro constant, NA = 6.02 × 1023 mol–1) (c) 0.02 (d) 3.125 × 10–2
EBD_7764
www.crackjee.xyz
C-2 Chemistry
12. Density of a 2.05M solution of acetic acid in exists as M3+ would be :
water is 1.02 g/mL. The molality of the solution (a) 7.01% (b) 4.08%[2013]
is [2006] (c) 6.05% (d) 5.08%
(a) 2.28 mol kg –1 (b) 0.44 mol kg–1 18. 3 g of activated charcoal was added to 50 mL of
(c) 1.14 mol kg –1 (d) 3.28 mol kg–1 acetic acid solution (0.06N) in a flask. After an
13. The density (in g mL–1) of a 3.60 M sulphuric hour it was filtered and the strength of the filtrate
acid solution that is 29% H 2 SO 4 was found to be 0.042 N. The amount of acetic
(molar mass = 98 g mol–1) by mass will be acid adsorbed (per gram of charcoal) is :
(a) 1.45 (b) 1.64 [2007] [JEE M 2015]
(c) 1.88 (d) 1.22 (a) 42 mg (b) 54 mg
(c) 18 mg (d) 36 mg
14. In the reaction, [2007]
19. At 300 K and 1 atm, 15 mL of a gaseous
2Al (s) + 6HCl(aq) ® hydrocarbon requires 375 mL air containing 20%
2Al3+ (aq) + 6Cl - (aq) + 3H 2 (g) O2 by volume for complete combustion. After
(a) 11.2 L H2(g) at STP is produced for every combustion the gases occupy 330 mL. Assuming
mole HCl(aq) consumed that the water formed is in liquid form and the
(b) 6 L HCl(aq) is consumed for every 3 L H2(g) volumes were measured at the same temperature
produced and pressure, the formula of the hydrocarbon
(c) 33.6 L H2 (g) is produced regardless of is: [JEE M 2016]
temperature and pressure for every mole (a) C4H8 (b) C4H10
Al that reacts (c) C3H6 (d) C3H8
(d) 67.2 H2(g) at STP is produced for every 20. The most abundant elements by mass in the
mole Al that reacts. body of a healthy human adult are :
15. The molality of a urea solution in which 0.0100 g Oxygen (61.4%) ; Carbon (22.9%), Hydrogen
of urea, [(NH2)2CO] is added to 0.3000 dm3 of (10.0%) ; and Nitrogen (2.6%). The weight which
water at STP is : [2011RS] a 75 kg person would gain if all 1H atoms are
(a) 5.55 ´ 10 -4 m (b) 33.3 m replaced by 2H atoms is [JEE M 2017]
(c) 3.33 × 10–2 m (d) 0.555 m (a) 15 kg (b) 37.5 kg
16. A gaseous hydrocarbon gives upon combustion (c) 7.5 kg (d) 10 kg
0.72 g of water and 3.08 g. of CO2. The empirical 21. 1 gram of a carbonate (M2CO3) on treatment
formula of the hydrocarbon is : [2013] with excess HCl produces 0.01186 mole of CO2.
(a) C2H4 (b) C3H4 The molar mass of M2CO3 in g mol–1 is :
(c) C6H5 (d) C7H8 [JEE M 2017]
17. Experimentally it was found that a metal oxide (a) 1186 (b) 84.3
has formula M0.98O. Metal M, present as M2+
(c) 118.6 (d) 11.86
and M3+ in its oxide. Fraction of the metal which

Answer Key
1 2 3 4 5 6 7 8 9 10 11 12 13 14 15
(c) (c) (a) (a) (d) (b) (a) (a) (b) (d) (d) (a) (d) (a) (a)
16 17 18 19 20 21
(d) (b) (c) (d) (c) (b)
www.crackjee.xyz
Some Basic Concepts of Chemistry C-3

1. (c) Percentage 6. (b) Moles of urea present in 100 ml of sol.=


R.N.A Simplest ratio
6.02 ´ 1020
9 3 6.02 ´ 10 23
C 9 = 3
12 4
6.02 ´10 20 ´1000
1 \M = = 0.01M
H 1 =1 4 6.02 ´10 23 ´100
1
[ Q M = Moles of solute present in 1L of
3.5 1 solution]
N 3.5 = 1
14 4 7. (a) N1V1 = N 2 V2
Empirical formula = C3H4N (Note : H3PO3 is dibasic \ M = 2N)
(C3H4N)n = 108
20 ´ 0.2 = 0.1´ V (Thus. 0.1 M = 0.2 N)
(12 × 3 + 4 × 1 + 14)n = 108
\ V = 40 ml
(54)n = 108
8. (a) H 2SO 4 is dibasic.
108
n= =2 0.1 MH 2SO 4 = 0.2NH 2SO 4 [
54
\ molecular formula = C6 H8N2 QM = 2´ N ]
2. (c) Among all the given options molarity is Meq of H 2SO 4 taken = = 100 ´ 0.2 = 20
correct because the term molarity involve
volume which increases on increasing Meq of H 2SO 4 neutralised by
temperature. NaOH = 20 ´ 0.5 = 10
558.5 Meq of H 2SO 4 neutralised by
3. (a) Fe (no. of moles) = = 10 moles
55.85 NH3 = 20 – 10 = 10 % of
C (no. of moles) in 60 g of C = 60/12 = 5 moles. 1.4 ´ M eq of acid neutrialised by NH3
N2 =
4. (a) 2BCl3 + 3H 2 ® 2B + 6HCl wt. of organic compound

3 1.4 ´ 10
or BCl3 + H 2 ® B + 3HCl = = 46.6
2 0.3
Now, since 10.8 gm boron requires 14 ´ 2 ´ 100
hydrogen % of nitrogen in urea = = 46.6
60

=
3
´ 22.4L at N.T.P
[ Mol.wt of urea =60]
2 Similarly % of Nitrogen in Benzamide
hence 21.6 gm boron requires hydrogen 14 ´ 100
= = 11.5% [C6H5CONH2 = 121]
3 22.4 121
´ ´ 21.6 < 67.2L at N.T. P..
2 10.8 14 ´ 1 ´ 100
5. (d) 25 × N = 0.1 × 35 ; N = 0.14 Acctamide = = 23.4%
59
Ba(OH)2 is diacid base [ CH3CONH2=59]
N 14 ´ 2 ´ 100
hence N = M × 2 or M = Thiourea = = 36.8%
2 76
[NH2CSNH2 = 76]
M = 0.07 M
Hence the compound must be urea.
EBD_7764
www.crackjee.xyz
C-4 Chemistry
9. (b) = 3.6 × 98 g = 352.8 g
\ 1000 ml solution has 352.8 g of H2SO4
From the molarity equation.
Given that 29 g of H2SO4 is present in =
M1V1 + M2V2 = MV
100 g of solution
Let M be the molarity of final mixture,
\ 352.8 g of H2SO4 is present in
M1V1 +M V2
2 100
M= where V = V1 + V2 = ´ 352.8 g of solution
V 29
= 1216 g of solution
480 ´ 1.5 + 520 ´ 1.2
M= = 1.344 M
480 + 520 Mass 1216
Density = =
10. (d) Relative atomic mass = Volume 1000
= 1.216 g/ml = 1.22 g/ml
Mass of one atom of the element
14. (a) 2Al(s) + 6HCl(aq) ® 2Al3+(aq) + 6Cl–
th
1/12 part of the mass of one atom of Carbon - 12 (aq) + 3H2(g)
Q 6 moles of HCl produces = 3 moles of H2
Mass of one atom of the element = 3 × 22.4 L of H2 at S.T.P
or ´ 12
mass of one atom of the C - 12 \ 1 mole of HCl produces
Now if we use 1 / 6 in place of 1 /12 the 3 ´ 22.4
formula becomes = L of H2 at S.T.P
6
Relative atomic mass = = 11.2 L of H2 at STP
15. (a) Molality = Moles of solute / Mass of
Mass of one atom of element solvent in kg
´6
Mass of one atom of carbon 0.01/ 60 0.01
Molality = = ;
\ Relative atomic mass decrease twice 0.3 60 ´ 0.3
11. (d) 1 Mole of Mg3(PO4)2 contains 8 mole of d = 1 g/ml
oxygen atoms
= 5.55 ´ 10-4 m
\ 8 mole of oxygen atoms º 1 mole of
Mg3(PO4)2 mole of Mg3(PO4)2 16. (d) Q 18 gm, H2O contains = 2 gm H
\ 0.72 gm H2O contains
1
0.25 mole of oxygen atom º ´ 0.25 mole 2
8 = ´ 0.72 gm = 0.08 gm H
18
of Mg3(PO4)2
Q 44 gm CO2 contains = 12 gm C
= 3.125 ´ 10 -2 mole of Mg3 (PO4)2 \ 3.08 gm CO2 contains
12. (a)
12
= ´ 3.08 = 0.84 gm C
æ1 M ö 44
Apply the formula d = M ç + 2 ÷
è m 1000 ø 0.84 0.08
\C: H= :
12 1
æ1 60 ö = 0.07 : 0.08 = 7 : 8
\ 1.02 = 2.05 ç + ÷
è m 1000 ø \ Empirical formula = C7H8
On solving we get, m = 2.288 mol/kg 17. (b) For one mole of the oxide
13. (d) Since molarity of solution is 3.60 M. It Moles of M = 0.98
means 3.6 moles of H2SO4 is present in its Moles of O2– = 1
1 litre solution. Let moles of M3+ = x
Mass of 3.6 moles of H2SO4 \ Moles of M2+ = 0.98– x
= Moles × Molecular mass On balancing charge
www.crackjee.xyz
Some Basic Concepts of Chemistry C-5
(0.98 – x) × 2 + 3x – 2 = 0 20. (c) Percentage (by mass) of elements given in
x = 0.04 the body of a healthy human adult is :-
0.04
\ % of M3+ = ´100 < 4.08% Oxygen = 61.4%, Carbon = 22.9%,
0.98
Hydrogen = 10.0% and Nitrogen = 2.6%
18. (c) Let the weight of acetic acid initially be w1
in 50 ml of 0.060 N solution. Q Total weight of person = 75 kg
Let the N = (Normality = 0.06 N)
10
0.06 = \ Mass due to 1H is = 75 ´ = 7.5 kg
Þ = 0.18 g = 180 mg. 100
After an hour, the strength of acetic acid =
0.042 N If 1H atoms are replaced by 2H atoms.
so, let the weight of acetic acid be w2 Mass gain by person would be = 7.5 kg
N=
21. (b) Given chemical eqn
0.042 =
Þ w2 = 0.126 g = 126 mg M2CO3 + 2HCl ® 2MCl + H2O + CO2
1gm 0.01186 mole
So amount of acetic acid adsorbed per 3g
= 180 – 126 mg = 54 mg from the balanced chemical eqn.
Amount of acetic acid adsorbed per g = 18 nM2CO3 = nCO2
19. (d) CxHy(g) + O2(g) ® xCO2(g) + H2O(l)
Volume of O2 used = = 75 ml 1
= 0.01186
\ From the reaction of combustion M 2 CO3
1 ml CxHy requires = 1
15 ml = \ M 2 CO3 =
0.01186
So, 4x + y = 20
Þ M = 84.3 g/mol
x= 3
y= 8
C3 H 8
EBD_7764
www.crackjee.xyz
C-6 Chemistry

Structure of Atom 2
1. In a hydrogen atom, if energy of an electron in 7. The de Broglie wavelength of a tennis ball of
ground state is 13.6. ev, then that in the 2nd mass 60 g moving with a velocity of 10 metres
excited state is [2002] per second is approximately [2003]
(a) 1.51 eV (b) 3.4 eV (a) 10–31 metres (b) 10–16 metres
(c) 6.04 eV (d) 13.6 eV. (c) 10–25 metres (d) 10–33 metres
2. Uncertainty in position of a minute particle of Planck’s constant, h = 6.63 × 10–34 Js
mass 25 g in space is 10-5 m. What is the uncertainty 8. Which of the following sets of quantum numbers
in its velocity (in ms-1)? (h = 6.6 ´ 10-34 Js) is correct for an electron in 4f orbital ? [2004]
[2002] (a) n = 4, l = 3, m = + 1, s = + ½
(a) 2.1 ´ 10–34 (b) 0.5 ´ 10 –34
(b) n = 4, l = 4, m = – 4, s = – ½
(c) 2.1 ´ 10–28 (d) 0.5 ´ 10–23. (c) n = 4, l = 3, m = + 4, s = + ½
3. The number of d-electrons retained in Fe2+ (At. (d) n = 3, l = 2, m = – 2, s = + ½
no. of Fe = 26) ion is [2003] 9. Consider the ground state of Cr atom (X = 24).
(a) 4 (b) 5 The number of electrons with the azimuthal
(c) 6 (d) 3 quantum numbers, l = 1 and 2 are, respectively
4. The orbital angular momentum for an electron [2004]
h (a) 16 and 4 (b) 12 and 5
revolving in an orbit is given by l (l + 1) . .
2p (c) 12 and 4 (d) 16 and 5
This momentum for an s-electron will be given 10. The wavelength of the radiation emitted, when
by [2003] in a hydrogen atom electron falls from infinity to
h stationary state 1, would be (Rydberg constant
(a) zero (b) = 1.097×107 m–1) [2004]
2p
1 h (a) 406 nm (b) 192 nm
(c) h (d) + .
2. 2 2p (c) 91 nm (d) 9.1×10–8 nm
2p
11. Which one of the following sets of ions
5. Which one of the following groupings represents represents the collection of isoelectronic
a collection of isoelectronic species ?(At. nos. : species? [2004]
Cs : 55, Br : 35) [2003] (a) K+, Cl–, Mg2+, Sc3+
(a) N3–, F–, Na+ (b) Be, Al3+, Cl– (b) Na+, Ca2+, Sc3+, F–
(c) Ca2+, Cs+, Br (d) Na+, Ca2+, Mg2+ (c) K+, Ca2+, Sc3+, Cl–
6. In Bohr series of lines of hydrogen spectrum, (d) Na+, Mg2+, Al3+, Cl–
the third line from the red end corresponds to
(Atomic nos. : F = 9, Cl = 17, Na = 11, Mg = 12,
which one of the following inter-orbit jumps of
Al = 13, K = 19, Ca = 20, Sc = 21)
the electron for Bohr orbits in an atom of
hydrogen [2003] 12. In a multi-electron atom, which of the following
orbitals described by the three quantum
(a) 5 ® 2 (b) 4 ® 1 members will have the same energy in the
(c) 2®5 (d) 3 ® 2 absence of magnetic and electric fields? [2005]
www.crackjee.xyz
Structure of Atom C-7
(A) n = 1, l = 0, m = 0 (B) n = 2, l = 0, m = 0 19. The ionization enthalpy of hydrogen atom is
(C) n = 2, l = 1, m = 1 (D) n = 3, l = 2, m = 1 1.312 × 106 J mol–1. The energy required to excite
(E) n = 3, l = 2, m = 0 the electron in the atom from n = 1 to n = 2 is
(a) (D) and (E) (b) (C) and (D) [2008]
(c) (B) and (C) (d) (A) and (B) (a) 8.51 × 105 J mol–1 (b) 6.56 × 105 J mol–1
13. Of the following sets which one does NOT (c) 7.56 × 105 J mol–1 (d) 9.84 × 105 J mol–1
contain isoelectronic species? [2005] 20. Calculate the wavelength (in nanometer)
associated with a proton moving at 1.0 × 103 ms –1.
(a) BO 33 - , CO 32 - , NO 3-
(Mass of proton = 1.67 × 10–27 kg and
(b) SO32 - , CO32 - , NO3- h = 6.63 × 10–34 Js) [2009]
(a) 0.40 nm (b) 2.5 nm
(c) CN - , N 2 , C 22 -
(c) 14.0 nm (d) 0.32 nm
(d) PO 34 - , SO 24 - , ClO -4 21. In an atom, an electron is moving with a speed
14. According to Bohr's theory, the angular of 600 m/s with an accuracy of 0.005%. Certainity
momentum of an electron in 5th orbit is [2006] with which the position of the electron can be
(a) 10 h / p (b) 2.5 h / p located is ( h = 6.6 × 10–34 kg m2s–1, mass of
(c) 25 h / p (d) 1.0 h / p electron, em = 9.1 × 10–31 kg) : [2009]
15. Uncertainty in the position of an electron (mass (a) 5.10 × 10 –3 m (b) 1.92 × 10 –3 m
= 9.1 × 10–31 kg) moving with a velocity 300 ms–1, (c) 3.84 × 10 –3 m (d) 1.52 × 10 –4 m
accurate upto 0.001% will be [2006] 22. The energy required to break one mole of Cl – Cl
(a) 1.92 × 10–2 m (b) 3.84 × 10–2 m bonds in Cl 2 is 242 kJ mol –1 . The longest
(c) 19.2 × 10–2 m (d) 5.76 × 10–2 m wavelength of light capable of breaking a single
(h = 6.63 × 10–34 Js) Cl – Cl bond is (c = 3 × 108 ms–1 and NA = 6.02 ×
16. Which one of the following sets of ions 1023 mol–1). [2010]
represents a collection of isoelectronic species? (a) 594 nm (b) 640 nm
[2006] (c) 700 nm (d) 494 nm
(a) N3–, O2–, F–, S2– 23. Ionisation energy of He+ is 19.6 × 10–18 J atom–1.
(b) Li+, Na+, Mg2+, Ca2+ The energy of the first stationary state (n = 1) of
(c) K+, Cl–, Ca2+, Sc3+ Li2+ is [2010]
(d) Ba2+, Sr2+, K+, Ca2+ (a) 4.41 × 10–16 J atom–1
17. Which of the following sets of quantum numbers (b) –4.41 × 10–17 J atom–1
represents the highest energy of an atom? (c) –2.2 × 10–15 J atom–1
[2007] (d) 8.82 × 10–17 J atom–1
(a) n = 3, l = 0, m = 0, s = +1/2
24. The frequency of light emitted for the transition
(b) n = 3, l = 1, m = 1, s = +1/2 n = 4 to n = 2 of the He+ is equal to the transition
(c) n = 3, l = 2, m = 1, s = +1/2 in H atom corresponding to which of the
(d) n = 4, l = 0, m = 0, s = +1/2. following? [2011RS]
18. Which one of the following constitutes a group (a) n = 2 to n = 1 (b) n = 3 to n = 2
of the isoelectronic species? [2008] (c) n = 4 to n = 3 (d) n = 3 to n = 1
2– –
(a) C2 , O2 , CO, NO 25. The electrons identified by quantum numbers n
(b) NO+ ,C2– – and l : [2012]
2 , CN , N 2
(A) n = 4, l = 1 (B) n = 4, l = 0
(c) CN – , N 2 ,O2– 2–
2 , C2 (C) n = 3, l = 2 (D) n = 3, l = 1
(d) N 2 ,O2– , NO+ ,CO can be placed in order of increasing energy as :
EBD_7764
www.crackjee.xyz
C-8 Chemistry
(a) (C) < (D) < (B) < (A) 29. Which of the following is the energy of a possible
(b) (D) < (B) < (C) < (A) excited state of hydrogen? [JEE M 2015]
(c) (B) < (D) < (A) < (C) (a) –3.4 eV (b) +6.8 eV
(d) (A) < (C) < (B) < (D) (c) +13.6 eV (d) –6.8 eV
30. A stream of electrons from a heated filaments
26. The increasing order of the ionic radii of the
was passed two charged plates kept at a
given isoelectronic species is : [2012]
potential difference V esu. If e and m are charge
– 2+ +
(a) Cl , Ca , K , S 2– (b)S , Cl , Ca2+ , K+
2– –
and mass of an electron, respectively, then the
2+ + –
(c) Ca , K , Cl , S 2– (d)K+, S2–, Ca2+, Cl– value of h/l (where l is wavelength associated
27. Energy of an electron is given by E = – 2.178 × with electron wave) is given by: [JEE M 2016]
æ Z2 ö (a) meV (b) 2meV
10-18 J ç 2 ÷ . Wavelength of light required to
èn ø (c) meV (d) 2meV
excite an electron in an hydrogen atom from level 31. The radius of the second Bohr orbit for hydrogen
n = 1 to n = 2 will be : [2013] atom is : [JEE M 2017]
(h = 6.62 × 10 –34 Js and c = 3.0 × 108 ms–1) (Plank's const. h = 6.6262 × 10–34 Js ; mass of
electron = 9.1091 × 10–31 kg ; charge of electron
(a) 1.214 × 10–7 m (b) 2.816 × 10.–7 m
e = 1.60210 × 10–19 C ; permittivity of vaccum
(c) 6.500 × 10–7 m (d) 8.500 × 10–7 m
Î0 = 8.854185 × 10–12 kg–1 m–3 A2)
28. The correct set of four quantum numbers for the
(a) 1.65Å (b) 4.76Å
valence electrons of rubidium atom (Z = 37) is:
(c) 0.529Å (d) 2.12Å
[2014]
32. The group having isoelectronic species is :
1 1 (a) O2– , F–, Na+, Mg2+ [JEE M 2017]
(a) 5, 0, 0, + (b) 5,1, 0, +
2 2 (b) O– , F–, Na, Mg+
1 1 (c) O2– , F–, Na , Mg2+
(c) 5,1,1, + (d) 5, 0,1, +
2 2 (d) O– , F–, Na+ , Mg2+

Answer Key
1 2 3 4 5 6 7 8 9 10 11 12 13 14 15
(a) (c) (c) (a) (a) (a) (d) (a) (b) (c) (c) (a) (b) (b) (a)
16 17 18 19 20 21 22 23 24 25 26 27 28 29 30
(c) (c) (b) (d) (a) (b) (d) (b) (a) (b) (c) (a) (a) (a) (b)
31 32
(d) (a)

1. (a) 2nd excited state will be the 3rd energy level.


6.62 ´ 10 -34
13.6 \ Dv =
13.6 4 ´ 3.14 ´ 0.025 ´ 10 -5
En = 2
eV or E = eV = 1.51 eV.
n 9
= 2.1 ´ 10 -28 ms -1
2. (c) 3. (c) Fe++ (26 – 2 = 24) = 1s2 2s2 2p6 3s2 3p6 4s0
h 3d6 hence no. of d electrons retained is 6.
h Dx.m.Dv =
Dx. Dp = ; or [Two 4s electron are removed]
4p 4p
www.crackjee.xyz
Structure of Atom C-9

4. (a) 13. (b) Calculating number of electrons


For s-electron, l = 0 3-
BO3 ¾¾® 5 + 8 ´ 3 + 3 = 32 ü
2- ï iso-electronic
\ Orbital angular momentum = 1. CO3 ¾¾® 6 + 8 ´ 3 + 2 = 32ý species
h -
® 7 + 8 ´ 3 + 1 = 32 ï
NO3 ¾¾
0(0 + 1) = 0 þ
2p
5. (a) N3–, F– and Na+ contain 10 electrons each.
6. (a) The lines falling in the visible region 2-
SO3 ® 16 + 8 ´ 3 + 2 = 42ü
¾¾
ï
comprise Balmer series. Hence the third line 2- ï not iso-electronic
2. CO3 ¾¾® 32 ý species
from red would be n1 =2, n 2 = 5 i.e. 5 ® 2. - ï
NO3 ¾¾® 32 ïþ
h 6.6 ´ 10 -34
7. (d) l= = = 10 -33 m
mv 60 ´ 10 -3 ´10 CN
-
® 6 + 7 + 1 = 14ü
¾¾
ï
ï iso-electronic
8. (a) The possible quantum numbers for 4f 3. N 2 ¾¾
® 7 ´ 2 = 14 ý species
electron are - ï
C 2 ¾¾
® 6 ´ 2 + 2 = 14 ï
n = 4, l = 3, m = – 3, –2 –1, 0, 1 , 2 , 3 and þ
1
s=± ® 15 + 8 ´ 4 + 3 = 50 ü
PO34- ¾¾
2 ï
Of various possiblities only option (a) is 2- ï iso-electronic
4. SO 4 ¾¾
® 16 + 8 + 2 = 50 ý species
possible. - ï
ClO4 ¾¾
® 17 + 8 ´ 4 + 1 = 50ï
9. (b) Electronic configuration of Cr atom þ
(z = 24) = 1s 2 , 2s2 2p 6 ,3s 2 3p 6 3d 5 , 4s1 Hence the species in option (b) are not
iso-electronic.
when l = 1, p - subshell,
14. (b) Angular momentum of an electron in nth
Numbers of electrons = 12 orbital is given by ,
when l = 2, d - subshell,
nh
Numbers of electrons = 5 mvr =
2p
10. (c) For n = 5, we have
Angular momentum of electron
1 æ 1 1 ö÷
= Rç - 5h 2.5h
l ç n2 n2 ÷ = =
è 1 2ø
2p p
1 æ1 1 ö 15. (a) Given m = 9.1 × 10–31kg
= 1.097 ´ 10 7 ç - ÷ = 1.097 ´ 10 7 h = 6.6 × 10–34Js
l è1 ¥ ø
300 ´ .001
l = 91.15 ´ 10 -9 m » 91nm Dv = = 0.003ms–1
100
+, 2+ , 3+ , -
11. (c) 19 K 20 Ca 21Sc 17 Cl From Heisenberg's uncertainity principle
each contains 18 electrons. 6.62 ´ 10 -34
12. (a) The energy of an orbital is given by (n + l) Dx =
4 ´ 3.14 ´ 0.003 ´ 9.1 ´ 10-31
in (d) and (c). (n + l) value is (3 + 2) = 5
hence they will have same energy, since = 1.92 ´ 10 -2 m
there n values are also same.
EBD_7764
www.crackjee.xyz
C-10 Chemistry
16. (c) (a) N3– = 7 + 3 = 10e–, O–– ––® 8 + 2 = 10e– DE is given by the relation,
E1 = – 1.312 × 106 J mol–1
F– = 9 + 1 = 10e–, S – – ––® 16 + 2 = 18e– \ DE = E2 – E1 = [–3.28 × 105]– [–1.312 ×
(not iso electronic) 106 ] J mol–1
(b) Li+ = 3+1= 4e–, Na+ = 11–1 = 10e–, = (–3.28 × 105 + 1.312 × 106) J mol–1
Mg++ = 12–2=10e– = 9.84 × 105 J mol–1
Ca++ = 20 – 2 = 18e– (not isoelectronic) Thus the correct answer is (d)
(c) K+ = 19 – 1= 18e–, Cl– =17 + 1 = 18e–,
h 6.63 ´ 10-34
Ca++ = 20 – 2 =18e, Sc3+ = 21–3 = 18e– 20. (a) l= =
mv 1.67 ´ 10 -27 ´ 1 ´ 103
(isoelectronic)
(d) Ba 56 – 2 = 54e, Sr++ 38–2 = 36e–
++ = 3.97 × 10–10 meter = 0.397 nanometer
21. (b) According to Heisenberg uncertainty
principle.
K+= 9–1 = 18e–, Ca++= 20–2 = 18e–
(not isoelectronic) h h
Dx.mDv = Dx =
17. (c) (a) n = 3, l = 0 means 3s-orbital and n + 4p 4 pmD v
l= 3 600 ´ 0.005
Here Dv = = 0.03
(b) n = 3, l = 1 means 3p-orbital n + l = 4 100
(c) n = 3, l = 2 means 3d-orbital n + l = 5 6.6 ´ 10-34
So, Dx =
(d) n = 4, l = 0 means 4s-orbital n + l = 4 4 ´ 3.14 ´ 9.1 ´ 10-31 ´ 0.03
Increasing order of energy among = 1.92 × 10–3 meter
these orbitals is 22. (d) Energy required to break one mole of Cl –
3s < 3p < 4s < 3d Cl bonds in Cl2
\ 3d has highest energy..
18. (b) Species having same number of electrons 242 ´ 103 hc
= =
are isoelectronic calculating the number 6.023 ´ 10 23 l
of electrons in each species given here, we
get. 6.626 ´ 10-34 ´ 3 ´ 108
=
CN– (6 + 7 + 1 = 14); N2 (7 + 7 = 14); l
O22–(8 + 8 +2 = 18) ; C22– (6 + 6 + 2 = 14);
6.626 ´ 10 -34 ´ 3 ´ 108 ´ 6.023 ´ 10 23
O2– (8 + 8 + 1 = 17) ; NO+ (7 + 8 – 1 = 14) \l=
CO (6 + 8 = 14) ; NO (7 + 8 = 15) 242 ´ 108
From the above calculation we find that all = 0.4947 × 10–6 m = 494.7 nm
the species listed in choice (b) have 14 Z2
electrons each so it is the correct answer. 23. (b) I. E = ´ 13.6 eV ...(i)
n2
19. (d) (DE), The energy required to excite an
electron in an atom of hydrogen from n = 1 I1 Z12 n22
to n = 2 is DE (difference in energy E2 and or I = 2 ´ 2 ...(ii)
2 n1 Z 2
E1)
Values of E2 and E1 are, Given I1 = – 19.6 × 10–18 , Z1 = 2,
n1 = 1 , Z2 = 3 and n2 = 1
-1.312 ´106 ´ (1)2 Substituting these values in equation (ii).
E2 =
(2)2 19.6 ´ 10 -18 4 1
– = ´
= –3.28 × 105 J mol–1 I2 1 9
www.crackjee.xyz
Structure of Atom C-11

-18 9 æ 1 1 ö hc
or I2 = -19.6 ´ 10 ´ 27. (a) DE = 2.178 ´ 10-18 ç 2 - 2 ÷ =
4 è1 2 ø l
= – 4.41 × 10–17 J/atom 3 hc
Þ 2.178 ´ 10-18 ´ =
24. (a) For He+ 4 l
1 æ 1 1ö 6.62 ´ 10-34 ´ 3 ´ 108
v= = RH Z 2 ç - =
l è 2 2 42 ÷ø l
For H 6.62 ´ 10 -34 3 ´ 108 ´ 4
l=
2.178 ´ 10 -18 ´3
æ ö –7
= 1.214 × 10 m
1 1 1
v= = RH ç - ÷ 28. (a) The electronic configuration of Rubidium
l ç n12 n2 ÷
è 2ø (Rb = 37) is
For same frequency, 1s 2 2s 2 2 p 6 3s 2 3 p 6 3d 10 4s 2 4 p 6 5s1
æ ö æ ö Since last electron enters in 5s orbital
1 1 1 1 1
z2 = ç 2 - 2 ÷ = ç 2 - 2 ÷ Hence n = 5, l = 0, m = 0, s = ±
çè 2 4 ÷ø çè n1 n2 ÷ø 2
29. (a) Total energy =
Since, z = 2
where n = 2, 3, 4 ....
1 1 1 1 Putting n = 2
\ - = -
n12 n22 12 2 2 ET =
\ n1 = 1 & n2 = 2 30. (b) As electron of charge ‘e’ is passed through
25. (b) (a) 4 p (b) 4 s ‘V’ volt, kinetic energy of electron will be
eV
(c) 3 d (d) 3 p
Wavelength of electron wave (l) =
Accroding to Bohr Bury's (n + l) l = Þ\ =
rule, increasing order of energy (D) < (B) < 31. (d) Radius of nth Bohr orbit in H-atom
(C) < (A). = 0.53 n2Å
Note : If the two orbitals have same value Radius of II Bohr orbit = 0.53 × (2)2
of (n + l) then the orbital with lower value = 2.12 Å
of n will be filled first. 32. (a) Isoelectronic species have same no. of
26. (c) Among isoelectronic species ionic radii electrons.
increases as the charge increases. ions O–2 F– Na+ Mg2+
Order of ionic radii Ca2+ < K+ < Cl– < S2– 8+2 9+1 11–1 12–2
The number of electrons remains the same No. of e– = 10 10 10 10
but nuclear charge increases with increase therefore O 2– , F – , Na + , Mg 2+ are
in the atomic number causing decrease in isoelectronic
size.
EBD_7764
www.crackjee.xyz
C-12 Chemistry

3
Classification of Elements
and Periodicity in Properties
1. According to the Periodic Law of elements, the 7. In which of the following arrangements, the
variation in properties of elements is related to their order is NOT according to the property indicated
[2003] against it? [2005]
(a) nuclear masses (a) Li < Na < K < Rb :
(b) atomic numbers Increasing metallic radius
(c) nuclear neutron-proton number ratios (b) I < Br < F < Cl :
(d) atomic masses Increasing electron gain enthalpy
2. Which one of the following is an amphoteric (with negative sign)
oxide ? [2003] (c) B < C < N < O
(a) Na2O (b) SO2 Increasing first ionization enthalpy
(c) B2O3 (d) ZnO (d) Al 3+ < Mg 2+ < Na + < F -
3. Which one of the following ions has the highest Increasing ionic size
value of ionic radius ? [2004] 8. Following statements regarding the periodic
(a) O2– (b) B3+ trends of chemical reactivity of the alkali metals
(c) Li+ (d) F– and the halogens are given. Which of these
statements gives the correct picture? [2006]
4. Among Al2O3, SiO2, P2O3 and SO2 the correct
order of acid strength is [2004] (a) Chemical reactivity increases with increase
in atomic number down the group in both
(a) Al2O3 < SiO2< SO2 < P2O3
the alkali metals and halogens
(b) SiO2< SO2 < Al2O3 < P2O3
(b) In alkali metals the reactivity increases but
(c) SO2< P2O3 < SiO2 < Al2O3 in the halogens it decreases with increase
(d) Al2O3 < SiO2< P2O3 < SO2 in atomic number down the group
5. The formation of the oxide ion O (2g-) requires (c) The reactivity decreases in the alkali metals
first an exothermic and then an endothermic step but increases in the halogens with increase
as shown below [2004] in atomic number down the group
O(g) + e - = O(g)
-
DHº = -142 kJmol -1 (d) In both the alkali metals and the halogens
O - (g) + e - = O (g)
2-
DHº = 844 kJmol -1 the chemical reactivity decreases with
This is because increase in atomic number down the group
(a) O– ion will tend to resist the addition of 9. In which of the following arrangements, the
another electron sequence is not strictly according to the
(b) Oxygen has high electron affinity property written against it? [2008]
(c) Oxygen is more elecronegative (a) HF < HCl < HBr , HI : increasing acid
strength
(d) O– ion has comparatively larger size than
oxygen atom (b) NH3 < PH3 < AsH3 <SbH3 : increasing
basic strength
6. Which of the following oxides is amphoteric in
(c) B < C < O < N : increasing first ionization
character? [2005]
enthalpy
(a) SnO2 (b) SiO 2
(d) CO2 < SiO2 < SnO2 < PbO2 : increasing
(c) CO2 (d) CaO oxidising power
www.crackjee.xyz
Classification of Elements and Periodicity in Properties C-13
10. The correct sequence which shows decreasing Ca, Ba, S, Se and Ar ? [2013]
order of the ionic radii of the elements is (a) Ca < S < Ba < Se < Ar
[2010] (b) S < Se < Ca < Ba < Ar
(a) Al3+ > Mg 2+ > Na + > F- > O2- (c) Ba < Ca < Se < S < Ar
(b) Na + > Mg 2 + > Al3+ > O 2- > F- (d) Ca < Ba < S < Se < Ar
(c) Na + > F- > Mg2 + > O2 - > Al3+ 13. The ionic radii (in Å) of N3–, O2– and F– are
(d) O2 - > F- > Na + > Mg 2 + > Al3+ respectively : [JEE M 2015]
11. The correct order of electron gain enthalpy with (a) 1.71, 1.40 and 1.36
negative sign of F, Cl, Br and I, having atomic (b) 1.71, 1.36 and 1.40
number 9, 17, 35 and 53 respectively, is : (c) 1.36, 1.40 and 1.71
[2011RS] (d) 1.36, 1.71 and 1.40
(a) F > Cl > Br > I (b) Cl > F > Br > I 14. Which of the following atoms has the highest
(c) Br > Cl > I > F (d) I > Br > Cl > F first ionization energy? [JEE M 2016]
12. Which of the following represents the correct (a) K (b) Sc
order of increasing first ionization enthalpy for (c) Rb (d) Na

Answer Key
1 2 3 4 5 6 7 8 9 10 11 12 13 14
(b) (d) (a) (d) (a) (a) (c) (b) (b) (d) (b) (c) (a) (b)

1. (b) According to modern periodic law, the SO 2 and P 2 O 3 are acidic as their
properties of the elements are repeated after corresponding acids H2SO3 and H3PO3 are
certain regular intervals when these strong acids.
elements are arranged in order of their 5. (a) O– ion exerts a force of repulsion on the
increasing atomic numbers. incoming electron. The energy is required
2. (d) Na2O (basic), SO2 and B2O3 (acidic) and to overcome it.
ZnO is amphoteric. 6. (a) CaO is basic as it form strong base Ca(OH)2
3. (a) O– – and F– are isoelectronic. Hence have on reaction with water.
same number of shells, therefore greater CaO + H2O –––––® Ca(OH)2
the nuclear charge smaller will be the size CO2 is acidic as it dissolve in water forming
i.e. unstable carbonic acid.
O– – > F– H2O + CO2 –––––® H2CO3
further Li + and B3+ are isoelectronic. Silica (SiO2) is insoluble in water and acts
therefore as a very weak acid.
Li+ > B3+ SnO2 is amphoteric as it reacts with both
Hence the correct order of atomic size is. acid and base.
O-- > F– > Li+ > B3+ SnO2 + 2H2SO4–––––® Sn(SO4)2 + 2H2O
4. (d) As the size increases the basic nature of SnO2 + 2KOH–––––® K2SnO3 + H2O
oxides changes to acidic nature i.e., acidic 7. (c) In a period the value of ionisation potential
nature increases. increases from left to right with breaks
SO 2 > P 2 O 3 > SiO 2 > Al 2 O 3 where the atoms have some what stable
Acidic Weak Amphoteric configuration. In this case N has half filled
acidic
EBD_7764
www.crackjee.xyz
C-14 Chemistry
stable orbitals. Hence has highest 11. (b) As we move down in a group electron gain
ionisation energy. Thus the correct order enthalpy becomes less negative because
is the size of the atom increases and the
B< C< O < N distance of added electron from the nucleus
and not as given in option (c) increases. Negative electron gain enthalpy
8. (b) The alkali metals are highly reactive because of F is less than Cl. This is due to the fact
their first ionisation potential is very low that when an electron is added to F, the
and hence they have great tendency to added electron goes to the smaller n = 2
loses electron to form unipositive ion. energy level and experiences significant
repulsion from the other electrons present
NOTE On moving down group- I from
in this level. In Cl, the electron goes to the
Li to Cs ionisation enthalpy decreases larger n = 3 energy level and consequently
hence the reactivity increases. The
occupies a larger region of space leading
halogens are most reactive elements due
to much less electron–electron repulsion.
to their low bond dissociation energy, high
So the correct order is
electron affinity and high enthalpy of
hydration of halide ion. However their Cl > F > Br > I.
reactivity decreases with increase in atomic 12. (c) On moving down a group size increases
number hence ionisation enthalpy decreases,
9. (b) In hydrides of 15th group elements, basic hence Se < S and Ba < Ca. Further, Ar being
character decreases on descending the an inert gas has maximum IE.
group i.e. 13. (a) For isoelectronic species, size of anion in-
NH3 > PH3 > AsH3 > SbH3. creases as negative charge increases. Thus
the correct order is
10. (d) All the given species contains 10 e– each
14. (b) Alkali metals have the lowest ionization
i.e. isoelectronic. energy in each period on the other hand Sc
For isoelectronic species anion having is a d - block element.
high negative charge is largest in size and Transition metals have smaller atomic radii
the cation having high positive charge is and higher nuclear charge leading to high
smallest. ionisation energy.
www.crackjee.xyz
Chemical Bonding and Molecular Structure C-15

4
Chemical Bonding and
Molecular Structure
1. In which of the following species the interatomic 7. The pair of species having identical shapes for
bond angle is 109° 28’? [2002] molecules of both species is [2003]
(a) NH3, (BF4) –1 +
(b) (NH4) , BF3 (a) XeF2, CO2 (b) BF3, PCl3
(c) NH3, BF4 (d) (NH2)–1, BF3. (c) PF5, IF5 (d) CF4, SF4
2. Which of the following are arranged in an 8. The correct order of bond angles (smallest first)
increasing order of their bond strengths? [2002] in H2S, NH3, BF3 and SiH4 is [2004]
(a) O2– < O2 < O2+ <O22– (a) H2S < NH3 < SiH4 < BF3
(b) O22– < O2– < O2 <O2+ (b) NH3 < H2S < SiH4 < BF3
(c) O2– < O22– < O2 <O2+ (c) H2S < SiH4 < NH3 < BF3
(d) O2+ < O2 < O2– <O22– (d) H2S < NH3 < BF3 < SiH4
3. Hybridisation of the underline atom changes in: 9. The bond order in NO is 2.5 while that in NO+ is 3.
[2002] Which of the following statements is true for
these two species ? [2004]
(a) AlH3 changes to AlH -4
(a) Bond length in NO+ is equal to that in NO
(b) H 2 O changes to H3O+ (b) Bond length in NO is greater than in NO+
(c) Bond length in NO+ is greater than in NO
(c) NH 3 changes to NH +4 (d) Bond length is unpredictable
(d) in all cases 10. The states of hybridization of boron and oxygen
4. An ether is more volatile than an alcohol having atoms in boric acid (H3BO3) are respectively
the same molecular formula. This is due to [2004]
[2003] (a) sp3 and sp2 (b) sp2 and sp 3
(a) alcohols having resonance structures (c) sp2 and sp2 (d) sp3 and sp3
(b) inter-molecular hydrogen bonding in ethers 11. Which one of the following has the regular
(c) inter-molecular hydrogen bonding in tetrahedral structure ? [2004]
alcohols (a) BF4- (b) SF4
(d) dipolar character of ethers
5. Which one of the following pairs of molecules (c) XeF4 (d) [ Ni(CN) 4 ]2 -
will have permanent dipole moments for both
(Atomic nos. : B = 5, S = 16, Ni =28, Xe = 54)
members ? [2003]
12. The maximum number of 90º angles between
(a) NO2 and CO2 (b) NO2 and O3
bond pair-bond pair of electrons is observed in
(c) SiF4 and CO2 (d) SiF4 and NO2 [2004]
6. Which one of the following compounds has the (a) dsp2 hybridization
smallest bond angle in its molecule ? [2003]
(b) sp3d hybridization
(a) OH2 (b) SH2
(c) dsp3 hybridization
(c) NH3 (d) SO2
(d) sp3d2 hybridization
EBD_7764
www.crackjee.xyz
C-16 Chemistry
13. Lattice energy of an ionic compound depends 21. Which one of the following pairs of species
upon [2005] have the same bond order? [2008]
(a) Charge on the ion and size of the ion (a) CN– and NO+ (b) CN– and CN+
(b) Packing of ions only (c) O–2 and CN– (d) NO+ and CN+
(c) Size of the ion only 22. The bond dissociation energy of B – F in BF3 is
(d) Charge on the ion only 646 kJ mol–1 whereas that of C – F in CF4 is 515
14. Which of the following molecules/ions does not kJ mol–1. The correct reason for higher B – F
bond dissociation energy as compared to that of
contain unpaired electrons? [2006]
C – F is [2008]
+
(a) N2 (b) O2 (a) stronger s bond between B and F in BF3 as
compared to that between C and F in CF4.
(c) O 22 - (d) B2
(b) significant pp – pp interaction between B
15. In which of the following molecules/ions are all and F in BF3 whereas there is no possibility
the bonds not equal? [2006] of such interaction between C and F in CF4.
(a) XeF4 (b) BF4 – (c) lower degree of pp – pp interaction between
(c) SF4 (d) SiF4 B and F in BF3 than that between C and F
16. The decreasing values of bond angles from NH3 in CF4.
(106º) to SbH3 (101º) down group-15 of the (d) smaller size of B– atom as compared to that
periodic table is due to [2006] of C– atom.
(a) decreasing lp-bp repulsion 23. Using MO theory, predict which of the following
(b) decreasing electronegativity species has the shortest bond length? [2008]
(c) increasing bp-bp repulsion (a) O +2 (b) O 2–
(d) increasing p-orbital character in sp3 (d) O 2+
(c) O 2–
2 2
17. Which of the following species exhibits the
24. The number of types of bonds between two
diamagnetic behaviour? [2007]
carbon atoms in calcium carbide is : [2011RS]
(a) NO (b) O2 2–
(a) One sigma, One pi (b) Two sigma, one pi
(c) O2+ (d) O2.
(c) Two sigma, two pi (d) One sigma, two pi
18. The charge/size ratio of a cation determines its
25. Ortho-Nitrophenol is less soluble in water than
polarizing power. Which one of the following
p- and m- Nitrophenols because : [2012]
sequences represents the increasing order of
the polarizing power of the cationic species, K+, (a) o-Nitrophenol is more volatile steam than
Ca2+, Mg2+, Be2+? [2007] those of m- and p-isomers.
(a) Ca2+ < Mg2+ < Be+ < K+ (b) o-Nitrophenol shows intramolecular
H-bonding
(b) Mg2+ < Be2+ < K+ < Ca2+
(c) o-Nitrophenol shows intermolecular
(c) Be2+ < K+ < Ca2+ < Mg2+
H-bonding
(d) K+ < Ca2+ < Mg2+ < Be2+.
(d) Melting point of o-Nitrophenol is lower
19. In which of the following ionization processes, than those of m- and p-isomers.
the bond order has increased and the magnetic
26. In which of the following pairs the two species
behaviour has changed? [2007]
are not isostructural ? [2012]
(a) N 2 ® N2 + (b) C2 ® C2 + - +
(a) CO23 - and NO3 (b) PCl 4 and SiCl4
(c) NO ® NO + (d) O 2 ® O 2 + .
20. Which of the following hydrogen bonds is the (c) PF5 and BrF5 (d) AlF63 - and SF6
strongest? [2007] 27. Which one of the following molecules is
(a) O – H - - - F (b) O – H - - - H expected to exhibit diamagnetic behaviour ?
(c) F – H - - - F (d) O – H - - - O. [2013]
www.crackjee.xyz
Chemical Bonding and Molecular Structure C-17
(a) C2 (b) N2 (a) It is diamagnetic in gaseous state
(c) O2 (d) S2 (b) It is neutral oxide
28. Which of the following is the wrong statement (c) It combines with oxygen to form nitrogen
[2013] dioxide
(a) ONCl and ONO– are not isoelectronic. (d) It’s bond order is 2.5
(b) O3 molecule is bent 31. The species in which the N atom is in a state of
(c) Ozone is violet-black in solid state sp hybridization is : [JEE M 2016]
(d) Ozone is diamagnetic gas. (a) NO 3- (b) NO 2
29. In which of the following pairs of molecules/
ions, both the species are not likely to exist ? (c) NO +2 (d) NO 2-
[2013] 32. Which of the following species is not
+ 2 - - 2-
(a) H 2 , He 2 (b) H 2 , He 2 paramagnetic ? [JEE M 2017]
(c) H 22 + , He2 (d) H -2 , He 22 + (a) NO (b) CO
30. Which one of the following properties is not (c) O2 (d) B2
shown by NO? [2014]

Answer Key
1 2 3 4 5 6 7 8 9 10 11 12 13 14 15
(a) (b) (a) (c) (b) (b) (a) (a) (b) (b) (a) (d) (a) (c) (d)
16 17 18 19 20 21 22 23 24 25 26 27 28 29 30
(b) (b) (d) (c) (c) (a) (b) (d) (d) (b) (c) (a, b) () (c) (a)
31 32
(c) (b)

1. (a) In NH3 and BF4- the hybridisation is sp3 1


Bond order = (8 –6 ) = 1
2
and the bond angle is almost 109º 28'.
2. (b) O2+ (15) = KK s 2s2, s* 2s2, s 2px2 , NOTE As we know that as the bond order

{p2py2 = p2p2z , {p* 2 p1y =p2p0z decreases, stability also decreases and
hence the bond strength also decreases.
1 5 Hence the correct order of their increasing
Bond order = (8–3) = = 2.5
2 2 bond strength is
O2 (16) = KK s 2s2, s* 2s2, s 2px2,
O 22 - < O2– < O2 < O2+
{ p2py2 = p2pz2,{ p* 2py1 = p* 2pz1
3. (a)
1
Bond order = (8 – 4) = 2
2 é æ No. of electronsö
O2– (17) = KK s 2s2, s* 2s2, s 2px2, 1 êç ÷+
{p2py2 = p2pz2,{p* 2py2 = p* 2pz1 Hybridisation =
2 ê ç in valence ÷
ê è shell of atom ø
1 ë
Bond order = (8 – 5) = 1.5
2 æ No.of monovalentö æ charge on ö
– ç +
O22– (18) = KK s 2s2 s* 2s2 s2px2, çè atoms around it ÷ø è cation ÷ø
{p2py2 = p 2pz2,{p* 2py2 = p* 2pz2
EBD_7764
www.crackjee.xyz
C-18 Chemistry
9. (b) Now since bond order of NO+ given (3) is
æ charge on ö higher than that of NO (2.5). Thus bond
çè anion ÷ø length of NO+ will be shorter.
(a) For AlH3, 10. (b) H 3
O sp
Hybridisation of Al atom = 1 [ 3 + 3 - 0 + 0]
2
= 3 = sp2
For AlH4–, B sp
2

Hybridisation of Al atom = 1 [ 3 + 4 - 0 + 1]
2
= 4 = sp3
sp3 O O
H
(b) For H2O, sp3
Hybridisation of O atom H
1
= [ 6 + 2 - 0 + 0] = 4 = sp3
2
For H3O+, Hybridisation of O atom
11. (a) XeF4 (sp3d 2 square planar),

1 [ Ni(CN) 4 ]2 - (dsp2 square planar ),


=
2
[ 6 + 3 - 1 + 0] = 4 = sp3
BF4- (sp3 tetrahedral), SF4 (sp3d see saw
(c) For NH3
shaped)
Hybridisation of N atom
12. (d)
1
= [ 5 + 3 - 0 + 0] = 4 = sp3
2 M
For NH +4 , Hybridisation of N atom

1
=
2
[ 5 + 4 - 1 + 0] = 4 = sp3
Thus hybridisation changes only in
option (a).
4. (c) In ether, there is no H-bonding while
alcohols have intermolecular H-bonding
5. (b) Both NO2 and O3 have angular shape and dsp2 hybridisation sp 3 d or dsp 3
hybridisation
hence will have net dipole moment.
Number of 90° angle Number of 90°
6. (b) In H2S, due to low electronegativity of
angle
sulphur the L.P. - L. P. repulsion is more
between bonds = 4 between bonds = 6
than B. P. - B. P. repulsion and hence the
bond angle is minimum.
SO 2 H 2 O H 2S NH3
Bond angle 119.5° 104.5° 92.5° 106.5°
7. (a) Both XeF2 and CO2 have a linear structure.
F — Xe — F O=C=O
8. (a) The order of bond angles sp3d2 hybridisation
BF3 > SiH 4 > NH 3 > H 2S Number of 90° angle
120º 109º 28¢ 107 º 92.5º between bonds = 12
www.crackjee.xyz
Chemical Bonding and Molecular Structure C-19
13. (a) The value of lattice energy depends on the more unpaired electrons as in
charges present on the two ions and the O2 ® s1s2 , s*1s 2 , s2 s2 , s* 2s 2 , s2 px2 ,
distance between them.
14. (c) The distribution of electrons in MOs is as {p 2 p 2y = p 2 pz2 ,
follows :
{ p* 2 p1y = p* 2 p1z - 2 unpaired electrons
N 2 + (electr ons 13) s 2 s *2 s 2 s *2
p2 s1 p* s*
p2 p* O+2 ® s1s 2 , s*1s 2 , s2s 2 , s* 2s 2 , s2 p 2,
x

O2 (electrons 16) s2 s*2 s2 s*2 s2 p2 { p 2 p 2y = p 2 p z2 {p *


2 p1y = p * 2 pz0 - 1 unpaired electron
p2
unpaired electron
p*1 s*
p*1 NO ® s1s 2 , s*1s 2 , s 2 s 2 , s* 2 s 2 , s2 p 2 ,
x
2
O22– (electrons 18) s2 s*2 s2 s*2 s2 p 2
p p 2 p z2 , {p* 2 p1y = p* 2 p 0z - 1 unpaired electron
p* s* 18. (d) Smaller the size and higher the charge more
p* will be polarising power of cation. Since
1 the order of the size of cation is
B2 (electrons 10) s2 s*2 s2 s*2 p1
p K + > Ca ++ > Mg ++ > Be++ . So the
Only O22– does not contain any unpaired correct order of polarising power is
electron. K+ < Ca2+ < Mg2+ < Be2+
15. (d) In SF4 the hybridisation is sp3d and the 19. (c) (a) N2 : bond order 3, paramagnetic
shape of molecule is N2– : bond order, 2.5, paramagnetic
F (b) C2 : bond order 2, diamagnetic
F C2+ : bond order 1.5, paramagnetic
S (c) NO : bond order 2.5, paramagnetic
NO+ : bond order 3, diamagnetic
F (d) O2 : bond order 2, paramagnetic
F
16. (b) The bond angle decreases on moving down O2+ : bond order 2.5, paramagnetic
the group due to decrease in bond pair- 20. (c) NOTE Greater the difference between
bond pair repulsion.
NH3 PH3 ASH3 SbH3 BiH3 electro-negativity of bonded atoms,
107º 94º 92º 91º 90º stronger will be bond. Since F is most
electronegative hence F – H ...... F is the
NOTE This can also be explained by the strongest bond.
fact that as the size of central atom increases 21. (a) For any species to have same bond order
sp3 hybrid orbital becomes more distinct we can expect them to have same number
with increasing size of central atom i.e. pure of electrons. Calculating the number of
p- orbitals are utilized in M–H bonding electrons in various species.
17. (b) Diamagnetic species have no unpaired O2– (8 + 8 + 1 = 17) ; CN - (6 + 7 + 1 = 14)
electrons
NO+(7 + 8 – 1= 14); CN+ (6 + 7 –1 = 12)
2 We find CN– and NO+ both have 14
O 2 2- Þ s1s2, s*1s2, s2s2, s*2s2, s 2 px ,
{p2py2 = p2pz2, {p*2py2 = p*2pz2 electrons so they have same bond order.
Whereas paramagnetic species has one or Correct answer is (a).
EBD_7764
www.crackjee.xyz
C-20 Chemistry
22. (b) NOTE The delocalised pp - pp {p 2p2y = p 2pz2 } s 2 p2x
bonding between filled p-orbital of F and Thus M.O. configuration suggests that it
vacant p-orbital of B leads to shortening contains one s & 2p bonds.
of B–F bond length which results in higher 25. (b) Compounds involved in chelation become
bond dissociation energy of the B–F bond. non-polar. Consequently such compounds
are soluble in non-polar solvents like ether,
benzene etc. and are only sparingly soluble
in water whereas meta and para isomers
F are more soluble in water & less soluble in
B F non-polar solvents.
F H
O O
Å
Vacant Filled N
2p-orbital 2p -orbital O intra-molecular H-bonding

F F 26. (c) PF5 trigonal bipyramidal


+
B=F B–F
+
F
F F F
+1/3
F P
+
F F F
+1/3
B–F B F F
+1/3
F F BrF5 square pyramidal (distorted)
23. (d) Bond order
F
No.of bonding electrons - No.of antibonding electrons F F
=
2 Br
10 - 5 F F
+
Bond order in O2 = = 2.5
2
27. (a, b) The molecular orbital structures of C2 and
- 10 - 7
Bond order in O 2 = = 1.5 N2 are
2
10 - 8
Bond order in O 22 - = =1 N2 = s1s2 s *1s2 s2s2 s *2s 2 s2 px2p2 py2p2 pz2
2
10 - 4
Bond order in O 22 + = =3
2 C2 = s1s 2 s *1s 2 s 2s 2 s * 2s 2 p2 py 2 p 2 Pz2
1 Both N2 and C2 have paired electrons,
Since Bond order µ hence they are diamagnetic.
Bond length
28. All options are correct,
\ Bond length is shortest in O 22 + .
24. (d) Calcium carbide exists as Ca2+ and C22–.
(a) ONCl = 8 + 7 + 17 = 32e - ü not
- - ý isoelectronic
According to the molecular orbital model, ONO = 8 + 7 + 8 + 1 = 24e þ
C 2 2– should have molecular orbital ..
configuration : (b) 7 8Å O 1.278A°
1. 2 The central atom is sp 2
s1s2 s *1s2 s 2s2 s *2s2 O 116.8° O hybridized with one lone pair.
www.crackjee.xyz
Chemical Bonding and Molecular Structure C-21
(c) It is a pale blue gas. At – 249.7°, it forms violet NO+2 = i.e. sp hybridisation
black crystals. NO–2 = i.e. sp2 hybridisation
NO–3 = i.e. sp2 hybridisation
(d) It is diamagnetic in nature due to absence of The lewis structure of NO2 shows a bent mo-
unpaired electrons. lecular geometry with trigonal planar electron
29. (c) H2+ 0
2 = s1s s*1s
0 pair geometry hence the hybridization will be
sp 2
1
Bond order for H2+
2 =
(0 - 0) = 0 32. (b)
2
He2 = s1s2s*1s2 1. NO ® one unpaired electron is present in p*
molecular orbit hence paramagnetic.
1
Bond order for He2 = (2 - 2) = 0 2 *2 2 *2 2 2 2
2 2. CO ® s1s , s1s , s2s , s2s , p2p x , p2p y , s2pz
so both H22+ and He2 does not exist.
30. (a) Nitric oxide is paramagnetic in the gaseous no unpaired electron hence diamagnetic.
state because of the presence of one 2 *2 2 *2 2 2 2 *1 *1
3. O2® s1s , s1s , s2s , s2s , s2pz , p2px , p2p y , p2p x , p2p y
unpaired electron in its outermost shell.
The electronic configuration of NO is two unpaired electron hence paramagnetic.
s12s s1*2s s 22 s s*2 2 2
2s s2 pz p2 px = p22 p y p*1 2 *2 2 *2 1 1
2 px 4. B2 ® s1s , s1s , s2s , s2s , p2p x , p2p y
31. (c) Hybridization (H) = [no. of valence electrons
of central atom + no. of Monovalent atoms at- B2 contains two unpaired electrons hence
tached to it + (–ve charge if any) – (+ve charge paramagnetic
if any)]
EBD_7764
www.crackjee.xyz
C-22 Chemistry

States of Matter
5
1. For an ideal gas, number of moles per litre in 7. Which one of the following statements is NOT
terms of its pressure P, gas constant R and true about the effect of an increase in temperature
temperature T is [2002] on the distribution of molecular speeds in a gas?
(a) PT/R (b) PRT [2005]
(c) P/RT (d) RT/P. (a) The area under the distribution curve
2. Value of gas constant R is [2002] remains the same as under the lower
(a) 0.082 litre atm temperature
(b) 0.987 cal mol–1 K–1 (b) The distribution becomes broader
(c) 8.3 J mol–1 K–1 (c) The fraction of the molecules with the most
probable speed increases
(d) 83 erg mol–1 K–1.
(d) The most probable speed increases
3. Kinetic theory of gases proves [2002]
8. If 10–4 dm3 of water is introduced into a 1.0 dm3
(a) only Boyle’s law
flask at 300 K, how many moles of water are in
(b) only Charles’ law the vapour ph ase when equilibrium is
(c) only Avogadro’s law established ? [2010]
(d) all of these. (Given : Vapour pressure of H2O at 300 K is 3170
4. According to the kinetic theory of gases, in an Pa; R = 8.314 J K–1 mol–1)
ideal gas, between two successive collisions a (a) 5.56× 10–3 mol (b) 1.53 × 10–2 mol
gas molecule travels [2003] (c) 4.46 × 10 mol (d) 1.27 × 10–3 mol
–2
(a) in a wavy path 9. When r, P and M represent rate of diffusion,
(b) in a straight line path pressure and molecular mass, respectively, then
(c) with an accelerated velocity
the ratio of the rates of diffusion (rA / rB ) of
(d) in a circular path
two gases A and B, is given as [2011RS]
5. As the temperature is raised from 20ºC to 40ºC,
the average kinetic energy of neon atoms (a) ( PA / PB ) ( M B / M A )1/ 2
changes by a factor of which of the following ? (b) ( PA / PB )1/ 2 ( M B / M A )
[2004]
(c) ( PA / PB ) ( M A / M B )1/ 2
(a) 313 (b) (313 / 293)
293 (d) ( PA / PB )1/ 2 ( M A / M B )
(c) 1 (d) 2 10. The molecular velocity of any gas is : [2011RS]
2
(a) inversely proportional to absolute
6. In van der Waals equation of state of the gas temperature.
law, the constant ‘b’ is a measure of [2004] (b) directly proportional to squar e of
(a) volume occupied by the molecules temperature.
(b) intermolecular attraction (c) directly proportional to square root of
(c) intermolecular repulsions temperature.
(d) intermolecular collisions per unit volume (d) inversely proportional to the square root
www.crackjee.xyz
States of Matter C-23
of temperature. 14. The ratio of masses of oxygen and nitrogen in a
11. The compressibility factor for a real gas at high particular gaseous mixture is 1 : 4. The ratio of
pressure is : [2012] number of their molecule is: [2014]
RT (a) 1 : 4 (b) 7 : 32
(a) 1 + (b) 1 (c) 1 : 8 (d) 3 : 16
pb
15. The intermolecular interaction that is dependent
pb pb on the inverse cube of distance between the
(c) 1 + (d) 1 – molecules is : [JEE M 2015]
RT RT
12. For gaseous state, if most probable speed is (a) London force
denoted by C*, average speed by C and mean (b) hydrogen bond
square speed by C, then for a large number of (c) ion - ion interaction
molecules the ratios of these speeds are: (d) ion - dipole interaction
[2013] 16. Two closed bulbs of equal volume (V) contain-
ing an ideal gas initially at pressure pi and tem-
(a) C* : C : C = 1.225 : 1.128 : 1
perature T1 are connected through a narrow
(b) C* : C : C = 1.128 : 1.225 : 1
tube of negligible volume as shown in the figure
(c) C* : C : C = 1 : 1.128 : 1.225 below. The temperature of one of the bulbs is
(d) C* : C : C = 1 : 1.225 : 1.128 then raised to T2. The final pressure pf is :
13. If Z is a compressibility factor, van der Waals [JEE M 2016]
equation at low pressure can be written as:
[2014] æ T ö æ T T ö
(a) 2 pi ç 2 ÷ (b) 2 pi ç 1 2 ÷
è T1 + T2 ø è T1 + T2 ø
RT a
(a) Z = 1+ (b) Z =1-
Pb VRT æ T T ö æ T1 ö
(c) pi ç 1 2 ÷ (d) 2 pi ç ÷
Pb Pb è T1 + T2 ø è T1 + T2 ø
(c) Z = 1- (d) Z = 1+
RT RT

Answer Key
1 2 3 4 5 6 7 8 9 10 11 12 13 14 15
(c) (c) (d) (b) (a) (a) (c) (d) (a) (c) (c) (c) (b) (b) (b)
16
(a)

1. (c) PV = nRT (number of moles = n/V) with the walls of the container and between
\ n/V = P/RT.. two successive collisions molecules travel
2. (c) Value of gas constant in a straight line path but show haphazard
( R ) = 0.0821L atm K–1 mol–1 motion due to collisions.
= 8.314 × 107 ergs K–1mol–1 3
= 8.314JK–1mol–1 = 1.987 cal K–1 mol–1 K ´ 313
K .E of neon at 40 °C 313
3. (d) Kinetic theory of gases proves all the given 5. (a) = 2 =
gas laws. K.E of neon at 20°C 3 293
K ´ 293
4. (b) According to kinetic theory the gas 2
molecules are in a state of constant rapid 6. (a) In van der waals equation ‘b’ is for volume
motion in all possible directions colloiding correction
in a random manner with one another and 7. (c) Distribution of molecular velocities at two
different temperature is given shown below.
EBD_7764
www.crackjee.xyz
C-24 Chemistry
Most probable 8RT
Fraction of molecules velocity Average Speed (C) =
pM
300 K 3RT
Root mean square velocity (C) =
M
600 K
2RT 8RT 3RT
V C*: C : C = : :
M pM M
Molecular velocity
4 3
NOTE At higher temperature more = 1: : = 1:1.128 :1.225
p 2
molecules have higher velocities and less PV
molecules have lower velocities. 13. (b) Compressibility factor ( Z ) =
As evident from fig. thus it is clear that RT
(For one mole of real gas)
With the increase in temperature the most van der Waals equation
probable velocity increase butthe fraction a
of such molecules decreases. (P + )(V - b ) = RT
8. (d) From the ideal gas equation : V2
At low pressure, volume is very large and
PV = nRT
hence correction term b can be neglected
PV 3170 ´ 10-3 in comparison to very large volume of V.
or n = = = 1.27 × 10–3
RT 8.314 ´ 300 i.e. V - b » V
P æ a ö
9. (a) r µ
m çè P + 2 ÷ø V = RT
V
rA PA M B a
= PV + = RT
rB PB M A V
10. (c) The different type of molecular velocities a
possessed by gas molecules are PV = RT -
V
2RT PV a
(i) Most probable velocity (a) = = 1-
M RT VRT
2RT a
(ii) Average velocity v = Hence, Z = 1 -
M VRT
(iii) Root mean square velocity in all three w
3RT
14. (b) Number of moles of O 2 =
cases v = 32
M 4w w
In all the above cases Number of moles of N 2 = =
28 7
Velocity × T
a
\ Ratio = w : w = 7 : 32
æ a ö 32 7
11. (c) çè P + 2 ÷ø (V - b) = RT at high pressure 2
V V 15. (b) Hydrogen bond is a type of strong
can be neglected electrostatic dipole-dipole interaction and
PV – Pb = RT and PV = RT + Pb dependent on the inverse cube of distance
PV Pb between the molecular ion-dipole
= 1+ interaction.
RT RT 16. (a) For a given mass of an ideal gas, the
Pb volume and amount (moles) of the gas are
Z = 1+ ; Z > 1 at high pressure
RT directly proportional if the temperature and
pressure are constant. i.e
2RT Hence in the given case.
12. (c) Most probable speed (C*) = Initial moles and final moles are equal
M
(nT)i = (nT)f
www.crackjee.xyz
Thermodynamics C-25

Thermodynamics
6
1. If an endothermic reaction is non-spontaneous change for the reaction
at freezing point of water and becomes feasible H 2 C = CH 2 (g) + H 2 (g) ® H 3C — CH 3 (g) a t
at its boiling point, then [2002] 298 K will be [2003]
(a) DH is –ve, DS is +ve (a) – 250 kJ (b) + 125 kJ
(b) DH and DS both are +ve (c) – 125 kJ (d) + 250 kJ
(c) DH and DS both are –ve 7. In an irreversible process taking place at constant
(d) DH is +ve, DS is -ve T and P and in which only pressure-volume work
2. A heat engine abosrbs heat Q1 at temperature is being done, the change in Gibbs free energy
T1 and heat Q2 at temperature T2. Work done (dG) and change in entropy (dS), satisfy the criteria
by the engine is J (Q1 + Q2). This data [2002] [2003]
(a) violates 1st law of thermodynamics (a) (dS)V, E > 0, (dG)T, P < 0
(b) violates 1st law of themodynamics if Q1 is –ve (b) (dS)V, E = 0, (dG)T, P = 0
(c) (dS)V, E = 0, (dG)T, P > 0
(c) violates 1st law of thermodynamics of Q2 is –ve
(d) (dS)V, E < 0, (dG)T, P < 0
(d) does not violate 1st law of themodynamics.
8. The correct relationship between free energy
3. For the reactions, [2002] change in a reaction and the corresponding
2C + O2 ® 2 CO2 ; DH = -393 J equilibrium constant Kc is [2003]
2Zn + O2 ® 2ZnO ; DH = -412 J (a) – DG = RT ln Kc (b) DGº = RT ln Kc
(a) carbon can oxidise Zn (c) – DGº = RT ln Kc (d) DG = RT ln Kc
(b) oxidation of carbon is not feasible 9. The enthalpy change for a reaction does not
(c) oxidation of Zn is not feasible depend upon [2003]
(d) Zn can oxidise carbon. (a) use of different reactants for the same
4. The heat required to raise the temperature of product
body by 1 K is called [2002] (b) the nature of intermediate reaction steps
(a) specific heat (b) thermal capacity (c) the differences in initial or final
(c) water equivalent (d) none of these. temperatures of involved substances
5. The internal energy change when a system goes (d) the physical states of reactants and
products
from state A to B is 40 kJ/mole. If the system goes
10. An ideal gas expands in volume from 1×10–3 to
from A to B by a reversible path and returns to
1 × 10–2 m3 at 300 K against a constant pressure
state A by an irreversible path what would be the
of 1×105 Nm–2. The work done is [2004]
net change in internal energy ? [2003]
(a) 270 kJ (b) – 900 kJ
(a) > 40 kJ (b) < 40 kJ
(c) – 900 (d) 900 kJ
(c) Zero (d) 40 kJ
11. The enthalpies of combustion of carbon and
6. If at 298 K the bond energies of C — H, C — C, carbon monoxide are – 393.5 and – 283 kJ mol–1
C = C and H — H bonds are respectively 414, respectively. The enthalpy of formation of
347, 615 and 435 kJ mol–1, the value of enthalpy carbon monoxide per mole is [2004]
EBD_7764
www.crackjee.xyz
C-26 Chemistry
(a) – 676.5 kJ (b) 676.5 kJ Cl2(g) = 2Cl(g), 242.3 kJ mol–1
(c) 110.5 kJ (d) – 110.5 kJ I2(g) = 2I(g), 151.0 kJ mol–1
12. Consider the reaction : N 2 + 3H 2 ® 2 NH 3 ICl(g) = I(g) + Cl(g), 211.3 kJ mol–1
carried out at constant temperature and I2(s) = I2(g), 62.76 kJ mol–1
pressure. If DH and DU are the enthalpy and Given that the standard states for iodine and
chlorine are I2 (s) and Cl 2 (g), the standard
internal energy changes for the reaction, which
enthalpy of formation for ICl(g) is : [2006]
of the following expressions is true ? [2005] –1
(a) +16.8 kJ mol (b) +244.8 kJ mol–1
(a) DH > DU (b) DH < DU
(c) –14.6 kJ mol–1 (d) –16.8 kJ mol–1
(c) DH = DU (d) DH = 0
17. (DH – DU) for the formation of carbon monoxide
13. If the bond dissociation energies of XY,
(CO) from its elements at 298 K is [2006]
X 2 and Y2 (all diatomic molecules) are in the (R = 8.314 J K–1 mol–1)
ratio of 1 : 1 : 0.5 and DHf for the formation of XY (a) –2477.57 J mol–1 (b) 2477.57 J mol–1
is – 200 kJ mole -1 . The bond dissociation (c) –1238.78 J mol–1 (d) 1238.78 J mol–1
energy of X 2 will be [2005] 18. In conversion of lime-stone to lime,
CaCO3(s) ® CaO(s) + CO2(g) the values of
-1
(a) 400 kJ mol (b) 300 kJ mol -1
DH° and DS° are + 179.1 kJ mol-1 and 160.2 J/K
-1 -1 respectively at 298 K and 1 bar. Assuming that
(c) 200 kJ mol (d) 100 kJ mol
14. An ideal gas is allowed to expand both reversibly DH° and DS° do not change with temperature,
and irreversibly in an isolated system. If Ti is temperature above which conversion of
the initial temperature and T f is the final limestone to lime will be spontaneous is[2007]
temperature, which of the following statements (a) 1118 K (b) 1008 K
is correct? [2006] (c) 1200 K (d) 845 K.
(a) (Tf)rev = (Tf)irrev 19. Assuming that water vapour is an ideal gas, the
(b) Tf = Ti for both reversible and irreversible
processes internal energy change (DU) when 1 mol of
(c) (Tf)irrev > (Tf)rev water is vapourised at 1 bar pressure and 100°C,
(d) Tf > Ti for reversible process but Tf = Ti (given : molar enthalpy of vapourisation of water
for irreversible process at 1 bar and 373 K = 41 kJ mol–1 and R = 8.3 J
15. The standard enthalpy of formation (DfHº) at mol–1 K–1) will be [2007]
298 K for methane, CH4 (g) is –74.8 kJ mol–1. (a) 41.00 kJ mol–1 (b) 4.100 kJ mol–1
The additional information required to determine (c) 3.7904 kJ mol–1 (d) 37.904 kJ mol–1
the average energy for C – H bond formation 20. Identify the correct statement regarding a
would be [2006] spontaneous process: [2007]
(a) the first four ionization energies of carbon (a) Lowering of energy in the process is the
and electron gain enthalpy of hydrogen only criterion for spontaneity.
(b) the dissociation energy of hydrogen (b) For a spontaneous process in an isolated
molecule, H2 system, the change in entropy is positive.
(c) the dissociation energy of H2 and enthalpy (c) Endothermic processes are never
of sublimation of carbon spontaneous.
(d) latent heat of vapourization of methane (d) Exothermic processes are always
16. The enthalpy changes for the following spontaneous.
processes are listed below : [2006] 21. Oxidising power of chlorine in aqueous solution
www.crackjee.xyz
Thermodynamics C-27
can be determined by the parameters indicated 26. The value of enthalpy change (DH) for the
below: reaction
1
D H Q C2 H5OH(l) + 3O2 (g) ®
1 2 diss D eg HQ
Cl2 (g) ¾¾¾¾¾ ® Cl(g) ¾¾¾¾ ® Cl– (g)
2 2CO 2 (g) + 3H 2O(l)
D Q
H at 27° C is – 1366.5 k J mol–1. The value of internal
Hyd
¾¾¾¾ ® Cl– (aq) energy change for the above reaction at this
(using the data, temperature will be : [2011RS]
–1
D diss H Cl = 240 kJ mol , (a) – 1369.0 kJ (b) – 1364.0 kJ
2
–1
(c) – 1361.5 kJ (d) – 1371.5 kJ
D eg H Cl = –349 kJ mol , 27. Consider the reaction :
–1
D hyd H Cl – = –381 kJ mol ), will be [2008] 4NO2 (g) + O 2 (g) ® 2N 2 O5 (g),
DrH = – 111 kJ.
(a) + 152 kJ mol–1 (b) – 610 kJ mol–1
If N2O5(s) is formed instead of N2O5(g) in the
(c) – 850 kJ mol–1 (d) + 120 kJ mol–1
above reaction, the DrH value will be :
22. Standard entropy of X2, Y2 and X Y3 are 60, 40
(given, DH of sublimation for N2O5 is 54 kJ
and 50 J K–1 mol –1 , respectively. For the
mol–1) [2011RS]
reaction,
(a) + 54 kJ (b) + 219 kJ
1 3 (c) – 219 J (d) – 165 kJ
X 2 + Y2 ® XY3 , DH = –30kJ , to be at
2 2 28. The incorrect expression among the following
equilibrium, the temperature will be [2008]
is: [2012]
(a) 1250 K (b) 500 K
(c) 750 K (d) 1000 K DGsystem
(a) = -T
23. On the basis of the following thermochemical DStotal
+
data : (Δ f G°H (aq) = 0) [2009] (b) In isothermal process,
+ –
H 2 O(l ) ® H (aq) + OH (aq); DH = 57.32kJ V
wreversible = –nRT ln f
1 Vi
H2 (g)+ O2 (g) ¾¾ ® H2 O(l); ΔH=–286.20kJ
2
The value of enthalpy of formation of OH– ion DHº -TDSº
(c) ln K =
at 25° C is: RT
(a) –228.88 kJ (b) +228.88 kJ (d) K = e–DGº/RT
(c) –343.52 kJ (d) –22.88 kJ 29. A piston filled with 0.04 mol of an ideal gas
24. The standard enthalpy of formation of NH3 expands reversibly from 50.0 mL to 375 mL at a
is – 46.0 kJ mol–1. If the enthalpy of formation constant temperature of 37.0ºC. As it does so, it
of H2 from its atoms is – 436 kJ mol–1 and that of
absorbs 208 J of heat. The values of q and w for
N2 is – 712 kJ mol–1, the average bond enthalpy
the process will be: [2013]
of N – H bond in NH3 is [2010]
(R = 8.314 J/mol K) (ln 7.5 = 2.01)
(a) – 964 kJ mol–1 (b) + 352 kJ mol–1
(c) + 1056 kJ mol–1 (d) – 1102 kJ mol–1 (a) q = + 208 J, w = – 208 J
25. For a particular reversible reaction at (b) q = – 208 J, w = – 208 J
temperature T, DH and DS were found to be both (c) q = – 208 J, w = + 208 J
+ve. If Te is the temperature at equilibrium, the (d) q = + 208 J, w = + 208 J
reaction would be spontaneous when 30. For complete combustion of ethanol,
(a) Te > T (b) T > Te [2010] C2 H5 OH ( l ) + 3O2 ( g ) ¾¾
® 2CO2 ( g ) + 3H 2 O ( l ) ,
(c) Te is 5 times T (d) T = Te
EBD_7764
www.crackjee.xyz
C-28 Chemistry
the amount of heat produced as measured in 32. The heats of combustion of carbon and carbon
bomb calorimeter, is 1364.47 kJ mol–1 at 25ºC. monoxide are –393.5 and –283.5 kJ mol–1,
respectively. The heat of formation (in kJ) of
Assuming ideality the enthalpy of combustion, carbon monoxide per mole is : [JEE M 2016]
DcH, for the reaction will be: (a) –676.5 (b) – 110.5
(R = 8.314 kJ mol–1) [2014] (c) 110.5 (d) 676.5
33. DU is equal to [JEE M 2017]
(a) -1366.95 kJ mol-1
(a) Isochoric work (b) Isobaric work
(b) -1361.95 kJ mol-1 (c) Adiabatic work (d) Isothermal work
34. Given [JEE M 2017]
(c) -1460.95 kJ mol-1
C(graphite) + O2(g) ® CO2(g) ;
(d) -1350.50 kJ mol-1 DrH° = –393.5 kJ mol–1
31. The following reaction is performed at 298 K.
[JEE M 2015] 1
H2 (g) + O (g) ® H2O(l) ; DrH° = –285.8 kJ
2NO(g) + O2(g) 2NO2(g) 2 2
The standard free energy of formation of NO(g) mol–1
is 86.6 kj/mol at 298 K. What is the standard free CO2(g) + 2H2O(l) ® CH4(g) + 2O2(g);
energy of formation of NO2(g) at 298 K? DrH° = + 890.3 kJ mol–1
(Kp = 1.6 × 1012) Based on the above thermochemical equations,
(a) 86600 – the value of DrH° at 298 K for the reaction
(b) 0.5[2 × 86,600 – R(298) ln(1.6 × 1012)] C(graphite) + 2H2(g) ® CH4(g) will be :
(c) R(298) ln(1.6 × 1012) – 86600 (a) + 74.8 kJ mol–1 (b) + 144.0 kJ mol–1
(d) 86600 + R(298) ln(1.6 × l012) (c) – 74.8 kJ mol–1 (d) – 144.0 kJ mol–1

Answer Key
1 2 3 4 5 6 7 8 9 10 11 12 13 14 15
(b) (a) (d) (b) (c) (c) (a) (c) (b) (c) (d) (b) (N) (c) (c)
16 17 18 19 20 21 22 23 24 25 26 27 28 29 30
(a) (d) (a) (d) (b) (b) (c) (a) (b) (b) (b) (d) (c) (a) (a)
31 32 33 34
(b) (b) (c) (c)

1. (b) DG = DH – TDS 2. (a) According to first law of thermodynamics


Since DG = DH – TDS for an endothermic energy can n either be created n or
reaction, destroyed although it can be converted
DH = +ve and at low temperature D S = + ve from one form to another.
Hence DG = (+) DH – T ( + )DS NOTE Carnot cycle is based upon this
and if T D S < DH (at low temp)
DG = +ve (non spontaneous) principle but during the conversion of heat
But at high temperature, reaction becomes into work some mechanical energy is
spontaneous i.e. DG = –ve. always converted to other form of energy
because at higher temperature TDS > DH. hence this data violates 1st law of
thermodynamics.
www.crackjee.xyz
Thermodynamics C-29
3. (d) DH negative shows that the reaction is 13. (N) X 2 + Y2 ¾¾® 2 XY , DH = 2(–200).
spontaneous. Higher negative value for Zn Let x be the bond dissociation energy of
shows that the reaction is more feasible. X2. Then
4. (b) The heat required to raise the temperature
of body by 1K is called thermal capacity or DH = -400 = x x - x + x y - y - 2x x - y
heat capacity. = x + 0.5 x – 2 x = -0.5 x
5. (c) For a cyclic process the net change in the
internal energy is zero because the change 400
or x = = 800 kJ mol -1
in internal energy does not depend on the 0.5
path. (In the question paper, this option was not
reversible path mentioned. So the answer has been marked ‘N’)
14. (c) NOTE In a reversible process the work
A B done is greater than in irreversible process.
Hence the heat absorbed in reversible
process would be greater than in the latter
irreversible path case. So
Tf (rev.) < Tf (irr.)
6. (c) CH2 = CH2 (g) + H2 (g) ® CH3 - CH3 15. (c) The standard enthalpy of formation of CH4
Enthalpy change = Bond energy of is given by the equation :
reactants – Bond energy of products. C (s ) + 2 H 2 (g ) ¾
¾® CH 4 (g )
D H = 1(C = C) + 4 (C – H) + 1 (H- H) - 1 (C - C) Hence, dissociation energy of hydrogen
- 6 (C - H) and enthalpy of sublimation of carbon is
= 1 (C = C) + 1 ( H – H) – 1 (C – C) – 2 (C– H) required.
= 615 + 435 – 347 – 2 × 414 = 1050 – 1175 = 16. (a) I 2 (s) + Cl 2 (g) ¾¾
® 2ICl(g)
–125 kJ.
DA = [DHI2(s) ®I2(g) + DHI–I + DHCl–Cl] –
7. (a) For spontaneous reaction, dS > 0 and dG
[DHI – Cl]
should be negative i.e. < 0.
= 151.0 + 242.3 + 62.76 –2 × 211.3 = 33.46
8. (c) DGº = – RT lnKc or – DGº = RT lnKc
33.46
9. (b) Enthalpy change for a reaction does not DH of ( ICl ) = = 16.73 kJ / mol
depend upon the nature of intermediate 2
reaction steps. 1
17. (d) For the reaction, C (s ) + O 2 (g ) ¾ ¾® CO
10. (c) w = –PDV = – 10–5 (1 ´ 10 –2 –1´ 10–3) 2
= – 900 J DH = DU + DnRT or DH – DU = DnRT
1 1
11. (d) (i) C + O2 CO2, DH = –393.5 kJmol–1 Dn = 1 - = ;
2 2
1 1
(ii) CO+ O 2 CO2, DH = –283.0 DH – DU = ´ 8.314 ´ 298
2 2
kJmol–1 = 1238.78 J mol–1
Operating (i) - (ii), we have 18. (a) DG° = DH° – TDS°
For a spontaneous reaction DG° < 0
1
C + O2 ® CO DH = -110.5 kJmol -1 DH °
2 or DH° – TDS° < 0 Þ T >
12. (b) DH = DU + DnRT for DS°
3
179.3 ´ 10
Þ T> > 1117.9K » 1118K
N 2 + 3H 2 ¾ ¾® 2 NH 3 160.2
Dng = 2 – 4 = – 2 19. (d) Given DH = 41 kJ mol–1 = 41000 J mol–1
\ D H = D U - 2 RT or D U T = 100°C = 273 + 100 = 373 K
n=1
= D H+ 2R T \ D U > D H
EBD_7764
www.crackjee.xyz
C-30 Chemistry
DU = DH – DnRT = 41000 – (2 × 8.314 × 373)
DH r = DH°f (H + .aq)+DH°f (OH – .aq)–
= 37898.88 J mol–1 ; 37.9 kJmol–1
20. (b) Spontaneity of reaction depends on DH°f (H2 O, l)
tendency to acquire minimum energy state
and maximum randomness. For a 57.32 = 0 + DH°f (OH – , aq) – DH°f (H 2 O, l) …(iii)
spontaneous process in an isolated system For reaction (ii)
the change in entropy is positive. DH r = DH°f (H 2 O, l) –
21. (b) The energy involved in the conversion of
1
1 DH°f (H 2 , g) – DH°f (O2 , g )
Cl2 (g) to Cl–1 (aq) is given by 2
2
1 –286.20 = DH°f (H 2 O, l)
DH = D diss H (–)
Cl2
+ D eg H (–) (–)
Cl + D hyl H Cl On replacing this value in equ. (iii) we have
2
Substituting various values from given
57.32 = DH°f (OH – , aq) – (–286.20)
data, we get
DH°f = –286.20 + 57.32
DH = æç ´ 240 ö÷ + (–349) + (–381) kJmol –1
1
è2 ø = –228.88 kJ
= (120 – 349 – 381) kJ mol–1 = – 610 kJ mol–1
i.e., the correct answer is (b) 24. (b) N 2 + 3H 2 ¾¾ ® 2NH 3 DH = 2 ´ -46.0
22. (c) For a reaction to be at equilibrium DG = 0. kJ mol–1
Since DG = DH – TDS so at equilibrium Let x be the bond enthalpy of N – H bond
then
DH – TDS = 0
[Note : Enthalpy of formation or bond
or DH = TDS formation enthalpy is given which is
For the reaction negative but the given reaction involves
1 3 bond breaking hence values should be
X 2 + Y2 ¾¾ ® XY3 ; DH = –30kJ
2 2 taken as positive.]
(given) DH = S Bond energies of products – S Bond
Calculating DS for the above reaction, we energies of reactants
get 2 × – 46 = 712 + 3 × (436) – 6x
é1 3 ù – 92 = 2020 – 6x
DS = 50 - ê ´ 60 + ´ 40ú JK -1 6x = 2020 + 92
ë2 2 û
=50 – (30 + 60) JK–1 = – 40 JK–1 6x = 2112
x = + 352 kJ/mol
At equilibrium, TDS = DH
25. (b) At equilibrium DG = 0
[Q DG = 0] Hence, DG = DH – TeDS = 0
\ DH
\ DH = TeDS or Te =
T ´ (–40) = –30 ´ 1000 [Q 1kJ = 1000J] DS
For a spontaneous reaction
–30 ´1000
or T= or 750 K DG must be negative which is possible only
–40 if DH < TDS
23. (a) Given, for reaction
® H + (aq.) + OH – (aq.);
(i)H2O (l) ¾¾ or T > DH ; Te < T
DS
DH r = 57.32 kJ 26. (b) C2 H5OH(l) + 3O 2 (g)
1
(ii) H 2 ( g ) + O 2 ( g ) ¾¾ ® H 2 O(l); ® 2CO 2 (g) + 3H 2O(l)
2 Dng = 2 - 3 = -1
DH r = –286.20 kJ
DU = DH - Dng RT
For reaction (i)
www.crackjee.xyz
Thermodynamics C-31
= -1366.5 - (-1) Given equation,
2NO(g) + O2 (g) 2NO2(g)
8.314
= -1366.5 - (1) ´ ´ 300 \ 2DG°NO2 – 2DG°NO = – R(298) ln (1.6 × 1012)
103
= – 1366.5 + 0.8314 × 3 = – 1364 kJ 2DG°NO2 – 2 × 86600 = – R(298) ln (1.6 × 1012)
27. (d) 2DG°NO2 = 2 × 86600 – R(298) ln (1.6 × 1012)
4NO 2 (g) + O 2 (g) ® 2N 2 O 5 (g), D rH = – 111 kJ DG°NO2 = [2 × 86600 – R(298) ln (1.6 × 1012]

– 54
= 0.5 [2 × 86600 – R (298) ln (1.6 × 1012)]
D H' 32. (b) Given
2N 2 O5 (s) C(s) + O2(g) ® CO2(g); DH= –393.5 kJ mol–1 …(i)
– 111 – 54 = D H' CO(g)+ O2(g) ® CO2(g); DH= –283.5 kJ mol–1…(ii)
D H' = – 165 kJ \ Heat of formation of CO = eqn(i) – eqn (ii)
28. (c) DG° = DH° – TDS° = –393.5 – (–283.5)
– RT lnK = DH° – TDS° = –110 kJ
DH° - TDS° 33. (c) From 1st law of thermodynamics
lnK = -
RT DU = q + w
29. (a) Process is isothermal reversible expansion, For adiabatic process :
hence q=0
DU = 0, therefore q = – w.
\ DU = w
Since q = + 208 J, w = – 208 J
34. (c) Given
30. (a) C 2 H 5 OH(l ) + 3O 2 ( g ) ¾¾
®
CO2(g) + 2H2O(l) ® CH4(g) + 2O2(g);
2CO2 ( g ) + 3H2O(l)
DrH° = 890.3...(i)
Bomb calorimeter gives DU of the reaction
C(graphite) + O2 (g) ® CO2(g) ;
Given, DU = –1364.47 kJ mol–1
Dng = – 1 DrH° = – 393.5 kJ mol–1 ...(ii)
DH = DU + DngRT = 1
H2(g) + O2(g) ® H2O(l);
1 ´ 8.314 ´ 298 2
-1364.47 - DrH° = – 285.8 kJ mol–1 ...(iii)
1000
= – 1366.93 kJ mol–1 D r H° = å (D r H°) products – å (Df H°)Reactants
31. (b) DG°NO(g) = 86.6k J/mol = 86600 J/mol 890.3 = é1´ ( D f H °)CH + 2 ´ 0 ù – [1´ ( -393.5 ) + 2 ( –285.8 ) ]
ë 4 û
= x J/mol
12 ( D f H °) CH = 890.3 – 965.1 = –74.8 kJ / mol
T = 298, KP = 1.6 × 10 4
DG° = – RT ln K
P
EBD_7764
www.crackjee.xyz

7
C-32 Chemistry

Equilibrium
7
1. 1 M NaCl and 1 M HCl are present in an aqueous 8. For the reaction equilibrium [2003]
solution. The solution is [2002] N2O4 (g) 2 NO2 (g)
(a) not a buffer solution with pH < 7 the concentrations of N 2 O 4 and NO 2 at
(b) not a buffer solution with pH > 7 equilibrium are 4.8 × 10–2 and 1.2 × 10–2 mol L–1
(c) a buffer solution with pH < 7 respectively. The value of Kc for the reaction is
(d) a buffer solution with pH > 7. (a) 3 × 10–1 mol L–1 (b) 3 × 10–3 mol L–1
2. Species acting as both Bronsted acid and base (c) 3 × 103 mol L–1 (d) 3.3 × 102 mol L–1
is [2002] 9. Consider the reaction equilibrium [2003]
(a) (HSO4) –1 (b) Na2CO3 2 SO2 (g) + O2(g) 2 SO3 (g) ; DHº = –198 kJ
(c) NH3 (d) OH–1. On the basis of Le Chatelier’s principle, the
3. Let the solubility of an aqueous solution of condition favourable for the forward reaction is
Mg(OH)2 be x then its Ksp is [2002] (a) increasing temperature as well as pressure
(a) 4x 3 (b) 108x 5
(b) lowering the temperature and increasing the
(c) 27x4 (d) 9x. pressure
4. Change in volume of the system does not alter (c) any value of temperature and pressure
which of the following equilibria? [2002] (d) lowering of temperature as well as pressure
(a) N2 (g) + O2 (g) 2NO (g) 10. When rain is accompanied by a thunderstorm,
(b) PCl5 (g) PCl3 (g) + Cl2 (g) the collected rain water will have a pH value
(c) N2 (g) + 3H2 (g) 2NH3 (g) [2003]
(d) SO2Cl2 (g) SO2 (g) + Cl2 (g). (a) slightly higher than that when the
5. For the reactionCO (g) + (1/2) O2 (g) = CO2 (g), thunderstorm is not there
Kp / Kc is [2002] (b) unin fluenced by occurrence of
(a) RT (b) (RT) –1 thunderstorm
(c) (RT)–1/2 (d) (RT)1/2 (c) which depends on the amount of dust in
6. Which one of the following statements is not air
true? [2003] (d) slightly lower than that of rain water without
(a) pH + pOH = 14 for all aqueous solutions thunderstorm.
(b) The pH of 1 × 10–8 M HCl is 8
11. The conjugate base of H 2 PO -4 is [2004]
(c) 96,500 coulombs of electricity when passed
through a CuSO4 solution deposits 1 gram (a) H3PO4 (b) P2O5
equivalent of copper at the cathode
(c) PO 34- (d) HPO 24 -
(d) The conjugate base of H 2 PO -4 is HPO 24-
7. The solubility in water of a sparingly soluble 12. What is the equilibrium expression for the reaction
salt AB2 is 1.0 × 10–5 mol L–1. Its solubility P4 (s) + 5O2 (g) P4 O10 (s) ? [2004]
product number will be [2003]
(a) 4 × 10–10 (b) 1 × 10–15 (a) Kc = [O 2 ]5
(c) 1 × 10–10 (d) 4 × 10–15
www.crackjee.xyz
Equilibrium C-33

(b) Kc = [P4O10 ] / 5[P4 ][O2 ] 18. The exothermic formation of CIF3 is represented
by the equation :
(c) Kc = [P4 O10 ]/[P4 ][O 2 ]5 Cl2 (g) + 3F2 (g) 2ClF3 (g) ;

(d) Kc = 1/[O 2 ]5 Δ H = – 329 kJ


13. For the reaction, Which of the following will increase the quantity
of CIF3 in an equilibrium mixture of
Kp Cl 2 , F2 and ClF3 ? [2005]
CO(g) + Cl 2 (g) COCl2 (g) the Kc
(a) Adding F2
is equal to [2004] (b) Increasing the volume of the container
(a) RT (b) RT (c) Removing Cl2
(d) Increasing the temperature
(c) 1 (d) 1.0
RT 19. For the reaction : [2005]
14. The equilibrium constant for the reaction 2NO2(g) 2NO (g ) + O 2(g ) ,
N 2 (g) + O 2 (g) 2NO2 (g) at temperature
(K c = 1.8 ´ 10- 6 at 184°C) (R = 0.0831 kJ/
T is 4×10–4. The value of Kc for the reaction
(mol. K))
1 1
NO2 (g) N 2 (g) + O2 (g) at the same When K p and K c are compared at 184°C, it is
2 2
found that
temperature is [2004]
–4
(a) Whether Kp is greater than, less than or
(a) 4 × 10 (b) 50 equal to Kc depends upon the total gas
(c) 2.5 × 102 (d) 0.02 pressure
15. The molar solubility (in mol L–1) of a sparingly (b) Kp = Kc
soluble salt MX4 is ‘s’. The corresponding (c) Kp is less than Kc
solubility product is Ksp. ‘s’ is given in term of
(d) Kp is greater than Kc
Ksp by the relation : [2004]
20. Hydrogen ion concentration in mol/L in a
(a) s = (256 K sp )1/ 5 (b) s = (128 K sp )1/ 4 solution of pH = 5.4 will be : [2005]
(a) 3.98 ´ 10 -6 (b) 3.68 ´ 10 -6
1/ 4 1/ 5
(c) s = ( K sp /128) (d) s = ( K sp / 256)
(c) 3.88 ´ 106 (d) 3.98 ´ 108
16. If a is the degree of dissociation of Na 2SO 4 , 21. What is the conjugate base of OH - ? [2005]
the Vant Hoff’s factor (i) used for calculating
(a) O2- (b) O -
the molecular mass is
(c) H2O (d) O 2
(a) 1 – 2 a (b) 1 + 2 a [2005]
(c) 1 – a (d) 1 + a 22. An amount of solid NH 4 HS is placed in a flask
17. The solubility product of a salt having general already containing ammonia gas at a certain
temperature and 0.50 atm pressure. Ammonium
formula MX 2 , in water is : 4 × 10 -12 . The hydrogen sulphide decomposes to yield
concentration of M 2+ ions in the aqueous NH 3 and H 2S gases in the flask. When the
solution of the salt is [2005] decomposition reaction reaches equilibrium, the
total pressure in the flask rises to 0.84 atm? The
(a) 4.0 ´ 10 -10 M (b) 1.6 ´ 10 -4 M
equilibrium constant for NH 4 HS decomposition
-4 -6
(c) 1.0 ´ 10 M (d) 2.0 ´ 10 M at this temperature is [2005]
EBD_7764
www.crackjee.xyz
C-34 Chemistry
(a) 0.11 (b) 0.17 stant Ksp of AgIO3 at a given temperature is
(c) 0.18 (d) 0.30 1.0 × 10–8, what is the mass of AgIO3 contained
23. Phosphorus pentachloride dissociates as in 100 ml of its saturated saolution? [2007]
follows, in a closed reaction vessel [2006] (a) 1.0 × 10– 4 g (b) 28.3 × 10–2 g
PCl5(g) PCl3(g) + Cl2(g) (c) 2.83 × 10–3 g (d) 1.0 × 10–7 g.
29. The equilibrium constants K p and K p2 for the
If total pressure at equilibrium of the reaction 1
mixture is P and degree of dissociation of PCl5 is reactions X ƒ2Y and Z ƒ P + Q,
x, the partial pressure of PCl3 will be respectively are in the ratio of 1 : 9. If the degree
of dissociation of X and Z be equal then the
æ x ö æ x ö
(a) çè ÷P (b) ç
è 1 - x ÷ø
P ratio of total pressures at these equilibria is
x - 1ø
[2008]
æ x ö æ 2x ö (a) 1: 36 (b) 1 : 1
(c) çè ÷P (d) ç
è 1 - x ÷ø
P
x + 1ø (c) 1 : 3 (d) 1 : 9
24. The equilibrium constant for the reaction 30. For the following three reactions a, b and c,
1 equilibrium constants are given: [2008]
SO3(g) SO 2 (g ) + O 2 (g)
2 (i) CO(g) ∗ H 2O(g) ƒ CO 2 (g) ∗ H 2 (g); K1
is Kc = 4.9 × 10–2. The value of Kc for the reaction (ii) CH4 (g) ∗ H2O(g) ƒ CO(g) ∗ 3H2 (g); K 2
2SO2(g) + O2(g) 2SO3(g)
(iii) CH4 (g) + 2H2O(g) ƒ CO2 (g) + 4H2 (g);K3
will be [2006]
(a) 9.8 × 10–2 (b) 4.9 × 10–2 (a) K1 K 2 = K3 (b) K 2 K3 = K1
(c) 416 (c) 2.40 × 10–3
(c) K3 = K1 K2 (d) K3 .K 23 = K12
25. Given the data at 25ºC
31. Four species are listed below: [2008]
Ag + I - ¾
¾® AgI + e - E º = 0.152 V – +
i. HCO3 ii. H3 O
¾® Ag+ + e -
Ag ¾ Eº = -0.800 V
What is the value of log Ksp for AgI? (2.303 RT/ iii. HSO4– iv. HSO3F
F = 0.059 V) [2006] Which one of the following is the correct
(a) –37.83 (b) –16.13 sequence of their acid strength?
(c) –8.12 (d) +8.612 (a) iv < ii < iii < i (b) ii < iii < i < iv
26. The first and second dissociation constants of (c) i < iii < ii < iv (d) iii < i < iv < ii
an acid H2A are 1.0 × 10–5 and 5.0 × 10–10 respec- 32. The pKa of a weak acid, HA, is 4.80. The pKb of
tively. The overall dissociation constant of the a weak base, BOH, is 4.78. The pH of an aqueous
acid will be [2007] solution of the correspondng salt, BA, will be
(a) 0.2 × 105 (b) 5.0 × 10–5 [2008]
(c) 5.0 × 1015 (d) 5.0 × 10–15. (a) 9.58 (b) 4.79
27. The pKa of a weak acid (HA) is 4.5. The pOH of (c) 7.01 (d) 9.22
an aqueous buffer solution of HA in which 50% 33. Solid Ba(NO3)2 is gradually dissolved in a 1.0 ×
of the acid is ionized is [2007] 10– 4 M Na2CO3 solution. At what concentration
(a) 7.0 (b) 4.5 of Ba2+ will a precipitate begin to form? (KSP for
for BaCO3 = 5.1 × 10–9) [2009]
(c) 2.5 (d) 9.5
28. In a saturated solution of the sparingly soluble (a) 5.1 × 10 –5 M (b) 8.1 × 10 –8 M
strong electrolyte AgIO3 (molecular mass = 283) (c) 8.1 × 10 –7 M (d) 4.1 × 10 –5 M
ˆˆ†
the equilibrium which sets in is AgIO3 (s) ‡ˆˆ 34. Three reactions involving H2PO4– are given
below : [2010]
Ag + (aq) + IO3-(aq) . If the solubility product con-
(i) H3PO4 + H2O® H3O+ + H2PO4–
www.crackjee.xyz
Equilibrium C-35
(ii) H2PO4– + H2O ® HPO42– + H3O+ 1 1
(iii) H2PO4– + OH– ® H3PO4 + O2– NO( g) ® N2 ( g ) + O 2 ( g ) at the same
2 2
In which of the above does H 2 PO-4 act as an temperature is:
acid ? (a) 0.02 (b) 2.5 × 102
(a) (ii) only (b) (i) and (ii) (c) 4 × 10–4 (d) 50.0
(c) (iii) only (d) (i) only 41. The pH of a 0.1 molar solution of the acid HQ is
35. In aqueous solution the ionization constants for 3. The value of the ionization constant, Ka of the
carbonic acid are acid is : [2012]
K1 = 4.2 × 10–7 and K2 = 4.8 × 10–11. (a) 3 × 10–1 (b) 1 × 10–3
Select the correct statement for a saturated 0.034 (c) 1 × 10–5 (d) 1 × 10–7
M solution of the carbonic acid. [2010] 42. How many litres of water must be added to 1
2 -
(a) The concentration of CO3 is 0.034 M. litre an aqueous solution of HCl with a pH of 1
(b) The concentration of CO32 - is greater than to create an aqueous solution with pH of 2 ?
that of HCO3- . [2013]
(c) The concentrations of H+ and HCO3- are (a) 0.1 L (b) 0.9 L
approximately equal. (c) 2.0 L (d) 9.0 L
(d) The concentration of H+ is double that of
CO32 - .
1 ˆˆˆˆˆˆ
†
43. For the reaction SO 2(g ) + O 2(g ) ‡ˆˆˆˆˆSO 3(g ) , if
2
36. Solubility product of silver bromide is 5.0 × 10– x
13. The quantity of potassium bromide (molar K P = KC ( RT ) where the symbols have usual
mass taken as 120 g mol–1) to be added to 1 litre meaning then the value of x is (assuming
of 0.05 M solution of silver nitrate to start the ideality): [2014]
1
precipitation of AgBr is [2010] (a) –1 (b) -
2
(a) 1.2 × 10–10 g (b) 1.2 × 10–9 g 1
(c) 6.2 × 10–5 g (d) 5.0 × 10–8 g (c) (d) 1
2
37. At 25°C, the solubility product of Mg(OH)2 is
1.0 × 10–11. At which pH, will Mg2+ ions start 44. The standard Gibbs energy change at 300 K for
precipitating in the form of Mg(OH)2 from a the reaction 2A B + C is 2494.2 J. At a given time,
solution of 0.001 M Mg2+ ions? [2010] the composition of the reaction mixture is [A] =,
(a) 9 (b) 10 [B] = 2 and [C] = . The reaction proceeds in the :
(c) 11 (d) 8 [R = 8.314 J/K/mol, e = 2.718] [JEE M 2015]
38. An acid HA ionises as (a) forward direction because Q < Kc
ˆˆ† H + + A -1
HA ‡ˆˆ (b) reverse direction because Q < Kc
The pH of 1.0 M solution is 5. Its dissociation (c) forward direction because Q > Kc
constant would be : [2011RS] (d) reverse direction because Q > Kc
(a) 5 (b) 5 ´ 10 -8 45. The equilibrium constant at 298 K for a reaction
(c) 1 ´ 10-5 (d) 1 ´ 10-10 A +B C+D is 100. If the initial concentration of
39. The Ksp for Cr(OH)3 is 1.6 × 10–30. The solubility all the four species were 1 M each, then equilib-
of this compound in water is : [2011RS] rium concentration of D (in mol L–1) will be :
(a) 4 1.6 ´ 10 -30 [JEE M 2016]
(a) 1.818 (b) 1.182
(b) 4 1.6 ´ 10 -30 / 27 (c) 0.182 (d) 0.818
(c) 1.6 ´ 10-30/ 27
46. pKa of a weak acid (HA) and pKb of a weak base
(d) 2 1.6 ´ 10 -30 (BOH) are 3.2 and 3.4, respectively. The pH of
40. The equilibrium constant (Kc) for the reaction their salt (AB) solution is [JEE M 2017]
N2(g) + O2(g) ® 2NO(g) at temperature T is (a) 7.2 (b) 6.9
4 × 10–4. The value of Kc for the reaction [2012]
(c) 7.0 (d) 1.0
EBD_7764
www.crackjee.xyz
C-36 Chemistry
Answer Key
1 2 3 4 5 6 7 8 9 10 11 12 13 14 15
(a) (a) (a) (a) (c) (b) (d) (b) (b) (d) (d) (d) (c) (b) (d)
16 17 18 19 20 21 22 23 24 25 26 27 28 29 30
(b) (c) (a) (d) (a) (a) (a) (c) (c) (b) (d) (d) (c) (a) (c)
31 32 33 34 35 36 37 38 39 40 41 42 43 44 45
(c) (c) (a) (a) (i) (c) (b) (b) (d) (b) (d) (c) (d) (b) (d) (a)
46
(b)

\ pH = – log [H+] = –log 11×10–8


1. (a) NOTE A buffer is a solution of weak
= –[log11 + 8 log 10]
acid and its salt with strong base and vice
= –[1.0414 – 8] = 6.9586
versa.
HCl is strong acid and NaCl is its salt with 7. (d) ˆˆ† A∗2 ∗ 2B,
AB2 ‡ˆˆ
strong base. pH is less than 7 due to HCl. [A] = 1.0 × 10–5, [B] = [2.0 × 10–5] ,
2. (a) (HSO4)– can accept and donate a proton Ksp = [B]2 [A] = [2 × 10–5]2 [1.0 × 10–5] = 4 × 10–15
(HSO4)– + H+ ® H2SO4 (acting as base)
(HSO4)– – H+ ® SO42–. (acting as acid) [NO2 ]2 [1.2 ´ 10 -2 ]2
8. (b) Kc = =
3. (a) Mg(OH)2 ® [Mg2+] + 2[OH–] [N 2O 4 ] [4.8 ´ 10 -2 ]
x 2x = 3 ´ 10–3 mol/L
Ksp = [Mg] [OH]2 = [x][2x]2 = x.4x2 = 4x3. 9. (b) Due to exothermicity of reaction low or
4. (a) In this reaction the ratio of number of moles optimum temperature will be required. Since
of reactants to products is same i.e. 2 : 2, 3 moles are changing to 2 moles.
hence change in volume will not alter the \ High pressure will be required.
number of moles. 10. (d) The rain water after thunderstorm contains
dissolved acid and therefore the pH is less
5. (c) K p = K c (RT)Dn ;
than rain water without thunderstorm.
æ 1ö 3 1 11. (d) NOTE Conjugate acid-base differ by H+
Dn = 1 - ç 1 + ÷ = 1 - = - .
è 2ø 2 2 +
–H
Kp H 2 PO 4 HPO 4
-1/ 2
\ K = (RT) Acid conjugate
ugate base
c 12. (d) For P4 (s) + 5O 2 (g) P4O10 (s)
6. (b) pH of an acidic solution should be less than
7. The reason is that from H2O. [H+] = 10– 1
7M which cannot be neglected in Kc = . The solids have
(O 2 ) 5
comparison to 10 –8M. The pH can be
calculated as. concentration unity
from acid, [H+] = 10–8M. 13. (c) K p = K c (RT) Dn ;
from H2O, [H+] = 10–7M Here D n = 1– 2 = –1
\ Total [H+] = 10–8 + 10–7 Kp 1
= 10–8 (1 + 10) = 11× 10–8 \ =
K c RT
www.crackjee.xyz
Equilibrium C-37

[ NO]2 1
14. (b) Kc = = 4 ´10 - 4 5.4 = log
[ N 2 ][O 2 ] [H + ]
On solving, [H+] = 3.98 × 10–6
[N 2 ]1/ 2 [O 2 ]1/ 2 1 21. (a) Conjugate acid-base pair differ by only one
K ¢c = =
[NO] Kc proton.

1 OH - ¾
¾® H + + O 2 - Conjugate base
= = 50
4 ´ 10 -4 of OH– is O2–
22. (a) NH 4 HS(s ) NH 3(g ) + H 2 S(g )
15. (d) MX4 M 4+ + 4X -
S 4S start 0 .5 atm 0 atm
At equib. 0 .5 + x atm x atm.
Ksp = [s] [4s]4 = 256 s5
Then 0.5 + x + x = 2x + 0.5 = 0.84 (given)
1/ 5
æ Ksp ö Þ x = 0.17 atm.
\s = ç ÷
è 256 ø
p NH3 = 0.5 + 0.17 = 0.67 atm
+ -–
16. (b) Na 2SO 4 2Na + SO 4
2a
; p H 2S = 0.17 atm
1- a a
2
1 - a + 2a + a K= pNH 3 ´ pH 2S = 0.67 ´ 0.17 atm
Vant. Hoff’s factor i = = 1 + 2a
1 = 0.1139 = 0.11
17. (c)
MX 2 M + + + 2X -
s 2s 23. (c) PCl5(g) PCl3(g) + Cl2(g)
Where s is the solubility of MX2
1–x x x
then Ksp = 4s3; s × (2s)2 = 4×10–12
Total moles after dissociation
= 4s3; s = 1 × 10–4
1–x+x +x =1+x
\ [M++] = s = 1[M++] = 1.0 × 10– 4
18. (a) The reaction given is an exothermic reaction p = mole fraction of
PCl 3
thus accordingly to Lechatalier’s principle æ x ö
lowering of temperature, addition of F2 and PCl3 × Total pressure = ç ÷P
è1+ x ø
or Cl2 favour the for ward direction and
hence the production of ClF3 . 1
24. (c) SO3(g) SO 2 (g ) + O 2 (g)
19. (d) For the reaction:- 2

ˆˆ
†
2NO 2 (g) ‡ˆˆ 2NO(g) + O 2 (g) [SO 2 ] [O2 ]½
Kc = = 4.9 ´ 10-2 ;
Given Kc = 1.8 × 10–6 at 184 ºC [SO3 ]
R = 0.0831 kj/mol. k On taking the square of the above reaction
Kp= 1.8 × 10–6 × 0.0831 × 457 = 6.836 × 10–
6
[SO 2 ]2 [O 2 ]
[ Q 184°C = (273 + 184) = 457 k, = 24.01´10 - 4
[SO 3 ]2
Dn = (2 + 1, –1) = 1]
Hence it is clear that Kp > Kc now K'C for 2SO2(g) + O2(g) 2SO3
1
20. (a) pH = –log [H+] = log [SO 3 ]2 1
[H + ] = = = 416
2
[SO 2 ] [O 2 ] 24.01´10 - 4
EBD_7764
w w w . c r a c k
C-38 Chemistry
+ -
25. (b) (i) Ag ¾
¾® Ag + e E º = - 0.800 V \ s = K sp = 1´10-8
(ii) Ag + I - ¾
¾® AgI + e - E º = 0 .152 V = 1.0 × 10–4 mol/lit = 1.0 × 10–4 × 283 g/lit
From (i) and (ii) we have, (Q Molecular mass of Ag IO3 = 283)
¾® Ag + + I - E º = -0.952 V
AgI ¾
1.0 ´10-4 ´ 283 ´100
= gm /100ml
0.059 1000
Eocell = log K
n = 2.83 × 10–3 gm/ 100 ml
0.059 29. (a) Let the initial moles of X be ‘a’ and that of
\ –0.952 = log [Ag+] [I–] Z be ‘b’ then for the given reactions, we
1
[ Q k = [Ag+][I–]]
X
  2Y
0.952 have  
or - = log K sp or –16.13 = log
0.059 Initial a moles 0
Ksp At equi. a(1 – a) 2aa
(moles)
26. (d) ˆˆ† H∗ ∗ HA,
H 2 A ‡ˆˆ
Total no. of moles = a (1 – a) + 2aa
+ - = a – aa + 2aa
\ K = 1.0 × 10–5 = [H ][HA ] (Given) = a (1 + a)
1
[H 2 A] Dn
(n y ) 2 æ PT ö
HA- ¾¾
® H+ + A-- Now, K P1 = ´ç 1 ÷
nx ç ån ÷
è ø
[H + ][A -- ] (Given)
\ K 2 = 5.0 ´ 10 -10 = (2aa)2 .PT1
[HA - ] or, K P1 =
[a(1 - a)][a(1 + a)]
[H + ]2 [A 2 - ]
K= = K1 ´ K2
[H 2 A] 
Z P + Q
= (1.0 × 10–5) × (5 × 10–10) = 5 × 10–15 

é salt ù Initial b moles 0 0


27. (d) For acidic buffer pH = pKa + log ê ú At equi. b(1 – a) ba ba
ë acid û
(moles)
éA- ù Total no . of moles = b(1 – a) + ba + ba
pH = pK + log ë û
or a
[ HA ] = b – ba + ba + ba
= b(1 + a)
Given pKa = 4.5 and acid is 50% ionised.
[HA] = [A–] (when acid is 50% ionised) Dn
nQ ´ nP é PT ù
\ pH = pKa + log 1 Now K P = ´ê 2 ú
ë ån û
2 nz
\ pH = pKa = 4.5
pOH = 14 – pH = 14 – 4.5 = 9.5 (ba)(ba).PT2
28. (c) Let s = solubility or K P2 =
[b(1 - a)][b(1 + a)]
ˆˆ† Ag ∗ ∗ IO3,
AgIO3 ‡ˆˆ
s s
K P1 4a 2 .PT1 (1 - a )2 4PT1
Ksp = [Ag+] [IO3–] = s × s = s2 = ´ =
or
Given Ksp = 1 × 10–8 K P2 (1 - a 2 ) PT2 .a 2 PT2
www.crackjee.xyz
Equilibrium C-39

PT1 1 é K P1 1 ù [HCO3- ][H3O + ] x´ x


= êQ = given ú K1 = =
or [H 2CO3 ] 0.034 - x
PT2 9 êë K P2 9 úû
PT1
1
= x2
or PT2 36 or 1 : 36 Þ 4.2 ≥10,7 ; Þ x < 1.195≥10,4
0.034
i.e., (a) is the correct answer.
As H 2 CO 3 is a weak acid so the
30. (c) Reaction (c) can be obtained by adding
concentration of
reactions (a) and (b) therefore K3 = K1. K2
H2CO3 will remain 0.034 as 0.034 >> x.
Hence (c) is the correct answer.
31. (c) The correct order of acidic strength of the x = [H+] = [ HCO3- ] = 1.195 × 10–4
given species in
Now, HCO3– (aq ) ∗ H 2O(l ) ƒ CO32– ( aq ) ∗ H3O∗ ( aq )
HSO3 F > H 3 O + > HSO 4 - > HCO3- x, y y y
(iv) (ii) (iii) (i)
As HCO3- is again a weak acid (weaker
or (i) < (iii) < (ii) < (iv)
It corresponds to choice (c) which is correct than H2CO3) with x >> y.
answer. [CO32– ][H3O+ ] y ´ (x + y)
32. (c) In aqueous solution BA(salt) hydrolyses K2 = =
[HCO3– ] ( x - y)
to give + +
Note : [H3O ] = H from first step (x) and
BA + H2O BOH + HA
from second step (y) = (x + y)
Base acid [As x > > y so x + y ; x and x – y ; x]
Now pH is given by
y≥ x
1 1 1 So, K 2 ; =y
pH = pKw + pKa - pKb x
2 2 2
substituting given values, we get Þ K 2 = 4.8 ´ 10-11 = y = [CO32 - ]
1 So the concentration of [H+] ; [ HCO3– ]=
pH = (14 + 4.80 - 4.78) = 7.01
2
concentrations obtained from the first
33. (a) Na 2 CO3 ¾¾
® 2Na + + CO32- step. As the dissociation will be very low
1 ´10-4 M 1´10-4 M 1´10-4 M
in second step so there will be no change
in these concentrations.
K SP(BaCO3 ) = [Ba 2+ ][CO 32– ] Thus the final concentrations are
5.1 ´ 10 -9 [H + ] = [ HCO3- ] = 1.195 × 10 –4 &
[Ba 2+ ] = = 5.1 ´ 10 -5 M
1 ´ 10 -4 [CO23 - ] = 4.8 ´ 10 -11
34. ® H3O + + H 2 PO4-
(a) (i) H3PO4 + H 2O4 ¾¾
acid1 base2 acid 2 base1 36. (b) ˆˆ† Ag∗ ∗ Br,
AgBr ‡ˆˆ
Ksp = [Ag+] [Br–]
-
® HPO4-- + H 3O +
(ii) H 2PO4 + H 2O ¾¾ For precipitation to occur
acid1 base 2 base1 acid2 Ionic product > Solubility product
H 2PO 4- + OH - ¾¾
® H3PO4 + O -- K sp 5≥10,13
(iii) acid 2 base2 [Br, ] < < < 10,11
[Ag∗ ]
base1 acid1
0.05
Hence only in (ii) reaction H2PO4– is acting
as an acid. i.e., precipitation just starts when 10–11
moles of KBr is added to 1l AgNO 3
35. (c) H 2 CO3 (aq )∗ H2 O(l ) ƒ HCO3– (aq) ∗ H3O+ (aq)
0.034, x x x
solution
\ Number of moles of Br – needed from
EBD_7764
www.crackjee.xyz
C-40 Chemistry
KBr = 10–11 M2 = 0.01 V2 = ?
\ Mass of KBr = 10–11 × 120 = 1.2 × 10–9 g From
M1V1 = M2V2
37. (b) Mg(OH)2 ‡ˆˆˆˆ† Mg∗∗ ∗ 2OH, 0.1 × 1 = 0.01 × V2
Ksp = [Mg++][OH–]2 V2 = 10 litres
1.0 × 10–11 = 10–3 × [OH–]2 \ Volume of water added = 10 – 1 = 9 litres
1
10,11 ˆˆ† SO3 (g)
SO2 (g) ∗ O2 (g) ‡ˆˆ
[OH, ] < < 10,4
43. (b)
2
10,3
\ pOH = 4 K P = KC ( RT ) x
\ pH + pOH = 14 where x = Dng = number of gaseous
\ pH = 10
38. (d) pH = 5 means moles in product
[H+] = 10–5 – number of gaseous moles in reactant
ˆˆ† H + + A -1
HA ‡ˆˆ æ 1ö 3 1
= 1 - ç1 + ÷ = 1 - = -
t=0 c 0 0 è 2ø 2 2
teq c (1 – a) ca ca 44. (d) DG° = 2494.2J
2A B + C.
[H+ ][A- ] (ca)2 [H+ ]2 R = 8.314 J/K/mol.
Ka = = = e = 2.718
[HA] c (1– a) c - [H+ ] [A] = , [B] = 2, [C] =
But, [H+] << C Q= =4
\ Ka = (10–5)2 = 10–10 DG° = – 2.303 RT log KC.
2494.2 J = – 2.303 × (8.314 J/K/mol) × (300K)
39. (b) ˆˆ† Cr 3+ (aq.) + 3OH - (aq.)
Cr(OH)3 (s) ‡ˆˆ logKC
Þ log KC = –
27 S 4 = K sp Þ log KC = – 0.4341
1/ 4 1/ 4 KC = 0.37
æ K sp ö æ 1.6 ´ 10-30 ö Q > KC.
S =ç ÷ =ç ÷
è 27 ø 27 45. (a) Given,
è ø
A + B C + D
40. (d) For the reaction No. of moles initially 1 1 1 1
N2 + O2 ¾¾ ® 2NO K = 4 × 10–4 At equilibrium 1–a 1–a 1+a
Hence for the reaction 1+a
\ Kc = = 100
1 1 \ = 10
NO ¾¾
® N2 + O2
2 2 On solving
1 1 a = 0.81
K'= = = 50 [D]At eq = 1 + a = 1 + 0.81 = 1.81
K 4×10 – 4 46. (b) Given
pKa (HA) = 3.2
[H + ]
41. (c) H+ = C a ; a = pKb (BOH) = 3.4
C The salt (AB) given is a salt is of weak acid
and weak base. Hence the pH can be
10 –3
or a = = 10-2 calculated by the formula
0.1 1 1
\ pH = 7 + pK a – pK b
Ka = C a 2 = 0.1´ 10-2 ´10-2 = 10–5 2 2
1 1
42. (d) Q pH = 1 ; H+ = 10–1 = 0.1 M = 7 + (3.2) – (3.4)
2 2
pH = 2 ; H+ = 10–2 = 0.01 M = 6.9
\ M1 = 0.1 V1 = 1
www.crackjee.xyz

8
Redox Reactions

1. Which of the following is a redox reaction?


Z
[2002] X Mn 2+ + 2Y CO 2 + H2 O
(a) NaCl + KNO3 ® NaNO3 + KCl 2
(b) CaC2O4 + 2HCl ® CaCl2 + H2C2O4 values of X, Y and Z respectively are
(c) Mg(OH)2 + 2NH4Cl ® MgCl2 +2NH4OH [Online May 12, 2012; 2013 Offline]
(d) Zn + 2AgCN ® 2Ag + Zn(CN)2. (a) 2, 5, 16 (b) 8, 2, 5
2. Several blocks of magnesium are fixed to the (c) 5, 2, 16 (d) 5, 8, 4
bottom of a ship to [2003] 5. The molecular formula of a commercial resin used
(a) make the ship lighter for exchanging ions in water softening is
(b) prevent action of water and salt C8H7SO3– Na+ (Mol. wt. 206. What would be
(c) prevent puncturing by under-sea rocks the maximum uptake of Ca2 + ions by the resin
(d) keep away the sharks when expressed in mole per gram resin?
3. Which of the following chemical reactions depict [JEE M 2015]
the oxidizing beahviour of H2SO4?
2 1
[2006] (a) (b)
(a) NaCl + H 2 SO 4 ¾¾® NaHSO 4 + HCl 309 412
(b) 2PCl5 + H 2SO 4 ¾¾ ® 1 1
(c) (d)
2POCl3 + 2HCl + SO 2 Cl2 103 206
(c) 2 HI + H 2 SO 4 ¾
¾® I 2 + SO 2 + 2 H 2 O 6. Which of the following reactions is an example
of a redox reaction? [JEE M 2017]
(d) Ca(OH) 2 + H 2SO 4 ¾¾ ®
(a) XeF4 + O2F2 ® XeF6 + O2
CaSO 4 + 2H 2 O (b) XeF2 + PF5 ® [XeF]+ PF6–
4. In the following balanced reaction, (c) XeF6 + H2O ® XeOF4 + 2HF
X MnO4- + Y C2 O24 - + Z H + (d) XeF6 + 2H2O ® XeO2F2 + 4HF

Answer Key
1 2 3 4 5 6
(d) (b) (c) (a) (b) (a)

2. (b) Magnesium provides cathodic protection


0 +1 –2e - 0 +2 and prevent rusting or corrosion.
1. (d) Zn + 2Ag CN ¾ ¾ ¾® Ag + Zn (CN)2
+ 2e - +6 +4
3. (c) 2HI -1 + H 2SO 4 ¾¾® I 02 + SO2 + 2H 2 O
The oxidation state shows a change only
in this reaction oxidation number of S is
in (d)
EBD_7764
www.crackjee.xyz
C-42 Chemistry
decreasing from + 6 to +4 hence undergoing On balancing
reduction and for HI oxidation Number of I
is increasing from –1 to 0 hence C2 O 4– – ¾¾
® 2CO 2 + 2e – .... (iv)
underegoing oxidation therefore H2SO4 is On multiplying eqn. (ii) by 5 and (iv) by
acting as oxidising agent. 2 and then adding we get
4. (a) X MnO 4– + Y C2 O 2–
4 +ZH
+ 2MnO 4– + 5C2 O4– – + 16H + ¾¾®
Z 2Mn ++ + 10CO2 + 8H 2 O
X Mn 2 + + 2Y CO 2 + H 2O
2 5. (b) 2 mole of water softner require 1 mole of
First half reaction Ca2+ ion
So, 1 mole of water softner require mole of
® Mn + +
MnO4– ¾¾ .... (i) Ca2+ ion
On balancing Thus, will be maximum uptake
6. (a) In the reaction
MnO4– + 8H + + 5e – ¾¾
® Mn ++ + 4H 2O
Oxidation
.... (ii) +4 +1 +6 0
Second half reaction XeF4 + O2F2 ® XeF6 + O2
Reduction
C2 O 4– – ¾¾
® 2CO 2 .... (iii)
www.crackjee.xyz
Hydrogen C-43

9
Hydrogen

1. Which of the following species is diamagnetic 5. In which of the following reactions H2O2 acts
in nature? as a reducing agent? [2014]
(a) H -2 (b) H +2 [2005] (a) H 2 O 2 + 2H + + 2e - ® 2H 2 O
+
(c) H 2 (d) He 2 (b) H 2 O 2 + 2e - ® O2 + 2H +
2. Which of the following statements in relation to (c) H 2 O 2 + 2e - ® 2OH -
the hydrogen atom is correct ? [2005]
(d) H 2 O 2 + 2OH - - 2e - ® O2 + 2H 2 O
(a) 3s, 3p and 3d orbitals all have the same
energy 6. From the following statements regarding H2O2,
(b) 3s and 3p orbitals are of lower energy than choose the incorrect statement : [JEE M 2015]
3d orbital (a) It has to be stored in plastic or wax lined
(c) 3p orbital is lower in energy than 3d orbital glass bottles in dark
(d) 3s orbital is lower in energy than 3p orbital (b) It has to be kept away from dust
3. In context with the industrial preparation of (c) It can act only as an oxidizing agent
hydrogen from water gas (CO + H2), which of (d) It decomposes on exposure to light
the following is the correct statement? [2008] 7. Which one of the following statements about
(a) CO and H2, are fractionally separated using water is FALSE ? [JEE M 2016]
differences in their densities (a) There is extensive intramolecular hydrogen
(b) CO is removed by absorption in aqueous bonding in the condensed phase.
Cu2Cl2 solution (b) Ice formed by heavy water sinks in normal
(c) H2 is removed through occlusion with pd water.
(c) Water is oxidized to oxygen during
(d) CO is oxidised to CO2 with steam in the
photosynthesis.
presen ce of a catalyst followed by
(d) Water can act both as an acid and as a
absorption of of CO2 in alkali
base.
4. Very pure hydrogen (99.9) can be made by which 8. The concentration of fluoride, lead, nitrate and
of the following processes ? [2012] iron in a water sample from an underground lake
(a) Reaction of methane with steam was found to be 1000 ppb, 40 ppb, 100 ppm and
(b) Mixing natural hydrocarbons of high 0.2 ppm, respectively. This water is unsuitable
molecular weight for drinking due to high concentration of :
(c) Electrolysis of water [JEE M 2016]
(d) Reaction of salts like hydrides with water (a) Nitrate (b) Iron
(c) Fluoride (d) Lead

Answer Key
1 2 3 4 5 6 7 8
(c) (a) (d) (d) (d) (c) (a) (a)
EBD_7764
www.crackjee.xyz
C-44 Chemistry

1. (c) 4. (d) Very pure hydrogen can be prepared by


the action of water on sodium hydride.
A diamagnetic substance contains no
unpaired electron. NaH + H2O ¾¾ ® NaOH + H2
H2 is diamagnetic as it contains all paired (very pure Hydrogen)
electrons 5. (d) The reducing agent loses electron during
redox reaction i.e. oxidises itself.
H 2 = sb2 , H 2+ = s1b , H 2- = sb2 , s*1 +
a ; He 2 = sb2 , s*1
a
(diamagnetic) (paramagnetic) (paramagnetic) (paramagnetic) -1
(paramagnetic) -2
(a) H 2O 2 + 2H + + 2e - ¾¾
® 2H 2 O (Re d.)
= sb2 , H +2 = s1b , H 2- = sb2 , s*1 +
a ; He2 = sb2 , s*1
a
magnetic) (paramagnetic) (paramagnetic) (paramagnetic) (paramagnetic) -1 0
(b) ® O 2 + 2H + + 2e- (Ox.)
H 2O 2 ¾¾
2. (a) NOTE In one electron species, such as
H-atom, the energy of orbital depends only -1 -2
on the principal quantum number, n. (c) H 2O 2 + 2e - ¾¾
® 2 OH - (Re d.)
Hence answer (d)
i.e. is < 2s = 2p < 3s = 3p = 3d < 4s 0
= 4p Þ 4d = 4g (d) H 2O 2-1 + 2OH - ¾¾
® O 2 + H 2O + 2e- (Ox.)
3. (d) On the industrial scale hydrogen is
prepared from water gas according to 6. (c) has oxidizing and reducing properties both.
following reaction sequence 7. (a) There is extensive intermolecular
catalyst hydrogen bonding in the condensed
CO + H 2 + H 2 O ¾¾¾¾
® CO2 + 2H 2
1424 3 { phase instead of intramolecular
water gas (steam) H-bonding.
2NaOH
8. (a) The maximum limit of nitrate in drinking
¾¾¾¾
® Na 2 CO3 + H 2 O water is 50 ppm. Excess nitrate in drinking
(alkali)
water can cause disease such as
From the above it is clear that CO is first
methemoglobinemia (‘blue baby’
oxidised to CO2 which is then absorbed in
syndrome).
NaOH.
www.crackjee.xyz
The s-Block Elements C-45

10
The s-Block Elements

1. KO2 (potassium super oxide) is used in oxygen (a) hydration energies of cations
cylinders in space and submarines because it (b) inter-ionic attraction
[2002] (c) entropy of solution formation
(a) absorbs CO2 and increases O2 content (d) lattice energies of solids
(b) eliminates moisture 7. Which one of the following processes will
(c) absorbs CO2 produce hard water ? [2003]
(d) produces ozone. (a) Saturation of water with MgCO3
2. The metallic sodium disolves in liquid ammonia (b) Saturation of water with CaSO4
to form a deep blue coloured solution. The deep
(c) Addition of Na2SO4 to water
blue colour is due to formation of: [2002]
(d) Saturation of water with CaCO3
(a) solvated electron, e(NH3 )-x
8. One mole of magnesium nitride on the reaction
(b) solvated atomic sodium, Na(NH3)y
with an excess of water gives : [2004]
(c) (Na+ + Na–)
(a) two moles of ammonia
(d) NaNH2 + H2
(b) one mole of nitric acid
3. A metal M readily forms its sulphate MSO4 which
(c) one mole of ammonia
is water-soluble. It forms its oxide MO which
becomes inert on heating. It forms an insoluble (d) two moles of nitric acid
hyroxide M(OH)2 which is soluble in NaOH 9. Based on lattice energy and other
solution. Then M is [2002] considerations which one of the following alkali
(a) Mg (b) Ba metal chlorides is expected to have the highest
(c) Ca (d) Be. melting point ? [2005]
4. In curing cement plasters water is sprinkled from (a) RbCl (b) KCl
time to time. This helps in [2003] (c) NaCl (d) LiCl
(a) developing interlocking needle-like crystals 10. The ionic mobility of alkali metal ions in aqueous
of hydrated silicates solution is maximum for [2006]
(b) hydrating sand and gravel mixed with (a) Li + (b) Na +

cement (c) K+ (d) Rb+


(c) converting sand into silicic acid 11. The products obtained on heating LiNO2 will
(d) keeping it cool be : [2011RS]
5. The substance not likely to contain CaCO3 is
(a) Li 2 O + NO2 + O2 (b) Li 3 N + O 2
(a) calcined gypsum (b) sea shells [2003]
(c) dolomite (d) a marble statue (c) Li 2 O + NO + O 2 (d) LiNO3 + O2
6. The solubilities of carbonates decrease down 12. What is the best description of the change that
the magnesium group due to a decrease in occurs when Na2O(s) is dissolved in water ?
[2003] [2011RS]
EBD_7764
www.crackjee.xyz
C-46 Chemistry
(a) Oxide ion accepts sharing in a pair of 16. Which one of the following alkaline earth metal
electrons sulphates has its hydration enthalpy greater
(b) Oxide ion donates a pair of electrons than its lattice enthalpy ? [JEE M 2015]
(c) Oxidation number of oxygen increases (a) BaSO4 (b) SrSO4
(d) Oxidation number of sodium decreases (c) CaSO4 (d) BeSO4
13. Which of the following on thermal 17. The main oxides formed on combustion of Li,
decomposition yields a basic as well as acidic Na and K in excess of air are, respectively:
oxide ? [2012] [JEE M 2016]
(a) NaNO3 (b) KClO3 (a) Li2O2, Na2O2 and KO2
(c) CaCO3 (d) NH4NO3 (b) Li2O, Na2O2 and KO2
14. The first ionisation potential of Na is 5.1 eV. The (c) Li2O, Na2O and KO2
value of electron gain enthalpy of Na+ will be
(d) LiO2, Na2O2 and K2O
[2013]
(a) – 2.55 eV (b) – 5.1 eV 18. Both lithium and magnesium display several
similar properties due to the diagonal
(c) – 10.2 eV (d) + 2.55 eV
relationship; however, the one which is incorrect
- is : [JEE M 2017]
15. Stability of the species Li 2 , Li 2 and
(a) Both form basic carbonates
Li +2 increases in the order of : [2013]
(b) Both form soluble bicarbonates
(a) Li 2 < Li 2+ < Li 2- (b) Li -2 < Li +2 < Li 2 (c) Both form nitrides
(d) Nitrates of both Li and Mg yield NO2 and
(c) Li 2 < Li 2- < Li 2+ (d) Li -2 < Li 2 < Li 2+
O2 on heating

Answer Key
1 2 3 4 5 6 7 8 9 10 11 12 13 14 15
(a) (a) (d) (a) (a) (a) (b) (a) (c) (d) (a) (b) (c) (b) (b)
16 17 18
(d) (b) (a)

1. (a) 2KO2 + 2 H2O ® 2 KOH + H2O2 + O2 . It is ammoniated electron which is


KO2 is used as an oxidising agent. It is used responsible for colour.
as air purifier in space capsules. Submarines 3. (d) Sulphate of alkaline earth metal are
and breathing masks as it produces oxygen sparingly soluble or almost not soluble in
and remove carbon dioxide. water whereas BeSO4 is soluble in water
2. (a) The alkali metals dissolve in liquid ammonia due to high degree of solvation. Be(OH)2
without evolution of hydrogen. The metal is insoluble in water but soluble in NaOH.
loses electrons and combine with ammonia BeO + 2NaOH––––––––® Na2BeO2 + H2O
molecule. 4. (a) Setting of cement is exothermic process
M––® M+ (in liquid ammonia) which develops interlocking crystals of
+ e– (ammoniated) hydrated silicates
M + (x + y) NH3 ®[M(NH3)x]+ + e–(NH3)y 5. (a) Gypsum is CaSO4.2H2O
Solvated electron
www.crackjee.xyz
The s-Block Elements C-47
6. (a) As we move down the group, the lattice
energies of car bonates remain ® Na + + e -
14. (b) Q For Na ¾¾ IE1 = 5.1 eV
approximately the same. However the
\ For Na + + e - ¾¾
® Na EF = -5.1 eV
hydration energies of the metal cation
decreases from Be++ to Ba++, hence the (because the reaction is reverse)
solubilities of carbonates of the alkaline 15. (b) Li2 = s1s2 s*1s2 s2s2
earth metal decrease down the group 1
mainly due to decreasing hydration \ Bond order = (4 - 2) = 1
2
energies of the cations from Be++ to Ba++. Li2+ = s1s2 s*1s2 s2s1
7. (b) Permanent hardness of water is due to
chlorides and sulphates of calcium and 1
B.O. = (3 - 2) = 0.5
magnesium i.e CaCl2, CaSO4, MgCl2 and 2
MgSO4. Li2 = s1s2 s*1s2s2s2s*2s1

8. (a) Mg3N2+ 6H2O 3Mg(OH ) 2 + 2 NH 3 1


B.O. = (4 - 3) = 0.5
9. (c) LiCl has partly covalent character. Other 2
halides are ionic in nature. Lattice energy The bond order of Li2+ and Li2– is same but
decreases with increase of ionic radius of Li2+ is more stable than Li2– because Li2+ is
cation, anion being the same. Larger is the smaller in size and has 2 electrons in
lattice energy, the higher will be m. pt. hence antibonding orbitals whereas Li2– has 3
NaCl will have highest lattice energy. electrons in antibonding orbitals. Hence Li2+
10. (d) Smaller the size of cation higher is its is more stable than Li2–.
hydration energy and greater is its ionic 16. (d) In alkaline earth metals, ionic size increases
mobility hence the correct order is down the group. The lattice energy remains
Li+ < Na+ < K+ < Rb+ constant because sulphate ion is so large, so
that small change in cationic size does not make
11. (a) 4LiNO3 ® 2Li 2 O + 4NO2 + O2 any difference. On moving down the group the
degree of hydration of metal ions decreases very
12. (b) O 2 - (base) + H 2 O(acid) ® OH - (C.B.) +
much leading to decrease in solubility.
OH - (C.A.) 17. (b) On heating with excess of air Li, Na and K forms
following oxides
O2– acts as Lewis base. 4 Li + O2 2Li2O Lithium monoxide
13. (c) Calcium carbonate on thermal 2Na + O2 Na2O2 Sodium peroxide
decomposition gives CaO (Basic oxide) and K + O2 KO2 Potassium superoxide
CO2 (Acidic oxide) 18. (a) Mg can form basic carbonate like

CaCO 3
D
CaO + CO 2­ 3MgCO3 . Mg(OH)2 . 3H2O ¯
Basic oxide Acidic oxide While Li can form only carbonate (Li2CO3)
not basic carbonate.
EBD_7764
www.crackjee.xyz
C-48 Chemistry

11
The p-Block Elements
(Group-13 and 14)
1. Alum helps in purifying water by [2002] (a) forming covalent halides
(a) forming Si complex with clay partiles (b) forming polymeric hydrides
(b) sulphate part which combines with the dirt (c) exhibiting maximum covalency in compounds
and removes it (d) exhibiting amphoteric nature in their oxides
(c) coagulaing the mud particles 6. Aluminium chloride exists as dimer, Al2Cl6 in
(d) making mud water soluble. solid state as well as in solution of non-polar
2. Graphite is a soft solid lubricant extremely solvents such as benzene. When dissolved in
difficult to melt. The reason for this anomalous water, it gives [2004]
behaviour is that graphite [2003]
(a) [Al(OH) 6 ]3- + 3HCl
(a) is an allotropic form of diamond
(b) has molecules of variable molecular masses (b) [Al(H 2 O) 6 ]3+ + 3Cl -
like polymers
(c) has carbon atoms arranged in large plates (c) Al 3+ + 3Cl -
of rings of strongly bound carbon atoms
(d) Al 2 O3 + 6HCl
with weak interplate bonds
(d) is a non-crystalline substance 7. Heating an aqueous solution of aluminium
chloride to dryness will give [2005]
3. Glass is a [2003]
(a) super-cooled liquid (a) Al(OH)Cl 2 (b) Al 2O 3
(b) gel
(c) Al 2 Cl 6 (d) AlCl3
(c) polymeric mixture
(d) micro-crystalline solid 8. In silicon dioxide [2005]
4. For making good quality mirrors, plates of float (a) there are double bonds between silicon and
glass are used. These are obtained by floating oxygen atoms
molten glass over a liquid metal which does not (b) silicon atom is bonded to two oxygen
solidify before glass. The metal used can be atoms
[2003] (c) each silicon atom is surrounded by two
(a) tin (b) sodium oxygen atoms and each oxygen atom is
(c) magnesium (d) mercury bonded to two silicon atoms
5. Beryllium and aluminium exhibit many properties (d) each silicon atom is surrounded by four
which are similar. But, the two elements differ in
oxygen atoms and each oxygen atom is
[2004]
bonded to two silicon atoms.
www.crackjee.xyz
The p-Block Elements (Group-13 and 14) C-49

9. The structure of diborane ( B2 H 6 ) contains 11. Among the following substituted silanes the
one which will give rise to cross linked silicone
[2005]
polymer on hydrolysis is [2008]
(a) four 2c-2e bonds and four 3c-2e bonds
(a) R4Si (b) R2SiCl2
(b) two 2c-2e bonds and two 3c-3e bonds (c) RSiCl3 (d) R3SiCl
(c) two 2c-2e bonds and four 3c-2e bonds
12. In view of the sings of D r G ° for the following
(d) four 2c-2e bonds and two 3c-2e bonds
reactions :
10. Which one of the following is the correct
statement? [2008] PbO 2 + Pb ® 2PbO, DrG° < 0

(a) Boric acid is a protonic acid SnO 2 + Sn ® 2SnO, D r G° > 0


(b) Beryllium exhibits coordination number of
which oxidation states are more characteristics
six
for lead and tin ? [2011RS]
(c) Chlorides of both beryllium and aluminium
(a) For lead + 2, for tin + 2
have bridged chloride structures in solid
(b) For lead + 4, for tin + 4
phase
(c) For lead + 2, for tin + 4
(d) B2 H 6 .2NH 3 is known as ‘inorganic
(d) For lead + 4, for tin + 2
benzene’

Answer Key
1 2 3 4 5 6 7 8 9 10 11 12
(c) (c) (a) (d) (c) (b) (b) (d) (d) (c) (c) (c)

1. (c) Alum furnishes Al3+ ions which bring about 3. (a) Glass is a translucent or transparent
coagulation of negatively charged clay amorphous supercooled solid solution or
particles, bacteria etc. we can say super cooled liquid of silicates
2. (c) In graphite, carbon is sp2 hybridized. Each and borats having a general formula R2O.
carbon is thus linked to three other carbon MO . 6 SiO2. where R = Na or K and M = Ca,
atoms forming hexagonal rings. Since only Ba, Zn or Pb.
three electrons of each carbon are used in 4. (d) It is mercury because it exists as liquid at
making hexagonal ring, fourth electron of room temperature.
each carbon is free to move. This makes 5. (c) The maximum valency of beryllium is +2
graphite a good conductors of heat and while that of aluminium is +3.
electricity.
Further graphite has a two dimensional sheet 6. ˆˆ†
(b) Al2Cl6 + 12H2O ‡ˆˆ
like structure. These various sheets are held 2[Al(H2O)6]3+ + 6 C1-
together by weak van der Waal’s force of
7. (b) The solution of aluminium chloride in water
attraction. due to these weak forces of
is acidic due to hydrolysis.
attraction, one layer can slip over the other.
AlCl3 + 3H2O –––––® Al(OH)3 + 3HCl.
This makes graphite soft and a good
On heating it till dryness Al(OH)3 is
lubricating agent.
converted into Al2O3
EBD_7764
www.crackjee.xyz
C-50 Chemistry
acid so (a) is incorrect statement.
Al(OH)3 ¾¾ D Al O + 3H O
® 2 3 2 The coordination number exhibited by
8. (d) In SiO2 (quartz), each of O-atom is shared beryllium is 4 and not 6 so statement (b) is
between two SiO44– tetrahedra. incorrect.
Both BeCl 2 and AlCl 3 exhibit bridged
structures in solid state so (c) is correct
statement.
11. (c) The cross linked polymers will be formed
by RSiCl3
OH
|
3nH 2O
9. (d) In diborane structure B2H6 there are two nRSiCl3 ¾¾¾¾ ® nR - Si - OH ¾¾
®
-3nHCl |
2c-2e bonds and two 3c–2e bonds (see
OH
structure of diborane).
Structure of B 2H6 :
| |
O O
Hb | |
R - Si - O - Si
Ht •• Ht | |
O O
B B | |
R - Si - O - Si - R
| |
Ht •• Ht
O O
Hb | |
(Cross linked polymer)
10. (c) The correct formula of inorganic benzene
is B3N3H6 so (d) is incorrect statement
12. (c) Negative D r G° value indicates that + 2
OH oxidation state is more stable for Pb2+. Also
|
Boric acid (H3BO3 or B - OH ) is a lewis it is supported by inert pair effect that + 2
| oxidation state is more stable for Pb and +
OH 4 oxidation state is more stable for Sn.
i.e. Sn++ < Pb++, Sn4+ > Pb4+
www.crackjee.xyz
Organic Chemistry-Some Basic Principles & Techniques C-51

12
Organic Chemistry-Some
Basic Principles & Techniques
1. Arrangement of (CH3)3 – C –, (CH3)2 – CH –, 2-methyl-3-butanol
CH3 – CH2 – when attached to benzyl or an
O
unsaturated group in increasing order of ||
inductive effect is [2002] (d) CH 3 – C H – C – CH 2 – CH 3 ® 2-methyl-
|
(a) (CH3)3 –C – < (CH3)2 – CH – < CH3 – CH2 – CH3
(b) CH3 –CH2– < (CH3)2– CH – < (CH3)3 – C – 3-pentanone
(c) (CH3)2 – CH– <(CH3)3 –C –< CH3 –CH2 – 6. The IUPAC name of CH3COCH(CH3)2 is[2003]
(d) (CH3)3 – C– < CH3 –CH2 –<(CH3)2 –CH – (a) 2-methyl-3-butanone
2. A similarity between optical and geometrical (b) 4-methylisopropyl ketone
isomerism is that [2002] (c) 3-methyl-2-butanone
(a) each forms equal number of isomers for a (d) Isopropylmethyl ketone
given compound 7. In which of the following species is the
(b) if in a compound one is present then so is underlined carbon having sp3 hybridisation?
the other [2002]
(c) both are included in stereoisomerism (a) CH3 COOH (b) CH3 CH2 OH
(d) they have no similarity. (c) CH3 COCH3 (d) CH2 = CH –CH3
3. Which of th e following does n ot show 8. Racemic mixture is formed by mixing two [2002]
geometrical isomerism? [2002] (a) isomeric compounds
(a) 1,2-dichloro-1-pentene (b) chiral compounds
(b) 1,3-dichloro-2-pentene (c) meso compounds
(c) 1,1-dichloro-1-pentene (d) enantiomers with chiral carbon.
(d) 1,4-dichloro-2-pentene 9. Following types of compounds (as I, II) [2002]
4. The functional group, which is found in amino CH3 CH === CHCH3 CH3 CHOH
acid is [2002] |
(a) – COOH group (b) – NH2 group CH 2CH3
(c) – CH3 group (d) both (a) and (b). are studied in terms of isomerism in:
5. Which of the following compounds has wrong (a) chain isomerism
IUPAC name? [2002] (b) position isomerism
(c) conformers
(a) CH3–CH2–CH2 –COO–CH2CH3 ® ethyl
(d) stereoisomerism
butanoate
10. The reaction: [2002]
(b) CH 3 - C H - CH 2 - CHO ® H 2O
| (CH3)3C – Br ¾¾¾® (CH3)3 – C –OH
CH (a) elimination reaction
3-methyl-butanal (b) substitution reaction
(c) free radical reaction
(c) CH 3 - C H - C H - CH 3 ®
| | (d) displacement reaction.
OH CH 3
EBD_7764
www.crackjee.xyz
C-52 Chemistry
11. In the anion HCOO– the two carbon-oxygen 17. Amongst the following compounds, th e
bonds are found to be of equal length. what is optically active alkane having lowest molecular
the reason for it ? [2003] mass is [2004]
(a) The C = O bond is weaker than the C — O
bond H
|
(b) The anion HCOO– has two resonating (a) CH 3 - C - <
structures |
(c) The anion is obtained by removal of a C 2 H5
proton from the acid molecule CH3
(d) Electronic orbitals of carbon atom are |
hybridised (b) CH 3 - CH 2 - C H - CH 3
12. The general formula CnH2nO2 could be for open
(c) CH 3 - CH 2 - CH 2 - CH 3
chain [2003]
(a) carboxylic acids (b) diols (d) CH 3 - CH 2 - C º CH
(c) dialdehydes (d) diketones 18. Consider the acidity of the carboxylic acids :
13. Among the following four structures I to IV,
(a) PhCOOH
[2003]
(b) o-NO2C6H4COOH
CH3 O CH3
| || | (c) p-NO2C6H4COOH
C 2 H 5 - CH - C3H 7 , CH 3 - C - C H - C 2 H 5 ,
(d) m-NO2C6H4COOH
( I)
( II ) Which of the following order is correct? [2004]
H (a) 2 > 4 > 1 > 3 (b) 2 > 4 > 3 > 1
| Å CH3
H- C |
C2 H5 - CH- C2H5 (c) 1 > 2 > 3 > 4 (d) 2 > 3 > 4 > 1
|
H , 19. Which of the following is the strongest base ?
( III ) ( IV ) [2004]
it is true that
(a) only I and II are chiral compounds (a)
(b) only III is a chiral compound
(c) only II and IV are chiral compounds
(d) all four are chiral compounds (b) NHCH3
14. The IUPAC name of the compound is
(c) NH2
HO
(a) 3, 3-dimethyl - 1- cyclohexanol [2004] (d) CH2NH2
(b) 1, 1-dimethyl-3-hydroxy cyclohexane
(c) 3, 3-dimethyl-1-hydroxy cyclohexane 20. Which of the following compounds is not chiral?
(d) 1, 1-dimethyl-3-cyclohexanol (a) 1-chloro-2-methyl pentane [2004]
15. Which one of the following does not have sp2 (b) 2-chloropentane
hybridized carbon ? [2004] (c) 1-chloropentane
(a) Acetonitrile (b) Acetic acid (d) 3-chloro-2-methyl pentane
(c) Acetone (d) Acetamide 21. Due to the presence of an unpaired electron,
16. Which of the following will have a mesoisomer free radicals are: [2005]
also? [2004] (a) cations
(a) 2, 3- Dichloropentane (b) anions
(b) 2, 3-Dichlorobutane
(c) chemically inactive
(c) 2-Chlorobutane
(d) chemically reactive
(d) 2-Hydroxypropanoic acid
www.crackjee.xyz
Organic Chemistry-Some Basic Principles & Techniques C-53
22. The decreasing order of nucleophilicity among • • •
(b) (CH3)2 C H < (CH3)3 C < (C6H5)2 C H <
the nucleophiles [2005] •
(C6H5)3 C
(A) CH 3C - O - • • •
(c) (CH3)2 C H < (CH3)3 C < (C6H5)2 C H <
||

O (C6H5)3 C
• • •
(B) CH 3 O - (d) (C6H5)3 C < (C6H5)2 C H < (CH3)3 C <

(CH3)2 C H
(C) CN -
27. CH 3 Br + Nu - ¾
¾® CH 3 - Nu + Br -
O The decreasing order of the rate of the above
||
(D) H 3C S – O – is reaction with nucleophiles (Nu–) A to D is
|| [2006]
O
[Nu = (A) PhO , (B) AcO , (C) HO–, (D)
– – –

(a) (C), (B), (A), (D) (b) (B), (C), (A), (D) CH3O–]
(c) (D), (C), (B), (A) (d) (A), (B), (C), (D) (a) A > B > C > D (b) B > D > C > A
23. The reaction [2004, 2005] (c) D > C > A > B (d) D > C > B > A
O

Me
||

||

R–C + Nu R–C +X D
28.
X Nu Å
is fastest when X is N Me
OH
(a) OCOR (b) OC 2 H 5 n-Bu Et
The alkene formed as a major product in the
(c) NH 2 (d) Cl above elimination reaction is [2006]
24. Which types of isomerism is shown by 2, 3- Me Me
dichlorobutane? (a) (b)
(a) Structural (b) Geometric [2005]
(c) Optical (d) Diastereo
(c) Me (d) CH2 = CH2
25. The IUPAC name of the compound shown below
29. Increasing order of stability among the three
is : main conformations (i.e. Eclipse, Anti, Gauche)
Cl of 2-fluoroethanol is [2006]
(a) Eclipse, Anti, Gauche
(b) Anti, Gauche, Eclipse
Br (c) Eclipse, Gauche, Anti
(a) 3-bromo-1-chlorocyclohexene [2006]
(d) Gauche, Eclipse, Anti
(b) 1-bromo-3-chlorocyclohexene
(c) 2-bromo-6-chlorocyclohex-1-ene
30. The IUPAC name of is
(d) 6-bromo-2-chlorocyclohexene
26. The increasing order of stability of the following
(a) 3-ethyl-4-4-dimethylheptane [2007]
free radicals is [2006]

(b) 1, 1-diethyl-2,2-dimethylpentane
• •
(a) (C6H5)2 C H < (C6H5)3 C < (CH3)3 C < (c) 4, 4-dimethyl-5,5-diethylpentane

(CH3)2 C H (d) 5, 5-diethyl-4,4-dimethylpentane.
EBD_7764
www.crackjee.xyz
C-54 Chemistry
31. Which of the following molecules is expected to 36. The correct decreasing order of priority for the
rotate the plane of plane-polarised light? [2007] functional groups of organic compounds in the
IUPAC system of nomenclature is [2008]
COOH CHO (a) – COOH, – SO3H, – CONH2, – CHO
(b) – SO3H, – COOH,– CONH2, – CHO
(a) H2N H (b) HO H
(c) – CHO, – COOH, – SO3H, – CONH2
H (d) – CONH2, – CHO, – SO3H, – COOH
CHOH
2
37. The IUPAC name of neopentane is [2009]
(c)
(a) 2, 2 dimethylpropane
SH (b) 2 methylpropane
H2N NH2 (c) 2, 2 dimethylbutane
(d) H H (d) 2- methylbutane
Ph Ph 38. Arrange the carbanions, [2009]
32. Presence of a nitro group in a benzene ring
(CH3 )3 C , C Cl3 , (CH3 ) 2 C H , C6 H5 CH 2
[2007]
(a) deactivates the ring towards electrophilic in order of their decreasing stability :
substitution (a) (CH3 ) 2 C H > CCl3 > C6 H5 C H2 > (CH3 )3 C
(b) activates the ring towards electrophilic
substitution (b) C Cl3 > C6 H5 CH 2 > (CH 3 )2 CH > (CH3 )3 C
(c) renders the ring basic (c) (CH3 )3 C > (CH3 )2 CH > C6 H 5 CH 2 > C Cl3
(d) deactivates the ring towards nucleophilic (d) C6 H5 CH 2 > C Cl3 > (CH3 )3 C > (CH 3 ) 2 CH
substitution.
33. Which one of the following conformations of 39. The alkene that exhibits geometrical isomerism is:
cyclohexane is chiral? [2007] (a) 2- methyl propene [2009]
(a) Boat (b) Twist boat (b) 2-butene
(c) Rigid (d) Chair. (c) 2- methyl -2- butene
34. The absolute configuration of [2008] (d) propene
HO2C CO2H 40. The number of stereoisomers possible for a
compound of the molecular formula
OH CH3 – CH = CH – CH(OH) – Me is: [2009]
HO H H (b) 2 (c) 4
(a) S, S (b) R, R
(d) 6 (d) 3
(b) R, S (c) S, R
41. The correct order of increasing basicity of the
35. The electrophile, E Å attacks the benzene ring given conjugate bases (R = CH3) is [2010]
to generate the intermediate s-complex. Of the (a) RCOO < HC º C < R < NH 2
following, which s-complex is lowest energy?
[2008] (b) R < HC º C < RCOO < NH 2
NO2
(c) RCOO < NH 2 < HC º C < R
H
+ E (d) RCOO < HC º C < NH 2 < R
(a) (b) +
42. Out of the following, the alkene that exhibits
H E optical isomerism is [2010]
NO2 NO2 (a) 3-methyl-2-pentene
H
(b) 4-methyl-1-pentene
(c) + E (d) + H (c) 3-methyl-1-pentene
E (d) 2-methyl-2-pentene
www.crackjee.xyz
Organic Chemistry-Some Basic Principles & Techniques C-55
43. The change in the optical rotation of freshly 48. Which of the following compounds will exhibit
prepared solution of glucose is known as: geometrical isomerism ? [JEE M 2015]
[2011RS] (a) 2 - Phenyl -1 - butene
(a) racemisation (b) specific rotation (b) 1, 1 - Diphenyl - 1 - propene
(c) mutarotation (d) tautomerism (c) 1 - Phenyl - 2 - butene
44. The non aromatic compound among the
(d) 3 - Phenyl -1 - butene
following is : [2011RS]
49. In Carius method of estimation of halogens, 250
mg of an organic compound gave 141 mg of AgBr.
(a) (b) The percentage of bromine in the compound is :
S [JEE M 2015]
(at. mass Ag =108; Br = 80)
(a) 48 (b) 60
(c) (d)
(c) 24 (d) 36
– 50. The absolute configuration of [JEE M 2016]
CO2H
45. A solution of ( – ) – 1 – chloro –1– phenylethane
in toluene racemises slowly in the presence of a H OH
small amount of SbCl5, due to the formation of :
H Cl
[2013]
(a) carbanion (b) carbene CH3
(c) carbocation (d) free radical is :
46. The order of stability of the following (a) (2S, 3S) (b) (2R, 3R)
carbocations : [2013] (c) (2R, 3S) (d) (2S, 3R)
Å 51. The hottest region of Bunsen flame shown in
CH 2
the figure below is : [JEE M 2016]
Å Å region 4
CH 2 = CH - C H 2 ; CH3 - CH 2 - CH 2 ; is :
I II region 3
III
(a) III > II > I (b) II > III > I region 2
(c) I > II > III (d) III > I > II
region 1
47. For which of the following molecule significant
m ¹ 0?
Cl CN
(a) region 3 (b) region 4
(i) (ii) [2014] (c) region 1 (d) region 2
52. Which of the following molecules is least
Cl CN resonance stabilized? [JEE M 2017]
OH SH
(a) (b)
O
(iii) (iv)

OH SH (c) (d)
(a) Only (i) (b) (i) and (ii) N O
(c) Only (iii) (d) (iii) and (iv)
EBD_7764
www.crackjee.xyz
C-56 Chemistry
Answer Key
1 2 3 4 5 6 7 8 9 10 11 12 13 14 15
(b) (c) (c) (d) (c) (c) (b) (d) (d) (b) (d) (a) (a) (a) (a)
16 17 18 19 20 21 22 23 24 25 26 27 28 29 30
(b) (a) (d) (d) (c) (d) (a) (d) (c) (a) (b) (c) (b) (a) (a)
31 32 33 34 35 36 37 38 39 40 41 42 43 44 45
(b) (a) (b) (b) (b) (a) (a) (b) (b) (b) (d) (c) (c) (d) (c)
46 47 48 49 50 51 52
(d) (d) (c) (c) (d) (d) (d)

1. (b) –CH3 group has +I effect, as number of 5. (c) The correct name is 3 - methylbut - 2 - ol
–CH3 group increases, the inductive effect
O CH 3
increases. 1 2| | 3| 4
Therefore the correct order is 6. (c) C H3 - C - C H - C H 3 ;
CH3– CH2– < (CH3)2 – CH– < (CH3)3 C – 3- methyl-2-butanone
2. (c) Sterioisomerism involve those isomers 7. (b) In molecules (a), (c) and (d), the carbon
which contain same ligands in their atom has a multiple bond, only (b) has sp3
co-ordination spheres but differ in the hybridization.
arrangement of these ligands in space. 8. (d) A mixtur e of equal amount of two
Stereo-isomerism is of two type geomerical enantiomers is called a racemic mixture. A
isomerism and optical isomerism. In racemic mixture does not rotate plane–
geomerical isomerism ligands occupy polarized light. They are optically inactive
different positions around the central metal because for every molecule in a racemic
atom or ion. mixture that rotate plane of polarized light
in one direction, there is a mirror image
NOTE In optical isomerism isomers molecule that rotates the plane in opposite
have same formula but differ in their ability direction.
to rotate directions of the plane of 9. (d)
polarised light.
Stereoisomerism is of two types i.e.,
Cl geometrical isomerism and optical
3. (c) C = CH - CH 2 - CH 2 CH3 does isomerism
Cl Both the structures shows sterioisomerism.
not show geometrical isomerism due to Structure I shows geometrical isomerism
presence of two similar Cl atoms on the as it contains two different atoms(H) and
same C-atom. Geometrical isomerism is groups (CH3 ) attached to each carbon
shown by compounds in which the groups/ containing double bond.
atoms attached to C = C are different. H3C CH3 H CH3
C=C C=C
H H H3C H
4. (d) Amino acids contain – NH2 and – COOH
Cis butene Trans butene
NH 2 Structure II shows optical isomerism as it
groups e.g Glycine CH 2
COOH
contains a chiral carbon (attached to four
different group) atom.
www.crackjee.xyz
Organic Chemistry-Some Basic Principles & Techniques C-57

CH3 H3C O
sp3 sp sp3 ||
H
*
C OH OH C H CH3 - C º N ; CH3 - C - NH 2
sp 2
Acetonitrile Acetamide
CH2CH3 H3CH2C
butyl alcohol (Two enantiomers) 16. (b) NOTE The compounds containing two

10. (b) The hydrolysis of t-butyl bromide is an similar assymmetric C-atoms have plane of
example of SN1 reaction. The reaction symmetry and exist in Meso form.
consists of two steps. CH3
CH3 CH3 H Cl
Slow step
(i) CH3 C Br ¾¾¾¾ ¾® CH3 C+ :Br plane of symmetry
H Cl
CH3 CH3
CH3
CH3 CH3
Meso 2, 3 dichlorobutane
+ Fast step
(ii) CH3 C+ OH – ¾¾¾¾® CH3 C OH 17. (a) Only 2- cylcopropyl butane has a chiral
|

|
|

CH3 centre.
CH3
11. –
(d) HCOO exists in following resonating H
structures *
CH3 – C – chiral centre
O OΠ
|| | C2H5
H , C, O, « H , C < O 18. (d) In aromatic acids presence of electron
Hence in it both the carbon oxygen bonds withdrawing substituent e.g. –NO 2
are found equal. disperses the negative charge of the anion
12. (a) CnH2nO2 is general formula for carboxylic and stablises it and hence increases the
acid acidity of the parent benzoic acid.
13. (a) CH3
Further o-isomer will have higher acidity
O CH3 Chiral
Carbon than corresponding m and p isomers. Since
C2H5 C* C3H7 CH3 C C* C2H5
nitro group at p-position have more
H Chiral H
(I) carbon pronounced electron withdrawing than
(II)
–NO 2 group at m-position hence the
H CH3 correct order is the one given above.
|
|
H — C* C2H5 — C — C2H5
| Achiral
H Carbon | Achiral COOH COOH
H carbon
(III) (IV)

<
NO2

COOH COOH
14. (a) 1 3 IUPAC name –
2 NO2
HO < <
3, 3-Dimethyl -1 cyclohexanol
NO2
O O
sp 3 || sp 3 sp3 ||
15. (a) H3 C - C - CH 3 ; CH3 - C - OH ; 19. (d) Lone pair of electrons present on the
sp 2 sp2 nitrogen of benzyl amine is not involved
Acetone Acetic acid
EBD_7764
www.crackjee.xyz
C-58 Chemistry
in resonance. Cl
20. (c) 1-chloropentane is not chiral while others 1
are chiral in nature 6 2
25. (a)
ClCl ClCl 5 3 Br
| | | | | | | | | | | | | | | | | | | | 4
- -CC
– –CC - -CC
- -CC
- -CC
; ; C-C-C C
-C- -CC
- -CC
-C-, -,
| | | | | | | | | | | |* |* | | | | | | 3-bromo-1chlorocyclohexene
1- chloropentane 2 - chloropentane
26. (b) The order of stability of free radicals
• •
| | (C6 H 5 ) 3 C > (C 6 H 5 ) 2 C H > (CH 3 ) 3
- Cl - C - –C – Cl
| | | | | | | | | | • •
- C – C – C– C– C – C– C – C – C – C – C > (CH3 ) 2 CH
| |* | | | | | |* | |
1-chloro-2-methyl 3-chloro-2-methyl The stabilisation of first two is due to
pentane pentane resonance and last two is due to inductive
21. (d) Free radicals are electrically neutral, effect.
unstable and very reactive on account of 27. (c)
the presence of odd electrons.
The stronger the acid, the weaker the
22. (a) In moving down a group, the basicity and conjugate base formed.
nucleophilicity are inversely related, i.e.
The acid character follows the order :
nucleophilicity increases while basicity
decreases. In going from left to right across CH3COOH > C6H5OH > H2O > CH3OH
a period, the basicity and nucleophilicity The basic character will follow the order
are directly related. Both of the CH3COO– < C6H5O– < O–H < CH3O–
characteristics decrease as the 28. (b) Hofmann's rule : When theoretically more
electronegativity of the atom bearing lone than one type of alkenes are possible in
pair of electrons increases. If the eliminations reaction, the alkene containing
nucleophilic centre of two or more species least alkylated double bond is formed as
is same, nucleophilicity parallels basicity, major product. Hence
i.e. more basic the species, stronger is its
Me Me
nucleophilicity. D
Hence based on the above facts, the +
OH N Me
correct order of nucleophilicity will be n-Bu |
O Et
– – –
H3C S— O– > CN > CH3O > CH3 C– O It is less stearically b-hydrogen is
O O removed
(D) (C) (B) (A)
H
29. (a) H H
O
||
23. (d) R - C - X ; when X is Cl the C–X bond is d–
H F
more polar and ionic which leaves the
d+
compound more reactive for nucleophilic O–H
substitution reaction.
Due to hydrogen bonding between H & F
Cl Cl gauche conformation is most stable hence
24. (c) CH3 CH3 . 2, 3-dichloro butane will the correct order is
H H
exhibit optical isomerism due to the Eclipse, Anti, Gauche
presence of two asymmetric carbon atom.
www.crackjee.xyz
Organic Chemistry-Some Basic Principles & Techniques C-59

CH3 O O
7 6 5 4| 3 2 1 || ||
30. (a) CH3 - CH 2 - CH 2 - C - CH - CH 2 - CH3 –COOH > –SO3 H > – C - NH 2 > - C - H
| |
CH3 CH 2 37. (a) CH3
| |
1 3 2
CH3 H3C- C - CH3
3- ethyl - 4,4 -dimethyl heptane |
CH3
31. (b) NOTE The organic compounds which Neopentane
have chiral carbon atom (a carbon atom or 2, 2- Dimethylpropane
attached to four different group or atoms
38. (b)
and do not have plane of symmetry rotate
plane polarised light. Cl
– – – –
CHO C Cl > C6H5CH2 > (CH3)2 CH > (CH3)3C
| Cl
HO - C*- H (* is asymmetric carbon) –ve charge –M effect +I effect of CH3 group
| highly dispersed delocalises intensifies the –ve charge
–ve charge
CH 2 OH due to – I effect

32. (a) Nitro group is electron withdrawing group, 39. (b) H3C CH3 H3C H
so it deactivates the ring towards C=C C=C
electrophilic substitution.
H H H CH3
33. (b) Chiral conformation will not have plane of
symmetry. Since twist boat does not have *
40. (b) CH 3 – CH = CH – CHCH 3
plane of symmetry it is chiral. |
OH
4 1 exhibits both geometrical as well as optical
isomerism.
cis - R cis - S
3 6
trans - R trans - S
41. (d) The correct order of basicity is
RCOO - < CH º C - < NH 2 - < R -
5 2
42. (c) For a compound to show optical isomerism,
34. (b) The absolute configuration is (R, R)
presence of chiral carbon atom is a
(using priority rules to get the absolute necessary condition.
configuration)
So the correct answer is (b) H
35. (b) In option (b) the complex formed is with |
benzene where as in other cases it is formed H 2 C < HC — C* — CH 2 , CH3
with nitrobenzene with –NO2 group in |
different position (o-, m-, p-). The complex CH3
formed with nitrobenzene in any position 3- methyl-1-pentene
of –NO2 group is less stable than the 43. (c) When either of the two forms of glucose is
complex formed with benzene so the correct dissolved in water there is change in
answer is (b) rotation till the equilibrium value of + 52.5º.
NOTE The most stable complex has This is known as mutarotation
a–D(+)Glucose Equilibrium Mixture
lowest energy.
+111.5º +52.5º
36. (a) The correct order of priority for the given
b–D–(+)Glucose
functional group is +111.5º +19.5º
EBD_7764
www.crackjee.xyz
C-60 Chemistry
H H H
44. (d) sp3 Carbon |
48. (c) H3C — C = CH — CH 2
|
Ph
Cyclopentadiene does not obey Huckel's 1- Phenyl-2-butene the two groups around
Rule, as it has sp3 carbon in the ring. each of the doubly bonded carbon
45. (c) Carbocations are planar hence can be attacked Because, all are different. This compound
on either side to form racemic mixture. can show cis-and trans-isomerism.
SbCl
Å 49. (c) Mass of substance = 250 mg = 0.250 g
Cl - CH- CH3 ¾¾¾¾ 5 ® Ph - C H - CH + SbCl- ¾¾
®
Toluene 3 6 Mass of AgBr = 141 mg = 0.141 g
| (carbocation) 1 mole of AgBr = 1 g atom of Br
Ph 188 g of AgBr = 80 g of Br
(- )
\ 188 g of AgBr contain bromine = 80 g
Ph - CH - CH3 + SbCl5 0.141 g of AgBr contain bromine =
| 80
Cl ´ 0.141
(d+l) mixture 188
This much amount of bromine present in
46. (d) Higher stability of allyl and aryl substituted
0.250 g of organic compound
methyl carbocation is due to dispersal of
\ % of bromine = = 24%
positive charge due to resonance
50. (d) CO2H
+ +
CH 2 = CH - C H 2 ¬¾® CH 2 - CH = CH 2 H 1 OH
Resonating structures of allyl carbocation
2
+
CH2 CH2 CH2 CH2 H Cl
Å Å CH3
At (1),
Å 3 1
S
Resonating structures of benzyl carbocation
whereas in alkyl carbocations dispersal of posi- 4 1 2 3
tive charge on different hydrogen atoms is due
to hyperconjugation. Hence the correct order of
stability will be 2 4
Å
CH2 It is ‘S’configurated
At. (2),
Å Å 2 1
> CH 2 = CH - C H 2 > CH3 - CH 2 - CH 2 R
Allyl, I Propyl, II
Benzyl, III 4 1 3 2
H
O O 3 4
47. (d)
H 51. (d) Region 2 (blue flame) will be the hottest region
H of Bunsen flame shown in given figure
S S O
H
In both the molecules the bond moments 52. (d) is nonaromatic and hence least
are not canceling with each other and hence
the molecules has a resultant dipole and
hence the molecule is polar. reasonance stabilized whereas other three
are aromatic.
www.crackjee.xyz
Hydrocarbons C-61

13
Hydrocarbons

1. Which of these will not react with acetylene? 8. Of the five isomeric hexanes, the isomer which
[2002] can give two monochlorinated compounds is
(a) NaOH (b) ammonical AgNO3 [2005]
(c) Na (d) HCl. (a) 2-methylpentane
2. What is the product when acetylene reacts with (b) 2, 2-dimethylbutane
hypochlorous acid? [2002] (c) 2, 3-dimethylbutane
(a) CH3COCl (b) ClCH2CHO (d) n-hexane
(c) Cl2CHCHO (d) ClCH2COOH. 9. The compound formed as a result of oxidation
3. On mixing a certain alkane with chlorine and of ethyl benzene by KMnO4 is [2007]
irradiating it with ultraviolet light, it forms only (a) benzyl alcohol (b) benzophenone
one monochloroalkane. This alkane could be
[2003] (c) acetophenone (d) benzoic acid.
(a) pentane (b) isopentane 10. Which of the following reactions will yield
(c) neopentane (d) propane 2, 2-dibromopropane? [2007]
4. Butene-1 may be converted to butane by (a) CH3 – CH = CH2 + HBr ®
reaction with [2003] (b) CH3 – C º CH + 2HBr ®
(a) Sn – HCl (b) Zn – Hg (c) CH3CH = CHBr + HBr ®
(c) Pd/H2 (d) Zn – HCl (d) CH º CH + 2HBr ®
5. Which one of the following has the minimum 11. The reaction of toluene with Cl2 in presence of
boiling point? [2004] FeCl3 gives predominantly [2007]
(a) 1 - Butene (b) 1 - Butyne (a) m-chlorobenzene
(c) n- Butane (d) isobutane (b) benzoyl chloride
6. 2-Methylbutane on reacting with bromine in the (c) benzyl chloride
presence of sunlight gives mainly [2005] (d) o- and p-chlorotoluene.
(a) 1-bromo-3-methylbutane 12. Toluene is nitrated and the resulting product is
(b) 2-bromo-3-methylbutane reduced with tin and hydrochloric acid. The
(c) 2-bromo-2-methylbutane product so obtained is diazotised and then
(d) 1-bromo-2-methylbutane heated wth cuprous bromide. The reaction
7. Reaction of one molecule of HBr with one mixture so formed contains [2008]
molecule of 1, 3-butadiene at 40°C gives (a) mixture of o- and p-bromotoluenes
predominantly [2005] (b) mixture of o- and p-dibromobenzenes
(a) 1-bromo-2-butene under kinetically (c) mixture of o- and p-bromoanilines
controlled conditions
(d) mixture of o- and m-bromotoluenes
(b) 3-bromobutene under thermodynamically
controlled conditions 13. In the following sequence of reactions, the
(c) 1-bromo-2-butene under thermodyna- alkene affords the compound ‘B’
mically controlled conditions CH 3 - CH = CH - CH 3 ¾¾3¾
O
® A ¾¾¾
H O
2 ® B.
(d) 3-bromobutene under kinetically controlled Zn
conditions The compound B is [2008]
EBD_7764
www.crackjee.xyz
C-62 Chemistry
(a) C H3CH2CHO (b) C H3COCH3 CH3
(c) C H3CH2COCH3 (d) C H3CHO CH3
14. The hydrocarbon which can react with sodium (a) (b) H3C
in liquid ammonia is [2008]
CH3
(a) CH 3CH 2 CH 2 C º CCH 2CH 2 CH3
CH3
(b) CH 3CH 2C º CH CH3
(c) CH 3CH = CHCH3
(c) CH3 (d)
(d) CH 3CH 2C º CCH 2CH3
CH3
15. The treatment of CH3MgX with CH 3C º C - H
produces [2008] 21. The product of the reaction given below is:
(a) CH 3 - CH = CH 2
1. NBS/hv
(b) CH 3C º C - CH3 X
H H 2. H 2O/K 2CO3
| |
(c) CH 3 - C = C - CH3
[JEE M 2016]
(d) CH4
16. One mole of a symmetrical alkene on ozonolysis O CO2H
gives two moles of an aldehyde having a
molecular mass of 44 u. The alkene is [2010]
(a) (b)
(a) propene (b) 1-butene
(c) 2-butene (d) ethene
17. Ozonolysis of an organic compound 'A' produces
OH
acetone and propionaldhyde in equimolar
mixture. Identify 'A' from the following
compounds: [2011RS] (c) (d)
(a) 1 – Pentene
(b) 2 – Pentene 22. The reaction of propene with HOCl (Cl2 + H2O)
(c) 2 – Methyl – 2 – pentene
proceeds through the intermediate:
(d) 2 – Methyl – 1 – pentene
[JEE M 2016]
18. Which branched chain isomer of the
hydrocarbon with molecular mass 72u gives only (a) CH 3 – CH ( OH ) - CH 2+
one isomer of mono substituted alkyl halide ?
[2012] (b) CH3 – CHCl - CH 2+
(a) Tertiary butyl chloride
(b) Neopentane (c) CH3 – CH + - CH 2 – OH
(c) Isohexane
(d) Neohexane (d) CH3 – CH + - CH 2 – Cl
19. 2-Hexyne gives trans-2-Hexene on treatment
with : [2012] 23. 3-Methyl-pent-2-ene on reaction with HBr in
(a) Pt/H2 (b) Li / NH3 presence of peroxide forms an addition product.
(c) Pd/BaSO4 (d) Li AlH4 The number of possible stereoisomers for the
20. Which compound would give 5 - keto - 2 - product is : [JEE M 2017]
methylhexanal upon ozonolysis ? (a) Six (b) Zero
[JEE M 2015] (c) Two (d) Four
www.crackjee.xyz
Hydrocarbons C-63

Answer Key
1 2 3 4 5 6 7 8 9 10 11 12 13 14 15
(a) (c) (c) (c) (d) (c) (c) (c) (d) (b) (d) (a) (d) (b) (d)
16 17 18 19 20 21 22 23
(c) (c) (b) (b) (d) (d) (d) (d)

1. (a) Acetylene reacts with the other three as: boiling points than the corrosponding
alkanes.
CH 2 Thus B.pt. follows the order
Na
CH º CNa CH º CH +HCl
liq. NH 3
CHCl alkynes > alkene > alkanes (straight chain)
CH3 > branched chain alkanes.
+HCl
CH3
CHCl CHCl2 |
Br
[AgNO3+NH4OH] 6. (c) CH3 - CH - CH2 - CH3 ¾¾®
¾2
CH º CH sun light
CH3
[AgNO +NH OH] |
AgCº CAg + NH4NO3 CH3 - C - CH2CH3
white ppt. |
Br
CHOH 2-bromo-2-methylbutane
2. (c) CH º CH + HOCl ¾¾
® ||
Ease of replacement of H-atom 3° > 2° > 1°.
CHCl
7. (c) CH2 = CH – CH = CH2CH2 + HBr
Br
HOCl é
CH (OH ) 2 ù CHO
- H 2O
¾¾¾®ê | ú ¾¾ ¾ ¾® |
CH2 = CH – CH – CH3
ëê CHCl 2 ûú CHCl 2
dichloroacetaldehyde At –80°C the product is
1, 2-addition
3. (c) In neopentane all the H atoms are same (1º).
CH2 – CH = CH – CH3
CH3
| Br
CH 3 - C - CH 3 At 40°C the product is
| 1, 4-addition
CH3
CH3 CH3
4. (c) Alkenes combine with hydrogen under | |
pressure and in presence of a catalyst (Ni, 8. (c) CH 3 - C H - C H - CH 3 . Since it contains
Pt or Pd) and form alkanes. only two types of H-atoms hence it will
H 2 / Pd give only two mono chlorinated
Butene - 1 ¾¾¾¾® Butane
CH3 CH3
5. (d) NOTE Among isomeric alkanes, the | |
straight chain isomer has higher boiling compounds viz. Cl.CH 2 - C H - C H - CH 3
1-chloro -2,3 -dimethyl butane
point than the branched chain isomer. The
CH 3 CH 3
greater the branching of the chain, the | |
lower is the boiling point. Further due to CH 3 - C - C H - CH 3
the presence of p electrons, these moleculs |
and Cl
are slightly polar and hence have higher 2 -chloro - 2,3-dimethyl butane
EBD_7764
www.crackjee.xyz
C-64 Chemistry
9. (d) When alkyl benzene are oxidised with alkaline
KMnO4, (strong oxidising agent) the entire CH3
alkyl group is oxidised to –COOH group (HNO3+ HSO)
2 4
regardless of length of side chain.
CH2CH3
( O ), KMnO4 / OH-
¾¾¾¾¾¾¾¾ ® CH3 CH3
Ethyl benzene (HNO + HSO)
NO2
COOH +

NO2
Benzoic aicd
o- p-
10. (b) The reaction follows Markownikoff rule on reduction with Sn/HCl they will form
which states that when unsymmetrical corresponding anilines in which –NO2 group
reagent adds across unsymmetrical double
changes to –NH2. The mixture now contains
or triple bond the negative part adds to
carbon atom having lesser number of CH3 CH3
hydrogen atoms. NH2
CH3 - C º CH + HBr ® and . These anilines

Br NH2
HBr |
CH3 - C = CH 2 ¾¾¾
® CH3 - C - CH3 when diazotized and then treated with CuBr
| | forms o-, p- bromotoluenes.
Br Br
13. (d) Completing the sequence of given reactions,
2, 2-
dibromo-propane O
CH 3 – CH = CH - CH 3 ¾¾3¾
®
11. (d) FeCl3 is Lewis acid. In presence of FeCl3 side
chain hydrogen atoms of toluene are
O
substituted.
CH3 Zn / H O
CH3 CH3– CH CH – CH3 ¾¾¾¾®
2
Cl
FeCl3
+ Cl2 ¾¾¾® + O O
Toluene o-chloro toluene ‘A’
(ozonide)
CH3

2CH3CHO+ H 2 O + ZnO
Cl 'B '
p-chloro toluene Thus ‘B’ is CH3CHO
Hence (d) is correct answer.
CH3
14. (b) Alkynes having terminal –C º H react with
Na in liquid ammonia to yield H2 gas of the
12. (a) NOTE Toluene ( ) contains –
given compounds CH3CH2C º CH can react
with Na in liquid NH3 so the correct answer
CH3 group which is o-, p- directing group so is (b).
on nitration of toluene the –NO2 group will
Na in
occupy o-, p- positions. CH3CH 2 C º CH ¾¾¾¾¾
®
liquid NH3
www.crackjee.xyz
Hydrocarbons C-65
19. (b) Anti addition of hydrogen atoms to the triple
1
CH3 CH 2 C º C – Na + + H 2 (g) bond occurs when alkynes are reduced with
2 sodium (or lithium) metal in ammonia,
15. (d) Writing the reaction we get ethylamine, or alcohol at low temperatures.
This reaction called, a dissolving metal
CH 3 MgX + CH 3 – C º C – H ¾¾
®
reduction, produces an (E)- or trans-alkene.
CH3 – C º CMgX + CH 4 (g) Sodium in liq. NH3 is used as a source of
So we find that CH4 is produced in this electrons in the reduction of an alkyne to a
trans alkene.
reaction.
16. (c) The given molecular formula suggests that CH3 CH2 CH2 C C CH3
2-Hexyne
the aldehyde formed will be acetaldehyde
Li/NH 3
hence the alkene will be CH3 CH2 CH2 H
Birch reduction
CH3CH = CH CH3 C C
H CH3
2- butene
Trans-2-Hexene
O3 H O 20. (d) When 1, 3-dimethylcyclopentene is heated with
¾¾® H
ozone and then with zinc and acetic acid,
C C
oxidative cleavage leads to keto - aldehyde.
O O
CH3 CH3
O
1O - 78° C
Zn / H 2O 2CH3CHO + H 2O 2 ¾ ¾ ¾3¾ ¾ ¾ ¾
® O C–H
¾¾¾¾® 2 - Zn- CH 3 COOH
17. (c) From the products formed it is clear that
the compound has 5 carbon atoms with a CH3 CH3
double bond and methyl group on 2 nd
carbon atom.
O O
CH3 || ||
| CH3 — C— CH 2 — CH 2 — CH— C— H
CH3 - C = CH - CH 2 - CH3 |
CH3
O / Zn, H O
¾¾¾¾¾¾
3 2 ® 5- keto – 2 – methylhexanal
21. (d) N – bromosuccinimide results into bromination
(2–Methyl–2–pentene) at allylic and benzylic positions
(A)

CH3 O
| NBS/hv
CH3 - C = O + CH3 - CH 2 - C

Acetone Propionaldehyde
18. (b)
More stable
CH3
Cl2/hv Br HO
H 2O/K 2CO 3
H3C C CH3 monohalogenation
single product NBS

CH3
neopentane
EBD_7764
www.crackjee.xyz
C-66 Chemistry
22. (d)

CH2 – CH – CH3 CH3 CH3


Å Cl More stable intermediate
CH2 = CH – CH3 + Cl – H Br Br H
Å
CH2 – CH – CH3
H CH3 H3C H
Cl

Å C2H5 C2H5
CH2 – CH – CH3 OH CH2 – CH – CH3 (I) (II)
Cl Cl OH
23. (d) If two chirality centres are created as a CH3 CH3
result of an addition reaction four
stereoisomers can be obtained as products. Br H H Br

CH3 CH3 H CH3 H3C H


HBr
C = C ¾¾ ¾®
– H2 O
H C2H5 C2H5 C2H5
cis-3, methyl pent-2-ene
(III) (IV)
Br CH3
| |
CH3 – CH – CH – CH2 – CH3
* *
2, Bromo, 3-methyl pentane
(2 chiral centre)
www.crackjee.xyz
Environmental Chemistry C-67

14
Environmental
Chemistry
1. The smog is essentially caused by the presence (c) Ozone absorbs infrared radiation.
of [2004] (d) Oxides of nitrogen in the atmosphere can
(a) Oxides of sulphur and nitrogen cause the depletion of ozone layer.
(b) O2 and N2 (c) O2 and O3 4. What is DDT among the following ? [2012]
(d) O3 and N2 (a) Greenhouse gas
2. Identify the wrong statement in the following: (b) A fertilizer
[2008] (c) Biodegradable pollutant
(a) Chlorofluorocarbons are responsible for (d) Non-biodegradable pollutant
ozone layer depletion 5. The gas leaked from a storage tank of the Union
(b) Greenhouse effect is responsible for global Carbide plant in Bhopal gas tragedy was: [2013]
warming (a) Methyl isocyanate
(c) Ozone layer does not permit infrared (b) Methylamine
radiation from the sun to reach the earth (c) Ammonia
(d) Acid rain is mostly because of oxides of (d) Phosgene
nitrogen and sulphur 6. A water sample has ppm level concentration of
3. Identify the incorrect statement from the following anions [2017]
following : [2011RS] – 2– –
F = 10; SO4 = 100; NO3 = 50
(a) Ozone absorbs the intense ultraviolet the anion/anions that make/makes the water
radiation of the sun. sample unsuitable for drinking is/are :
(b) Depletion of ozone layer is because of its (a) only NO3– (b) both SO42– and NO3–
chemical reactions with chlorofluoro –
alkanes. (c) only F (d) only SO42–

Answer Key
1 2 3 4 5 6
(a) (c) (c) (d) (a) (c)

1. (a) Photochemical smog is caused by oxides N2O + h u ¾ ¾® NO + N


of sulphur and nitrogen. reactive nitric oxide
2. (c) NOTE Ozone layer acts as a shield and NO + O 3 ¾ ¾® NO 2 + O 2
does not allow ultraviolet radiation from O3 + h u ¾ ¾® O 2 + O
sun to reach earth. It does not prevent infra-
red radiation from sun to reach earth. NO 2 + O ¾ ¾® NO + O 2
Thus option (c) is wrong statement and so 2 O3 + h u ¾ ¾® 3 O 2 (Net reaction)
it is the correct answer. The presence of oxides of nitrogen increase
3. (c) The ozone layer, existing between 20 to the decomposition of O 3 .
35 km above the earth’s surface, shield the
earth from the harmful U. V. radiations from 4. (d) DDT is a non-biodegradable pollutant.
5. (a)
the sun. 6. (c) Above 2 ppm concentration of F – in
Depletion of ozone is caused by oxides of drinking water cause brown mottling of
nitrogen teeth.
EBD_7764
www.crackjee.xyz
C-68 Chemistry

15
The Solid State

1. Na and Mg crystallize in BCC and FCC type


20 3 24 3
crystals respectively, then the number of atoms (c) pr (d) pr
of Na and Mg present in the unit cell of their 3 3
respective crystal is [2002] 6. In a compound, atoms of element Y form ccp
(a) 4 and 2 (b) 9 and 14 lattice and those of element X occupy 2/3rd of
(c) 14 and 9 (d) 2 and 4. tetrahedral voids. The formula of the compound
will be [2008]
2. How many unit cells are present in a cube-
shaped ideal crystal of NaCl of mass 1.00 g ? (a) X4 Y3 (b) X2 Y3
[2003] (c) X2 Y (d) X3 Y4
[Atomic masses : Na = 23, Cl = 35.5] 7. Copper crystallises in fcc with a unit cell length
(a) 5.14 × 1021 unit cells of 361 pm. What is the radius of copper atom?
[2009]
(b) 1.28 × 1021 unit cells
(a) 127 pm (b) 157 pm
(c) 1.71 × 1021 unit cells
(c) 181 pm (d) 108 pm
(d) 2.57 × 1021 unit cells
8. The edge length of a face centered cubic cell of
3. What type of crystal defect is indicated in the
an ionic substance is 508 pm. If the radius of the
diagram below ? [2004]
cation is 110 pm, the radius of the anion is
Na + Cl - Na + Cl - Na + Cl - (a) 288 pm (b) 398 pm [2010]
Cl– c Cl– Na+ c Na+ (c) 618 pm (d) 144 pm
Na+Cl– c Cl– Na+ Cl– 9. Percentages of free space in cubic close packed
Cl– Na+ Cl–Na+ c Na+ structure and in body centered packed structure
(a) Interstitial defect are respectively [2010]
(a) 30% and 26% (b) 26% and 32%
(b) Schottky defect
(c) 32% and 48% (d) 48% and 26%
(c) Frenkel defect
10. Copper crystallises in fcc lattice with a unit cell
(d) Frenkel and Schottky defects
edge of 361 pm. The radius of copper atom is :
4. An ionic compound has a unit cell consisting of [2011RS]
A ions at the corners of a cube and B ions on the
(a) 108 pm (b) 128 pm
centres of the faces of the cube. The empirical
(c) 157 pm (d) 181 pm
formula for this compound would be [2005]
11. Lithium forms body centred cubic structure. The
(a) A 3B (b) AB3 length of the side of its unit cell is 351 pm. Atomic
radius of the lithium will be : [2012]
(c) A 2B (d) AB
(a) 75 pm (b) 300 pm
5. Total volume of atoms present in a face-centred (c) 240 pm (d) 152 pm
cubic unit cell of a metal is (r is atomic radius)
12. Which of the following exists as covalent
[2006]
crystals in the solid state ? [2013]
12 3 16 3 (a) Iodine (b) Silicon
(a) pr (b) pr
3 3 (c) Sulphur (d) Phosphorus
www.crackjee.xyz
The Solid State C-69
13. CsCl crystallises in body centered cubic lattice. (c) It contains Cs3+ and I– ions.
If ‘a’ is its edge length then which of the (d) It contains Cs+, I– and lattice I2 molecule.
following expressions is correct? [2014] 15. Sodium metal crystallizes in a body centred cubic
(a) r +r = 3a lattice with a unit cell edge of 4.29Å. The radius
Cs + Cl -
of sodium atom is approximately : [JEE M 2015]
3a (a) 5.72Å (b) 0.93Å
(b) r +r =
Cs + Cl - 2 (c) 1.86Å (d) 3.22Å
16. A metal crystallises in a face centred cubic
3 structure. If the edge length of its unit cell is 'a',
(c) r +r = a
Cs + Cl - 2 the closest approach between two atoms in
metallic crystal will be : [JEE M 2017]
(d) r +r = 3a
Cs + Cl - (a) 2a (b) 2 2 a
14. The correct statement for the molecule, CsI3 is:
(a) It is a covalent molecule. [2014] a
(c) 2a (d)
2
(b) It contains Cs+ and I3- ions.

Answer Key
1 2 3 4 5 6 7 8 9 10 11 12 13 14 15
(d) (d) (b) (b) (b) (a) (a) (d) (b) (b) (d) (b) (c) (b) (c)
16
(d)

1. (d) In bcc - points are at corners and one in the


centre of the unit cell. 1.0 ´ 6.02 ´10 23
\ units cells = = 2.57 × 1021
Number of atoms per unit cell 58.5 ´ 4
unit cells.
1
= 8´ +1 = 2 . 3. (b) When equal number of cations and anions
8 are missing from their regular lattice positions,
In fcc - points are at the corners and also we have schottky defect.
centre of the six faces of each cell. This type of defects are more common in
Number of atoms per unit cell ionic compounds with high co-ordination
1 1 number and where the size of positive and
= 8´ + 6 ´ = 4 . negative ions are almost equal e.g. NaCl
8 2
KCl etc.
2. (d) Since in NaCl type of structure 4 formula 4. (b) Number of A ions in the unit cell.
units form a cell. Number of formulas in
1
1. 0 ´8 = 1
=
cube shaped crystals = ´ 6.02 ´ 10 23 8
58.5 Number of B ions in the unit cell
No. of unit cells present in a cubic crystal 1
´6 = 3
=
3
P ´ a ´ NA m ´ NA 2
= = Hence empirical formula of the compound
M´Z M´Z = AB3
EBD_7764
w w w . c r a c k j e
C-70 Chemistry
5. (b) The face centered cubic unit cell contains
11. (d) For BCC structure 3 a = 4r
4 atom
4 16 3 3
\ Total volume of atoms = 4 ´ pr 3 = pr 3 r= a = ´ 351 = 152 pm.
3 3 4 4
6. (a) From the given data, we have 12. (b)
Number of Y atoms in a unit cell = 4
Number of X atoms in a unit cell 13. (c)
Cl – Cl –
2 16
= 8´ = Cl –
3 3 Cl
From the above we get the formula of the
compound as X16 / 3Y4 or X 4 Y3 Cs+

7. (a) For fcc unit cell, 4r = 2a


Cl – Cl
2 ´ 361 Cl –
r= = 127 pm Cl –
4
8. (d) For an Fcc crystal
Relation between radius of cation, anion
edge length and edge length of the cube
rcation + ranion =
2
2r + 2r = 3a
Cs + Cl-
508
110 + ranion = 3a
2 r +r =
Cs + Cl-
2
ranion = 254 – 110 = 144 pm
9. (b) Packing fraction is defined as the ratio of 14. (b) CsI3 dissociates as CsI3 ® Cs+ + I3–
the volume of the unit cell that is occupied 15. (c) In bcc the atoms touch along body
by the spheres to the volume of the unit diagonal
cell. \ 2r + 2r = 3a
P.F. for ccp and bcc are 0.74 and 0.68
respectively. 3a 3 ´ 4.29
\ r= = = 1.857Å
So, the free space in ccp and bcc are 26% 4 4
& 32% respectively.
16. (d) For a FCC unit cell
10. (b) fcc lattice
a = 361 pm 2a
r=
a 2 = 4r 4

361 ´ 2 2a a
r= = 127.6 » 128pm \ closest distance (2r) = =
4 4 2
www.crackjee.xyz
Solutions C-71

16
Solutions

1. Freezing point of an aqueous solution is 6. Which one of the following aqueous solutions
(–0.186)°C. Elevation of boiling point of the same will exihibit highest boiling point ? [2004]
solution is Kb = 0.512°C,Kf = 1.86°C, find the (a) 0.015 M urea (b) 0.01 M KNO3
increase in boiling point. [2002] (c) 0.01 M Na2SO4 (d) 0.015 M glucose
(a) 0.186°C (b) 0.0512°C 7. For which of the following parameters the
(c) 0.092°C (d) 0.2372°C. structural isomers C2 H5 OH and CH3 OCH3
2. In mixture A and B components show -ve would be expected to have the same
deviation as values?(Assume ideal behaviour) [2004]
(a) D Vmix > 0 [2002] (a) Boiling points
(b) D Hmix < 0 (b) Vapour pressure at the same temperature
(c) A – B interaction is weaker than A – A and B (c) Heat of vaporization
– B interaction (d) Gaseous densities at the same temperature
(d) A – B interaction is stronger than A – A and and pressure
B – B interaction. 8. Which of the following liquid pairs shows a
3. If liquids A and B form an ideal solution [2003] positive deviation from Raoult’s law ? [2004]
(a) the entropy of mixing is zero (a) Water - nitric acid
(b) the free energy of mixing is zero (b) Benzene - methanol
(c) the free energy as well as the entropy of (c) Water - hydrochloric acid
mixing are each zero (d) Acetone - chloroform
(d) the enthalpy of mixing is zero 9. Which one of the following statements is FALSE?
4. In a 0.2 molal aqueous solution of a weak acid [2004]
HX the degree of ionization is 0.3. Taking kf for (a) The correct order of osmotic pressure for
water as 1.85, the freezing point of the solution 0.01 M aqueous solution of each
will be nearest to [2003] compound is
(a) – 0.360º C (b) – 0.260º C BaCl 2 > KCl > CH 3COOH > sucrose
(c) + 0.480º C (d) – 0.480º C (b) The osmotic pressure (p) of a solution is
5. A pressure cooker reduces cooking time for food given by the equation p = MRT, where M is
because [2003] the molarity of the solution
(a) boiling point of water involved in cooking (c) Raoult’s law states that the vapour pressure
is increased of a component over a solution is
(b) the higher pressure inside the cooker proportional to its mole fraction
crushes the food material (d) Two sucrose solutions of same molality
(c) cooking involves chemical changes helped prepared in different solvents will have the
by a rise in temperature same freezing point depression
(d) heat is more evenly distributed in the 10. Benzene and toluene form nearly ideal solution.
cooking space At 20°C, the vapour pressure of benzene is 75
torr and that of toluene is 22 torr. The partial
EBD_7764
www.crackjee.xyz
C-72 Chemistry
vapour pressure of benzene at 20°C for a (a) 210.0 g mol–1 (b) 90.0 g mol–1
solution containing 78 g of benzene and 46 g of (c) 115.0 g mol–1 (d) 105.0 g mol–1.
toluene in torr is [2005] 17. At 80° C, the vapour pressure of pure liquid ‘A’
(a) 53.5 (b) 37.5 is 520 mm Hg and that of pure liquid ‘B’ is 1000
(c) 25 (d) 50 mm Hg. If a mixture solution of ‘A’ and ‘B’ boils
11. Equimolar solutions in the same solvent have at 80° C and 1 atm pressure, the amount of ‘A’ in
[2005] the mixture is (1 atm = 760 mm Hg) [2008]
(a) Different boiling and different freezing (a) 52 mol percent (b) 34 mol percent
points (c) 48 mol percent (d) 50 mol percent
(b) Same boiling and same freezing points 18. The vapour pressure of water at 20° C is 17.5
(c) Same freezing point but different boiling mm Hg. If 18 g of glucose (C6 H12 O 6) is added
points to 178.2 g of water at 20° C, the vapour pressure
(d) Same boiling point but different freezing of the resulting solution will be [2008]
points (a) 17.325 mm Hg (b) 15.750 mm Hg
12. Among the following mixtures, dipole-dipole as (c) 16.500 mm Hg (d) 17.500 mm Hg
the major interaction, is present in [2006] 19. A binary liquid solution is prepared by mixing n-
(a) KCl and water heptane and ethanol. Which one of the following
(b) benzene and carbon tetrachloride statements is correct regarding the behaviour
(c) benzene and ethanol of the solution? [2009]
(d) acetonitrile and acetone (a) The solution is non-ideal, showing – ve
deviation from Raoult’s Law.
13. 18 g of glucose (C6H12O6) is added to 178.2 g of
water. The vapour pressure of water for this (b) The solution is non-ideal, showing + ve
aqueous solution at 100ºC is [2006] deviation from Raoult’s Law.
(a) 76.00 Torr (b) 752.40 Torr (c) n-heptane shows + ve deviation while
ethanol shows – ve deviation from Raoult’s
(c) 759.00 Torr (d) 7.60 Torr
Law.
14. A mixture of ethyl alcohol and propyl alcohol
(d) The solution formed is an ideal solution.
has a vapour pressure of 290 mm at 300 K. The
vapour pressure of propyl alcohol is 200 mm. If 20. Two liquids X and Y form an ideal solution. At
the mole fraction of ethyl alcohol is 300 K, vapour pressure of the solution
0.6, its vapour pressure (in mm) at the same containing 1 mol of X and 3 mol of Y is 550 mmHg.
temperature will be [2007] At the same temperature, if 1 mol of Y is further
added to this solution, vapour pressure of the
(a) 360 (b) 350
solution increases by 10 mmHg. Vapour pressure
(c) 300 (d) 700
( in mmHg) of X and Y in their pure states will be,
15. Equal masses of methane and oxygen are mixed respectively: [2009]
in an empty container at 25°C. The fraction of
(a) 300 and 400 (b) 400 and 600
the total pressure exerted by oxygen is [2007]
(c) 500 and 600 (d) 200 and 300
(a) 1/2 (b) 2/3
21. If sodium sulphate is considered to be
1 273 completely dissociated into cations and anions
(c) ´ (d) 1/3.
3 298 in aqueous solution, the change in freezing point
of water (DTf), when 0.01 mol of sodium sulphate
16. A 5.25% solution of a substance is isotonic with a
is dissolved in 1 kg of water, is (Kf = 1.86 K kg
1.5% solution of urea (molar mass = 60 g mol–1) in
mol–1) [2010]
the same solvent. If the densities of both the
solutions are assumed to be equal to 1.0 g cm–3, (a) 0.372 K (b) 0.0558 K
molar mass of the substance will be [2007] (c) 0.0744 K (d) 0.0186 K
www.crackjee.xyz
Solutions C-73
22. On mixing, heptane and octane form an ideal (a) 0.875 M (b) 1.00 M
solution. At 373 K, the vapour pressures of the (c) 1.75 M (d) 0.975 M
two liquid components (heptane and octane) are 27. Consider separate solutions of 0.500 M
105 kPa and 45 kPa respectively. Vapour pressure C2H5OH(aq), 0.100 M Mg3 (PO4)2 (aq), 0.250
of the solution obtained by mixing 25.0 g of M KBr(aq) and 0.125 M
heptane and 35 g of octane will be
Na3PO4(aq) at 25°C. Which statement is true
(molar mass of heptane = 100 g mol–1 and of about these solutions, assuming all salts to be
octane = 114 g mol–1) [2010] strong electrolytes? [2014]
(a) 72.0 kPa (b) 36.1 kPa (a) They all have the same osmotic pressure.
(c) 96.2 kPa (d) 144.5 kPa
(b) 0.100 M Mg3(PO4)2(aq) has the highest
23. A 5% solution of cane sugar (molar mass 342) is osmotic pressure.
isotonic with 1% of a solution of an unknown
(c) 0.125 M Na3 PO 4 (aq) has the highest
solute. The molar mass of unknown solute in g/
osmotic pressure.
mol is : [2011RS]
(d) 0.500 M C2 H5OH(aq) has the highest
(a) 171.2 (b) 68.4
osmotic pressure.
(c) 34.2 (d) 136.2
28. The vapour pressure of acetone at 20°C is 185
24. The density of a solution prepared by dissolving torr. When 1.2 g of a non-volatile substance was
120 g of urea (mol. mass = 60 u) in 1000 g of dissolved in 100 g of acetone at 20°C, its vapour
water is 1.15 g/mL. The molarity of this solution pressure was 183 torr. The molar mass (g mol–1)
is : [2012] of the substance is : [JEE M 2015]
(a) 0.50 M (b) 1.78 M (a) 128 (b) 488
(c) 1.02 M (d) 2.05 M
(c) 32 (d) 64
25. Kf for water is 1.86 K kg mol–1. If your automobile
radiator holds 1.0 kg of water, how many grams 29. The freezing point of benzene decreases by
of ethylene glycol (C2H6O2) must you add to 0.45°C when 0.2g of acetic acid is added to 20 g
get the freezing point of the solution lowered to of benzene. If acetic acid associates to form a
–2.8ºC ? [2012] dimer in benzene, percentage association of
(a) 72 g (b) 93 g acetic acid in benzene will be : [JEE M 2017]
(c) 39 g (d) 27 g (Kf for benzene = 5.12 K kg mol –1)
26. The molarity of a solution obtained by mixing (a) 64.6% (b) 80.4%
750 mL of 0.5(M) HCl with 250 mL of 2(M) HCl will (c) 74.6% (d) 94.6%
be : [2013]
Answer Key
1 2 3 4 5 6 7 8 9 10 11 12 13 14 15
(b) (d) (d) (d) (a) (c) (d) (b) (d) (d) (d) (d) (b) (b) (d)
16 17 18 19 20 21 22 23 24 25 26 27 28 29
(a) (d) (a) (b) (b) (b) (a) (b) (d) (b) (a) (a) (d) (d)

WB DTb K b DTb
1. (b) DTb = K b ´1000 ; = =
M B ´ WA DTf Kf -0.186

WB 0.512
DTf = K f ´1000 ; = = 0.0512°C.
M B ´ WA 1.86
EBD_7764
www.crackjee.xyz
C-74 Chemistry
2. (d) In solution containing A and B component between ethanol molecules which weaken
showing negative deviation A–A and B–B intermolecular forces. This results in
interactions are weaker than that of A–B increase in vapour pressure.
interactions. For such solutions. 9. (d) DTf = K f ´ m ´ i . Since Kf has different
DH = –ve and DV = –ve
values for different solvents, hence even
3. (d) When A and B form an ideal solution,
DHmix = 0 if the m is the same DTf will be different
4. (d) DTf = Kf × m × i ; 10. (d) Given, Vapour pressure of benzene = 75
DTf = 1.85 × 0.2 × 1.3 = 0.480º C torr
\ Tf = 0 – 0.480ºC = – 0.480ºC Vapour pressure of benezene = 22 torr
+ mass of benzene in = 78g
( HX H + X - , i = 1.3) 78
1- 0.3 0. 3 0.3
hence moles of benzene = = 1mole
5. (a) NOTE On increasing pressure, the 78
(mol.wt of benzene = 78)
temperature is also increased. Thus in mass of toluence in solution = 46g
pressure cooker due to increase in pressure
46
the b.p. of water increases. hence moles of toluene = = 0.5 mole
6. (c) Q DTb° = Tb – Tbº 92
Where Tb = b.pt of solution now partial pressure of benezene
Tb° = b.pt of solvent or Tb = Tb° + DTb 1 1
= Pºb. Xb = 75 × = 50 torr = 75 ×
NOTE Elevation in boiling point is a 1 + 0.5 1.5
colligative property, which depends upon 2
the no. of particles.Thus greater the = 75 × = 50
3
number of particles, greater is it elevation
and hence greater will be its boiling point. 11. (d) Equimolar solutions of normal solutes in
the same solvent will have the same b. pts
Na2SO4 2Na + SO4 and same f. pts.
Since Na2SO4 has maximum number of d+ d-
particles (3) hence has maximum boiling 12. (d) Acetonitrile ( CH 3 - C º N ) and acetone
point. d+
7. (d) Gaseous densities of ethanol and dimethyl (CH3)
d–
ether would be same at same temperature C = O both are polar
and pressure. The heat of vaporisation, V.P. d–
and b.pts will differ due to (CH3)
H-bonding in ethanol. molecules, hence
dipole-dipole interaction exist between
8. (b) NOTE Positive deviations are shown
them. Between KCl and water ion-dipole
by such solutions in which solvent- interaction is found and in Benzene ethanol
solvent and solute-solute interactions are and Benzene–Carbon tetra chloride
stronger than the solvent interactions. In dispersion force is present
such solution, the intcractions among
molecules becomes weaker. Therefore their 13. (b) Moles of glucose = 18 = 0.1
escaping tendency increases which results 180
in the increase in their partial vapour Moles of water = 1 78 .2
= 9.9
pressures. 18
In a solutions of benzene and methanol Total moles = 0.1 + 9.9 = 10
there exists inter molecular H– bonding.
p H 2O = Mole fraction × Total pressure
9.9
= ´ 760
10
In this solution benzene molecules come = 752.4 Torr
www.crackjee.xyz
Solutions C-75

14. (b) PAo = ? , Given PBo


= 200mm , xA = 0.6, or 760 = 520X A + 1000 - 1000X A
xB = 1 – 0.6 = 0.4, P = 290 or 480X A = 240
P = PA + PB = PAo x A + PBo x B 240 1
or X A = = or 50 mol. percent
Þ 290 = PAo × 0.6 + 200 × 0.4 480 2
\ PAo
i.e., The correct answer is (d)
= 350 mm
15. (d) Let the mass of methane and oxygen = m 18. (a) NOTE On addition of glucose to water,,
gm. vapour pressure of water will decrease. The
Mole fraction of O2 vapour pressure of a solution of glucose
in water can be calculated using the relation
Moles of O2
=
Moles of O 2 + Moles of CH 4 Po - Ps Moles of glucose in solution
=
Ps moles of water in solution
m / 32 m / 32 1
= = =
m / 32 + m /16 3m / 32 3 17.5 - Ps 18/180
or = [Q Po = 17.5 ]
Partial pressure of O2 = Total pressure × Ps 178.2/18
1 1
mole fraction of O2 , PO2 = P × = P
3 3 or 17.5 – Ps = 0.1 ´ Ps or Ps = 17.325 mm Hg.
9.9
16. (a) Hence (a) is correct answer.
Osmotic pressure ( p ) of isotonic solutions 19. (b) For this solution intermolecular interactions
between n-heptane and ethanol aare
are equal. For solution of unknown weaker than n-heptane - n-heptane &
substance (p = CRT) ethanol-ethanol interactions hence the
5.25 / M solution of n-heptane and ethanol is non-
C1 = ideal and shows positive deviation from
V
Raoult’s law.
For solution of urea, C2 (concentration) =
20. (b) Ptotal = PA° X A + PB° X B
1.5 / 60
1 3
V 550 = PA° ´ + PB° ´
Given, p1 = p2 4 4
Q p = CRT PA° + 3PB° = 550 ´ 4 ...(i)
\ C1RT = C2RT or C1 = C2 In second case
5.25 / M 1.8 / 60 1 4
or = Ptotal = PA° ´ + PB° ´
V V 5 5
\ M = 210 g/mol
17. (d) At 1 atmospheric pressure the boiling point PA° + 4PB° = 560 ´ 5 ...(ii)
of mixture is 80°C. Subtract (i) from (ii)
At boiling point the vapour pressure of
mixture, PT = 1 atmosphere = 760 mm Hg. \ PB° = 560 ´ 5 - 550 ´ 4 = 600
Using the relation, Q PA° = 400
PT = PAo X A + PBo X B , we get 21. (b) Sodium sulphate dissociates as
PT = 520X A + 1000(1 - X A ) ® 2Na + + SO -4 -
Na 2SO 4 (s) ¾¾
hence van’t hoff factor i = 3
{Q PAo = 520mm Hg ,
Now D T f = i k f .m
PBo = 1000 mm Hg , X A + X B = 1 } = 3 × 1.86 × 0.01 = 0.0558 K
EBD_7764
www.crackjee.xyz
C-76 Chemistry
22. (a) PTotal = = P° A x A + P° B X B M V + M 2V2
M= 1 1 where V = total volume
V
= P°Heptane X Heptane + P°Octane X Octane 750 ´ 0.5 + 250 ´ 2
=
25 /100 35 /114 1000
= 105 ´ 25 35 + 45 ´ 25 35 = 0.875 M
+ + 27. (a) p = i CRT
100 114 100 114 p
C2 H5 OH = 1´ 0.500 ´ R ´ T = 0.5 RT
0.25 0.3
= 105 ´ + 45 ´ p
Mg3 (PO 4 )2
0.25 + 0.3 0.25 + 0.3
105 ´ 0.25 45 ´ 0.3 26.25 + 13.5
= 5 ´ 0.100 ´ R ´ T = 0.5 RT
= + =
0.55 0.55 0.55 pKBr = 2 ´ 0.250 ´ R ´ T = 0.5 RT
= 72 kPa p Na PO = 4 ´ 0.125 ´ RT = 0.5 RT
3 4
23. (b) For isotonic solutions Since the osmotic pressure of all the given
p1 = p 2 solutions is equal. Hence all are isotonic
solution.
C1 = C2 28. (d) Using relation,
5 / 342 1/ M p° - ps w 2 M1
= =
0.1 0.1 ps w1M 2
5 1 where w1, M1 = mass in g and mol. mass of
= solvent
342 M
w2, M2 = mass in g and mol. mass of
342 solute
Þ M = = 68.4 gm/mol
5 Let M2 = x
moles of solute p° = 185 torr
24. (d) Molarity =
volume of solution(l) ps = 183 torr
Mass of solution = 1000 + 120 = 1120 185 - 183
=
M M 1120 183
d = ;v = = mL (Mol. mass of acetone = 58)
v d 1.15
x = 64
120 ´ 1.15 \ Molar mass of substance = 64
= ´ 1000 = 2.05 M
60 ´ 1120 29. (d) In benzene
25. (b) DTf = i × Kf × m 2CH3COOH ƒ (CH3COOH)2
Given DTf = 2.8, Kf = 1.86 K kg mol–1 i = 1 1–a a/2
(ethylene glygol is a non- electrolyte) i = 1 – a + a/2 = 1 – a/2
Here a is degree of association
wt. of solvent = 1 kg
DTf = iKfm
Let of wt of solute = x
Mol. wt of ethylene glycol = 62 æ 0.2 ö
çè ÷
æ aö 60 ø
x 0.45 = ç1 – ÷ (5.12)
2.8 = 1 × 1.86 × è 2ø 20
62 ´ 1 1000
2.8 ´ 62 a
or x = = 93 gm 1– = 0.527
1.86 2
26. (a) From molarity equation : a = 0.945
M1V1 + M2V2 = M × V % degree of association = 94.6%
www.crackjee.xyz
Electrochemistry C-77

Electrochemistry
17
1. Conductivity (unit Siemen’s S) is directly (a) 29.5 × 10–2 (b) 10
proportional to area of the vessel and the (c) 1 × 1010 (d) 1 × 10–10
concentration of the solution in it and is 7. Standard reduction electrode potentials of three
inversely proportional to the length of the metals A, B & C are respectively + 0.5 V, – 3.0 V &
vessel then the unit of the constant of –1.2 V. The reducing powers of these metals are
proportionality is [2003]
(a) Sm mol–1 (b) Sm2 mol–1 [2002] (a) A > B > C (b) C > B > A
(c) S–2m2 mol (d) S2m2 mol–2. (c) A > C > B (d) B > C > A
2. EMF of a cell in terms of reduction potential of 8. When during electrolysis of a solution of AgNO3
its left and right electrodes is [2002] 9650 coulombs of charge pass through the
(a) E = Eleft - Eright (b) E = Eleft + Eright electroplating bath, the mass of silver deposited
(c) E = Eright - Eleft (d) E = – (Eright + Eleft). on the cathode will be [2003]
3. What will be the emf for the given cell [2002] (a) 10.8 g (b) 21.6 g
Pt | H2 (P1) | H+ (aq) | | H2 (P2) | Pt (c) 108 g (d) 1.08 g
RT P2 9. For the redox reaction : [2003]
RT P
(a) log e 1 (b) 2 F loge P Zn (s) + Cu 2 + (0.1 M ) ® Zn 2 + (1 M ) + Cu (s)
F P2 2
RT P º
taking place in a cell, E cell is 1.10 volt. Ecell for
(c) log e 2 (d) none of these.
F P1
the cell will be æç 2.303 ö
RT
4. Which of the following reaction is possible at = 0.0591÷ [2003]
anode? [2002] è F ø
(a) 2 Cr + 7H2O ® Cr2O7 + 14H
3+ 2– + (a) 1.80 volt (b) 1.07 volt
(b) F2 ® 2F – (c) 0.82 volt (d) 2.14 volt
10. In a hydrogen-oxygen fuel cell, combustion of
(c) (1/2) O2 + 2H+ ® H2O
hydrogen occurs to [2004]
(d) none of these. (a) produce high purity water
5. When the sample of copper with zinc impurity is (b) create potential difference between two
to be purified by electrolysis, the appropriate electrodes
electrodes are [2002] (c) generate heat
Cathode Anode (d) remove adsorbed oxygen from elctrode
(a) pure zinc pure copper surfaces
(b) impure sample pure copper 11. Consider the following Eº values
(c) impure zinc impure sample Eº = +0.77 V ; E º = -0.14 V
(d) pure copper impure sample. Fe3 + / Fe 2 + Sn 2 + / Sn

6. For a cell reaction involving a two-electron Under standard conditions the potential for the
change, the standard e.m.f. of the cell is found reaction
to be 0.295 V at 25ºC. The equilibrium constant
of the reaction at 25ºC will be [2003] Sn(s) + 2Fe3+ (aq) ® 2Fe 2+ (aq) + Sn 2+ (aq) is
EBD_7764
www.crackjee.xyz
C-78 Chemistry
(a) 0.91 V (b) 1.40 V [2004] To prepare 5.12 kg of aluminium metal by this
(c) 1.68 V (d) 0.63 V method we require [2005]
12. The standard e.m.f. of a cell involving one
(a) 5.49 × 101 C of electricity
electron change is found to be 0.591 V at 25ºC.
The equilibrium constant of the reaction is (b) 5.49 × 10 4 C of electricity
(F = 96,500 C mol–1; R = 8.314 JK–1 mol–1)
(a) 1.0 × 1010 (b) 1.0 × 105 [2004] (c) 1.83 × 10 7 C of electricity
(c) 1.0 × 10 1 (d) 1.0 × 1030
(d) 5.49 × 10 7 C of electricity
13. The limiting molar conductivities Lº for NaCl,
Electrolyte: KCl KNO 3 HCl NaOAc NaCl
KBr and KCl are 126, 152 and 150 S cm2 mol–1 19.
2 –1
L ¥ (S cm mol ) : 149.9 145 426.2 91 126.5
respectively. The Lº for NaBr is [2004]
(a) 278 S cm2 mol–1 (b) 176 S cm2 mol–1 ¥
Calculate L HOAc using appropriate molar
(c) 128 S cm2 mol–1 (d) 302 S cm2 mol–1 conductances of the electrolytes listed above
14. In a cell that utilises the reaction at infinite dilution in H 2 O at 25°C [2005]
Zn(s) + 2H + (aq) ® Zn 2+ (aq) + H 2 (g)
addition of H2SO4 to cathode compartment, will (a) 217.5 (b) 390.7
[2004] (c) 552.7 (d) 517.2
(a) increase the E and shift equilibrium to the o o
20. The molar conductivities L NaOAc and L HCl
right
at infinite dilution in water at 25ºC are 91.0 and
(b) lower the E and shift equilibrium to the right
426.2 S cm2/mol respectively. To calculate
(c) lower the E and shift equlibrium to the left
o
(d) increase the E and shift equilibrium to the left L HOAc , the additional value required is [2006]
15. The E º 3+ 2 + values for Cr, Mn, Fe and Co o o
M /M (a) L NaOH (b) L NaCl
are – 0.41, + 1.57, + 0.77 and + 1.97V respectively. o
o
For which one of these metals the change in (c) LH O (d) L KCl
2
oxidation state from +2 to +3 is easiest?
21. Resistance of a conductivity cell filled with a
(a) Fe (b) Mn [2004]
solution of an electrolyte of concentration 0.1
(c) Cr (d) Co M is 100 W. The conductivity of this solution is
16. For a spontaneous reaction the D G, equilibrium 1.29 S m–1. Resistance of the same cell when
filled with 0.2 M of the same solution is 520 W.
constant (K) and E oCell will be respectively
The molar conductivity of 0.2 M solution of
[2005] electrolyte will be [2006]
(a) –ve, >1, –ve (b) –ve, <1, –ve (a) 1.24 × 10–4 S m2 mol–1
(c) +ve, >1, –ve (d) –ve, >1, +ve (b) 12.4 × 10–4 S m2 mol–1
17. The highest electrical conductivity of the (c) 124 × 10–4 S m2 mol–1
following aqueous solutions is of [2005]
(d) 1240 × 10–4 S m2 mol–1
(a) 0.1 M difluoroacetic acid
22. The equivalent conductances of two strong elec-
(b) 0.1 M fluoroacetic acid trolytes at infinite dilution in H2O (where ions
(c) 0.1 M chloroacetic acid move freely through a solution) at 25°C are given
(d) 0.1 M acetic acid below : [2007]
18. Aluminium oxide may be electrolysed at 1000°C
LoCH3COONa = 91.0 S cm 2 / equiv.
to furnish aluminium metal (At. Mass = 27 amu;
1 Faraday = 96,500 Coulombs). The cathode L o HCl = 426.2 S cm 2 / equiv.
reaction is– Al 3+ + 3e- ® Al ° What additional information/ quantity one needs
www.crackjee.xyz
Electrochemistry C-79
(a) 0.385 V (b) 0.770 V
to calculate Lo of an aqueous solution of acetic
(c) –0.270 V (d) –0.072 V
acid?
27. The Gibbs energy for the decomposition of
(a) Lo of chloroacetic acid (ClCH2COOH) Al2O3 at 500°C is as follows :
(b) Lo of NaCl 2 4
Al2 O3 ® Al + O2 , D r G = + 966 kJ mol -1
3 3
(c) L o of CH3COOK
(d) the limiting equivalent coductance of The potential difference needed for electrolytic
reduction of Al2O3 at 500°C is at least [2010]
H + (l° ). (a) 4.5 V (b) 3.0 V
H+
23. The cell, (c) 2.5 V (d) 5.0 V
Zn | Zn 2+ (1 M) || Cu 2+ (1 M) | Cu (E°cell = 1.10 V)
28. The correct order of E ° 2 + values with
M /M
was allowed to be completely discharged at 298 negative sign for the four successive elements
K. The relative concentration of Zn 2+ to Cu2+ Cr, Mn, Fe and Co is [2010]
(a) Mn > Cr > Fe > Co (b) Cr < Fe > Mn > Co
æ [Zn 2+ ] ö
ç ÷ (c) Fe > Mn > Cr > Co (d) Cr > Mn > Fe > Co
ç [Cu 2+ ] ÷ is [2007]
è ø 29. Resistance of 0.2 M solution of an electrolyte is
(a) 9.65 × 104 (b) antilog (24.08) 50 W. The specific conductance of the solution
is 1.3 S m–1. If resistance of the 0.4 M solution
(c) 37.3 (d) 1037.3.
of the same electrolyte is 260 W, its molar
24. Given Eº = –0.72 V, Eº
Cr 3+ / Cr Fe2 + / Fe conductivity is : [2011RS]
–4
(a) 6.25 × 10 S m mol 2 –1
= – 0.42 V. The potential
for the cell (b) 625 × 10–4 S m2 mol–1
Cr|Cr3+ (0.1M)|| Fe2 + (0.01 M)| Fe is [2008] (c) 62.5 S m2 mol–1
(a) 0.26 V (b) 0.336 V (d) 6250 S m2 mol–1
(c) – 0.339 (d) 0.26 V 30. The standard reduction potentials for Zn2+/Zn,
25. In a fuel cell methanol is used as fuel and oxygen Ni2+/Ni and Fe2+/Fe are –0.76,–0.23 and –0.44 V
gas is used as an oxidizer. The reaction is respectively.

CH 3OH(l ) + 3/2O 2 (g) ¾¾


® The reaction X +Y 2 + ¾¾ ® X 2+ + Y will be
spontaneous when : [2012]
CO2 (g) + 2H 2O(l ) (a) X = Ni, Y = Fe (b) X = Ni, Y = Zn
At 298 K standard Gibb’s energies of formation (c) X= Fe, Y = Zn (d) X= Zn, Y = Ni
for CH3OH(l), H2O(l) and and CO2 (g) are –166.2 31. Given :
–237.2 and –394.4 kJ mol–1 respectively. If
standard enthalpy of combustion of methonal Eo 3+ = -0.74 V; Eo - = 1.51 V
Cr / Cr MnO 4 / Mn 2+
is – 726 kJ mol–1, efficiency of the fuel cell will
be: [2009] Eo 2 - 3+ = 1.33 V; Eo = 1.36 V
Cr O / Cr
2 7 Cl / Cl-
(a) 87% (b) 90%
Based on the data given above, strongest
(c) 97% (d) 80%
oxidising agent will be : [2013]
26. Given:
(a) Cl (b) Cr3+
E° 3+ = –0.036V,
Fe / Fe (c) Mn 2+ (d) MnO4 –
E° = –0.439 V 32. Four successive members of the first row
Fe 2+ / Fe
The value of standard electrode potential for transition elements are listed below with atomic
the change, numbers. Which one of them is expected to have

Fe3+ (aq) + e – ¾¾
® Fe 2+ (aq) will be: [2009]
the highest Eo 3+ 2+ value ? [2013]
M /M
EBD_7764
www.crackjee.xyz
C-80 Chemistry

( )
(a) Cr(Z = 24) (b) Mn(Z = 25)
(c) Fe(Z = 26) (d) Co(Z = 27) 2 Mn3+ + e - ® Mn 2+ ; E o = +1.51V
33. Resistance of 0.2 M solution of an electrolyte is
50 W. The specific conductance of the solution The Eo for 3Mn 2 + ® Mn + 2Mn 3+ will be:
is 1.4 S m–1. The resistance of 0.5 M solution of [2014]
the same electrolyte is 280 W. The molar (a) –2.69 V; the reaction will not occur
conductivity of 0.5 M solution of the electrolyte (b) –2.69 V; the reaction will occur
in S m2 mol–1 is: [2014] (c) –0.33 V; the reaction will not occur
(a) 5 × 10–4 (b) 5 × 10–3 (d) –0.33 V; the reaction will occur
(c) 5 × 103 (d) 5 × 102 36. Two Faraday of electricity is passed through a
34. The equivalent conductance of NaCl at solution of CuSO 4 . The mass of copper
concentration C and at infinite dilution are lC deposited at the cathode is
(at. mass of Cu = 63.5 amu) [JEE M 2015]
and l¥ , respectively. The correct relationship (a) 2g (b) 127 g
between lC and l¥ is given as: (c) 0 g (d) 63.5 g
(Where the constant B is positive) [2014] 37. Given [JEE M 2017]
(a) lC = l ¥ + ( B ) C Eo
Cl /Cl –
= 1.36V, E o 3 +
Cr / Cr
= –0.74V,
2
(b) lC = l ¥ - ( B ) C E o
= 1.33V, Eo = 1.51V.
Cr /O2– /Cr3+ – /Mn 2 +
MnO4
2 7
(c) lC = l¥ - ( B ) C
Among the following, the strongest reducing
(d) lC = l¥ + ( B ) C agent is
(a) Cr (b) Mn 2+
35. Given below are the half-cell reactions:
(c) Cr3+ (d) Cl–
Mn 2 + + 2e - ® Mn ; E o = -1.18V

Answer Key
1 2 3 4 5 6 7 8 9 10 11 12 13 14 15
(b) (c) (b) (a) (d) (c) (d) (a) (b) (b) (a) (a) (c) (a) (c)
16 17 18 19 20 21 22 23 24 25 26 27 28 29 30
(d) (a) (d) (b) (b) (b) (b) (d) (d) (c) (b) (c) (a) (a) (d)
31 32 33 34 35 36 37
(d) (d) (a) (c) (a) (d) (a)

3. (b) Oxidation half call:-


area ´ conc km 2 mol
1. (b) given S µ = H2(g) ––––––– ®– 2H+(1M) + 2e–
l m ´ m3 P1
\k = Sm2 mol-1 Reduction half cell
2H+ (1M) + 2e– ––––––– ®
– H2(g)
2. (c) Ecell = Reduction potential of cathode
P2
(right)
The net cell reaction
– reduction potential of anode (left)
H2(g) ––––––– ®– H2(g)
= Eright – Eleft.
P1 P2
www.crackjee.xyz
Electrochemistry C-81
E ºcell = 0.00 V n=2
11. (a) Fe3+ + e - ® Fe2+ DG° = -1´ F ´ 0.77
RT
\ Ecell = Eºcell – logeK
nF
Sn 2+ + 2e - ® Sn(s) DG° = -2 ´ F(-0.14)
RT P
=0– log e 2
nF P1 for Sn(s) + 2Fe3+ (aq) ®
RT P2 2Fe 2 + (aq) + Sn 2 + (aq)
or Ecell = log e P
2F 1 \ Standar d potential for the given
reaction
4. (a) 2Cr 3+ + 7 H 2 O ® Cr2 O 72 - + 14H +
O.S. of Cr changes from +3 to +6 by loss of or E ocell = E o + Eo
electrons. At anode oxidation takes place. Sn / Sn 2 + Fe3+ / Fe 2 +
5. (d) Pure metal always deposits at cathode. = 0.14 + 0.77 = 0.91 V
6. (c) The equilibrium constant is related to the 0.059
standard emf of cell by the expression 12. (a) E°cell = Eocell – log K c
n
n 2
log K = Eº cell ´ = 0.295 ´ 0.0591
0.059 0.059 or 0 = 0.591 – log K c
1
590
log K = = 10 or K = 1 × 1010 or log K c =
0.591
= 10 or Kc = 1 ´ 1010
59 0.0591
7. (d) A B C
+0.5C –3.0V –1.2V 13. (c) L º NaCl = l° Na + + lCl - ....(i)

NOTE The higher the negative value of L° KBr = l°K + + l°Br - ....(ii)
reduction potential, the more is the
reducing power. L° KCl = l°K + + lCl- ....(iii)
Hence B > C > A. operating (i) + (ii) - (iii)
8. (a) When 96500 coulomb of electricity is
passed through the electroplating bath the L° NaBr = l° Na + + l º Br –
amount of Ag deposited = 108g
= 126 + 152 - 150 = 128 S cm 2 mol -1
\ when 9650 coulomb of electricity is
passed deposited Ag. 14. (a) Zn(s) + 2H + + (aq) Zn 2+ (aq) + H 2 (g)
108
= ´ 9650 = 10.8 g 0.059 [Zn 2 + ][H 2 ]
96500 E cell = E °cell - log
2 [H + ]2
0.059 [Cu +2 ] Addition of H2SO4 will increase [H+]and
9. (b) E cell = E º cell + log
n [ Zn + 2 ] Ecell will also increase and the equilibrium
will shift towards RHS
0.059 15. (c) The given values show that Cr has
= 1.10 + log[0.1]
2 maximum oxidation potental, therefore its
= 1.10 - 0.0295 = 1.07 V oxidation will be easiest. (Change the sign
to get the oxidation values)
10. (b) In H 2 - O 2 fuel cell, the combustion of
H2 occurs to create potential difference 16. (d) NOTE For spontaneous reaction DG
between the two electrodes should be negative. Equilibrium constant
should be more than one
EBD_7764
www.crackjee.xyz
C-82 Chemistry
(DG = – 2.303 RT log Kc, If Kc = 1 then m (molar conductivity) = ?
DG = 0; If Kc < 1 m = k × V (k can be calculated as
1 æ1ö
then DG = +ve). Again DG = - nFE ocell . k= now
R çè a ÷ø
E ocell must be +ve to have DG –ve. cell constant is known.)
Hence,
17. (a) Thus difluoro acetic acid being strongest
1 1000
acid will furnish maximum number of ions m= ´ 129 ´ ´ 10 -6 m 3
showing highest electrical conductivity. 520 0.2
The decreasing acidic strength of the = 12.4 ×10–4 Sm2 mol–1
carboxylic acids given is
22. (b) NOTE According to Kohlrausch’s law,,
difluoro acetic acid > fluoro acetic acid
> chloro acetic acid > acetic acid. molar conductivity of weak electrolyte
18. (d) 1 mole of e– = 1F = 96500 C acetic acid (CH3COOH) can be calculated
as follows:
27g of Al is deposited by 3 × 96500 C
5120 g of Al will be deposited by (
L oCH3COOH = L oCH3COONa + L oHCl - L oNaCl )
=
3 ´ 96500 ´ 5120
= 5.49 ´ 10 7 C \ Value of L o NaCl should also be known
27
for calculating value of LoCH 3COOH .
23. (d) E cell = 0; when cell is completely
mFz
q=
We know, Q discharged.
M
5120 ´ 96500 ´ 3 æ é Zn 2 + ù ö
0.059
q=
\Q = 5.49 ´ 107 C Ecell = E°cell - log ç ë û÷
27 ç + ÷
2 ç é Cu ù ÷
2
èë ûø
19. (b) L¥
HCl = 426.2 (i)
æ é Zn 2 + ù ö

AcONa = 91.0 (ii) 0.059
or 0 = 1.1 - log ç ë û÷
2 ç é 2+ ù ÷
ç Cu ÷

NaCl = 126.5 (iii) èë ûø
æ é Zn 2+ ù ö
¥
L AcOH = (i) + (ii) - (iii) log ç ë û ÷ = 2 ´1.1 = 37.3
ç é 2+ ù ÷ 0.059
ç Cu ÷
= [426.2 + 91.0 - 126.5] = 390.7 èë ûø

20. (b) o
L CH is given by the following æ é Zn 2 + ù ö
3COOH çë û ÷ = 1037.3
\ ç 2+ ù ÷
equation é
ç Cu ÷
èë ûø
LoCH COOH
3 (o
= L CH
3COONa )(
+ L oHCl - L oNaCl ) 24. (d) From the given representation of the cell,
o
Ecell can be found as follows.
Hence L NaCl is required.
0.059 [Cr3+ ]
2
1ælö Ecell = Eo - Eo - log
21. (b) R = 100 W , k = ç ÷ , Fe 2+
/ Fe Cr 3+
/ Cr 6 [Fe2+ ]3
R è aø
l [Nernst -Equ.]
(cell constant) = 1.29 × 100m–1
a 0.059 (0.1)2
Given, R = 520W, C = 0.2 M, = –0.42 – (–0.72) - log
6 (0.01)3
www.crackjee.xyz
Electrochemistry C-83
0.059 0.1´ 0.1 \ The potential difference needed for the
= –0.42 + 0.72 - log reduction = 2.5 V.
6 0.01´ 0.01´ 0.01
0.059 10-2 0.059 28. (a) The value of E o for given metal ions
= 0.3 - log = 0.3 - ´4 M 2∗ M
6 -6 6
10 are
= 0.30 – 0.0393 = 0.26 V Eo < ,1.18 V,
Hence option (d) is correct answer. Mn 2∗ Mn

3 Eo < ,0.9 V,
25. (c) CH3OH (l) + O (g) ® CO2 (g) + 2H2O Cr 2∗ Cr
2 2
(l) Eo < ,0.44 V and
Fe 2∗ Fe
DG r = DG f (CO 2 , g) + 2DG f (H 2 O, l) -
Eo < ,0.28 V.
3 Co 2∗ Co
DG f (CH3OH, l ) – DG f (O2 ,g)
2 The correct order of E o 2∗ values
= – 394.4 + 2 (–237.2) – (–166.2) – 0 M M
= – 394.4 – 474.4 + 166.2 = – 702.6 k J without considering negative sign would
be
702.6
% efficiency = ´ 100 = 97% Mn2+ > Cr2+ > Fe2+ > Co2+.
726
26. (b) Given 1 l
29. (a) k= ´
R A
Fe3+ + 3e - ® Fe ,
1 l
E° 3+ = –0.036 V … (i) 1.3 = ´
Fe / Fe 50 A
Fe 2 + + 2e - ® Fe , l
= 65m-1
E° = –0.439V … (ii) A
Fe 2+ / Fe
we have to calculate k ´ 1000
L=
molarity
Fe3+ + e - ® Fe2 + , DG = ? [molarity is in moles/litre but 1000 is used
To obtain this equation subtract equ (ii) to convert liter into cm3]
from (i) we get
æ 1 ö
çè ´ 65 m -1 ÷ ´ 1000 cm 3
Fe3+ + e – ® Fe 2 + … (iii) 260 ø
=
As we know that DG = –nFE 0.4 moles
Thus for reaction (iii) 650 m -1 1
DG = DG1 - DG = ´ m3
260 ´ 4 mol 1000
– nFE° = – nFE1 – (–nFE2) = 6.25 × 10–4 S m2 mol–1
–nFE° = nFE2 – nFE1 30. (d) For a spontaneous reaction DG must be –ve
–1FE° = 2× 0.439F – 3 × 0.036 F
Since DG = – nFE°
–1 FE° = 0.770 F
\ E° = – 0.770V Hence for DG to be -ve DE° has to be
positive. Which is possible when X = Zn,
O -- > F- > Na + > Mg ++ > Al3+ Y = Ni
27. (c) DG = – nFE Zn + Ni++ ¾¾
® Zn++ + Ni
ΧG 966´10 3
E° + E°
or E < < < ,2.5 V Zn / Zn +2 Ni 2+ / Ni
,nF 4´96500
EBD_7764
www.crackjee.xyz
C-84 Chemistry
= 0.76 + (– 0.23) = + 0.53 (positive)
2.5 ´ 10-3 ´ 1000
31. (d) Higher the value of standard reduction =
potential, stronger is the oxidising agent, 0.5
hence MnO4– is the strongest oxidising = 5 S cm2 mol–1 = 5 × 10–4 S m2 mol–1
agent. 34. (c) According to Debye Huckle onsager
equation,
E° = – 0.41 V E°Fe3+ / Fe 2 + = +
32. (d) Cr 3+ / Cr 2 +
lC = l¥ - B C
0.77 V
35. (a) (a)
E ° 3+ = + 1.57 V,,
Mn / Mn 2 +
Mn 2+ + 2e- ® Mn; E o = -1.18V ; ...(i)
E° = + 1.97 V
Co3+ / Co 2 +
33. (a) Given for 0.2 M solution (b) Mn 3+ + e ® Mn 2+ ; E o = -1.51V ; ...(ii)
R = 50 W Now multiplying equation (ii) by two and
k = 1.4 S m–1 = 1.4 × 10–2 S cm–1 subtracting from equation (i)
l 1 l 3Mn 2+ ® Mn + + 2Mn 3+ ;
Now, R = r = ´
a k a
l E o = EOx. + ERed.
-2
Þ = R ´ k = 50 ´1.4 ´ 10
a = – 1.18 + (– 1.51) = – 2.69 V
For 0.5 M solution (–ve value of EMF (i.e. DG = +ve) shows
R = 280 W that the reaction is non-spontaneous)
k =? 36. (d) Cu 2+ + 2e – ¾¾
® Cu
l
= 50 ´ 1.4 ´ 10-2 2F i.e. 2 × 96500 C deposit Cu = 1 mol = 63.5g
a
37. (a) E° – / Mn 2 + = 1.51V
l 1 l MnO 4
Þ R=r = ´
a k a E° = 1.36V
Cl2 /Cl –
1
Þ k= ´ 50 ´ 1.4 ´ 10 -2
280 E° = 1.33V
Cr2 O72 - /Cr3+
1
= ´ 70 ´ 10-2
280 E° = – 0.74
Cr3+ / Cr
= 2.5 × 10–3 S cm–1
Since Cr 3+ is having least reducing
k ´1000
Now, L m = potential, so Cr is the best reducing agent.
M
www.crackjee.xyz
Chemical Kinetics C-85

Chemical Kinetics
18
1. Units of rate constant of first and zero order 6. The half-life of a radioactive isotope is three
reactions in terms of molarity M unit are hours. If the initial mass of the isotope were 256
respectively [2002] g, the mass of it remaining undecayed after 18
(a) sec–1 , Msec–1 (b) sec–1, M hours would be [2003]
(c) Msec–1 , sec–1 (d) M, sec–1. (a) 8.0 g (b) 12.0 g
2. For the reaction A + 2B ® C, rate is given by (c) 16.0 g (d) 4.0 g
R = [A] [B]2 then the order of the reaction is 7. In respect of the equation k = Ae - E a / RT in
chemical kinetics, which one of the following
[2002]
statements is correct ? [2003]
(a) 3 (b) 6
(a) A is adsorption factor
(c) 5 (d) 7.
(b) Ea is energy of activation
3. The differential rate law for the reaction (c) R is Rydberg’s constant
H2 + I2 ® 2HI is [2002] (d) k is equilibrium constant
d[ H 2 ] d[ I 2 ] d[ HI ] 8. For the reaction system : [2003]
(a) - =- =- 2NO(g) + O 2 (g) ® 2 NO 2 (g) volume is
dt dt dt
suddenly reduced to half its value by increasing
d[H 2 ] d[I 2 ] 1 d[Hl ] the pressure on it. If the reaction is of first order
(b) = = with respect to O2 and second order with respect
dt dt 2 dt
to NO, the rate of reaction will
1 d[H 2 ] 1 d[I 2 ] d[Hl] (a) diminish to one-eighth of its initial value
(c) = =-
2 dt 2 dt dt (b) increase to eight times of its initial value
(c) increase to four times of its initial value
d[H 2 ] d[I 2 ] d[HI]
(d) -2 = -2 = (d) diminish to one-fourth of its initial value
dt dt dt 9. In a first order reaction, the concentration of the
4. If half-life of a substance is 5 yrs, then the total reactant, decreases from 0.8 M to 0.4 M is 15
amount of substance left after 15 years, when minutes. The time taken for the concentration to
initial amount is 64 grams is [2002] change from 0.1 M to 0.025 M is [2004]
(a) 16 grams (b) 2 grams (a) 7.5 minutes (b) 15 minutes
(c) 32 grams (d) 8 grams. (c) 30 minutes (d) 60 minutes
5. The integrated rate equation is [2002] 10. The rate equation for the reaction 2A + B ® C is
Rt = log C0 - logCt. found to be: rate = k[A][B]. The correct statement
The straight line graph is obtained by plotting in relation to this reaction is that the [2004]
(a) rate of formation of C is twice the rate of
1 disappearance of A
(a) time vs logCt (b) vs C
time t
(b) t1 / 2 is a constant
1 1 (c) unit of k must be s–1
(c) time vs Ct (d) vs (d) value of k is independent of the initial
time Ct
concentrations of A and B
EBD_7764
www.crackjee.xyz
C-86 Chemistry
11. The half-life of a radioisotope is four hours. If the 16. A reaction was found to be second order with
initial mass of the isotope was 200 g, the mass respect to the concentration of carbon monoxide.
remaining after 24 hours undecayed is [2004] If the concentration of carbon monoxide is
(a) 3.125 g (b) 2.084 g doubled, with everything else kept the same,
(c) 1.042 g (d) 4.167 g the rate of reaction will [2006]
12. A reaction involving two different reactants (a) increase by a factor of 4
can never be [2005] (b) double
(a) bimolecular reaction (c) remain unchanged
(b) second order reaction
(c) first order reaction (d) triple
(d) unimolecular reaction 17. Rate of a reaction can be expressed by Arrhenius
equation as : [2006]
13. A schematic plot of ln K eq versus inverse of
k = A e–E/RT
temperature for a reaction is shown below [2005]
In this equation, E represents
6.0 (a) the total energy of the reacting molecules
at a temperature, T
ln Keq

(b) the fraction of molecules with energy


greater than the activation energy of the
reaction
2.0
1.5 ´ 10 - 3 1 (K -1 ) 2.0 ´ 10 - 3
(c) the energy above which all the colliding
T molecules will react
The reaction must be (d) the energy below which all the colliding
(a) highly spontaneous at ordinary temperature molecules will react
(b) one with negligible enthalpy change 18. The following mechanism has been proposed
(c) endothermic for the reaction of NO with Br 2 to form NOBr :
(d) exothermic NO(g) + Br2(g) NOBr2(g)
14. t 1 can be taken as the time taken for the NOBr 2 (g ) + NO (g ) ¾
¾® 2 NOBr ( g )
4
If the second step is the rate determining step,
3 the order of the reaction with respect to NO(g)
concentration of a reactant to drop to of its
4 is [2006]
initial value. If the rate constant for a first order (a) 3 (b) 2
t1 (c) 1 (d) 0
reaction is K, the can be written as 19. The energies of activation for forward and reverse
4
reactions for A2 + B2 ƒ 2AB are 180 kJ mol–1
(a) 0.75/K (b) 0.69/K [2005]
and 200 kJ mol–1 respectively. The presence of a
(c) 0.29/K (d) 0.10/K
catalyst lowers the activation energy of both
15. Consider an endothermic reaction X ® Y with
(forward and reverse) reactions by 100 kJ mol–1.
the activation energies E b and E f for the The enthalpy change of the reaction
backward and forward reactions, respectively. (A2 + B2 ® 2AB) in the presence of a catalyst
In general [2005] will be (in kJ mol–1) [2007]
(a) there is no definite relation between (a) 20 (b) 300
E b and E f (c) 120 (d) 280
(b) E b = Ef 20. Consider the reaction, 2A + B ® products.
When concentration of B alone was doubled,
(c) E b > Ef the half-life did not change. When the
concentration of A alone was doubled, the rate
(d) Eb < Ef
www.crackjee.xyz
Chemical Kinetics C-87
increased by two times. The unit of rate constant 26. A reactant (A) froms two products : [2011RS]
for this reaction is [2007] k
A ¾¾
1 ® B, Activation Energy Ea
(a) s – 1 (b) L mol–1 s–1 k
1
(c) no unit (d) mol L–1 s–1. A ¾¾®
2
C, Activation Energy Ea2
21. A radioactive element gets spilled over the floor If Ea2 = 2 Ea1, then k1 and k2 are related as :
of a room. Its half-life period is 30 days. If the (a) k2 = k1e Ea1 / RT (b) k2 = k1e Ea2 / RT
initial velocity is ten times the permissible value,
after how many days will it be safe to enter the (c) k1 = Ak2e Ea1 / RT (d) k1 = 2k2e Ea2 / RT
room? [2007] 27. For a first order reaction (A) ® products the
(a) 100 days (b) 1000 days concentration of A changes from 0.1 M to 0.025
(c) 300 days (d) 10 days. M in 40 minutes.
The rate of reaction when the concentration of
1 A is 0.01 M is : [2012]
22. For a reaction A ® 2B, rate of disappearance
2 (a) 1.73 × 10–5 M/min (b) 3.47 × 10–4 M/min
of ‘A’ is related to the rate of appearance of ‘B’ (c) 3.47 × 10–5 M/min (d) 1.73 × 10–4 M/min
by the expression [2008] 28. The rate of a reaction doubles when its
d[A] 1 d[B] d[A] 1 d[B] temperature changes from 300 K to 310 K.
(a) – = (b) – = Activation energy of such a reaction will be : (R
dt 2 dt dt 4 dt = 8.314 JK–1 mol–1 and log 2 = 0.301) [2013]
d[A ] d[B] d[A] d[B] (a) 53.6 kJ mol –1 (b) 48.6 kJ mol–1
(c) – = (d) – =4 –1
dt dt dt dt (c) 58.5 kJ mol (d) 60.5 kJ mol–1
23. The half life period of a first order chemical 29. For the non - stoichiometric reaction
reaction is 6.93 minutes. The time required for 2A + B ® C + D, the following kinetic data
the completion of 99% of the chemical reaction were obtained in three separate experiments, all
will be (log 2 = 0.301) [2009] at 298 K.
(a) 23.03 minutes (b) 46.06 minutes
(c) 460.6 minutes (d) 230.03 minutes Initial Initial In itial rate of
Con centration Concentration formation of C
24. The time for half life period of a certain reaction –1 –1
A ¾¾ ® Products is 1 hour. When the initial (A ) (B ) (mol L s )
concentration of the reactant ‘A’, is 2.0 mol L–1, 0.1 M 0.1 M 1.2 × 10
–3

how much time does it take for its concentration 0.1 M 0.2 M 1.2 × 10 –3
to come from 0.50 to 0.25 mol L–1 if it is a zero –3
order reaction ? [2010] 0.2 M 0.1 M 2.4 × 10
(a) 4 h (b) 0.5 h
(c) 0.25 h (d) 1 h The rate law for the formation of C is: [2014]
25. Consider the reaction : dc dc
= k [ A][ B ] = k [ A] [ B ]
2

Cl 2 (aq) + H 2S(aq) ® (a) (b)


dt dt
S(s) + 2H + (aq) + 2Cl - (aq)
dc dc
The rate equation for this reaction is = k [ A][ B ] = k [ A]
2
(c) (d)
rate = k[Cl 2 ][H 2S] dt dt
Which of these mechanisms is/are consistent 30. Higher order (>3) reactions are rare due to :
with this rate equation? [2010] [JEE M 2015]
+ - +
A. Cl 2 + H 2S ® H + Cl + Cl + HS (slow) -
(a) shifting of equilibrium towards reactants
Cl + + HS- ® H + + Cl- + S (fast) due to elastic collisions
(b) loss of active species on collision
B. H 2S ƒ H + + HS- (fast equilibrium) (c) low probability of simultaneous collision
Cl 2 + HS- ® 2Cl - + H + + S (Slow) of all the reacting species
(a) B only (b) Both A and B (d) increase in entropy and activation energy
(c) Neither A nor B (d) A only as more molecules are involved
EBD_7764
www.crackjee.xyz
C-88 Chemistry
31. Decomposition of H2O2 follows a first order 32. Two reactions R1 and R2 have identical pre-
reaction. In fifty minutes the concentration of exponential factors. Activation energy of R1
H2O2 decreases from 0.5 to 0.125 M in one such
exceeds that of R2 by 10 kJ mol–1. If k1 and k2
decomposition. When the concentration of
are rate constants for reactions R1 and R2
H2O2 reaches 0.05 M, the rate of formation of
O2 will be: [JEE M 2016] respectively at 300 K, then ln(k2/k1) is equal to :
–1
(a) 2.66 L min at STP (R = 8.314 J mol–1K–1) [JEE M 2017]
(b) 1.34 × 10–2 mol min–1 (a) 8 (b) 12
(c) 6.96 × 10–2 mol min–1 (c) 6 (d) 4
(d) 6.93 × 10–4 mol min–1

Answer Key
1 2 3 4 5 6 7 8 9 10 11 12 13 14 15
(a) (a) (d) (d) (a) (d) (b) (b) (c) (d) (a) (d) (d) (c) (d)
16 17 18 19 20 21 22 23 24 25 26 27 28 29 30
(a) (c) (b) (a) (b) (a) (b) (b) (c) (d) (c) (b) (a) (d) (c)
31 32
(d) (d)

1. (a) For a zero order reaction.


2d [ H 2 ] 2d [ I2 ] d [ HI]
rate =k[A]º i.e. rate = k or – =- =
hence unit of k = M.sec–1 dt dt dt
For a first order reaction. 4. (d) t1/2 = 5 years, T = 15 years hence total num-
rate = k [A] ber of half life periods = 15 = 3 .
k = M.sec–1/M = sec–1 5
2. (a) NOTE Order is the sum of the power of \ Amount left = 64 = 8g
the concentrations terms in rate law ( 2) 3
expression. 5. (a) Rt = log Co – log Ct
Hence the order of reaction is = 1 + 2 = 3 It is clear from the equation that if we plot a
graph between log Ct and time, a straight
1 d [ HI]
3. (d) rate of appearance of HI = k
2 dt line with a slope equal to – and
2.303
-d éë H 2 ùû intercept equal to log [Ao] will be obtained.
rate of formation of H2 =
dt 6. (d) t1/2 = 3hrs. T = 18 hours
Q T = n ´ t1/ 2
-d [ I 2 ]
rate of formation of I2 = 18
dt
\n= =6
3
-d [ H 2 ] d [ I2 ] 1 d [ HI] Initial mass (C0) = 256 g
hence =- =
dt dt 2 dt
www.crackjee.xyz
Chemical Kinetics C-89

C0 256 256 2.303 1 2.303 4


\ Cn = = = = 4g. 14. (c) t1/ 4 = log = log
n 6 64 K 3/ 4 K 3
2 ( 2)

7. (b) In equation k = Ae - E a / RT ; 2.303


A = Frequency factor = (log 4 - log 3)
K
k = velocity constant, R = gas constant
and Ea = energy of activation 2.303
= (2 log 2 - log 3)
8. (b) r = k [O2][NO]2. When the volume is K
reduced to 1/2, the conc. will double 2.303 0.29
\ New rate = k [2O2][2 NO]2 = 8 k [O2][NO]2 = (2 ´ 0.301 - 0.4771) =
K K
The new rate increases to eight times of its 15. (d) Enthalpy of reaction (DH) = Ea – Ea
initial. (f) (b)
9. (c) As the concentration of reactant for an endothermic reaction DH = +Ve
decreases from 0.8 to 0.4 in 15 minutes hence for DH to be negative
Ea < Ea
(b) (f)
hence the t1 / 2 is 15 minutes. To fall the
16. (a) Since the reaction is 2nd order w.r.t CO.
concentration from 0.1 to 0.025 we need Thus, rate law is given as.
two half lives i.e., 30 minutes. r = k [CO]2
10. (d) The velocity constant depends on Let initial concentration of CO is a i.e. [CO]
temperature only. It is independent of =a
concentration of reactants. \ r1 = k (a)2 = ka2
n when concentration becomes doubled,
æ 1ö
11. (a) Nt = N 0 ç ÷ where n is number of half i.e.[CO] = 2a
è 2ø
\ r2 = k (2a)2 = 4ka2 \ r2 = 4r1
life periods. So, the rate of reaction becomes 4 times.
Total time 24 17. (c) In Arrhenius equation k = A e–E/RT, E is the
n= = =6 energy of activation, which is required by
half life 4
the colliding molecules to react resulting
6 in the formation of products.
æ 1ö
\ N t = 200 ç ÷ = 3.125g . 18. (b) (i) NO(g) + Br2(g) NOBr2(g)
è 2ø
12. (d) The molecularity of a reaction is the number (ii) NOBr2 (g ) + NO (g ) ¾
¾® 2 NOBr ( g )
of reactant molecules taking part in a single Rate law equation = k[NOBr2] [NO]
step of the reaction. But NOBr2 is intermediate and must not
appear in the rate law equation
NOTE The reaction involving two
[NOBr2 ]
different reactant can never be unimolecular. from Ist step K C =
13. (d) The graph show that reaction is [NO] [Br2 ]
exothermic. \ [NOBr2] = KC [NO] [Br2]
\ Rate law equation = k . KC [NO]2 [Br2]
-DH
log k = +1 hence order of reaction is 2 w.r.t. NO.
RT
19. (a) DH R = E f - E b = 180 – 200 = – 20 kJ/mol
For exothermic reaction DH < 0
The nearest correct answer given in
1 choices may be obtained by neglecting
\ log k Vs would be negative straight sign.
T
line with positive slope.
EBD_7764
www.crackjee.xyz
C-90 Chemistry
0.693 0.693 2.303 ´ 2
20. (b) For a first order reaction t1/ 2 = i.e. =
K 6.93 t
t = 46.06 min
for a first order reaction t1/ 2 does not
24. (c) For the reaction
depend up on the concentration. From the A ® Product
given data, we can say that order of
given t1/ 2 = 1 hour
reaction with respect to B = 1 because
change in concentration of B does not for a zero order reaction
change half life.
tcompletion =
[ A0 ] = initial conc.
Order of reaction with respect to A = 1 k rate constant
because rate of reaction doubles when
concentration of B is doubled keeping \ t1 / 2 =
[ A0 ]
concentration of A constant. 2k
\ Order of reaction = 1 + 1 = 2 and units of
second order reaction are L mol–1 sec–1. or k =
[ A0 ] =
2
= 1 mol lit –1 hr–1
21. (a) Suppose activity of safe working = A 2 t1/ 2 2 ´1
Given A0 = 10A Further for a zero order reaction
0.693 0.693 dx change in concentration
l= = k= =
t1/ 2 30 dt time
0.50 - 0.25
2.303 A 2.303 10A 1=
t½ = log 0 = log time
l A 0.693 / 30 A
\ time = 0.25 hr.
2.303 ´ 30 25. (d) Since the slow step is the rate determining
= ´ log10 = 100 days.
0.693 step hence if we consider option (1) we
find
22. (b) The rates of reactions for the reaction
Rate = k [ Cl2 ][ H 2S]
1
A ¾¾ ® 2B Now if we consider option (2) we find
2
Rate = k [ Cl2 ] éë HS ùû
can be written either as -
...(1)
d From equation (i)
-2 [A] with respect to ‘A’’
dt
é H + ù éHS- ù
1 d ë ûë û
or [B] with respect to ‘B’ k=
2 dt H 2S
From the above, we have
- k [ H 2 S]
d 1 d or éë HS ùû =
–2 [A] = [B] H+
dt 2 dt Substituting this value in equation (1) we
d 1 d find
or – [A] = [B]
dt 4 dt
Rate = k [ Cl2 ] K
[ H 2S] = k ' [ Cl2 ][ H 2S]
i.e., correct answer is (b) H+éH+ ù
23. (b) For first order reaction, ë û
hence only , mechanism (1) is consistent
2.303 100
k= log with the given rate equation.
t 100 - 99
26. (c) k1 = A1e - Ea1 / RT .........(i)
0.693 2.303 100
= log
6.93 t 1 k2 = A2 e- Ea2 / RT ........(ii)
www.crackjee.xyz
Chemical Kinetics C-91
On dividing eqn (i) from eqn. (ii) We find, x = 1
k1 A d [C ]
= 1 ( Ea1 - Ea1 ) / RT Hence = k[ A]1 [ B ]0
k 2 A2 dt
........(iii) 30. (c) Reactions of higher order (>3) are very rare due
to very less chances of many molecules to
Given Ea2 = 2 Ea1 undergo effective collisions.
On substituting this value in eqn. (iii) 1
31. (d) H2O2(aq) ® H2O(aq) + O (g)
Ea / RT 2 2
k1 = k 2 A ´ e 1
For a first order reaction
27. (b) For a first order reaction
2.303 a
2.0303 a 2.303 0.1 k= log
k= log = log t (a - x)
t a-x 40 0.025 Given a = 0.5, (a – x) = 0.125, t = 50 min
2.303 2.303 ´ 0.6020 2.303 0.5
= log 4 = \ k= log
40 40 50 0.125
= 2.78 ´ 10–2 min–1
= 3.47 ´ 10 –2 r = k[H2O2] = 2.78 ´ 10–2 ´ 0.05
R = k (A)1 = 3.47 × 10–2 × 0.01 = 1.386 ´ 10–3 mol min–1
= 3.47 × 10–4 Now
28. (a) Activation energy can be calculated from d [ H 2 O2 ] d [ H2 O] 2d [ O2 ]
the equation - = =
dt dt dt
log k2 - Ea æ 1 1 ö
= - 2d [ O2 ] d [ H 2 O2 ]
log k1 2.303 R çè T2 T1 ÷ø \ = -
dt dt

Given
k2
= 2 ; T2 = 310 K ; T1 = 300 K d [ O2 ] 1 d[H 2 O 2 ]
k1 \ = ´
dt 2 dt

= log 2 =
- Ea æ 1
-
1 ö 1.386 ´10-3
ç ÷ = = 6.93 ´ 10–4 mol min–1
2.303 ´ 8.314 è 310 300 ø 2
Ea = 53598.6 J/mol = 53.6 kJ/mol. 32. (d) From arrhenius equation,
d [C ] – Ea
29. (d) Let rate of reaction = = k[A]x [B]y k = A.e RT
t
Now from the given data –E a /RT
so, k1 = A.e 1 .....(1)
1.2 × 10 – 3 = k [0.1]x[0.1]y .....(i)
1.2 × 10 – 3 = k [0.1]x[0.2]y .....(ii) –E a / RT
k 2 = A.e 2 .....(2)
2.4 × 10 – 3 = k [0.2]x[0.1]y .....(iii)
Dividing equation (i) by (ii) On dividing equation (2) (1)
-3 x y
1.2 ´10 k[0.1] [0.1]
Þ = (E a – Ea )
-3
1.2 ´10 k [0.1]x [0.2] y k 1 2
Þ 2 =e RT
We find, y = 0 k1
Now dividing equation (i) by (iii)
æ k ö Ea – Ea 2 10, 000
1.2 ´10-3 k[0.1]x [0.1] y ln ç 2 ÷ = 1 = =4
Þ = è k1 ø RT 8.314 ´ 300
2.4 ´10 -3 k [0.2] x [0.1] y
EBD_7764
www.crackjee.xyz
C-92 Chemistry

19
Surface Chemistry

1. The formation of gas at the surface of tungsten (b) Mixing the sols has no effect
due to adsorption is the reaction of order [2002] (c) Sodium sulphate solution causes
(a) 0 (b) 1 coagulation in both sols
(c) 2 (d) insufficient data. (d) Magnesium chloride solution coagulates,
the gold sol more readily than the iron (III)
2. Which one of the following characteristics is
hydroxide sol
not correct for physical adsorption ? [2003] 6. In Langmuir's model of adsorption of a gas on a
(a) Adsorption increases with increase in solid surface [2006]
temperature (a) the mass of gas striking a given area of
(b) Adsorption is spontaneous surface is proportional to the pressure of
(c) Both enthalpy and entropy of adsorption the gas
are negative (b) the mass of gas striking a given area of
(d) Adsorption on solids is reversible surface is independent of the pressure of
3. Identify the correct statement regarding enzymes the gas
[2004] (c) the rate of dissociation of adsorbed
(a) Enzymes are specific biological catalysts molecules from the surface does not
that cannot be poisoned depend on the surface covered
(d) the adsorption at a single site on the surface
(b) Enzymes are normally heterogeneous
may involve multiple molecules at the same
catalysts that are very specific in their
time
action
7. Gold numbers of protective colloids A, B, C and D
(c) Enzymes are specific biological catalysts
are 0.50, 0.01, 0.10 amd 0.005, respectively. the
that can normally function at very high
correct order of their protective powers is [2008]
temperatures (T~1000K) (a) D < A < C < B (b) C < B < D < A
(d) Enzymes are specific biological catalysts (c) A < C < B < D (d) B < D < A < C
that possess well-defined active sites 8. Which of the following statements is incorrect
4. The volume of a colloidal particle, VC as regarding physissorptions? [2009]
compared to the volume of a solute particle in a (a) More easily liquefiable gases are adsorbed
readily.
true solution VS , could be [2005]
(b) Under high pressure it results into multi
VC ~ 10 3 VC ~ - 3 molecular layer on adsorbent surface.
(a) - (b) - 10
VS VS (c) Enthalpy of adsorption ( DH adsorption) is
VC VC ~ low and positive.
(c) ~ 10 23
- (d) -1 (d) It occurs because of van der Waal’s forces.
VS VS 9. According to Freundlich adsorption isotherm
5. The disperse phase in colloidal iron (III) which of the following is correct? [2012]
hydroxide and colloidal gold is positively and x
negatively charged, respectively. Which of the (a) µ p0
m
following statements is NOT correct ? [2005]
x
(a) Coagulation in both sols can be brought (b) µ p1
about by electrophoresis m
www.crackjee.xyz
Surface Chemistry C-93
x 12. The Tyndall effect is observed only when
(c) µ p1/n following conditions are satisfied: [JEE M 2017]
m
(d) All the above are correct for different (i) The diameter of the dispersed particles is
ranges of pressure much smaller than the wavelength of the
10. The coagulating power of electrolytes having light used.
ions Na+, Al3+ and Ba2+ for arsenic sulphide sol
increases in the order : [2013] (ii) The diameter of the dispersed particle is
(a) Al3+ < Ba2+ < Na+ (b) Na+ < Ba2+ < Al3+ not much smaller than the wavelength of
(c) Ba2+ < Na+ < Al3+ (d) Al3+ < Na+ < Ba2+ the light used.
11. For a linear plot of log (x/m) versus log p in a (iii) The refractive indices of the dispersed
Freundlich adsorption isotherm, which of the phase and dispersion medium are almost
following statements is correct? (k and n are
constants) [JEE M 2016] similar in magnitude.
(a) Only 1 n appears as the slope. (iv) The refractive indices of the dispersed
phase and dispersion medium differ
(b) log (1 n ) appears as the intercept. greatly in magnitude.
(c) Both k and 1 n appear in the slope term. (a) (i) and (iv) (b) (ii) and (iv)
(d) 1 n appears as the intercept. (c) (i) and (iii) (d) (ii) and (iii)

Answer Key
1 2 3 4 5 6 7 8 9 10 11 12
(b) (a) (d) (a) (b) (a) (c) (c) (d) (c) (a) (b)

1. (b) It is zero order reaction 5. (b) When oppositely charged sols are mixed
their charges are neutralised. Both sols may
[ NOTE Adsorption of gas on metal sur--
be partially or completely precipitated.
face is of zero order] 6. (a) According to Langmuir's Model of
2. (a) As adsorption is an exothermic process. adsorption of a gas on a soild surface the
\ Rise in temperature will decrease adsorption mass of gas adsorbed(x)per gram of the
(according to Le-chatelier principle). adsorbent (m) is directly proportional to
3. (d) Enzymes are very specific biological the pressure of the gas (p) at constant
catalysts possessing well - defined active temperature i.e.
sites x
4. (a) Particle size of colloidal particle = 1mµ to µp
m
100 mµ 7. (c) For a protective colloid µ lesser the value
(suppose 10 mµ) of gold number better is the protective
power.
4 3 4 3
Vc = pr Vc = p(10) Thus the correct order of protective power
3 = 3 of A, B, C and D is
Particle size of true solution particle = 1mm Þ (A) < (C) < (B) < (D)
Gold number 0.50 0.10 0.01 0.005
4 3 Vc 3
Vs = p(1) hence now = 10 Hence (c) is the correct answer
3 Vs
EBD_7764
www.crackjee.xyz
C-94 Chemistry
8. (c) Adsorption is an exothermic process, 11. (a) According to Freundlich adsorption isotherm
hence DH will always be negative x 1
9. (d) The Freundlich adsorption isotherm is log = log K + log P
m n
mathematically represented as
Thus if a graph is plotted between log(x/m) and
x log P, a straight line will be obtained
= kP1/n
m n
1/
at high pressure 1/n = 0. Hence, x / m µ =
pe
P° s lo

log x/m
at low pressure 1/n = 1 Hence, x/m µ P¢

Intercept = log K
10. (c) According to Hardy Schulze rule, greater
the charge on cation, greater is its log P
coagulating power for negatively charged The slope of the line is equal to 1/n and the
sol (As2S3), hence the correct order of intercept on log x/m axis will correspond to log
coagulating power : Na+ < Ba2+ < Al3+ K.
12. (b)
www.crackjee.xyz
General Principles and Processes of Isolation of Elements C-95

20
General Principles and
Processes of Isolation of Elements
1. Aluminium is extracted by the electrolysis of 6. Which method of purification is represented by
[2002] the following equation ? [2012]
(a) bauxite
523K
(b) alumina Ti(s) + 2I 2 (g) ¾¾¾®
(c) alumina mixed with molten cryolite
1700K
(d) molten cryolite. TiI 4 (g) ¾¾¾¾
® Ti(s) + 2I 2 (g)
2. The metal extracted by leaching with a cyanide
(a) Zone refining (b) Cupellation
is [2002]
(a) Mg (b) Ag (c) Polling (d) Van Arkel
(c) Cu (d) Na. 7. The metal that cannot be obtained by
3. Which one of the following ores is best electrolysis of an aqueous solution of its salts
concentrated by froth-flotation method? [2004] is: [2014]
(a) Galena (b) Cassiterite (a) Ag (b) Ca
(c) Magnetite (d) Malachite (c) Cu (d) Cr
4. During the process of electrolytic refining of
copper, some metals present as impurity settle 8. In the context of the Hall - Heroult process for
as ‘anode mud’. These are [2005] the extraction of Al, which of the following
(a) Fe and Ni (b) Ag and Au statements is false ? [JEE M 2015]
(c) Pb and Zn (d) Sn and Ag 3+
(a) Al is reduced at the cathode to form Al
5. Which of the following factors is of no (b) Na3AlF6 serves as the electrolyte
significance for roasting sulphide ores to the (c) CO and CO2 are produced in this process
oxides and not subjecting the sulphide ores to
(d) Al2O3 is mixed with CaF2 which lowers the
carbon reduction directly? [2008]
(a) Metal sulphides are thermodynamically melting point of the mixture and brings
more stable than CS2 conductivity
(b) CO2 is thermodynamically more stable than 9. Which one of the following ores is best
CS2 concentrated by froth floatation method?
(c) Metal sulphides are less stable than the [JEEM 2016]
corresponding oxides (a) Galena (b) Malachite
(d) CO2 is more volatile than CS2 (c) Magnetite (d) Siderite

Answer Key
1 2 3 4 5 6 7 8 9
(c) (b) (c) (b) (c) (d) (b) (b) (a)
EBD_7764
www.crackjee.xyz
C-96 Chemistry

1. (c) Pure aluminium can be obtained by From this we find that on thermodynamic
electrolysis of a mixture containing alumina, considerations CO2 is more stable than CS2
crayolite and fluorspar in the ratio 20 : 24 : and the metal sulphides are more stable
20. The fusion temperature of this mixture than corresponding oxides.
is 900ºC and it is a good conductor of In view of above the factor listed in choice
electricity. (c) is incorrect and so is of no significance.
2. (b) Silver ore forms a soluble complex with Hence the correct answer is (c)
NaCN from which silver is precipitated 6. (d) Van Arkel is a method in which heat
using scrap zinc. treatment is used to purify metal in this
Zn
Ag 2 S + 2 NaCN ® Na[Ag (CN ) 2 ] ¾¾® process metals are converted into other
metal compound for loosly coupled like as
Na 2 [ Zn (CN ) 4 ] + Ag ¯
iodine to make metal iodide which are easily
sodargento-cynanide
(soluble) decomposed and give pure metal.
The process is known as Van Arkel
3. (c) NOTE Galena is PbS and thus purified method.
by froth floatation method. 7. (b) On electrolysis of aqueous solution of
Froath floatation method is used to s-block elements H2 gas discharge at
concentrate sulphide ores. This method is cathode.
based on th e preferential wetting
1
properties with the froathing agent and At cathode: H 2O + e- ® H 2 + OH -
water. 2
4. (b) During the process of electrolytic refining 8. (b) In the metallurgy of aluminium, purified Al2O3
Ag and Au are obtained as anode mud. is mixed with Na3AlF6 or CaF2 which lowers
the melting point of the mix and brings
5. (c) NOTE The reduction of metal sulphides conductivity.
9. (a) Froth floatation method is mainly applicable
by car bon reduction process is not
for sulphide ores.
spontaneous because DG for such a (1) Malachite ore : Cu(OH)2 . CuCO3
process is positive. The reduction of metal (2) Magnetite ore : Fe3O4
oxide by carbon reduction process is (3) Siderite ore : FeCO3
spontaneous as DG for such a process is (4) Galena ore : PbS (Sulphide Ore)
negative.
www.crackjee.xyz

The p-Block Elements


(Group 15, 16, 17 & 18)
21
1. In XeF2, XeF4, XeF6 the number of lone pairs on solution which appears like a cloudy smoke.
Xe are respectively [2002] (c) due to strong affin ity for water,
(a) 2, 3, 1 (b) 1, 2, 3 concentrated hydrochloric acid pulls
(c) 4, 1, 2 (d) 3, 2, 1. moisture of air towards itself. This moisture
2. In case of nitrogen, NCl3 is possible but not forms droplets of water and hence the
NCl5 while in case of phosphorous, PCl 3 as well cloud.
as PCl5 are possible. It is due to [2002] (d) concentrated hydrochloric acid emits
(a) availability of vacant d orbitals in P but not strongly smelling HCl gas all the time.
in N 7. Which one of the following substances has the
(b) lower electronegativity of P than N highest proton affinity ? [2003]
(c) lower tendency of H-bond formation in P (a) H2S (b) NH3
than N (c) PH3 (d) H2O
(d) occurrence of P in solid while N in gaseous 8. Which among the following factors is the most
state at room temperature. important in making fluorine the strongest
3. Number of sigma bonds in P4O10 is [2002] oxidizing halogen ? [2004]
(a) 6 (b) 7 (a) Hydration enthalpy
(c) 17 (d) 16. (b) Ionization enthalpy
4. Oxidation number of Cl in CaOCl2 (bleaching (c) Electron affinity
power) is: [2002] (d) Bond dissociation energy
(a) zero, since it contains Cl2 9. Excess of KI reacts with CuSO4 solution and
(b) – 1, since it contains Cl – then Na2S2O3 solution is added to it. Which of
(c) + 1, since it contains ClO– the statements is incorrect for this reaction ?
(d) + 1 and – 1 since it contains ClO– and Cl– [2004]
5. What may be expected to happen when (a) Na2S2O3 is oxidised
phosphine gas is mixed with chlorine gas ? (b) CuI2 is formed
[2003] (c) Cu2I2 is formed
(a) PCl3 and HCl are formed and the mixture (d) Evolved I2 is reduced
warms up 10. Which one of the following statement regarding
(b) PCl5 and HCl are formed and the mixture helium is incorrect ? [2004]
cools down (a) It is used to produce and sustain powerful
(c) PH3 . Cl2 is formed with warming up superconducting magnets
(d) The mixture only cools down (b) It is used as a cryogenic agent for carrying
6. Concentrated hydrochloric acid when kept in out experiments at low temperatures
open air sometimes produces a cloud of white (c) It is used to fill gas balloons instead of
fumes. The explanation for it is that [2003] hydrogen because it is lighter and non-
(a) oxygen in air reacts with the emitted HCl inflammable
gas to form a cloud of chlorine gas (d) It is used in gas-cooled nuclear reactors
(b) strong affinity of HCl gas for moisture in air 11. The number of hydrogen atom(s) attached to
results in forming of droplets of liquid phosphorus atom in hypophosphorous acid is
[2005]
EBD_7764
www.crackjee.xyz
C-98 Chemistry
(a) three (b) one 19. Which of the following has maximum number of
(c) two (d) zero lone pairs associated with Xe ? [2011RS]
12. The correct order of the thermal stability of (a) XeF4 (b) XeF6
hydrogen halides (H–X) is [2005] (c) XeF2 (d) XeO3
(a) HI > HCl < HF > HBr 20. The molecule having smallest bond angle is :
(b) HCl< HF > HBr < HI [2012]
(c) HF > HCl < HBr > HI (a) NCl3 (b) AsCl3
(c) SbCl3 (d) PCl3
(d) HI < HBr > HCl < HF
21. Which among the following is the most reactive ?
13. Which of the following statements is true?
[JEE M 2015]
[2006]
(a) I2 (b) IC1
(a) HClO4 is a weaker acid than HClO3
(c) Cl2 (d) Br2
(b) HNO3 is a stronger acid than HNO2
22. Assertion: Nitrogen and oxygen are the main
(c) H3PO3 is a stronger acid than H2SO3 components in the atmosphere but these do not
(d) In aqueous medium HF is a stronger acid react to form oxides of nitrogen.
than HCl Reason: The reaction between nitrogen and
14. The increasing order of the first ionization oxygen requires high temperature.
enthalpies of the elements B, P, S and F (Lowest [JEE M 2015]
first) is [2006] (a) The assertion is incorrect, but the reason
(a) B < P < S < F (b) B < S < P < F is correct
(c) F < S < P < B (d) P < S < B < F (b) Both the assertion and reason are incorrect
15. What products are expected from the (c) Both assertion and reason are correct, and
disproportionation reaction of hypochlorous the reason is the correct explanation for
acid? [2006] the assertion
(a) HCl and Cl2O (b) HCl and HClO3 (d) Both assertion and reason are correct, but
(c) HClO3 and Cl2O (d) HClO2 and HClO4 the reason is not the correct explanation
16. Identify the incorrect statement among the fol- for the assertion
lowing. [2007] 23. Which one has the highest boiling point ?
(a) Br 2 reacts with hot and strong NaOH [JEE M 2015]
solution to give NaBr and H2O. (a) Kr (b) Xe
(b) Ozone reacts with SO2 to give SO3. (c) He (d) Ne
(c) Silicon reacts with NaOH(aq) in the pres- 24. The pair in which phosphorous atoms have a
ence of air to give Na2SiO3 and H2O. formal oxidation state of + 3 is : [JEE M 2016]
(a) Orthophosphorous and hypophosphoric
(d) Cl2 reacts with excess of NH3 to give N2
acids
and HCl. (b) Pyrophosphorous and pyrophosphoric
17. Regular use of the following fertilizers increases acids
the acidity of soil? [2007] (c) Orthophosphorous and
(a) Ammonium sulphate pyrophosphorous acids
(b) Potassium nitrate (d) Pyrophosphorous and hypophosphoric
(c) Urea acids
25. The reaction of zinc with dilute and
(d) Superphosphate of lime.
concentrated nitric acid, respectively, produces:
18. Which one of the following reactions of xenon [2016]
compounds is not feasible? [2009] (a) NO and N2O (b) NO2 and N2O
(a) 3Xe F4 + 6H 2 O ¾¾ ® (c) N2O and NO2 (d) NO2 and NO
2 Xe + Xe O3 +12HF +1.5O 2 26. The products obtained when chlorine gas reacts
(b) 2Xe F2 + 2H 2 O ¾¾
® 2 Xe + 4HF + O 2 with cold and dilute aqueous NaOH are :
[2017]
(c) Xe F6 + RbF ¾¾
® Rb[Xe F7 ] –
(a) ClO and ClO3 – (b) ClO2 and ClO3–

(d) Xe O3 + 6HF ¾¾® Xe F6 + 3H 2 O –
(c) Cl and ClO – (d) Cl– and ClO2–
www.crackjee.xyz
The p-Block Elements (Group 15, 16, 17 & 18) C-99

Answer Key
1 2 3 4 5 6 7 8 9 10 11 12 13 14 15
(d) (a) (d) (d) (d) (a) (b) (d) (b) (c) (c) (c) (b) (b) (b)
16 17 18 19 20 21 22 23 24 25 26
(d) (a) (d) (c) (c) (b) (c) (b) (c) (c) (c)

1. (d) In the formation of XeF 2 , sp 3 d are available. Hence phosphorous can from
hybridisation occurs which gives the pentahalides also but nitrogen can not form
molecule a trigonal bipyramidal structure. pentahalide due to absence of d-orbitals
F
O
••

••
Xe ||
5s 5p 5d
Ground state P
••
configuration F O O
3. (d)
O
O P P O
||

||
Excited state
O
configuration O P
In the formation of XeF 4 , sp 3 d 2 || O
hybridization occurs which gives the O
molecule an octahedral structure.
F F 4. (d) CaOCl2 –– or it can also be written as
Ca (OCl) Cl
xe x1 x2
Excited state config.
F F hence oxidation no of Cl in OCl– is
–2 + x2 = –1
In the formation of XeF 6 , sp 3 d 3
hybridization occurs which gives the
x2 = 2–1 = +1
molecule a pentagonal bipyramidal
structure. now oxidation no. of another Cl is –1 as it
F is present as Cl–.
F 5. (d) On mixing phosphine with chlorine gas PCl5
F and HCl forms. The mixture cools down.
xe
PH3 + 4Cl2 –––––® PCl5 + 3HCl
6. (a) 4HCl + O2 ® 2Cl2 + 2H2O
F F
air cloud of white fumes
F . .
2. (a) 2 2 3 2 2 6 2 3
7N = 1s 2s 2p ; 15P = 1s 2s 2p 3s 3p 7. (b) Among the given compounds, the N H 3 is
NOTE In phosphorous the 3d- orbitals most basic. Hence has highest proton af-
finity
EBD_7764
www.crackjee.xyz
C-100 Chemistry
8. (d) The fluorine has low dissociation energy atom having fully or half filled stable
of F - F bond and reaction of atomic fluorine orbitals.
is exothermic in nature In this case P has a stable half filled
-1
electronic configuration hence its ionisation
0
9. (b) 4 KI +2CuSO4 ® I 2 + Cu 2 I 2 + 2 K 2SO 4 enthalapy is greater in comparision to S.
Hence the correct order is B < S < P<F.
0 2+ +2.5 -1 15. (b) During disproportionation same compound
I 2 + 2Na 2 S2 O3 ® Na 2 S4 O6 + 2 NaI undergo simultaneous oxidation and
In this CuI2 is not formed. reduction.
10. (c) Helium is heavier than hydrogen although Oxidation
it is non-inflammable redution
- + -1 +5
3HO Cl ¾ ¾® 2HCl+ HClO 3
H
| 16. (d) Chlorine reacts with excess of ammonia to
11. (c) Hypophosphorous acid H - O - P ® O produce ammonium chloride and nitrogen.
|
H 17. (a) (NH4)2SO4 + 2H2O¾¾ ® 2H2SO4 + NH4OH
H2SO4 is strong acid and increases the
Two H-atoms are attached to P atom.
acidity of soil.
12. (c) The H–X bond strength decreases from HF 18. (d) The products of the concerned reaction
to HI. i.e. HF > HCl > HBr > HI. Thus HF is react each other forming back the reactants.
most stable while HI is least stable. This is
XeF6 + 3H 2 O ¾¾ ® XeO3 + 6HF .
evident from their decomposition reaction:
HF and HCl are stable upto 1473K, HBr
decreases slightly and HI dissociates F
considerably at 713K. The decreasing
stability of the hydrogen halide is also
reflected in the values of dissociation
energy of the H–X bond 19. (c) XeF2 : Xe

H-F H - Cl H - Br H-I
135kcal mol-1 103kcal mol -1 87kcal mol-1 71kcal mol -1

+5 +3
F
13. (b) The HNO3 is stronger than HNO 2 . The 3l p
more the oxidation state of N, the more is
the acid character.
14. (b) The correct order of ionisation enthalpies F F
is
F>P>S>B Xe

NOTE On moving along a period XeF4 : F F


ionization enthalapy increases from left to
right and decreases from top to bottom in a 2l p
group. But this trend breaks up in case of
www.crackjee.xyz
The p-Block Elements (Group 15, 16, 17 & 18) C-101
21. (b) ICl Order of reactivity of halogens
Cl2 > Br2 > I2
F But, the interhalogen compounds are
generally more reactive than halogens
(except F2), since the bond between two
dissimilar electronegative elements is
weaker than the bond between two similar
F
atoms i.e, X – X
F Xe
22. (c) Nitrogen and oxgen in air do not react to
XeF6 : F F form oxides of nitrogen in atmosphere be-
F cause the reaction between nitrogen and
oxygen requires high temperature.
1l p 23. (b) Xe. As we move down the group, the
melting and boiling points show a regular
increase due to corresponding increase in
the magnitude of their van der waal forces
of attraction as the size of the atom increases.
24. (c) Phosphorous acid contain P in +3 oxidation
state.
Xe Acid Formula Oxidation
state of
XeO3 : O O Phosphorus
O
Pyrophosphorous acid H4P2O 5 +3
1l p Pyrophosphoric acid H4P2O 7 +5
Orthophosphorous acid H3PO 3 +3
Hence XeF2 has maximum no. of lone pairs Hypophosphoric acid H4P2O 6 +4
25. (c) Reaction of Zn with dil. HNO3
of electrons.
4Zn + 10HNO 3(dil) 4Zn(NO 3) 2 +
20. (c) All the members form volatile halides of 5H2O + N2O
the type (Zn reacts differently with very
AX3. All halides are pyramidal in shape. dilute HNO3)
The bond angle decreases on moving Reaction of Zn with conc. HNO3
down the group due to decrease in bond Zn + 4HNO 3 (conc.) Zn(NO 3) 2 +
2H2O + 2NO2
pair-bond pair repulsion.
26. (c) Cl2 + NaOH ® NaCl + NaClO + H2O
NCl3 PCl3 AsCl3
[cold and dilute]
107° 94° 92°
EBD_7764
www.crackjee.xyz
C-102 Chemistry

The d-and f-Block Elements


22
1. Most common oxidation states of Ce (cerium) (a) In acidic solutions protons coordinate with
are [2002]
ammonia molecules forming NH +4 ions and
(a) +2, +3 (b) +2, +4
NH3 molecules are not available
(c) +3, +4 (d) +3, +5.
(b) In alkaline solutions insoluble Cu(OH)2 is
2. Arrange Ce+3, La+3, Pm+3 and Yb+3 in increasing
precipitated which is soluble in excess of
order of their ionic radii. [2002]
+3 +3 +3 +3
any alkali
(a) Yb < Pm < Ce < La
(c) Copper hydroxide is an amphoteric
(b) Ce+3 < Yb+3 < Pm+3 < La+3 substance
(c) Yb+3 < Pm+3 < La+3 < Ce+3 (d) In acidic solutions hydration protects
(d) Pm+3 < La+3 < Ce+3 < Yb+3. copper ions
3. Which of the following ions has the maximum 8. A red solid is insoluble in water. However it
magnetic moment? [2002] becomes soluble if some KI is added to water.
(a) Mn +2 (b) Fe+2 Heating the red solid in a test tube results in
(c) Ti+2 (d) Cr+2. liberation of some violet coloured fumes and
4. The most stable ion is [2002] droplets of a metal appear on the cooler parts of
the test tube. The red solid is [2003]
(a) [Fe(OH)3]3- (b) [Fe(Cl)6]3-
(a) HgI2 (b) HgO
(c) [Fe(CN)6]3- (d) [Fe(H2O)6]3+. (c) Pb3O4 (d) (NH4)2Cr2O7
5. When KMnO4 acts as an oxidising agent and 9. A reduction in atomic size with increase in atomic
ultimately forms [MnO4]–2, MnO2, Mn2O3, Mn+2 number is a characteristic of elements of
then the number of electrons transferred in each [2003]
case respectively is [2002] (a) d-block (b) f-block
(a) 4, 3, 1, 5 (b) 1, 5, 3, 7 (c) radioactive series (d) high atomic masses
10. What would happen when a solution of
(c) 1, 3, 4, 5 (d) 3, 5, 7, 1.
potassium chromate is treated with an excess of
6. The radius of La3+ (Atomic number of La = 57) is dilute nitric acid? [2003]
1.06Å. Which one of the following given values
will be closest to the radius of Lu3+ (Atomic (a) Cr2 O 72- and H2O are formed
number of Lu = 71) ? [2003]
(b) CrO 24- is reduced to +3 state of Cr
(a) 1.40 Å (b) 1.06 Å
(c) 0.85 Å (d) 1.60 Å (c) CrO 24- is oxidized to +7 state of Cr
7. Ammonia forms the complex ion [Cu(NH3)4]2+ (d) Cr3+ and Cr2O 72- are formed
with copper ions in alkaline solutions but not in 11. Which one of the following nitrates will leave
acidic solutions. What is the reason for it ? behind a metal on strong heating ? [2003]
[2003]
www.crackjee.xyz
The d-and f-Block Elements C-103
(a) Copper nitrate (b) Manganese nitrate 17. Heating mixture of Cu2O and Cu2S will give
(c) Silver nitrate (d) Ferric nitrate [2005]
12. Of the following outer electronic configurations (a) Cu2SO3 (b) CuO + CuS
of atoms, the highest oxidation state is achieved (c) Cu + SO3 (d) Cu + SO2
by which one of them ? [2004] 18. The oxidation state of chromium in the final
(a) (n – 1)d3 ns2 (b) (n – 1)d5 ns1 product formed by the reaction between Kl and
(c) (n – 1)d8 ns2 (d) (n – 1)d5 ns2 acidified potassium dichromate solution is:
13. The soldiers of Napolean army while at Alps [2005]
during freezing winter suffered a serious problem
(a) + 3 (b) + 2
as regards to the tin buttons of their uniforms.
White metallic tin buttons got converted to grey (c) + 6 (d) + 4
power. This transformation is related to 19. Calomel (Hg 2Cl 2 ) on reaction with ammonium
[2004] hydroxide gives [2005]
(a) a change in the partial pressure of oxygen
(a) HgO
in the air
(b) a change in the crystalline structure of tin (b) Hg 2O
(c) an interaction with nitrogen of the air at
(c) NH 2 – Hg – Hg – Cl
very low temperature
(d) an interaction with water vapour contained (d) Hg NH 2 Cl
in the humid air
20. The lanthanide contraction is responsible for
14. Among the properties (a) reducing (b) oxidising
the fact that [2005]
(c) complexing, the set of properties shown by
(a) Zr and Zn have the same oxidation state
CN– ion towards metal species is [2004]
(a) c, a (b) b, c (b) Zr and Hf have about the same radius
(c) a, b (d) a, b, c (c) Zr and Nb have similar oxidation state
15. Cerium (Z = 58) is an important member of the (d) Zr and Y have about the same radius
lanthanoids. Which of the following statements 21. The value of the ‘spin only’ magnetic moment
about cerium is incorrect? [2004] for one of the following configurations is 2.84
(a) The +4 oxidation state of cerium is not BM. The correct one is [2005]
known in solutions
(b) The +3 oxidation state of cerium is more (a) d 5 (in strong ligand field)
stable than the +4 oxidation state (b) d 3 (in weak as well as in strong fields)
(c) The common oxidation states of cerium are
+3 and +4 (c) d 4 (in weak ligand fields)
(d) Cerium (IV) acts as an oxidizing agent
16. The correct order of magnetic moments (spin (d) d 4 (in strong ligand fields)
only values in B.M.) anong is [2004] 22. Which of the following factors may be regarded
as the main cause of lanthanide contraction?
(a) [Fe(CN ) 6 ]4 - > [MnCl 4 ]2 - > [CoCl 4 ]2 - [2005]
(a) Greater shielding of 5d electrons by 4f
(b) [MnCl 4 ]2- > [Fe(CN ) 6 ]4 - > [CoCl 4 ]2 -
electrons
(b) Poorer shielding of 5d electrons by 4f
(c) [MnCl 4 ]2- > [CoCl 4 ]2 - > [Fe(CN ) 6 ]4 -
electrons
(d) [Fe(CN ) 6 ]4 - > [CoCl 4 ]2 - > [MnCl 4 ]2 - (c) Effective shielding of one of 4f electrons
by another in the subshell
(Atomic nos. : Mn = 25, Fe = 26, Co = 27) (d) Poor shielding of one of 4f electron by
another in the subshell
EBD_7764
www.crackjee.xyz
C-104 Chemistry
23. A metal, M forms chlorides in its +2 and +4 (a) the 5f orbitals extend further from the
oxidation states. Which of the following nucleus than the 4f orbitals
statements about these chlorides is (b) the 5f orbitals are more buried than the 4f
correct? [2006] orbitals
(a) MCl2 is more ionic than MCl4 (c) there is a similarity between 4f and 5f
(b) MCl2 is more easily hydrolysed than MCl4 orbitals in their angular part of the wave
(c) MCl2 is more volatile than MCl4 function
(d) MCl2 is more soluble in anhydrous ethanol (d) the actinoids are more reactive than the
than MCl4 lanthanoids.
24. Lanthanoid contraction is caused due to 29. Larger number of oxidation states are exhibited
[2006] by the actinoids than those by the lanthanoids,
the main reason being [2008]
(a) the same effective nuclear charge from Ce
to Lu (a) 4f orbitals more diffused than the 5f orbitals
(b) the imperfect shielding on outer electrons (b) leasser energy difference between 5f and
by 4f electrons from the nuclear charge 6d than between 4f and 5d orbitals
(c) the appreciable shielding on outer (c) more energy difference between 5f and 6d
electrons by 4f electrons from the nuclear than between 4f and 5d orbitals
charge (d) more reactive nature of the actionids than
(d) the appreciable shielding on outer the lanthanoids
electrons by 5d electrons from the nuclear 30. Amount of oxalic acid present in a solution can
charge be determined by its titration with KMnO4
25. The "spin-only" magnetic moment [in units of solution in the presence of H2SO4.The titration
Bohr magneton, (µB )] of Ni 2+ in aqueous gives unsatisfactory result when carried out in
solution would be (At. No. Ni = 28) [2006] the presence of HCl, because HCl [2008]
(a) 6 (b) 1.73 (a) gets oxidised by oxalic acid to chlorine
(c) 2.84 (d) 4.90 (b) furnishes H+ ions in addition to those from
oxalic acd
26. The stability of dihalides of Si, Ge, Sn and Pb
increases steadily in the sequence [2007] (c) reduces permanganate to Mn 2+
(d) Oxidises oxalic acid to carbon doxide and
(a) PbX 2 << SnX 2 << GeX 2 << SiX 2 water
(b) GeX2 << SiX2 << SnX2 << PbX2 31. Knowing that the chemistry of lanthanoids(Ln)
(c) SiX2 << GeX2 << PbX2 << SnX2 is dominated by its + 3 oxidation state, which of
(d) SiX2 << GeX2 << SnX2 << PbX2. the following statements is incorrect? [2009]
27. Identify the incorrect statement among the (a) The ionic size of Ln (III) decrease in general
following: [2007] with increasing atomic number
(a) 4f and 5f orbitals are equally shielded. (b) Ln (III) compounds ar e generally
(b) d-Block elements show irregular and erratic colourless.
chemical properties among themselves. (c) Ln (III) hydroxide are mainly basic in
(c) La and Lu have partially filled d-orbitals character.
and no other partially filled orbitals. (d) Because of the large size of the Ln (III) ions
(d) The chemistry of various lanthanoids is the bonding in its compounds is
very similar. predominantly ionic in character.
28. The actinoids exhibit more number of oxidation 32. Iron exhibits +2 and + 3 oxidation states. Which
states in general than the lanthanoids. This is of the following statements about iron is
because [2007] incorrect ? [2012]
www.crackjee.xyz
The d-and f-Block Elements C-105
(a) Ferrous oxide is more basic in nature than +
the ferric oxide. (b) é CoCl ( NH3 ) ù + 5H +
ë 5û
(b) Ferrous compounds are relatively more
ionic than the corresponding ferric ® CO 2 + + 5NH +4 + Cl-
¾¾
compounds.
2+ 4-
(c) Ferrous compounds are less volatile than (c) é Mg ( H 2 O ) ù + ( EDTA )
ë 6û
the corresponding ferric compounds.
(d) Ferrous compounds ar e more easily 2+
® éë Mg ( EDTA ) ùû
excess NaOH + 6H 2 O
hydrolysed than the corresponding ferric ¾¾¾¾¾¾
compounds. (d) CuSO 4 + 4KCN ¾¾
®
33. Which of the following arrangements does not
represent the correct order of the property stated K 2 éëCu ( CN )4 ùû + K 2SO4
against it ? [2013]
(a) V2+ < Cr2+ < Mn2+ < Fe2+ : paramagnetic 36. Match the catalysts to the correct processes :
behaviour [JEE M 2015]
(b) Ni2+ < Co2+ < Fe2+ < Mn2+ : ionic size Catalyst Process
(c) Co3+ < Fe3+ < Cr3+ < Sc3+ : stability in (A) TiCl4 (i) Wacker process
aqueous solution (B) PdCl2 (ii) Ziegler - Natta
(d) Sc < Ti < Cr < Mn : number of oxidation polymerization
states (C) CuCl2 (iii) Contact process
34. Which series of reactions correctly represents (D) V2O5 (iv) Deacon's process
chemical reactions related to iron and its (a) (A) - (ii), (B) - (iii), (C) - (iv), (D) - (i)
compound? [2014] (b) (A) - (iii), (B) - (i), (C) - (ii), (D) - (iv)
dil. H SO H SO , O
(c) (A) - (iii), (B) - (ii), (C) - (iv), (D) - (i)
(a) Fe ¾¾¾¾¾
2 4 ® FeSO ¾¾¾¾¾
4
2 4 2®
(d) (A) - (ii), (B) - (i), (C) - (iv), (D) - (iii)
37. The color of KMnO4 is due to : [JEE M 2015]
Fe 2 ( SO4 )3 ¾¾¾
heat
® Fe (a) L ® M charge transfer transition
O , heat dil. H SO
(b) s - s* transition
(b) Fe ¾¾¾¾
2 ® FeO ¾¾¾¾¾
2 4®
(c) M ® L charge transfer transition
heat (d) d – d transition
FeSO 4 ¾¾¾
® Fe
38. Which of the following compounds is metallic
(c) Cl , heat
Fe ¾¾¾¾®
2 FeCl3 ¾¾¾¾®
heat , air and ferromagnetic? [JEE M 2016]
(a) VO2 (b) MnO2
Zn
FeCl 2 ¾¾® Fe (c) TiO2 (d) CrO2
39. In the following reactions, ZnO is respectively
(d) O , heat CO , 600°C
Fe ¾¾¾¾
2 ® Fe3O 4 ¾¾¾¾¾
® acting as a/an: [2017]
CO , 700°C (A) ZnO + Na2O ® Na2ZnO2
FeO ¾¾¾¾¾
® Fe
(B) ZnO + CO2 ® ZnCO3
35. The equation which is balanced and represents
the correct product(s) is: [2014] (a) base and acid (b) base and base
(c) acid and acid (d) acid and base
(a) Li 2 O + 2KCl ¾¾
® 2LiCl + K 2 O

Answer Key
1 2 3 4 5 6 7 8 9 10 11 12 13 14 15
(c) (a) (a) (c) (c) (c) (a) (a) (b) (a) (c) (d) (b) (a) (a)
16 17 18 19 20 21 22 23 24 25 26 27 28 29 30
(c) (d) (a) (d) (b) (d) (d) (a) (b) (c) (d) (a) (a) (b) (c)
31 32 33 34 35 36 37 38 39
(b) (d) (a) (c) (b) (d) (a) (d) (d)
EBD_7764
www.crackjee.xyz
C-106 Chemistry

1. (c) Common oxidation states of Ce(Cerium) are 8. (a) When KI is added to mercuric iodide it
+ 3 and + 4 disssolve in it and form complex.
2. (a) In lanthanides there is a regular decrease HgI 2 ∗ KI ® K 2 [HgI 4 ]
in the atomic radii as well as ionic radii of red, solid (so lub le)
trivalent ions as the atomic number (inso lub le)
increases from Ce to Lu. This decrease in On heating HgI2 decomposes as
size of atoms and ions is known as
HgI 2 ƒ Hg + I 2
lanthanide contraction. Although the
(violet vapours)
atomic radii do show some irregualrities but
ionic radii decreases from La to Lu. Thus 9. (b) f-block elements show a regular decrease
the correct order is. in atomic size due to lanthanide/actinide
contraction.
Yb+3 < Pm +3 < Ce +3 < La +3
10. (a) When a solution of potassium chromate is
86.8pm 97pm 102pm 103pm
treated with an excess of dilute nitric acid.
3. (a) Mn++ –5 unpaired electrons Potassium dichromate and H2O are formed.
Fe++ – 4 unpaired electrons 2K2CrO4 + 2HNO3 –® K2Cr2O7 + 2KNO3 +
Ti++ – 2 unpaired electrons H2O
Cr++ – 4 unpaired electrons Hence Cr2O7– and H2O are formed.
hence maximum no. of unpaired electron is 11. (c) AgNO3 on heating till red hot
present in Mn++.
decomposes as follows :
NOTE Magnetic moment µ number of 1
AgNO3 ® Ag + NO 2 + O2
unpaired electrons 2
4. (c) The cyano and hydroxo complexes are far 12. (d) (n–1)d5ns2 attains the maximum O.S. of + 7
more stable than those formed by halide 13. (b) Grey tin white tin
ion. This is due to the fact that CN– and
OH– are strong lewis bases (nucleophiles). Grey tin is brittle and crumbles down to
powder in very cold climate
Further [Fe(OH)5]3– is not formed. hence
most stable ion is [Fe(CN)6]3- The conversion of grey tin to white tin is
acompained by increase in volume., This
+3 +7 +6
5. (c) - 4e - -e - 2- is knwon as tin plaque or tin disease.
Mn2 O 3 ¬¾¾ ¾ [KMnO4 ] ¾ ¾¾® [MnO 4 ]
14. (a) CN– ion acts good complexing as well as
– 5e

– 3e

reducing agent.
+4
Mn
2+
MnO2 15. (a) The +4 oxidation state of cerium is also
known in solution.
1 4–
6. (c) Ionic radii µ 16. (c) [Fe (CN)6] ®
z
– no of unpaired electron = 0
z 1.06 71
Thus, 2 Þ < [MnCl4] ®
2–
z1 3+
(Ionic radii of Lu ) 57
– no of unpaired electron = 5
Þ Ionic radii of Lu3+ = 0.85 Å 2–
.. + [CoCl4] ®
7. (a) N H3 + H + (acid medium) N H4 – no of unpaired electron = 3
www.crackjee.xyz
The d-and f-Block Elements C-107

NOTE The greater the number of


22. (d) In lanthanides, there is poorer shielding of
5d electrons by 4 f electrons resulting in
unpaired electrons, greater the magnitude greater attraction of the nucleus over 5 d
of magnetic moment. Hence the correct electrons and contraction of the atomic
order will be radii.
[MnCl4]– – > [CoCl4]– – > [Fe(CN)6]4– 23. (a) Metal atom in the lower oxidation state
17. (d) 2Cu 2 O + Cu 2S ¾¾® 6Cu + SO 2 forms the ionic bond and in the higher
self reduction. oxidation state the covalent bond. because
2- - + higher oxidation state means small size and
18. (a) Cr2O7 + 6I + 14H ¾¾
® great polarizing power and hence greater
3+ the covalent character. Hence MCl2 is more
3I2 + 7H 2O + 2 Cr
ionic than MCl4.
oxidation state of Cr is 3+.
24. (b) The configuration of Lanthanides show
19. (d) Hg 2 Cl 2 + 2NH 4OH ¾¾ ® that the additional electron enters the 4f
HgNH 2Cl +NH 4Cl + 2H 2O subshell. The shielding of one 4 f electron
20. (b) NOTE In vertical columns of transition
by another is very little or imperfect. The
imperfect shielding of f electrons is due to
elements, there is an increase in size from the shape of f orbitals which is very much
first member to second member as expected diffused. Thus as the atomic number
but from second member to third member, increases, the nuclear charge increases by
there is very small chang in size and some unity at each step. While no comparable
times sizes are same. This is due to increase in the mutual shielding effect of
lanthanide contraction this is the reason 4f occurs. This causes a contraction in the
for Zr and Hf to have same radius. size of the 4 f subshell. as a result atomic
21. (d) d 5 –––– strong ligand field and ionic radii decreases gradually from La
to Lu.
25. (c) The number of unpaired electrons in
t2g eg Ni2+(aq) = 2
m = n n + 2 = 3 = 1.73BM Water is weak ligand hence no pairing will
take place spin magnetic moment
d 3–– in weak as well as in strong field
= n(n + 2) = 2(2 + 2)

t2g eg = 8 = 2.82
26. (d) Reluctance of valence shell electrons to
m = 3(5) = 15 = 3.87 B.M. participate in bonding is called inert pair
d 4– in weak ligand field effect. The stability of lower oxidation state
(+2 for group 14 element) increases on
going down the group. So the correct order
t2g eg is
SiX2 < GeX2 < SnX2< PbX2
m = 4(8) = 24 = 4.89 27. (a) 4f orbital is nearer to nucleus as compared
d 4– in strong ligand field to 5f orbital therefore, shielding of 4 f is
more than 5f.

t2g eg 28. (a) NOTE More the distance between


nucleus and outer orbitals, lesser will be
m = 2(4) = 8 = 2.82. force of attraction on them. Distance
EBD_7764
www.crackjee.xyz
C-108 Chemistry
between nucleus and 5 f orbitals is more as Co3+ / Co = + 1.97, Fe3+ / Fe = + 0.77 ;
compared to distance between 4f orbital Cr3+ / Cr2+ = – 0.41, Sc 3+ is highly stable as it
and nucleus. does not show + 2 O. S.
So actinoids exhibit more number of (d) Sc – (+ 2), (+ 3)
oxidation states in general than the Ti – (+ 2), (+ 3), (+ 4)
lanthanoids. Cr – (+ 1), (+ 2), (+ 3), (+ 4), (+ 5), (+ 6)
29. (b) NOTE The main reason for exhibiting Mn – (+ 2), (+ 3), (+ 4), (+ 5), (+ 6), (+ 7)
i.e. Sc < Ti < Cr = Mn
larger number of oxidation states by
34. (c) In equation (i) Fe2(SO4)3 and in equation
actinoids as compared to lanthanoids is
lesser energy difference between 5 f and 6d (ii) Fe2(SO4)3 on decomposing will form
orbitals as compared to that between 4f and oxide instead of Fe.
5d orbitals. The correct sequence of reactions is
In case of actinoids we can remove O ,heat CO,600°C
Fe ¾¾¾¾
2 ® Fe3 O 4 ¾¾¾¾¾ ®
electrons from 5f as also from 6 d and due
to this actinoids exhibit larger number of D
Fe2 (SO 4 )3 ¾¾
® Fe
oxidation state than lanthanoids. Thus the
correct answer is option (b) 35. (b) The complex [CoCl(NH3 )5 ]+
30. (c) The titration of oxalic acid with KMnO4 in
decomposes under acidic medium, so
presence of HCl gives unsatisfactory result
because of the fact that KMnO4 can also [CoCl(NH3 )5 ]+ + 5H + ¾¾
®
oxidise HCl along with oxalic acid. HCl on
Co 2 + + 5NH 4 + + Cl -
oxidation gives Cl 2 and HCl reduces
KMnO4 to Mn2+ thus the correct answer 36. (d) (A) - (ii), (B) - (i), (C) - (iv), (D) - (iii)
is (c). 37. (a) L ® M charge transfer spectra. KMnO4 is
31. (b) Most of the Ln3+ compounds except La3+ colored because it absorbs light in the
and Lu3+ are coloured due to the presence visible range of electromagnetic radiation.
of f-electrons. The permanganate ion is the source of
32. (d) Fe3+ is easily hydrolysed than Fe2+ due to color, as a ligand to metal, (L ® M) charge
more positive charge. transfer takes place between oxygen's p
33. (a) orbitals and the empty d-orbitals on the
(a) V = 3d 3 4s 2 ; V2+ = 3d 3 = 3 unpaired electrons metal. This charge transfer takes place
Cr = 3d 5 4s1 ; Cr2+ = 3d 4 = 4 unpaired electrons when a photon of light is absorbed, which
Mn = 3d 5 4s 2 ; Mn 2+ = 3d 5 = 5 unpaired leads to the purple color of the compound.
electrons 38. (d) Out of all the four given metallic oxides CrO2
Fe = 3d 6 4s 2 ; Fe2+ = 3d 6 = 4 unpaired electrons is attracted by magnetic field very strongly.
Hence the correct order of paramagnetic The effect persists even when the magnetic
behaviour field is removed. Thus CrO2 is metallic and
V2+ < Cr 2+ = Fe2+ < Mn2+
ferromagnetic in nature
(b) For the same oxidation state, the ionic radii
39. (d) Although ZnO is an amphoteric oxide but
generally decreases as the atomic number
in given reaction.
increases in a particular transition series. hence
the order is (A) ZnO + Na2O ® Na2ZnO2
Mn++ > Fe++ > Co++ > Ni++ acid base salt
(c) In solution, the stability of the compound (B) ZnO + CO2 ® ZnCO3
depends upon electrode potentials, SEP of the base acid salt
transitions metal ions are given as
www.crackjee.xyz
Co-ordination Compounds C-109

Co-ordination Compounds
23
1. A square planar complex is formed by (d) the number of only anionic ligands bonded
hybridisation of which atomic orbitals? to the metal ion.
[2002] 7. Which one of the following complexes is an
outer orbital complex ? [2004]
(a) s, px , py , dyz (b) s, px , p y , d x2 - y2 3+ 4–
(a) [Co(NH3)6] (b) [Mn(CN)6]
(c) s, p x , p y , d z 2 (d) s, py , pz , dxy (c) [Fe(CN)6]4– (d) [Ni(NH3)6]2+
2. The type of isomerism present in (Atomic nos. : Mn = 25; Fe = 26; Co = 27, Ni = 28)
nitropentammine chromium (III) chloride is 8. Coordination compounds have great importance
[2002] in biological systems. In this context which of
(a) optical (b) linkage the following statements is incorrect ? [2004]
(c) ionization (d) polymerisation. (a) Cyanocobalamin is B12 and contains cobalt
3. CH3 – Mg – Br is an organo metallic compound (b) Haemoglobin is the red pigment of blood
due to [2002] and contains irons
(a) Mg – Br bond (b) C – Mg bond (c) Chlorophylls are green pigments in plants
and contain calcium
(c) C – Br bond (d) C – H bond.
(d) Carboxypeptidase - A is an exzyme and
4. One mole of the complex compound
contains zinc.
Co(NH3 ) 5 Cl 3 , gives 3 moles of ions on
dissolution in water. One mole of the same 9. Which one of the following has largest number
complex reacts with two moles of AgNO3 of isomers? [2004]
solution to yield two moles of AgCl (s). The (a) [Ir(PR 3 ) 2 H(CO)]2 +
structure of the complex is [2003]
(a) [Co(NH3)3Cl3]. 2 NH3 (b) [Co( NH 3 )5 Cl]2+
(b) [Co(NH3)4Cl2] Cl . NH3
(c) [Co(NH3)4Cl] Cl2. NH3 (c) [Ru ( NH 3 ) 4 Cl 2 ]+
(d) [Co(NH3)5Cl] Cl2
(d) [Co(en ) 2 Cl 2 ]+
5. In the coordination compound, K4[Ni(CN)4], the
oxidation state of nickel is [2003] (R = alkyl group, en = ethylenediamine)
(a) 0 (b) +1 10. The oxidation state Cr in [Cr ( NH 3 ) 4 Cl 2 ]+ is
(c) +2 (d) –1
(a) 0 (b) + 1 [2005]
6. The coordination number of a central metal atom
in a complex is determined by [2004] (c) + 2 (d) + 3
(a) the number of ligands around a metal ion 11. The IUPAC name of the coordination compound
bonded by sigma and pi-bonds both K 3[Fe(CN ) 6 ] is [2005]
(b) the number of ligands around a metal ion (a) Tripotassium hexacyanoiron (II)
bonded by pi-bonds (b) Potassium hexacyanoiron (II)
(c) the number of ligands around a metal ion (c) Potassium hexacyanoferrate (III)
bonded by sigma bonds (d) Potassium hexacyanoferrate (II)
EBD_7764
www.crackjee.xyz
C-110 Chemistry
12. Which of the following compounds shows (a) 6 and 2 (b) 4 and 2
optical isomerism? [2005] (c) 4 and 3 (d) 6 and 3
(a) [Co(CN) 6 ]3 - (b) [Cr(C 2 O 4 )3 ]3 - 20. In which of the following complexes of the Co
(at. no. 27), will the magnitude of D0 be the
(c) [ ZnCl4 ]2 - (d) [Cu ( NH 3 ) 4 ]2 + hightest? [2008]
(a) [Co(CN)6] 3– (b) [Co(C2O4)3]3–
13. Which one of the following cyano complexes 3+
would exhibit the lowest value of paramagnetic (c) [Co(H2O)6] (d) [Co(NH3)6]3+
behaviour ? [2005] 21. Which of the following has an optical isomer
[2009]
(a) [Co(CN) 6 ]3 - (b) [Fe(CN) 6 ]3 - (a) [Co(en) (NH3)2] 2+

(b) [Co(H2O)4(en)]3+
(c) [Mn (CN) 6 ]3 - (d) [Cr (CN) 6 ]3 -
(c) [Co(en)2 (NH3)2]3+
(At. Nos : Cr = 24, Mn = 25, Fe = 26, Co = 27)
(d) [Co(NH3)3Cl] +
14. The IUPAC name for the complex
22. Which of the following pairs represent linkage
[Co(NO2)(NH3)5]Cl2 is : [2006]
isomers? [2009]
(a) pentaammine nitrito-N-cobalt(II) chloride
(a) [Pd(P Ph3)2(NCS)2 ] and [Pd ( P Ph3)2
(b) pentaammine nitrito-N-cobalt(III) chloride (SCN)2]
(c) nitrito-N-pentaamminecobalt(III) chloride
(b) [Co(NH3)5NO3] SO4 and
(d) nitrito-N-pentaamminecobalt(II) chloride [Co(NH3)5SO4]NO3
15. Nickel (Z = 28) combines with a uninegative (c) [PtCl2(NH3)4] Br2 and [Pt Br 2 (NH3)4] Cl2
monodentate ligand X– to form a paramagnetic
(d) [Cu(NH3)4][Pt Cl4] and [Pt(NH3)4][CuCl4]
complex [NiX4 ]2–. The number of unpaired
electron(s) in the nickel and geometry of this 23. A solution containing 2.675 g of CoCl3. 6 NH3
complex ion are, respectively : [2006] (molar mass = 267.5 g mol–1) is passed through
a cation exchanger. The chloride ions obtained
(a) one, square planar (b) two, square planar
in solution were treated with excess of AgNO3
(c) one, tetrahedral (d) two, tetrahedral
to give 4.78 g of AgCl (molar mass = 143.5 g
16. In Fe(CO)5, the Fe – C bond possesses [2006] mol–1). The formula of the complex is
(a) ionic character
(At. mass of Ag = 108 u) [2010]
(b) s-character only
(c) p-character (a) [Co(NH3 )6 ]Cl3
(d) both s and p characters (b) [CoCl2 (NH3 )4 ]Cl
17. How many EDTA (ethylenediaminetetraacetic
acid) molecules are required to make an (c) [CoCl3 (NH3 )3 ]
octahedral complex with a Ca2+ ion? [2006] (d) [CoCl(NH3 )5 ]Cl2
(a) One (b) Two
24. Which one of the following has an optical
(c) Six (d) Three
isomer? [2010]
18. Which of the following has a square planar
(a) [Zn(en) (NH3)2]2+ (b) [Co(en)3]3+
geometry? [2007]
(c) [Co(H2O)4(en)]3+ (d) [Zn(en)2]2+
(a) [PtCl4]2– (b) [CoCl4]2–
(en = ethylenediamine)
(c) [FeCl4]2– (d) [NiCl4]2–
(At. nos.: Fe = 26, Co = 27, Ni = 28, Pt = 78) 25. Which one of the following complex ions has
geometrical isomers ? [2011]
19. The coordination number and the oxidation
state of the element ‘E’ in the complex (a) [Ni(NH3)5Br]+ (b) [Co(NH3)2(en)2]3+
[E (en)2 (C2O4)]NO2 (where (en) is ethylene (c) [Cr(NH3)4(en)2]3+ (d) [Co(en)3]3+
diamine) are, respectively, [2008] (en ethylenediamine)
www.crackjee.xyz
Co-ordination Compounds C-111
26. Which among the following will be named as (d) K3[Co(NO2)6]
dibromidobis (ethylene diamine) chromium (III) 30. The number of geometric isomers that can exist
bromide? [2012] for square planar complex [Pt (Cl) (py) (NH3)
(a) [Cr (en)3]Br3 (b) [Cr(en)2Br2]Br (NH2OH)]+ is (py = pyridine) : [JEE M 2015]
(c) [Cr(en)Br4]– (d) [Cr(en)Br2]Br (a) 4 (b) 6
27. Which of the following complex species is not (c) 2 (d) 3
expected to exhibit optical isomerism ? [2013] 31. Which one of the following complexes shows
(a) [Co(en)3]3+ optical isomerism? [JEE M 2016]
(b) [Co(en)2 Cl2]+ (a) trans [Co(en)2Cl2]Cl
(c) [Co(NH3)3 Cl3] (b) [Co(NH3)4Cl2]Cl
(d) [Co(en) (NH3)2 Cl2]+ (c) [Co(NH3)3Cl3]
28. The octahedral complex of a metal ion M3+ with (d) cis[Co(en)2Cl2]Cl
four monodentate ligands L1, L2, L3 and L4 (en = ethylenediamine)
absorb wavelengths in the region of red, green, 32. The pair having the same magnetic moment is:
yellow and blue, respectively. The increasing [At. No.: Cr = 24, Mn = 25, Fe = 26, Co = 27]
order of ligand strength of the four ligands is: [JEE M 2016]
[2014]
(a) [Mn(H2O)6] and [Cr(H2O)6]2+
2+

(a) L4 < L3 < L2 < L1 (b) [CoCl4]2– and [Fe(H2O)6]2+


(c) [Cr(H2O)6]2+ and [CoCl4]2–
(b) L1 < L3 < L2 < L4
(d) [Cr(H2O)6]2+ and [Fe(H2O)6]2+
(c) L3 < L2 < L4 < L1 33. On treatment of 100 mL of 0.1 M solution of
CoCl3 . 6H2O with excess AgNO3; 1.2 × 1022
(d) L1 < L2 < L4 < L3
ions are precipitated. The complex is : [2017]
29. Which of the following compounds is not (a) [Co(H2O)4 Cl2]Cl.2H2O
colored yellow ? [JEE M 2015]
(b) [Co(H2O)3Cl3].3H2O
(a) (NH4)3 [As(Mo3O10)4 ]
(c) [Co(H2O)6]Cl3
(b) BaCrO4
(c) Zn2[Fe(CN)6] (d) [Co(H2O)5Cl]Cl2.H2O

Answer Key
1 2 3 4 5 6 7 8 9 10 11 12 13 14 15
(b) (b) (b) (d) (a) (c) (d) (c) (d) (d) (c) (b) (a) (b) (d)
16 17 18 19 20 21 22 23 24 25 26 27 28 29 30
(d) (a) (a) (d) (a) (c) (a) (a) (b) (b) (b) (c) (b) (c) (d)
31 32 33
(d) (d) (d)

1. (b) A square planar complex is formed by


é Cr ( NH3 ) NO2 ù Cl2
hybridisation of ë 5 û
s, px, py and d 2 2 atomic orbitals It can exist in following two structures
x -y
2. (b) The chemical formula of nitropentammine éCr ( NH3 ) NO2 ù Cl2 and
chromium (III) chloride is ë 5 û
EBD_7764
www.crackjee.xyz
C-112 Chemistry
nitropentammine chromium (III) chloride
[Ir(PR 3 ) 2 H(CO)]2+ , [Co(en ) 2 Cl 2 ]+
éCr ( NH3 ) ONOù Cl2 cis and trans cis and trans and
ë 5 û optical isomers
Nitropentammine chromium (III) chloride
Therefore the type of isomerism found in 10. (d) Oxidation state of Cr in [Cr ( NH 3 ) 4 Cl 2 ]+ .
this compound is linkage isomerism as nitro Let it be x, 1 × x + 4 × 0 + 2 × (–1) = 1
group is linked through N as –NO2 or Therefore x = 3.
through O as – ONO.
11. (c) K 3 [Fe (CN ) 6 ] is potassium
3. (b) Compounds that contain at least one
hexacyanoferrate (III).
carbon metal bond are known as
organ ometallic compounds. In 12. (b) C2O4 C2O 4
CH3-Mg-Br (Grignard's reagent) a bond is
present between carbon and Mg (Metal) Cr C2O 4 C2O4 Cr
hence it is an organometallic compound. C2O4
C2O4
4. (d) Co (NH3)5 Cl3 [Co(NH3)5Cl]+2 + 2Cl-
\ Structure is [Co (NH3)5 Cl] Cl2.
Non-superimposable mirror images, hence
Now [Co(NH3 )5 Cl]Cl2 + 2AgNO3 optical isomers.
® [Co(NH3 )5 Cl](NO3 )2 + 2AgCl 13. (a) No. of unpaired electron
5. (a) Let the O. No of Ni in K4[Fe(CN)6] be = x (a) Co3+ 4
then (b) Fe3+ 1
4 (+ 1) + x + (–1) × 4 = 0 (c) Mn 3+ 4
Þ 4+ x–4=0 (d) Cr3+ 3
x=0 The effective magnetic moment is given by
the number of unpaired electrons in a
6. (c) The coordination number of central metal
substance, the lesser the number of
atom in a complex is equal to number of
unpaired electrons lower is its magnetic
monovalent ligands, twice the number of
moment in Bohr – Magneton and lower
bidentate ligands and so on, around the
shall be its paramagnetism
metal ion bonded by coordinate bonds.
14. (b) [Co(NO)2(NH3)5]Cl2
Hence coordination number = no. of
pentaammine nitrito-N-cobalt (III) chloride
s bonds formed by metals with ligands
7. (d) Hybridisation 15. (d) [Ni X4]2–, the electronic configuration of
Ni2+ is
[Fe(CN)6 ]4 -,[Mn(CN) 6 ]4- ,
d 2sp3 d 2sp3

[Co(NH 3 ] 3+ ,[Ni(NH 3 ) 6 ] 2+
d 2sp 3 sp 3d 2 It contains two unpaired electrons and the
Hence [ Ni( NH3 ) 6 ] 2+
is outer orbital hybridisation is sp3 (tetrahedral).
complex. 16. (d) Due to some backbonding by sidewise
overlapping of between d-orbitals of metal
8. (c) The chlorophyll molecule plays an and p-orbital of carbon, the Fe–C bond in
important role in photosynthesis, contain Fe(CO)5 has both s and p character.
porphyrin ring and the metal Mg not Ca.
17. (a) EDTA has hexadentate four donor O atoms
9. (d) Isomers and 2 donor N atoms and for the formation
[Ru(NH 3 ) 4 Cl2 ]+ ,[Co(NH 3 )5 Cl]2 + , of octahedral complex one molecule is
cis and trans none required
www.crackjee.xyz
Co-ordination Compounds C-113
18. (a) Complexes with dsp2 hybridisation are 24. (b) For a substance to be optical isomer
square planar. So [PtCl4]2– is square planar following conditions should be fulfiled
in shape. (a) A coordination compound which can
19. (d) In the given complex we have two bidentate rotate the plane of polarised light is
ligands said to be optically active.
(i.e en and C2O4), so coordination number (b) When the coordination compounds
of E is 6 have same formula but differ in their
(2 × 2 + 1 × 2 = 6) abilities to rotate directions of the
Let the oxidation state of E in complex be x, plane of polarised light are said to
then exhibit optical isomerism and the
[x + (–2) = 1] or x – 2 = 1 molecules are optical isomers. The
optical isomers are pair of molecules
or x = + 3, so its oxidation state is + 3
which are non-superimposable mirror
Thus option (d) is correct.
images of each other.
20. (a) In octahedral complex the magnitude of Do (c) This is due to the absence of elements
will be highest in a complex having of symmetry in the complex.
strongest ligand. Of the given ligands CN–
(d) Optical isomerism is expected in
is strongest so D o will be highest for
tetrahedral complexes of the type
[Co(CN)6]3–. Thus option (a) is correct.
Mabcd.
21. (c) H3N NH3
3+ 3+
Based on this only option (2) shows
NH3
optical isomerism
NH3
Co en Co [Co(en)3]3+

en en en

en 3+ en 3+
3+
Enantiomers of cis- éë Co(en) 2 (NH 3 ) 2 ûù
en Co Co
22. (a) The SCN– ion can coordinate through S or en
N atom giving rise to linkage isomerism
en en
M ¬ SCN thiocyanato
d–form Mirror –form
M ¬ NCS isothiocyanato.
23. (a) ® xCl-
CoCl3.6NH3 ¾¾
2.675g

xCl - + AgNO3 ¾¾
® x AgCl ¯ en Co en en
4.78g
Number of moles of the complex
2.675 'Meso' or optically inactive form
= = 0.01 moles
267.5
Number of moles of AgCl obtained Complexes of Zn++ cannot show optical
isomerism as they are tetrahedral complexes
4.78
= = 0.03 moles with plane of symmetry.
143.5
\ No. of moles of AgCl obtained [Co(H 2 O)4 (en)]3+ have two planes of
= 3 × No. of moles of complex symmetry hence it is also optically inactive.
0.03 Hence the formula of the complex is
\n= =3 [Co(NH3)6]Cl3
0.01
EBD_7764
www.crackjee.xyz
C-114 Chemistry
create a complex with smaller D, which will
25. (b) 3+ absorbs light of longer l and thus lower
en
frequency. Conservely, stronger field
NH3
ligands create a larger D, absorb light of
Co shorter l and thus higher v i.e. higher
energy.
en NH3 Red < Yellow < Green < Blue
l = 650 nm 570 nm 490 nm 450 nm
cis- So order of ligand strength is
L1 < L3 < L2 < L4
NH3 3+ 29. (c)
30. (d) Square planar complexes of type M[ABCD]
form three isomers. Their position may be
obtained by fixing the position of one
en Co en ligand and placing at the trans position any
one of the remaining three ligands one by
one.
HOH2N Cl
NH3
26. (b) [Cr(en)2Br2]Br
Pt
dibromidobis(ethylenediamine) chromium
(III) Bromide.
27. (c) Octahedral coordination entities of the type py NH3
trans
Ma3b3 exhibit geometrical isomerism. The
HOH2N NH3
compound exists both as facial and meridi-
onal isomers, both contain plane of sym-
metry
Pt

NH3 NH3 py Cl
cis
Cl NH3 NH3 Cl HOH2N NH3
Co Co
NH3 Cl Cl
Cl Pt
Cl NH3
fac- mer Cl py
cis
31. (d) Optical isomerism occurs when a molecule is
28. (b) B non-super imposable with its mirror image hence
the complex
V G cis-[Co(en)2Cl2]Cl is optically active.
+ +
en en
Cl Cl

R Y CO CO

Cl
O en
Cl
en

For a given metal ion, weak field ligands Cis-d-isomer Cis-d-isomer


www.crackjee.xyz
Co-ordination Compounds C-115
32. (d) 100 ´ 0.1
Complex Metal ion = = 0.01 mole
1000
Configuration Magnetic moment
m = n(n + 2) Moles of ions precipitated with excess of
(a) [Cr(H 2O)6 ]2+ Cr+2
1.2 ´ 1022
d4 24 AgNO3 = = 0.02 moles
(b) [Fe(H 2O)6]2+ Fe2+ 6.02 ´ 1023
d6 24 0.01 × n = 0.02
(c) [CoCl4]2– Co2+
\ n =2
d7 15
It means 2Cl– ions present in ionization
(d) [Mn(H 2O)6]2+ Mn 2+ d5 35
Since (a) and (b), each has 4 unpaired electron sphere
they will have same magnetic moment \ complex is [Co(H2O)5Cl]Cl2.H2O
33. (d) Moles of complex

Molarity ´ Volume(ml)
=
1000
EBD_7764
www.crackjee.xyz
C-116 Chemistry

Haloalkanes and Haloarenes


24
1. Bottles containing C6H5I and C6 H5CH2I lost 6. Phenyl magnesium bromide reacts with methanol
their original labels. They were labelled A and B to give [2005]
for testing. A and B were separately taken in test (a) a mixture of toluene and Mg(OH)Br
tubes and boiled with NaOH solution. The end (b) a mixture of phenol and Mg(Me)Br
solution in each tube was made acidic with dilute (c) a mixture of anisole and Mg(OH)Br
HNO3 and then some AgNO3 solution was
(d) a mixture of benzene and Mg(OMe)Br
added. Substance B gave a yellow precipitate.
7. Fluorobenzene (C6H5F) can be synthesized in
Which one of the following statements is true for
the laboratory [2006]
this experiment ? [2003]
(a) by direct fluorination of benzene with F2
(a) A and C6H5CH2I
gas
(b) B and C6H5I
(b) by reacting bromobenzene with NaF
(c) Addition of HNO3 was unnecessary
solution
(d) A was C6H5I
(c) by heating phenol with HF and KF
2. The compound formed on heating
(d) from aniline by diazotisation followed by
chlorobenzene with chloral in the presence of
heating the diazonium salt with HBF4
concentrated sulphuric acid, is [2004]
8. Reaction of trans 2-phenyl-1
(a) freon (b) DDT
bromocyclopentane on reaction with alcoholic
(c) gammexene (d) hexachloroethane KOH produces [2006]
3. Tertiary alkyl halides are practically inert to (a) 1-phenylcyclopentene
substitution by S N 2 mechanism because of (b) 3-phenylcyclopentene
[2005] (c) 4-phenylcyclopentene
(a) steric hindrance (b) inductive effect (d) 2-phenylcyclopentene
(c) instability (d) insolubility 9. The structure of the major product formed in the
following reaction [2006]
4. Alkyl halides react with dialkyl copper reagents
to give [2005] CH2Cl
NaCN
(a) alkenyl halides DMF is
(b) alkanes
(c) alkyl copper halides I
(d) alkenes CH2Cl
5. Elimination of bromine from 2-bromobutane (a)
results in the formation of – [2005]
(a) Predominantly 2-butyne CN
(b) Predominantly 1-butene CH2CN
(c) Predominantly 2-butene (b)
(d) equimolar mixture of 1 and 2-butene
I
www.crackjee.xyz
Haloalkanes and Haloarenes C-117

CH2CN
(c) CH2Br
CH2Br
CN (c) (d)
CH3
CH2Cl CH3

(d) CN 15. Among the following oxoacids, the correct


decreasing order of acid strength is: [2014]
I
(a) HOCl > HClO2 > HClO3 > HClO4
10. Which of the following is the correct order of
decreasing SN2 reactivity? [2007] (b) HClO4 > HOCl > HClO 2 > HClO3
(a) R2CH X > R3C X > RCH2 X
(c) HClO4 > HClO3 > HClO2 > HOCl
(b) RCH X > R3C X > R2CH X
(c) RCH2 X > R2CH X > R3C X (d) HClO2 > HClO 4 > HClO3 > HOCl
(d) R3C X > R2CH X > RCH2 X. 16. In SN2 reactions, the correct order of reactivity
(X is a halogen) for the following compounds: [2014]
11. The organic chloro compound, which shows
CH3Cl, CH3CH2Cl, (CH3)2CHCl and (CH3)3CCl
complete sterochemical inversion during a SN2
is:
reaction, is [2008]
(a) (C2H5)2CHCl (b) (CH3)3CCl (a) CH3 Cl > ( CH3 )2 CHCl
(c) (CH3)2 CHCl (d) CH3Cl
> CH3CH 2Cl > ( CH3 )3 CCl
12. Consider the following bromides :
Me Me (b) CH 3Cl > CH 3CH 2Cl
Me Br Me
Br Br > ( CH3 ) 2 CHCl > ( CH3 )3 CCl
(A) (B)
(c) CH3CH 2 Cl > CH3Cl
The correct order of SN1 reactivity is [2010]
(a) B > C > A (b) B > A > C > ( CH3 )2 CHCl > ( CH3 )3 CCl
(c) C > B > A (d) A > B > C
13. How many chiral compounds are possible on (d) ( CH3 )2 CHCl > CH3CH 2Cl
monochlorination of 2- methyl butane ? [2012] > CH 3Cl > ( CH3 )3 CCl
(a) 8 (b) 2
(c) 4 (d) 6 17. The major organic compound formed by the
14. Compound (A), C 8 H 9 Br, gives a white reaction of 1, 1, 1-trichloroethane with silver
precipitate when warmed with alcoholic AgNO3. powder is: [2014]
Oxidation of (A) gives an acid (B), C8H6O4. (B) (a) Acetylene (b) Ethene
easily forms anhydride on heating. Identify the (c) 2 - Butyne (d) 2 - Butene
compound (A). [2013] 18. The synthesis of alkyl fluorides is best
accomplished by : [JEE M 2015]
CH2Br C2H5 (a) Finkelstein reaction
(a) (b) (b) Swarts reaction
Br (c) Free radical fluorination
CH3 (d) Sandmeyer's reaction
EBD_7764
www.crackjee.xyz
C-118 Chemistry
19. 2-chloro-2-methylpentane on reaction with 21. Which of the following , upon treatment with
sodium methoxide in methanol yields: tert-BuONa followed by addition of bromine
[JEE M 2016] water, fails to decolourize the colour of bromine?
O
CH3
(a)
(i) C2H5CH2C OCH3 Br
C6H5
CH3
(b)
(ii) C2H5CH2C = CH2 Br
O
CH3
(c)
(iii) C2H5CH = C – CH3 Br
O
CH3
(a) (iii) only (b) (i) and (ii) (d)
(c) All of these (d) (i) and (iii) Br
22. The major product obtained in the following
20. The increasing order of the reactivity of the
reaction is : [2017 ]
following halides for the SN1 reaction is
CH3CHCH2CH3 CH3CH2CH2Cl Br
|
Cl (II) H
C6H5 t-BuOK
(I)
D
p-H3CO–C6 H4–CH2Cl C6H5
(III) (a) (±)C6H5CH(OtBu)CH2C6H5
(a) (III) < (II) < (I) (b) (II) < (I) < (III) (b) C6H5CH = CHC6H5
(c) (I) < (III) < (II) (d) (II) < (III) < (I) (c) (+)C6H5CH(OtBu)CH2C6H5
(d) (–)C6H5CH(OtBu)CH2C6H5

Answer Key
1 2 3 4 5 6 7 8 9 10 11 12 13 14 15
(d) (b) (a) (b) (c) (d) (d) (a) (b) (c) (d) (a) (c) (d) (c)
16 17 18 19 20 21 22
(b) (c) (b) (a) (b) (a) (b)

+ AgNO
1. NaOH C H ONa HNO3 /H
(d) C6H5I ¾¾ ® 6 5 ¾¾ ¾3¾® yellow ppt.
¾¾¾ ®
AgNO
Since benzyl iodide gives yellow ppt.
C6H5OH ¾¾¾¾
3
® No yellow ppt. hence this is compound B and A was
NaOH phenyl iodide (C6H5I).
C6H5CH2I ¾¾¾¾ ® C6H5CH2ONa
+ 2. (b) DDT is prepared by heating chlorbenzene
HNO3/H
¾¾¾¾¾ ® C6H5CH2OH and chloral with concentrated sulphuric
acid
www.crackjee.xyz
Haloalkanes and Haloarenes C-119
8. (a) The reaction is dehydrohalogenation
Br
CCl3CHO + 2 H Cl
alc. KOH
C6H5
C6H5
Cl 1-phenyl cyclopentene
H2SO4 9. (b)
CCl3CH
–H2O
CH2Cl CH2CN
NaCN
Cl ¾¾ ¾
¾®
DM F

1,1,1-trichloro–2,2 bis
(p-chlorophenyl) ethane
I I
or Nuclear substitution will not take place.
DDT
10. (c) In S N2 mechanism transition state is
pentavelent. For bulky alkyl group it will
3. (a) Due to steric hindrance tertiary alkyl halide have sterical hinderance and smaller alkyl
do not react by SN2 mechanism they react group will favour the SN2 mechanism. So
by SN1 mechanism. SN2 mechanisam is the decreasing order of reactivity of alkyl
followed in case of primary and secondary halides is
alkyl halides
RCH2X > R2CHX > R3CX
The order is
11. (d) SN2 reaction is favoured by small groups
CH3 – X > CH3– CH2X > (CH3)2 – CH.X >
on the carbon atom attached to halogen.
(CH3)3 – C–X
4. (b) In Corey House synthesis of alkanes alkyl So, the order of reactivity is
halide react with lithium dialkyl cuprate CH 3Cl > (CH 3 ) 2 CHCl > (CH 3 ) 3 CCl
R 'X + LiR 2 Cu ¾¾ ® R '- R + RCu + LiX > (C 2 H 5 ) 2 CHCl
Br
|
Alc. KOH NOTE SN2 reaction is shown to maximum extent by
5. (c) CH3 - CH - CH 2 - CH3 ¾¾¾¾¾
®
primary halides. The only primary halides
CH3 - CH = CH - CH 3 + HBr given is CH3Cl so the correct answer is
The formation of 2-butene is in accordance (d).
to Saytzeff’s rule. The more substituted 12. (a)
alkene is formed.
6. (d) CH 3OH + C 6 H 5 MgBr ¾¾ ® + –
Me Br ionisation +Br
CH 3O.MgBr + C6 H 6 Me
7. (d) A

NH2 N2Cl Me Me
ionisation Me
+ + Br –
Na NO + HCl HBF Br
¾ ¾ ¾ ¾2¾ ¾¾® ¾ ¾ ¾4 ®
0 - 5 º diazotisation B

Me
N2+BF4 F Me ionisation Me
+ Br

Me +
Be
D
¾¾® + BF3 + N2 C

Benzene diazonium (Balz-Schiemann Since SN1 reactions involve the formation


tetrafluoroborate reaction)
of carbocation as intermediate in the rate
EBD_7764
www.crackjee.xyz
C-120 Chemistry
determining step, more is the stability of
Cl
carbocation higher will be reactivity of |
alkyl halides towards SN1 route. Now we 17. (c) 2Cl - C - CH3 + 6Ag
know that stability of carbocations follows |
the order : 3° > 2° > 1°, so SN1 reactivity Cl
should also follow the same order.
3° > 2° > 1° > Methyl (SN1 reactivity) ¾¾
® CH3C º CCH3 + 6AgCl
2 - butyne
Cl
1, 1, 1-trichloroethane
13. (c) CH2 CH CH2 CH3 18. (b) Alkyl fluorides are more conveniently
prepared by heating suitable chloro – or
CH3 bromo-alkanes with organic fluorides such
( R + S) as AsF3, SbF3, CoF2, AgF, Hg2F2 etc. This
reaction is called Swarts reaction.
Cl CH 3Br + AgF ¾¾ ® CH 3 F + AgBr

CH3 CH CH CH3 2CH3CH 2 Cl + Hg 2 F2 ¾¾


®
2CH3CH 2 F + Hg 2Cl 2
CH3
19. (a) When tert -alkyl halides are used in
(R + S)
Williamson synthesis elimination occurs
Four monochloro derivatives are chiral. rather than substitution resulting into
formation of alkene. Here alkoxide ion
CH2Br abstract one of the b-hydrogen atom along
14. (d) Alcholic with acting as a nucleophile.
AgBr¯
CH3 AgNO3
A CH3
Oxidation +– CH3OH
CH3 CH2 CH2 C CH3 + Na OCH3
COOH CO Cl
D
O
2-Chloro-2-methylpentane
COOH CO
Acid (B) Phthalic Anhydride
H CH3
15. (c) Acidic strength increases as the CH OH
CH3CH2 C C CH3 + CH3OH + NaBr
oxidation number of central atom
increases. 2-Methyl-pent-2-ene

Hence acidic strength order is


(+7) (+5) (+3) (+1)
20. (b) Since SN1 reactions involve the formation
HClO4 > HClO3 > HClO2 > HClO
of carbocation as intermediate in the rate
16. (b) Steric congestion around the carbon atom determining step, more is the stability of
undergoing the inversion process will slow carbocation higher will be the reactivity of
down the S N 2 reaction, h ence less alkyl halides towards SN1 route.
congestion faster will the reaction. So, the
order is Since stability of carbocation follows order.
CH3Cl > (CH3)CH2 – Cl > (CH3)2CH – Cl > Å
(CH3)3CCl CH3 – CH2 – CH 2
www.crackjee.xyz
Haloalkanes and Haloarenes C-121
Å O O
< CH3 – CH – CH2 – CH3
Å (d) tert - BuONa
¾ ¾ ¾ ¾ ¾®
< p – H3CO – C6H4 – CH 2 Br
(it decolorises bromine
solution)
Hence correct order is II < I < III
Products formed in option (2), (3) & (4)
21. (a) decolorises bromine solution due to presence
of double bond.
O O
tert - BuONa 22. (b) Elimination reaction is highly favoured if
(a) ¾¾ ¾ ¾ ¾
®
Br O-tBu
(a) Bulkier base is used
(fails to decolorise the (b) Higher temperature is used
colour of bromine) Hence in given reaction biomolecular
due to unsaturation
elimination reaction provides major
C6H5 C6H5 product.
(b) ¾¾¾¾¾
®
tert - BuONa

Br Br H
(it decolorises
bromine solution)
C6H5 t-BuOK
C6H5
O O 1, 2 elimination
(c) tert - BuONa
¾ ¾ ¾ ¾ ¾® H H
Br
(it decolorises bromine OtBu
solution)

C6H5
+ t-BuOH + Br
C6H5
EBD_7764
www.crackjee.xyz
C-122 Chemistry

Alcohols, Phenols
and Ethers 25
1. During dehydration of alcohols to alkenes by
CH 3
heating with conc. H2SO4 the initiation step is
[2003]
(a)
(a) formation of carbocation CH 2 COOH
(b) elimination of water OH
(c) formation of an ester
(d) protonation of alcohol molecule CH 3
CH 2 COOH
2. Among the following compounds which can be
(b)
dehydrated very easily is [2004]
OH
CH3
|
(a) CH3 CH 2 C CH 2 CH3 CH 3
|
OH
(c)
OH CH(OH)COOH
| OH
(b) CH3CH 2 CH 2CHCH3
CH 3
(c) CH 3CH 2 CH 2 CH 2 CH 2 OH
CH(OH)COOH
(d) CH3 CH 2 CHCH 2 CH 2 OH (d)
|
CH3 OH
3. The best reagent to convert pent-3-en-2-ol into 5. HBr reacts with CH2 = CH – OCH3 under
pent-3-en-2-one is [2005] anhydrous conditions at room temperature to
(a) Pyridinium chloro-chromate give [ 2006]
(a) BrCH2 – CH2 – OCH3
(b) Chromic anhydride in glacial acetic acid
(b) H3C – CHBr – OCH3
(c) acidic dichromate
(c) CH3CHO and CH3Br
(d) Acidic permanganate
(d) BrCH2CHO and CH3OH
4. p-cresol reacts with chloroform in alkaline 6. Among the following the one that gives positive
medium to give the compound A which adds iodoform test upon reaction with I2 and NaOH
hydrogen cyanide to form, the compound B. The is [ 2006]
latter on acidic hydrolysis gives chiral carboxylic
acid. The structure of the carboxylic acid is CH3
|
[2005] (a) CH 3 - C HCH 2 OH
www.crackjee.xyz
Alcohols, Phenols and Ethers C-123
(b) PhCHOHCH3 11. The major product obtained on interaction of
(c) CH3CH2CH(OH)CH2CH3 phenol with sodium hydroxide and carbon
(d) C6H5CH2CH2OH dioxide is [2009]
7. The structure of the compound that gives a (a) salicylaldehyde (b) salicylic acid
tribromo derivative on treatment with bromine (c) phthalic acid (d) benzoic acid
water is [2006] 12. From amongst the following alcohols the one
CH 3 CH3 that would react fastest with conc. HCl and
anhydrous ZnCl2, is [2010]
OH
(a) (b) (a) 2-Butanol
(b) 2- Methylpropan-2-ol
(c) 2-Methylpropanol
OH
(d) 1- Butanol
CH3 CH2OH 13. The main product of the following reaction is
conc.H SO
C6 H5CH 2CH(OH)CH(CH 3 )2 ¾¾¾¾¾
2 4¾
®?
(c) (d)
OH [2010]
– +
(a) H 5C 6 H
OH O Na C=C
8. + CHCl3 + NaOH H CH(CH3)2
CHO
(b) C6H5CH2 CH3
The electrophile involved in the above reaction C=C
is [2006] H CH3

(c) H5C6CH2CH2
(a) trichloromethyl anion (CCl3) C = CH 2
H 3C
Å
(b) formyl cation (CHO) (d) C6H5 CH(CH3)2
Å
C=C
(c) dichloromethyl cation (CHCl2) H H
(d) dichlorocarbene (: CCl2) 14. Consider thiol anion (RSQ ) and alkoxy anion
9. In the following sequence of reactions,
(ROQ ) . Which of the following statements is

P +I Mg HCHO
correct ? [2011RS]
CH 3CH 2 OH ¾¾¾
2 ® A ¾¾¾® B ¾¾¾¾
®
ether
(a) RSQ is less basic but more nucleophilic
H O
C ¾¾¾
2 ®D
than ROQ
the compound D is [2007]
(b) RSQ is more basic and more nucleophilic
(a) propanal (b) butanal than ROQ
(c) n-butyl alcohol (d) n-propyl alcohol
(c) RSQ is more basic but less nucleophilic
10. Phenol, when it first reacts with concentrated
sulphuric acid and then with concentrated nitric than ROQ
acid, gives [2008]
(d) RSQ is less basic and less nucleophilic
(a) 2, 4, 6-trinitrobenzene
(b) o-nitrophenol than ROQ
(c) p-nitrophenol 15. The correct order of acid strength of the
(d) nitrobenzene following compounds : [2011RS]
EBD_7764
www.crackjee.xyz
C-124 Chemistry
(A) Phenol (B) p–Cresol 19. The most suitable reagent for the conversion
(C) m–Nitrophenol (D) p–Nitrophenol of R - CH 2 - OH ® R - CHO is: [2014]
(a) D > C > A > B (b) B > D > A > C (a) KMnO4
(c) A > B > D > C (d) C > B > A > D (b) K2Cr2O7
16. Consider the following reaction : (c) CrO3
C2H5OH + H2SO4 ¾¾ ® Product (d) PCC (Pyridinium Chlorochromate)
20. Sodium phenoxide when heated with CO2 under
Among the following, which one cannot be
pressure at 125ºC yields a product which on
formed as a product under any conditions ?
acetylation produces C [2014]
[2011RS]
(a) Ethylene (b) Acetylene ONa 125° H+
+ CO 2 ¾¾¾¾
® B ¾¾¾® C
(c) Diethyl ether (d) Ethyl-hydr ogen 5 Atm Ac2O
sulphate The major product C would be
17. Arrange the following compounds in order of
decreasing acidity : [2013] OCOCH3

OH OH OH OH COOH
(a)

; ; ; ;
OH

Cl CH3 NO2 OCH3 COCH3


(I) (II) (III) (IV) (b)

(a) II > IV > I > III (b) I > II > III > IV COCH3
(c) III > I > II > IV (d) IV > III > I > II
18. An unknown alochol is treated with the “Lucas OH
reagent” to determine whether the alcohol is
primary, secondary or tertiary. Which alcohol COOCH3
reacts fastest and by what mechanism : (c)
[2013]
(a) secondary alcohol by SN1
OCOCH3
(b) tertiary alcohol by SN1
(c) secondary alcohol by SN2
(d) tertiary alcohol by SN2 (d)
COOH

Answer Key
1 2 3 4 5 6 7 8 9 10 11 12 13 14 15
(d) (a) (a) (c) (b) (b) (c) (d) (d) (b) (b) (b) (a) (a) (a)
16 17 18 19 20
(b) (c) (b) (d) (a)
www.crackjee.xyz
Alcohols, Phenols and Ethers C-125

1. (d) The dehydration of alcohol to form alkene


CH3 CH3
occurs in following three step. Step (1) is
initiation step. OH HOH
Step (1) Formation of protonated alcohol. C CN CH(OH) COOH

+ H OH H OH
+
CH3–CH2 – O –H + H CH3CH2 –– O Cyanohydrin
H
(Protonated
ethanol) 5. (b) Methyl vinyl ether under anhydrous
Step (2) Formation of carbocation condition at room temperature undergoes
addition reaction.
+ H Slow +
CH3–CH2–– O ––––––® HBr
H Ethyl
CH 2= CH - OCH3 ¾¾¾
® CH3 - CH - O - CH3
|
carbocation Br
Step (3)Elimination of a proton to form
ethene 6. (b) Only those alcohols which contain
– CHOHCH3 group undergo haloform
+ Fast + reaction. Among the given options only
H– CH 2––– CH2 CH2 = CH 2+ H
(b) contain this group, hence undergo
ethene
haloform reaction.
2. (a) 3-methyl pentan-3-ol will be dehydrated
most readily since it produces tertiary 7. (c) NOTE OH group activates the benzene
carbonium ion as intermediate. nucleus and
CH3 OH OH
|
CH3 - CH 2 - C - CH 2 - CH3 Br Br
| Br2 / H 2O
¾¾ ¾ ¾
¾®
OH CH3 CH3
Br
CH3
|
H+ 8. (d) NOTE This is Riemer-Tiemann reaction
¾¾¾
® CH3 - CH 2 - C - CH 2 - CH3
Å
and the electrophile is dichlorocarbene.
OH Cl
|
3. (a) CH3 - CH - CH = CH - CH3 ¾¾
® | ••
H – C – Cl + NaOH Cl – C – Cl + NaCl + H2O
|
O Cl
dichlorocarbene
||
CH3 - C - CH = CH - CH3 a-elimination

Pyridiminum chloro-chromate (PCC) is P +I


specific for the conversion. 9. (d) CH3CH 2 OH ¾¾¾
2 ® CH CH I
3 2
A
CH3 CH3
Mg
CHCl3 + NaOH
¾¾¾® CH3CH 2 MgI
4. (c) HCN Ether
Reimer Tiemman reaction (B)
C=O

OH OH H
EBD_7764
www.crackjee.xyz
C-126 Chemistry
Mechanism
CH 2 CH3
|
HCHO
¾¾¾¾
® H - C - OMgI CH3
|
|
H Step 1 : CH3 — C — OH + H - Cl
|
(C) CH3
2 Methyl Propan-2-ol
H2 O
¾¾¾
® ∗
CH 2 CH3 ˆˆ† (CH3 )3 C — OH 2 ∗ Cl,
‡ˆˆ
|
H - C - OH ∗
| Step 2 : ˆˆ† (CH 3 )3 C∗ ∗ H 2 O
(CH3 )3 C , OH 2 ‡ˆˆ
H 3° Carbocation
(D) ∗ , ˆˆ†
n - propyl alcohol Step 3 : (CH3 )3 C ∗ Cl ‡ˆˆ (CH3 )3 C , Cl
t,Butylchloride
10. (b) Phenol on reaction with conc. H2SO4 gives 13. (a) Whenever dehydration can produce two
a mixture of o- and p- products (i.e., –SO3H different alkenes, major product is formed
group, occupies o-, p- position). At room according to Saytzeff rule i.e. more
temperature o-product is more stable, which substituted alkene (alkene having lesser
on treatment with conc. HNO3 will yield number of hydrogen atoms on the two
o-nitrophenol. doubly bonded carbon atoms) is the major
product.
OH
Such reactions which can produce two
Conc.H2SO4 or more structural isomers but one of them
in greater amounts than the other are
OH OH called regioselective ; in case a reaction
SO3H is 100% regioselective, it is termed as
Conc.H SO regiospecific.
+
In addition to being regioselective, alcohol
o- dehydrations are stereoselective (a
SO3H reaction in which a single starting
p-
At room temperature o- product is more material can yield two or more
stereoisomeric products, but gives one of
stable
them in greater amount than any other).
OH OH C6H5 ,CH2 ,CH, CH ,CH3 ¾¾¾¾¾
2 4¾

Conc.H SO
SO3H NO2 | |
Conc. HNO 3 OH CH3

o- nitrophenol H H H CH(CH3)2
Hence (b) is the correct answer. C=C + C=C
C6H5 CH(CH3)2 C6H5 H
OH OH cis trans
NaOH
11. (b) CO2
(minor) (major)
COOH 14. (a) On moving down a group, the basicity &
12. (b) Tertiary alcohols react fastest with conc. nucleophilicity are inversely related, i.e.
HCl and anhydrous ZnCl2 (lucas reagent) nucleophilicity increases while basicity
as its mechanism proceeds through the decreases. i.e RSQ is more nucleophilic but
formation of stable tertiary carbocation.
less basic than ROQ . This opposite
www.crackjee.xyz
Alcohols, Phenols and Ethers C-127
behaviour is because of the fact that – CH3, – OCH3 decreases acidity. hence
basicity and nucleophilicity depends upon the correct order of acidity will be
different factors. Basicity is directly related OH OH OH OH
to the strength of the
H–element bond, while nucleophilicity is
indirectly related to the electronegativity > > >
of the atom to which proton is attached.
NO2 Cl CH3 OCH3
OH OH OH
III I II IV
15. (a) (–M, –I) (–I > +M) (+I, + HC ) (+ M)
18. (b) Reaction of alcohols with Lucas reagent
NO2 proceeds through carbocation formation,
CH3 SN1 mechanism.
+I –I Further 3° carbocations (from tertiary
effect effect alcohols) are highly stable thus reaction
(A) (B) (C) proceeds through SN1 mechanism.
19. (d) An excellent reagent for oxidation of 1°
OH alcohols to aldehydes is PCC.
PCC
R - CH 2 - OH ¾¾¾
® R - CHO

ONa
NO2 20. (a) + CO2 ¾¾
®
– M, – I
Sodium
effect Phenoxide
(D)
OH
Electron withdrawing substituents H SO
increases the acidity of phenols; while ¾¾¾¾
2 4®

electron releasing substituents decreases COONa


acidity. Since the + I effect is maximum in
ortho position, followed by meta and least OH
in para, thus the correct order of acidity
will COOH
D > C >A> B Salicylic acid
16. (b) (CH 3 CO) 2 O
C2 H5 - OH + H2 SO4 433K CH 2 = CH 2
ethylene
413 K
CH3 - CH 2 - O - CH2 - CH3 O
diethyl ether ||
383 K O - C - CH3
CH3CH 2 HSO4 + H 2 O
ethyl hydrogen sulphate + CH3COOH
Acetylene is not formed under any COOH
conditions. Aspirin
17. (c) Electron withdrawing substituents like (Acetyl Salicylate)
–NO2, –Cl increase the acidity of phenol
while electron releasing substituents like
EBD_7764
www.crackjee.xyz
C-128 Chemistry

Aldehydes, Ketones and


Carboxylic Acids 26
Cl 2 4. When CH2 = CH — COOH is reduced with
1. ¾¾® A ¾alc.
CH3CH2COOH ¾red
KOH
¾ ¾¾® B. LiAlH4, the compound obtained will be
P [2003]
What is B? [2002] (a) CH2 = CH — CH2OH
(a) CH3CH2COCl (b) CH3CH2CHO (b) CH3 — CH2 — CH2OH
(c) CH2=CHCOOH (d) ClCH2CH2COOH. (c) CH3 — CH2 — CHO
2. On vigorous oxidation by permanganate (d) CH3 — CH2 — COOH
solution. 5. On mixing ethyl acetate with aqueous sodium
(CH3)2C = CH - CH2 - CHO gives chloride, the composition of the resultant
[2002] solution is [2004]
CH OH (a) CH3COCl+ C2H5OH + NaOH
| | (b) CH3COONa + C2H5OH
(a) CH3 - C - CH - CH 2CH 3 (c) CH3COOC2H5 + NaCl
|
CH (d) CH3Cl + C2H5COONa
6. Acetyl bromide reacts with excess of CH3MgI
CH3 followed by treatment with a saturated solution
(b)
COOH + CH3CH2COOH of NH4Cl gives [2004]
CH3 (a) 2-methyl-2propanol
CH3 (b) acetamide
(c) CH – OH + CH2CH2CH2OH (c) acetone
CH3 (d) acetyl iodide
CH3 7. Which one of the following is reduced with zinc
(d) . C = O + CH2CH2CHO and hydrochloric acid to give the corresponding
CH3 hydrocarbon? [2004]
3. Picric acid is: [2002] (a) Acetamide (b) Acetic acid
COOH (c) Ethyl acetate (d) Butan-2-one
COOH
8. Which one of the following undergoes reaction
with 50% sodium hyroxide solution to give the
(a) (b)
corresponding alcohol and acid? [2004]
NO2 OH (a) Butanal (b) Benzaldehyde
COOH (c) Phenol (d) Benzoic acid
OH 9. Among the following acids which has the lowest
O2N NO2 pK a value? [2005]
(c) (d)
NH2 (a) CH 3CH 2COOH

NO2 (b) (CH 3 ) 2 CH - COOH


www.crackjee.xyz
Aldehydes, Ketones and Carboxylic Acids C-129
(c) HCOOH 16. Iodoform can be prepared from all except: [2012]
(d) CH 3COOH (a) Ethyl methyl ketone
(b) Isopropyl alcohol
10. Reaction of cyclohexanone with dimethylamine (c) 3-Methyl 2-butanone
in the presence of catalytic amount of an acid
(d) Isobutyl alcohol
forms a compound if water during the reaction
17. In the given transformation, which of the
is continuously removed. The compound formed
following is the most appropriate reagent ?
is generally known [2005]
[2012]
(a) an amine (b) an imine
(c) an anemine (d) a Schiff’s base CH CH COCH3
11. The increasing order of the rate of HCN addition
to compound A – D is [2006] HO
(A) HCHO (B) CH3COCH3 Reagent
(C) PhCOCH3 (D) PhCOPh ¾¾¾¾®
(a) D < C < B < A (b) C < D < B < A CH CH CH2 CH3
(c) A < B < C < D (d) D < B < C < A
12. The correct order of increasing acid strenght of HO
the compounds [2006] Q
(A) CH3CO2H (B) MeOCH2CO2H (a) NH2 NH2 , OH (b) Zn – Hg/ HCl
Me (c) Na, Liq NH3 (d) NaBH4
(C) CF3CO2H (D) CO2H 18. An organic compound A upon reacting with NH3
Me
gives B. On heating B gives C. C in presence of
is KOH reacts with Br2 to given CH3CH2NH2. A
(a) D < A < B < C (b) A < D < B < C is : [2013]
(c) B < D < A < C (d) D < A < C < B (a) CH3COOH
13. A liquid was mixed with ethanol and a drop of (b) CH3CH2CH2COOH
concentrated H2SO4 was added. A compound
(c) CH 3 - CH - COOH
with a fruity smell was formed. The liquid was : |
[2009] CH3
(a) HCHO (b) CH3COCH3
(c) CH3COOH (d) CH3OH (d) CH3CH2COOH
14. Which of the following on heating with aqueous 19. In the reaction,
KOH, produces acetaldehyde? [2009] LiAlH
CH 3COOH ¾¾¾¾
4® A
(a) CH3CH2Cl (b) CH2ClCH2Cl
(c) CH3CHCl2 (d) CH3COCl PCl Alc.KOH
¾¾¾
5 ® B ¾¾¾¾® C,
15. In Cannizzaro reaction given below
the product C is: [2014]
Π
: ΝΓ .. (a) Acetaldehyde (b) Acetylene
2PhCHO ¾¾¾ ↑ PhCH 2 OH + PhCOΠ 2 (c) Ethylene (d) Acetyl chloride
the slowest step is : [2009]
(a) the transfer of hydride to the carbonyl 20. The correct sequence of reagents for the
group following conversion will be :
(b) the abstraction of proton from the O HO CH3
carboxylic group
(c) the deprotonation of Ph CH2OH
Π
(d) the attack of : OH at the carboxyl group HO–CH3
CHO CH3
EBD_7764
www.crackjee.xyz
C-130 Chemistry
(a) [Ag(NH3)2]+ OH–, H+/CH3OH, CH3MgBr OH
(b) CH3MgBr, H+/CH3OH, [Ag(NH3)2]+ OH–
(c) CH3MgBr, [Ag(NH3)2]+ OH–, H+/CH3OH (a) CHO
(d) [Ag(NH3)2]+ OH–, CH3MgBr, H+/CH3OH
COOH
21. Sodium salt of an organic acid 'X' produces
effervescence with conc. H2SO4. 'X' reacts with OH
the acidified aqueous CaCl2 solution to give a
white precipitate which decolourises acidic (b) CHO
solution of KMnO4. 'X' is :
(a) C6H5COONa (b) HCOONa CHO
(c) CH3COONa (d) Na2C2O4
22. The major product obtained in the following
(c) CHO
reaction is :
O
COOH
O

(d) CHO
DIBAL–H
CHO

COOH

Answer Key
1 2 3 4 5 6 7 8 9 10 11 12 13 14 15
(c) (b) (c) (a) (c) (a) (d) (b) (c) (c) (a) (a) (c) (c) (a)
16 17 18 19 20 21 22
(d) (a) (d) (c) (a) (d) (b)

1.
Cl2
(c) CH3CH2COOH ¾¾¾® CH3CHClCOOH CH2 = CH - CH 2OH
red P
5. (c) There is no reaction hence the resultant
alc.KOH mixture contains CH3 COOC2H5 + NaCl.
¾¾ ¾¾® CH 2 = CHCOOH
- HCl Acrylic acid

2. (b) Aldehydic group gets oxidised to O


||
carboxylic group. Double bond breaks and 6. (a) (i)CH MgI
CH 3 - C- Br ¾¾¾¾¾¾¾¾
3 ®
(ii)Saturated NH 4Cl
carbon gets oxidised to carboxylic group.
3. (c) 2,4,6-Trinitrophenol is also known as picric
CH3
acid. |
4. (a) LiAlH4 can reduce COOH group and not CH3 – C – OH
|
the double bond. CH3
2-methyl-2-propanol
LiAlH
CH 2 = CH - COOH ¾¾¾¾
4 ® CH
2
www.crackjee.xyz
Aldehydes, Ketones and Carboxylic Acids C-131
7. (d) It is Clemmensen’s reduction (Me)2CHCOOH < CH3COOH <
O MeOCH2COOH < CF3COOH
||
Zn - Hg
CH3 - C - CH 2 - CH3 ¾¾¾¾® [ NOTE Electron withdrawing groups
Conc.HCl
Butane - 2 -one increase the acid strength and electron
Zn Hg
donating groups decrease the acid
¾¾¾¾® CH3CH 2 - CH 2 CH3 strength.]
Conc.HCl
(Butane)
13. (c) Fruity smell is due to ester formation which
8. (b) This reaction is known as cannizzaro’s is formed between ethanol and acid.
reaction. In this reaction benzaldehyde in
Conc. H 2SO 4
CH3COOH + C2H5OH ¾¾¾¾¾¾
presence of 50%. NaOH undergoes ®
disproportionation reaction and form one CH3COOC2H5 + H2O
mol of Benzyl alcohol (Red. product) and
aq.KOH
14. (c) CH 3CHCl 2 ¾¾¾¾
one mole of sod. benzoate (ox. product) ® CH 3CH(OH) 2

CHO CH2OH -H O
¾¾¾¾
2 ® CH CHO
3
50% NaOH

O
+ – ||
fast
COO Na
15. (a) ˆˆˆ†
Ph – C –H + OH – ‡ˆˆˆ
+
– O
O
Ph – C – H
9. (c) pKa = –log Ka; HCOOH is the strongest Ph – C – H slow
acid and hence it has the highest Ka or
OH
lowest pKa value.
10. (c) O O–
|| |
OH Ph – C + Ph – C –H
| |
O + HN(CH3)2 N (CH3)2
OH H

–H2O
O OH
N (CH3)2 + || |
H exchange
¾¾¾¾¾¾ ® Ph – C + Ph – C –H
enamine
fast | |
O– H
11. (a) NOTE Addition of HCN to carbonyl 16. (d) Iodoform test is given by methyl ketones,
compounds is nucleophilic addition acetaldehyde and methyl secondary
reaction. The order of reactivity of carbonyl alcohols.
compounds is CH3 CH CH2 OH
Aldehydes (smaller to higher) Ketones
(smaller to higher), Then CH3
HCHO > CH3COCH3 > PhCOCH3 > PhCOPh isobutyl alcohol is a primary alcohol hence
NOTE The lower reactivity of Ketones does'nt give positive iodoform test.
17. (a) Aldehydes and ketones can be reduced to
is due to presence of two alkyl group which hydrocarbons by the action (i) of
shows +I effect. The reactivity of Ketones amalgamated zinc and concentrated
decreases as the size of alkyl group hydrochloric acid (Clemmensen reduction),
increases. or (b) of hydrazine (NH2NH2) and a strong
12. (a) The correct order of increasing acid base like NaOH, KOH or potassium tert-
strength butoxide in a high-boiling alcohol like
EBD_7764
www.crackjee.xyz
C-132 Chemistry
ethylene glycol or triethylene glycol (Wolf-
Kishner reduction) O O
CH CH COCH3
[Ag(NH3)2]OH
20. (a)
Tollens reagent
HO

NH 2NH 2/OH
CHO CO2H
Wolf-kishner
Reduction H+/CH3OH
CH CH CH2 CH3 (esterification)

HO O
–OH group and alkene are acid-sensitive
groups so clemmensen reduction can not
be used.Acid sensitive substrate should
be reacted in the Wolf-Kishner reduction
which utilise strongly basic conditions.
C
18. (d) O OCH3
NH
3 ® B ¾¾ D Br
2 ® CH CH NH
A ¾¾¾ ® C ¾¾¾¾¾ 3 2 2 CH3MgBr
(I) II KOH,(III)

Reaction (III) is a Hofmann bromamide


reaction. Now formation of CH3CH2NH2 CH3
is possible only from a compound
CH3CH2CONH2(C) which can be obtained
from the compound CH3CH2COO– NH+4
(B).
Thus (A) should be CH3CH2COOH H3C – C – CH3
O OH
P
↑ CH3CH 2COO, NH∗ 21. (d) Na2C2O4 + H2SO4 ® Na2SO4 + CO­ +
NH3
CH3CH 2 , C, OH ¾¾¾ 4
CO2­ + H2O
(A) (B)
'x' (conc.)
Χ
¾¾↑ CH 3CH 2 CONH 2 Na2C2O4 + CaCl2 ® CaC2O4¯ + 2NaCl
(C) 'x' (white ppt.)

5CaC2O4 ¯ + 2KMnO4 + 8H2SO4


KOH Br2
(purple)
CH 3CH 2 NH 2
K2SO4 + 5CaSO4 + 2MnSO4 + 10CO2 +
8H2O
19. (c) CH3 COOH ¾¾¾¾
LiA1H
4 ® CH CH OH (colourless)
3 2
(A)
22. (b) DIBAL-H is an electrophilic reducing
PCl5 agent. It reduces both ester and carboxylic
group into an aldehyde at low temperature.
CH3CH2Cl O
(B) O OH O
Alc. KOH
H
DIBAL - H
¾ ¾ ¾ ¾®
CH2 = CH2
(C) CHO
CO2H
Hence the product (C) is ethylene.
www.crackjee.xyz

Amines
27
1. When primary amine reacts with chloroform in (a) Curtius reaction (b) Wurtz reaction
ethanolic KOH then the product is [2002] (c) Hofmann method (d) Hinsberg method
(a) an isocyanide (b) an aldehyde 6. Amongst the following the most basic
(c) a cyanide (d) an alcohol. compound is [2005]
2. The reaction of chloroform with alcoholic KOH (a) p -nitroaniline (b) acetanilide
and p-toluidine forms [ 2003] (c) aniline (d) benzylamine
7. An organic compound having molecular mass
60 is found to contain C = 20%, H = 6.67% and
(a) H3C N2Cl
N = 46.67% while rest is oxygen. On heating it
gives NH3 alongwith a solid residue. The solid
residue give violet colour with alkaline copper
(b) H3C NHCHCl2 sulphate solution. The compound is [2005]
(a) CH 3CH 2CONH 2 (b) ( NH 2 ) 2 CO
(c) CH 3CONH 2 (d) CH 3 NCO
(c) H3C NC
8. Which one of the following is the strongest
base in aqueous solution ?
[2007]
(d) H3C CN (a) Methylamine (b) Trimethylamine
(c) Aniline (d) Dimethylamine
3. The correct order of increasing basic nature for
9. In the chemical reaction,
the bases NH3, CH3NH2 and (CH3)2NH is
[ 2003] CH3CH2NH2 + CHCl3 + 3KOH ®
(a) (CH3)2NH < NH3 < CH3NH2 (A) + (B) + 3H2O, the compounds (A) and (B)
are respectively [2007]
(b) NH3 < CH3NH2 < (CH3)2NH
(a) C2H5NC and 3KCl
(c) CH3NH2 < (CH3)2NH < NH3
(b) C2H5CN and 3KCl
(d) CH3NH2 < NH3 < (CH3)2NH
(c) CH3CH2CONH2 and 3KCl
4. Ethyl isocyanide on hydrolysis in acidic medium
generates [2003] (d) C2H5NC and K2CO3.
(a) propanoic acid and ammonium salt 10. In the chemical reactions,
(b) ethanoic acid and ammonium salt NH 2
(c) methylamine salt and ethanoic acid
(d) ethylamine salt and methanoic acid NaNO2 HBF 4
HCl, 278 K
A B
5. Which one of the following methods is neither
meant for the synthesis nor for separation of
amines? [2005]
EBD_7764
www.crackjee.xyz
C-134 Chemistry
the compounds ‘A’ and ‘B’ respectively are NaNO /HCl CuCN/KCN
[2010] ¾¾¾¾¾2
0–5° C
® D ¾¾¾¾¾
D
® E + N2
(a) nitrobenzene and fluorobenzene the product E is :
(b) phenol and benzene
CN
(c) benzene diazonium chloride and
fluorobenzene
(d) nitrobenzene and chlorobenzene (a)
11. In the chemical reactions : [2011RS]
NH 2 CH3

NaNO CuCN CH3


¾¾¾¾¾ 2 ® A ¾¾¾¾® B,
HCl, 278K D

the compounds A and B respectively are : (b)


(a) Benzene diazonium chloride and
benzonitrile COOH
(b) Nitrobenzene and chlorobenzene
(c) Phenol and bromobenzene
(c)
(d) Fluorobenzene and phenol
12. A compound with molecular mass 180 is acylated
with CH3COCl to get a compound with molecular CH3
mass 390. The number of amino groups present
per molecule of the former compound is : (d) H3C CH3
[2013]
(a) 2 (b) 5 16. In the Hofmann bromamide degradation
(c) 4 (d) 6 reaction, the number of moles of NaOH and Br 2
13. On heating an aliphatic primary amine with used per mole of amine produced are :
[JEE M 2016]
chloroform and ethanolic potassium hydroxide,
the organic compound formed is: [2014] (a) Two moles of NaOH and two moles of Br 2.
(a) an alkanol (b) an alkanediol (b) Four moles of NaOH and one mole of Br 2.
(c) an alkyl cyanide (d) an alkyl isocyanide (c) One mole of NaOH and one mole of Br 2.
14. Considering the basic strength of amines in (d) Four moles of NaOH and two moles of Br 2.
aqueous solution, which one has the smallest 17. Which of the following compounds will form
pKb value? [2014] significant amount of meta product during mono-
(a) (CH3)2NH (b) CH3NH2 nitration reaction ? [2017]
(c) (CH3)3N (d) C6H5NH2 OH OCOCH3
15. In the reaction [JEE M 2015]
(a) (b)
NH2
NH2 NHCOCH3

(c) (d)
CH3
www.crackjee.xyz
Amines C-135

Answer Key
1 2 3 4 5 6 7 8 9 10 11 12 13 14 15
(a) (c) (b) (d) (b) (d) (b) (d) (a) (c) (a) (b) (d) (a) (a)
16 17
(b) (c)

1. (a) C2H5NH2 + CHCl3 + 3KOH ..


CH2NH2
® C2H5N º C + 3KCl + 3HCl 6. (d) Benzylamine is most
(Ethyl isocyanide)

NH2 basic. In others the basic character is


suppressed due to Resonan ce (see
2. (c) +CHCl3+ 3KOH applications of resonance).
7. (b)
CH3
(a) (b) (c) (d)
N=C C 20% 20/12 = 1.66 1.66 / 1.66 = 1
H 6.67% 6.67 / 1 = 6.67 6.67 / 1.66 = 4.16
+3KCl+3H2 O
N 46.67% 46.67/14 = 3.33 3.33 / 1.66 = 2.02
CH3
O 26.64% 26.64 / 16 = 1.66 1.66 / 1.66 = 1.0
3. (b) The alkyl groups are electron releasing The compound is CH4N2O
group (+ I), thus increases the electron Empirical weight = 60; Mol. wt. = 60;
density around the nitrogen thereby
60
increasing the availability of the lone pair \n = =1
of electrons to proton or lewis acid and 60
making the amine more basic. Hence more
O
the no. of alkyl group more basic is the ||
amine. Therefore the correct order is Molecular formula = CH4N2O; NH 2 - C - NH 2
NH3 < CH3NH2 < (CH3)2 NH
On heating urea loses ammonia to give Biuret
4. (d) Ethyl isocyanide on hydrolysis form
primary amines. 2NH 2CONH 2 ¾¾
® H 2 NCO.NH.CONH 2 + NH 3
+ Biuret with alkaline CuSO4 gives violet colour.
H
= C + H2 O ¾¾¾
CH3CH2 N ® ®
Test for –CONH– group.
CH3CH2NH2 + HCOOH
8. (d) NOTE Aromatic amines are less basic than
Therefore it gives only one mono
chloroalkane. aliphatic amines. Among aliphatic amines the
5. (b) Wurtz reaction is for the preparation of order of basicity is 2° > 1° > 3°. The electron
hydrocarbons from alkyl halide density is decreased in 3° amine due to
crowding of alkyl group over N atom which
RX + 2 Na + XR ¾
¾® R – R + 2 NaX makes the approach and bonding by a proton
EBD_7764
www.crackjee.xyz
C-136 Chemistry
relatively difficult. Therefore the basicity O
decreases. Further Phenyl group show – I P
12. (b) R - NH 2 + CH3 - C - Cl ¾¾¾®
effect, thus decreases the electron density { - HCl
on nitrogen atom and hence the basicity. Mol.Mass=16

\ dimethylamine (2° aliphatic amine) is O


P
strongest base among given choices. R - NH - C - CH3
\ The correct order of basic strength is 144244 3
Mol.mass =58
Dimethylamine > Methyl amine > Trimethyl
amine > Aniline. Now since the molecular mass increases by 42
9. (a) This is carbylamine reaction. unit as a result of the reaction of one mole of
CH3CH2NH2 + CHCl3 + 3KOH CH3COCl with one-NH2 group and the given
increase in mass is 210. Hence the number of
¾¾ ® C2H5NC + 3KCl + 3H2O –NH2 groups is = 210/42 = 5.
10. (c) Primary aromatic amines react with nitrous 13. (d) R – CH2 – NH2 + CHCl3 + 3KOH (alc)
acid to yield arene diazonium salts. Carbyl amine reaction
ArNH2 + NaNO2 + 2HX ¾¾¾ cold
® ¾¾
® R – CH2 – NC + 3KCl + 3H2O
1° Aromatic amine Alkyl isocynide
Ar—N = N+X– + NaX + 2H2O
14. (a) Arylamines are less basic than alkyl amines
Arene diazonium salt
and even ammonia. This is due to
The diazonium group can be replaced by
resonance. In aryl amines the lone pair of
fluorine by treating the diazonium salt with
electrons on N is partly shared with the
fluoroboric acid (HBF4). The precipitated
ring and is thus less available for sharing
diazonium fluoroborate is isolated, dried and
with a proton.
heated until decomposition occurs to yield
the aryl fluoride. This reaction is known as In alkylamines, the electron releasing alkyl
Balz-Schiemann reaction. group increases the electron density on
nitrogen atom and thus also increases the
Ar—N2+X– ¾¾¾®
HBF4 Ar—N2+BF4–¯ ¾¾¾
heat
® ability of amine for protonation. Hence more
Ar—F + BF3 + N2 the no. of alkyl groups higher should be
11. (a) the basicity of amine. But a slight
NH 2 N +2 Cl- discrepancy occurs in case of trimethyl
amines due to steric effect. Hence the
correct order is
NaNO2 CuCN
HCl, 278K (CH 3 )2 NH > CH3 NH 2
Diazotization Benzene diazonium > (CH3 )3 N > C6 H5 NH 2
chloride
(A) 15. (a)

C ºN + –
NH2 N = NCl CN

¾ NaNO
¾ ¾ 2¾/HCl
¾¾ ® ¾ CuCN/KCN
¾ ¾ ¾ ¾®
0 - 5° C D

Benzonitrile CH3 CH3 CH3


(B)
Sandmayer reaction
www.crackjee.xyz
Amines C-137
16. (b) 4 moles of NaOH and one mole of Br2 is In strongly acidic nitration medium, the
required during production of one mole amine is converted into anilinium ion
of amine during Hoffmann's bromamide (– NH3+); substitution is thus controlled
degradation reaction. not by – NH2 group but by – NH3+ group
which, because of its positive charge,
O
directs the entering group to the meta-
||
position instead of ortho, and para.
R–C–NH2 + Br2 + 4NaOH ® R–NH2 + K2CO3 + 2NaBr
+ 2H2O Å
: NH2 NH3
17. (c) Nitration takes place in presence of
concentrated
HNO3 + concentrated H2SO4 Conc. HNO
¾ Conc. 3
¾ ¾H¾SO¾®
2 4

– NH2 gp : o,p -directing – NH+3 gp : m-directing


EBD_7764
www.crackjee.xyz
C-138 Chemistry

Biomolecules
28
1. RNA is different from DNA because RNA
(b) C1' and C5' respectively of the sugar
contains [2002]
(a) ribose sugar and thymine molecule
(b) ribose sugar and uracil (c) C '2 and C5' respectively of the sugar
(c) deoxyribose sugar and thymine
molecule
(d) deoxyribose sugar and uracil.
2. Complete hydrolysis of cellulose gives [2003] (d) C5' and C '2 respectively of the sugar
(a) D-ribose (b) D-glucose molecule
(c) L-glucose (d) D-fructose 8. The term anomers of glucose refers to [2006]
3. The reason for double helical structure of DNA (a) enantiomers of glucose
is operation of [2003] (b) isomers of glucose that differ in
(a) dipole-dipole interaction configuration at carbon one (C-1)
(b) hydrogen bonding (c) isomers of glucose that differ in
(c) electrostatic attractions configurations at carbons one and four
(d) van der Waals’ forces (C-1 and C-4)
4. Which base is present in RNA but not in DNA ? (d) a mixture of (D)-glucose and (L)-glucose
(a) Guanine (b) Cytosine [2004] 9. The pyrimidine bases present in DNA are [2006]
(c) Uracil (d) Thymine (a) cytosine and thymine
5. Insulin production and its action in human body (b) cytosine and uracil
are responsible for the level of diabetes. This (c) cytosine and adenine
compound belongs to which of the following (d) cytosine and guanine
categories ? [2004] 10. The secondary structure of a protein refers to
(a) An enzyme (b) A hormone [2007]
(c) A co-enzyme (d) An antibiotic (a) fixed configuration of the polypeptide
6. Which of the following is a polyamide? [2005] backbone
(a) Bakelite (b) Terylene (b) a– helical backbone
(c) Nylon-66 (d) Teflon (c) hydrophobic interactions
7. In both DNA and RNA, heterocylic base and (d) sequence of a– amino acids.
phosphate ester linkages are at – [2005] 11. a - D-(+)-glucose and b-D-(+)-glucose are [2008]
(a) C5' and C1' respectively of the sugar (a) conformers (b) epimers
(c) anomers (d) enatiomers
molecule
www.crackjee.xyz
Biomolecules C-139

12. The two functional groups present in a typical (a) Quinoline (b) Adenine
carbohydrate are: [2009] (c) Cytosine (d) Thymine
(a) – CHO and – COOH 18. Which of the vitamins given below is water
(b) > C = O and – OH soluble ? [2015]
(c) – OH and – CHO (a) Vitamin E (b) Vitamin K
(d) – OH and – COOH (c) Vitamin C (d) Vitamin D
13. Biuret test is not given by [2010] 19. Thiol group is present in : [2016]
(a) Cysteine (b) Methionine
(a) carbohydrates (b) polypeptides
(c) Cytosine (d) Cystine
(c) urea (d) proteins
20. Which of the following compounds will behave
14. Which of the following compounds can be
as a reducing sugar in an aqueous KOH solution?
detected by Molisch's Test ? [2012]
(a) Nitro compounds
HOH2C O CH2OH
(b) Sugars
(c) Amines (a) HO OCOCH
3
(d) Primary alcohols
15. Which one of the following statements is OH
correct? [2012]
(a) All amino acids except lysine are optically HOH2C O CH2OH
active
(b) All amino acids are optically active (b) HO
(c) All amino acids except glycine are optically
active OH
(d) All amino acids except glutamic acids are
optically active HOH2C O CH2OH
16. Synthesis of each molecule of glucose in
photosynthesis involves : [2013] (c) HO OCH
3
(a) 18 molecules of ATP
OH
(b) 10 molecules of ATP
(c) 8 molecules of ATP
(d) 6 molecules of ATP HOH2C
17. Which one of the following bases is not present O CH2OCH3
in DNA? [2014] (d) OH
OH
OH
Answer Key
1 2 3 4 5 6 7 8 9 10 11 12 13 14 15
(b) (b) (b) (c) (b) (c) (b) (b) (a) (b) (c) (c) (a) (b) (c)
16 16 17 18 19 20
(b) (a) (a) (c) (a) (a)
EBD_7764
www.crackjee.xyz
C-140 Chemistry

1. (b) In RNA, the sugar is D–ribose and base is


uracil where as in DNA, the sugar is D-2
deoxyribose and the nitrogenous base is
thymine.
2. (b) Cellulose is a linear polymer of
b –D– glucose in which C1 of one glucose
unit is connected to C4 of the other through
b –D glucosidic linkage. It does not
undergo hydrolysis easily. However on
heating with dilute H2SO4 under pressure.
It does undergo hydrolysis to give only
D– glucose.
H+
(C6 H10O5 )n+nH 2O ¾¾® nC6 H12O6
D-Glucose
3. (b) DNA consists of two polynucleotide
chains, each chain forms a right handed
spiral with ten bases in one turn of the
spiral. The two chains coil to double helix
and run in opposite direction held together
by hydrogen bonding.

4. (c) RNA contains cytosine and uracil as pyrimidine bases while DNA has cytosine and thymine.
Both have the same purine bases i.e., Guanine and adenine.
5. (b) Insulin is a biochemically active peptide harmone secreted by pancreas.
6. (c) Nylon is a general name for all synthetic fibres forming polyamides.
7. (b) In DNA and RNA heterocyclic base and phosphate ester are at C 1' and C5' respectively of the sugar
molecule.

HO N
| N
5
HO – P – O – C H 2 4¢ O N
|| N
C
O C¢
H H
H | |
C3––– C2 H
| |
OH OH

8. (b) Cyclization of the open chain structure of D-(+)-glucose has created a new stereocenter at C 1 which
explains the existence of two cyclic forms of D-(+)-glucose, namely a– and b–. These two cyclic
forms are diasteromers, such diastereomers which differ only in the configuration of chiral carbon
developed on hemiacetal formation (it is C1 in glucose and C2 in fructose) are called anomers and
the hemiacetal carbon (C1 or C2) is called the anomeric carbon.
www.crackjee.xyz
Biomolecules C-141

9. (a) The pyrimidine bases present in DNA are 13. (a) Biuret test produces violet colour on
cytosine and thymine. addition of dilute CaSO 4 to alkaline
10. (b) The secondary structure of a protein refers solution of a compound containing
to the shape in which a long peptide chain can peptide linkage.
exist. There are two different conformations of Polypeptides, proteins and urea have
the peptide linkage present in protein, these
- C- NH - (peptide) linkage while
are a-helix and b-conformation. The a-helix ||
always has a right handed arrangement. In O

b-conformation all peptide chains are carbohydrates have glycosidic llinkages.


streched out to nearly maximum extension So, test of carbohydtrates should be
and then laid side by side and held together different from that of other three.
by intermolecular hydrogen bonds. The 14. (b) Molisch's Test : This is a general test for
structure resembles the pleated folds of carbohydrates. One or two drops of alcoholic
drapery and therefore is known as b-pleated solution of a-naphthol is added to 2 ml
sheet. glucose solution. 1 ml of conc. H2SO4
11. (c) Since a - D - (+) - glucose and solution is added carefully along the sides of
the test-tube. The formation of a violet ring
b - D - (+) glucose differ in at the junction of two liquids confirms the
configuration at C – 1 atom so they are presence of a carbohydrate or sugar.
anomers. 15. (c) With the exception of glycine all the 19 other
NOTE Anomers are those diastereomers common amino acids have a uniquely
different functional group on the central
that differ in configuration at C – 1 atom.
tetrahedral alpha carbon.
i.e., (c) in the correct answer.
H
12. (c) NOTE Glucose is considered as a typical |
H — C — COOH
carbohydrate which contains –CHO and |
NH 2
–OH group.
glycine
EBD_7764
www.crackjee.xyz
C-142 Chemistry
16. (a) 6CO2 + 12NADPH + 18ATP ® C6H12O6 + 20. (a)
12NADP + 18ADP
17. (a) DNA contains ATGC bases HOCH2 O CH2OH HOCH2 O CH2OH
O ¾¾¾¾¾Aq.KOH
So quinoline is not present in DNA. ®
HO – CH3COOK HO –
18. (c) Water-soluble vitamins dissolve in water O – C – CH3 O
and are not stored by the body. The water OH
OH OH
soluble vitamins include the vitamin B- Hemiketal
complex group and vitamin C. Ring opening
19. (a) Among 20 naturally occuring amino acids
HOCH2 OH
"Cysteine" has '– SH' or thiol functional CH2OH
group. Å vesilver
mirror
Tollen 's
¬¾¾¾ ¾
OH O
Reagent
test
Þ General formula of amino acid OH
® R–CH—COOH—NH2 (a Reducing sugar) a-hydroxy ketone
Þ Value of R = –CH2–SH in Cysteine.
www.crackjee.xyz
Polymers C-143

Polymers
29
1. Polymer formation from monomers starts by (a) teflon (b) nylon 6, 6
[2002] (c) polystyrene (d) natural rubber
(a) condensation reaction between monomers 7. Thermosetting polymer, Bakelite is formed by
(b) coordinate reaction between monomers the reaction of phenol with : [2011RS]
(c) conversion of monomer to monomer ions (a) CH3CHO (b) HCHO
by protons (c) HCOOH (d) CH3CH2CHO
(d) hydrolysis of monomers. 8. The species which can best serve as an initiator
2. Nylon threads are made of [2003] for the cationic polymerization is : [2012]
(a) polyester polymer (a) LiAlH4 (b) HNO3
(b) polyamide polymer (c) AlCl3 (d) BaLi
(c) polyethylene polymer 9. Which one is classified as a condensation
(d) polyvinyl polymer polymer? [2014]
3. Which of the following is fully fluorinated (a) Dacron (b) Neoprene
polymer? [2005]
(a) PVC (b) Thiokol (c) Teflon (d) Acrylonitrile
(c) Teflon (d) Neoprene 10. Which polymer is used in the manufacture of
4. Bakelite is obtained from phenol by reacting with paints and lacquers ? [2015]
[2008] (a) Polypropene (b) Polyvinyl chloride
(a) (CH2OH)2 (b) CH3CHO
(c) CH3 COCH3 (d) HCHO (c) Bakelite (d) Glyptal
5. Buna-N synthetic rubber is a copolymer of : 11. Which of the following statements about low
[2009] density polythene is FALSE? [2016]
(a) H2C = CH – CH = CH2 and H5C6 – CH = CH2
(a) Its synthesis requires dioxygen or a
(b) H2C = CH – CN and H2C = CH – CHCH2 peroxide initiator as a catalyst.
(c) H2C = CH – CN and H 2 C = CH – C = CH 2
| (b) It is used in the manufacture of buckets,
C H3 dust-bins etc.
Cl (c) Its synthesis requires high pressure.
|
(d) H 2 C = CH – C = CH 2 and (d) It is a poor conductor of electricity.

H 2C = CH – CH = CH 2 12. The formation of which of the following


6. The polymer containing strong intermolecular polymers involves hydrolysis reaction?[2017]
forces e.g. hydrogen bonding, is [2010] (a) Nylon 6 (b) Bakelite
(c) Nylon 6, 6 (d) Terylene

Answer Key
1 2 3 4 5 6 7 8 9 10 11 12
(a) (b) (c) (d) (b) (b) (b) (c) (a) (d) (b) (a)
EBD_7764
www.crackjee.xyz
C-144 Chemistry

1. (a) Polymerisation starts either by


condensation or addition reactions
between monomers. Condensation
polymers are formed by the combination
of monomers with the elimination of simple
molecules.Where as the addition polymers
are formed by the addition together of the
molecules of the monomer or monomers to
form a large molecule without elimination
of any thing.
2. (b) Nylon is a polyamide polymer.
3. (c) Teflon is polymer of CF2 = CF2.
4. (d) Bakelite is formed by the reaction of 5. (b) Buna – N is a copolymer of butadiene
formaldehyde (HCHO) and phenol so the (CH2 = CH – CH = CH2) and acrylonitrile
correct answer is (d). (CH2 = CHCN).
OH 6. (b) Nylon 6, 6 has amide linkage capable of
forming hydrogen bonding.

OH
n + n HCHO

O
OH H2N
HO + NH2
O
Adipic acid Hexamethylenediamine
H2O

H
O O
H2N N OH
NH2 + HO NH2 + HO
O O
Hexamethylenediamine Adipic acid

H2O H2O
Polymerization

H H
O O
N N
N N
O
H H
Nylon

7. (b)
8. (c) Lewis acids are the most common compounds used for initiation of cationic polymerisation. The
more popular Lewis acids are SnCl4, AlCl3, BF3 and TiCl4.
www.crackjee.xyz
Polymers C-145
9. (a) Except Dacron all are additive polymers. dustbins, bottles, pipes etc. Low density
Terephthalic acid condenses with ethylene polythene is used for insulating electric wires
glycol to give Dacron. and in the manufacture of flexible pipes, toys,
coats, bottles etc.

HOOC COOH + 12. (a) Formation of Nylon-6 involves hydrolysis


of caprolactum, (its monomer) in initial
Terephthalic acid state.

– CH2 – CH2 – OH ¾¾
HO O ® O
Ethylene glycol
NH H 2O
H2 N(CH2)5COOH
[
CO CH 2 - CH 2 - O - ] n Caprolactam e-Amino Caproic acid

Dacron (Polyester)
O O
10. (d) Glyptal is used in the manufacture of paints
and lacquers. –(NH(CH2)5–C–NH–(CH2)5–C)–n
Polymerise
11. (b) High density polythene is used in the Nylon-6
manufacture of housewares like buckets,
EBD_7764
www.crackjee.xyz

Chemistry in
Everyday Life 30
OCOCH3 4. Aspirin is known as : [2012]
COOH (a) Acetyl salicylic acid
1. The compound is used as [2002] (b) Phenyl salicylate
(c) Acetyl salicylate
(a) antiseptic (b) antibiotic (d) Methyl salicylic acid
(c) analgesic (d) pesticide. 5. Which of the following compounds is not an
2. Which of the following could act as a propellant antacid ? [2015]
for rockets? [2003] (a) Phenelzine
(a) Liquid oxygen + liquid argon (b) Ranitidine
(b) Liquid hydrogen + liquid oxygen (c) Aluminium hydroxide
(c) Liquid nitrogen + liquid oxygen (d) Cimetidine
6. Which of the following is an anionic detergent?
(d) Liquid hydrogen + liquid nitrogen [2016]
3. Which one of the following types of drugs (a) Cetyltrimethyl ammonium bromide.
reduces fever ? [2005] (b) Glyceryl oleate.
(a) Tranquiliser (b) Antibiotic (c) Sodium stearate.
(c) Antipyretic (d) Analgesic (d) Sodium lauryl sulphate.
Answer Key
1 2 3 4 5 6
(c) (b) (c) (a) (a) (d)

1. (c) The given compound is aspirin which is 5. (a) Phenelzine is an antidepressant, while
antipyretic and analgesic others are antacids.
2. (b) Liquid hydrogen and liquid oxygen are used 6. (d) Sodium lauryl sulphate (C11H23CH2OSO–
+
as excellent fuel for rockets. H2(l) has low mass 3Na ) is an anionic detergent. Glyceryl
and high enthalpy of combustion whereas oleate is a glyceryl ester of oleic acid.
oxygen is a strong supporter of combustion. Sodium stearate (C17H35COO–Na+) is a
3. (c) An antipyretic is a drug which is responsible soap. Cetyltrimethyl ammonium bromide
for lowering the temperature of the feverish é + ù -
organism to normal but has no effect on ê CH3 (CH 2 )15 N(CH 3 )3 ú Br
normal temperature states. ë û
O–COCH3 is a cationic detergent.
COOH
4. (a) Aspirin (Acetyl salicylic acid)
www.crackjee.xyz
Analytical Chemistry C-147

Analytical Chemistry 31
1. When H2S is passed through Hg2S we get (a) Fe4[Fe(CN)6]3 (b) Na3[Fe(CN)6]
[2002] (c) Fe(CN)3 (d) Na4[Fe(CN)5NOS]
(a) HgS (b) HgS + Hg2S 5. 29.5 mg of an organic compound containing
(c) Hg2S + Hg (d) None of these. nitrogen was digested according to Kjeldahl’s
2. How do we differentiate between Fe3+ and Cr3+ method and the evolved ammonia was absorbed
in group III? [2002] in 20 mL of 0.1 M HCl solution. The excess of
(a) by taking excess of NH4OH solution the acid required 15 mL of 0.1 M NaOH solution
(b) by increasing NH4+ ion concentration for complete neutralization. The percentage of
nitrogen in the compound is [2010]
(c) by decreasing OH– ion concentration
(a) 59.0 (b) 47.4
(d) both (b) and (c)
(c) 23.7 (d) 29.5
3. Which one of the following statements is correct ?
6. For the estimation of nitrogen, 1.4 g of an organic
[2003]
compound was digested by Kjeldahl method and
(a) From a mixed precipitate of AgCl and AgI,
the evolved ammonia was absorbed in 60 mL of
ammonia solution dissolves only AgCl
(b) Ferric ions give a deep green precipitate M
sulphuric acid. The unreacted acid required
on adding potassium ferrocyanide solution 10
(c) On boiling a solution having K+, Ca2+ and
M
HCO 3- ions we get a precipitate of 20 mL of sodium hydroxide for complete
10
K2Ca(CO3)2
neutralization. The percentage of nitrogen in the
(d) Manganese salts give a violet borax bead
compound is: [2014]
test in the reducing flame
4. The compound formed in the positive test for (a) 6% (b) 10%
nitrogen with the Lassaigne solution of an (c) 3% (d) 5%
organic compound is [2004]

Answer Key
1 2 3 4 5 6
(c) (b) (a) (a) (c) (b)
EBD_7764
www.crackjee.xyz
C-148 Chemistry

1. (c) When H2S is passed through Hg2S we get 5. (c) Moles of HCl taken = 20 × 0.1 × 10– 3
a mixture of mercurous sulphide and = 2 × 10–3
mercury (Hg 2S + Hg) . Moles of HCl neutralised by NaOH solution
2. (b) When we add NH4Cl, it suppresses the = 15 × 0.1 × 10–3 = 1.5 × 10–3
ionisation of NH4OH and prevents the Moles of HCl neutralised by ammonia
precipitation of higher group hydroxide in
= 2 × 10–3 – 1.5 × 10–3
gp(III).
= 0.5 × 10–3
NOTE Further ferric chloride and
1.4 ´ N ×V
chromium chloride form different colour % of nitrogen = ´ 100
w.t. of Substance
precipitates with NH4OH.
® Fe(OH)3 ¯ + 3NH4Cl
FeCl3 + 3NH4OH ––––– 1.4 ´ 0.5 ´ 10 -3
= ´ 100
reddish brown 29.5 ´ 10 -3
® Cr(OH)3 + 3NH4Cl
CrCl3 + 3NH4OH –––––
= 23.7%
Bluish green.
3. (a) Between AgCl and AgI, AgI is less soluble, 1.4 ´ meq. of acid
hence ammonia can dissolve ppt. of AgCl 6. (b) % of N =
mass of organic compound
only due to formation of complex as given
below: M
AgCl + 2NH3 ® [Ag (NH3)2]Cl meq. of H2SO4 = 60´ ´ 2 = 12
10
4. (a) Prussian blue Fe 4 [Fe(CN) 6 ]3 is formed
M
in lassaigne test for nitrogen. meq. of NaOH = 20´ = 2
10
3Na 4 [Fe(CN) 6 + 4Fe3+ ¾¾
® \ meq. of acid consumed = 12 – 2 = 10

Fe4 [Fe(CN)4 ]6 + 12Na + 1.4 ´ 10


Prussian blue \ % of N = = 10%
1.4
www.crackjee.xyz

Topic-wise Solved Papers Mathematics

Sets 1
1. If A, B and C are three sets such that (a) 52 (b) 35
A Ç B = A Ç C and A È B = A È C , then (c) 25 (d) 53
[2009] æ1 ö
(a) A = C (b) B = C
3. ç ÷÷ = 3x , x ¹ 0 and
If f(x) + 2f ç
èx ø
S = {x Î R : f(x) = f(–x)}; then S: [2016]
(c) AÇB = f (d) A = B
(a) contains exactly two elements.
2. Let X ={1,2,3,4,5}. The number of different (b) contains more than two elements.
ordered pairs (Y,Z) that can formed such that (c) is an empty set.
Y Í X , Z Í X and Y Ç Z is empty is : [2012] (d) contains exactly one element.

Answer Key
1 2 3
(b) (b) (a)

1. (b) Let x Î A and x Î B Û x Î A È B æ1ö 1


Adding (1) and (2) Þ f (x) + f ç ÷=x+
Û x Î A È C (Q A È B = A È C ) èxø x
Û x ÎC \ B = C. Substracting (1) from (2)
Let x Î A and x Î B Û x Î A Ç B æ1ö 3
Þ f (x) - f ç ÷ = - 3x
Û x Î A Ç C (Q A Ç B = A Ç C ) èxø x
Û x ÎC \B = C On adding the above equations
2. (b) Let X = {1,2,3,4,5} 2
Total no. of elements = 5 Þ f (x) = -x
x
Each element has 3 options. Either set Y or
set Z or none. (Q Y Ç Z = f) 2 -2 2
f (x) = f (- x) Þ -x= +xÞx =
So, number of ordered pairs = 35 x x x
æ1ö
3. (a) f (x) + 2f ç ÷ = 3x .......(1) x2 = 2 or x = 2, - 2
èxø
1 3
f ( ) + 2f (x) = .......(2)
x x
EBD_7764
www.crackjee.xyz

Relations
and Functions 2
1. Domain of definition of the function 3. The graph of the function y = f(x) is symmetrical
3 about the line x = 2, then [2004]
f ( x) = + log10 ( x 3 - x) , is [2003]
4 - x2 (a) f ( x ) = - f (- x )
(a) ( -1,0) È (1,2) È ( 2, ¥) (b)
(a, 2) (b) f (2 + x ) = f (2 - x )
(c) ( -1,0) È (a,2) (d) (1,2) È (2, ¥) . (c) f ( x ) = f ( - x)
2. If f : R ® R satisfies f ( x + y ) = f ( x) + f ( y ) , (d) f ( x + 2) = f ( x - 2)
n
for all x, y Î R and f(1) = 7, then S f (r ) is 1
r=1 4. The domain of the function f ( x) = is
[2003] x -x
7 n (n + 1) 7n
(a) (b) [2011]
2 2 (a) (0, ¥ )
7 (n + 1)
(b) (– ¥ , 0)
(c) (d) 7n + (n + 1) . (c) (– ¥ , ¥ ) – {0} (d) (– ¥, ¥)
2
Answer Key
1 2 3 4
(a) (a) (b) (b)

3 Y
1. (a) f ( x) = 2
+ log10 ( x 3 - x) -x x
4- x
4 - x 2 ¹ 0; x 3 - x > 0;
x ¹ ± 4 and - 1 < x < 0 or 1 < x < ¥
– +
+ –
–1 0 1
X
{ 4} x1 x2
\ D = ( -1, 0) È (1, ¥) -
x=2
D = ( -1, 0) È (1, 2) È (2, ¥). From the figure
2. (a) f ( x + y ) = f (x ) + f ( y ) . f ( x1 ) = f ( x2 ), where x1 = 2 - x
Function should be f (x) = mx and x2 = 2 + x
f (1) = 7; \ m = 7, f ( x ) = 7 x \ f (2 - x) = f (2 + x)
n n
7 n ( n + 1)
S f (r ) = 7 S r = 1
4. (b) f ( x) = , define if | x | – x > 0
r =1 12 x -x
3. (b) Let us consider a graph symm. with respect
to line x = 2 as shown in the figure. Þ | x | > x, Þ x < 0
Hence domain of f(x) is (– ¥, 0)
www.crackjee.xyz

Trigonometric
Functions 3
1. The number of solution of tan x + sec x = 2cos x
in [0, 2 p ) is [2002] (4 + 7 ) (1 + 7 )
(c) – (d)
(a) 2 (b) 3 3 4
(c) 0 (d) 1 6. Let A and B denote the statements
2. Let a, b be such that p < a - b < 3p . A : cos a + cos b + cos g = 0
21 27 B : sin a + sin b + sin g = 0
If sin a + sin b = - and cos a + cos b = - , 3
65 65 If cos (b – g) + cos (g – a) + cos (a – b) = - ,
a -b 2
then the value of cos [2004] then : [2009]
2
3 (a) A is false and B is true
-6
(a) (b) (b) both A and B are true
65 130
(c) both A and B are false
(d) A is true and B is false
6 3
(c) (d) -
65 130 4 5
7. Let cos (a + b ) = and sin (a - b) = ,
5 13
3. If u = a 2 cos 2 q + b 2 sin 2 q + a 2 sin 2 q + b 2 cos 2 q
then the difference between the maximum and p
where 0 £ a , b £ . Then tan 2a = [2010]
minimum values of u2 is given by [2004] 4
(a) ( a - b) 2 (b) 2 a 2 + b2 56 19
(a) (b)
33 12
(c) ( a + b) 2 (d) 2(a 2 + b2 )
4. The number of values of x in the interval [0, 3p] 20 25
(c) (d)
7 16
satisfying the equation 2sin 2 x + 5 sin x - 3 = 0 is 8. If A = sin2 x + cos4x, then for all real x :
[2006] [2011]
(a) 4 (b) 6
13
(a) £ A £1 (b) 1 £ A £ 2
(c) 1 (d) 2 16

1 3 13 3
5. If 0 < x < p and cos x + sin x = , then tan x (c) £ A£ (d) £ A £1
2 4 16 4
is [2006] 9. The possible values of q Î( 0, p) such that

(a) (1 - 7 ) (b) (4 - 7 ) sin ( q) + sin ( 4q) + sin ( 7q) = 0 are


4 3
[2011RS]
EBD_7764
www.crackjee.xyz
M-4 Mathematics

(a)
π 5π π 2π 3π 8π
, , , ,
4 12 2 3 4 9
, 13. Let f k ( x ) =
1
k
( )
sin k x + cosk x where x Î R

2p p p 2p 3p 35p and k ³ 1. Then f 4 ( x ) - f 6 ( x ) equals


(b) , , , , ,
9 4 2 3 4 36 [2014]
2p p p 2p 3p 8p 1 1
(c) , , , , , (a) (b)
9 4 2 3 4 9 4 12
2π π 4π π 3π 8π 1 1
(d) , , , , , (c) (d)
9 4 9 2 4 9 6 3
10. The equation esinx – e–sinx – 4 = 0 has : 1 14. If the angles of elevation of the top of a tower
[2012] from three collinear points A, B and C, on a line
(a) infinite number of real roots leading to the foot of the tower, are 30°, 45° and
(b) no real roots 60° respectively, then the ratio, AB : BC, is :
(c) exactly one real root [2015]
(d) exactly four real roots
(a) 1: 3 (b) 2 : 3
11. ABCD is a trapezium such that AB and CD are
parallel and BC ^ CD. If ÐADB = q, BC = p and (c) 3 :1 (d) 3: 2
CD = q, then AB is equal to : [2013] 15. A man is walking towards a vertical pillar in a
( p 2 + q 2 ) sin q
straight path, at a uniform speed. At a certain
(a) point A on the path, he observes that the angle
p cos q + q sin q of elevation of the top of the pillar is 30°. After
walking for 10 minutes from A in the same
p 2 + q 2 cos q
(b) direction, at a point B, he observes that the angle
p cos q + q sin q of elevation of the top of the pillar is 60°. Then
the time taken (in minutes) by him, from B to
p2 + q2 reach the pillar, is: [2016]
(c)
p 2 cos q + q 2 sin q (a) 20 (b) 5
(c) 6 (d) 10
( p 2 + q 2 ) sin q 16. If 0 £ x < 2p, then the number of real values of
(d)
( p cos q + q sin q) 2 x, which satisfy the equation
cos x + cos 2x + cos 3x + cos 4x = 0 is: [2016]
tan A cot A (a) 7 (b) 9
12. The expression +
1 - cot A 1 - tan A (c) 3 (d) 5
can be written as : [2013] 17. If 5(tan 2x – cos2x) = 2cos 2x + 9, then the value
(a) sinA cosA + 1 of cos 4x is : [2017]
(b) secA cosecA + 1 7 3
(c) tanA + cotA (1) - (2) -
9 5
(d) secA + cosecA
1 2
(3) (4)
3 9

Answer Key
1 2 3 4 5 6 7 8 9 10 11 12 13 14 15
(b ) (d) (a) (a) (c) (b) (a) (d) (d) (b) (a) (b) (b) (c) (b)
16 17
(a) (a)
www.crackjee.xyz
Trigonometric Functions M-5

1. (b) The given equation is tanx + secx = 2 cos x; 2 2


æ -21 ö æ -27 ö
Þ sin x + 1 = 2cos2 x + cos 2 b + 2 cos a cos b = ç ÷ +ç ÷
è 65 ø è 65 ø
Þ sin x + 1 = 2(1 – sin 2 x);
Þ 2sin2x + sin x – 1= 0; 1170
Þ 2 + 2 ( cos a cos b + sin a sin b ) =
Þ (2sin x – 1)(sin x + 1) = 0 4225

1 1170
Þ sin x = , –1.; Þ 2 éë1 + cos ( a - b ) ùû =
2 4425
Þ x = 30°, 150°, 270°.
æ a - b ö 1170
2. (d) p < a - b < 3p Þ 4 cos 2 ç ÷=
è 2 ø 4425
p a - b 3p a-b
Þ < < Þ cos <0 a -b 9
2 2 2 2 Þ cos 2 =
2 130
21
sin a + sin b = -
65 a-b -3
Þ cos =
2 130
a+b a-b 21
Þ 2 sin cos =- ....(1) Negative sign is taken because
2 2 65
p a - b 3p
27 < <
cos a + cos b = - 2 2 2
65

a+b a -b 27 ( a 4 + b 4 ) cos 2 q sin 2 q


Þ 2 cos cos =- ....(2) 3. (a) u 2 = a2 + b2 + 2
2 2 65 + a 2b 2 (cos 4 q + sin 4 q)
Square and add (1) and (2)
… (1)
a - b (21) 2 + (27) 2 1170 4 4 2 2
4 cos 2 = = Now (a + b ) cos q sin q
2 (65) 2 65 ´ 65
+ a 2 b2 (cos4 q + sin 4 q )
a -b 9 a -b 3
\ cos 2 = Þ cos =-
2 130 2 130 = (a4 + b4 ) cos 2 q sin 2 q

+ a 2 b2 (1 - 2cos 2 q sin 2 q )
21
Given that sin a + sin b = ....(1)
65 = (a4 + b4 - 2a 2b 2 ) cos 2 q sin 2 q + a 2 b2
-27
cos a + cos b = ....(2) sin 2 2q
65 = (a 2 - b 2 )2 . + a 2b2 …(2)
4
Squaring and adding equations (1) and
(2) we get Q 0 £ sin 2 2q £ 1
sin 2 a + sin 2 b + 2sin a sin b + cos 2 a
sin 2 2q (a 2 - b 2 ) 2
Þ 0 £ (a 2 - b 2 )2 £
4 4
EBD_7764
www.crackjee.xyz
M-6 Mathematics

sin 2 2q as tan x < 0 \ tan x =


-4 - 7
Þ a 2b2 £ (a 2 - b 2 )2 + a 2b2
4 3
6. (b) We have
1 cos (b – g ) + cos ( g – a) + cos (a – b)
£ (a - b ) . + a 2b2
2 2 2
....(3)
4
3
\ from (1) , (2) and (3) = -
2
Minimum value of
Þ 2 [cos (b – g ) + cos ( g – a)
u 2 = a 2 + b 2 + 2 a 2 b 2 = ( a + b) 2 + cos (a – b)] + 3 = 0
Þ 2 [cos (b – g ) + cos ( g – a) + cos (a – b)]
Maximum value of u2
+ sin2 a + cos2 a + sin2 b + cos2 b
+ sin2 g + cos2 a = 0
(a ) 1
2
= a 2 + b2 + 2 2
- b2 . + a2b 2
4 Þ [sin a + sin b + sin 2 g + 2 sin a sin b + 2
2 2

sin b sin g + 2 sin g sina ] + [cos2a +


2 cos2 b+ cos2 g + 2cosa cos b
= a 2 + b2 + (a 2 + b2 )2 = 2(a 2 + b2 )
2 + 2 cos b cos g + 2cos g cos a] = 0
\ Max value - Min value Þ [sina + sin b + sin g ]2 + (cos a + cos b +
= 2(a 2 + b2 ) - (a + b 2 ) = (a - b)2 cos g )2= 0
Þ sina + sin b + sin g = 0 and cos a + cos b
y
+ cos g =0
\ A and B both are true.
1
y=
2 4 3
4. (a) x 7. (a) cos(a + b) = Þ tan(a + b) =
O 3p 5 4
y = sin x
5 5
sin(a - b) = Þ tan(a - b) =
2
2sin x + 5 sin x - 3 = 0 13 12
Þ (sin x + 3)(2sin x - 1) = 0 tan 2a = tan [ (a + b) + (a - b)]
1 3 5
Þ sin x = and sin x ¹ -3 +
2 56
= 4 12 =
\ In [0, 3p] , x has 4 values. 3 5 33
1- .
4 12
1 1
5. (c) cos x + sin x = Þ 1 + sin 2 x =
2 4 8. (d) A = sin 2 x + cos 4 x
3 = sin 2 x + cos 2 x(1 - sin 2 x )
Þ sin 2 x = - , so x is obtuse and
4
1
2 tan x 3 = sin 2 x + cos2 x - (2sin x.cos x) 2
=- 4
2 4
1 + tan x 1
= 1 - sin 2 (2 x )
Þ 3 tan 2 x + 8 tan x + 3 = 0 4
Now 0 £ sin 2 (2 x) £ 1
-8 ± 64 - 36 -4 ± 7
\ tan x = =- 1 2 1
6 3 Þ 0 ³ - sin (2 x ) ³ -
4 4
www.crackjee.xyz
Trigonometric Functions M-7
1 2 1
Þ 1 ³ 1 - sin (2 x ) ³ 1 - p 2 + q 2 sin q
4 4 AB= =
sin q cos a + cos q sin a
3
Þ 1³ A ³
4 ( p 2 + q 2 ) sin q
9. (d) sin 4θ + 2sin 4θ cos 3 θ = 0 q sin q + p cos q

æ q p ö
sin 4q (1 + 2 cos 3q) = 0 ççQ cos a = and sina = ÷÷
è p2 + q2 p2 + q2 ø
1 12. (b) Given expression can be written as
sin 4q = 0 or cos 3q = -
2
sin A sin A cos A cos A
4q = n p ; n Î I ´ + ´
cos A sin A - cos A sin A cos A - sin A
2p
or 3q = 2np ± ,n ÎI
3 æ sin A ö
Q tan A = and
ç cos A ÷
p p 3p 2p 8p 4 p ç ÷
q= , , or q = , , cos A
4 2 4 9 9 9 ç cot A = ÷
è sin A ø
[Q q, Î (0, p)]
10. (b) Given equation is esinx – e–sinx – 4 = 0
1 ìï sin 3 A - cos3 A ïü
Put esin x = t in the given equation, we get = í ý
t2 – 4t – 1 = 0 sin A - cos A îï cos A sin A þï

4 ± 16 + 4 4 ± 20 sin 2 A + sin A cos A + cos 2 A


Þ t= = =
2 2 sin A cos A
4±2 5 = 1 + sec A cosec A
= =2± 5
2 1
13. (b) Let f k ( x) = (sin k x + cosk x )
Þ e
sin x
= 2 ± 5 Q t = e sin x ( ) Consider
k

Þ e sin x = 2 - 5 and e sin x = 2 + 5 1


f 4 ( x) - f 6 ( x) = (sin 4 x + cos 4 x)
4
Þ e sin x = 2 - 5 < 0
1
(
and sin x = ln 2 + 5 > 1 ) - (sin 6 x + cos6 x)
6
So, rejected.
Hence given equation has no solution.
1 1
\ The equation has no real roots. = [1 - 2sin 2 x cos-2 x][1 - 3sin 2 x cos 2 x]
4 6
11. (a) From Sine Rule
1 1 1
= - =
AB p +q 2 2 4 6 12
=
sinq sin( p - (q + a ))
EBD_7764
w w w . c r a c k j e e . x y
M-8 Mathematics
14. (c)
From (1) and (2)
P
3a = x + a Þ x = 2a
Here, the speed is uniform
15°
15° h
So, time taken to cover x = 2 (time taken to
cover a)
10
\ Time taken to cover a = minutes = 5
30° 45° 60° 2
A B C Q minutes
16. (a) cos x + cos 2x + cos 3x + cos 4x = 0
Q PB bisects ÐAPC, therefore Þ 2 cos 2x cos x + 2 cos 3x cos x = 0
AB : BC = PA : PC æ 5x xö
Þ 2cos x ç
ç2 cos cos ÷
÷= 0
h è 2 2ø
Also in DAPQ, sin30° = Þ PA = 2h
PA 5x x
cos x = 0, cos = 0 , cos = 0
2 2
h 2h
and in DCPQ, sin60° = Þ PC = p 3p p 3p 7p 9p
PC 3 x =p, , , , , ,
2 2 5 5 5 5
2h 17. (a) We have
\ AB : BC = 2h : = 3 :1 5 tan2 x – 5 cos2 x = 2 (2 cos2 x –1 ) + 9
3
Þ 5 tan2 x – 5 cos2 x = 4 cos2 x –2 + 9
h Þ 5 tan2 x = 9 cos2 x + 7
15. (b) tan 30° =
x+a Þ 5 (sec2 x – 1) = 9 cos2 x + 7
1 h
Þ = Þ 3h = x + a ...(1) Let cos2 x = t
3 x +a 5
h h Þ - 9t - 12 = 0
tan 60° = Þ 3 = t
a a
Þ 9t2 + 12t – 5 = 0
Þ h = 3a ...(2) Þ 9t2 + 15t – 3t – 5 = 0
Þ (3t – 1) (3t + 5) = 0
1 5
Þt= as t ¹ – .
h 3 3
æ1ö 1
30° 60° cos 2x = 2 cos2 x – 1 = 2 ç ÷ – 1 = –
è3ø 3
A x B a 2
æ 1ö 7
cos 4x = 2 cos2 2x – 1 = 2 ç - ÷ - 1 = -
è 3ø 9
www.crackjee.xyz

Principle of
Mathematical Induction 4
1. If an = 7 + 7 + 7 + ... ... having n radical 2. Let S ( K ) = 1 + 3 + 5... + (2 K - 1) = 3 + K 2 .
signs then by methods of mathematical induction Then which of the following is true [2004]
which is true [2002] (a) Principle of mathematical induction can
be used to prove the formula
(a) an > 7 " n ³ 1 (b) an < 7 " n ³ 1
(b) S ( K ) Þ S ( K + 1)

(c) an < 4 " n ³ 1 (d) an < 3 " n ³ 1 (c) S (K ) Þ


/ S ( K + 1)

(d) S (1) is correct

Answer Key
1 2
(b) (b)

1. (b) a1 = S (1) :1 = 3 + 1, which is not true


7 < 7. Let am < 7
Q S (1) is not true.
Then am + 1 = 7 + am Þ a2m + 1 = 7 + am \ P.M.I cannot be applied
Let S(K) is true, i.e.
< 7 + 7 < 14.
1 + 3 + 5.... + (2 K - 1) = 3 + K 2
Þ am + 1 < 14 < 7; So by the principle
Þ 1 + 3 + 5.... + (2 K - 1) + 2 K + 1
of mathematical induction an < 7 " n.
\ r = 0,8,16,24,........256 , total 33 values. = 3 + K 2 + 2 K + 1 = 3 + ( K + 1) 2
2. (b) S(K) = 1+3+5+...+(2K – 1) = 3 + K2 \ S ( K ) Þ S ( K + 1)
EBD_7764
www.crackjee.xyz
M-10 Mathematics

Complex Numbers
and Quadratic Equations 5
1. z and w are two non zero complex numbers such 7. If p and q are the roots of the equation
that | z | = | w| and Arg z + Arg w = p then z equals x2 + px + q = 0, then [2002]
[2002] (a) p = 1, q = –2 (b) p = 0, q = 1
(a) w (b) – w (c) p = –2, q = 0 (d) p = – 2, q = 1
(c) w (d) – w 8. If z and w are two non-zero complex numbers
2. If | z – 4 | < | z – 2 |, its solution is given by p
such that zw = 1 and Arg ( z ) - Arg (w ) = ,
[2002] 2
(a) Re(z) > 0 (b) Re(z) < 0 then zw is equal to [2003]
(a) – 1 (b) 1
(c) Re(z) > 3 (d) Re(z) > 2 (c) – i (d) i
3. The locus of the centre of a circle which touches 9. Let Z1 and Z 2 be two roots of the equation
the circle | z – z1 | = a and | z – z2 | = b externally
(z, z1 & z2 are complex numbers) will be Z 2 + aZ + b = 0 , Z being complex. Further ,
[2002] assume that the origin, Z1 and Z 2 form an
(a) an ellipse (b) a hyperbola equilateral triangle. Then [2003]
(c) a circle (d) none of these (a) a 2 = 4b (b) a2 = b
4. If a ¹ b but a2 = 5a – 3 and b2 = 5b – 3 then the
equation having a/b and b/a as its roots is (c) a 2 = 2b (d) a 2 = 3b
[2002] x
2 2 æ1+ i ö
(a) 3x – 19x + 3 = 0 (b) 3x + 19x – 3 10. If ç ÷ = 1 then [2003]
è1- i ø
=0
(c) 3x2 – 19x – 3 = 0 (d) x2 – 5x + 3 = 0. (a) x = 2n + 1 , where n is any positive integer
5. Difference between the corresponding roots of (b) x = 4n , where n is any positive integer
x2+ax+b=0 and x2+bx+a=0 is same and a ¹ b, (c) x = 2n , where n is any positive integer
then [2002] (d) x = 4n + 1 , where n is any positive integer..
(a) a + b + 4 = 0 (b) a + b – 4 = 0 11. The value of 'a' for which one root of the quadratic
(c) a – b – 4 = 0 (d) a – b + 4 = 0
equation (a 2 - 5a + 3) x 2 + (3a - 1) x + 2 = 0 is
6. Product of real roots of the equation
t2 x2 + | x | + 9 = 0 [2002] twice as large as the other is [2003]
(a) is always positive 1 2
(a) - (b)
(b) is always negative 3 3
(c) does not exist
2 1
(d) none of these (c) - (d)
3 3
www.crackjee.xyz
Complex Numbers & Quadratic Equations M-11

12. The number of real solutions of the equation 2


(c) – 1, 1 – 2 w , 1 – 2 w
2
x 2 - 3 x + 2 = 0 is [2003] (d) – 1, 1 + 2 w , 1 + 2 w
19. If z1 and z2 are two non- zero complex
(a) 3 (b) 2
(c) 4 (d) 1 numbers such that | z1 + z2 | = | z1 | + | z2 | , then
13. Let z and w be complex numbers such that arg z1 – arg z2 is equal to [2005]
z + i w = 0 and arg zw = p. Then arg z equals p
(a) (b) – p
[2004] 2
5p p
(a) (b) -p
4 2 (c) 0 (d)
2
3p p
(c) (d) z
4 4 20. If w = and | w | = 1, then z lies on
1
1 z- i
3
14. If z = x-i y an d z3 = p + iq, th en [2005]
(a) an ellipse (b) a circle
æ x yö 2 2
çè p + q ÷ø ( p + q ) is equal to [2004] (c) a straight line (d) a parabola
p æ Pö
(a) –2 (b) –1 21. In a triangle PQR, Ð R = . If tan ç ÷ and
(c) 2 (d) 1 2 è 2ø
æ Qö
15. If | z 2 - 1|=| z |2 +1, then z lies on [2004] – tan ç ÷ are the roots of ax 2 + bx + c = 0, a
è 2ø
(a) an ellipse ¹ 0 then
(b) the imaginary axis [2005]
(c) a circle (a) a = b + c (b) c = a + b
(d) the real axis
(c) b = c (d) b = a + c
16. If (1 - p) is a root of quadratic equation
22. If both the roots of the quadratic equation
x 2 + px + (1 - p) = 0 then its root are [2004] x 2 - 2 kx + k 2 + k – 5 = 0 are less than 5, then k
lies in the interval [2005]
(a) –1, 2 (b) –1, 1 (a) (5, 6] (b) (6, ¥ )

(c) 0, –1 (d) 0, 1 (c) (– ¥ , 4) (d) [4, 5]


10
17. If one root of the equation x 2 + px + 12 = 0 is æ 2k p 2k p ö
23. The value of å çè sin 11
+ i cos
11 ø
÷ is
k =1
4, while the equation x 2 + px + q = 0 has equal [2006]
roots , then the value of ‘q’ is [2004] (a) i (b) 1
(a) 4 (b) 12 (c) – 1 (d) – i

49 24. If z 2 + z + 1 = 0 , where z is complex number,,


(c) 3 (d) then the value of
4 2 2 2
æ 1ö æ 2 1ö æ 3 1ö
18. 2
If the cube roots of unity are 1, w , w then the çè z + ÷ø + çè z + 2 ÷ø + çè z + 3 ÷ø +
z z z
roots of the equation ( x –1)3 + 8 = 0, are [2005] 2
......... + æç z 6 + 1ö
÷ is [2006]
è z6 ø
2
(a) –1, –1 + 2 w , – 1 – 2 w (a) 18 (b) 54
(b) –1, – 1, – 1
(c) 6 (d) 12
EBD_7764
www.crackjee.xyz
M-12 Mathematics
25. If the roots of the quadratic equation 31. If the roots of the equation bx2 + cx + a = 0 be
imaginary, then for all real values of x, the
x 2 + px + q = 0 are tan30° and tan15°, expression 3b2x2 + 6bcx + 2c2 is : [2009]
respectively, then the value of 2 + q – p is (a) less than 4ab
[2006] (b) greater than – 4ab
(a) 2 (b) 3 (c) 1ess than – 4ab
(c) 0 (d) 1 (d) greater than 4ab
26. All the values of m for which both roots of the 4
32. If z - = 2 , then the maximum value of |z| is
2 2 z
equation x - 2mx + m - 1 = 0 are greater than
– 2 but less than 4, lie in the interval [2006] equal to : [2009]
(a) -2 < m < 0 (b) m>3 (a) 5 +1 (b) 2
(c) -1 < m < 3 (d) 1 < m < 4
(c) 2+ 2 (d) 3 +1
27. If | z + 4 | £ 3, then the maximum value of 33. The number of complex numbers z such that
| z + 1 | is [2007] |z – 1| = |z + 1| = |z – i| equals [2010]
(a) 6 (b) 0 (a) 1 (b) 2
(c) 4 (d) 10 (c) ¥ (d) 0
28. If the difference between the roots of the 34. If a and b are the roots of the equation
x2 – x + 1 = 0, then a2009 + b2009 = [2010]
equation x2 + ax + 1 = 0 is less than 5 , then the
(a) –1 (b) 1
set of possible values of a is [2007]
(c) 2 (d) –2
(a) (3, ¥) (b) (- ¥, - 3) 35. Let a, b be real and z be a complex number. If
(c) (– 3, 3) (d) (-3, ¥) . z2 + az + b = 0 has two distinct roots on the line
Re z =1, then it is necessary that : [2011]
1 (a) b Î (-1, 0) (b) b =1
29. The conjugate of a complex number is then
i –1
that complex number is [2008] (c) b Î (1, ¥) (d) b Î (0,1)
–1 1 36. If w( ¹ 1) is a cube root of unity, and
(a) (b)
i –1 i +1
(1 + w )7 = A + Bw. Then (A, B) equals [2011]
–1 1
(c) (d) (a) (1, 1) (b) (1, 0)
i +1 i –1
(c) (–1, 1) (d) (0, 1)
30. The quadratic equations x2 – 6x + a = 0 and
x2 – cx + 6 = 0 have one root in common. The 37. Sachin and Rahul attempted to solve a quadratic
other roots of the first and second equations equation. Sachin made a mistake in writing down
are integers in the ratio 4 : 3. Then the common the constant term and ended up in roots (4,3).
root is [2009] Rahul made a mistake in writing down coefficient
(a) 1 (b) 4 of x to get roots (3,2). The correct roots of
equation are : [2011 RS]
(c) 3 (d) 2
(a) 6, 1 (b) 4, 3
(c) – 6, – 1 (d) – 4, – 3
www.crackjee.xyz
Complex Numbers & Quadratic Equations M-13
38. Let for a ¹ a1 ¹ 0, (where [x] denotes the greatest integer £ x ) has
no integral solution, then all possible values of
f ( x ) = ax 2 + bx + c, g ( x) = a1 x 2 + b1 x + c1and p x = f axlie-ing the
x .interval: [2014]
+ bx + c, g ( x) = a1 x + b1 x + c1and p ( x ) = f ( x ) - g ( x ) . If p (x) = 0 only for x (a) ( -2, -1)
= -1 and p (– 2) = 2, then the value of p (2) is : (b) ( -¥, -2 ) È ( 2, ¥ )
[2011 RS]
(a) 3 (b) 9 (c) ( -1, 0 ) È ( 0,1)
(c) 6 (d) 18
2
(d) (1, 2 )
39. If z ¹ 1 and z is real, then the point 44. A complex number z is said to be unimodular if |z| = 1.
z -1 Suppose z1 and z2 are complex numbers such that
represented by the complex number z lies :
z1 - 2z 2
[2012]
(a) either on the real axis or on a circle passing 2 - z1 z2 is unimodular and z2 is not unimodular. Then
through the origin. the point z1 lies on a: [2015]
(b) on a circle with centre at the origin (a) circle of radius 2.
(c) either on the real axis or on a circle not (b) circle of radius 2.
passing through the origin. (c) straight line parallel to x-axis
(d) on the imaginary axis. (d) straight line parallel to y-axis.
40. If the equations x2 + 2x + 3 = 0 and ax2 + bx + c 45. Let a and b be the roots of equation x2 – 6x – 2 =
= 0, a,b,c Î R, have a common root, then a : b : c 0. If an = an – bn, for n ³ 1, then the value of
is [2013] a10 - 2a 8
(a) 1 : 2 : 3 (b) 3 : 2 : 1 2a 9 is equal to : [2015]
(c) 1 : 3 : 2 (d) 3 : 1 : 2
(a) 3 (b) – 3
41. If z is a complex number of unit modulus and (c) 6 (d) – 6
æ 1+ z ö 46. The sum of all real values of x satisfying the
argument q, then arg ç equals: [2013]
è 1 + z ÷ø 2
equation (x 2 - 5 x+ 5)x + 4x - 60
= 1 is :
p
(a) –q (b) –q [2016]
2 (a) 6 (b) 5
(c) q (d) p – q (c) 3 (d) – 4
42. If z is a complex number such that z ³ 2, then 2 + 3i sinq
47. A value ofqfor which is purely
1 1 - 2i sinq
the minimum value of z + : [2014] imaginary, is: [2016]
2
æ 3ö æ 1 ö
5 sin -1 ç ÷ sin -1 çç ÷÷
(a) is strictly greater than (a) ç 4 ÷ (b)
2 è ø è 3ø
3 5 p p
(b) is strictly greater than but less than (c) (d)
2 2 3 6
5 48. If, for a positive integer n, the quadratic
(c) is equal to equation, x(x + 1) + (x + 1) (x + 2) + ..... + (x +
2
(d) lie in the interval (1, 2) n - 1 ) (x + n) = 10n has two consecutive
43. If a Î R and the equation integral solutions, then n is equal to : [2017]
(a) 11 (b) 12
- 3 ( x - [ x ]) + 2 ( x - [ x ]) + a 2 = 0
2
(c) 9 (d) 10
EBD_7764
www.crackjee.xyz
M-14 Mathematics
Answer Key
1 2 3 4 5 6 7 8 9 10 11 12 13 14 15
(b) (c) (b) (a) (a) (a) (a) (a) (d) (b) (b) (c) (c) (a) (b)
16 17 18 19 20 21 22 23 24 25 26 27 28 29 30
(c) (d) (c) (c) (c) (b) (c) (d) (d) (b) (c) (a) (c) (c) (d)
31 32 33 34 35 36 37 38 39 40 41 42 43 44 45
(b) (a) (a) (b) (c) (a) (a) (d) (a) (a) (c) (d) (c) (a) (a)
46 47 48
(c) (b) (a)

1. (b) Let | z | = | w | = r \ a + b = –a, ab = b and g + d = –b, g d = a.


\ z = reiq, w = reif where q + f = p. Given |a – b| = |g – d| Þ (a – b)2 = (g – d)2
\ z = rei(p–f) = reip . e–if = –re–if = – w . Þ (a+ b)2 – 4ab = (g + d)2 – 4gd
Þ a – 4b = b2 – 4a
2
[Q w = re–if]
Þ (a2 – b2) + 4(a – b)= 0
2. (c) Given | z – 4 | < | z – 2 | Let z = x + iy Þ a + b + 4 = 0 (Q a ¹ b)
Þ | (x – 4) + iy) | < | (x – 2) + iy |
9
Þ (x – 4)2 + y2 < (x – 2)2 + y2 6. (a) Product of real roots = 2 > 0, " t Î R
Þ x2 – 8x + 16 < x2 – 4x + 4 Þ 12 < 4x t
Þ x > 3 Þ Re(z) > 3 \ Product of real roots is always positive.
3. (b) Let the circle be |z – z0| = r. Then according 7. (a) p + q = – p and pq = q Þ q (p – 1) = 0
to given conditions |z0 – z1| = r + a and Þ q = 0 or p = 1.
|z0 – z2|= r + b. Eliminating r, If q = 0, then p = 0. i.e.p = q
we get |z0 – z1| –|z0 – z2| = a – b. \ p = 1 and q = –2.
\ Locus of centre z0 is |z – z1| –|z – z2| 8. (a) | z w |=| z || w |=| z || w |=| z w |= 1
= a – b, which represents a hyperbola.
4. (a) We have a2 = 5a – 3 and b2 = 5b – 3; Arg( z w) = arg( z ) + arg(w)
Þ a & b are roots of equation, x2 = 5x – 3
p
or x2 – 5x + 3 = 0 = - arg( z ) + arg w = -
\ a + b = 5 and ab = 3 2
\ z w = -1
a b
Thus, the equation having & as its
b a
iq if
roots is Let z = r1e and w = r2e , \ z = r1e -iq
æ a b ö ab
x2 - x ç + ÷ + =0 Now | zw |= 1 Þ r1r2 ei( q+f) = 1 Þ r1 r2 =1
è b aø ab

æ a 2 + b2 ö p p
Þ x2 - x ç
Also arg (z) –arg (w) = Þ q-f =
÷ +1 = 0 2 2
è ab ø
or 3x2 – 19x +3 = 0 Now z w = r1e - iq .r2eif
5. (a) Let a, b and g, d be the roots of the ip
-i( q-f ) -
equations x2 + ax + b = 0 and = r1r2e =e 2 = –1
x2 + bx + a = 0 respectively.
www.crackjee.xyz
Complex Numbers & Quadratic Equations M-15

9. (d) Z 2 + aZ + b = 0 ; 13. (c) arg zw = p Þ arg z + arg w = p...(1)


Z1 + Z 2 = - a & Z1Z 2 = b z + iw = 0 Þ z = -iw
0, Z1, Z 2 form an equilateral D p
\ z = iw Þ arg z = + arg w
2
\ 02 + Z12 + Z 2 2 = 0.Z1 + Z1.Z 2 + Z 2 .0
p
(for an equilateral triangle, Þ arg z = + p - arg z (from (1))
2
Z12 + Z 22 + Z32 = Z1Z 2 + Z 2 Z 3 + Z3 Z1 ) 3p
\ arg z =
2 2
Þ Z1 + Z 2 = Z1Z 2 4
1
Þ ( Z1 + Z 2 ) 2 = 3Z1Z 2 14. (a) z3 = p + iq
\ a = 3b 2
Þ z = p3 + (iq )3 + 3 p (iq )( p + iq )
x
æ 1+ i ö
x é (1 + i )2 ù Þ x - iy = p3 - 3 pq 2 + i (3 p 2 q - q 3 )
10. (b) çè ÷ø = 1 Þ ê ú =1
1- i 2
êë 1 - i úû x
\ x = p3 - 3 pq 2 Þ = p 2 - 3q2
x p
æ 1 + i 2 + 2i ö x +
ç ÷ = 1 Þ (i ) = 1; \ x = 4n ; n Î I y = q3 - 3 p 2 q Þ
y
= q2 - 3 p2
è 1 + 1 ø q
11. (b) Let the roots of given equation be a and
x y
2a then \ + = -2 p 2 - 2q2
p q
1 - 3a
a + 2a = 3a = æ x yö
2
a - 5a + 3 \ ç + ÷ ( p 2 + q 2 ) = -2
è p qø
2
and a.2a = 2a 2 = 2
a - 5a + 3 15. (b) | z 2 - 1|=| z |2 +1 Þ| z 2 - 1|2 = ( zz + 1) 2

1 - 3a Þ ( z 2 - 1)( z 2 - 1) = ( zz + 1) 2
Þ a=
(
3 a 2 - 5a + 3 ) Þ z 2 z 2 - z 2 - z 2 + 1 = z 2 z 2 + 2zz + 1

é 1 (1 - 3a)2 ù 2 2 ( z + z )2 = 0 Þ z = - z
\ 2 ê ú= 2 Þ z 2 + 2 zz + z Þ =0
9 2 2
ëê (a - 5a + 3) ûú a - 5a + 3 Þ z is purely imaginary
1
(1 - 3a) 2 Let z = r (cosq + i sinq)
= 9 or 9a 2 - 6 a + 1
2
(a - 5a + 3) Then | z 2 - 1|=| r 2 (cos 2q + i sin 2q) - 1 |
= 9 a 2 - 45a + 27 and
= r 4 - 2r 2 cos 2q + 1
2
or 39a = 26 or a = | z 2 - 1|2 = (| z |2 +1) 2
3
Þ r 4 - 2r 2 cos 2q + 1 = r 4 + 2r 2 + 1
2 2
12. (c) x - 3 x + 2 = 0 Þ| x | -3 | x | +2 = 0
p
( x - 2)( x - 1) = 0 Þ 2 cos 2 q = 0 Þ cos q = ±
2
x = 1, 2 or x = ±1, ±2 \ z lies on imaginary axis.
\ No.of solution = 4
EBD_7764
www.crackjee.xyz
M-16 Mathematics
2 æ 1ö
ç 0, ÷ is same hence z lies on bisector
We know that, if z1 + z2 = z1 + z2 è 3ø
then origin, z1 and z2 are collinear of the line joining points (0, 0) and (0, 1/
Þ arg ( z1 ) = arg ( z2 ) 3).
Hence z lies on a straight line.
As per question z + ( -1) = z + -1
2 2
æ Pö æ Qö
21. (b) tan ç ÷ , tan ç ÷ are the roots of
è 2ø è 2ø
( )
Þ arg z 2 = arg ( -1)
ax 2 + bx + c = 0
p æ Pö æ Qö b
Þ 2arg ( z ) = p Þ arg ( z ) = tan ç ÷ + tan ç ÷ = -
2 è 2ø è 2ø a
Þ z lies on imaginary axis.
16. (c) Let the second root be a. æ Pö æ Qö c
tan ç ÷ × tan ç ÷ =
Then a + (1 - p ) = - p Þ a = -1 è 2ø è 2ø a
Also a.(1 - p) =1 - p æ Pö æ Qö
tan ç ÷ + tan ç ÷
Þ (a - 1)(1 - p) = 0 Þ p =1[Q a = -1] è 2ø è 2ø æ P Qö
= tan ç + ÷ = 1
æ Pö æ Qö è 2 2ø
\ Roots are a = -1 and p - 1 = 0 1 - tan ç ÷ tan ç ÷
è 2ø è 2ø
17. (d) 4 is a root of x 2 + px + 12 = 0
b
Þ 16 + 4 p + 12 = 0 Þ p = -7 -
a =1 b a c
Þ Þ - = -
Now, the equation x 2 + px + q = 0 c a a a
1-
has equal roots. a
Þ – b = a – c or c = a + b.
p 2 49
\ p 2 - 4q = 0 Þ q = = 22. (c) both roots are less than 5
4 4
Y axis
1/ 3
18. (c) ( x - 1)3 + 8 = 0 Þ ( x - 1) = (-2) (1)
Þ x – 1 = – 2 or -2w or - 2w 2 0 X axis
or x = – 1 or 1 – 2 w or 1 – 2 w 2 . x=5
19. (c) | z1 + z2 | = | z1 | + | z2 | Þ z1 and
z2 are collinear and are to the same side then (i) Discriminant ³ 0
(ii) p(5) > 0
of origin; hence arg z1 – arg z2 = 0.
Sum of roots
z (iii) <5
20. (c) As given w = 2
1 Hence (i) 4k2– 4(k2 + k – 5) ³ 0
z- i
3 4k2 – 4k2 – 4k + 20 ³ 0
|z| 4k £ 20 Þ k £ 5
Þ |w|= =1 (ii) Þ f(5) > 0 ; 25 – 10 k + k2 + k – 5 > 0
1
|z- i| or k2 – 9k + 20 > 0
3 or k (k – 4) –5(k – 4) > 0
1 or (k – 5) (k – 4) > 0
Þ z = z- i
3 Þ k Î ( – ¥ , 4 ) U ( – ¥ , 5)
Þ distance of z from origin and point Sum of roots b 2k
(iii) Þ =– = <5
2 2a 2
www.crackjee.xyz
Complex Numbers & Quadratic Equations M-17
The intersection of (i), (ii) & (iii) 2 2
gives 26. (c) Equation x - 2mx + m - 1 = 0
k Î ( – ¥ , 4 ). ( x - m) 2 - 1 = 0 or
10
2k p 2k p ö ( x - m + 1)( x - m - 1) = 0
23. (d) å æçè sin 11
+ i cos
11 ø
÷ x = m - 1, m + 1
k =1
m – 1 > –2 and m + 1 < 4
10
æ
= i å ç cos
2k p 2k p ö Þ m > - 1 and m < 3 or,, -1 < m < 3
- i sin ÷
è 11 11 ø 27. (a) z lies on or inside the circle with centre
k =1
(–4, 0) and radius 3 units.
2k p ì 10 - 2k p i ü Y
10 - i ï ï
= iå e 11 = i í å e 11 - 1ý Im.
k =1 ïî k =0 ïþ
é 2p 4p ù
- i - i
ê
= i 1+ e 11 + e 11 + ....11 terms ú - i (-7, 0) (-4, 0) (-1, 0) Real
ê ú X
ë û X'
é
= iê e ( )
2 p 11 ù
ê1 - - 11 ú é 1 - e - 2p i ù
ú-i = iê ú
2p - i
ê 2p
- i ú
ê - iú Y'
ëê 1 - e 11 ûú ë1 - e 11 û From the Argand diagram maximum value
=i×0–i [Q e -2 pi = 1] of | z + 1| is 6
=–i |z+1|=|z+4–3|
24. (d) z 2 + z + 1 = 0 Þ z = w or w 2 £ | z + 4 | + | –3 | £ | 3 | + | – 3|
1 2 Þ | z + 1 | £ 6 Þ | z + 1|max = 6
So, z + = w + w = -1
z 28. (c) Let a and b are roots of the equation
1 x2 + ax + 1 = 0
z 2 + 2 = w 2 + w = -1,
z a + b = – a and ab = 1
1
z 3 + 3 = w3 + w3 = 2 given | a - b | < 5
z
1 1 Þ (a + b)2 - 4ab < 5
z + 4 = -1, z 5 + 5 = -1
4
z z (Q (a - b) 2 = (a + b ) 2 - 4ab )
1
and z 6 + 6 = 2 Þ a 2 - 4 < 5 Þ a2 – 4 < 5
z
\ The given sum = 1+1 + 4 + 1 + 1 + 4 Þ a2 – 9 < 0 Þ a2 < 9 Þ – 3 < a < 3
= 12 Þ a Î (–3, 3)

25. (b) x 2 + px + q = 0 29. (c) æç 1 ö÷ = 1 = –1


Sum of roots = tan30° + tan15° = – p è i –1ø –i –1 i + 1
Product of roots = tan30° . tan15° = q 30. (d) Let the roots of equation x2 – 6x + a = 0
tan 30° + tan15° -p be a and 4 b and that of the equation
tan 45° = = =1 x2 –cx + 6 = 0 be a and 3 b .Then
1 - tan 30°. tan15° 1 - q
a + 4b = 6 ; 4a b = a
Þ – p = 1- q Þ q - p = 1 and a + 3b = c ; 3a b = 6
\ 2+ q - p = 3 Þ a=8
\ The equation becomes x2 – 6x + 8 = 0
EBD_7764
w w w . c r
M-18 Mathematics
Þ (x –2) (x – 4) = 0
1± 1- 4
Þ roots are 2 and 4 34. (b) x2 - x + 1 = 0 Þ x =
Þ a = 2, b = 1 \ Common root is 2. 2
31. (b) Given that roots of the equation 1± 3 i
bx2 + cx + a = 0 are imaginary x=
2
\ c2 – 4ab < 0 ....(i)
2 2
Let y = 3b x + 6 bc x + 2c 2 1 3
a= +i = -w2
Þ 3b2x2 + 6 bc x + 2c2 – y = 0 2 2
As x is real, D ³ 0
1 i 3
Þ 36 b2c2 – 12 b2 (2c2 – y ) ³ 0 b= - = -w
2 2
Þ 12 b2 (3 c2 – 2 c2+ y ) ³ 0
p p p p
Þ c2 + y ³ 0 Þ y ³ – c2 a = cos + i sin , b = cos - i sin
3 3 3 3
But from eqn. (i), c < 4ab or – c2 > – 4ab
2

\ we get y ³ – c2 > – 4ab a2009 + b2009 = (-w2 )2009 + ( -w)2009


Þ y > – 4 ab = -w2 - w = 1
4 35. (c) Since both the roots lie in the line Re z = 1
32. (a) Given that z - =2 i.e., x = 1, hence real part of both the roots
z are 1.
N o w Let both roots be 1 + ia and 1 – ia
4 4 4 4 Product of the roots, 1 + a2 = b
z = z- + £ z- +
z -z z z Q a2 + 1 ³ 1
4 \b ³ 1 Þ Q b Î (1, ¥)
Þ z £ 2+
z 36. (a) (1 + w)7 = A + Bw
2
(–w2)7 = A + Bw
Þ z –2 z -4£ 0 – w2 = A + Bw
æ 1 + w = A + Bw
2 + 20 ö æ 2 - 20 ö
Þ çç z – ÷÷ çç z – ÷÷ £ 0 Þ A = 1, B = 1.
è 2 ø è 2 ø 37. (a) Let the correct equation be
Þ ( z – (1 + 5) ) ( z – (1 - 5) ) £ 0 ax 2 + bx + c = 0
Now Sachin’s equation
Þ ( - 5 + 1) £ z £ ( 5 + 1) ax 2 + bx + c ' = 0
Roots found by Sachin’s are 4 and 3
Þ z max = 5 + 1 Hence T is an
equivalence relation. Rahul’s equation, ax 2 + b ' x + c = 0
Roots found by Rahul’s are 3 and 2
33. (a) Let z = x + iy
b
- =7 ....(i)
z - 1 = z + 1 ( x - 1) 2 + y 2 = ( x + 1) 2 + y 2 a
c
Þ Re z = 0 Þ x=0 =6 ...(ii)
a
z - 1 = z - i ( x - 1) 2 + y 2 = x 2 + ( y - 1) 2 From (i) and (ii), roots of the correct
equation x 2 - 7 x + 6 = 0 are 6 and 1.
Þ x= y
38. (d) p (x) = 0
z + 1 = z - i ( x + 1) 2 + y 2 = x 2 + ( y - 1) 2 Þ f ( x ) = g ( x)
Only (0, 0) will satisfy all conditions.
Þ ax2 + bx + c = a1x 2 + b1 x + c1
Þ Number of complex number z = 1
www.crackjee.xyz
Complex Numbers & Quadratic Equations M-19

Þ (a - a1 ) x + (b - b1 ) x + (c - c1 ) = 0.
2
æ 1+ z ö æ 1+ z ö
\ arg ç = arg = arg (z) = q.
It has only one solution, x = – 1 è 1 + z ÷ø ç 1÷
çè 1 + ÷ø
Þ b - b1 = a - a1 + c - c1 ...(1) z
vertex = ( -1, 0) 42. (d) We know minimum value of |Z1 + Z2| is | |Z1|
b - b1 1
Þ = -1 – |Z2|| Thus minimum value of Z + is
2 (a - a1 ) 2

Þ Þ b - b1 = 2 ( a - a1 ) ....... 2 ...(2) |Z |-
1
£ Z+
1
£| Z | +
1
Now p (– 2) = 2 2 2 2
Þ f (– 2) – g (– 2) = 2 Since, | Z |³ 2 therefore
Þ 4a – 2b + c – 4a1 + 2b1 – c1 = 2
Þ 4 (a – a1) – 2 (b – b1) + (c – c1) = 2 ...(3) 1 1 1
From equations, (1), (2) and (3) 2- < Z + < 2+
2 2 2
1
a - a1 = c - c1 = ( b - b1 ) = 2 3 1 5
2 Þ < Z+ <
2 2 2
Now, p ( 2) = f ( 2) - g (2) 43. (c) Consider –3(x – [x])2 + 2 [x – [x]) + a2 = 0
= 4 ( a - a1 ) + 2 ( b - b1 ) + ( c - c1 ) Þ 3{x}2 – 2{x} –a2 = 0 (Q x – [x] =
{x})
= 8 + 8 + 2 = 18
æ 2 ö
39. (a) Since we know z = z if z is real. Þ 3 ç {x}2 - {x} ÷ = a2 , a ¹ 0
è 3 ø
z2 z2
Therefore, = æ 2ö
z-1 z -1 Þ a 2 = 3{x} ç {x} - ÷
2 2
è 3ø
Þ zzz - z = z. z . z - z
2 2
Þ z .z - z 2 = z .z - z 2
1/3
Þ z 2 ( z - z ) - ( z - z )( z + z ) = 0
2
(
Þ (z - z ) z -(z + z ) = 0 ) –1/3 2/3

Either z - z = 0 or z 2 - ( z + z ) = 0
Either z = z Þ real axis
-2
or z 2 = z + z Þ zz - z - z = 0 Now, {x} Î (0,1) and £ a2 < 1
3
represents a circle passing through origin. (by graph)
40. (a) Given equations are Since , x is not an integer
x2 + 2x + 3 = 0 …(i)
\ a Î (-1,1) - {0}
ax2 + bx + c = 0 …(ii)
Roots of equation (i) are imaginary roots. Þ a Î (-1, 0) È (0,1)
According to the question (ii) will also have
both roots same as (i). Thus z1 - 2z 2
44. (a) =1
a b c 2 - z1 z2
= = = l (say)
1 2 3
Þ a = l, b = 2l, c = 3l 2 2
Þ z1 - 2z 2 = 2 - z1z2
Hence, required ratio is 1 : 2 : 3
41. (c) Given | z | = 1, arg z = q
Þ (z1 - 2z 2 )(z1 - 2z 2 ) = (2 - z1z2 )(2 - z1z2 )
1
As we know, z = Þ (z1 - 2z 2 )(z1 - 2z2 ) = (2 - z1z2 )(2 - z1z 2 )
z
EBD_7764
www.crackjee.xyz
M-20 Mathematics
Þ (z1z1) - 2z1z2 - 2z1z 2 + 4z 2 z2 Case II
x2 – 5x + 5 = –1 and x2 + 4x – 60 has to be an
even number
= 4 - 2z1z 2 - 2z1z2 + z1z1z 2 z2
Þ x = 2, 3
2 2 2 2 where 3 is rejected because for x = 3,
Þ z1 + 4 z 2 = 4 + z1 z2 x2 + 4x – 60 is odd.
Case III
2 2 2 2
Þ z1 + 4 z 2 – 4 – z1 z2 =0 x2 – 5x + 5 can be any real number and
x2 + 4x – 60 = 0
(z 1
2
)(
- 4 1 - z2
2
)=0 Þ x = –10, 6
Þ Sum of all values of x = –10 + 6 + 2 + 1 + 4 = 3
z2 ¹ 1 47. (b) Rationalizing the given expression
Q
(2 + 3isin q)(1 + 2isin q)
2
\ z1 =4 1 + 4sin 2 q
For the given expression to be purely imaginary,
Þ z1 = 2 real part of the above expression should be equal
to zero.
Þ Point z1 lies on circle of radius 2.
2 - 6 sin 2 q 1
Þ =0 Þ sin 2 q =
6 ± 36 + 8 1 + 4 sin q 2 3
45. (a) a, b = = 3 ± 11
2 1
Þ sin q = ±
a = 3 + 11 , b = 3 - 11 3
48. (a) We have
( ) ( )
n n
\ an = 3 + 11 – 3 – 11 n

a10 – 2a 8 å (x + r - 1)(x + r) = 10n


r =1
2a 9 n
å (x 2 + xr + (r – 1)x + r 2 - r = 10n
=
( 3 + 11 ) (
10
– 3 – 11 )
10
(
– 2 3 + 11) ( 8
+ 2 3 - 11 ) 8
r= 1

(
é
) - (3 - 11) ùúû n
9 9
2 ê 3 + 11
Þ å (x + (2r - 1)x + r(r - 1) = 10n
2
ë
r= 1
( 3 + 11)8 éêë( 3 + 11)2 – 2ùúû + ( 3 - 11)8 éêë2 - (3 - 11)2 ùúû Þ nx2 + {1 + 3 + 5 + .... + (2n – 1) }x + {1.2 + 2.3
+.... + (n – 1) n} = 10 n
=
2 ê( 3 + 11 ) - (3 - 11) ú
é 9 9ù
(n - 1) n(n + 1)
ë û Þ nx2 + n2 x + = 10n
3
( 3 + 11)8 ( 9 + 11 + 6 11 – 2) + (3 - 11)8 (2 - 9 - 11 + 6 11) Þ x2 + nx +
n 2 - 31
=0
=
2 ê( 3 + 11 ) – ( 3 - 11) ú 3
é 9 9ù
ë û Let a and a + 1 be its two solutions
(Q it has two consequtive integral solutions)
( )9 – 6 (3 - 11)9 6
6 3 + 11 Þ a + (a + 1) = – n
= = =3 -n - 1
2 ê( 3 + 11) - ( 3 - 11 ) ú
é 9 9ù 2 Þa= ...(1)
ë û 2
2 n 2 - 31
46. (c) (x 2 - 5 x + 5) x + 4x - 60
=1 Also a (a+1) = ...(2)
3
Case I Putting value of (1) in (2), we get
x2 – 5x + 5 = 1 and x2 + 4x – 60 can be any real 2
æ n + 1ö æ 1 - n ö n - 31
number -ç ÷ ç ÷ =
Þ x = 1, 4 è 2 øè 2 ø 3
Þ n2 = 121 Þ n = 11
www.crackjee.xyz

Linear Inequality 6
1. If a, b, c are distinct +ve real numbers and a2 + 3. Statement-1 : For every natural number n³ 2,
b2 + c2 = 1 then ab + bc + ca is [2002]
1 1 1
(a) less than 1 (b) equal to 1 + + ......... + > n.
(c) greater than 1 (d) any real no. 1 2 n
Statement-2 :For every natural number n ³ 2,
3x 2 + 9 x + 17
2. If x is real, the maximum value of n(n + 1) < n + 1. [2008]
3x 2 + 9 x + 7
(a) Statement -1 is false, Statement-2 is true
is [2006]

1 (b) Statement -1 is true, Statement-2 is true;


(a) (b) 41 Statement -2 is a correct explanation for
4
Statement-1
17 (c) Statement -1 is true, Statement-2 is true;
(c) 1 (d) Statement -2 is not a correct explanation
7
for Statement-1
(d) Statement -1 is true, Statement-2 is false
Answer Key
1 2 3
(a) (b) (b)

1. (a) Q (a – b)2 + (b – c)2 + (c – a)2 > 0 \ Max value of y is 41


Þ 2(a2 + b2 + c2 – ab – bc – ca) >0
Þ 2 > 2(ab + bc + ca) Þ ab + bc + ca < 1
3x 2 + 9 x + 17
Given f ( x) =
3 x 2 + 9 x + 17 3 x2 + 9 x + 7
2. (b) y=
3 x2 + 9 x + 7
10
2 Þ f ( x) = 1 + 2
3x ( y - 1) + 9 x ( y - 1) + 7 y - 17 = 0 3x + 9 x + 7
D ³ 0 Q x is real Clearly f(x) is maximum when g(x) = 3x2 +
9x + 7 is min.
81( y - 1)2 - 4 ´ 3( y - 1)(7 y - 17) ³ 0
æ 9ö 27
Þ ( y - 1)( y - 41) £ 0 Þ 1 £ y £ 41 Here g(x) = 3 ç x 2 + 3x + ÷ + 7 -
è 4 ø 4
EBD_7764
www.crackjee.xyz
M-22 Mathematics
2 1 1
1
= 3 çæ x + ÷ö +
3 Now 2< nÞ >
è 2ø 4 2 n

-3 1 1
which is minimum when x = 3< nÞ > ;
2 3 n

10 1 1
\ f max = 1 + = n£ nÞ ³
9 3 n n
3´ -9´ +7
4 2
1 1
Also > \ Adding all, we get
10 ´ 4 1 n
1+ = 41
27 – 54 + 28
1 1 1 1 n
3. (b) Statement 2 is n(n + 1) < n + 1, n ³ 2 + + + ............ + > = n
1 2 3 n n
Þ n < n + 1, n ³ 2 which is true Hence both the statements are correct
and statement 2 is a correct explanation
Þ 2 < 3 < 4 < 5 < ------- n
of statement-1.
www.crackjee.xyz

Permutations and
Combinations 7
1. Total number of four digit odd numbers that can 8. How many ways are there to arrange the letters
be formed using 0, 1, 2, 3, 5, 7 (using repetition in the word GARDEN with vowels in
allowed) are [2002] alphabetical order [2004]
(a) 216 (b) 375 (a) 480 (b) 240
(c) 400 (d) 720 (c) 360 (d) 120
2. Number greater than 1000 but less than 4000 is
9. The number of ways of distributing 8 identical
formed using the digits 0, 1, 2, 3, 4 (repetition
balls in 3 distinct boxes so that none of the
allowed). Their number is [2002] boxes is empty is [2004]
(a) 125 (b) 105
(c) 375 (d) 625 (a) 8 (b) 21
C3
3. Five digit number divisible by 3 is formed using
0, 1, 2, 3, 4, 6 and 7 without repetition. Total (c) 38 (d) 5
number of such numbers are [2002] 10. If the letters of the word SACHIN are arranged
(a) 312 (b) 3125 in all possible ways and these words are written
(c) 120 (d) 216 out as in dictionary, then the word SACHIN
4. The sum of integers from 1 to 100 that are appears at serial number [2005]
divisible by 2 or 5 is [2002] (a) 601 (b) 600
(a) 3000 (b) 3050 (c) 603 (d) 602
(c) 3600 (d) 3250
11. At an election, a voter may vote for any number
5. n
If Cr denotes the number of combination of n of candidates, not greater than the number to be
things taken r at a time, then the expression elected. There are 10 candidates and 4 are of be
selected, if a voter votes for at least one
n
Cr +1 + nC r -1 + 2´n Cr equals [2003] candidate, then the number of ways in which he
can vote is [2006]
(a) n +1 (b) n+ 2
Cr +1 Cr (a) 5040 (b) 6210
(c) n+2
Cr +1 (d) n +1
Cr . (c) 385 (d) 1110
6. A student is to answer 10 out of 13 questions 12. The set S = {1, 2, 3, ......., 12} is to be partitioned
in an examination such that he must choose at into three sets A, B, C of equal size.
least 4 from the first five questions. The number Thus A È B È C = S,
of choices available to him is [2003] A Ç B = B Ç C = A Ç C = f. The number of
(a) 346 (b) 140 ways to partition S is [2007]
(c) 196 (d) 280
7. The number of ways in which 6 men and 5 women 12! 12!
(a) (b)
can dine at a round table if no two women are (4!) 3
(4!) 4
to sit together is given by [2003]
(a) 6! × 5! (b) 6 × 5 12! 12!
(c) (d)
(c) 30 (d) 5 × 4 3!(4!) 3
3!(4!) 4
EBD_7764
www.crackjee.xyz
M-24 Mathematics
13. How many different words can be formed by (c) Statement-1 is true, Statement-2 is false
jumbling the letters in the word MISSISSIPPI in (d) Statement-1 is false, Statement-2 is true
which no two S are adjacent? [2008] 18. There are 10 points in a plane, out of these 6 are
(a) 8. 6C4. 7C4 (b) 6.7. 8C4 collinear. If N is the number of triangles formed
(c) 6. 8. 7C4. (d) 7. 6C4. 8C4 by joining these points. Then : [2011RS]
14. From 6 different novels and 3 different (a) N £ 100
dictionaries,4 novels and 1 dictionary are to be
selected and arranged in a row on a shelf so that (b) 100 < N £ 140
the dictionary is always in the middle. Then the (c) 140 < N £ 190
number of such arrangement is: [2009] (d) N > 190
(a) at least 500 but less than 750
(b) at least 750 but less than 1000
(c) at least 1000
{
19. If X = 4n - 3n - 1 : n Î N and}
(d) less than 500 Y = {9 ( n - 1) : n Î N } , where N is the set of
15. There are two urns. Urn A has 3 distinct red natural numbers, then X È Y is equal to:
balls and urn B has 9 distinct blue balls. From [2014]
each urn two balls are taken out at random and (a) X (b) Y
then transferred to the other. The number of ways (c) N (d) Y – X
in which this can be done is [2010] (c) 8 (d) 64
(a) 36 (b) 66 20. Let A and B be two sets containing four and two
(c) 108 (d) 3 elements respectively. Then the number of
16. Statement-1: The number of ways of distributing subsets of the set A × B, each having at least
10 identical balls in 4 distinct boxes such that no three elements is : [2013, 2015]
box is empty is 9C3 . (a) 275 (b) 510
Statement-2: The number of ways of choosing (c) 219 (d) 256
any 3 places from 9 different places is 9C3. 21. The number of integers greater than 6,000 that
[2011] can be formed, using the digits 3, 5, 6, 7 and 8,
(a) Statement-1 is true, Statement-2 is true; without repetition, is : [2015]
Statement-2 is not a correct explanation for (a) 120 (b) 72
Statement-1. (c) 216 (d) 192
(b) Statement-1 is true, Statement-2 is false.
22. If all the words (with or without meaning) having
(c) Statement-1 is false, Statement-2 is true.
five letters, formed using the letters of the word
(d) Statement-1 is true, Statement-2 is true; SMALL and arranged as in a dictionary; then the
Statement-2 is a correct explanation for position of the word SMALL is : [2016]
Statement-1. (a) 52nd (b) 58th
17. Statement - 1 : For each natural number n, (n + 1)7–1 (c) 46th (d) 59th
is divisible by 7. 23. A man X has 7 friends, 4 of them are ladies
Statement - 2 : For each natural number and 3 are men. His wife Y also has 7 friends, 3
of them are ladies and 4 are men. Assume X
n, n 7 - n is divisible by 7. [2011 RS]
and Y have no common friends. Then the
(a) Statement-1 is true, Statement-2 is true; total number of ways in which X and Y
Statement-2 is a correct explanation for together can throw a party inviting 3 ladies and
Statement-1. 3 men, so that 3 friends of each of X and Y are in
(b) Statement-1 is true, Statement-2 is true; this party, is : [2017]
Statement-2 is NOT a correct explanation (a) 484 (b) 485
for Statement-1 (c) 468 (d) 469
www.crackjee.xyz
Permutations and Combinations M-25

Answer Key
1 2 3 4 5 6 7 8 9 10 11 12 13
(d) (c) (d) (b) (c) (c) (a) (c) (b) (a) (c) (a) (d)
14 15 16 17 18 19 20 21 22 23
(c) (c) (a) (a) (a) (b) (c) (d) (b) (b)

1. (d) Required number of numbers (i) Selecting 4 out of first five question and
= 5 ´ 6 ´ 6 ´ 4 = 36 ´ 20 = 720. 6 out of remaining question
2. (c) Required number of numbers
= 5C4 ´8 C6 = 140 choices.
= 3 ´ 5 ´ 5 ´ 5 = 375
3. (d) We know that a number is divisible by 3 (ii) Selecting 5 out of first five question and
only when the sum of the digits is divisible 5 out of remaining 8 questions
by 3. The given digits are 0, 1, 2, 3, 4, 5. = 5C5 ´8 C5 = 56 choices.
Here the possible number of combinations
Therefore, total number of choices
of 5 digitsout of 6 are5C4 = 5, which are as
=140 + 56 = 196.
follows–
7. (a) No. of ways in which 6 men can be arranged
1 + 2 + 3 + 4 + 5 = 15 = 3 × 5
at a round table = (6 - 1)! = 5!
0 + 2 + 3 + 4 + 5 = 14 (not divisible by 3)
0 + 1 + 3 + 4 + 5 = 13 (not divisible by 3) Now women can be arranged in 6 P5
0 + 1 + 2 + 4 + 5 = 12 = 3 × 4 = 6! Ways.
0 + 1 + 2 + 3 + 5 = 11 (not divisible by 3) Total Number of ways = 6! × 5!
0 + 1 + 2 + 3 + 4 = 10 ( not divisible by 3)
8. (c) Total number of arrangements of letters in
Thus the number should contain the digits
the word GARDEN = 6 ! = 720 there are
1, 2, 3, 4, 5 or the digits 0, 1, 2, 4, 5. two vowels A and E, in half of the
Taking 1, 2, 3, 4, 5, the 5 digit numbers are
arrangements A preceeds E and other half
= 5! = 120 A follows E.
Taking 0, 1, 2, 4, 5, the 5 digit numbers are So, vowels in alphabetical order in
= 5! – 4! = 96
\ Total number of numbers = 120 + 96 1
´ 720 = 360
= 216 2
4. (b) Required sum 9. (b) We know that the number of ways of
= (2 + 4 + 6 + ... + 100) distributing n identical items among r
+ (5 + 10 + 15 + ... + 100) persons, when each one of them receives
– (10 + 20 + ... + 100) at least one item is n -1
Cr -1
= 2550 + 1050 – 530 = 3050.
\ The required number of ways
5. (c) n
Cr +1 + n Cr -1 + 2 n Cr
8 -1 7! 7´ 6
n n n n = C3-1 = 7C2 == = 21
= Cr -1 + Cr + Cr + Cr +1 2!5! 2 ´1
10. (a) Alphabetical order is
= n +1Cr + n +1Cr +1 = n + 2 Cr +1 A, C, H, I, N, S
6. (c) As for given question two cases are No. of words starting with A – 5!
possible. No. of words starting with C – 5!
EBD_7764
www.crackjee.xyz
M-26 Mathematics
No. of words starting with H – 5!
= 10 -1 C4 -1 = 9 C3
No. of words starting with I – 5!
No. of words starting with N – 5! 17. (a) Statement 2 :
SACHIN – 1 P ( n ) : n 7 - n is divisible by 7
\ sachin appears at serial no 601
Put n = 1, 1 – 1 = 0 is divisible by7, which is true
10 10 10 10
11. (c) C1 + C2 + C3 + C4 Let n = k, P (k) : k7 – k is divisible by 7, true
= 10 + 45 + 120 + 210 = 385 Put n = k + 1
12. (a) Set S = {1, 2, 3, ...... 12} \ P (k + 1) : (k + 1) – ( k + 1) is div. by 7
7

AÈ B È C = S, AÇ B = B ÇC = A ÇC = f P(k + 1) : k7 + 7C1k6 + 7C2k2 +......+ 7C6k


\ The number of ways to partition + 1 – k – 1, is div. by 7.
= 12C4 × 8C4 × 4C4 P(k + 1) : (k7 – k) + (7C1k6 + 7C2k5 +........+
7C k) is div. by 7.
12! 8! 4! 12! 6
= ´ ´ = Since 7 is coprime with 1, 2, 3, 4, 5, 6.
4!8! 4!4! 4!0! (4!)3
13. (d) First let us arrange M, I, I, I, I, P, P So 7 C1 , 7 C 2 , ......7 C 6are all divisible by 7

7! \ P(k + 1) is divisible by 7
Which can be done in ways Hence P(n) : n7 – n is divisible by 7
4!2!
Ö MÖ IÖIÖ IÖIÖ PÖPÖ Statement 1 : n7 - n is divisible by 7
Now 4 S can be kept at any of the ticked
places in 8C4 ways so that no two S are Þ ( n + 1) 7 - ( n + 1) is divisible by 7
adjacent.
Total required ways Þ ( n + 1) 7 - n7 - 1 + ( n7 - n)
7! 8 7! 8 is divisible by 7
= C4 = C4 = 7 ´ 6C4 ´ 8C4
4!2! 4!2! Þ ( n + 1) 7 - n7 - 1 is divisible by 7
14. (c) 4 novels, out of 6 novels and 1 dictionary
Hence both Statements 1 and 2 are correct
out of 3 can be selected in 6 C4 ´3C1 ways and Statement 2 is the correct explanation
Then 4 novels with one dictionary in the of Statement -1.
middle can be arranged in 4! ways. 18. (a) Number of required triangles = 10 C3 -6 C3
\ Total ways of arrangement
10 ´ 9 ´ 8 6 ´ 5 ´ 4
= 6 C4 ´3C1 ´ 4! = 1080 = - = 120 - 20 = 100
6 6
15. (c) Total number of ways = 3 C2 ´ 9C2 19. (b) 4n – 3n – 1 = (1 + 3)n – 3n –1
9´8 = [nC0 + nC1.3 + nC2.32 +......+ nCn3n] – 3n – 1
=3× = 3 ´ 36 = 108 = 9 [nC2 +nC3.3+....+nCn.3n–2]
2
16. (a) The number of ways of distributing 10 \ 4n – 3n – 1 is a multiple of 9 for all n.
identical balls in 4 distinct boxes such that \ X = {x : x is a multiple of 9}
no box empty is same as the number of Also, Y = {9 (n – 1) : n ÎN}
ways of selecting (r – 1) places out of = {All multiples of 9}
(n – 1) different places, that is n -1
Cr -1 . Clearly X Ì Y. \ X È Y = Y
20. (c) Given
Hence require number of ways
n(A) = 2, n(B) = 4, n(A × B) = 8
www.crackjee.xyz
Permutations and Combinations M-27
Required number of subsets =
8C + 8C +.... + 8C = 28 – 8C – 8C – 8C SMALL ® 58 th word
3 4 8 0 1 2
= 256 – 1 – 8 – 28 = 219
21. (d) Four digits number can be arranged in 3 × 4 ladies 3 ladies
4! ways. 23. (b) X Y
Five digits number can be arranged in 5!
ways. 3 men 4 men
Number of integers = 3 × 4! + 5! = 192. Possible cases for X are
22. (b) ALLMS (1) 3 ladies, 0 man
No. of words starting with (2) 2 ladies, 1 man
(3) 1 lady, 2 men
4!
A: A
_____ = 12 (4) 0 ladies, 3 men
2!
Possible cases for Y are
L : L
_ _ _ _ _ 4! = 24 (1) 0 ladies, 3 men
4! (2) 1 lady, 2 men
M: M
_____ = 12 (3) 2 ladies, 1 man
2!
(4) 3 ladies, 0 man
3! No. of ways = 4C3 . 4C3 + (4C2 . 3C1)2 +
S : S
_A_____ =3
2! (4C1 . 3C2)2 + (3C3)2
: S_ L_ _ _ _ 3! = 6 = 16 + 324 + 144 + 1 = 485
EBD_7764
www.crackjee.xyz
M-28 Mathematics

Binomial Theorem 8
1. The coefficients of xp and xq in the expansion of
(1+ x )p+q are [2002]
(a) ( -1)n -1 n (b) ( -1)n (1 - n)
(a) equal (c) ( -1)n -1 (n - 1) 2 (d) (n - 1)
(b) equal with opposite signs
(c) reciprocals of each other 6
(d) none of these 9. The value of 50
C4 + å 56 - r C3 is [2005]
2. If the sum of the coefficients in the expansion of r =1
(a + b)n is 4096, then the greatest coefficient in 55
55 C3
the expansion is [2002] (a) C4 (b)
(a) 1594 (b) 792 56 56
(c) 924 (d) 2924 (c) C3 (d) C4
3. The positive integer just greater than 11
é ù
10. If the coefficient of x in ê ax 2 + æç 1 ö÷ ú
7
(1 + 0.0001)10000 is [2002]
(a) 4 (b) 5 ë è bx ø û
(c) 2 (d) 3 11
-7 é 1 öù
4. r and n are positive integers r > 1, n > 2 and equals the coefficient of x in ê ax - æç ú ,
coefficient of (r+2)th term and 3rth term in the ë è bx 2 ÷ø û
expansion of (1 + x)2n are equal, then n equals then a and b satisfy the relation [2005]
[2002] (a) a – b = 1 (b) a + b = 1
(a) 3r (b) 3r + 1
(c) 2r (d) 2r + 1 a
(c) =1 (d) ab = 1
5. If x is positive, the first negative term in the b
expansion of (1 + x)27 5 is [2003] 11.
3
If x is so small that x and higher powers of x
(a) 6th term (b) 7th term 3 3
(c) 5th term (d) 8th term æ 1 ö
6. The number of integral terms in the expansion (1 + x) 2 - ç 1 + x÷
è 2 ø
may be neglected, then
of ( 3 + 8 5 )256 is [2003] 1
(a) 35 (b) 32 (1 - x ) 2
(c) 33 (d) 34
may be approximated as [2005]
7. The coefficient of the middle term in the binomial
3 3
expansion in powers of x of (1 + ax )4 and of (a) 1 - x2 (b) 3x + x2
8 8
is the same if a equals [2004]
3 x 3 2
3 10 (c) - x2 (d) - x
(a) (b) 8 2 8
5 3
-3 -5 12. For natural numbers m, n if (1 - y ) m (1 + y ) n
(c) (d)
10 3 = 1 + a1 y + a2 y 2 + ....... and a1 = a2 = 10, then
8. The coefficient of x n in expansion of
(m, n) is [2006]
(1 + x )(1 - x )n is [2004]
www.crackjee.xyz
Binomial Theorem M-29
(a) (20, 45) (b) (35, 20) (c) Statement -1 is false, Statement -2 is true .
(c) (45, 35) (d) (35, 45) (d) Statement - 1 is true, Statement 2 is true ;
13. In the binomial expansion of (a – b)n, n ³ 5, the Statement -2 is a correct explanation for
Statement -1.
sum of 5th and 6th terms is zero, then a/b equals
18. The coefficient of x7 in the expansion of (1– x –
[2007]
x2 + x3 )6 is [2011]
n -5 n-4 (a) –132 (b) –144
(a) (b)
6 5 (c) 132 (d) 144
19. If n is a positive integer , then
5 6
(c) (d) .
( ) ( )
2n 2n
n-4 n -5 3 +1 - 3 -1 is : [2012]
14. The sum of the series [2007] (a) an irrational number
(b) an odd positive integer
20
C0 - 20
C1 + 20
C2 - 20
C3 + .....-..... + 20
C10 (c) an even positive integer
(d) a rational number other than positive
is
integers
(a) 0 (b) 20
C10 20. The term independent of x in expansion of
1 20 x +1 x -1 ö
10
(c) - 20 C10 (d) C10 æ
2 çè 2 / 3 1/3 - ÷ is [2013]
x - x + 1 x - x1/ 2 ø
n (a) 4 (b) 120
15. Statement -1 : å (r + 1) nCr = (n + 2)2n –1. (c) 210 (d) 310
r =0 21. Let Tn be the number of all possible triangles
n formed by joining vertices of an n-sided regular
Statement-2: å (r + 1) nCr x r polygon. If Tn+1 – Tn = 10, then the value of n
r =0 is : [2013]
n n –1 (a) 7 (b) 5
= (1 + x) + nx(1 + x ) . [2008]
(a) Statement -1 is false, Statement-2 is true (c) 10 (d) 8
(b) Statement -1 is true, Statement-2 is true; 22. The sum of coefficients of integral power of x in

( )
50
Statement -2 is a correct explanation for the binomial expansion 1 - 2 x is :
Statement-1
(c) Statement -1 is true, Statement-2 is true; [2015]
Statement -2 is not a correct explanation 1 50 1 50
for Statement-1 (a)
2
(
3 -1 ) (b)
2
2 +1 ( )
(d) Statement -1 is true, Statement-2 is false
1 50 1 50
16. The remainder left out when 82n – (62)2n+1 is
divided by 9 is: [2009]
(c)
2
(
3 +1 ) (d)
2
3 ( )
(a) 2 (b) 7 23. If the number of terms in the expansion of
(c) 8 (d) 0
10 10
æ 2 4 ön
çç1 - + ÷÷ , x ¹ 0, is 28, then the sum of the
17. Let S1 = å j ( j - 1) 10
C J , S2 = å j10C j è x x2 ø
j =1 j =1
coefficients of all the terms in this expansion, is :
10
and S3 = å j 2 10C j . [2010] [2016]
(a) 243 (b) 729
j =1
(c) 64 (d) 2187
Statement -1 : S3 = 55 × 29.
24. The value of
Statement - 2: S1 = 90 × 28 and S2 = 10 × 28 .
(a) Statement -1 is true, Statement -2 is true ; (21C1 – 10C1) + (21C2 – 10C2) + (21C3 – 10C3) +
Statement -2 is not a correct explanation or (21C4 – 10C4) + .... + (21C10 – 10C10) is : [2017]
Statement -1. (a) 220 – 210 (b) 221 – 211
(b) Statement -1 is true, Statement -2 is false. (c) 221 – 210 (d) 220 – 29
EBD_7764
www.crackjee.xyz
M-30 Mathematics
A n sw er K ey
1 2 3 4 5 6 7 8 9 10 11 12 13 14 15
(a) (c) (d) (c) (d) (c) ( c) ( b) (d) (d) (c) (d) (b) (d) (b)
16 17 18 19 20 21 22 23 24
(a) (b) (b) (a) (c) (b) ( c) ( b) (a)

1. (a) We have tp + 1 = p + qCp xp and tq + 1 = p+qCq


6. (c) Tr +1 = 256
Cr ( 3)256 - r (8 5)r
xq p + qC
=p
p + qC .
[ Remember
q
nC
r
256 - r
= nCn – r ] 256
2. (c) We have 2n = 4096 = 212 Þ n = 12; = Cr (3) 2 (5) r / 8
the greatest coeff = coeff of middle term.
So middle term Terms will be integral if 256 - r & r both
2 8
= t7.; t7 = t6 + 1
are +ve integer, which is so if r is an integral
12! multiple of 8. As 0 £ r £ 256
Þ coeff of t7 = 12C6 = = 924.
6!6! 7. (c) The middle term in the expansion of

æ 1ö
n
(1 + ax) 4 = T3 =4 C2 (ax )2 = 6a 2 x 2
3. (d) (1 + 0.0001)10000 = ç1 + ÷ , n = 10000
è nø The middle term in the expansion of

1 n(n - 1) 1 n( n - 1)(n - 2) 1 (1 - ax)6 = T4 =6 C3 ( -ax )3 = -20a 3 x3


= 1 + n. + + + ...
n 2! n 2 3! n3 According to the question

1 æ 1 ö 1 æ 1 ö æ 2ö 3
= 1+ 1+ ç1 - ÷ + ç1 - ÷ + ç1 - ÷ + ...
6a 2 = -20a 3 Þ a = -
2! è n ø 3! è n ø è n ø 10
8. (b) Coeff. of xn in ( 1+x) (1 – x)n
1 1 1 1
< 1+ + + + ........+ = coeff of xn in
1! 2! 3! (9999)!

1 1
= 1+ + + .......¥ = e < 3 (1 + x )(1 - n C1 x + n C2 x 2 - .... + (-1)nn Cn x n )
1! 2!
4. (c) tr + 2 = 2nCr + 1 xr + 1;t3r = 2nC3r – 1 x3r – 1 n n
= ( -1) n C n + ( -1) n -1 C n -1 = ( -1) n + ( -1) n -1 .n
Given 2nCr + 1 = 2nC3r – 1 ;
Þ 2nC2n – (r + 1) = 2nC3r – 1 = ( -1)n (1 - n)
Þ 2n – r – 1 = 3r – 1 Þ 2n = 4r Þ n = 2r
5. (d)
Coeff of x n in (1 + x ) (1 - x )
n
n (n - 1)(n - 2).........( n - r + 1) r
Tr +1 = ( x)
r! = Coeff of xn in
For first negative term,
n - r +1 < 0 Þ r > n +1
(1 - x) n + Coeff of x n -1 in (1 - x )
n

32 æ 27 ö = ( -1)n nCn + ( -1) n -1 nCn-1


Þr > \ r = 7 . çèQ n = 5 ÷ø
5
Therefore, first negative term is T8 . = ( -1) n 1 + ( -1)n -1 n
www.crackjee.xyz
Binomial Theorem M-31

= ( -1) n [1 - n] Þ 11
C5 (a)6 (b) - 5 = 11C6 a5 ´ (b) - 6
Þ ab = 1.
6
11. (c) Q x3 and higher powers of x may be
9. (d) 50
C4 + å 56 - r
C3
neglected
r =1
3 3
(1 + x) 2 - æçè1 +

é 55 C3 + 54C3 + 53C3 + 52 C3 ù ÷
= 50 C4 + ê ú \ 2ø
êë + 51C3 + 50C3 úû

n +1
(1-
1
x2 )
én
We know ë Cr + Cr -1 =
n
Cr ù
û
éæ 3 1 ö ù
-1 êç 3 . æ 3 x 3.2 x 2 ö ú
50 50 2 2 2÷
= ( C4 + C3 ) = (1 - x ) 2 êçè 1 + x + x ÷ - ç1 + + ÷ú
2 2! ø è 2 2! 4 ø úû
êë
+51 C3 +52 C3 + 53 C3 +54 C3 +55 C3
= é 1 3 ù
ê x .
= ê1 + + 2 2 x 2 ú é -3 x 2 ù = -3 x 2
(51 C4 + 51C+352
) C3 + 53 C3 + 54 C3 + 55 C3 úê ú 8
ë 2 2! ûë8 û
Proceeding in the same way, we get (as x3 and higher powers of x can be
= 55
C4 + 55
C3 = 56
C4 . neglected)

10. (d) Tr +1 in the expansion 12. (d) (1 - y ) m (1 + y ) n

11 r = [1 - m C1 y + m C2 y 2 - ......]
é 2 1 ù 11 2 11 - r æ 1ö
ê ax + bx ú = Cr (ax ) çè ÷ø
ë û bx [1 + n C1 y + n C 2 y 2 + .....]

= 11 Cr (a)11 - r (b) - r ( x)22 - 2r - r ì m( m - 1) n(n - 1) ü


= 1 + ( n - m) + í + - mný y 2 + .....
For the Coefficient of x7, we have î 2 2 þ
22 – 3r = 7 Þ r = 5
\ a1 = n - m = 10
\ Coefficient of x 7
m2 + n2 - m - n - 2mn
11 6 -5 and a2 = = 10
= C5 (a) (b) 2
...(1)
So, n – m = 10 and (m - n) 2 - (m + n) = 20
Again T r + 1 in the expansion
Þ m + n = 80
11 r \ m = 35, n = 45
é 1 ù æ 1 ö
ê ax - 2 ú = 11Cr (ax 2 )11 - r ç - 13. (b) Tr + 1 = (–1)r. nCr (a)n – r. (b)r is an expansion
ë bx û è bx 2 ÷ø of (a – b)n
11 11 - r r -r - 2r 11 - r \ 5th term = t5 = t4+1
= Cr (a ) ( -1) ´ (b ) ( x ) ( x)
= (–1)4. nC4 (a)n–4.(b)4 = nC4 . an–4 . b4
–7
For the Coefficient of x , we have 6th term = t6 = t5+1 = (–1)5 nC5 (a)n–5 (b)5
Now 11 – 3r = – 7 Þ 3r = 18 Þ r = 6 Given t5 + t6 = 0
\ Coefficient of x - 7 \ nC4 . an–4 . b4 + (– nC5 . an–5 . b5) = 0

= 11C6 a5 ´ 1 ´ (b) - 6 n! an n! a n b5
Þ . .b4 - . =0
\ Coefficient of x7 = Coefficient of x–7 4!(n - 4)! a 4 5!(n - 5)! a5
EBD_7764
www.crackjee.xyz
M-32 Mathematics

n !.a n b 4
é 1 b ù + 2 n +1C2 (63)2 n -1 – ........+ (–1)2n+1 2n+1C2+1 ù
Þ - =0 û
4 ê ( n - 4) 5.a ú
4!( n - 5)!.a ë û = 63 ×
1 b a n-4 é nC0 (63)n -1 + n C1 (63)n - 2 + n C2 (63)n -3
or, - =0 Þ = ë
n - 4 5a b 5
14. (d) We know that, (1 + x) = C0 + 20C1x +
20 20 +........] + 1 – 63 ×
20C x2 + ...... 20C x10 + ..... 20C x20
2 10 20 é 2n+1C (63) 2n - 2n+1C (63)2n -1 + .......ù + 1
ë 0 1 û
Put x = –1, (0) = 20C0 – 20C1 + 20C2 – 20C3
= 63 × some integral value + 2
+ ...... + 20C10 – 20C11 .... + 20C20 = 82n – (62)2n+1when divided by 9 leaves
Þ 0 = 2[20C0 – 20C1 + 20C2 – 20C3 2 as the remainder.
+ ..... – 20C9] + 20C10 10 10
Þ 20C = 2[20C – 20C + 20C – 20C
10 0 1 2 3
17. (b) S2 = å j 10
C j = å 10 9 C j -1
j =1 j =1
+ ...... – 20C9 + 20C10]
Þ 20C – 20C + 20C – 20C + .... + 20C
0 1 2 3 10 = 10 é 9 C0 + 9C1 + 9 C2 + .... + 9 C9 ù = 10.29
1 ë û
= 20C10
2 18. (b) (1 – x – x2 + x3)6 = [(1– x) – x2 (1 – x)]6
15. (b) We have
= (1– x)6 (1 – x2)6
n n
n = (1 – 6x + 15x2 – 20x3 + 15x4 – 6x5 + x6)
å (r + 1) nCr x r = å r. nCr x r + å nCr xr × (1 – 6x2 + 15x4 – 20x6 + 15x8 – 6x10 + x12)
r =0 r =0 r =0
Coefficient of x7 = (– 6) (– 20) + (– 20)(15)
n + (– 6) (–6) = – 144
n
=år× r
n –1
Cr –1 x r + (1 + x)n
( ) ( )
2n 2n
r =1 19. (a) Consider 3 +1 - 3 -1
n
= nx å
n –1
Cr –1 x r –1 +(1 + x )n
( 3) ( 3)
2 n -1 2 n -3
= 2 é 2n C1 + 2 n C3
r =1 êë
= nx (1+ x) n–1 + (1+ x) n = RHS
( 3)
2n-5
\ Statement 2 is correct. + ....ù
+ 2 n C5
úû
Putting x = 1, we get
n
(Using binomial expansion of (a + b) and
n
(a – b)n)
å (r + 1)n Cr = n × 2n –1 + 2n = (n + 2) × 2n –1. = which is an irrational number.
r=0
\ Statement 1 is also true and statement 2
is a correct explanation for statement 1. 20. (c) Given expression can be written as
16. (a) (8)2n – (62) 2n + 1 10
= (64) n – (62)2n + 1 æ 1/ 3 æ x +1 ö ö
ç ( x + 1) - çç ÷÷
= (63 + 1)n – (63 – 1)2n + 1 ç
è è x ÷ø ÷ø
=
10
é nC (63)n + nC (63)n -1 + nC (63)n - 2 æ 1 ö
ë 0 1 2 = ç x1/ 3 + 1 - 1 - ÷
è xø
+ ........+
n
Cn-1 (63) + nCn ù = (x1/3 – x–1/2)10
û
General term = Tr+1
2n +1
= éë C0 (63) 2n +1 - 2n +1C1 (63)2 n = 10Cr (x1/3)10–r(–x–1/2)r
www.crackjee.xyz
Binomial Theorem M-33
10 - r
-r 50
10
Cr x 3 · ( -1) r · x 2 +... + C3 (2 x )3 - 50
C 4 (2 x ) 4 ...(2)
=
Adding equation (1) and (2)
10- r - r
50 50
= 10 Cr ( -1) r · x 3 2 (1 - 2 x) + (1 + 2 x )
Term will be independent of x when
= 2 éë 50 C0 + 50C2 22 x + 50
C4 23 x 2 + ...ùû
10 - r r
- = 0
3 2 350 + 1
Putting x = 1, we get above as
Þ r=4 2
So, required term = T5 = 10C4 = 210 23. (b) Total number of terms = n+2C2 = 28
21. (b) We know, (n + 2) (n + 1) = 56; x = 6
Tn = nC3, Tn+1 = n+1C3 24. (a) We have (21C1 + 21C2 ...... + 21C10)
ATQ, Tn+1 – Tn = n+1C3 – nC3 = 10 – (10C1 + 10C2 ..... 10C10)
Þ nC2 = 10 1 21
Þ n = 5. = [( C1 + .... + 21C10 ) + (21C11 + .... 21C20)]
2
– (210 – 1)
(1 - 2 x )50 =50C0 -50C1 2 x +50C2 ( 2 x )
2
22. (c)
(Q 10C1 + 10C2 + .... + 10C10 = 210 – 1)
...(1)
1 21
50 50 50 50 2 = [2 - 2] - (210 - 1)
(1 + 2 x ) = C0 + C1 2 x - C 2 (2 x ) 2
= (220 – 1) – (210 – 1) = 220 – 210
EBD_7764
www.crackjee.xyz
M-34 Mathematics

Sequences and Series


9
1. If 1, log9 (31–x + 2), log3 (4.3x – 1) are in A.P. then n n
1 r t
x equals
(a) log3 4
[2002]
(b) 1 – log3 4
8. If Sn = å nC and t n = å nC , then n
Sn
r =0 r r =0 r
(c) 1 – log4 3 (d) log4 3 is equal to [2004]
2. The value of 21/4. 41/8. 81/16 ... ¥ is [2002]
(a) 1 (b) 2 2n –1 1
(a) (b) n -1
(c) 3/2 (d) 4 2 2
3. Fifth term of a GP is 2, then the product of its 9 1
terms is [2002] (c) n – 1 n(d)
2
(a) 256 (b) 512
9. Let Tr be the rth term of an A.P. whose first term
(c) 1024 (d) none of these
is a and common difference is d. If for some
4. Sum of infinite number of terms of GP is 20 and
sum of their square is 100. The common ratio of 1
positive integers m, n, m ¹ n, Tm = and
GP is [2002] n
(a) 5 (b) 3/5
1
(c) 8/5 (d) 1/5 Tn = , then a – d equals [2004]
m
5. 13 – 23 + 33 – 43 +...+93 = [2002]
(a) 425 (b) –425 1 1
(a) + (b) 1
(c) 475 (d) –475 m n
6. The sum of the series [2003]
1
1 1 1 (c) (d) 0
- + .......... .. up to ¥ is equal to mn
1.2 2.3 3.4
10. The sum of the first n terms of the series
æ 4ö
(a) log e ç ÷ (b) 2 log e 2
èeø 12 + 2.2 2 + 3 2 + 2.4 2 + 5 2 + 2.6 2 + ...
(c) log e 2 - 1 (d) log e 2
n(n + 1)2
7. If the sum of the roots of the quadratic equation is when n is even. When n is odd the
2
ax 2 + bx + c = 0 is equal to the sum of the sum is [2004]
a b c
squares of their reciprocals, then , and é n(n + 1) ù
2
n2 (n + 1)
c a b (a) (b)
ê 2 ú
are in [2003] ë û 2
(a) Arithmetic - Geometric Progression
(b) Arithmetic Progression n(n + 1)2 3n(n + 1)
(c) (d)
(c) Geometric Progression 4 2
(d) Harmonic Progression.
www.crackjee.xyz
Sequences and Series M-35

16. Let a1, a2 , a3 ............ be terms on A.P. If


1 1 1
11. The sum of series + + + ..... is [2004]
2! 4! 6! a1 + a2 + ...........a p p2 a6
= , p ¹ q , then
a1 + a2 + ........... + aq q 2
a21
(e2 - 2) (e - 1)2
(a) (b)
e 2e equals [2006]
41 7
(e2 - 1) (e2 - 1) (a) (b)
(c) (d) 11 2
2e 2
12. Let two numbers have arithmetic mean 9 and 2 11
geometric mean 4. Then these numbers are the (c) (d)
7 41
roots of the quadratic equation [2004]
17. If a1, a2, .........., an are in H.P., then the expression
(a) x 2 - 18 x - 16 = 0 a1a2 + a2a3 + .......... + an–1an is equal to
[2006]
(b) x 2 - 18 x + 16 = 0
(a) n(a1 - an ) (b) (n - 1)(a1 - an )
(c) x 2 + 18 x - 16 = 0
(c) na1an (d) (n - 1)a1an
(d) x 2 + 18 x + 16 = 0
13. If the coefficients of rth, (r + 1)th, and (r + 2)th 1 1 1
18. The sum of series - + - ....... upto
terms in the the binomial expansion of (1 + y ) m 2! 3! 4!
are in A.P., then m and r satisfy the equation infinity is [2007]
[2005] (a) 1
(b) +
1
-
2 e 2 e 2
(a) m 2 – m (4r – 1) + 4 r – 2 = 0
(c) e–2 (d) e–1
2
(b) m2 – m (4r + 1) + 4 r + 2 = 0 19. In a geometric progression consisting of positive
(c) 2 2
m – m (4r + 1) + 4 r – 2 = 0 terms, each term equals the sum of the next two
terms. Then the common ratio of its progression
(d) m2 – m (4r – 1) + 4 r 2 + 2 = 0 is equals [2007]
¥ ¥ ¥
14. If x = å an , y = å bn , z = å cn where a, (a) 5 (b)
1
2
( )
5 -1
n =0 n=0 n=0
b, c are in A.P and |a | < 1, | b | < 1, | c | < 1 then x,
y, z are in [2005]
(c)
1
2
1- 5 ( ) (d)
1
2
5.
(a) G. P. 20. The first two terms of a geometric progression
(b) A.P. add up to 12. the sum of the third and the fourth
(c) Arithmetic - Geometric Progression
terms is 48. If the terms of the geometric
(d) H.P.
progression are alternately positive and
15. The sum of the series [2005]
negative, then the first term is [2008]
1 1 1 (a) –4 (b) –12
1+ + + + ....................ad inf. is
4.2! 16.4! 64.6 ! (c) 12 (d) 4
21. The sum to infinite term of the series
e -1 e +1
(a) (b) 2 6 10 14 ... is [2009]
e e 1+ + + + +
3 32 33 34
e -1 e +1 (a) 3 (b) 4
(c) (d)
2 e 2 e (c) 6 (d) 2
EBD_7764
www.crackjee.xyz
M-36 Mathematics
22. A person is to count 4500 currency notes. Let an 27. The sum of first 20 terms of the sequence 0.7,
denote the number of notes he counts in the nth 0.77, 0.777,....., is [2013]
minute. If a1 = a2 = ... = a10 = 150 and a10, a11, ... 7
are in an AP with common difference –2, then the (a) (179 - 10-20 )
81
time taken by him to count all notes is [2010]
(a) 34 minutes (b) 125 minutes 7
(b) (99 - 10-20 )
(c) 135 minutes (d) 24 minutes 9
23. A man saves ` 200 in each of the first three
months of his service. In each of the subsequent 7
(c) (179 + 10 -20 )
months his saving increases by ` 40 more than 81
the saving of immediately previous month. His
7
total saving from the start of service will be (d) (99 + 10 -20 )
` 11040 after [2011] 9
(a) 19 months (b) 20 months 28. If x, y, z are in A.P. and tan–1x, tan–1y and tan–1z
(c) 21 months (d) 18 months are also in A.P., then [2013]
(a) x = y = z (b) 2x = 3y = 6z
24. Let an be the n th term of an A.P. If
(c) 6x = 3y = 2z (d) 6x = 4y = 3z
100 100 29. Let a and b be the roots of equation px2 + qx + r
å a2r = a and å a2r –1 = β, then the common
1 1
r =1 r =1 = 0, p ¹ 0. If p, q, r are in A.P and + = 4, then
difference of the A.P. is [2011] a b
a -b the value of | a – b| is: [2014]
(a) a -b (b)
100 34 2 13
(a) (b)
α–β 9 9
(c) b-a (d)
200
25. Statement-1: The sum of the series 1 + (1 + 2 + 4) 61 2 17
(c) (d)
+ 9 9
(4 + 6 + 9) + (9 + 12 + 16) + .... + (361 + 380 + 400)
is 8000.
9 1
( ) 2
30. If (10 ) + 2 (11) 10 + 3 (11) (10 ) + .....
8 7

n
Statement-2: å ( k 3 - ( k - 1)3 ) = n3, for any 9 9
+10 (11) = k (10 ) , then k is equal to: [2014]
k =1
(a) 100 (b) 110
natural number n. [2012]
(a) Statement-1 is false, Statement-2 is true. 121 441
(c) (d)
(b) Statement-1 is true, statement-2 is true; 10 100
statement-2 is a correct explanation for 31. Three positive numbers form an increasing G. P.
Statement-1. If the middle term in this G.P. is doubled, the new
(c) Statement-1 is true, statement-2 is true; numbers are in A.P. then the common ratio of
statement-2 is not a correct explanation for the G.P. is: [2014]
Statement-1.
(a) 2- 3 (b) 2+ 3
(d) Statement-1 is true, statement-2 is false.
26. If 100 times the 100th term of an AP with non zero (c) 2+ 3 (d) 3 + 2
common difference equals the 50 times its 50th 32. The sum of first 9 terms of the series.
term, then the 150th term of this AP is : [2012]
13 13 + 23 13 + 23 + 33
(a) – 150 + + + .... [2015]
(b) 150 times its 50th term 1 1+ 3 1+ 3 + 5
(c) 150 (a) 142 (b) 192
(d) Zero (c) 71 (d) 96
www.crackjee.xyz
Sequences and Series M-37
33. If m is the A.M. of two distinct real numbers l and
16
n(l, n > 1) and G1, G2 and G3 are three geometric is m, then m is equal to : [2016]
5
means between l and n, then G14 + 2G 24 + G 34 (a) 100 (b) 99
equals. [2015] (c) 102 (d) 101
(a) 4 lmn2 (b) 4 l2m2n2 36. For any three positive real numbers a, b and c,
(c) 4 l2 mn (d) 4 lm2n 9(25a2 + b2) + 25(c2 – 3ac) = 15b(3a + c). Then :
34. If the 2nd, 5th and 9th terms of a non-constant [2017]
A.P. are in G.P., then the common ratio of this
(a) a, b and c are in G.P.
G.P. is : [2016]
(b) b, c and a are in G.P.
7 (c) b, c and a are in A.P.
(a) 1 (b)
4 (d) a, b and c are in A.P.
8 4 37. Let a, b, c Î R. If f(x) = ax2 + bx + c is such that a
(c) (d) + b + c = 3 and f(x + y) = f(x) + f(y) + xy, " x, y Î R,
5 3
35. If the sum of the first ten terms of the series 10
then å f ( n ) is equal to : [2017]
æ 3 ö2 æ 2 ö2 æ 1 ö2 2 æ 4 ö2 n =1
çç1 ÷÷ + çç2 ÷÷ + çç3 ÷÷ + 4 + çç4 ÷÷ + ......., (a) 255 (b) 330
è 5ø è 5ø è 5ø è 5ø (c) 165 (d) 190

A n sw er K ey
1 2 3 4 5 6 7 8 9 10 11 12 13 14 15
(b) (b) (b) (b ) (a) ( a) (d) (d ) (d) (b) (b) ( b) (c) (d) (d)
16 17 18 19 20 21 22 23 24 25 26 27 28 29 30
(d) (d) (d) (b ) (b) ( a) (a) (c) (b) (b) (d) (c) (a) (b) (a)
31 32 33 34 35 36 37
(b) (d) (d) (d ) (d) ( c) (b)

1. (b) 1, log9 (31 – x + 2), log3 (4.3x – 1) are in A.P.


æ 3ö
Þ 2 log9 (31– x+2) = 1 + log3 (4.3x – 1) Þ x = log3 ç ÷ or x = log3 3 – log3 4
è 4ø
Þ log3 (31 – x + 2) = log33 + log3 (4.3x – 1)
Þ log3 (31– x + 2) = log3 [3(4 × 3x – 1)] Þ x = 1 – log3 4
Þ 31– x + 2 = 3 (4.3x – 1) 2. (b) The product is P = 21/ 4.22 / 8.23 /16.........
Þ 3.3–x + 2 = 12.3x – 3.
= 21 / 4 + 2 / 8 + 3 / 16+..........¥
Put 3x = t
3 Now let S = 1 + 2 + 3 + ........¥ ......(1)
Þ + 2 = 12t - 3 or 12t2 – 5t – 3 = 0; 4 8 16
t
1 1 2
1 3 S = + + ........¥ ......(2)
Hence t = - , 2 8 16
3 4
Subtracting (2) from (1)
3
Þ 3x = (as 3 x ¹ - ve ) 1 1 1 1
4 Þ S = + + + ........¥
2 4 8 16
EBD_7764
www.crackjee.xyz
M-38 Mathematics
1 1/ 4 1
or S= = Þ S =1 2c b2
2 1 - 1/ 2 2 2 2 -
a +b b a2 a
a +b = - =
\ P = 2S = 2 a 2b 2 a c2
3. (b) ar4 = 2
a2
2 3 4 5 6 7 8
a ´ ar ´ ar ´ ar ´ ar ´ ar ´ ar ´ ar ´ ar
= a9 r36 = (ar4)9 = 29 = 512 On simplification 2a 2 c = ab 2 + bc 2
4. (b) Let a = first term of G.P. and r = common 2a c b c a b
ratio of G.P.; Then G.P. is a, ar, ar2 Þ = + Þ , , are in A.P..
b a c a b c
a
Given S ¥ = 20 Þ = 20 a b c
1- r \ , , & are in H.P..
c a b
Þ a = 20(1 – r)... (i)
Also a2 + a2r2 + a2r4 + ... to ¥ =100 1 1 1 1
8. (d) Sn = + + + .... +
n n n n
C0 C1 C2 Cn
a2
Þ =100
1 - r2 0 1 2 n
tn = + + + .... +
Þ a2 = 100(1 – r)(1 + r)... (ii) n
C0 n
C1 n
C2 n
Cn
From (i), a2 = 400(1 – r)2;
From (ii), we get 100(1 – r)(1 + r) = 400(1 – r)2 n n -1 n-2 0
tn = + + + .... +
Þ 1 + r = 4 – 4r Þ 5r = 3 Þ r = 3/5. n
Cn n
Cn-1 n
Cn- 2 n
C0
5. (a) 13 – 23 + 33 – 43 + ........+ 93
= 13 + 23 + 33 +.......+ 93 – 2(23 + 43 + 63 + 83) Add,
é 1 1 ù
[ ] 1
2
é 9 ´10 ù =2tn (n) ê n += + .... ú nS n
=ê ú - 2.2 3 13 + 2 3 + 33 + 4 3 n n
ë 2 û êë C0 C1 Cn úû

2 t n
é4´5ù =\ n
= (45) 2 - 16.ê ú = 2025 – 1600 = 425 Sn 2
ë 2 û
1
1 1 1 9. (d) Tm = a+ (m – 1) d = .....(1)
6. (a) - + ...............¥ n
1.2 2.3 3.4
1
1 æ1 1 ö Tn = a + (n - 1)d = .....(2)
Tn = =ç - ÷ m
n (n + 1) è n n + 1ø
S = T1 - T2 + T3 - T4 + T5 ............¥ 1 1 1
(1) – (2) Þ (m - n )d = - Þd=
n m mn
æ1 1 ö æ 1 1 ö æ 1 1 ö æ 1 1 ö 1
= ç - ÷ - ç - ÷ + ç - ÷ - ç - ÷.....
è1 2 ø è 2 3 ø è 3 4 ø è 4 5 ø From (1) a = Þa-d =0
mn
é1 1 1 1 ù
= 1 - 2 ê - + - ................¥ú 10. (b) If n is odd, the required sum is
ë 2 3 4 5 û
æ4ö 12 + 2.22 + 32 + 2.42 + ...... + 2.(n - 1) 2 + n2
= 1 - 2[- log(1 + 1) + 1] = 2 log 2 - 1 = logç ÷.
-b cè e ø (n - 1)(n - 1 + 1) 2
7. (d) ax 2 + bx + c = 0, a + b = , ab = = + n2
a a
2
1 1 [Q (n–1) is even
As for given condition, a + b = 2
+
a b2
www.crackjee.xyz
Sequences and Series M-39
\ using given formula for the sum of a, b, c are in A.P. OR 2b = a + c
(n–1) terms.]
æ 1ö 1 1
2 2 ç1 - ÷ = 1 - + 1 -
æ n - 1 ö 2 n (n + 1) è yø x y
=ç + 1÷ n =
è 2 ø 2
2 1 1
= + Þ x, y, z are in H.P..
1 1 1 y x z
11. (b) We know that e = 1 + + + + .......
1! 2! 3!
15. (d) e x + e- x x 2 x 4 x6
-1 1 1 1 = 1+ + + .............
and e = 1 - + - + ....... 2 2! 4! 6!
1! 2! 3!
1
é 1 1 ù
\ e + e -1 = 2 ê1 + + + ....ú Putting x =
2
we get
ë 2! 4! û
1 1 1
1 1 1 e + e -1 1+ + + + ......
\ + + + ...... = -1 4.2! 16.4! 64.6!
2! 4! 6! 2
1 -1 1
e+
e 2 + 1 - 2e (e - 1)2 e2 + e 2
e e +1
= = ¥= = =
2e 2e 2 2 2 e
a+b
12. (b) Let two numbers be a and b then =9 p
2 [2a + ( p - 1)d ]
2 1 p2
16. (d) =
and ab = 4 q 2
[2a1 + (q - 1)d ] q
\ Equation with roots a and b is 2
x 2 - (a + b ) x + ab = 0 2a1 + ( p - 1)d p
2
Þ 2a + (q - 1)d = q
Þ x - 18 x + 16 = 0 1

m m m
13. (c) Given Cr -1 , Cr , Cr +1 are in A.P.. æ p - 1ö
a1 + ç d
è 2 ÷ø p
2m Cr = m Cr -1 + m Cr +1 =
æ q - 1ö q
a1 + ç d
m è 2 ÷ø
Cr -1 m Cr +1
Þ 2= +
m m
Cr Cr a6 a 11
For , p = 11, q = 41 Þ 6 =
r m-r a21 a21 41
= +
m - r +1 r +1 1 1 1 1 1 1
2 2 17. (d) a - a = a - a =..........= - =d
Þ m - m(4r + 1) + 4r - 2 = 0 . 2 1 3 2 an an -1
¥
(say)
1 1
14. (d) x = å an =
1- a
a = 1-
x Then a1a2 =
a1 - a2
, a2 a3 =
a2 - a3
,
n=0 d
d
¥ an -1 - an
1 1
y= å bn = 1 - b b = 1-
y
..........., an -1an =
d
n=0
\ a1a2 + a2 a3 + ......... + an-1an
¥ 1
1
z = å cn = c = 1-
n=0
1- c z
EBD_7764
www.crackjee.xyz
M-40 Mathematics
a1 - a2 a2 - a3 a -a 21. (a) We have
= + + .... + n -1 n
d d d 2 6 10 14
S = 1 + + 2 + 3 + 4 + .......¥ ....(1)
3 3 3 3
1
= [a1 - a2 + a2 - a3 + .... + an -1 - an ]
d 1
Multiplying both sides by we get
a1 - an 3
=
d 1 1 2 6 10
S = + 2 + 3 + 4 + ........¥ ....(2)
1 1 3 3 3 3 3
Also, = + (n - 1)d
an a1 Subtracting eqn. (2) from eqn. (1) we get
2 1 4 4 4
a1 - an S = 1 + + 2 + 3 + 4 + ........¥
Þ = (n - 1)d 3 3 3 3 3
a1an
2 4 4 4 4
a -a Þ S = + 2 + 3 + 4 + ........¥
Þ 1 n = (n - 1)a1an 3 3 3 3 3
d
Which is the required result. 4
2 3 = 4´3 Þ S =3
x 2 x3 Þ S=
18. (d) We know that ex = 1 + x + + + ........¥ 3 1 3 2
2! 3! 1-
3
Put x = – 1
22. (a) Till 10th minute number of counted notes
\ 1 1 1 = 1500
e–1 = 1 - 1 + - + ........¥
2! 3! 4! n
3000 = [ 2 ´ 148 + ( n - 1)( -2) ] = n [148 - n + 1]
1 1 1 1 2
\ e–1 = - + - ........¥
2! 3! 4! 5! n 2 - 149 n + 3000 = 0
19. (b) Let the series a, ar, ar2, ..... are in geometric Þ n = 125, 24
progression. But n = 125 is not possible
given, a = ar + ar 2 \ total time = 24 + 10 = 34 minutes.
Þ 1= r + r2 Þ r2 + r – 1 = 0 23. (c) Let required number of months = n
-1 ± 1 - 4 ´ -1 \ 200 × 3 + (240 + 280 + 320 + ... + (n – 3)th
Þ r= term)
2
= 11040
-1 ± 5
Þ r= n-3
2 Þ [ 2 ´ 240 + (n - 4) ´ 40]
2
5 -1 = 11040 - 600
Þ r= [Q terms of G.P. are positive
2 Þ (n - 3)[240 + 20n - 80] = 10440
\ r should be positive] Þ(n - 3)(20n + 160) = 10440
20. (b) As per question,
Þ(n - 3)(n + 8) = 522
a + ar = 12 …(1)
ar2 + ar3 = 48 …(2) Þn2 + 5n - 546 = 0
(n + 26) (n – 21) = 0
Þ
ar 2 (1 + r ) 48 \ n = 21
Þ = Þ r2 = 4, Þ r = –2
a(1 + r ) 12 24. (b) Let A.P. be a, a + d , a + 2d ,.........
(Q terms are = + ve and –ve alternately) a2 + a4 + ........... + a200 = a
Þ a = –12
www.crackjee.xyz
Sequences and Series M-41
100
Þ é 2 ( a + d ) + (100 - 1) d ùû = a ....(i) éæ 1ö æ 1 ö æ 1 öù
2 ë 7 êç1 - ÷ + ç1 - 2 ÷ + ç1 - 3 ÷ ú
= êè 10 ø è 10 ø è 10 ø ú
and a1 + a3 + a5 + ......... + a199 = b 9
100 ëê +¼...up to 20 terms ûú
Þ
2
[2a + (100 – 1) d ] = β ....(ii)
On solving (i) and (ii), we get é 1 æ æ 1ö öù
20
ê ç ç ÷ ÷ú
1 -
a -b 7ê 10 è è 10 ø ø ú
d= = ê 20 - ú
100 9ê 1
1- ú
25. (b) nth term of the given series ê 10 ú
ë û
2
= Tn = ( n - 1) + ( n - 1) n + n 2
7 é179 1 æ 1 ö
20 ù

=
( ( n - 1)3 - n 3 ) 3
= n - ( n - 1) 3
= ê + ç ÷
9 êë 9 9 è 10 ø
ú
úû
( n - 1) - n
7
n = [179 + (10)–20]
Þ Sn = å é k 3 - ( k - 1) 3 ù Þ 8000 = n3
ë û
28.
81
(a) Since, x, y, z are in A.P.
k =1
Þ n = 20 which is a natural number. \ 2y = x + z
Now, put n =1,2,3,....20 Also, we have
T1 = 13 – 03 2 tan–1 y = tan–1x + tan–1 (z)
T2 = 23 – 13 æ 2y ö -1 æ x + z ö
M Þ tan–1 ç ÷ = tan çè 1 - xz ÷ø
è 1 - y2 ø
T20 = 203 – 193
Now, T1 + T2 + --- + T20 = S20 x+ z x+z
Þ S20 = 203 – 03 = 8000 Þ = (Q 2y = x + z)
1- y 2
1 - xz
Hence, both the given statement is true.
26. (d) Let 100th term of an AP is a + (100 –1) d Þ y2 = xz or x + z = 0 Þ x = y = z
= a + 99d where 'a' is the first term of A.P 29 (b) Let p, q, r are in AP
and 'd' is the common difference of A.P. Þ 2q = p + r ...(i)
Similarly, 50th term = a + (50 – 1) d 1 1
= a + 49d Given + =4
Now, According to the question a b
100 (a + 99d) = 50 (a + 49d) a +b
Þ 2a + 198 d = a + 49d Þ a + 149 d = 0 Þ =4
ab
This is the 150th term of an A.P.
Hence, T150 = a + 149 d = 0 r
27. (c) Given sequence can be written as We have a + b = – q/p and ab =
p
7 77 777
+ + + ...¼+ up to 20 terms
10 100 103 q
-
p
é 1 11 111 ù Þ = 4 Þ q = - 4r ....(ii)
= 7ê + + +¼... + up to 20 termsú r
ë10 100 103 û p
Multiply and divide by 9
From (i), we have
7 é 9 99 999 ù 2( – 4r) = p + r
= ê + + + ...¼up to 20 termsú
9 ë10 100 1000 û p = –9r
EBD_7764
www.crackjee.xyz
M-42 Mathematics
q = – 4r
r=r 32. (d) nth term of series

Now | a - b | = (a + b ) 2 - 4ab é n(n + 1) ù


2
êë 2 úû 1
2 = = (n + 1) 2
æ -q ö 4r q 2 - 4 pr n 2 4
= ç ÷ - =
è pø p | p|
1 2
Sum of n term = S (n + 1)
4
16 r 2 + 36 r 2 2 13
= =
| -9 r | 1é 2
9 = Sn + 2 Sn + n ù
30. (a) Let 109 + 2. (11)(10)8 + 3(11)2 (10)7 4ë û
+ ... + 10(11)9 = k(10)9 1 é n(n + 1)(2n + 1) 2n(n + 1) ù
Let x = 109 + 2.(11)(10)8 + 3(11)2(10)7 + ... + = + + nú
4 êë 6 2 û
10(11)9
Sum of 9 terms
11
Multiplied by on both the sides 1 é 9 ´ 10 ´ 19 18 ´ 10 ù
10 = + + 9ú
4 êë 6 2 û
11
x = 11.108 + 2.(11)2.(10)7 + ...+ 9(11)9 + 384
10 = = 96
1110 4

æ 11ö l+n
x ç1 - ÷ = 109 + 11 (10)8 + 112 × (10)7 33. (d) m= and common ratio of G.P.
è 10 ø 2
+ ... + 119 – 1110 1

= r = æç ö÷ 4
n
é æ 11 ö10 ù èlø
ê ç ÷ - 1ú
x
- = 109 ê è ø
10 ú - 1110 \ G1 = l3/4n1/4, G2 = l1/2n1/2, G3 = l1/4 n3/4
Þ ê 11 ú
10
ê -1 ú 3 2 2
G14 + 2G 42 + G34 = l n + 2l n + ln
3
êë 10 úû
= ln (l + n)2
= ln × 2m2
x
Þ - = (1110 - 1010 ) - 1110 = -1010 = 4lm2n
10 34. (d) Let the GP be a, ar and ar 2 then a = A + d;
Þ x = 1011 = k.109 Given ar= A + 4d; ar2 = A + 8d
Þ k = 100
31. (b) Let a, ar, ar2 are in G.P. ar 2 - ar (A + 8d)-(A + 4d)
According to the question Þ =
ar - a (A + 4d)-(A + d)
a, 2ar, ar2 are in A.P.
Þ 2 × 2ar = a + ar2 4
Þ 4r = 1 + r2 Þ r2 – 4r + 1 = 0 r=
3
4 ± 16 - 4 35. (d)
r= = 2± 3
2 æ 8 ö2 æ12 ö2 æ16 ö2 æ 20 ö2 æ 44 ö2
Since r > 1 çç 5 ÷÷ + çç 5 ÷÷ + çç 5 ÷÷ + çç 5 ÷÷ ... + çç 5 ÷÷
è ø è ø è ø è ø è ø
\ r = 2 - 3 is rejected
16 2 2 2
Hence, r = 2 + 3 S=
25 (
2 + 3 + 4 + ... + 112 )
www.crackjee.xyz
Sequences and Series M-43
Þ a + b = 2c
16 æ11(11 + 1)(22 + 1) ö
= ç
ç - 1÷
÷ Þ b, c, a are in A.P.
25 è 6 ø 37. (b) f(x) = ax2 + bx + c
16 16 f(1) = a + b + c = 3 Þ f (1) = 3
= ´505 = ´101 Now f(x + y) = f(x) + f(y) + xy ...(1)
25 5
Put x = y = 1 in eqn (1)
16 16 f(2) = f(1) + f(1) + 1
Þ m = ´101 = 2f(1) + 1
5 5
f(2) = 7
Þ m = 101. Þ f(3) = 12
36. (c) We have Now, Sn = 3 + 7 + 12 + ......... tn ...(1)
9(25a2 + b2) + 25 (c2 – 3ac) = 15b (3 a + c) Sn = 3 + 7 + ......... tn–1 + tn ...(2)
Þ 225a2 + 9b2 + 25c2 – 75ac = 45ab + 15bc Subtract (2) from (1)
Þ (15a)2 + (3b)2 + (5c)2 – 75ac – 45ab–15 bc= 0 tn = 3 + 4 + 5 + .... upto n terms
1
[(15 a – 3b)2 + (3b – 5c)2 + (5c – 15a)2 ] = 0 (n 2 + 5n)
2 tn =
2
it is possible when 15a – 3b = 0, 3b – 5 c =
0 and 5c – 15a = 0 (n 2 + 5n)
Þ 15a = 3b = 5 Sn = å tn = å 2
5c c Sn=
Þb= ,a =
3 3 1 é n(n + 1)(2n + 1) 5n(n + 1) ù n(n + 1)(n + 8)
+ ú=
2 êë 6 2 û 6
c 5c 6c
Þa+b= + =
3 3 3 10 ´11 ´18
S10 = = 330
6
EBD_7764
www.crackjee.xyz
M-44 Mathematics

Straight Lines &


Pair of Straight Lines 10
1. A triangle with vertices (4, 0), (–1, –1), (3, 5) is (a) y (cos a + sin a ) + x(cos a - sin a ) = a
[2002]
(b) y (cos a - sin a ) - x(sin a - cos a ) = a
(a) isosceles and right angled
(b) isosceles but not right angled (c) y (cos a + sin a ) + x (sin a - cos a) = a
(c) right angled but not isosceles (d) y (cos a + sin a ) + x (sin a + cos a ) = a .
(d) neither right angled nor isosceles
2. Locus of mid point of the portion between the 6. If the pair of straight lines x 2 - 2 pxy - y 2 = 0
axes of x cos a + y sina = p whre p is constant and x 2 - 2qxy - y 2 = 0 be such that each pair
is [2002] bisects the angle between the other pair, then
4 [2003]
(a) x2 + y2 = (b) x2 + y2 = 4p2
p2 (a) pq = –1 (b) p = q
(c) p = –q (d) pq = 1.
1 1 2 1 1 4 7. Locus of centroid of the triangle whose vertices
(c) + = (d) 2
+ 2
=
x 2
y 2
p 2 x y p2
are (a cos t , a sin t ), (b sin t , - b cos t ) and (1, 0),
3. If the pair of lines ax2 + 2hxy + by2 + 2gx + 2fy + where t is a parameter, is [2003]
c = 0 intersect on the y-axis then
(a) (3x + 1) 2 + (3 y ) 2 = a 2 - b 2
[2002]
(a) 2fgh = bg2 + ch2 (b) bg2 ¹ ch2
(b) (3x - 1) 2 + (3 y ) 2 = a 2 - b 2
(c) abc = 2fgh (d) none of these
4. The pair of lines represented by (c) (3x - 1) 2 + (3 y ) 2 = a 2 + b 2
3ax2 + 5xy + (a2 – 2)y2 = 0
are perpendicular to each other for [2002] (d) (3x + 1) 2 + (3 y ) 2 = a 2 + b 2 .
(a) two values of a (b) " a
8. If x1, x2 , x3 and y1, y2 , y3 are both in G.P. with
(c) for one value of a (d) for no values of a
5. A square of side a lies above the x-axis and has the same common ratio, then the points
one vertex at the origin. The side passing through ( x1, y1 ), ( x 2 , y 2 ) and ( x3 , y3 ) [2003]
æ pö (a) are vertices of a triangle
the origin makes an angle aç 0 < a < ÷ with the (b) lie on a straight line
è 4ø
(c) lie on an ellipse
positive direction of x-axis. The equation of its
(d) lie on a circle.
diagonal not passing through the origin is
[2003]
www.crackjee.xyz
Straight Lines & Pair of Straight Lines M-45

9. If the equation of the locus of a point 13. If one of the lines given by

equidistant from the point (a1, b1 ) and 6 x 2 - xy + 4cy 2 = 0 is 3x + 4y = 0, then c


equals [2004]
(a2, b2 ) is (a1 - b2 ) x + (a1 - b2 ) y + c = 0 , (a) –3 (b) 1
then the value of `c` is [2003] (c) 3 (d) 1
14. The line parallel to the x- axis and passing through
(a) a12 + b12 - a 2 2 - b2 2
the intersection of the lines ax + 2by + 3b = 0 and
1 2 bx – 2ay – 3a = 0, where (a, b) ¹ (0, 0) is
(b) a2 + b2 2 - a12 - b12
2 [2005]

(c) a12 - a 2 2 + b12 - b2 2 3


(a) below the x - axis at a distance of from it
2
1 2
(d) ( a1 + a2 2 + b12 + b2 2 ) .
2 2
(b) below the x - axis at a distance of from it
10. Let A(2, - 3) and B ( -2, 3) be vertices of a 3

triangle ABC. If the centroid of this triangle 3


(c) above the x - axis at a distance of from it
moves on the line 2 x + 3 y = 1, then the locus 2
of the vertex C is the line [2004]
2
(d) above the x - axis at a distance of from it
(a) 3x - 2 y = 3 (b) 2 x - 3 y = 7 3
(c) 3x + 2 y = 5 (d) 2 x + 3 y = 9 15. If a vertex of a triangle is (1, 1) and the mid points
11. The equation of the straight line passing of two sides through this vertex are (–1, 2) and
through the point (4, 3) and making intercepts (3, 2) then the centroid of the triangle is
on the co-ordinate axes whose sum is –1 is [2005]
[2004]
æ 7ö æ -1 7 ö
(a) ç - 1, ÷ (b) ç , ÷
x y x y è 3ø è 3 3ø
(a) - = 1 and + =1
2 3 -2 1
æ 7ö
(d) æç , ö÷
1 7
x y x y (c) ç1, ÷
(b) - = -1 and + = -1 è 3ø è3 3ø
2 3 -2 1
16. A straight line through the point A (3, 4) is such
x y x y that its intercept between the axes is bisected
(c) + = 1 and + = 1
2 3 2 1 at A. Its equation is [2006]

x y x y (a) x + y = 7 (b) 3 x - 4 y + 7 = 0
(d) + = -1 and + = -1
2 3 -2 1 (c) 4 x + 3 y = 24 (d) 3 x + 4 y = 25
12. If the sum of the slopes of the lines given by
17. If (a, a 2 ) falls inside the angle made by the
x 2 - 2cxy - 7 y 2 = 0 is four times their product
x
c has the value [2004] lines y = , x > 0 and y = 3x , x > 0 , then a
2
(a) –2 (b) –1
(c) 2 (d) 1 belong to [2006]
EBD_7764
www.crackjee.xyz
M-46 Mathematics
23. The lines p(p2 +1)x – y + q = 0 and
(a) æç 0, 1 ö÷ (b) (3, ¥) (p2 + 1)2x + (p2 + 1)y + 2q = 0 are perpendicular
è 2ø to a common line for : [2009]
(a) exactly one values of p
æ1 ö æ 1ö
(c) ç , 3 ÷ (d) ç - 3, - ÷ (b) exactly two values of p
è 2 ø è 2ø
(c) more than two values of p
18. Let A (h, k), B(1, 1) and C (2, 1) be the vertices (d) no value of p
of a right angled triangle with AC as its 24. Three distinct points A, B and C are given in
hypotenuse. If the area of the triangle is 1square the 2-dimensional coordinates plane such that
unit, then the set of values which 'k' can take is the ratio of the distance of any one of them
given by [2007] from the point (1, 0) to the distance from the
(a) {–1, 3} (b) {–3, –2} 1
(c) {1, 3} (d) {0, 2} point (–1, 0) is equal to . Then the
3
19. Let P = (–1, 0), Q = (0, 0) and R = (3, 3 3 ) be circumcentre of the triangle ABC is at the point:
three point. The equation of the bisector of the [2009]
angle PQR is [2007] æ5 ö æ5 ö
(a) çè , 0÷ø (b) çè , 0÷ø
3 4 2
(a) x+ y =0 (b) x + 3 y = 0
2
æ5 ö
(c) çè , 0÷ø (d) (0, 0)
3 3
(c) 3x + y = 0 (d) x + y = 0.
2 25. The lines L1 : y – x = 0 and L2 : 2x + y = 0
20. If one of the lines of my2 + (1– m2) xy – mx2= 0 intersect the line L3 : y + 2 = 0 at P and Q
is a bisector of the angle between the lines xy = respectively. The bisector of the acute angle
0, then m is [2007] between L1 and L2 intersects L3 at R. [2011]
(a) 1 (b) 2 Statement-1: The ratio PR : RQ equals
(c) –1/2 (d) –2. 2 2: 5
21. The perpendicular bisector of the line segment Statement-2: In any triangle, bisector of an
j o i n i n g angle divides the triangle into two similar
P (1, 4) and Q(k, 3) has y-intercept –4. Then a triangles.
possible value of k is [2008] (a) Statement-1 is true, Statement-2 is true;
(a) 1 (b) 2 Statement-2 is not a correct explanation for
(c) –2 (d) – 4 Statement-1.
22. The shortest distance between the line y – x = (b) Statement-1 is true, Statement-2 is false.
1 and the curve x = y2 is : [2009] (c) Statement-1 is false, Statement-2 is true.
2 3 3 2 (d) Statement-1 is true, Statement-2 is true;
(a) (b) Statement-2 is a correct explanation for
8 5
Statement-1.
3 3 2 26. The lines x + y = a and ax – y = 1 intersect
(c) (d)
4 8 each other in the first quadrant. Then the set of
all possible values of a in the interval :
www.crackjee.xyz
Straight Lines & Pair of Straight Lines M-47
[2011RS] the reflected ray is [2013]
(a) ( 0, ¥ ) (b) [1, ¥) (a) y = x + 3 (b) 3y = x – 3

(c) ( -1, ¥) (d) ( -1,1) (c) y = 3x - 3 (d) 3y = x -1


27. If A (2, – 3) and B (– 2, 1) are two vertices of a 31. The x-coordinate of the incentre of the triangle
triangle and third vertex moves on the line that has the coordinates of mid points of its
2 x + 3 y = 9, then the locus of the centroid of sides as (0, 1) (1, 1) and (1, 0) is : [2013]
the triangle is : [2011RS] (a) 2 + 2 (b) 2 - 2
(a) x - y = 1 (b) 2 x + 3 y = 1 (c) 1 + 2 (d) 1 - 2
(c) 2 x + 3 y = 3 (d) 2 x - 3 y = 1
28. If the line 2x + y = k passes through the point 32. Let PS be the median of the triangle vertices
which divides the line segment joining the P(2, 2), Q(6, –1) and R(7, 3). The equation of
points (1,1) and (2,4) in the ratio 3 :2, then k the line passing through (1, –1) and parallel to
equals : [2012] PS is: [2014]
(a) 4x + 7y + 3 = 0 (b) 2x – 9y – 11 = 0
29 (c) 4x – 7y – 11 = 0 (d) 2x + 9y + 7 = 0
(a) (b) 5
5 33. Let a, b, c and d be non-zero numbers. If the
11 point of intersection of the lines 4ax + 2ay + c =
(c) 6 (d)
5 0 and 5bx + 2by + d =0 lies in the fourth quadrant
29. A line is drawn through the point (1,2) to meet and is equidistant from the two axes then
the coordinate axes at P and Q such that it [2014]
forms a triangle OPQ, where O is the origin. If (a) 3bc – 2ad = 0 (b) 3bc + 2ad = 0
the area of the triangle OPQ is least, then the (c) 2bc – 3ad = 0 (d) 2bc + 3ad = 0
slope of the line PQ is : [2012] 34. Two sides of a rhombus are along the lines,
x – y + 1 = 0 and 7x – y – 5 = 0. If its diagonals
1
(a) - (b) – 4 intersect at (–1, –2), then which one of the
4
following is a vertex of this rhombus?
1 [2016]
(c) – 2 (d) -
2
æ 1 -8 ö æ -10 -7 ö
30. A ray of light along x + 3 y = 3 gets (a) ç , ÷ (b) ç , ÷
è3 3 ø è 3 3 ø
reflected upon reaching x-axis, the equation of (c) (–3, –9) (d) (– 3, – 8)

A n sw er K ey
1 2 3 4 5 6 7 8 9 10 11 12 13 14 15
(a) (d) (a) (a) (a) (a) (c) (b) (b) (d) (a) (c) (a) (a) (c)
16 17 18 19 20 21 22 23 24 25 26 27 28 29 30
(c) (c) (a) (c) (a) (d) (d) (a) (a) (b) (b ) (b) (c) (c) (b)
31 32 33 34
(b) (d) (a) (a)
EBD_7764
www.crackjee.xyz
M-48 Mathematics

For unique point of intersection f 2 – bc = 0


1. (a) AB = ( 4 + 1) 2 + (0 + 1) 2 = 26 ;
Þ af 2 – abc = 0.
BC = (3 + 1) 2 + (5 + 1) 2 = 52 Since abc + 2fgh – af 2 –bg2 – ch2 = 0
Þ 2fgh – bg2 – ch2 = 0
CA = ( 4 - 3) 2 + (0 - 5) 2 = 26 ; 4. (a) 3a + a2 – 2 = 0 Þ a2 + 3a – 2 = 0.;
In isosceles triangle side AB = CA
- 3 ± 9 + 8 - 3 ± 17
For right angled triangle, BC2 = AB2 + AC2 Þa = =
2 2
So, here BC = 52 or BC2 = 52 5. (a) Co-ordinates of A = (a cos a , a sin a )
or ( 26 )2 + ( 26 )2 = 52 Equation of OB,
So, the given triangle is right angled and also B
Y
isosceles
2. (d) Equation of AB is C
x cos a + y sin a = p;
p A
Y
4
a
O
B X

M (x1, y1) æp ö
y = tan ç + a ÷ x
è4 ø

O A
X CA ^ r to OB
æp ö
x cos a y sin a \ slope of CA = - cot ç + a ÷
Þ + = 1; è4 ø
p p Equation of CA
x y æp ö
Þ + =1 y - a sin a = - cot ç + a ÷( x - a cos a )
p / cos a p / sin a è4 ø
So co-ordinates of A and B are æ æp öö
Þ (y - a sin a) ç tan ç + a ÷ ÷ = (a cos a - x)
æ p ö æ p ö è è4 øø
çè , 0÷ and ç 0, ;
cos a ø è sin a ÷ø
æ p ö
So coordinates of midpoint of AB are tan + tan a
ç 4 ÷
æ p p ö Þ (y - a sin a ) ç ÷ (a cos a - x)
p
çè , ÷ = ( x1 , y1 )(let ) ; ç 1 - tan tan a ÷
2 cos a 2sin a ø è 4 ø
p p Þ (y - a sin a ) (1 + tan a )
x1 = & y1 = ;
2 cos a 2 sin a = (a cos a - x)(1 - tan a)
Þ cos a = p/2x1 and sin a = p/2y1 ; Þ (y - a sin a ) (cos a +=sin
(aacos
) a - x)(cos a - sin a )
cos2 a + sin2 a = 1
Þ y(cos + sin a ) - a sin a cos a - a sin 2 a
1 1 4
Locus of (x1, y1) is + = . 2
x2 y 2 p 2 = a cos a - a cos a sin a - x(cos a - sin a)
3. (a) Put x = 0 in the given equation Þ y(cos a + sin a ) + x(cos a - sin a ) = a
Þ by2 + 2 fy + c = 0. y (sin a + cos a ) + x(cos a - sin a ) = a.
www.crackjee.xyz
Straight Lines & Pair of Straight Lines M-49
6. (a) Equation of bisectors of second pair of æ h -2 + k ö
straight lines is, or çè , ÷ . It lies on 2x + 3y = 1
3 3 ø
qx 2 + 2 xy - qy 2 = 0 ....(1) 2h
It must be identical to the first pair Þ - 2 + k = 1 Þ 2h + 3k = 9
3
x 2 - 2 pxy - y 2 = 0 ....(2) Þ Locus of C is 2x + 3y = 9
from (1) and (2) x y
11. (a) Let the required line be + = 1 ....(1)
q 2 -q a b
= = Þ pq = -1 . then a + b = –1 ....(2)
1 - 2 p -1
4 3
a cos t + b sin t + 1 (1) passes through (4, 3) , Þ + =1
7. (c) x= a b
3
Þ a cos t + b sin t = 3 x - 1 Þ 4b + 3a = ab ....(3)
a sin t - b cos t Eliminating b from (2) and (3), we get
y=
3 a 2 - 4 = 0 Þ a = ±2 Þ b = -3 or 1
Þ a sin t - b cos t = 3 y \ Equations of straight lines are
Squaring and adding, x y x y
+ = 1 or + =1
(3 x - 1) 2 + (3 y ) 2 = a 2 + b 2 2 -3 -2 1
8. (b) Taking co-ordinates as 12. (c) Let the lines be y = m1x and y = m2x then
æx yö 2c 1
ç , ÷; ( x, y ) & ( xr , yr ) . m1 + m2 = - and m1m2 = -
èr rø 7 7
Then slope of line joining Given m1 + m2 = 4 m1m2

æ 1ö 2c 4
y ç1 - ÷ Þ =- Þc=2
æ x yö è rø y 7 7
çè , ÷ø , ( x, y ) = =
13. (a) 3 x + 4 y = 0 is one of the lines of the pair
r r æ 1ö x
x ç1 - ÷
è rø 3
6 x 2 - xy + 4cy 2 = 0 , Put y = - x ,
and slope of line joining (x, y) and (xr, yr) 4
y ( r - 1) y 3 2 æ 3 ö
2
= = we get 6 x 2 + x + 4c ç - x ÷ = 0
x ( r - 1) x 4 è 4 ø
\ m1 = m2 3 9c
Þ Points lie on the straight line. Þ 6+ + = 0 Þ c = -3
4 4
9. (b) ( x - a1 ) 2 + ( y - b1 ) 2 14. (a) The line passing through the intersection
2
= ( x - a2 ) + ( y - b2 ) 2 of lines ax + 2by = 3b = 0 and
(a1 - a2 ) x + (b1 - b2 ) y bx - 2ay - 3a = 0 is
1 ax + 2by + 3b + l (bx – 2ay – 3a) = 0
+ ( a22 + b22 - a12 - b12 ) = 0
2 Þ (a + b l ) x + (2b – 2a l )y + 3b – 3 l a = 0
1 2 As this line is parallel to x-axis.
c= ( a2 + b2 2 - a12 - b12 )
2 \ a + b l = 0 Þ l = – a/b
10. (d) Let the vertex C be (h, k), then the a
centroid of Þ ax + 2by + 3b – (bx – 2ay – 3a) = 0
b
æ 2 - 2 + h -3 + 1 + k ö 2a 2 3a 2
DABC is ç , ÷ø Þ ax + 2by + 3b – ax + y+ =0
è 3 3 b b
EBD_7764
www.crackjee.xyz
M-50 Mathematics
æ 17. (c) Clearly for point P,
2a 2 ö 3a 2
y ç 2b + ÷ + 3b + =0
b ø b y
è y = 3x

æ 2b 2 + 2a 2 ö æ 3b2 + 3a 2 ö 2
yç ÷ = -ç ÷ • P(a, a )
è b ø è b ø x
y=
2 2 2
-3(a + b ) -3
y= = O x
2 2 2
2(b + a )
So it is 3/2 units below x-axis.
15. (c) Vertex of triangle is (1, 1) and midpoint of a 1
sides through this vertex is (– 1, 2) and (3, 2) a 2 - 3a < 0 and a 2 - > 0 Þ <a<3
22
18. (a) Given : The vertices of a right angled
A (1, 1) triangle A(l, k), B(1, 1) and C(2, 1) and Area
of DABC = 1 square unit

Y
(-1, 2) (3, 2) A (1, k)

B C
C (2, 1)
B (1, 1)
Þ vertex B and C come out to be (– 3, 3) O X
and (5, 3)
1- 3+ 5 1+ 3+ 5 We know that, area of right angled triangle
\ Centroid is , 1
3 3 1
= × BC × AB = 1 = (1) | (k – 1)|
2 2
æ 7ö
Þ ç1, ÷ Þ ± (k - 1) = 2 Þ k = – 1, 3
è 3ø
19. (c) Given : The coordinates of points P, Q, R
16. (c) y
are (–1, 0), (0, 0), (3,3 3) respectively..
P(0, b)
Y R (3, 3 3 )
A(3, 4)
M
Q(a, 0)
x
O

2p / 3 p/3
X' X
Q A is the mid point of PQ , therefore P (-1, 0) Q (0, 0)
a+0 0+b
= 3, = 4 Þ a = 6, b = 8 Y'
2 2
x y
\ Equation of line is + = 1 y2 - y1 3 3
6 8 Slope of QR = =
or 4x + 3y = 24 x2 - x1 3
p
Þ tan q = 3 Þ q =
3
www.crackjee.xyz
Straight Lines & Pair of Straight Lines M-51

Þ ÐRQX = p
a2 - a + 1 1 é 1 2 3ù
3 D= = êë (a - 2 ) + 4 úû
p 2p 2 2
\ ÐRQP = p - =
3 3 1
Let QM bisects the ÐPQR , It is min when a = and Dmin
2
2p 3 3 2
\ Slope of the line QM = tan =– 3 = =
3 4 2 8
\ Equation of line QM is (y – 0) = – 3 (x – 0) 23. (a) If the lines p (p2 + 1) x – y + q = 0
and (p2 + 1)2 x + (p2 + 1) y +2q = 0
Þ y= – 3 x Þ 3 x + y= 0 are perpendicular to a common line then
20. (a) Equation of bisectors of lines, xy = 0 are y = these lines must be parallel to each other,
±x p ( p 2 + 1) ( p 2 + 1) 2
\ m1 = m2 Þ - =-
y -1 p2 + 1
Þ (p2 + 1) ( p + 1 ) = 0
y=x Þ p=–1
y = -x
\ p can have exactly one value.
24. (a) Given that P (1, 0), Q (– 1, 0)
x
(0, 0) AP BP CP 1
and = = =
AQ BQ CQ 3
\ Put y = ± x in the given equation Þ 3AP = AQ
my2 + (1 – m2)xy – mx2 = 0
Let A = (x, y) then
\ mx2 + (1 – m2)x2 – mx2 = 0
3AP = AQ Þ 9 AP2= AQ2
Þ 1 – m2 = 0 Þ m = ± 1
Þ 9 (x – 1)2 + 9y2 = (x + 1)2 + y2
3–4 –1 Þ 9 x2 – 18x + 9 + 9y2 = x2 +2x +1 + y2
21. (d) Slope of PQ = =
k –1 k –1 Þ 8x2 – 20x + 8y2 + 8 = 0
\ Slope of perpendicular bisector of 5
PQ = ( k –1) Þ x2 + y2 – x +1 = 0 ....(1)
3
æ k +1 7ö \ A lies on the circle given by eq (1). As B
Also mid point of PQ ç , .
è 2 2 ÷ø and C also follow the same condition, they
\ Equation of perpendicular bisector is must lie on the same circle.
\ Centre of circumcircle of D ABC
7 æ k + 1ö
y – = (k –1) ç x – ÷ æ5 ö
2 è 2 ø = Centre of circle given by (1) = ç , 0÷
è4 ø
Þ 2y – 7 = 2(k –1) x –(k2 –1)
Þ 2(k – 1)x – 2y + ( 8 – k2) = 0
L3
2 L1
8–k 25. (b)
\ y-intercept = – = –4
–2 0
x= P(–2, –2)
Þ 8 – k2 = –8 or k2 = 16 Þ k = ± 4 y–

22. 2
(d) Let (a , a) be the point of shortest distance R(–1, –2)
on x = y2 O
Then distance between (a2, a) and line (0, 0) 2x
+y Q
x – y + 1 = 0 is given by =0

L2
EBD_7764
www.crackjee.xyz
M-52 Mathematics
L1 : y – x = 0
L2 : 2x + y = 0 æ a 2 + 1ö a2 + 1
Þ -ç ÷ > 0 Þ <0
L3 : y + 2 = 0 è a +1 ø a +1
On solving the equation of line L1 and L2
Since a2+ 1 > 0
we get their point of intersection (0, 0) i.e.,
origin O. \ a+1<0
On solving the equation of line L1 and L3, Þ a < –1 .... (6)
we get P = (– 2, – 2).
–1
Similarly, we get Q = (– 1, – 2) From (5) and (6), a Î f
We know that bisector of an angle of a
Hence Case-II is not possible.
triangle, divide the opposite side the
triangle in the ratio of the sides including So, correct answer is a Î[1, ¥)
the angle [Angle Bisector Theorem of a
Triangle] 27. (b) A(2, –3)

PR OP (-2)2 + (-2) 2 2 2
\ = = =
RQ OQ 2
(-1) + (-2) 2 5

26. (b) x+ y = a G
and ax - y = 1 (h, k)
Case I : If a > 0
x+ y = a .... (1) B(–2, 1) C (a, b)
ax - y = 1 .... (2)
On adding equation (1) and (2), we get a = 3h
b - 2 = 3k
x (1 + a ) = 1 + a Þ x = 1
y=a–1 b = 3k + 2
It is in first quadrant
Third vertex (α, β ) lies on the line
so a – 1 ³ 0
2x + 3 y = 9
Þ a ³1
2a + 3b = 9
Þ a Î[1, ¥)
Case II : If a < 0 2 ( 3h) + 3 ( 3k + 2) = 9
x + y = -a .... (3)
2h + 3k = 1
ax - y = 1 .... (4) 2x + 3 y =1
On adding equation (3) and (4), we get 28. (c) Let the joining points be A (1,1) and B (2,4).
x (1 + a) = 1 - a Let point C divides line AB in the ratio 3 :
2.
1- a a -1 So, by section formula we have
x= >0Þ <0
1+ a a +1
æ 3´ 2 + 2´1 3 ´ 4 + 2´1 ö
Since a – 1 < 0 C=ç , ÷
\ a+1>0 è 3+2 3+2 ø
Þ a > –1 .... (5) æ 8 14 ö
=ç , ÷
–1 è5 5 ø
Since Line 2x + y = k passes through
1- a -a - a2 - 1 + a
y = -a - >0 = >0
1+ a 1+ a æ 8 14 ö
Cç , ÷
è5 5 ø
www.crackjee.xyz
Straight Lines & Pair of Straight Lines M-53
\ C satisfies the equation 2x + y = k. m 2
2 + 8 14 Let Area = f (m) = 2 - -
Þ + =k Þ k=6 2 m
5 5 -1 2
29. (c) Equation of a line passing through (x1,y1) Now, f ' ( m ) = +
having slope m is given by y – y1 = m (x – 2 m2
x1 ) Put f ¢ (m) = 0
Since the line PQ is passing through (1,2) Þ m2 = 4 Þ m = ± 2
therefore its equation is (y – 2) = m (x – 1) -4
Now, f '' ( m ) =
where m is the slope of the line PQ. m3
Now, point P (x,0) will also satisfy the
1
equation of PQ f '' ( m ) m = 2 = - < 0
\ y –2 = m (x –1) Þ 0 – 2 = m (x – 1) 2
-2 1
f '' ( m ) m = - 2 = > 0
Þ – 2 = m (x – 1) Þ x – 1 = 2
m
Area will be least at m = –2
-2 Hence, slope of PQ is –2.
Þ x= +1
m
30. (b) Suppose B(0, 1) be any point on given line
Also, OP = ( x - 0 ) 2 + ( 0 - 0 )2 = x
and co-ordinate of A is ( 3, 0). So,
-2
= +1 equation of
m
Similarly, point Q (0,y) will satisfy equation (0, –1)
of PQ B
\ y –2 = m (x– 1)
Þ y – 2 = m (–1)
Þ y = 2 – m and OQ = y = 2 – m
1 A 3, 0
Area of DPOQ = ( OP )( OQ )
2
1æ 2ö
= ç 1 - ÷ ( 2 - m) B' (0, –1)
2è mø
-1 - 0 y -0
1 Reflected Ray is =
(Q Area of D = ´ base ´ height ) 0- 3 x- 3
2
1é 4 ù
Þ 3y = x - 3
= 2 - m - + 2ú
2 êë m û 31. (b) From the figure, we have

1é æ 4 öù a = 2, b = 2 2, c = 2
= 4 - ç m + ÷ú
2 êë è m øû x1 = 0, x2 = 0, x3 = 2

m 2
=2- -
2 m
Q

(1,2)

P
O
EBD_7764
w w w . c r a c k j
M-54 Mathematics
Now, x-co-ordinate of incentre is given as The point of intersection will be
ax1 + bx2 + cx3 x -y 1
= =
a+b+c 2 ad - 2bc 4ad - 5bc 8ab - 10ab
Þ x-coordinate of incentre 2(ad - bc) bc - ad
Þ x= =
2 ´ 0 + 2 2.0 + 2.2 -2ab ab
=
2+ 2+ 2 2 5bc - 4ad 4 ad - 5bc
Þ y= =
2 -2ab 2 ab
= = 2- 2 Q Point of intersection is in fourth quadrant
2+ 2 so x is positive and y is negative.
Also distance from axes is same
32. (d) Let P, Q, R, be the vertices of DPQR So x = – y (Q distance from x-axis is –y as
y is negative)
P (2, 2)
bc - ad 5bc - 4ad
= Þ 3bc – 2ad = 0
ab 2ab

34. (a) D x – y +1= 0 C


Q (6, – 1) S R (7, 3)
0

5=0
+m=
Since PS is the median
S is mid-point of QR O (–1,–2)


7x – y

7x – y
æ 7 + 6 3 - 1ö æ 13 ö
So, S = ç , ÷ = ç ,1÷
è 2 2 ø è2 ø
A x–y +l =0 B
2 -1 2
Now, slope of PS = =- Let other two sides of rhombus are
13 9
2-
2 x–y+l=0
Since, required line is parallel to PS and 7x –y + m = 0
therefore then O is equidistant from AB and DC and
from AD and BC
slope of required line = slope of PS
Now, eqn of line passing through (1, –1) \ -1 + 2 + 1 = -1 + 2 +l Þl= – 3
2
and having slope - is and -7 + 2 - 5 = -7 + 2 +m Þm= 15
9
\ Other two sides are x – y – 3 = 0 and 7x – y +
2
y - (-1) = - ( x - 1) 15 = 0
9 On solving the eqns of sides pairwise, we get
9y + 9 = –2x + 2 Þ 2x + 9y + 7 = 0 the vertices as
33. (a) Given lines are æ 1 -8 ö æ -7 -4 ö
4ax + 2ay + c = 0 ç , ÷
ç ÷ , (1, 2), ç
ç , ÷ ÷ , (-3, -6)
è3 3 ø è3 3ø
5bx + 2by + d = 0
www.crackjee.xyz

Conic Sections 11
1. If the chord y = mx + 1 of the circle x2+y2=1
6. If the two circles ( x - 1) 2 + ( y - 3) 2 = r 2 and
subtends an angle of measure 45° at the major
segment of the circle then value of m is x 2 + y 2 - 8 x + 2 y + 8 = 0 intersect in two
[2002]
distinct point, then [2003]
(a) 2 ± 2 (b) –2 ± 2 (a) r > 2 (b) 2 < r < 8
(c) r < 2 (d) r = 2.
(c) –1 ± 2 (d) none of these
7. The lines 2 x - 3 y = 5 and 3x - 4 y = 7 are
2. The centres of a set of circles, each of radius 3,
diameters of a circle having area as 154
lie on the circle x2+y2=25. The locus of any point
sq.units.Then the equation of the circle is
in the set is [2002]
[2003]
(a) 4 £ x2 + y2 £ 64 (b) x2 + y2 £ 25
(a) x 2 + y 2 - 2 x + 2 y = 62

(c) x2 + y2 ³ 25 (d) 3 £ x2+ y2 £ 9 (b) x 2 + y 2 + 2 x - 2 y = 62


3. The centre of the circle passing through (0, 0)
and (1, 0) and touching the circle x2 + y2 = 9 is (c) x 2 + y 2 + 2 x - 2 y = 47
[2002] (d) x 2 + y 2 - 2 x + 2 y = 47 .
æ1 1ö æ1 ö 8. The normal at the point (bt12 , 2bt1 ) on a
(a) ç , ÷ (b) ç ,- 2 ÷
è2 2ø è 2 ø
parabola meets the parabola again in the point
æ3 1ö æ1 3ö (bt 2 2 , 2bt 2 ) , then [2003]
(c) ç , ÷ (d) ç , ÷
è2 2ø è2 2ø 2 2
(a) t 2 = t1 + (b) t 2 = -t1 -
4. The equation of a circle with origin as a centre t1 t1
and passing through equilateral triangle whose 2 2
(c) t2 = -t1 + (d) t 2 = t1 -
median is of length 3a is [2002] t1 t1
(a) x2 + y2 = 9a2 (b) x2 + y2 = 16a2
x2 y 2
(c) x2 + y2 = 4a2 (d) x2 + y2 = a2 9. The foci of the ellipse + = 1 and the
16 b2
5. Two common tangents to the circle x2 + y2 = 2a2
and parabola y2 = 8ax are [2002] x2 y 2 1
hyperbola - = coincide. Then the
(a) x = ± ( y + 2 a ) 144 81 25

(b) y = ± ( x + 2a) value of b 2 is [2003]


(a) 9 (b) 1
(c) x = ± ( y + a)
(d) y = ± ( x + a) (c) 5 (d) 7
EBD_7764
w w w . c r a c k j e e . x y z
M-56 Mathematics
10. If a circle passes through the point (a, b) and 15. The eccentricity of an ellipse, with its centre at
cuts the circle x 2 + y 2 = 4 orthogonally, then 1
the origin, is . If one of the directrices is x = 4,
2
the locus of its centre is [2004]
then the equation of the ellipse is: [2004]
(a) 2ax - 2by - (a 2 + b 2 + 4) = 0 (a) 4 x2 + 3 y 2 = 1
(b) 2ax + 2by - (a 2 + b 2 + 4) = 0 (b) 3 x 2 + 4 y 2 = 12
(c) 4 x 2 + 3 y 2 = 12
(c) 2ax - 2by + (a 2 + b 2 + 4) = 0
(d) 3x 2 + 4 y 2 = 1
(d) 2ax + 2by + (a 2 + b 2 + 4) = 0
11. A variable circle passes through the fixed point 16. If the circles x 2 + y 2 + 2ax + cy + a = 0 and
A( p, q ) and touches x-axis . The locus of the other x 2 + y 2 – 3ax + dy – 1 = 0 intersect in two
end of the diameter through A is [2004] distinct points P and Q then the line 5x + by – a
(a) ( y - q)2 = 4 px (b) ( x - q)2 = 4 py = 0 passes through P and Q for [2005]
(a) exactly one value of a
(c) ( y - p)2 = 4qx (d) ( x - p)2 = 4qy (b) no value of a
(c) infinitely many values of a
12. If the lines 2 x + 3 y + 1 = 0 and 3x - y - 4 = 0 (d) exactly two values of a
lie along diameter of a circle of circumference 17. A circle touches the x- axis and also touches the
10p, then the equation of the circle is [2004] circle with centre at (0,3 ) and radius 2. The locus
of the centre of the circle is [2005]
(a) x 2 + y 2 + 2 x - 2 y - 23 = 0
(a) an ellipse (b) a circle
(b) x 2 + y 2 - 2 x - 2 y - 23 = 0 (c) a hyperbola (d) a parabola
18. If a circle passes through the point (a, b) and
(c) x 2 + y 2 + 2 x + 2 y - 23 = 0 cuts the circle x 2 + y 2 = p 2 orthogonally, then
the equation of the locus of its centre is
(d) x 2 + y 2 - 2 x + 2 y - 23 = 0
[2005]
13. Intercept on the line y = x by the circle
(a) x 2 + y 2 – 3ax – 4by + ( a 2 + b 2 - p 2 ) = 0
x 2 + y 2 - 2 x = 0 is AB. Equation of the circle
on AB as a diameter is [2004] (b) 2ax + 2by – ( a 2 - b 2 + p 2 ) = 0
(a) x 2 + y 2 + x - y = 0 (c) x 2 + y 2 – 2ax – 3by + ( a2 - b2 - p 2 ) = 0
(b) x 2 + y 2 - x + y = 0 2 2 2
(d) 2ax + 2by – ( a + b + p ) = 0
(c) x 2 + y 2 + x + y = 0 19. If the pair of lines ax 2 + 2 (a + b)xy + by 2 = 0 lie
(d) x 2 + y 2 - x - y = 0 along diameters of a circle and divide the circle
into four sectors such that the area of one of the
14. If a ¹ 0 and the line 2bx + 3cy + 4d = 0 passes sectors is thrice the area of another sector then
through the points of intersection of the parabolas [2005]
y 2 = 4ax and x 2 = 4ay, then [2004] (a) 3a 2 - 10 ab + 3b 2 = 0
(a) d 2 + (3b - 2c) 2 = 0 (b) 3a 2 - 2ab + 3b 2 = 0
2 2
(b) d + (3b + 2c) = 0 (c) 3a 2 + 10 ab + 3b 2 = 0
(c) d 2 + (2b - 3c) 2 = 0
(d) 3a 2 + 2ab + 3b 2 = 0
(d) d 2 + (2b + 3c) 2 = 0
www.crackjee.xyz
Conic Sections M-57
20. Let P be the point ( 1, 0 ) and Q a point on the 105 3
(a) xy = (b) xy =
2 4
locus y = 8 x . The locus of mid point of PQ is 64
35
[2005] (c) xy = (d) xy = 64
(a) y 2 – 4x + 2 = 0 (b) y 2 + 4x + 2 = 0 16 105
26. In an ellipse, the distance between its foci is 6
2 2 and minor axis is 8. Then its eccentricity is
(c) x + 4y + 2 = 0 (d) x – 4y + 2 = 0
[2006]
21. The locus of a point P (a, b) moving under the
(a) 3 (b) 1
condition that the line y = ax + b is a tangent to 5 2
x2 y2 (c) 4 (d) 1
the hyperbola - = 1 is [2005]
5
a2 b2 5
(a) an ellipse (b) a circle 27. Consider a family of circles which are passing
(c) a parabola (d) a hyperbola through the point (– 1, 1) and are tangent to x-
22. An ellipse has OB as semi minor axis, F and F ' axis. If (h, k) are the coordinate of the centre of
the circles, then the set of values of k is given
its focii and the angle FBF ' is a right angle. by the interval [2007]
Then the eccentricity of the ellipse is
[2005] 1 1 1
(a) - £k£ (b) k£
1 1 2 2 2
(a) (b)
2 2 1 1
(c) 0£ k £ (d) k³
1 1 2 2
(c) (d)
4 3 x2 y2
28. For the Hyperbola - = 1 , which of
23. If the lines 3 x - 4 y - 7 = 0 and 2 x - 3 y - 5 = 0 cos 2 a sin 2 a
are two diameters of a circle of area 49p square the following remains constant when a varies = ?
units, the equation of the circle is [2006] [2007]
(a) abscissae of vertices
(a) x 2 + y 2 + 2 x - 2 y - 47 = 0 (b) abscissae of foci
(c) eccentricity (d) directrix.
(b) x 2 + y 2 + 2 x - 2 y - 62 = 0 29. The equation of a tangent to the parabola
y2 = 8x is y = x + 2. The point on this line from
(c) x 2 + y 2 - 2 x + 2 y - 62 = 0 which the other tangent to the parabola is
perpendicular to the given tangent is [2007]
(d) x 2 + y 2 - 2 x + 2 y - 47 = 0 (a) (2, 4) (b) (–2, 0)
(c) (–1, 1) (d) (0, 2)
24. Let C be the circle with centre (0, 0) and radius 3
30. The point diametrically opposite to the point
units. The equation of the locus of the mid points
P(1, 0) on the circle x2 + y2 + 2x + 4y – 3 = 0 is
of the chords of the circle C that subtend an [2008]
angle of 2p at its center is [2006] (a) (3, – 4) (b) (–3, 4)
3 (c) (–3, –4) (d) (3, 4)
31. A focus of an ellipse is at the origin. The directrix
(a) x 2 + y 2 = 3 (b) x 2 + y 2 = 1
2 1
27 9 is the line x = 4 and the eccentricity is . Then
2
(c) x + y =2 (d) x 2 + y 2 = 2
4 4 the length of the semi-major axis is [2008]
25. The locus of the vertices of the family of
8 2 4 5
3 2 2 (a) (b) (c) (d)
parabolas y = a x + a x - 2a is [2006] 3 3 3 3
3 2
EBD_7764
www.crackjee.xyz
M-58 Mathematics
32. A parabola has the origin as its focus and the 39. Equation of the ellipse whose axes are the axes
line x = 2 as the directrix. Then the vertex of the of coordinates and which passes through the
parabola is at [2008] 2
(a) (0, 2) (b) (1, 0) point (–3, 1) and has eccentricity is [2011]
5
(c) (0, 1) (d) (2, 0)
(a) 5x2 + 3y2 – 48 = 0 (b) 3x2 + 5y2 – 15 = 0
33. If P and Q are the points of intersection of the
(c) 5x2 + 3y2 – 32 = 0 (d) 3x2 + 5y2 – 32 = 0
circles x 2 + y2 + 3 x + 7 y + 2 p - 5 = 0 and 40. The equation of the circle passing through the
point (1, 0) and (0, 1) and having the smallest
x2 + y2 + 2x + 2y – p2=0 then there is a circle
radius is [2011 RS]
passing through P, Q and (1, 1) for: [2009]
(a) all except one value of p (a) x2 + y 2 - 2 x - 2 y + 1 = 0
(b) all except two values of p (b) x2 + y2 – x – y = 0
(c) exactly one value of p (c) x2 + y2 + 2x + 2y – 7= 0
(d) all values of p (d) x2 + y2 + x + y – 2 = 0
41. The equation of the hyperbola whose foci are
34. The ellipse x 2 + 4 y 2 = 4 is inscribed in a
(– 2, 0) and (2, 0) and eccentricity is 2 is given
rectangle aligned with the coordinate axes, which by : [2011RS]
in turn is inscribed in another ellipse that passes (a) x2 – 3y2 = 3 (b) 3x2 – y2 = 3
through the point (4, 0). Then the equation of (c) – x2 + 3y2 = 3 (d) – 3x2 + y2 = 3
the ellipse is : [2009] 42. The length of the diameter of the circle which
touches the x-axis at the point (1,0) and passes
(a) x 2 + 12 y 2 = 16 (b) 4 x 2 + 48 y 2 = 48 through the point (2,3) is: [2012]
(c) 4 x 2 + 64 y 2 = 48 (d) x 2 + 16 y 2 = 16 10 3
(a) (b)
35. The circle x2 + y2 = 4x + 8y + 5 intersects the line 3 5
3x – 4y = m at two distinct points if [2010] 6 5
(a) – 35 < m < 15 (b) 15 < m < 65 (c) (d)
5 3
(c) 35 < m < 85 (d) – 85 < m < – 35
43. Statement-1 : An equation of a common tangent
36. If two tangents drawn from a point P to the
parabola y2 = 4x are at right angles, then the to the parabola y 2 = 16 3 x and the ellipse
locus of P is [2010] 2 x 2 + y 2 = 4 is y = 2x + 2 3
(a) 2x + 1 = 0 (b) x=–1
(c) 2x – 1 = 0 (d) x= 1 4 3
, (m ¹ 0)
Statement-2 : If the line y = mx +
37. The two circles x + y = ax and x + y = c2 (c >
2 2 2 2 m
0) touch each other if [2011] is a common tangent to the parabola
(a) | a | = c (b) a = 2c y 2 = 16 3 x and the ellipse 2x2 + y2 = 4, then m
(c) | a | = 2c (d) 2 | a | = c satisfies m4 + 2m2 = 24 [2012]
38. The shortest distance between line y – x =1 and (a) Statement-1 is false, Statement-2 is true.
curve x = y2 is [2011] (b) Statement-1 is true, statement-2 is true;
8 statement-2 is a correct explanation for
3 2
(a) (b) Statement-1.
8 3 2
(c) Statement-1 is true, statement-2 is true;
4 statement-2 is not a correct explanation for
3
(c) (d) Statement-1.
3 4 (d) Statement-1 is true, statement-2 is false.
www.crackjee.xyz
Conic Sections M-59
44. An ellipse is drawn by taking a diameter of the 49. The locus of the foot of perpendicular drawn
circle (x – 1)2 + y2 = 1 as its semi-minor axis and from the centre of the ellipse x2 + 3y2 = 6 on any
a diameter of the circle x2 + (y – 2)2 = 4 is semi- tangent to it is [2014]
major axis. If the centre of the ellipse is at the
( x2 + y2 )
2
origin and its axes are the coordinate axes, then (a) = 6 x2 + 2 y 2
the equation of the ellipse is : [2012]
(a) 4x2 + y2 = 4 (b) x2 + 4y2 = 8
( x2 + y2 )
2
(b) = 6 x2 - 2 y 2
(c) 4x2 + y2 = 8 (d) x2 + 4y2 = 16
( x2 - y2 )
2
45. The chord PQ of the parabola y2 = x, where one (c) = 6x2 + 2 y2
end P of the chord is at point (4, – 2), is
perpendicular to the axis of the parabola. Then
( x2 - y 2 )
2
the slope of the normal at Q is [2012] (d) = 6 x2 - 2 y 2
1 50. Let C be the circle with centre at (1, 1) and radius
(a) – 4 (b) -
4 = 1. If T is the circle centred at (0, y), passing
1 through origin and touching the circle C
(c) 4 (d)
4 externally, then the radius of T is equal to
46. The circle passing through (1, –2) and touching [2014]
the axis of x at (3, 0) also passes through the
1 1
point [2013] (a) (b)
(a) (–5, 2) (b) (2, –5) 2 4
(c) (5, –2) (d) (–2, 5)
3 3
47. The equation of the circle passing through the (c) (d)
2 2
x2 y 2 51. The slope of the line touching both the parabolas
foci of the ellipse + = 1, and having
16 9
centre at (0, 3) is [2013] y 2 = 4 x and x 2 = -32 y is [2014]
(a) x2 + y2 – 6y – 7 = 0 1 2
(b) x2 + y2 – 6y + 7 = 0 (a) (b)
8 3
(c) x2 + y2 – 6y – 5 = 0
(d) x2 + y2 – 6y + 5 = 0 1 3
(c) (d)
48. Given : A circle, 2x2 + 2y2 = 5 and a parabola, y2 2 2
52. Let O be the vertex and Q be any point on the
= 4 5 x.
parabola, x2 = 8y. If the point P divides the line
Statement-1 : An equation of a common tangent
segment OQ internally in the ratio 1 : 3, then locus
to these curves is y = x + 5 . of P is : [2015]
2
(a) y = 2x 2
(b) x = 2y
5
Statement-2 : If the line, y = mx + (m ¹ 0) is their (c) x2 = y (d) y2 = x
m 53. The number of common tangents to the circles x2
common tangent, then m satisfies m4 – 3m2 + 2 = 0. + y2 – 4x – 6x – 12 = 0 and x2 + y2 + 6x + 18y + 26
[2013] = 0, is : [2015]
(a) Statement-1 is true; Statement-2 is true; (a) 3 (b) 4
Statement-2 is a correct explanation for (c) 1 (d) 2
Statement-1.
54. The area (in sq. units) of the quadrilateral formed
(b) Statement-1 is true; Statement-2 is true; by the tangents at the end points of the latera
Statement-2 is not a correct explanation for
Statement-1. x 2 y2
(c) Statement-1 is true; Statement-2 is false. recta to the ellipse + = 1, is : [2015]
9 5
(d) Statement-1 is false; Statement-2 is true.
EBD_7764
www.crackjee.xyz
M-60 Mathematics
58. If one of the diameters of the circle, given by the
27
(a) (b) 27 equation, x2 + y2 – 4x + 6y – 12 = 0, is a chord of
2 a circle S, whose centre is at (–3, 2), then the
radius of S is: [2016]
27
(c) (d) 18 (a) 5 (b) 10
4
(c) 5 2 (d) 5 3
55. Locus of the image of the point (2, 3) in the line
59. Let P be the point on the parabola, y2 = 8x which
(2x – 3y + 4) + k (x – 2y + 3) = 0, k Î R, is a :
is at a minimum distance from the centre C of the
[2015]
circle, x2 + (y + 6)2 = 1. Then the equation of the
(a) circle of radius 2. circle, passing through C and having its centre
at P is: [2016]
(b) circle of radius 3.
x
(c) straight line parallel to x-axis (a) x 2 + y2 -
+ 2y - 24 = 0
4
(d) straight line parallel to y-axis
(b) x2 + y2 – 4x + 9y + 18 = 0
56. The centres of those circles which touch the (c) x2 + y2 – 4x + 8y + 12 = 0
circle, x2 + y2 – 8x – 8y – 4 = 0, externally and (d) x2 + y2 – x + 4y – 12 = 0
also touch the x-axis, lie on: [2016] 60. A hyperbola passes through the point
(a) a hyperbola
(b) a parabola
P ( )
2, 3 and has foci at (± 2, 0). Then the
(c) a circle tangent to this hyperbola at P also passes
(d) an ellipse which is not a circle through the point : [2017]
57. The eccentricity of the hyperbola whose length
of the latus rectum is equal to 8 and the length
(a) (- 2, - 3 ) (b) (3 2, 2 3 )
of its conjugate axis is equal to half of the
distance between its foci, is : [2016]
(c) (2 2,3 3) (d) ( 3, 2 )
61. The radius of a circle, having minimum area,
2 which touches the curve y = 4 – x2 and the lines,
(a) (b) 3
3 y = |x| is : [2017]

4 4 (a) 4( 2 +1 ) (b) 2 ( 2 +1 )
(c) (d)
2( 2 - 1) 4( 2 - 1)
3 3
(c) (d)

Answer Key
1 2 3 4 5 6 7 8 9 10 11 12 13 14 15
(c) (a) (b) (c) (b) (b) (d) (b) (d) (b) (d) (d) (d) (d) (b)
16 17 18 19 20 21 22 23 24 25 26 27 28 29 30
(b) (d) (d) (d) (a) (d) (a) (d) (d) (a) (a) (d) (b) (b) (c)
31 32 33 34 35 36 37 38 39 40 41 42 43 44 45
(a) (b) (a) (a) (a) (b) (a) (a) (d) (b) (b) (a) (b) (d) (a)
46 47 48 49 50 51 52 53 54 55 56 57 58 59 60
(c) (a) (b) (a) (b) (c) (b) (a) (b) (a) (b) (a) (d) (c) (c)
61
(None)
www.crackjee.xyz
Conic Sections M-61

1. (c) Equation of circle x2 + y2 = 1 = (1)2


9 1
Þ x2 + y2 = (y – mx)2 Þ f2 = - =2 \f =± 2.
4 4
Þ x2 = m2x2 – 2 mxy;
Hence, the centres of required circle are
Þ x 2 (1 – m 2 ) + 2mxy = 0. Which
represents the pair of lines between which æ1 ö æ1 ö
ç , 2 ÷ or ç ,- 2 ÷
the angle is 45o. è2 ø è2 ø
4. (c) Let ABC be an equilateral triangle, whose
2 m2 - 0 ±2 m
tan 45 = ± = ; median is AD. A
1 - m2 1 - m2
Þ 1 – m2 = ± 2m Þ m2 ± 2m – 1 = 0
O
-2 ± 4 + 4
Þm=
2 B D C

-2 ± 2 2
= = -1 ± 2 . Given AD = 3a.
2
2. (a) For any point P (x, y) in the given circle, In D ABD, AB2 = AD2 + BD2 ;
Þ x2 = 9a2 + (x2/4) where AB = BC = AC
Y = x.
B
3 2
P x = 9a2 Þ x2 = 12a2.
A C 4
X In D OBD, OB2 = OD2 + BD2
O
x2
Þ r2 = (3a – r)2 +
we should have 4
OA £ OP £ OB Þ r2 = 9a2 – 6ar + r2 + 3a2 ; Þ 6ar = 12a2
Þ r = 2a
Þ (5 - 3) £ x 2 + y 2 £ 5 + 3 So equation of circle is x2 + y2 = 4a2
Þ 4 £ x 2 + y 2 £ 64 5. (b) Any tangent to the parabola y2 = 8ax is
3. (b) Let the required circle be 2a
y = mx+ ...(i)
x2 + y2 + 2gx + 2fy + c = 0 m
Since it passes through (0, 0) and (1, 0) If (i) is a tangent to the circle, x2 + y2 = 2a2
1 2a
Þ c = 0 and g = - then, 2a = ±
2
m m2 + 1
Points (0, 0) and (1, 0) lie inside the circle x2
+ y2 = 9, so two circles touch internally Þ m2(1 + m2) = 2 Þ ( m2 + 2)(m2 – 1) = 0
Þ c1c2 = r1 – r2 Þ m = ± 1.
So from (i), y = ± (x + 2a).
3 6. (b) r1 - r2 < C1C2 for intersection
\ g 2 + f 2 = 3 - g 2 + f 2 Þ g2 + f 2 =
2
Þ r -3< 5Þ r <8 ...(1)
EBD_7764
www.crackjee.xyz
M-62 Mathematics
and r1 + r2 > C1C2 , r + 3 > 5 Þ r > 2 ...(2) 10. (b) Let the variable circle is
From (1) and (2), 2 < r < 8. x 2 + y 2 + 2 gx + 2 fy + c = 0 ......(1)

7. (d) pr 2 = 154 Þ r = 7 It passes through (a, b)


For centre on solving equation \ a 2 + b2 + 2 ga + 2 fb + c = 0 ......(2)
2 x - 3 y = 5& 3x - 4 y = 7
(1) cuts x 2 + y 2 = 4 orthogonally
we get x = 1, y = -1
\ centre = (1, –1 ) \ 2( g ´ 0 + f ´ 0) = c - 4 Þ c = 4

Equation of circle, ( x - 1)2 + ( y + 1)2 = 7 2 \ from (2) a 2 + b2 + 2 ga + 2 fb + 4 = 0


\ Locus of centre (–g,–f) is
x 2 + y 2 - 2 x + 2 y = 47
8. (b) Equation of the normal to a parabola a 2 + b2 - 2ax - 2by + 4 = 0

y 2 = 4bx at point (bt


2
1 , 2bt1 ) is or 2ax + 2by = a 2 + b 2 + 4
11. (d) Let the variable circle be
y = – t1 x + 2bt1 + bt13
As given, it also passes through x 2 + y 2 + 2 gx + 2 fy + c = 0 ....(1)

(bt 2
2 , 2bt 2 ) then \ p 2 + q 2 + 2 gp + 2 fq + c = 0 ....(2)
Circle (1) touches x-axis,
2bt2 = – t1 bt22 + 2 bt1 + bt13
\ g 2 - c = 0 Þ c = g 2 . From (2)
(
2t2 – 2t1 = – t1 t22 – t12 ) p 2 + q 2 + 2 gp + 2 fq + g 2 = 0 ....(3)
= –t1(t2 + t1) (t2 – t1)
Let the other end of diameter through (p,
2 q) be (h, k), then
Þ 2 = – t1(t2 + t1) Þ t2 + t1 = – t
1
h+ p k+q
= - g and =-f
2 2 2
Þ t2 = – t1 –
t1 Put in (3)
2
x2 y2 æ h + pö æ k + qö æ h + pö
9. (d) - =
1 p2 + q 2 + 2 p ç - ÷ + 2q ç - ÷ +ç ÷ =0
è 2 ø è 2 ø è 2 ø
144 81 25
144 81 81 15 5
a= ,b = , e = 1+ = = Þ h2 + p 2 - 2hp - 4kq = 0
25 25 144 12 4
\ locus of (h, k)
\ Foci = ( ±3 , 0)
\ foci of ellipse = foci of hyperbola is x 2 + p2 - 2 xp - 4 yq = 0
\ for ellipse ae = 3 but a = 4,
Þ ( x - p )2 = 4qy
3
\ e= 12. (d) Two diameters are along
4
2 x + 3 y + 1 = 0 and 3x - y - 4 = 0
Then b 2 = a 2 (1 - e 2 )
solving we get centre (1, –1)
Þ b 2 = 16æç1 - 9 ö÷ = 7 circumference = 2pr = 10p
è 16 ø \ r = 5.
www.crackjee.xyz
Conic Sections M-63
\ The two equations should represent
Required circle is, ( x - 1)2 + ( y + 1)2 = 52
the same line
Þ x 2 + y 2 - 2 x + 2 y - 23 = 0 a c - d a +1
Þ = = Þ a + 1 = -a 2
13. (d) Solving y = x and the circle 1 b -a

x 2 + y 2 - 2 x = 0, we get a2 + a + 1 = 0
No real value of a.
x = 0, y = 0 and x = 1, y = 1 17. (d) Equation of circle with centre (0, 3) and
\ Extremities of diameter of the required
radius 2 is x 2 + ( y - 3)2 = 4
circle are (0, 0) and (1, 1). Hence, the
equation of circle is Let locus of the variable circle is (a , b )
( x - 0)( x - 1) + ( y - 0)( y - 1) = 0 Q It touches x - axis.
\ It¢s equation is
Þ x2 + y 2 - x - y = 0
( x - a )2 + ( y + b)2 = b 2
14. (d) Solving equations of parabolas
y 2 = 4ax and x 2 = 4ay
we get (0, 0) and ( 4a, 4a)
Substituting in the given equation of line c1
r1
2bx + 3cy + 4d = 0,
r2 c2
we get d = 0 and 2b +3c = 0 (a , b )

Þ d 2 + (2b + 3c )2 = 0

1 a
15. (b) e = . Directrix , x = = 4
2 e
Circle touch externally Þ c1c2 = r1 + r2
1
\a = 4´ = 2
2 \ a 2 + (b - 3) 2 = 2 + b
1
\b = 2 1- = 3 a 2 + (b - 3) 2 = b 2 + 4 + 4b
4
Equation of ellipse is Þ a 2 = 10(b - 1 / 2)

x2 y 2
\ Locus is x 2 = 10 æç y - ö÷
1
+ = 1 Þ 3x 2 + 4 y 2 = 12
4 3 è 2ø
16. (b) s1 = x 2 + y 2 + 2ax + cy + a = 0 Which is parabola.
18. (d) Let the centre be (a, b)
s2 = x 2 + y 2 - 3ax + dy - 1 = 0
2 2 2
Equation of common chord of circles Q It cuts the circle x + y = p
s1 and s2 is given by s1 - s2 = 0 orthogonally

Þ 5ax + (c - d ) y + a + 1 = 0 \ Using 2 g1 g2 + 2 f1 f 2 = c1 + c2 ,
we get
Given that 5x + by – a = 0 passes through
P and Q 2(-a ) ´ 0 + 2(- b) ´ 0 = c1 - p2
EBD_7764
w w w . c r a c k j e e . x y
M-64 Mathematics
c1 = p 2 h +1 k+0
a= , b=
Let equation of circle is 2 2
2 a -1 = h 2 b = k.
x 2 + y 2 - 2ax - 2by + p 2 = 0
It passes through (2b) 2 = 8(2a - 1) Þ b 2 = 4a - 2

(a, b) Þ a 2 + b 2 - 2aa - 2bb + p 2 = 0 Þ y2 - 4x + 2 = 0 .


\ Locus of (a, b) is
x2 y2
21. (d) Tangent to the hyperbola - = 1 is
\ 2ax + 2by - (a 2 + b 2 + p 2 ) = 0 . a2 b2

y = mx ± a 2 m2 - b2
q Given that y = a x + b is the tangent of
3q hyperbola
19. (d)
Þ m = a and a 2 m2 - b2 = b 2
\ a 2a 2 - b 2 = b 2
As per question area of one sector = 3
area of another sector Locus is a 2 x 2 - y 2 = b2 which is
Þ angle at centre by one sector hyperbola.
= 3 ´ angle at centre by another sector 22. (a) Q ÐFBF ' = 90° Þ FB 2 + F ' B 2 = FF ' 2
Let one angle be q then other = 3q
Clearly q + 3q = 180 Þ q = 45o
( ) ( )
2 2
\ Angle between the diameters repre- \ a 2e2 + b2 + a 2e 2 + b2 = (2ae) 2
sented by combined equation
b2
ax 2 + 2 ( a + b ) xy + by 2 = 0 is 45o 2
Þ 2(a 2 e 2 + b2 ) = 4a 2 e 2 Þ e = 2
a
B (0, b)
2 h2 - ab
\ Using tan q =
a+b
F' (-ae, 0) O F (ae, 0)
we get tan 45o =
2 ( a + b) 2 - ab
a+b

2 a 2 + b2 + ab Also e 2 = 1 - b 2 / a 2 = 1 - e2
Þ1=
a+b
1
Þ 2e 2 = 1, e = .
2
(
Þ ( a + b) = 4 a + b + ab
2 2
) 2
23. (d) Point of intersection of 3 x - 4 y - 7 = 0
Þ a 2 + b 2 + 2 ab = 4a 2 + 4b 2 + 4ab and 2 x - 3 y - 5 = 0 is (1, - 1) which is
2 2
Þ 3a + 3b + 2 ab = 0 the centre of the circle and radius = 7

20. (a) P = (1, 0) Q = (h, k) Such that K 2 = 8h \ Equation is ( x - 1)2 + ( y + 1) 2 = 49

Let (a, b) be the midpoint of PQ Þ x 2 + y 2 - 2 x + 2 y - 47 = 0


www.crackjee.xyz
Conic Sections M-65
24. (d) Let M(h, k) be the mid point of chord AB 3 3
\ e= =
2p a 5
where ÐAOB =
3 27. (d) Equation of circle whose centre is (h, k)
i.e (x – h)2 + (y – k)2 = k2

O (0, 0)
3
p/3 (h, k)
A M(h, k) B
(-1,1)
X' X
p p 3
\ ÐAOM = . Also OM = 3cos =
3 3 2

3 9
Þ h2 + k 2 = Þ h2 + k 2 = (radius of circle = k because circle is tangent
2 4
to x-axis)
2 2 9 Equation of circle passing through (–1, +1)
\ Locus of (h, k) is x + y =
4 \ (–1 –h)2 + (1 – k)2 = k2
Þ 1 + h2 + 2h + 1 + k2 – 2k = k2
a3 x 2 a 2 x Þ h2 + 2h – 2k + 2 = 0
25. (a) Given parabola is y = + - 2a
3 2 D³0
\ (2)2 – 4 × 1.(–2k + 2) ³ 0
a3 æ 3 3 9 ö 3a
Þy= çè x + 2a x + ÷- - 2a Þ 4 – 4(–2k + 2) ³ 0 Þ 1 + 2k – 2 ³ 0
3 16a 2 ø 16
1
3 2
Þ k³
35a a æ 3ö 2
Þy+ = çx+ ÷
16 3 è 4a ø 28. (b) Given, equation of hyperbola is

æ -3 -35a ö x2 y2
\ Vertex of parabola is çè , ÷ -
=1
4a 16 ø cos 2 a sin 2 a
To find locus of this vertex, We know that the equation of hyperbola is
-3 -35a x2 y2
x= and y = - = 1 Here, a 2 = cos 2 a and
4a 16 a 2
b 2

-3 16 y
Þ a= and a = - b 2 = sin 2 a
4x 35
We know that, b 2 = a 2 (e2 - 1)
-3 -16 y
Þ = Þ 64xy = 105
4x 35 Þ sin 2 a = cos 2 a (e 2 - 1)
105 Þ sin 2 a + cos2 a = cos 2 a.e2
Þ xy = which is the required locus.
64
Þ e 2 = 1 + tan 2 a = sec 2 a
26. (a) 2ae = 6 Þ ae = 3 ; 2b = 8 Þ b = 4 Þ e = sec a
2 2 2
b 2 = a 2 (1 - e 2 ) ; 16 = a - a e 1
\ ae = cos a . =1
cos a
Þ a 2 = 16 + 9 = 25 Þ a = 5
EBD_7764
www.crackjee.xyz
M-66 Mathematics
Co-ordinates of foci are (± ae, 0)
Y
i.e. ( ± 1, 0)
Hence, abscissae of foci remain constant æa ö
çè - ae÷ø
when a varies. e
29. (b) Parabola y2 = 8x X´ X
O S
(ae, 0)
Y y 2 = 8x
Y´ x= a/e

(2,0) 8
X' X Þa =
F 3
x+2=0

\ Semi major axis = 8/3


32. (b) Vertex of a parabola is the mid point of
Y' focus and the point

We know that the locus of point of Y


intersection of two perpendicular tangents
to a parabola is its directrix.
Point must be on the directrix of parabola
Q equation of directrix x + 2 = 0 OA X
B
Þ x = –2
Hence the point is (–2, 0)
Y¢ x= 2
30. (c) The given circle is x2 + y2 + 2x + 4y –3 = 0
where directrix meets the axis of the
parabola.
Here focus is O(0, 0) and directrix meets
P(1,0) Q(a,b) the axis at B(2, 0)
C(–1, –2) \ Vertex of the parabola is (1, 0)
33. (a) The given circles are
S1 º x2 + y2 + 3x + 7y + 2p – 5 = 0....(1)
Centre (–1, –2) S 2 º x2 + y2 + 2x + 2y – p2 = 0 ....(2)
Let Q ( a, b) be the point diametrically \ Equation of common chord PQ is
opposite to the point P(1, 0), S1 – S2 = 0
1+ a 0+b Þ L º x + 5 y + p2 + 2 p - 5 = 0
then = –1 and = –2
2 2 Þ Equation of circle passing through P and
Þ a = –3, b = – 4 Q is
So, Q is (–3, –4) S1 + l L = 0
31. (a) Perpendicular distance of directrix from
Þ (x2 + y2 + 3x + 7y + 2p – 5)
focus
a + l (x + 5y + p2 +2p – 5) = 0
= – ae = 4 As it passes through (1, 1), therefore
e
(7 + 2p ) + l (2p + p2 + 1) = 0
æ 1ö
Þaç2 – ÷ = 4
è 2ø 2p + 7
Þ l =–
( p + 1) 2
www.crackjee.xyz
Conic Sections M-67
which does not exist for p = – 1 a a
2 2 So, = c- Þ a =c
x y 2 2
34. (a) The given ellipse is + =1 38. (a) Shortest distance between two curve
4 1
So A = (2, 0) and B= (0, 1) occurred along the common normal, so –
If PQRS is the rectangle in which it is 2t = – 1
inscribed, then Þ t = 1/2
P = (2, 1).
y
x2 y2
Let + = 1 be the ellipse
a 2 b2
circumscribing the rectangle PQRS. (t2, t)

Q B (0,1) P (2, 1) x
O
A
O (2,0) (4,0)

R S
3 2
So shortest distance between them is
8
Then it passed through P (2,1 )
4 1 x2 y2
\ + = 1 ....(A) 39. (d) Let the ellipse be =1 +
2
a 2
b2 a b2
Also, given that, it passes through (4, 0) It passes through (– 3, 1) so
16 9 1
\ 2 + 0 = 1 Þ a 2 = 16 2
+ 2 = 1 ..(i)
a a b
Þ b2 = 4/3 [substituting a2 = 16 in eqn (A)] Also, b 2 = a 2 (1 - 2 / 5)
Þ 5b2 = 3a 2 ...(ii)
x2 y2
\ The required ellipse is + =1 32 32
16 4 / 3 Solving (i) and (ii) we get a2 = , b2 =
or x2 + 12y2 =16 3 5
So, the equation of the ellipse is
35. (a) Circle x 2 + y 2 - 4 x - 8 y - 5 = 0
Centre = (2, 4), Radius =
3x 2 + 5 y 2 = 32
4 + 16 + 5 = 5 40. (b) Circle whose diametric end points are (1,0)
If circle is intersecting line 3 x - 4 y = m, at and (0,1) will be of smallest radius. Equation
two distinct points. of this smallest circle is
Þ length of perpendicular from centre to (x – 1) (x – 0) + (y – 0) (y – 1) = 0
the line < radius Þ x2 + y2 – x – y = 0
41. (b) ae = 2
6 - 16 - m
Þ < 5 Þ 10 + m < 25 e=2
5 \ a =1
Þ –25 < m + 10 < 25 Þ – 35 < m < 15
36. (b) The locus of perpendicular tangents is (
b 2 = a 2 e2 - 1)
directrix i.e., x = - a; x = -1 b = 1( 4 - 1)
2

37. (a) If the two circles touch each other, then b2 = 3


they must touch each other internally.
EBD_7764
www.crackjee.xyz
M-68 Mathematics
x 2
y 2 On comparing (1) and (2), we get
Equation of hyperbola, - =1
a 2
b2 4 3
= ± 2m 2 + 4
m
x2 y 2
Þ - =1 Squaring on both the sides, we get
1 3
16 (3) = (2m2 + 4) m2
3 x2 - y 2 = 3 Þ 48 = m2 (2m2 + 4)
42. (a) Let centre of the circle be (1,h)
[Q circle touches x-axis at (1,0)] Þ 2m4 + 4m2 – 48 = 0
Þ m4 + 2m2 –24 = 0
Y Þ (m2 + 6)(m2 – 4) = 0
Þ m2= 4 (Q m2 ¹ – 6) Þ m = ± 2
Þ Equation of common tangents are
(1,h) (2,3) y = ± 2x ± 2 3
C B Thus, statement-1 is true.
Statement-2 is obviously true.
A(1,0) X
44. (d) Equation of circle is (x – 1)2 + y2 =1
Þ radius = 1 and diameter = 2
\ Length of semi-minor axis is 2.
Let the circle passes through the point B (2,3)
Equation of circle is x2 + (y – 2)2 = 4 = (2)2
\ CA = CB (radius)
2 2
Þ radius = 2 and diameter = 4
Þ CA = CB \ Length of semi major axis is 4
Þ (1 – 1)2 + (h – 0)2 = (1 – 2)2 + ( h – 3)2 We know, equation of ellipse is given by
Þ h2 = 1 + h2 + 9 – 6h
x2 y2
10 5 + =1
Þ h= = ( Major axis ) 2
( Minor axis )2
6 3
43. (b) Given equation of ellipse is 2x2 + y2 = 4 x2 y2 2 2
2 2 2 2 Þ 2
+ 2
=1 Þ x +y =1
2x y x y (4) (2) 16 4
Þ + =1 Þ + =1
4 4 2 4
Equation of tangent to the ellipse Þ x2 + 4y2 =16
45. (a) Point P is (4, –2) and PQ ^ x-axis
x2 y 2
+ = 1 is So, Q = (4, 2)
2 4
y = mx ± 2m2 + 4 ...(1) Y
nt
Tange
(Q equation of tangent to the ellipse Q
x2 y2 Normal
2
+ 2 =1
a b
X
2 2 2
is y = mx + c where c = ± a m + b )
Now, Equation of tangent to the parabola P
4 3 (4, – 2)
y 2 = 16 3 x is y = mx + ...(2)
m
(Q equation of tangent to the parabola
a Equation of tangent at (4, 2) is
y2 = 4ax is y = mx + )
m
www.crackjee.xyz
Conic Sections M-69

1 7
yy1 = (x + x1) Þ e=
2 4
1 Now, radius of this circle = a2 = 16
Þ 2y = (x + 2) Þ 4y = x + 2
2 Þ Focii = (± 7, 0)
x 1 Now equation of circle is (x – 0)2 + (y – 3)2
Þy= + = 16
4 2
x2 + y2 – 6y – 7 = 0
1
So, slope of tangent = 48. (b) Let common tangent be
4
\ Slope of normal = – 4 5
y = mx +
46. (c) Since circle touches x-axis at (3, 0) m
\ the equation of circle be Since, perpendicular distance from centre
(x – 3)2 + (y – 0)2 + ly = 0 of the circle to the common tangent is equal
to radius of the circle, therefore

5
m 5
=
1+ m 2 2
A (3, 0) On squaring both the side, we get
m2 (1 + m2) = 2
Þ m4 + m2 – 2 = 0
A
(1, –2) Þ (m2 + 2)(m2 – 1) = 0
Þ m= ±1 (Q m ¹ ± 2 )

As it passes through (1, –2)


( )
y = ± x + 5 , both statements are

\ Put x = 1, y = –2 correct as m = ±1 satisfies the given


Þ (1 – 3)2 + (–2)2 + l(–2) = 0 equation of statement-2.
Þ l=4 49. (a) Given equation of ellipse can be written as
\ equation of circle is x2 y 2
(x – 3)2 + y2 – 8 = 0 + =1
6 2
Now, from the options (5, –2) satisfies
Þ a2 = 6, b2 = 2
equation of circle.
Now, equation of any variable tangent is
47. (a) From the given equation of ellipse, we have

9 y = mx ± a 2 m2 + b2 ...(i)
a = 4, b = 3, e = 1 -
16 where m is slope of the tangent
So, equation of perpendicular line drawn
from centre to tangent is
-x
y= ...(ii)
m
Eliminating m, we get

( x4 + y 4 + 2 x2 y 2 ) = a 2 x2 + b2 y 2
EBD_7764
www.crackjee.xyz
M-70 Mathematics
ALTERNATIVE METHOD:
Þ ( x 2 + y 2 )2 = a 2 x 2 + b2 y 2
1
Let tangent to y2 = 4x be y = mx +
Þ ( x2 + y 2 )2 = 6 x2 + 2 y 2 m
Since this is also tangent to x2 = – 32y
50. (b) æ 1ö
C \ x 2 = -32 ç mx + ÷
è mø
(0, y) (1, 1) 32
Þ x2 + 32mx + =0
T m
Now, D = 0
æ 32 ö
Equation of circle (32) 2 - 4 ç ÷ = 0
è mø
C º ( x - 1)2 + ( y - 1)2 = 1 4 1
3
Radius of T = | y | Þm = Þ m=
32 2
T touches C externally therefore,
52. (b) Let P(h, k) divides
Distance between the centres = sum of their
radii OQ in the ratio 1 : 3
Let any point Q on x2 = 8y is (4t, 2t2).
Þ (0 - 1)2 + ( y - 1)2 = 1+ | y |
Þ (0 – 1)2 + (y –1)2 = (1 + |y|)2
Q (4t, 2t2)
Þ 1 + y2 + 1 – 2y = 1 + y2 + 2| y | P3
1
2 | y | = 1 – 2y
O
1
If y > 0 then 2y = 1 – 2y Þ y =
4
If y < 0 then –2y = 1 – 2y Þ 0 = 1
(not possible)
Then by section formula
1
\ y=
4 t2
51. (c) Given parabolas are Þ k= and h = t
2
y2 = 4x ...(i)
2
x = –32y ...(ii) Þ 2k = h 2
Let m be slope of common tangent Required locus of P is x2 = 2y
Equation of tangent of parabola (1) 53. (a) x2 + y2 – 4x – 6y – 12 = 0 ...(i)

1 Centre, C1 = (2, 3)
y = mx + ...(i)
m Radius, r1 = 5 units
Equation of tangent of parabola (2) x2 + y2 + 6x + 18y + 26 = 0 ...(ii)
y = mx + 8m2 ...(ii) Centre, C2 = (–3, –9)
(i) and (ii) are identical
Radius, r2 = 8 units
1 1 1
Þ = 8m 2 Þ m3 = Þ m =
m 8 2 C1C2 = (2 + 3)2 + (3 + 9)2 = 13 units
r1 + r2 = 5 + 8 = 13
www.crackjee.xyz
Conic Sections M-71
\ C1 C2 = r1 + r2 By symmetry area of quadrilateral
27
= 4 × (Area DOAB) = 4 ´ = 27 sq. units.
4
55. (a) Intersection point of 2x – 3y + 4 = 0 and x – 2y
C1 + 3 = 0 is (1, 2)

C2 A(2, 3)

Therefore there are three common tangents.


P
54. (b) The end point of latus rectum of ellipse (1, 2)

x 2
y 2 æ b 2ö
+ = 1 in first quadrant is ç ae, ÷
2 2 è aø B(a, b)
a b
and the tangent at this point intersects x-axis at
Since, P is the fixed point for given family of
æa ö lines
çè , 0÷ø and y-axis at (0, a).
e So, PB = PA
(a – 1)2 + (b – 2)2 = (2 – 1)2 + (3 – 2)2
x2 y 2
The given ellipse is + =1
9 5 (a – 1)2 + (b –2)2 = 1 + 1 = 2
Then a2 = 9, b2 = 5
(x – 1)2 + (y – 2)2 = ( 2) 2
5 2 (x – a)2 + (y – b)2 = r2
Þ e = 1– =
9 3
Therefore, given locus is a circle with centre (1,
\ end point of latus rectum in first quadrant is
2) and radius 2.
L (2, 5/3) 56. (b)

2x y
Equation of tangent at L is + =1
9 3
It meets x-axis at A (9/2, 0) and y-axis at B (0, 3) C(4, 4)
1 9 27
\ Area of DOAB = ´ ´3= 6
2 2 4
k
Y P(h, k)
B
(0, 3) k

X' X
L(2,5/3)
For the given circle,
C O S A X centre : (4, 4)
(9/2, 0) radius = 6
2
6+ k = (h - 4) + (k - 4) 2
D
(h – 4)2 = 20k + 20
EBD_7764
www.crackjee.xyz
M-72 Mathematics
\ locus of (h, k) is Centre of new circle = P(2t2, 4t)
(x – 4)2 = 20(y + 1), = P(2, – 4)
which is a parabola.
Radius = PC = (2 – 0) 2 + (–4 + 6)2
2b 2 1
57. (a) =8 and 2b = (2ae)
a 2 =2 2
\ Equation of circle is :
Þ 4b2 = a 2 e2 Þ 4a 2 (e 2 - 1) = a 2 e 2
2
2
(x –2)2 + (y + 4) = 2 2 ( )
Þ 3e2 = 4 Þ e =
3 Þ x2 + y2 – 4x + 8y + 12 = 0

x2 y2
60. (c) Equation of hyperbola is - =1
58. (d) S a2 b2
(–3, 2)
O foci is (±2, 0) Þ ae = 2 Þ a2e2 = 4
Since b2 = a2 (e2 – 1)
b2 = a2 e2 – a2 \ a2 + b2 = 4 ...(1)
5 2
5
B A Hyperbola passes through ( 2, 3 )
(2, –3)
2 3
\ - =1 ...(2)
2
a b2
Centre of S : O (–3, 2) centre of given circle
2 -3
A(2, –3) =1
4 - b b2 2
Þ OA = 5 2
Þ b4 + b2 – 12 = 0
Also AB = 5 (Q AB = r of the given circle)
Þ (b2 – 3) (b2 + 4) = 0
Þ Using pythagoras theorem in DOAB
Þ b2 = 3
r=5 3 b2 = – 4 (Not possible)
59. (c) Minimum distance Þ perpendicular distance For b2 = 3
Eqn of normal at p111(2t2, 4t)
y = –tx + 4t + 2t3 2 2
It passes through C(0, –6) Þ a2 = 1 \ x - y = 1
1 3
Þ t3 + 2t + 3 = 0 Þ t = – 1

2x 3y
Equation of tangent is - =1
1 3

Clearly (2 2,3 3) satisfies it.


61. (None)
(Let the equation of circle be
2 x2 + (y – k)2 = r2
P (2t , 4t)
It touches x – y = 0
C
www.crackjee.xyz
Conic Sections M-73
It touches y = 4 – x2 as well
\ Solving the two equations
k2
Þ 4 – y + (y – k)2 =
2

k2
Þ 1y2 – y(2k + 1) + + 4= 0
2
It will give equal roots \ D = 0
0
æ k2 ö
Þ (2k + 1)2 = 4 ç + 4 ÷
è 2 ø
Þ 2k2 + 4k – 15 = 0

-2 + 34
Þ k=
0-k 2
Þ =r
2 k -2 + 34
\ r= =
Þ k= r 2 2 2 2
\ Equation of circle becomes Which is not matching with any of the option
given here.1
k2
x2 + (y – k)2 = ...(i)
2
EBD_7764
www.crackjee.xyz
M-74 Mathematics

Limits and Derivatives 12


[2002]
1 - cos 2 x
1. lim is [2002] (a) 2 (b) 4
x ®0 2x (c) 1 (d) 1/2
(a) 1 (b) –1
log(3 + x ) - log(3 - x)
(c) zero (d) does not exist 7. If lim = k , the value of
x®0 x
x k is [2003]
æ x2 + 5 x + 3 ö
Lim ç ÷ 2
2. [2002] (a) -
x ®¥ çè x 2 + x + 2 ÷ø 3
(b) 0

(a) e4 (b) e2 1 2
(c) - (d)
3 3
(c) e3 (d) 1
é æ xö ù
3. Let f (x) = 4 and f ¢ (x) = 4. Then ê1 - tan çè 2 ÷ø ú [1 - sin x]
ë û
8. lim is [2003]
xf (2) - 2 f ( x) x®
p é æ ö
x ù 3
lim is given by [2002] 2 ê1 + tan çè ÷ø ú [ p - 2 x ]
x®2 x-2 ë 2 û
1
(a) 2 (b) –2 (a) ¥ (b)
8
(c) – 4 (d) 3
1
(c) 0 (d)
1 p + 2 p + 3 p + ..... + n p 32
4. lim is [2002]
n ®¥ n p +1 2x
lim æ a b ö = e2 , then the values of
9. If x®¥ çç1+ + ÷÷
1 1 è x x2 ø
(a) (b)
p +1 1- p a and b, are [2004]
1 1 1 (a) a = 1 and b = 2 (b) a = 1, b Î R
(c) - (d)
p p -1 p+2 (c) a Î R, b = 2 (d) a Î R, b Î R
10. Let a and b be the distinct roots of
log x n - [ x]
5. lim , n Î N , ([x] denotes greatest 2
[ x]
ax 2 + bx + c = 0 , then lim 1 - cos(ax + bx + c )
x ®0
2
integer less than or equal to x) [2002] x®a ( x - a)
(a) has value -1 (b) has value 0 is equal to [2005]
(c) has value 1 (d) does not exist a2
(a) (a - b ) 2 (b) 0
2
f ( x) - 1
6. If f (1) = 1, f ¢(1) = 2, then lim is -a 2 1
x ®1 x -1 (c) (a - b ) 2 (d) (a - b) 2
2 2
www.crackjee.xyz
Limits and Derivatives M-75
11. Let f : R ® R be a positive increasing function (a) 0 (b) 1
(c) 2 (d) 3
f (3x ) f (2 x)
with lim = 1 then lim = (1 - cos 2 x )(3 + cos x )
x ®¥ f ( x) x ®¥ f ( x) 14. lim is equal to [2013]
x®0 x tan 4 x
[2010]
1 1
2 3 (a) - (b)
(a) (b) 4 2
3 2
(c) 1 (d) 2
(c) 3 (d) 1

æ 1 - cos{2( x - 2)} ö 15. lim


(
sin p cos 2 x ) is equal to: [2014]
12. lim ç ÷÷ [2011] x®0 x2
x ®2 ç x-2
è ø
(a) -p (b) p
(a) equals 2 (b) equals – 2 p
(c) (d) 1
2
1
(c) equals (d) does not exist
2 cot x - cos x
16. lim equals : [2017]
13. Let f : R ® [0, ¥) be such that lim f(x) exists x®
p
( p - 2x )3
x® 5 2

( f ( x ) )2 - 9 = 0 1 1
and lim [2011RS] (a)
4
(b)
24
x® 5 x -5
1 1
Then lim
x® 5
f(x) equals : (c) (d)
16 8

Answer Key
1 2 3 4 5 6 7 8 9 10 11 12 13 14 15
(d) (a) (c) (a) (d) (a) (d) (d) (b) (a) (d) (d) (d) (d) (b)
16
(c)

1. (d) x
æ x 2 + 5 x + 3ö
2. (a) lim ç 2 ÷
1 - cos 2 x 1 - (1 - 2 sin 2 x) x ®¥ è x + x + 2 ø
lim Þ lim ;
2x 2x
x
æ 4x +1 ö
= lim ç1 + 2 ÷
2 sin 2 x | sin x | x ®¥ è x + x + 2ø
lim Þ lim
x®0 2x x®0 x ( 4 x +1) x
é x2 + x + 2 ù x2 + x + 2
The limit of above does not exist as êæ 4 x + 1 ö 4 x+1 ú
LHS = –1 ¹ RHL = 1 = lim êç1 + 2 ÷ ú
x ®¥ ê è x + x + 2ø ú
ëê ûú
EBD_7764
www.crackjee.xyz
M-76 Mathematics
4x2 + x æ yö
tan ç - ÷ .(1 - cos y )
lim é 1
lù è 2ø
êQ lim (1 + lx ) x = e ú
2
= e x®¥ x + x+ 2 = lim
ë x®¥ û y®0 ( -2 y )3
y y
4+
1 - tan 2sin 2
x = lim 2 2
lim
x ®¥ 1+ 1 + 2 y®0 y3
x =ex 2 4 (-8). .8
=e 8
3. (c) Apply L H Rule
xf (2) - 2 f ( x ) æ 0ö y
We have, lim çè ÷ø tan é 2
x®2 x-2 0 1 2 sin y / 2 ù 1
= lim .ê ú =
y ®0 32 æ ö ë y / 2 û
y 32
ç ÷
= lim f (2) - 2 f ¢( x ) = f (2) - 2 f ¢ (2) è2ø
x® 2
1
= 4 – 2 ´ 4 = –4. 9. (b) We know that lim (1 + x ) x = e
x ®¥
1p + 2 p + .... + n p
4. (a) We have lim ; 2x
n® ¥ n p+1 æ a bö
\ lim ç1 + + ÷ = e2
1 1 x ®¥ è x x2 ø
n
rp é x p +1 ù 1
lim å
n ®¥ r =1 n p × n
= ò x p
dx = ê
p + 1
ú =
p +1 æa b ö
0 ëê ûú0 é æ 1 öù
2xç + ÷
è x x2 ø
êæ a b ö ç a + b ÷ ú
5. (d) Since lim [ x ] does not exist, hence the Þ lim ê ç1 + + ÷ çè x 2 ÷ø ú = e2
x ®0 x®¥ è x x2 ø x
ê ú
ë û
required limit does not exist.
é bù
æ0ö
f ( x) - 1 lim 2 ê a + ú 2a 2
6. (a) lim ç ÷ form using L’’ Þ e x®¥ ë xû
= e2 Þ e = e
x ®1 x -1 è0ø
1
f ¢ ( x) Þ a = 1 and b Î R
2 f ( x)
Hospital’s rule = lim
x ®1 1/ 2 x
2x
lim æ a bö
f ¢ (1) 2 x ®¥ ç 1 + + = e2
= = =2. è x x 2 ÷ø
f (1) 1 lim æ a b ö
x ®¥ ç 1+ + 2 -1÷ 2 x
log(3 + x) - log(3 - x) Þe è x x ø = e2
7. (d) lim =k
x ®0 x
æ 2b ö
(by L 'Hospital rule) Þlim
x®¥ ç 2a + ÷=2
è xø
1 -1
- Þ 2a + 0 = 2, b Î R Þ a = 1, b Î R
Þ lim 3 + x 3 -x =k \2 = k 10. (a) Given limit =
x ®0 1 3
1 - cos a ( x - a )( x - b )
lim
æ p xö x ®a ( x - a )2
tan ç - ÷ .(1 - sin x )
è 4 2ø
8. (d) lim
p ( p - 2 x )3 æ ( x - a )( x - b) ö
x® 2sin 2 ç a ÷ø
2 è 2
= lim
p x® a ( x - a )2
Let x = + y; y ® 0
2
www.crackjee.xyz
Limits and Derivatives M-77
= (1 - cos 2 x ) (3 + cos x) x
lim ·
æ ( x - a ) ( x - b) ö x®0 x 2 1 tan 4 x
sin 2 ç a ÷ø
2 è 2
lim ´ 2sin 2 x 3 + cos x x
x ® a ( x - a)2 a 2 ( x - a ) 2 ( x - b) 2 = lim 2
· ·
x®0 x 1 tan 4x
4
sin 2 x x
a 2 ( x - a )2 ( x - b)2 = 2 lim · lim 3 + cos x · lim
´ x®0 x 2 x®0 x ®0 tan 4 x
4
a 2 (a - b)2 1 4x 1
= . = 2.4 lim = 2.4. = 2
2 4 x®0 tan 4 x 4

11. (d) f(x) is a positive increasing function.


sin( p cos 2 x)
\ 0 < f ( x ) < f (2 x ) < f (3 x ) 15. (b) Consider lim
x®0 x2
f (2 x ) f (3 x)
Þ 0 < 1 < f ( x) < f ( x)
sin éëp(1 - sin 2 x) ùû
= lim
f (2 x ) f (3 x ) x ®0 x2
Þ xlim 1 £ lim £ lim
®¥ x ®¥ f ( x) x ®¥ f ( x)
(p - p sin 2 x)
By Sandwich Theorem. = lim sin [Q sin (p – q) =
x ®0 x2
f (2 x ) sin q]
Þ xlim =1
®¥ f ( x)
(p sin 2 x) p sin 2 x
= lim sin ´
1 - cos{2( x - 2)} x ®0 p sin 2 x x2
12. (d) lim
x ®2 x-2 æ sin x ö
2
= lim 1´ p ç ÷ =p
2 sin( x - 2) x ®0 è x ø
= lim
x®2 x-2 cot x(1 - sin x)
16. (c) lim
p 3
L.H.L = - lim 2 sin( x - 2) x® æ pö
= -1 2 -8 ç x - ÷
(at x = 2) x®2 ( x - 2) è 2ø
R.H.L = lim 2 sin( x - 2)
=1
(at x = 2) x®2 ( x - 2) cot x(1 - sin x)
= limp 3
Thus L.H.L ¹ R.H.L x® æp ö
(at x = 2) (at x = 2) 8ç - x ÷
2
è 2 ø
p p
1 - cos{2( x - 2)} Put - x = t Þ as x ® Þ t ® 0
Hence, lim does not 2 2
x® 2 x-2 æp öæ æ p öö
exist. cot ç - t ÷ ç 1 - sin ç - t ÷ ÷
è2 øè è 2 øø
= lim
( f ( x)) 2 - 9 t®0 3
8t
13. (d) lim =0
x® 5 x -5 tan t(1 - cos t)
= lim
t®0 8t 3
lim [(f (x) )2 – 9] = 0 Þ lim f (x) = 3 tan t 1 - cos t
x® 5 x®5
= lim .
t ®0 8t t2
14. (d) Multiply and divide by x in the given 1 1 1
expression, we get = .1. =
8 2 16
EBD_7764
www.crackjee.xyz
M-78 Mathematics

Mathematical Reasoning 13
DIRECTIONS: Given below question contains Statement-2 is not a correct explanation for
two statements: Statement-1(Assertion) and Statement-1.
Statement-2(Reason). This question also has (b) Statement-1 is true, Statement-2 is false.
four alternative choices, only one of which is (c) Statement-1 is false, Statement-2 is true.
the correct answer. You have to select the correct
(d) Statement-1 is true, Statement-2 is true,
choice.
Statement-2 is a correct explanation for
1. Let p be the statement “x is an irrational number”,
statement -1
q be the statement “y is a transcendental number”,
and r be the statement “ x is a rational number iff 4. Let S be a non-empty subset of R. Consider the
y is a transcendental number”. [2008] following statement :
Statement-1 : r is equivalent to either q or p P : There is a rational number x Î S such that
Statement-2 : r is equivalent to ~(p«~q). x > 0.
(a) Statement -1 is false, Statement-2 is true Which of the following statements is the
negation of the statement P ? [2010]
(b) Statement -1 is true, Statement-2 is true;
Statement -2 is a correct explanation for (a) There is no rational number x Î S such than x < 0.
Statement-1 (b) Every rational number x Î S satisfies x < 0.
(c) Statement -1 is true, Statement-2 is true; (c) x Î S and x < 0 Þ x is not rational.
Statement -2 is not a correct explanation (d) There is a rational number x Î S such that x < 0.
for Statement-1 5. Consider the following statements [2011]
(d) Statement -1 is true, Statement-2 is false P : Suman is brilliant
2. The statement p ® (q®p) is equivalent to Q : Suman is rich
[2008] R : Suman is honest
(a) p ® (p® q) (b) p ® (p Ú q) The negation of the statement “Suman is
(c) p ® (p Ù q) (d) p ® (p «q) brilliant and dishonest if and only if Suman is
rich” can be expressed as
DIRECTIONS: Given below question contains two
statements: Statement-1(Assertion) and Statement (a) ~ (Q « ( P Ù ~ R))
2(Reason). This question also has four alternative (b) ~ Q «~ P Ù R
choices, only one of which is the correct answer. You (c) ~ ( P Ù ~ R) « Q
have to select the correct choice.
(d) ~ P Ù (Q «~ R )
3. Statement-1 : ~ ( p «~ q) is equivalent to
6. The only statement among the following that is
p «q.
a tautology is [2011RS]
Statement-2 : ~ ( p «~ q) is a tantology (a) A Ù (A Ú B)
[2009]
(b) A Ú (A Ù B)
(a) Statement-1 is true, Statement-2 is true;
www.crackjee.xyz
Mathematical Reasoning M-79
(c) [A Ù (A ® B)] ® B 9. The statement : ( p « : q ) is: [2014]
(d) B ® [A Ù (A ® B)] (a) a tautology
7. The negation of the statement
(b) a fallacy
"If I become a teacher, then I will open a school",
is : [2012] (c) equavalent to p « q
(a) I will become a teacher and I will not open (d) equivalent to : p « q
a school.
(b) Either I will not become a teacher or I will 10. The negation of ~ s Ú (~ r Ù s) is equivalent to :
not open a school. [2015]
(c) Neither I will become a teacher nor I will
open a school. (a) s Ú (r Ú ~ s) (b) sÙ r
(d) I will not become a teacher or I will open a (c) s Ù ~ r (d) s Ù (r Ù ~ s)
school.
11. The Boolean Expression
8. Consider
Statement-1 : (p ^ ~ q) ^ (~ p ^ q) is a fallacy. (p Ù : q) Ú qÚ (: p Ù q) is equivalent to:
Statement-2 : (p ® q) « (~ q ® ~ p) is a [2016]
tautology. [2013] (a) p Ú q (b) p Ú : q
(a) Statement-1 is true; Statement-2 is true; (c) : p Ù q (d) p Ù q
Statement-2 is a correct explanation for
12. The following statement [2017]
Statement-1.
(p ® q) ® [(~p ® q) ® q] is :
(b) Statement-1 is true; Statement-2 is true;
(a) a fallacy
Statement-2 is not a correct explanation for
(b) a tautology
Statement-1.
(c) equivalent to ~ p ® q
(c) Statement-1 is true; Statement-2 is false.
(d) Statement-1 is false; Statement-2 is true. (d) equivalent to p ® ~q

Answer Key
1 2 3 4 5 6 7 8 9 10 11 12
(None) (b) (b) (b) (a) (c) (a) (b) (c) (b) (a) (b)

1. (None) From columns (1), (2) and (3), we observe,


p : x is an irrational number none of the these statements are equivalent
q : y is a transcendental number to each other.
r : x is a rational number iff y is a \ Statement 1as well as statement 2 both
transcendental number. are false.
clearly r : : p « q \ None of the options is correct.
Let us use truth table to check th e 2. (b) Let us make the truth table for the given
equivalence of ‘r’ and ‘q or p’; ‘r’ and statements, as follows :
: ( p «: q) p q pÚq q®p p ®(q®p) p ®(pÚq)
T T T T T T
1 2 3 T F T T T T
p q ~p ~q ~p «q q or p p«~q ~(p«~q) F T T F T T
T T F F F T F T F F F T T T
T F F T T T T F
F T T F T T T F From table we observe
F F T T F F F T
p ® (q®p) is equivalent to p®(pÚq)
EBD_7764
www.crackjee.xyz
M-80 Mathematics
3. (b) The truth table for the logical statements, But ~ (p « ~q) is not a tautology as all
involved in statement 1, is as follows : entries in its column are not T.
\ Statement-1 is true but statement-2 is
p q : q p « : q : ( p « : q) p « q
false.
T T F F T T
4. (b) P : there is a rational number x Î S such
T F T T F F
that x > 0
F T F T F F ~ P : Every rational number x Î S satisfies
F F T F T T
x£0
We observe the columns for ~ (p « ~q) 5. (a) Suman is brilliant and dishonest if and only
and p « q are identical, therefore if Suman is rich is expressed as
~(p « ~q) is equivalent to p « q Q « ( P Ù ~ R)
Negation of it will be ~ (Q « ( PÙ ~ R))
6. (c)
A B A Ú B A Ù B A Ù (A Ú B) A Ú (A Ù B) A ® B A Ù (A ® B) [A Ù (A ® B) ®B] [B ® [A Ù (A ®B)]
T F T F T T F F T T
F T T F F F T F T F
T T T T T T T T T T
F F F F F F T F T T

\ It is tautology.
7. (a) Let p : I become a teacher. Clearly equivalent to p « q
q : I will open a school
Negation of p ® q is ~ (p ® q) = p ^ ~q 10. (b) :[:sÚ(:r Ù s)]
i.e. I will become a teacher and I will not = sÙ:(:r Ù s)
open a school. = sÙ(r Ú:s)
8. (b) Statement-2 : (p ® q) « (~q ® ~p) = (s Ù r) Ú (s Ù: s)
º (p ® q) « (p ® q) = (s Ù r) Ú 0
which is always true. =sÙr
So statement 2 is true
Statement-1: (p ^ ~q) ^ (~p ^ q) 11. (a) (pÙ : q) Ú q Ú (: p Ù q)
= p ^ ~q ^ ~p ^ q
= p ^ ~p ^ ~q ^ q 40. (1) (pÙ : q) Ú q Ú (: p Ù q)
= f^f=f Þ {(p Ú q) Ù (: q Ú q)} Ú (: p Ù q)
So statement-1 is true
Þ {(p Ú q) Ù T} Ú (: p Ù q)
9. (c) p q ~q p « ~ q ~ ( p « ~ q) Þ (p Ú q) Ú (: p Ù q)
F F T F T Þ {(p Ú q) Ú : p} Ù (p Ú q Ú q)
F T F T F Þ TÙ (p Ú q)
T F T T F Þ pÚq
T T F F T

12. (b) We have


p q ~ p p ® q ~ p ® q (~ p ® q) ® q (p ® q) ® ((~p ® q)® q)
T F F F T F T
T T F T T T T
F F T T F T T
F T T T T T T
\ It is tautology.
www.crackjee.xyz

Statistics 14
1. In a class of 100 students there are 70 boys whose 6. If in a frequency distribution, the mean and
average marks in a subject are 75. If the average
median are 21 and 22 respectively, then its mode
marks of the complete class is 72, then what is
is approximately [2005]
the average of the girls? [2002]
(a) 22.0 (b) 20.5
(a) 73 (b) 65
(c) 25.5 (d) 24.0
(c) 68 (d) 74
2. The median of a set of 9 distinct observations 7. Let x1 , x 2 , .............. xn be n observations such
is 20.5. If each of the largest 4 observations of
that å xi = 400 and å xi = 80. Then the
2
the set is increased by 2, then the median of the
new set [2003] possible value of n among the following is
(a) remains the same as that of the original set [2005]
(b) is increased by 2 (a) 15 (b) 18
(c) is decreased by 2 (c) 9 (d) 12
(d) is two times the original median. 8. Suppose a population A has 100 observations
3. In an experiment with 15 observations on x, the 101, 102, ............., 200 and another population B
following results were available: [2003] has 100 obsevrations 151, 152, ................ 250. If
Sx 2 = 2830, Sx = 170 VA and VB represent the variances of the two
One observation that was 20 was found to be VA
wrong and was replaced by the correct value populations, respectively then is [2006]
VB
30. The corrected variance is [2003]
(a) 8.33 (b) 78.00 9
(c) 188.66 (d) 177.33 (a) 1 (b)
4
4. Consider the following statements :
(A) Mode can be computed from histogram 4 2
(B) Median is not independent of change of scale (c) (d)
9 3
(C) Variance is independent of change of origin
and scale. 9. The average marks of boys in class is 52 and
Which of these is / are correct ? [2004] that of girls is 42. The average marks of boys
(a) (A), (B) and (C) (b) only (B) and girls combined is 50. The percentage of
(c) only (A) and (B) (d) only (A) boys in the class is [2007]
5. In a series of 2 n observations, half of them equal (a) 80 (b) 60
a and remaining half equal –a. If the standard (c) 40 (d) 20.
deviation of the observations is 2, then |a| 10. The mean of the numbers a, b, 8, 5, 10 is 6 and
equals. [2004] the variance is 6.80. Then which one of the
2 following gives possible values of a and b?
(a) (b) 2 [2008]
n
(a) a = 0, b = 7 (b) a = 5, b = 2
1
(c) 2 (d) (c) a = 1, b = 6 (d) a = 3, b = 4
n
EBD_7764
www.crackjee.xyz
M-82 Mathematics
DIRECTIONS:This question contains two always under reported every fish weight by 2
statements: statement-1 (Assertion) and gm. The correct mean and standard deviation
statement-2 (Reason). This question also has (in gm) of fishes are respectively : [2011RS]
four alternative choices, only one of which is (a) 32, 2 (b) 32, 4
the correct answer. You have to select the correct (c) 28,2 (d) 28, 4
choice. 16. Let x1 , x2,...., xn be n observations, and let x be their
11. Statement-1 : The variance of first n even natural arithmetic mean and s2 be the variance.
Statement-1 : Variance of 2x1, 2x2, ..., 2xn is 4s2.
n2 –1
numbers is . Statement-2 : Arithmetic mean 2x1, 2x2, ..., 2xn is 4 x .
4
[2012]
Statement-2 : The sum of first n natural numbers (a) Statement-1 is false, Statement-2 is true.
n(n + 1) (b) Statement-1 is true, statement-2 is true;
is and the sum of squares of first n statement-2 is a correct explanation for
2
Statement-1.
natural numbers is n (n + 1)(2n + 1) . [2009] (c) Statement-1 is true, statement-2 is true;
6 statement-2 is not a correct explanation for
(a) Statement-1 is true, Statement-2 is true Statement-1.
(d) Statement-1 is true, statement-2 is false.
Statement-2 is not a correct explanation for 17. All the students of a class performed poorly in
Statement-1. Mathematics. The teacher decided to give grace
(b) Statement-1 is true, Statement-2 is false. marks of 10 to each of the students. Which of
(c) Statement-1 is false, Statement-2 is true. the following statistical measures will not
(d) Statement-1 is true, Statement-2 is true. change even after the grace marks were given ?
Statement-2 is a correct explanation for [2013]
Statement-1. (a) mean (b) median
12. If the mean deviation of the numbers 1, 1 + d, (c) mode (d) variance
1 + 2d, .... 1 + 100d from their mean is 255, then d 18. The variance of first 50 even natural numbers is
is equal to : [2009] [2014]
(a) 20.0 (b) 10.1 437
(a) 437 (b)
(c) 20.2 (d) 10.0 4
13. For two data sets, each of size 5, the variances are 833
(c) (d) 833
given to be 4 and 5 and the corresponding means 4
are given to be 2 and 4, respectively. The variance 19. The mean of the data set comprising of 16
of the combined data set is [2010] observations is 16. If one of the observation
valued 16 is deleted and three new observations
11
(a) (b) 6 valued 3, 4 and 5 are added to the data, then the
2 mean of the resultant data, is: [2015]
13 5 (a) 15.8 (b) 14.0
(c) (d)
2 2 (c) 16.8 (d) 16.0
14. If the mean deviation about the median of the 20. If the standard deviation of the numbers 2, 3, a
numbers a, 2a,.......,50a is 50, then | a | equals [2011] and 11 is 3.5, then which of the following is true?
(a) 3 (b) 4 [2016]
(c) 5 (d) 2 2
(a) 3a – 34a + 91 = 0
15. A scientist is weighing each of 30 fishes. Their
mean weight worked out is 30 gm and a (b) 3a2 – 23a + 44 = 0
standarion deviation of 2 gm. Later, it was found (c) 3a2 – 26a + 55 = 0
that the measuring scale was misaligned and (d) 3a2 – 32a + 84 = 0
www.crackjee.xyz
Statistics M-83

Answer Key
1 2 3 4 5 6 7 8 9 10 11 12 13 14 15
(b) (a) (b) (c) (c) (d) (b) (a) (a) (d) (c) (b) (a) (b) (a)
16 17 18 19 20
(d) (d) (d) (b) (d)

1. (b) Total student = 100; 6. (d) Mode + 2Mean = 3 Median


for 70 students total marks = 75 ´ 70 = 5250 7. (b) We know that for positive real numbers
Þ Total marks of girls = 7200 – 5250 x1, x2, ...., xn,
= 1950 A.M. of kth powers of x'i s ³ kth the
power of A.M. of x'i s
1950
Average of girls = = 65 2 2
30
Þ
å x12 ³ æç å x1 ö÷ Þ
400 æ 80 ö
³ç ÷
th n n ø n è nø
æ 9 +1ö th è
2. (a) n = 9 then median term = ç ÷ =5
è 2 ø Þ n ³ 16 . So only possible value for
term. Last four observations are increased n = 18
by 2. The median is 5th observation which Þ Mode = 3 × 22 – 2 × 21 = 66 – 42 = 24.
is remaining unchanged.
\ there will be no change in median.
8. (a) s 2x =
å di2
(Here deviations are taken
n
3. (b) Sx = 170, Sx 2 = 2830 increase in
from the mean). Since A and B both have
Sx = 10 , then 100 consecutive integers, therefore both
Sx' = 170 + 10 = 180 have same standard deviation and hence
Increase in Sx 2 = 900 - 400 = 500 then the variance. \ V A = 1
2
Sx' = 2830 + 500 = 3330 VB

1 2 æ1 ö
2 (As å di2 is same in both the cases)
Variance = Sx' -ç Sx' ÷
n èn ø 9. (a) Let the number of boys be x and that of
2 girls be y.
1 æ1 ö Þ 52x + 42y = 50(x + y)
= ´ 3330 - ç ´ 180 ÷ = 222 - 144 = 78.
15 è 15 ø Þ 52x – 50x = 50y – 42y
4. (c) Only first (A) and second (B) statements
x 4 x 4
are correct. Þ 2x = 8y Þ = and =
5. (c) Clearly mean A = 0 y 1 x+ y 5

Standard deviation s = å ( x - A)2 Required % of boys =


x
´ 100 =
2n x+ y

( a - 0)2 + ( a - 0) 2 + ...(0 - a ) 2 + ... 4


2= ´ 100 = 80 %
2n 5
10. (d) Mean of a, b, 8, 5, 10 is 6
a 2 .2 n
= =| a | a + b + 8 + 5 + 10
2n Þ = 10
Hence | a | = 2 5
Þ a+b=7 ...(i)
EBD_7764
www.crackjee.xyz
M-84 Mathematics
Variance of a, b, 8, 5, 10 is 6.80 101´ 255
Þ d= = 10.1
(a - 6)2 + (b – 6) 2 + (8 – 6) 2 + (5 – 6) 2 + (10 – 6) 2 50 ´ 51
Þ
5
= 6.80 13. (a) s 2x = 4, s 2y = 5, x = 2, y = 4

Þ a 2 –12a + 36 + (1 – a ) 2 + 21 = 34 1 1
5
å xi2 - (2)2 = 4; å yi2 - (4)2 = 5
5
[using eq. (i)]
Þ 2a2 –14a + 24 = 0 Þ a2 – 7a + 12 = 0 å xi2 = 40; å yi2 = 105
Þ a = 3 or 4 Þ b = 4 or 3
Þ å ( xi + yi ) = 145
2 2
\ The possible values of a and b are a
= 3 and
b= 4 Þ å ( xi + yi ) = 5(2) + 5(4) = 30
or, a = 4 and b = 3 Variance of combined data
11. (c) For the numbers 2, 4, 6, 8, ......., 2n 2

x =
2[ n ( n + 1)]
= (n + 1) =
1
10
( )
æ1 ö
å xi2 + yi2 - çè 10 å ( xi + yi ) ÷ø
2n
S ( x – x )2 S x2 =
145
-9 =
11
And Var = = – ( x )2
2n n 10 2
14. (b) Median is the mean of 25th and 26th
4S n 2 observation
== – (n + 1)2
n 25a + 26a
\ M = = 25.5a
4n (n + 1) (2n + 1) 2
= – (n + 1)2
6n å xi - M
2(2n + 1) ( n + 1) M .D (M ) =
= – ( n + 1)2 N
3
é 4n + 2 – 3n – 3 ù 1
= (n + 1) ê Þ 50 = [2 ´ a ´ (0.5 + 1.5 + 2.5 + ....24.5)]
ë 3 úû 50
25
=
( n + 1)( n –1) = n2 - 1 Þ 2500 = 2 a ´
2
(25)
3 3 Þ a =4
\ Statement-1 is false. Clearly, statement -
2 is true. 15. (a) Correct mean = observed mean + 2 = 30 + 2
= 32
101 + d(1 + 2 + 3 + ......+100) Correct S. D. = observed S.D. = 2
12. (b) Mean =
101 16. (d) A.M. of 2x1, 2x2, ..., 2xn is
d × 100 × 101 2 x1 + 2x2 + ... + 2xn
=1+ =1 + 50 d
101 × 2 n
Q Mean deviation from the mean = 255
æ x + x2 + .... + xn ö
Þ 1 =2ç 1 ÷ = 2x
[| 1 - (1 + 50d ) | + | (1 + d ) - (1 + 50 d ) | è n ø
101
æ sum of observations ö
+ | (1 + 2d )-(1 + 50d ) | +....+ | (1 + 100 d ) - (1 + 50 d ) |]= çQ Mean = ÷
255 è Number of observations ø
So statement-2 is false.
Þ 2d [1 + 2 + 3 + ... + 50] = 101´ 255
variance (2xi) = 22 variance (xi) = 4s2
50 ´ 51 where i = 1, 2,......n
Þ 2d ´ = 101´ 255
2 So statement-1 is true.
www.crackjee.xyz
Statistics M-85
17. (d) If initially all marks were xi then
æ 50 ´ 51 ´ 101ö
= 4ç - (51)2 = 3434 – 2601
å ( xi - x )2 è 50 ´ 6 ÷ø
s12 = i
Þ s2 = 833
N
19. (b) Sum of 16 observations = 16 × 16 = 256
Now each is increased by 10 Sum of resultant 18 observations
= 256 – 16 + (3 + 4+5) = 252
å ( xi - x )
2
å [ ( xi +10) -( x +10) ]2
i
= i 252
s12 = = s12 Mean of observations = = 14
N N 18
Hence, variance will not change even after 2 + 3 + a + 11 a
the grace marks were given. 20. (d) x = = +4
4 4
18. (d) First 50 even natural numbers are 2, 4 , 6
....., 100 x i2
( )
2
s= å - x
Variance =
å xi2
- ( x) 2
n
N 2
4 + 9 + a 2 +121 æ a ö
2 2 2 Þ3.5= - çç + 4 ÷÷
2 + 4 + ... + 100 4 è4 ø
Þ s2 =
50
2 49 4(134 + a 2 ) - (a 2 + 256 + 32a)
æ 2 + 4 + ... + 100 ö Þ =
-ç ÷ø 4 16
è 50
Þ 3a 2 - 32a + 84 = 0
2 2 2 2
4(1 + 2 + 3 + .... + 50 )
= - (51)2
50
EBD_7764
www.crackjee.xyz
M-86 Mathematics

Probability 15
1. A and B are events such that P(A È B)=3/4, (a) equally likely and mutually exclusive
P(A Ç B)=1/4, P( A ) =2/3 then P ( A Ç B) is (b) equally likely but not independent
[2002] (c) independent but not equally likely
(a) 5/12 (b) 3/8 (d) mutually exclusive and independent
(c) 5/8 (d) 1/4 5. A die is thrown. Let A be the event that the
2. Events A, B, C are mutually exclusive events number obtained is greater than 3. Let B be the
event that the number obtained is less than 5.
3x + 1 1- x Then P(AÈB) is [2008]
such that P ( A) = , P( B) = and
3 4
3
(a) (b) 0
1 - 2x 5
P (C ) = The set of possible values of x
2 2
are in the interval. [2003] (c) 1 (d)
5
é1 1 ù 6. Four numbers are chosen at random (without
(a) [0 , 1] (b) ê 3 , 2 ú replacement) from the set {1, 2, 3, ...20}.
ë û
[2010]
é1 2 ù é 1 13 ù
(c) ê 3 , 3 ú (d) ê 3 , 3 ú Statement -1: The probability that the chosen
ë û ë û
numbers when arranged in some order will form
3. Five horses are in a race. Mr. A selects two of
the horses at random and bets on them. The 1
an AP is .
probability that Mr. A selected the winning horse 85
is [2003] Statement -2 : If the four chosen numbers form
2 4 an AP, then the set of all possible values of
(a) (b)
5 5 common difference is (±1, ±2, ±3, ±4, ±5) .
3 1 (a) Statement -1 is true, Statement -2 is true ;
(c) (d) Statement -2 is not a correct explanation for
5 5
4. Let A and B be two events such that Statement -1
(b) Statement -1 is true, Statment -2 is false
1 1 1
P ( A È B ) = , P ( A Ç B ) = and P ( A) = , (c) Statement -1 is false, Statment -2 is true.
6 4 4
(d) Statement -1 is true, Statement -2 is true ;
where A stands for complement of event A. Statement -2 is a correct explanation for
Then events A and B are [2005] Statement -1.
www.crackjee.xyz
Probability M-87
7. Assuming the balls to be identical except for 1
difference in colours, the number of ways in = P(Exactly one of C or A occurs) = and
4
which one or more balls can be selected from 10 1
white, 9 green and 7 black balls is : [2012] P(All the three events occur simultaneously) = .
16
(a) 880 (b) 629 Then the probability that at least one of the
events occurs, is : [2017]
(c) 630 (d) 879
3 7
8. For three events A, B and C, (a) (b)
P(Exactly one of A or B occurs) 16 32
= P(Exactly one of B or C occurs) 7 7
(c) (d)
16 64

Answer Key
1 2 3 4 5 6 7 8
(a) (b) (a) (c) (c) (b) (d) (c)

P ( A È B È C ) = P ( A) + P ( B ) + P ( C )
1. (a) P (A È B) = P (A) + P (B) – P (A Ç B);
3x + 1 1 - x 1 - 2 x
3 1 ÞP ( A È B È C ) = + +
Þ =1 – P( A ) + P(B) – 3 4 2
4 4
1 + 3x 1 - x 1 - 2 x
2 2 \ 0£ + + £1
Þ 1=1– + P(B) Þ P(B) = ; 3 4 2
3 3
0 £ 13 - 3x £ 12 Þ 1 £ 3 x £ 13
Now, P( A Ç B ) = P(B) – P ( A Ç B )
1 13
2 1 5 Þ £x£
= – = . 3 3
3 4 12
Considering all inequations, we get
2. (b) 3x + 1 1- x
P ( A) = , P( B) = , ì 1 1 1ü ì 2 1 13 ü
3 4 maxí- ,-3,- , ý £ x £ min í ,1, , ý
î 3 2 3þ î3 2 3 þ
1 - 2x
P (C ) = 1 1 é1 1 ù
2 £ x £ Þ xÎê , ú
3 2 ë3 2 û
Q For any event E , 0 £ P ( E ) £ 1
3. (a) Let 5 horses are H1, H2, H3, H4 and H5. Selected
Þ 0 £ 3 x + 1 £ 1, 0 £ 1 - x £ 1 pair of horses will be one of the 10 pairs (i.e.;
3 4
5
C2 ): H1 H2, H1 H3, H1 H4, H1 H5, H2H3, H2
1 - 2x
and 0 £ £1 H4, H2 H5, H3 H4, H3 H5 and H4 H5.
2
Þ -1 £ 3x £ 2, - 3 £ x £ 1 and - 1 £ 2 x £ 1 Any horse can win the race in 4 ways.
For example : Horses H2 win the race in 4
1 2 ways H1 H2, H2H3, H2H4 and H2H5.
Þ - £ x £ £ -3 £ x £ 1, and
3 3
4 2
1 1 Hence required probability = =
- £x£ 10 5
2 2
Also for mutually exclusive events A, B, C,
EBD_7764
w w w . c r a c k j e e . x y
M-88 Mathematics

4. (c)
1 1 17 + 14 + 11 + 8 + 5 + 2 1
P ( A È B) = , P ( A Ç B ) = and Prob. = =
6 4 20 85
C4
1 Statement -2 is false, because common
P ( A) =
4 difference can be 6 also.
5 3 7. (d) Number of white balls = 10
Þ P ( A È B ) = , P( A) = Number of green balls = 9
6 4
Also and Number of black balls = 7
\ Required probability
Þ P ( A È B ) = P ( A) + P ( B ) - P ( A Ç B )
= (10 + 1) (9 + 1) (7 + 1) – 1
5 3 1 1 = 11.10.8 –1 = 879
Þ P( B) = - + =
6 4 4 3 [Q The total number of ways of selecting
3 1 1 one or more items from p identical items of
Þ P ( A) P ( B ) =
- = = P( A Ç B) one kind, q identical items of second kind;
4 3 4
r identical items of third kind is
Hence A and B are independent but not
equally likely. (p + 1) (q + 1) (r + 1) –1 ]
5. (c) A º number is greater than 3 64
=
3 1 127
Þ P( A) = = 8. (c) P (exactly one of A or B occurs)
6 2
B º number is less than 5 1
= P(A) + P (B) – 2P (A Ç B) = ...(1)
4 2 4
Þ P( B) = = P (Exactly one of B or C occurs)
6 3
A Ç B º number is greater than 3 but 1
less than 5. = P(B) + P (C) – 2P (B Ç C) = ...(2)
4
1 P (Exactly one of C or A occurs)
Þ P( A Ç B) =
6 1
\ P(A È B) = P(A) + P(B) – P(A Ç B) = P(C) + P(A) – 2P (C Ç A) = ...(3)
4
1 2 1 3 + 4 –1 Adding (1), (2) and (3),we get
= + – = =1
2 3 6 6 3
6. (b) n(S) = 20C4 2SP(A) – 2SP (A Ç B) =
4
Statement-1:
common difference is 1; total number of 3
\ SP(A) – SP (A Ç B) =
cases = 17 8
common difference is 2; total number of 1
cases = 14 Now, P (A Ç B Ç C) =
common difference is 3; total number of 16
cases = 11 \ P (A È B È C)
common difference is 4; total number of = SP (A) – SP (A Ç B) + P (A Ç B Ç C)
cases = 8 3 1 7
common difference is 5; total number of = + =
8 16 16
cases = 5
common difference is 6; total number of
cases = 2
www.crackjee.xyz

Relations and Functions 16


1. The period of sin 2 q is [2002] interval of S is [2004]
(a) [ –1, 3] (b) [–1, 1]
(a) p 2 (b) p
(c) [ 0, 1] (d) [0, 3]
(c) 2 p (d) p /2
2. Which one is not periodic? [2002] 6. Let R = {(1,3), (4, 2), (2, 4), (2, 3), (3,1)} be a
(a) | sin3x | +sin 2x relation on the set A = {1, 2,3, 4}. . The relation
(b) cos x + cos2x R is [2004]
(c) cos 4x + tan2x (a) reflexive (b) transitive
(d) cos2x + sinx (c) not symmetric (d) a function
7. Let f : (– 1, 1) ® B, be a function defined by
3. ( 2
The function f ( x) = log x + x + 1 , is ) f (x) = tan
-1 2x
, then f is both one - one and
[2003] 1 - x2
(a) neither an even nor an odd function onto when B is the interval [2005]
(b) an even function é pö
æ pö
(c) an odd function (a) ç 0, ÷ (b) ê0, ÷
è 2ø ë 2ø
(d) a periodic function.
4. A function f from the set of natural numbers to é p pù æ p pö
(c) (d) ç- , ÷
integers defined by [2003] ê- 2 , 2 ú è 2 2ø
ë û
8. A real valued function f (x) satisfies the
ìn -1 functional equation
ïï 2 , when n is odd
=
f (n) í is f (x – y) = f (x) f (y) – f (a – x) f (a + y)
ï - n , when n is even where a is a given constant and f (0) = 1,
ïî 2
f (2a – x) is equal to [2005]
(a) neither one -one nor onto (a) – f (x) (b) f (x)
(c) f (a) + f (a – x) (d) f (– x)
(b) one-one but not onto
9. Let R = {(3, 3), (6, 6), (9, 9), (12, 12), (6, 12), (3, 9),
(c) onto but not one-one
(3, 12), (3, 6)} be a relation on the set
(d) one-one and onto both. A = {3, 6, 9, 12}. The relation is [2005]
5. If f : R ® S , defined by (a) reflexive and transitive only
(b) reflexive only
f ( x) = sin x - 3 cos x + 1, is onto, then the (c) an equivalence relation
(d) reflexive and symmetric only
EBD_7764
www.crackjee.xyz
M-90 Mathematics
10. Let W denote the words in the English dictio- Statement-2 is not a correct explanation for
nary. Define the relation R by R = {(x, y) Î W × W| Statement-1.
the words x and y have at least one letter in 14. For real x, let f (x) = x3 + 5x + 1, then [2009]
common.} Then R is [2006] (a) f is onto R but not one-one
(a) not reflexive, symmetric and transitive (b) f is one-one and onto R
(b) relexive, symmetric and not transitive (c) f is neither one-one nor onto R
(c) reflexive, symmetric and transitive (d) f is one-one but not onto R
(d) reflexive, not symmetric and transitive 15. Consider the following relations:
11. Let f: N®Y be a function defined as f(x) = 4x + 3 R = {(x, y) | x, y are real numbers and x = wy for
where Y = {y Î N : y = 4x + 3 for some x Î N}. some rational number w};
Show that f is invertible and its inverse is æ m pö
[2008] S = {ç , ÷ | m,n, p and q are integers such
è n qø
3y + 4 y+3 that n, q ¹ 0 and qm = pn}. Then [2010]
(a) g ( y) = (b) g ( y ) = 4 +
3 4 (a) Neither R nor S is an equivalence relation
(b) S is an equivalence relation but R is not an
y+3 y –3
(c) g ( y) = (d) g ( y ) = equivalence relation
4 4 (c) R and S both are equivalence relations
12. Let R be the real line. Consider the following (d) R is an equivalence relation but S is not an
subsets of the plane R × R: equivalence relation
S ={(x, y): y = x + 1 and 0 < x < 2} 16. Let R be the set of real numbers. [2011]
T ={(x, y): x – y is an integer}, Statement-1: A = {(x, y) Î R × R : y – x is an
Which one of the following is true? [2008] integer} is an equivalence relation on R.
(a) Neither S nor T is an equivalence relation on R Statement-2: B = {(x, y) Î R × R : x = ay for some
(b) Both S and T are equivalence relation on R rational number a} is an equivalence relation on R.
(c) S is an equivalence relation on R but T is not (a) Statement-1 is true, Statement-2 is true;
(d) T is an equivalence relation on R but S is not Statement-2 is not a correct explanation for
13. DIRECTIONS : This question contains two Statement-1.
statements: (b) Statement-1 is true, Statement-2 is false.
Statement-1 (Assertion ) and Statement-2 (c) Statement-1 is false, Statement-2 is true.
(Reason). (d) Statement-1 is true, Statement-2 is true;
This question also has four alternative choices, Statement-2 is a correct explanation for
only one of which is the correct answer. You Statement-1.
have to select the correct choice. 17. Let f be a function defined by
2
Let f(x) = ( x + 1) –1, x ³ –1 f ( x) = ( x -1) +1, ( x ³1) .
2
[2011RS]
Statement -1 : The set {x : f(x) = f –1(x) = {0, –1} Statement - 1 :
Statement-2 : f is a bijection. [2009]
(a) Statement-1 is true, Statement-2 is true. { }
The set x : f ( x) = f ( x) = {1, 2} .
-1

Statement-2 is not a correct explanation for


Statement - 2:
Statement-1.
(b) Statement-1 is true, Statement-2 is false. f is a bijection and f -1 ( x ) = 1 + x - 1, x ³ 1.
(c) Statement-1 is false, Statement-2 is true.
(d) Statement-1 is true, Statement-2 is true. (a) Statement-1 is true, Statement-2 is true;
Statement-2 is a correct explanation for
Statement-1.
www.crackjee.xyz
Relations and Functions M-91
(b) Statement-1 is true, Statement-2 is true; (c) 1 + x5 (d) 5x4
Statement-2 is NOT a correct explanation for é 1 1ù
Statement-1. 19. The function f : R ® ê - , ú defined as f(x) =
ë 2 2û
(c) Statement-1 is true, Statement-2 is false. x
, is : [2017]
(d) Statement-1 is false, Statement-2 is true. 1 + x2
18. If g is the inverse of a function f and (a) neither injective nor surjective
(b) invertible
1
f '( x ) = , then g ¢ ( x ) is equal to:[2014] (c) injective but not surjective
1 + x5 (d) surjective but not injective
1
(b) 1 + { g ( x )}
5
(a)
1 + { g ( x )}
5

Answer Key
1 2 3 4 5 6 7 8 9 10 11 12 13 14 15
(b) (b) (c) (d) (a) (c) (d) (a) (a) (b) (d) (d) (b) (b) (b)
16 17 18 19
(a) (a) (b) (d)

\ f is onto.
21 - cos 2q 2p Also each negative integer is an image of
1. (b) sin q = ; Period = =p
2 2 even natural number and each positive
integer is an image of odd natural number.
2. (b) Q cos x is non periodic
\ f is onto.
\ cos x + cos 2 x can not be periodic. Hence f is one one and onto both.
5. (a) f ( x) is onto \ S = range of f (x)
3. (c) f ( x ) = log( x + x 2 + 1)
Now f (x) = sin x - 3cos x + 1
{
f( - x ) = log - x + x 2 + 1 } æ pö
= 2sin ç x - ÷ + 1
è 3ø
ìï - x 2 + x 2 + 1üï
= log í æ pö
ý Q -1 £ sin ç x - ÷ £ 1
ïî x + x 2 + 1 ïþ è 3ø
æ pö
= - log( x + x 2 + 1) = - f ( x ) -1 £ 2sin ç x - ÷ + 1 £ 3
è 3ø
Þ f(x) is an odd function. \ f ( x) Î[ -1, 3] = S
4. (d) We have f : N ® I
If x and y are two even natural numbers, We know that
-x - y
then f ( x) = f ( y ) Þ
= Þx=y - a 2 + b2 £ a sin q + b cos q £ a 2 + b2
2 2
Again if x and y are two odd natural \ -2 £ sin x - 3 cos x £ 2
numbers then
x -1 y - 1
f ( x) = f ( y) Þ = Þx=y
2 2
EBD_7764
www.crackjee.xyz
M-92 Mathematics
Also f (x) = 4x + 3 = y
Þ -1 £ sin x - 3 cos x + 1 £ 3
y –3 y –3
\ f ( x) Î[ -1, 3] Þx= \ g ( y) =
4 4
6. (c) Q (1, 1) Ï R Þ R is not reflexive (2,3) Î R 12. (d) Given S = {(x , y) : y = x + 1 and 0 < x < 2}
but (3, 2) Ï R Q x ¹ x + 1 for any x Î(0, 2)
\ R is not symmetric Þ (x, x) Ï S
æ 2x ö \ S is not reflexive.
7. (d) Given f (x) = tan -1 ç = 2tan–1x
è 1 - x 2 ÷ø Hence S in not an equivalence relation.
for x Î (-1, 1) Also T ={x, y): x – y is an integer}
æ -p pö Q x – x = 0 is an integer " x Î R
If x Î( -1, 1) Þ tan -1 x Î ç , ÷ \ T is reflexive.
è 4 4ø
If x – y is an integer then y – x is also an
æ -p p ö \T is symmetric
Þ 2 tan -1 x Î ç , integer
è 2 2 ÷ø If x – y is an integer and y – z is an
æ p pö integer then
Clearly, range of f (x) = ç - , ÷ (x – y) + (y– z) = x – z is also an integer.
è 2 2ø
For f to be onto, codomain = range \ T is transitive
\ Co-domain of function = B = 13. (b) Given that f (x) = (x + 1)2 –1, x ³ –1
Clearly Df = [–1, ¥ ) but co-domain is not
æ p pö
ç- , ÷ . given. Therefore f (x) need not be
è 2 2ø necessarily onto.
8. (a) f (2a – x) = f (a – (x – a)) But if f (x) is onto then as f (x) is one one
= f (a) f (x – a) – f (0) f (x) = f (a) f (x –a) – f (x) also, (x + 1) being something +ve,
= – f (x) f –1(x) will exist where
(x + 1)2 –1 = y
[Q x = 0, y = 0, f (0) = f 2 (0) - f 2 (a)
Þ x +1 = y +1
2
Þ f (a) = 0 Þ f (a) = 0 (+ve square root as x +1 ³ 0 )
Þ f (2a - x) = - f ( x ) Þ x =–1+ y+1
9. (a) Reflexive and transitive only.
e.g. (3, 3), (6, 6), (9, 9), (12, 12) [Reflexive] Þ f –1 (x) = x + 1 – 1
(3, 6), (6, 12), (3, 12) [Transitive]. Then f (x) = f –1 (x)
(3, 6) Î R but (6, 3) Ï R [ non symmetric] Þ (x + 1)2 – 1 = x + 1 –1

10. (b) Clearly ( x, x) Î R"x ÎW. So R is reflexive. Þ (x + 1)2 = x + 1 Þ (x + 1)4 = (x + 1)


Þ (x + 1) [ (x + 1)3 – 1] = 0 Þ x = – 1, 0
Let ( x, y) Î R , then ( y, x) Î R as x and y
\ The statement-1 is correct but
have at least one letter in common. So, R statement-2 is false.
is symmetric. 14. (b) Given that f (x) = x3 + 5x + 1
But R is not transitive for example \f ' (x) = 3x2 + 5 > 0,
Let x = INDIA, y = BOMBAY and z =JOKER
"xÎR
then ( x, y ) Î R (A is common) and
Þ f (x) is strictly increasing on R
( y, z ) Î R (O is common) but ( x, z ) Ï R . (as Þ f (x) is one one
no letter is common) \ Being a polynomial f (x) is continuous
11. (d) Clearly f is one one and onto, so invertible and increasing.
on R with lim f ( x ) = -¥
x ®¥

and xlim f ( x) = ¥
®¥
www.crackjee.xyz
Relations and Functions M-93
\ Range of f = ( - ¥, ¥) = R Hence statement-1 is correct
Hence f is onto also. So, f is one one and 18. (b) Since f (x) and g(x) are inverse of each other
onto R. 1
15. (b) x Ry need not implies yRx \ g'( f (x)) =
f '( x )
m p
S: s
n q Þ g '( f ( x)) = 1 + x5
p m
Given qm = pn Þ = æ 1 ö
q n
çèQ f ¢ ( x) = ÷
m m m p p m 1 + x5 ø
\ s reflexive n s q Þ q s n symmetric
n n Here x = g(y)
m p p r
g ¢( y ) = 1 + [ g ( y )]
s , s 5
qm = pn, ps = rq \
n q q s Þ
p m r Þ g ¢ ( x ) = 1 + ( g ( x) ) 5
Þ q = n = s Þ ms = rn transitive.
S is an equivalence relation. é 1 1ù
19. (d) we have f : R ® ê - , ú ,
16. (a) Let for statement 1: xRy = x – y Î I . As xRx ë 2 2û
is an integer and yRx as well as xRz (for x
xRy and yRz) is also an integer. f (x) = "x Î R
1 + x2
Hence equivalence.
Similarly as x = ay hence a =1 for reflexive (1 + x 2 ).1 - x.2x -(x + 1)(x - 1)
Þ f ¢(x) = 2 2
=
1 (1 + x ) (1 + x 2 ) 2
and being a rational for symmetric for
a – + –
some non zero a and product of rationals
also being rational Þ equivalence x = –1 x=1
But not symmetric because of a = 0 case sign of f¢ (x)
Both relations are equivalence but not the Þ f¢ (x) changes sign in different intervals.
correct explanation. \ Not injective
f ( x ) = ( x - 1) + 1, x ³ 1
2
17. (a) x
Now y =
Since f is a bijective function 1 + x2
Þ y + yx2 = x
\ f :[1, ¥) ® [1, ¥)
Þ yx2 – x + y = 0
Þ y = ( x - 1) + 1 Þ ( x - 1) = y - 1
2 2 For y ¹ 0, D = 1 – 4y2 ³ 0
é -1 1 ù
Þ x = 1± y - 1 Þ f -1 ( y ) = 1 ± y - 1 Þ y Î ê , ú - {0}
ë 2 2û
Þ f -1 ( x ) = 1 + x - 1 {\ x ³ 1} For y = 0 Þ x = 0
Hence statement-2 is correct é -1 1 ù
\ Range is ê , ú
Now f ( x ) = f -1
( x) ë 2 2û
Þ Surjective but not injective
Þ f ( x ) = x Þ ( x - 1) + 1 = x
2

Þ x 2 - 3 x + 2 = 0 Þ x = 1, 2
EBD_7764
www.crackjee.xyz
M-94 Mathematics

Inverse Trigonometric
Functions 17
cot -1 ( cos a ) - tan -1 ( cos a ) = x , æx ö
2
1. f ( x ) = 4- x + cos -1 ç - 1÷ + log (cos x) , is defined, is
è2 ø
then sin x = [2002]
[2007]
æaö æaö
(a) tan2 ç ÷ (b) cot2
ç ÷ é p pö é pö
è2ø è2ø (a) (b) ê 0, 2 ÷
ê- 4 , 2 ÷ ë ø
æaö ë ø
(c) tan a (d) cot ç 2 ÷ p p
è ø (c) [0, p] (d) æç - , ö÷
2. The domain of sin-1 [log 3 (x/3)] is [2002] è 2 2ø
(a) [1, 9] (b) [–1, 9]
æ 5 2ö
(c) [–9, 1] (d) [–9, –1] 8. The value of cot ç cos ec –1 + tan –1 ÷ is
-1 -1
è 3 3ø
3. The trigonometric equation sin x = 2 sin a
6 3
has a solution for [2003] (a) (b)
17 17
1 1 1
(a) a £ (b) < a < 4 5
2 2 2 (c) (d)
17 17
1
(c) all real values of a (d) a < –1 æ 2x ö
2 9.
–1 –1
Let tan y = tan x + tan çè ÷ ,where or
4. The domain of the function 1 - x2 ø
sin -1 ( x - 3) 1
f ( x) = is x < . Then a value of y is : [2015]
9- x 2 3
(a) [1, 2] (b) [2, 3) [2004] 3x - x 3 3x + x 3
(c) [1, 2 ] (d) [2, 3] (a) (b)
y 1 + 3x 2 1 + 3x 2
5. If cos -1 x - cos -1 = a , then 4 x 2 - 4 xy
2 3x – x 3 3x + x 3
(c) (d)
cos a + y 2 is equal to [2005] 1 – 3x 2 1 - 3x 2
(a) 2 sin 2 a (b) 4 æ 1ö
10. If for x Îç 0, ÷ , the derivative of
(c) 4 sin a 2
(d) – 4 sin a 2 è 4ø
æ x ö æ 5 ö p
6. If sin -1 ç ÷ + cosec -1 ç ÷ = , then the æ 6x x ö
è 5ø è 4ø 2 tan -1 ç ÷ is x.g ( x ) , then g(x) equals :
values of x is [2007] è 1 - 9x 3 ø
(a) 4 (b) 5 [2017]
(c) 1 (d) 3.
3 9
(a) (b)
7. The largest interval lying in æç -p , p ö÷ for which 1 + 9x 3
1 + 9x3
è 2 2ø
the function, 3x x 3x
(c) (d)
1 - 9x 3 1 - 9x3
www.crackjee.xyz
Inverse Trigonometric Functions M-95

Answer Key
1 2 3 4 5 6 7 8 9 10
(a) (a) (a) (b) (c) (d) (b) (a) (c) (b)

1
\a £
1. (a) cot–1 ( cos a ) – tan–1 ( cos a ) = x 2
æ 1 ö sin -1 ( x - 3)
tan–1 çç ÷÷ – tan–1 ( cos a ) = x 4. (b) f ( x) = is defined
è cos a ø 9 - x2
1
- cos a if (i) - 1 £ x - 3 £ 1 Þ 2 £ x £ 4
cos a
Þ tan–1 =x and (ii) 9 - x 2 > 0 Þ -3 < x < 3
1
1+ . cos a
cos a Taking common solution of (i) and (ii),
1 - cos a we get 2 £ x < 3 \ Domain = [2, 3)
Þ tan–1 =x y
2 cos a 5. (c) cos - 1 x - cos - 1 = a
2
1 - cos a 2 cos a æ æ ö
Þ tan x = OR cot x = xy y2 ö
2 cos a 1 - cos a cos - 1 ç + (1 - x 2 ) ç1 - ÷ ÷ = a
çè 2 è 4 ø÷
ø
[Considering a D with perpendicular
= (1– cosa) and base = 2 cos a ]
æ xy + 4 - y 2 - 4 x 2 + x 2 y 2 ö
1 - cos a 1 - (1 - 2 sin 2 a / 2) cos -1
ç ÷ =a
Þ sin x = = çè 2 ÷ø
1 + cos a 1 + 2 cos 2 a / 2 - 1
a Þ 4 - y 2 - 4 x2 + x 2 y 2
or sin x = tan2
2
= 4cos 2 a + x 2 y 2 - 4 xy cos a
æ æ xö ö
2. (a) f (x)=sin–1 ç log 3 ç ÷ ÷ exists Þ 4 x 2 + y 2 - 4 xy cosa = 4 sin 2 a .
è è 3ø ø
æ xö æ 5ö p
6. (d) sin -1 ç ÷ + cosec -1 ç ÷ =
æ xö x è 5ø è 4ø 2
if -1 £ log ç ÷ £ 1 Û 3-1 £ £ 31
è 3 ø 3 æ xö p æ 5ö
3
Þ sin -1 ç ÷ = - cosec -1 ç ÷
è 5ø 2 è 4ø
Û 1 £ x £ 9 or x Î [1, 9]
æ xö p æ 4ö
3. (a) sin -1 x = 2 sin -1 a Þ sin -1 ç ÷ = - sin -1 ç ÷
è 5ø 2 è 5ø

p p p p [Q sin -1 x + cos -1 x = p / 2]
- £ sin -1 x £ ; \ - £ 2sin -1 a £
2 2 2 2
æ xö æ 4ö
p p -1 1 Þ sin -1 ç ÷ = cos -1 ç ÷ ....(i)
- £ sin -1 a £ or £a£ è 5ø è 5ø
4 4 2 2
4 4
Let cos -1 = A Þ cos A =
5 5
EBD_7764
www.crackjee.xyz
M-96 Mathematics
C
é –1 17 ù
= cot ê tan
5 ë 6 úû
3
æ –1 6 ö 6
= cot çè cot ÷=
A B
17 ø 17
4
Þ A = cos–1 (4/5) é 2x ù
9. (c) tan–1 y = tan -1 x + tan -1 ê ú
3 ë1 - x 2 û
Þ sin A =
5
= tan -1 x + 2 tan -1 x = 3 tan–1x
3
Þ A = sin –1
5 é 3ù
-1 3x - x
\ cos (4/5) = sin–1 (3/5)
–1 tan–1 y = tan ê 2ú
\ equation (i) become, ëê 1 - 3x ûú

x 3 x 3
sin -1 = sin -1 Þ = Þ x= 3x - x 3
5 5 5 5 Þ y=
3 1 - 3x 2
2 æx ö æ 6x x ö
7. (b) f (x) = 4- x + cos -1 ç - 1÷ + log(cos x) Let F(x) = tan –1 çç
è2 ø 10. (b) 3 ÷÷ where x
è 1 - 9x ø
æx ö æ 1ö
f (x) is defined if – 1 £ ç - 1÷ £ 1 and Î ç 0, ÷ .
è2 ø è 4ø
cos x > 0
æ 2.(3x3 / 2 ) ö
x p p = tan–1 çç ÷ = 2 tan–1 (3x3/2)
or 0 £ £ 2 and - < x < 3/ 2 2 ÷
2 2 2 è 1 - (3x ) ø
p p æ 3ö
or 0 £ x £ 4 and - < x< As 3x3/2 Î ç 0, ÷
2 2 è 8ø
é pö
\ x Î ê 0, ÷ é 1 3/ 2 1 3ù
ë 2ø êQ 0 < x < 4 Þ 0 < x < Þ 0 < 3x 3/ 2 < ú
ë 8 8û
æ 5 2ö
8. (a) cot ç cos ec –1 + tan –1 ÷ = dF(x) 1 3
è 3 3ø So =2× 3 × 3 × × x1/2
dx 1 + 9x 2
é 3 2ù
cot ê tan –1 + tan –1 ú 9
ë 4 3û = x
1 + 9x 3
é æ 3 2 öù On comparing
ê –1 ç 4 + 3 ÷ ú
= cot ê tan ç ú 9
ê 3 2÷ \ g(x) =
ç1– ´ ÷ ú 1 + 9x 3
êë è 4 3 ø úû
www.crackjee.xyz
Matrices M-97

10-13 delet

Matrices 18
éa b ù
If A = ê 2 éa b ù of B¢s such that AB = BA
1.
b a ú and A = ê b a ú , then (c) there exists exactly one B such that AB = BA
ë û ë û
(d) there exist infinitely many B¢s such that
[2003]
AB = BA
2
(a) a = 2 ab, b = a + b 2
5. The number of 3 × 3 non-singular matrices, with
2 2 four entries as 1 and all other entries as 0, is
(b) a = a + b , b = ab
[2010]
(c) a = a 2 + b 2 , b = 2ab
(a) 5 (b) 6
(d) a = a 2 + b 2 , b = a 2 - b 2 . (c) at least 7 (d) less than 4
6. Let A and B be two symmetric matrices of order 3.
é1 0ù é1 0 ù
2. If A = ê ú and I = ê ú , then which one of [2011]
ë1 1 û ë0 1û Statement-1: A(BA) and (AB)A are symmetric
the following holds for all n ³ 1, by the principle matrices.
of mathematical induction [2005] Statement-2: AB is symmetric matrix if matrix
(a) An = nA – (n – 1) I multiplication of A with B is commutative.
(a) Statement-1 is true, Statement-2 is true;
(b) An = 2n - 1 A – (n – 1) I Statement-2 is not a correct explanation for
(c) An = nA + (n – 1) I Statement-1.
n (b) Statement-1 is true, Statement-2 is false.
(d) A = 2n - 1 A + (n – 1) I
(c) Statement-1 is false, Statement-2 is true.
3. If A and B are square matrices of size n × n such
(d) Statement-1 is true, Statement-2 is true;
that A2 - B 2 = ( A - B)( A + B) , then which of Statement-2 is a correct explanation for
the following will be always true? [2006] Statement-1.
(a) A = B 7. If w ¹ 1is the complex cube root of unity and
(b) AB = BA
éω 0ù
(c) either of A or B is a zero matrix matrix H = ê ú , then H70 is equal to
(d) either of A or B is identity matrix ë0 ωû
[2011RS]
æ 1 2ö æ a 0ö
4. Let A = ç ÷ and B = ç , a, b Î N. (a) 0 (b) –H
è 3 4ø è 0 b÷ø
(c) H2 (d) H
Then [2006]
(a) there cannot exist any B such that AB = BA
(b) there exist more than one but finite number
EBD_7764
www.crackjee.xyz
M -9 8 Mathematics
+ b is equal to : [2016]
é1 2 2 ù (a) 4 (b) 13 (c) –1 (d) 5
ê ú
8. If A = ê 2 1 -2ú is a matrix satisfying the é 2 -3ù
10. If A = ê 2
êë a 2 b úû ú , then adj (3A + 12A) is equal to
ë -4 1 û
equation AAT = 9I, where I is 3 × 3 identity : [2017]
matrix, then the ordered pair (a, b) is equal to: é 72 -63ù é 72 -84 ù
[2015] (a) ê ú (b) ê ú
ë -84 51 û ë -63 51 û
(a) (2, 1) (b) (–2, – 1)
(c) (2, – 1) (d) (–2, 1) é 51 63ù é 51 84 ù
(c) ê ú (d) ê ú
é5a - b ù ë84 72 û ë 63 72 û
9. If A = ê ú and A adj A = A AT, then 5a
ë 3 2 úû
ê

Answer Key
1 2 3 4 5 6 7 8 9 10
(c) (a) (b) (d) (c) (a) (d) (b) (d) (c)

3. (b) A2 - B 2 = ( A - B )( A + B )
éa b ù é a b ù é a b ù
1. (c) A2 = ê ú=ê úê ú
ë b a û ë b a û ëb a û A2 - B 2 = A2 + AB - BA - B 2
éa 2 + b 2 2ab ù Þ AB = BA
=ê ú
êë 2ab a 2 + b2 úû é1 2ù éa 0ù
A= ê B = ê
4 úû b úû
4. (d)
a = a 2 + b 2 ; b = 2ab ë3 ë0

2. (a) We observe that é a 2b ù


AB = ê ú
é 1 0 ù 3 é1 0 ù ë3a 4b û
A2 = ê ú , A = ê3 1 ú and we can
ë2 1 û ë û é a 0 ù é1 2 ù é a 2 a ù
BA = ê úê ú=ê ú
é 1 0ù ë 0 b û ë3 4û ë3b 4b û
prove by induction that An = ê ú
ë n 1û Hence, AB = BA only when a = b
é n 0ù é n - 1 0 ù \ There can be infinitely many B¢s for
Now nA - ( n - 1) I = ê n n ú - ê 0 n - 1úû
ë û ë which AB = BA
é 1 0ù n
= ê n 1ú = A é 1 ... ...ù
ë û ê... 1 ...ú
5. (c) ê ú are 6 non-singular matrices
\ nA - ( n - 1) I = An êë... ... 1 úû
www.crackjee.xyz
Matrices M-99

because 6 blanks will be filled by 5 zeros é 1+ 4 + 4 2+ 2- 4 a + 4 + 2b ù é9 0 0ù


ê ú
and 1 one.
Þê 2 + 2 - 4 4 +1+ 4 2a + 2 - 2b ú = ê0 9 0ú
ê ú
ê 2 2 ú ê0 0 9 ú
é... ... 1 ù ëa + 4 + 2b 2a + 2 - 2b a + 4 + b û ë û
Similarly, ê... 1 ...ú are 6 non-singular
ê ú Þ a + 4 + 2b = 0 Þ a + 2b = – 4 ...(i)
êë 1 ... ...úû 2a + 2 – 2b = 0 Þ 2a – 2b = – 2
Þ a – b = –1 ...(ii)
matrices.
So, required cases are more than 7, non- On solving (i) and (ii) we get
singular 3 × 3 matrices. – 1 + b + 2b = –4 ...(i)
6. (a) \ A¢ = A – 1 + 3b = – 4
B¢ = B 3b = – 3
Now (A(BA))¢ = (BA)¢A¢ b = –1
and a = – 2
= (A¢B¢)A¢ = (AB)A = A(BA)
(a, b) = (–2, –1)
Similarly ((AB)A)¢ = (AB)A
So, A(BA) and (AB)A are symmetric 9. (d) A(adj A) = A AT
matrices.
Þ A–1A (adj A) = A–1A AT
Again (AB)¢ = B¢A¢ = BA
Now if BA = AB, then AB is symmetric adj A = AT
matrix. é 2 b ù é5a 3 ù
Þê ú=ê ú
êë-3 5a úû êë- b 2 úû
éω 0 ù é ω 0 ù éω 0 ù
2
7. (d) H2 = ê = ê ú 2
úê ú Þ a = and b = 3
ë0 ωû ë 0 ωû êë0 ω2 úû 5
Þ 5a + b = 5
éωk 0ù
k
If H =ê ú then Hk + é 2 -3ù
êë0 ω úû 10. (c) We have A = ê -4 1 ú
ë û
éωk +1 0 ù é 16 -9 ù
1 =ê ú Þ A2 = ê ú
êë0 ωk +1 úû ë -12 13 û
So by principle of mathematical induction, é 48 -27 ù
Þ 3A2 = ê -36 39 ú
éω 70
0 ù éω w 0 ù éω
69
0ù ë û
H 70 = ê ú =ê ú =ê ú=H
êë0 ω70 úû êë0 w 69ωúû ë0 ωû é 24 -36 ù
Also 12A = ê ú
ë -48 12 û
é 1 2 2 ù é 1 2 a ù é 9 0 0ù \ 3A2 + 12A
ê 2 1 -2ú ê2 1 2ú = ê0 9 0ú
8. (b) ê úê ú ê ú é 48 -27 ù é 24 -36 ù é 72 -63ù
= ê -36 +
ëê a 2 b ûú êë2 -2 b ûú ëê0 0 9 ûú ë 39 úû êë -48 12 úû = êë -84 51 úû

é 51 63ù
adj (3A2 + 12A) = ê ú
ë84 72 û
EBD_7764
www.crackjee.xyz
M-100 Mathematics
10-13 delet

Determinants 19
1. If a > 0 and discriminant of ax2+2bx+c is –ve,
(a) A2 = I
a ax + b
b
(b) A = (–1) I, where I is a unit matrix
then b bx + c is equal to
c
-1
ax + b bx + c 0 (c) A does not exist
(d) A is a zero matrix
[2002]
(a) +ve æ 1 -1 1 ö æ 4 2 2ö
(b) (ac-b2)(ax2+2bx+c) 5. ç ÷
Let A = 2 1 -3 . and B = ç -5 0 a ÷ .
ç ÷ ç ÷
(c) –ve è1 1 1 ø è 1 -2 3 ø
(d) 0
2. If the system of linear equations [2003] If B is the inverse of matrix A, then a is [2004]
(a) 5 (b) –1
x + 2ay + az = 0 ; x + 3by + bz = 0 ; (c) 2 (d) –2
x + 4cy + cz = 0 has a non - zero solution, then 6. If a1, a2 , a3 ,......, an ,.... are in G.P., then the value
a, b, c. of the determinant [2004]
(a) satisfy a + 2b + 3c = 0 (b) are in A.P
log an log an+1 log an+ 2
(c) are in G..P (d) are in H.P.
log an+ 3 log an+ 4 log an +5
, is
3. If 1, w, w 2 are the cube roots of unity, then log an+ 6 log an+ 7 log an+8

1 wn w 2n (a) –2 (b) 1
(c) 2 (d) 0
D = wn w2n 1 is equal to [2003] 7. The system of equations
2n n
w 1 w ax+y +z= a –1
x + a y+ z = a – 1
(a) w 2 (b) 0 x+ y+ az = a –1
has infinite solutions, if a is [2005]
(c) 1 (d) w
(a) – 2 (b) either – 2 or 1
æ 0 0 -1ö (c) not – 2 (d) 1
4. Let A = ç 0 -1 0 ÷ . The only correct 8.
2 2 2
If a + b + c = – 2 and [2005]
ç ÷
è -1 0 0 ø
1 + a2 x (1 + b 2 ) x (1 + c 2 ) x
statement about the matrix A is [2004]
2 2 2
f (x) = (1 + a ) x 1 + b x (1 + c ) x ,
(1 + a2 ) x (1 + b 2 ) x 1 + c 2 x
www.crackjee.xyz
Determinants M-101
then f (x) is a polynomial of degree for Statement-1
(a) 1 (b) 0
(c) 3 (d) 2 (d) Statement -1 is true, Statement-2 is false
9. If a1 , a2 , a3 , ............, an , ...... are in G. P., then 14. Let a, b, c be any real numbers. Suppose that
the determinant there are real numbers x, y, z not all zero such
that x = cy + bz, y = az + cx, and z = bx + ay. Then
log an log an + 1 log an + 2 a2 + b2 + c2 + 2abc is equal to [2008]
D = log an + 3 log an + 4 log an + 5 (a) 2 (b) –1
log an + 6 log an + 7 log an + 8 (c) 0 (d) 1
is equal to [2005] 15. Let A be a square matrix all of whose entries are
(a) 1 (b) 0 integers. Then which one of the following is true?
(c) 4 (d) 2 [2008]
2 (a) If det A = ± 1, then A–1 exists but all its
10. If A – A + I = 0 , then the inverse of A is
[2005] entries are not necessarily integers
(a) A + I (b) A (b) If det A ¹ ± 1, then A–1 exists and all its
(c) A – I (d) I – A entries are non integers
1 1 1 (c) If det A = ± 1, then A–1 exists but all its
11. If D = 1 1 + x 1 for x ¹ 0, y ¹ 0 , then D entries are integers
1 1 1+ y (d) If det A = ± 1, then A–1 need not exists
is [2007] 16. Let A be a 2 × 2 matrix
(a) divisible by x but not y Statement -1 : adj (adj A) = A
(b) divisible by y but not x
(c) divisible by neither x nor y Statement -2 : |adj A |= |A| [2009]
(d) divisible by both x and y (a) Statement-1 is true, Statement-2 is true.
5 5a a Statement-2 is not a correct explanation for
2
12. Let A = 0 a 5a . If A = 25 , then a Statement-1.
0 0 5
(b) Statement-1 is true, Statement-2 is false.
equals [2007]
(c) Statement -1 is false, Statement-2 is true.
(a) 1/5 (b) 5
(d) Statement-1 is true, Statement -2 is true.
(c) 52 (d) 1
Statement-2 is a correct explanation for
13. Let A be a 2 × 2 matrix with real entries. Let I be
Statement-1.
the 2 × 2 identity matrix. Denote by tr(A), the sum
of diagonal entries of a. Assume that A2 = I. 17. Let a, b, c be such that b(a + c) ¹ 0 if [2009]
[2008] a a + 1 a –1
Statement-1 : If A ¹ I and A ¹ –I, then det (A) = –1 –b b + 1 b –1
+
Statement-2 : If A ¹ I and A ¹ –I, then tr (A) ¹ 0. c c –1 c + 1
(a) Statement -1 is false, Statement-2 is true
a +1 b +1 c –1
(b) Statement -1 is true, Statement-2 is true;
Statement -2 is a correct explanation for a –1 b -1 c + 1 = 0,
Statement-1 (-1)n+ 2 a (-1)n +1b (-1)n c
(c) Statement -1 is true, Statement-2 is true; then the value of n is :
Statement -2 is not a correct explanation (a) any even integer
EBD_7764
www.crackjee.xyz
M-102 Mathematics
(b) any odd integer Statement - 2 :
(c) any integer For any matrix A, det (A)T= det (A) and det (– A)
(d) zero = – det (A).
18. Let A be a 2 × 2 matrix with non-zero entries and Where det (B) denotes the determinant of matrix
B. Then : [2011RS]
let A2 = I , where I is 2 × 2 identity matrix. Define
(a) Both statements are true
Tr(A) = sum of diagonal elements of A and
(b) Both statements are false
|A| = determinant of matrix A. (c) Statement-1 is false and statement-2 is true
Statement - 1 : Tr(A) = 0. (d) Statement-1 is true and statement-2 is false
Statement -2 : |A| = 1. [2010] 23. Consider the following relation R on the set of
(a) Statement -1 is true, Statement -2 is true ; real square matrices of order 3. [2011RS]
Statement -2 is not a correct explanation for
R = { ( A, B) A = P BP for some invertible
–1
Statement -1.
(b) Statement -1 is true, Statement -2 is false. matrix P}
(c) Statement -1 is false, Statement -2 is true . Statement-1 : R is equivalence relation.
(d) Statement - 1 is true, Statement 2 is true ; Statement-2 : For any two invertible 3 ´ 3
Statement -2 is a correct explanation for
matrices M and N, ( MN )
-1
Statement -1. = N -1 M -1 .
19. Consider the system of linear equations; (a) Statement-1 is true, statement-2 is true and
x1 + 2x2 + x3 = 3 [2010] statement-2 is a correct explanation for
2x1 + 3x2 + x3 = 3 statement-1.
3x1 + 5x2 + 2x3 = 1 (b) Statement-1 is true, statement-2 is true;
The system has statement-2 is not a correct explanation for
(a) exactly 3 solutions statement-1.
(b) a unique solution (c) Statement-1 is true, stement-2 is false.
(c) no solution (d) Statement-1 is false, statement-2 is true.
(d) infinite number of solutions æ1 0 0ö
20. The number of values of k for which the linear 24. Let A = çç 2 1 0 ÷÷ . If u1 and u2 are column
equations 4x + ky + 2z = 0 , kx + 4y + z = 0 and 2x ç 3 2 1÷
+ 2y + z = 0 possess a non-zero solution is è ø
æ1ö æ0ö
[2011] ç ÷ ç ÷
(a) 2 (b) 1 matrices such that Au1 = ç 0 ÷ and Au2 = ç 1 ÷ ,
ç0÷ ç0÷
(c) zero (d) 3 è ø è ø
21. If the trivial solution is the only solution of the then u1 + u2 is equal to : [2012]
system of equations [2011RS] æ -1 ö æ -1 ö
x - ky + z = 0 ç ÷ ç ÷
(a) ç 1÷ (b) ç 1÷
kx + 3 y - kz = 0 ç0÷ ç -1 ÷
è ø è ø
3x + y - z = 0
æ -1 ö æ1ö
then the set of all values of k is : ç ÷ ç ÷
(c) ç -1 ÷ (d) ç -1 ÷
(a) R - { 2, -3} (b) R - { 2} ç0÷ ç -1 ÷
è ø è ø
(c) R - { -3} (d) { 2, -3} 25. Let P and Q be 3 ´ 3 matrices P ¹ Q. If P3= Q3
22 Statement - 1: and P2Q = Q2P then determinant of (P2 + Q2) is
Determinant of a skew-symmetric matrix of order equal to : [2012]
3 is zero. (a) – 2 (b) 1
(c) 0 (d) – 1
www.crackjee.xyz
Determinants M-103
26. The number of values of k, for which the system (a) contains two elements.
of equations : (b) contains more than two elements
(k + 1) x + 8y = 4k (c) is an empty set.
kx + (k + 3)y = 3k – 1 (d) is a singleton
has no solution, is [2013] 31. The system of linear equations [2016]
x + ly – z = 0
(a) infinite (b) 1
lx – y – z = 0
(c) 2 (d) 3 x + y – lz = 0
é 1 a 3ù has a non-trivial solution for:
(a) exactly two values of l.
27. If P = ê 1 3 3ú is the adjoint of a 3 × 3 matrix (b) exactly three values of l.
ê ú
êë 2 4 4úû (c) infinitely many values of l.
A and |A| = 4, then a is equal to : [2013] (d) exactly one value of l.
32. Let k be an integer such that triangle with
(a) 4 (b) 11 vertices (k, –3k), (5, k) and (–k, 2) has area 28 sq.
(c) 5 (d) 0 units. Then the orthocentre of this triangle is at
28. If a, b ¹ 0, and f ( n ) = a n + b n and the point : [2017]
æ 1ö æ 1ö
3 1 + f (1) 1 + f ( 2 ) (a) çè 2, ÷ø (b) ç 2, - ÷
2 è 2ø
1 + f (1) 1 + f ( 2 ) 1 + f ( 3)
æ 3ö æ 3ö
1 + f ( 2 ) 1 + f ( 3) 1 + f ( 4 ) (c) çè1, ÷ø (d) ç1, - ÷
4 è 4ø
33. If S is the set of distinct values of ‘b’ for which
= K (1 - a ) (1 - b ) ( a - b)
2 2 2
, the following system of linear equations
then K is equal to: [2014] x + y+ z = 1 [2017]
(a) 1 (b) –1 x + ay + z = 1
ax + by + z = 0
1 has no solution, then S is :
(c) ab (d)
ab (a) a singleton
29. If A is an 3 × 3 non-singular matrix such that AA' (b) an empty set
= A'A and B = A–1A', then BB' equals: [2014] (c) an infinite set
(d) a finite set containing two or more elements
(a) B –1 (b) ( B )¢
-1 34. Let w be a complex number such that 2w + 1 = z

(c) I + B (d) I 1 1 1
30. The set of all values of l for which the system of 2 2
where z = -3 . If 1 -w - 1 w = 3k, then
linear equations : [2015]
2x1 – 2x2 + x3 = lx1 1 w2 w7
2x1 – 3x2 + 2x3 = lx2 k is equal to : [2017]
–x1 + 2x2 = lx3 (a) 1 (b) –z
has a non-trivial solution, (c) z (d) –1

Answer Key
1 2 3 4 5 6 7 8 9 10 11 12 13 14 15
(c) (d) (b) (a) (a) (d) (a) (d) (b) (d) (d) (a) (d) (d) (c)
16 17 18 19 20 21 22 23 24 25 26 27 28 29 30
(a) (b) (b) (c) (a) (a) (d) (b) (d) (c) (b) (b) (a) (d) (a)
31 32 33 34
(b) (a) (a) (b)
EBD_7764
www.crackjee.xyz
M-104 Mathematics

a b ax + b é 0 0 -1ù é 0 0 -1ù
1. (c) We have b c bx + c Also A = ê 0 -1 0 ú ê 0 -1 0 ú
2
ê úê ú
ax + b bx + c 0 ëê -1 0 0 úû êë -1 0 0 ûú
By R3 ® R3 – (xR1 + R2);
é1 0 0ù
a b ax + b = ê0 1 0ú = I
ê ú
= b c bx + c êë0 0 1úû
2
0 0 -(ax + 2bx + c) é 4 2 2ù
= (ax2 + 2bx + c)(b2 – ac) = (+)(–) = –ve. 5. (a) Given that 10 B = ê -5 0 a ú
2. (d) For homogeneous system of equations to ê ú
have non zero solution, D = 0 êë 1 -2 3 úû
1 2a a é 4 2 2ù
1 ê
-5 0 a ú
2
1 3b b = 0 C2 ® C2 - 2C3 b - 4ac ÞB=
10 ê ú
1 4c c êë 1 -2 3 úû

1 0 a Also since, B = A-1 Þ AB = I


1 b b = 0 R3 ® R3 - R2 , R2 ® R2 - R1 é 1 -1 1 ù é 4 2 2 ù é1 0 0 ù
1 ê úê ú ê ú
1 2c c Þ 2 1 -3ú ê -5 0 a ú = ê0 1 0 ú
10 ê
2 1 1 êë 1 1 1 úû êë 1 -2 3 úû êë0 0 1 úû
On simplification, = +
b a c
é10 0 5 - 2 ù é1 0 0ù
\ a, b, c are in Harmonic Progression. 1 ê ú ê ú
Þ ê 0 10 -5 + a ú = ê0 1 0ú
1 wn w 2n 10
êë 0 0 5 + a úû êë0 0 1 úû
3. (b) D = wn w 2n 1
5-a
Þ =0Þa =5
w 2n 1 wn 10

( ) ( ) ( )
3n n 2n 2n 2n n 4n
6. (d) Let r be the common ratio, then
= 1 w -1 - w w - w + w w - w
log an log an+1 log an+ 2
3n 3n 6n
=w -1- 0 + w -w log an+ 3 log an+ 4 log an +5
= 1 - 1 + 1 - 1 = 0 Q w3n = 1ù
é log an+ 6 log an+ 7 log an+8
ë û
é 0 0 -1ù
ê ú log a1r n -1 log a1r n log a1r n +1
4. (a) A = ê 0 -1 0 ú
êë -1 0 0 úû = log a1r n + 2 log a1r n + 3 log a1r n + 4
clearly A ¹ 0. Also |A| = -1 ¹ 0 log a1r n +5 log a1r n + 6 log a1r n + 7
\ A-1 exists, further log a1 + ( n - 1) log r log a1 + n log r log a1 + (n + 1) log r
é -1 0 0 ù = log a1 + ( n + 2) log r log a1 + (n + 3) log r log a1 + (n + 4) log r

( -1) I = ê 0 -1 0 ú ¹ A log a1 + ( n + 5) log r log a1 + (n + 6)log r log a2 + (n + 7) log r


ê ú
êë 0 0 -1úû
www.crackjee.xyz
Determinants M-105

é 1 1 ù 9. (b) Q a1, a2 , a3 ,....... are in G.P..


= 0 ê Apply c2 ® c2 - c1 - c3 ú
ë 2 2 û \ Using an = ar n -1 ,we get the given
7. (a) ax + y + z = a - 1 determinant, as
x + a y + z = a – 1;
x+ y+ z a = a –1 log ar n -1 log ar n log ar n +1

a 1 1 log ar n + 2 log ar n + 3 log ar n + 4


D= 1 a 1 log ar n + 5 log ar n + 6 log ar n+ 7
1 1 a
Operating C3 - C2 and C2 - C1 and
2
= a(a - 1) - 1(a - 1) + 1(1 - a) m
using log m - log n = log we get
= a (a - 1)(a + 1) - 1(a - 1) - 1(a - 1) n
For infinite solutions, D = 0
log ar n -1 log r log r
2
Þ (a - 1)[a + a - 1 - 1] = 0
= log ar n+ 2 log r log r
2
Þ (a - 1)[a + a - 2] = 0 log ar n+5
log r log r
2
Þ (a – 1) [a + 2a - a - 2] = 0 = 0 (two columns being identical)
Þ (a - 1)[a (a + 2) - 1(a + 2)] = 0 10. (d) Given A2 - A + I = 0
(a - 1) = 0, a + 2 = 0
A-1 A2 - A-1 A + A-1.I = A-1.0
Þ a = – 2, 1;
(Multiplying A-1 on both sides)
But a ¹ 1 .
-1
\a=–2 Þ A - 1 + A-1 = 0 or A = I - A .
8. (d) Applying, C1 ® C1 + C2 + C3 we get
1 1 1
2 2 2 2 2
1 + (a + b + c + 2) x (1 + b ) x (1 + c ) x 11. (d) Given, D = 1 1 + x 1
2 2
f (x) = 1 + (a + b + c + 2) x 2
1+ b x 2 2
(1 + c x ) 1 1 1+ y
2 2 2 2
1 + (a + b + c + 2) x (1 + b ) x 1 + c2 x Apply R2 ® R2 – R1 and R ® R3 – R1
1 1 1
1 (1 + b2 ) x (1 + c 2 ) x
\ D = 0 x 0 = xy
= 1 1 + b2 x (1 + c 2 x)
0 0 y
1 (1 + b2 ) x 1 + c2 x
Hence, D is divisible by both x and y
[As given that a2 + b2 + c2 = –2] é5 5a a ù
2 2 2 12. (a) Given A = ê0 a 5a ú and | A2 | = 25
\ a +b +c +2=0 êë0 0 5 úû
Applying R1 ® R1 - R2 , R2 ® R2 - R3
é5 5a a ù é5 5a a ù
0 x -1 0 \ A2 = ê0 a 5a ú ê0 a 5a ú
êë0 0 5 úû êë0 0 5 úû
\ f(x) = 0 1- x x -1
1 (1 + b 2 ) x 1 + c 2 x é 25 25a + 5a 2 5a + 25a 2 + 5a ù
ê ú
=ê0 a2 5a 2 + 25a ú
f (x) = ( x - 1)2
ê0 0 25 ú
Hence degree = 2. ë û
EBD_7764
www.crackjee.xyz
M-106 Mathematics
\ | A2 | = 25 (25a 2 ) a a + 1 a -1
-b b + 1 b - 1 +
1 17. (b)
\ 25 = 25 (25a 2 ) Þ | a | = c c -1 c +1
5
éa b ù 2 a +1 b +1 c -1
13. (d) Let A = ê ú then A = I
ë c d û a -1 b -1 c +1 = 0
Þ a2 + bc = 1 and ab + bd =0 n+2 n +1
(-1) a (-1) b (-1)n c
ac + cd = 0 and bc + d2 = 1
From these four relations, a a + 1 a -1
a2 + bc = bc + d2 Þ a2 = d2 -b b + 1 b - 1 +
and b(a + d) = 0 = c( a + d) Þ a = – d Þ
c c -1 c +1
We can take a = 1, b = 0, c = 0, d = –1 as
one possible set of values, then
a +1 a -1 ( -1)n + 2 a
é1 0 ù
A= ê ú b +1 b -1 ( -1)n +1 b =0
ë0 –1û
Clearly A ¹ I and A ¹ –I and det A = –1 c -1 c + 1 ( -1) n c
\ Statement 1 is true.
(Taking transpose of second determinant)
Also if A ¹ I then tr(A) = 0
\ Statement 2 is false. C1 Û C3
14. (d) The given equations are a a + 1 a -1
–x + cy + bz = 0 -b b + 1 b - 1 -
Þ
cx –y + az = 0 c c -1 c +1
bx + ay – z = 0
Q x, y, z are not all zero ( -1)n + 2 a a -1 a + 1
\ The above system should not have
unique (zero) solution ( -1)n+ 2 (-b) b -1 b + 1 = 0

–1 c b (-1)n + 2 c c + 1 c -1
ÞD=0Þ c –1 a =0
C2 Û C3
b a –1
a a + 1 a -1
Þ –1(1– a2) – c(– c – ab) + b(ac + b) = 0
Þ -b b + 1 b - 1 + (-1)n+ 2
Þ –1 + a2 + b2 + c2 + 2abc = 0
Þ a2 + b2 + c2 + 2abc = 1 c c -1 c +1
15. (c) Q All entries of square matrix A are a a + 1 a -1
integers, therefore all cofactors should
-b b + 1 b - 1 = 0
also be integers.
If det A = ± 1 then A–1 exists. Also all c c -1 c + 1
entries of A–1 are integers.
a a + 1 a -1
16. (a) We know that | adj (adj A) | = | A | n–2 A.
Þ é1 + (-1)n+ 2 ù -b b + 1 b - 1 = 0
= | A |0 A = A ë û
Also | adj A | = | A | n–1 = | A | 2–1 = | A | c c -1 c + 1
\ Both the statements are true but C2 – C1, C3 – C1
statement-2 is not a correct explanation for
statement-1 .
www.crackjee.xyz
Determinants M-107

a 1 -1 21. (a) x - ky + z = 0
n+ 2 ù
Þ éë1 + (-1) û
-b 2b + 1 2b - 1 = 0 R1 + R3 kx + 3 y - kz = 0
c -1 1 3x + y - z = 0
The given system of equations will have non
a+c 0 0
trivial solution, if
Þ é1 + ( -1) n+ 2 ù -b 2b + 1 2b - 1 = 0
ë û
c -1 1 1 -k 1
Þ [1+ (– 1)n + 2](a + c) (2b + 1+ 2b – 1) = 0 k 3 -k =0
Þ 4b (a + c) [1 + (–1)n + 2] = 0 3 1 -1
Þ 1 + (–1)n + 2 = 0 as b (a + c) ¹ 0
Þ n should be an odd integer. Þ 1(-3 + k ) + k ( - k + 3k ) + 1( k - 9) = 0
æa b ö Þ k - 3 + 2k 2 + k - 9 = 0
18. (b) Let A = ç ÷ where a, b, c, d ¹ 0
èc d ø Þ k2 + k - 6 = 0
Þ k = -3, k = 2
æa b öæa bö So the equation will have only trivial
A2 = ç ÷ç ÷
èc d øèc dø solution,
when k Î R – {2, – 3}
æ a 2 + bc ab + bd ö
Þ A2 = ç ÷ 22. (d) Statement-1 : Determinant of skew
ç ac + cd bc + d 2 ÷ symmetric matrix of odd order is zero.
è ø
Þ a 2 + bc = 1, bc + d 2 = 1
T
( )
Statement-2 : det A = det (A).
det (– A) = – (– 1)n det (A).
ab + bd = ac + cd = 0 where A is a n ´ n order matrix.
c ¹ 0 and b ¹ 0 Þ a + d = 0 23. (b) For reflexive

| A |= ad - bc = -a 2 - bc = -1 ( A, A) Î R
1 2 1 A = P -1 AP is true,
19. (c) For P = I, which is an invertible matrix.
D= 2 3 1 =0
\ R is reflexive.
3 5 2 For symmetry
3 2 1 As ( A, B) ÎR for matrix P
D1 = 3 3 1 ¹ 0
A = P-1 BP
1 5 2
Þ PAP-1 = B
Þ Given system, does not have any solution. Þ B = PAP -1
Þ No solution
( )
-1
20. (a) D = 0 Þ B = P –1 A (P–1)
4 k 2
\ (B, A) Î R for matrix P -1
Þ k 4 1 =0 \ R is symmetric.
2 2 1 For transitivity
Þ 4(4 - 2) - k (k - 2) + 2(2k - 8) = 0 A = P-1 BP
Þ 8 - k 2 + 2k + 4k - 16 = 0 and B = P–1CP
k 2 - 6k + 8 = 0 Þ (
A = P –1 P -1CP P )
Þ (k - 4)(k - 2) = 0 Þ k = 4, 2
EBD_7764
www.crackjee.xyz
M-108 Mathematics
If |P2 + Q2| ¹ 0 then P2 + Q2 is invertible.
Þ A= P ( ) CP
-1 2 2
Þ P–Q=0 Þ P=Q
A = (P ) C (P )
2 -1 2 Which gives a contradiction (Q P ¹ Q)
Þ
Hence |P2 + Q2| = 0
\ (A, C) ÎR for matrix P
2
26. (b) From the given system, we have
\ R is transitive. k +1 8 4k
So R is equivalence = ¹
k k + 3 3k - 1
æ1ö æ0ö (Q System has no solution)
ç ÷ ç ÷
24. (d) Let Au1 = ç 0 ÷ and Au2 = ç 1 ÷ Þ k2 + 4k + 3 = 8k
ç0÷ ç0÷ Þ k = 1, 3
è ø è ø
8 4.1
æ 1ö æ0ö If k = 1 then ¹ which is false
ç ÷ ç ÷ 1+ 3 2
Then, Au1 + Au2 = ç 0 ÷ + ç 1 ÷
ç0÷ ç0÷ 8 4.3
è ø è ø and if k = 3 then ¹ which is true,
6 9 -1
æ 1ö therefore k = 3
ç ÷ Hence for only one value of k. System has
Þ A ( u1 + u2 ) = ç 1 ÷ ...(1)
ç0÷ no solution.
è ø 27. (b) | P | = 1(12 – 12) – a(4 – 6) + 3(4 – 6) = 2a –
6
æ1 0 0ö Now, adj A = P Þ | adj A | = | P |
ç ÷
Also, A = ç 2 1 0 ÷ Þ | A |2 = | P |
ç 3 2 1÷ Þ | P | = 16
è ø
Þ 2a – 6 = 16
Þ |A| = 1(1) – 0 (2) + 0 (4–3) = 1 Þ a = 11
We know, 28. (a) Consider
1 1 + f (1) 1 + f (2)
A-1 = adjA 3
A 1 + f (1) 1 + f (2) 1 + f (3)
Þ A -1 = adj ( A ) (Q A = 1) 1 + f (2) 1 + f (3) 1 + f (4)
Now, from equation (1), we have
1+1 +1 1+ a + b 1 + a 2 + b2
æ 1ö
-1 ç ÷ = 1+ a + b 1 + a 2 + b2 1 + a 3 + b3
u1 + u2 = A ç 1÷
ç0÷ 1 + a 2 + b2 1 + a3 + b3 1 + a 4 + b4
è ø
1 1 1 1 1 1
é 1 0 0ù æ 1 ö é 1 ù
ç ÷ a b ´ 1 a b
= êê -2 1 0 úú ç 1 ÷ = êê -1úú = 1
êë 1 -2 1úû èç 0 ø÷ êë -1úû 1 a2 b2 1 a2 b2

25. (c) Given P3 = Q3 ...(1) 2


1 1 1
and P2Q = Q2P ...(2)
= 1 a b
Subtracting (1) and (2), we get
P3 – P2Q = Q3 – Q2P 1 a2 b2
Þ P2 (P–Q) + Q2 (P – Q) = 0 = [(1 – a) (1 – b) (a – b)]2
Þ (P2 + Q2) (P–Q) = 0 So, K = 1
www.crackjee.xyz
Determinants M-109
So no real solution exist
29. (d) BB' = B ( A-1 A ') ' = B ( A ') '( A-1 ) '
For orthocentre
= BA (A–1)' BH ^ AC
= ( A -1 A ')( A( A -1) ') æ b - 2 öæ 8 ö
\ ç ÷ç ÷ = –1
= A–1A .A'.(A–1)' {as AA' = A'A} è a - 5 øè -4 ø
= I(A–1A)' = I.I = I 2 = I Þ a – 2b = 1
...(1)
30. (a) 2x1 - 2x 2 + x 3 = lx1ü
ï Also CH ^ AB
2x1 - 3x 2 + 2x 3 = lx 2 ý
æ b - 2 öæ 8 ö
- x1 + 2x 2 = lx3 ïþ \ ç ÷ç ÷ = –1
è a + 2 øè 3 ø
Þ (2 – l)x1 – 2x2 + x3 = 0 Þ 3a + 8b = 1 ... (2)
2x1 – (3 + l) x2 + 2x3 = 0 Solving (1) and (2), we get
– x1 + 2x2 – lx3 = 0
For non-trivial solution, 1
a = 2, b =
D=0 2
2-l -2 1 æ 1ö
orthocentre is çè 21 ÷ø
i.e. 2 -(3 + l ) 2 =0 2
-1 2 -l 1 1 1
Þ (2 – l) [l(3 + l) – 4] + 2[–2l + 2] + 1[4 – (3 + l)] = 0 1 a 1
Þ l3 + l2 – 5l + 3 = 0 33. (a) D= =0
Þ l = 1, 1, 3 a b 1
Hence l has 2 values. Þ 1 [a – b] – 1 [1 – a] + 1 [b – a2] = 0
31. (b) For trivial solution, Þ (a – 1)2 = 0
Þa=1
1 l -1
For a = 1, First two equations are identical
l - 1 -1 = 0 ie. x + y + z = 1
1 1 -l To have no solution with x + by + z = 0
b= 1
( 1)(l- 1) = 0
Þ -ll+ So b = {1} Þ It is singleton set.
Þl= 0, +1, –1 34. (b) Given 2w + 1 = z;
32. (a) We have z = 3i

k -3k 1 3i - 1
1 Þ w=
5 k 1 = 28 2
2 Þ w is complex cube root of unity
-k 2 1
Applying R1 ® R1 + R2 + R3
Þ 5k2 + 13k – 46 = 0
or 5k2 + 13k + 66 = 0 3 0 0
Now, 5k2 + 13k – 46 = 0 1 -w - 1 w2
2
=
-13 ± 1089 -23 1 w2 w
Þ k= \ k= ;k = 2
10 5
= 3 (–1 – w – w) = –3 (1 + 2w) = – 3z
since k is an integer, \ k = 2
Þk=–z
Also 5k2 + 13k + 66 = 0
-13 ± -1151
Þ k=
10
EBD_7764
www.crackjee.xyz
M-110 Mathematics

10-13 delet

Continuity and Differentiability


20
1. f is defined in [-5, 5] as [2002] (d) neither differentiable nor continuous at x = 0
f (x) = x if x is rational
6. If f ( x) = x n , then the value of [2003]
= – x if x is irrational. Then
(a) f (x) is continuous at every x, except x = 0 f ' (1) f ' ' (1) f ' ' ' (1) ( -1) n f n (1)
(b) f (x) is discontinuous at every x, except x = 0 f (1) - + - + ..........
1! 2! 3! n!
(c) f (x) is continuous everywhere
is
(d) f (x) is discontinuous everywhere
(a) 1 (b) 2 n
2. If f ( x + y ) = f ( x ). f ( y )"x. y and f (5) = 2,
(c) 2 n - 1 (d) 0.
f '(0) = 3, then f ¢ (5) is [2002]
7. Let f (x) be a polynomial function of second
(a) 0 (b) 1
(c) 6 (d) 2 degree. If f(1) = f(-1) and a, b, c are in A. P , then
f '(a), f ¢ (b), f '(c) are in [2003]
d2y dy
3. If y = (x + 1 + x 2 )n, then (1 + x2) +x is (a) Arithmetic -Geometric Progression
dx2 dx
(b) A.P
(a) n2y (b) – n2y [2002] (c) G..P
(c) –y (d) 2x2y (d) H.P.
4. Let f (a) = g (a) = k and their nth derivatives 1 - tan x p é pù
8. Let f ( x) = , x ¹ , x Î ê 0, ú .
n n
f (a) , g (a) exist and are not equal for some 4x - p 4 ë 2û
n. Further if é pù æ pö
If f (x) is continuous in ê 0, ú , then f çè ÷ø is
f ( a ) g ( x) - f ( a ) - g ( a ) f ( x ) + f ( a ) ë 2û 4
lim =4
x ®a g ( x) - f ( x) [2004]
then the value of k is [2003] 1
(a) 0 (b) 4 (a) –1 (b)
2
(c) 2 (d) 1
1
(c) - (d) 1
ì æ 1 1ö 2
ï -ç + ÷
5. If f ( x ) = í xe è x x ø , x ¹ 0 then f(x) is y +L to ¥ dy
ï0 9. If x = e y + e , x > 0, then is [2004]
î ,x = 0 dx

(a) discontinuous every where [2003] 1+ x 1


(a) (b)
(b) continuous as well as differentiable for all x x x
(c) continuous for all x but not differentiable at
1- x x
x=0 (c) (d)
x 1+ x
www.crackjee.xyz
Continuity and Differentiability M-111

10. Suppose f (x) is differentiable at x = 1 and


(c) f (x) ³ 1 for all x Î R
1 (d) f (x) is not differentiable at x = 1
lim f (1 + h) = 5 , then f '(1) equals [2005]
h®0 h 17. The function f : R /{0} ® R given by [2007]
(a) 3 (b) 4 1 2
(c) 5 (d) 6 f ( x) =- 2x
x e -1
11. Let f be differentiable for all x. If f (1) = – 2 and
can be made continuous at x = 0 by defining f (0)
f '( x) ³ 2 for x Î [1, 6], then [2005] as
(a) 0 (b) 1
(a) f (6) ³ 8 (b) f (6) < 8 (c) 2 (d) – 1
(c) f (6) < 5 (d) f (6) = 5
ì 1
12. If f is a real valued differentiable function ï( x –1) sin if x ¹ 1
18. Let f ( x) = í x –1 [2008]
satisfying | f (x) – f (y) | £ ( x - y )2 , x, y Î R and ïî 0 if x = 1
f (0) = 0, then f (1) equals [2005] Then which one of the following is true?
(a) – 1 (b) 0 (a) f is neither differentiable at x = 0 nor at x =1
(c) 2 (d) 1 (b) f is differentiable at x = 0 and at x =1
(c) f is differentiable at x = 0 but not at x = 1
13. The value of a for which the sum of the squares
(d) f is differentiable at x = 1 but not at x = 0
of the roots of the equation x2 – (a – 2) x – a – 19. Let y be an implicit function of x defined by
1 = 0 assume the least value is [2005] x2x – 2xx cot y – 1= 0. Then y¢(1) equals
(a) 1 (b) 0 [2009]
(c) 3 (d) 2 (a) 1 (b) log 2
(c) –log 2 (d) –1
x
14. The set of points where f ( x ) = is 20. Let f : (–1, 1) ® R be a differentiable function
1+ | x | with f (0) = – 1 and f ¢ (0) = 1. Let g(x) = [f (2f (x) +
differentiable is [2006] 2)]2. Then g¢(0) = [2010]
(a) (-¥,0) È (0, ¥) (a) –4 (b) 0
(c) –2 (d) 4
(b) (-¥,-1) È (-1, ¥) 21. The values of p and q for which the function
[2011]
(c) (-¥, ¥)
ì
(d) (0, ¥) ï sin( p + 1) x + sin x , x < 0
ï x
ï
f ( x) = í q , x = 0 is continuous for
dy
15. If x m . y n = ( x + y ) m+ n , then is [2006] ï 2
dx ï x+ x - x
ïî ,x > 0
x3 / 2
y x+ y
(a) (b) all x in R, are
x xy
5 1
x (a) p = ,q =
(c) xy (d) 2 2
y
16. Let f : R ® R be a function defined by 3 1
(b) p = - ,q =
2 2
f (x) = min {x + 1, x + 1} ,Then which of the
1 3
following is true ? (c) p = ,q =
2 2
(a) f (x) is differentiable everywhere [2007]
1 3
(b) f (x) is not differentiable at x = 0 (d) p = ,q = -
2 2
EBD_7764
www.crackjee.xyz
M-112 Mathematics

d2x (b) a f (a ) - a 2 f ' ( a )


22. equals : [2011]
dy 2
(c) 2af ( a) - a 2 f ' ( a)
-1
æ d 2 y ö æ dy ö-3 2a f (a) +a2 f '(a)
(a) -ç 2 ÷ ç ÷ (d)
ç dx ÷ è dx ø
è ø 25. If f : R ® R is a function defined by f (x) = [x]

æ d 2 y ö æ dy ö -2 æ 2x - 1 ö
cos ç ÷ p , where [x] denotes the greatest
(b) çç 2 ÷÷ ç dx ÷ è 2 ø
è dx ø è ø integer function, then f is . [2012]
(a) continuous for every real x.
æ d 2 y ö æ dy ö-3
(c) -ç 2 ÷ç ÷ (b) discontinuous only at x = 0
ç dx ÷ è dx ø
è ø (c) discontinuous only at non-zero integral
values of x.
-1
æ d2y ö (d) continuous only at x = 0.
(d) çç 2 ÷÷
è dx ø 26. Consider the function, f (x) = | x – 2 | `+ | x – 5 |,
x Î R.
23. Define f (x) as the product of two real function
Statement-1 : f ¢(4) = 0
[2011RS]
Statement-2 : f is continuous in [2,5],
ì 1 differentiable in (2,5) and f (2) = f (5). [2012]
ïsin , if x ¹ 0
f1 ( x ) = x, x Î R, and f 2 ( x ) = í x (a) Statement-1 is false, Statement-2 is true.
ï if x = 0 (b) Statement-1 is true, statement-2 is true;
î0,
statement-2 is a correct explanation for
as follows :
Statement-1.
ïì f1 ( x ) . f 2 ( x) , if x = 0 (c) Statement-1 is true, statement-2 is true;
f ( x) = í
ïî 0 if x = 0 statement-2 is not a correct explanation for
Statement-1.
Statement - 1 : f (x) is continuous on R. (d) Statement-1 is true, statement-2 is false.
Statement - 2 : f1 ( x) and f 2 ( x) are continuous
dy
on R. 27. If y = sec(tan–1x), then at x = 1 is equal to :
dx
(a) Statement -1 is true, Statement-2 is true;
[2013]
Statement-2 is a correct explanation for
Statement-1. 1 1
(b) Statement-1 is true, Statement-2 is true; (a) (b)
2 2
Statement-2 is NOT a correct explanation for
Statement-1 (c) 1 2(d)
(c) Statement-1 is true, Statement-2 is false 28. If f and g are differentiable functions in [0, 1]
(d) Statement-1 is false, Statement-2 is true satisfying f (0) = 2 = g(1), g(0) = 0 and f (1) = 6,
24. If function f (x) is differentiable at x = a,
then for some c Î]0,1[ [2014]
x f (a) - a f ( x)
2 2
(a) f ¢(c) = g¢(c) (b) f ¢(c) = 2g ¢(c)
then xlim
®a
is : [2011RS]
x-a (c) 2f ¢(c) = g¢(c) (d) 2f ¢(c) = 3g¢(c)
(a) -a 2 f ' ( a )
www.crackjee.xyz
Continuity and Differentiability M-113

29. If the function. (b) g is differentiable at x = 0 and g'(0) = –


sin(log2)
ïì k x + 1, 0 £ x £ 3 (c) g is not differentiable at x = 0
g(x) = í is differentiable, (d) g'(0) = cos(log2)
ïî m x + 2, 3 < x £ 5
dy
then the value of k + m is : [2015] 31. If (2 + sin x) + (y + 1) cos x = 0 and y(0) = 1,
dx
10 æ pö
(a) (b) 4 then y ç ÷ is equal to :
3 è 2ø

16 4 1
(c) 2 (d) (a) (b)
5 3 3
30. For x Î R, f(x) = |log2 – sinx| and g(x) = f(f(x)),
2 1
then : [2016] (c) - (d) -
3 3
(a) g'(0) = – cos(log2)

Answer Key
1 2 3 4 5 6 7 8 9 10 11 12 13 14 15
(b) (c) (a) (b) (c) (d) (b) (c) (c) (c) (a) (b) (a) (c) (a)
16 17 18 19 20 21 22 23 24 25 26 27 28 29 30
(a) (b) (c) (d) (a) (b) (c) (c) (c) (a) (c) (a) (b) (c) (d)
31
(b)

1. (b) Let a is a rational number other than 0, in [– 2. (c) f (x + y) = f (x) ´ f (y)


5, 5], then f (a) = a and lim f ( x) = -a Differentiate with respect to x, treating y as
x®a constant
[As in the immediate neighbourhood of a f ¢ (x + y) = f ¢ (x) f (y)
rational number, we find irrational numbers]
\ f (x) is not continuous at any rational Putting x = 0 and y = x, we get f '(x)= f '(0) f
number (x) ; Þ f ¢ (5) = 3 f (5) = 3 × 2 = 6.
If a is irrational number, then
3. (a) y = ( x + 1 + x 2 )n
f (a) = – a and lim f ( x ) = a
x® a
dy æ 1 ö
\ f (x) is not continuous at any irrational = n( x + 1 + x 2 )n -1 ç 1 + (1 + x 2 ) -1/ 2 . 2 x; ÷
dx è 2 ø
number clearly lim f ( x) = f (0) = 0
x®0
dy ( 1 + x 2 + x)
\ f (x) is continuous at x = 0 = n( x + 1 + x 2 ) n -1
dx 1 + x2
EBD_7764
www.crackjee.xyz
M-114 Mathematics

n( 1 + x 2 + x ) n 7. (b) f ( x) = ax 2 + bx + c
=
1 + x2 f (1) = f (-1)
2 dy Þ a + b + c = a - b + c or b = 0
or 1 + x = ny or 1 + x 2 y 1= ny
dx \ f ( x) = ax 2 + c or f '( x ) = 2ax
dy
( y1 = ) Squaring, Now f '(a); f '(b); and f '( c)
dx
(1 + x 2 ) y12 = n2 y 2 are 2a(a); 2a(b); 2a(c )
Differentiating, i.e. 2a2, 2ab, 2ac.
Þ If a, b, c are in A.P. then
(1 + x 2 )2 y1 y2 + y12 .2 x = n2 .2 yy 1
f '(a); f '(b) and f '(c ) are also in A.P..
or (1+x2)y2 + xy1 = n2y
1 - tan x
f (a) g ¢ ( x ) - g (a ) f ¢ ( x ) 8. (c) f ( x) = is continuous in
4. (b) lim =4 4x - p
x® a g ¢( x ) - f ¢ ( x )
(By L’ Hospital rule) é pù
ê0, 2 ú
k g ¢ ( x) - k f ¢ ( x) ë û
lim =4
x ® a g ¢ ( x) - f ¢ ( x) æ pö
\ f ç ÷ = lim p f ( x) = lim + f ( x )
\ k = 4. è 4 ø x® p
4 x®
4
æ 1 1ö
-ç + ÷ æp ö
5. (c) f (0) = 0; f ( x ) = xe è x xø = lim f ç + h÷
h ®0 è 4 ø
h
R.H.L. lim (0 + h)e -2 / h = lim =0 æp ö
h® 0 h ®0 e 2 / h 1 - tan ç + h÷
è4 ø
æ 1 1ö
-ç - ÷ = lim ,h > 0
è h hø h ®0 æ p ö
L.H.L. lim (0 - h)e =0 4 ç + h÷ - p
h® 0 è4 ø
therefore, f (x) is continuous.
æ 1 1ö 1 + tan h
-ç + ÷ 1-
(0 + h)e è h hø
-0 1 - tan h
R.H.D = lim =0 = lim
h® 0 h h® 0 4h
æ 1 1ö
-ç - ÷
-2 tan h -2 1
è h hø = lim . = =-
(0 - h)e -0 h ®0 1 - tanh 4 h 4 2
L.H.D. = lim =1
h® 0 -h
therefore, L.H.D. ¹ R.H.D.
p
f (x) is not differentiable at x = 0. Q f (x) is continuous at x =
4
6. (d) f ( x ) = x n Þ f (1) = 1
æ pö 1 - tan x
f ' ( x ) = nx n-1 Þ f ' (1) = n \ f çè ÷ø = limp f ( x ) = limp
4 x® x®
4x - p
f '' ( x ) = n ( n - 1) x n- 2 Þ f '' (1) = n ( n - 1) 4 4

.........................
- sec2 x
......................... f n
( x ) = n ! Þ f (1) = n!
n = limp [using L' Hospital’s rule]

4
n n ( n - 1) n ( n - 1)( n - 2 ) n n! 4
= 1- + - + ¼ + ( -1) p
1! 2! 3! n! - sec 2
=n C0 -n C1 + n C2 - n C3 + ¼ + ( -1)
n n
Cn = 0 = 4 = -2 = -1
4 4 2
www.crackjee.xyz
Continuity and Differentiability M-115

y +L¥
9. (c) x = e y+ e Þ x = e y+ x . ì x
x<0
Taking log. ïï1 - x ,
14. (c) f ( x) = í
1 dy
log x = y + x Þ ï x , x³0
= +1
x dx îï1 + x
dy 1 1- x
\ = -1 = ì x
dx x x ï 2
, x<0
ï (1 - x )
f (1 + h) - f (1) Þ f '( x) = í
10. (c) f '(1) = lim ; ï x , x³0
h®0 h ïî (1 + x )2
As function is differentiable so it is
\ f '( x) exist at everywhere.
f (1 + h)
continuous as it is given that lim 15. (a) x m . y n = ( x + y ) m+ n
h®0 h
= 5 and hence f (1) = 0 Þ mlnx + nlny = (m + n)ln(x + y)
Differentiating both sides.
f (1 + h)
Hence f '(1) = lim =5 m n dy m + n æ dy ö
h®0 h \ + = ç1 + ÷
x y dx x + y è dx ø
11. (a) As f (1) = – 2 & f '( x) ³ 2 " x Î [1, 6]
Applying Lagrange’s mean value
theorem æ m m + nö æ m + n n ö dy
Þ çè x - x + y ø÷ = çè x + y - y ÷ø dx
f (6) - f (1)
= f '(c) ³ 2
5
my - nx æ my - nx ö dy
Þ f (6) ³ 10 + f (1) Þ =
x( x + y ) èç y ( x + y ) ø÷ dx
Þ f (6) ³ 10 – 2 Þ f (6) ³ 8.
f ( x + h) - f ( x ) dy y
Þ =
12. (b) f '(x) = lim dx x
h®0 h
f ( x + h ) - f ( x) 16. (a) f (x) = min {x + 1, | x | + 1} Þ f (x) = x + 1 Ú
| f '( x) | = lim x ÎR
h®0 h
(h)2 Y
£ lim
h®0 h y=-x+1 y=x+1
Þ | f '( x) | £ 0 Þ f '( x) = 0 (0, 1)
Þ f (x) = constant
As f (0) = 0 X' X
(-1, 0)
Þ f (1) = 0.
13. (a) x 2 - (a - 2) x - a - 1 = 0 Y'

Þ a + b = a - 2 ; a b = -(a + 1) Hence, f (x) is differentiable everywhere for


all x Î R.
a + b = (a + b) 2 - 2 a b
2 2
1 2
= a2 - 2a + 6 = (a - 1)2 + 5 17. (b) Given, f (x) = -
x e2 x - 1
For min. value of a2 + b2 where a is an
integer 1 2
Þ f (0) = lim -
Þ a = 1. x®0 2
x e -1x
EBD_7764
w w w . c r a c k j
M-116 Mathematics

( e2 x - 1) - 2 x
é0 ù dy æ 1 ö du
= lim – 2cosec2 y = ç1 + 2 ÷
; ê form ú dx è u ø dx
x ® 0 x(e 2 x - 1) ë 0 û
\ using, L'Hospital rule where u = xx Þ log u = x log x
1 du
4e 2 x Þ = 1 + log x
f (0) = lim u dx
x ® 0 2( xe 2 x 2 + e 2 x .1) + e 2 x .2
du
Þ = x x (1 + log x)
4e 2 x é0 ù dx
= lim êë 0 form úû \ We get
x ® 0 4 xe 2 x + 2e 2 x + 2e 2 x
– 2 cosec2 y
4e 2 x 4.e0 dy
= lim = =1 = (1 + x -2 x ).x x (1 + log x)
x ® 0 4( xe 2 x + e 2 x ) 4(0 + e0 ) dx
18. (c) We have
ì Þ
dy
=
( )
x x + x - x (1 + log x)
…(i)
æ 1 ö
ï(x – 1) sin ç , if x ¹ 1 dx -2(1 + cot 2 y )
f (x) = í è x – 1÷ø
ïî 0 , if x = 1 Now when x = 1, x2x – 2xx cot y – 1 = 0,
gives
f (1 + h) – f (1) 1 – 2 cot y – 1 = 0
Rf ' (1) = lim
h ®0 h Þ cot y = 0
\ From equation (i), at x = 1 and
1
h sin –0 cot y = 0, we get
h 1
= lim = lim sin
h ®0 h h ®0 h (1 + 1) (1 + 0)
= a finite number y ' (1) = = -1
-2(1 + 0)
Let this finite number be l
20. (a)
f (1 - h) - f (1)
L f ¢(1) = lim æ d ö
h®0 -h g '( x ) = 2 ( f (2 f ( x) + 2) ) ç ( f (2 f ( x) + 2) ) ÷
è dx ø
æ 1 ö
-h sin ç ÷
= lim è -h ø = 2 f (2 f ( x) + 2) f '(2 f ( x)) + 2).(2 f '( x))
h ®0 -h
æ 1 ö æ 1ö Þ g '(0) = 2 f (2 f (0) + 2). f '(2 f (0) + 2)
= lim sin ç ÷ = - lim sin çè ÷ø
h ®0 è -h ø h®0 h .2 f '(0) = 4 f (0)( f '(0)) 2 = 4(–1)(1)2 = – 4
= – (a finite number) = – l
Thus Rf ¢(1) ¹ Lf ¢(1) 21. (b) L.H .L = lim- f ( x)
( at x = 0) x ®0
\ f is not differentiable at x = 1
Also, sin{( p + 1)( - h )} - sinh
= lim
h®0 -h
1 x –1 æ 1 öù
f '(0) = sin – cos ç ÷ú = p+1+1=p+2
(x – 1) (x – 1) 2 è x – 1 ø úû x =0
= –sin 1 + cos 1 1 1
R.H .L = lim+ f ( x) = lim = =
\ f is differentiable at x = 0 ( at x =0) x ®0 h ®0 1+ 1 2
19. (d) x2x – 2xx cot y – 1 = 0
Þ 2 cot y = xx – x – x 3 1
f (0) = q Þ p = - , q =
1 2 2
Þ 2 cot y = u – where u = xx
u
Differentiating both sides with respect to x, we
get
www.crackjee.xyz
Continuity and Differentiability M-117

d 2x d æ dx ö d æ dx ö dx æ 2x - 1 ö
= \ f (x) = [x] cos ç ÷ is
22. (c) 2 ç ÷= ç ÷ è 2 ø
dy dy è dy ø dx è dy ø dy continuous for every real x.
d æ 1 ö dx
= ç ÷ ìx - 2 , x - 2 ³ 0
dx è dy / dx ø dy 26. (c) f (x) = x - 2 = í
î2 - x , x - 2 £ 0
1 d2 y 1 1 d2 y ìx - 2 , x ³ 2
=- . . =– =í
æ dy ö
2
dx 2 dy 3
æ dy ö dx
2
î2 - x , x £ 2
ç ÷ dx ç ÷
è dx ø è dx ø Similarly,
ìï x sin (1/ x ) , x ¹ 0 ìx - 5 , x ³ 5
23. (c) f (x) = í at x = 0 f (x) = x - 5 = í
ïî 0 , x = 0 î5 - x , x £ 5
\ f (x) = x - 2 + x - 5
ì æ 1ö ü
LHL = lim– í - h sin çè - ÷ø ý = {x - 2 + 5 - x = 3, 2 £ x £ 5}
h®0 î h þ
= 0 × a finite quantity betwen – 1 Thus f (x) = 3 , 2 £ x £ 5
and 1= 0 f ¢ (x) = 0 , 2 < x < 5
f ¢ (4) = 0
lim 1
RHL = h®0+
h sin =0 Y
h
Also, f (0) = 0
Thus LHL = RHL = f ( 0)
\ f ( x) is continuous on R.
f2 ( x) is not continuous at x = 0 X
2 5
x f ( a ) - a f ( x)
2 2
24. (c) xlim
®a
x-a Clearly, statement-2 is also true.
2 xf ( a ) - a 2 f ' ( x ) Q f (2) = 0 + |2 – 5| = 3
= xlim
®a and f (5) = |5 – 2| + 0 = 3
1
= 2af ( a ) - a 2 f ' ( a ) a = 0, b = 0 and c is any real number.
æ 2x - 1 ö 27. (a) Let y = sec(tan–1 x) and tan–1 x = q.
25. (a) Let f ( x ) = [ x ] cos ç ÷ Þ x = tan q
è 2 ø
Doubtful points are x = n, n Î I
æ 2x - 1 ö
L.H.L = lim- [ x ] cos ç ÷p
x ®n è 2 ø
æ 2n - 1 ö
= ( n - 1) cos ç ÷p = 0
è 2 ø
(Q [x] is the greatest integer function)
Thus, we have y = sec q
æ 2x - 1 ö
R.H.L = lim+ [ x ] cos ç ÷p Þ y = 1 + x 2 (Qsec2q = 1 + tan 2q)
x ®n è 2 ø
æ 2n - 1 ö dy 1
= n cos ç · 2x
÷p = 0 Þ =
è 2 ø dx 2 1 + x2
Now, value of the function at x = n is At x = 1,
f (n) = 0 dy 1
Since, L.H.L = R.H.L. = f(n) = .D
dx 2
EBD_7764
www.crackjee.xyz
M-118 Mathematics
28. (b) Since, f and g both are continuous function on 30. (d) g (x) = f (f (x))
[0, 1] and differentiable on (0, 1) then $ c Î (0,1) In the neighbourhood of x = 0,
f(x) = | log2 – sin x| = (log 2 – sin x)
such that
\ g (x) = |log 2 – sin| log 2 – sin x ||
f (1) - f (0) 6 - 2 = (log 2 – sin(log 2 – sin x))
f ¢(c ) = = =4
1 1 \ g (x) is differentiable
g (1) - g (0) 2 - 0
and g'(x) = – cos(log 2 – sin x) (– cos x)
and g ¢(c ) = = =2 Þ g'(0) = cos (log 2)
1 1
Thus, we get f ¢ ( c) = 2 g ¢ ( c) dy
31. (b) We have (2 + sinx) + (y + 1) cos x = 0
29. (c) Since g (x) is differentiable, it will be continuous dx
at x = 3 d
Þ (2 + sin x)(y + 1) = 0
\ xlim g(x) = lim+ g(x) dx
®3- x®3
On integrating, we get
2k = 3m + 2 ...(1)
(2 + sin x) (y + 1) = C
Also g(x) is differentiable at x = 0
At x = 0, y = 1 we have
\ xlim g ¢(x) = lim+ g¢ (x) (2 + sin 0) (1 + 1) = C
®3– x ®3
ÞC=4
k 4
=m Þ y+ 1 =
2 3 +1 2 + sin x
k= 4m ...(2) 4
y= -1
Solving (1) and (2), we get 2 + sin x

2 8 æ pö 4
m= , k= Now y ç ÷ = -1
5 5 è 2 ø 2 + sin p
k+m =2 4 1 2
= -1 =
3 3
www.crackjee.xyz
Application of Derivatives M-119

Application of Derivatives
21
1. The maximum distance from origin of a point on
æ9 9ö
æ at ö (a) ç , ÷
è8 2ø
(b) (2, - 4)
the curve x = a sin t–b sin çè ÷ø , y = a cos t – b
b
æ -9 9ö
æ at ö (c) ç , ÷
è 8 2ø
(d) (2, 4)
cos çè ÷ø ,both a, b > 0 is [2002]
b
(a) a – b (b) a + b 6. A function y = f ( x ) has a secon d or der

(c) a 2 + b2 derivative f "( x) = 6( x - 1). If its graph passes


(d) a 2 - b2
through the point (2,1) and at that point the
2. If 2a + 3b + 6c = 0, (a, b, c Î R) then the quadratic
tangent to the graph is y = 3x – 5, then the
equation ax2 + bx + c = 0 has [2002]
function is [2004]
(a) at least one root in [0, 1]
(b) at least one root in [2, 3] (a) ( x + 1)2 (b) ( x - 1)3
(c) at least one root in [4, 5]
(d) none of these (c) ( x + 1)3 (d) ( x - 1)2

3. If the function f ( x) = 2 x3 - 9ax 2 + 12a 2 x + 1 , 7. The normal to the curve x = a(1 + cos q), y = a
where a > 0 , attains its maximum and minimum sinq at ‘q’ always passes through the fixed point
2
at p and q respectively such that p = q , then [2004]
a equals [2003] (a) (a, a) (b) (0, a)
(c) (0, 0) (d) (a, 0)
1
(a) (b) 3 8. If 2a + 3b + 6c = 0, then at least one root of the
2
(c) 1 (d) 2 equation ax 2 + bx + c = 0 lies in the interval
4. The real number x when added to its inverse [2004]
gives the minimum value of the sum at x equal to (a) (1, 3) (b) (1, 2)
(a) –2 (b) 2 [2003] (c) (2, 3) (d) (0, 1)
(c) 1 (d) –1 9. Area of the greatest rectangle that can be

5. A point on the parabola y 2 = 18x at which the x2 y2


inscribed in the ellipse + = 1 is
ordinate increases at twice the rate of the a2 b2
abscissa is [2004] [2005]
(a) 2ab (b) ab
a
(c) ab (d)
b
EBD_7764
www.crackjee.xyz
M-120 Mathematics
10. The normal to the curve [2005] 14. A lizard, at an initial distance of 21 cm behind an
x = a (cos q + q sin q ), y = a (sin q – q cos q ) insect, moves from rest with an acceleration of
at any point q is such that 2 cm / s 2 and pursues the insect which is
(a) it passes through the origin crawling uniformly along a straight line at a speed
p of 20 cm/s. Then the lizard will catch the insect
(b) it makes an angle + q with the x- axis after [2005]
2
(a) 20 s (b) 1 s
æ p ö (c) 21 s (d) 24 s
(c) it passes through ç a , - a÷
è 2 ø 15. Two points A and B move from rest along a
(d) It is at a constant distance from the origin straight line with constant acceleration f and f '
11. A spherical iron ball 10 cm in radius is coated respectively. If A takes m sec. more than B and
with a layer of ice of uniform thickness that melts
describes ‘n’units more than B in acquiring the
at a rate of 50 cm3/min. When the thickness of
ice is 5 cm,then the rate at which the thickness same speed then [2005]
of ice decreases is [2005] (a) ( f - f ')m2 = ff ' n
1 1
(a) cm/min. (b) cm/min. (b) ( f + f ')m2 = ff ' n
36 p 18 p
1
1 5 (c) ( f + f ')m = ff ' n 2
(c) cm/min. (d) cm/min 2
54 p 6p
1
12. If the equation an x n + an -1 x n -1 + ............. + (d) ( f '- f ) n = ff ' m2
2
a1 x = 0
x 2
a1 ¹ 0, n ³ 2, has a positive root x = a , then 16. The function f ( x ) = + has a local minimum
2 x
n -1 n -2
the equation nan x + (n – 1) an -1 x + ......... at [2006]
+ a1 = 0 has a positive root, which is [2005] (a) x=2 (b) x = -2
(a) greater than a (c) x = 0 (d) x = 1
(b) smaller than a 17. A triangular park is enclosed on two sides by a
(c) greater than or equal to a fence and on the third side by a straight river bank.
The two sides having fence are of same length x.
(d) equal to a
The maximum area enclosed by the park is
13. A function is matched below against an interval
[2006]
where it is supposed to be increasing. Which of the
following pairs is incorrectly matched? [2005] 3 2 x3
(a) x (b)
Interval Function 2 8
(a) (– ¥ , ¥ ) x3 - 3 x 2 + 3 x + 3 (c) 1 2 (d) px 2
x
2
(b) [2, ¥ ) 2 x3 - 3 x 2 - 12 x + 6
18. value of c for which conclusion of Mean Value
æ 1ù Theorem holds for the function f (x) = loge x on
(c) ç - ¥, ú 3 x2 - 2 x + 1 the interval [1, 3] is [2007]
è 3û
(a) log3e (b) loge3
(d) (– ¥ , – 4) x3 + 6 x 2 + 6 1
(c) 2 log3e (d) log e
2 3
www.crackjee.xyz
Application of Derivatives M-121
19. The function f (x) = tan –1(sin x + cos x) is an 23. The differential equation of the family of circles
increasing function in [2007] with fixed radius 5 units and centre on the line y
= 2 is [2009]
æ pö æ p pö 2
(a) (x – 2)y' = 25 –(y – 2) 2
(a) çè 0, ÷ø (b) çè - , ÷ø
2 2 2 (b) (y – 2)y'2 = 25 –(y – 2)2
(c) (y – 2)2y'2 = 25 –(y – 2)2
(c) æ p pö (d) æ p pö
çè , ÷ø çè - , ÷ø (d) (x – 2)2 y'2 = 25 –(y – 2)2
4 2 2 4
24. Let f (x) = x | x | and g (x) = sin x.
20. If p and q are positive real numbers such that p2 Statement-1 : gof is differentiable at x = 0 and
+ q2 = 1, then the maximum value of (p + q) is its derivative is continuous at that point.
[2007] Statement-2 : gof is twice differentiable at x = 0.
[2009]
1 1
(a) (b) (a) Statement-1 is true, Statement-2 is true;
2 2 Statement-2 is not a correct explanation for
(c) 2 (d) 2. Statement-1.
21. Suppose the cubic x3 – px + q has three distinct (b) Statement-1 is true, Statement-2 is false.
real roots where p > 0 and q > 0. Then which one (c) Statement-1 is false, Statement-2 is true.
of the following holds? [2008] (d) Statement-1 is true, Statement-2 is true;
Statement-2 is a correct explanation for
p Statement-1.
(a) The cubic has minima at and maxima
3 25. Given P(x) = x4 + ax3 + bx2 + cx + d such that x
= 0 is the only real root of P' (x) = 0. If P(–1)
p
at – < P(1), then in the interval [ –1, 1] : [2009]
3
(a) P(–1) is not minimum but P(1) is the
p maximum of P
(b) The cubic has minima at – and maxima (b) P(–1) is the minimum but P(1) is not the
3
maximum of P
p (c) Neither P(–1) is the minimum nor P(1) is
at
3 the maximum of P
(d) P(–1) is the minimum and P(1) is the
p
(c) The cubic has minima at both and maximum of P
3
26. The equation of the tangent to the curve
p 4
– y = x+ , that is parallel to the x-axis, is [2010]
3 x2
(a) y = 1 (b) y = 2
p (c) y = 3 (d) y = 0
(d) The cubic has maxima at both and
3 27. Let f : R ® R be defined by


p
3
f ( x) = { k2-x+23,x, ifif xx £>--11
If f has a local minimum at x = – 1 , then a
22. How many real solutions does the equation
possible value of k is [2010]
x7 + 14x5 + 16x3 + 30x – 560 = 0 have? [2008]
(a) 7 (b) 1 1
(a) 0 (b) -
(c) 3 (d) 5 2
(c) –1 (d) 1
EBD_7764
www.crackjee.xyz
M-122 Mathematics
28. Let f : R ® R be a continuous function defined 31. A spherical balloon is filled with 4500p cubic
meters of helium gas. If a leak in the balloon
1
by f ( x ) = x [2010] causes the gas to escape at the rate of 72p cubic
e + 2e - x meters per minute, then the rate (in meters per
minute) at which the radius of the balloon
1
Statement -1 : f (c) = , for some c Î R. decreases 49 minutes after the leakage began is:
3
9 7
1 (a) (b) [2012]
Statement -2 : 0 < f (x) £ , for all x Î R 7 9
2 2
2 9
(a) Statement -1 is true, Statement -2 is true ; (c) (d)
9 2
Statement -2 is not a correct explanation for
32. Let a, b Î R be such that the function f given by
Statement -1.
f (x) = ln | x | + bx2 + ax, x ¹ 0 has extreme values
(b) Statement -1 is true, Statement -2 is false.
at x = –1 and x = 2
(c) Statement -1 is false, Statement -2 is true . Statement-1 : f has local maximum at x = –1 and
(d) Statement - 1 is true, Statement 2 is true ; at x = 2.
Statement -2 is a correct explanation for
1 -1
Statement -1. Statement-2 : a = and b = [2012]
2 4
29. Let f be a function defined by - [2011RS]
(a) Statement-1 is false, Statement-2 is true.
ì tan x (b) Statement-1 is true, statement-2 is true;
ï , x¹0
f ( x) = í x statement-2 is a correct explanation for
ï1, x=0 Statement-1.
î
(c) Statement-1 is true, statement-2 is true;
Statement - 1 : x = 0 is point of minima of f statement-2 is not a correct explanation for
Statement - 2 : f ¢ ( 0 ) = 0. Statement-1.
(a) Statement-1 is true, statement-2 is true; (d) Statement-1 is true, statement-2 is false.
statement-2 is a correct explanation for 33. The real number k for which the equation, 2x3 +
statement-1. 3x + k = 0 has two distinct real roots in [0, 1]
(b) Statement-1 is true, statement-2 is true; [2013]
statement-2 is NOT a correct explanation for (a) lies between 1 and 2
statement-1. (b) lies between 2 and 3
(c) Statement-1 is true, statement-2 is false. (c) lies between .1 and 0
(d) Statement-1 is false, statement-2 is true. (d) does not exist.
30. The curve that passes through the point (2, 3), 34. If x = –1 and x = 2 are extreme points of
and has the property that the segment of any
f ( x ) = a log x + b x 2 + x then [2014]
tangent to it lying between the coordinate axes
is bisected by the point of contact is given by : 1 1
[2011RS] (a) a = 2, b = - (b) a = 2, b =
2 2
6
(a) 2 y - 3x = 0 (b) y = 1 1
x (c) a = -6, b = (d) a = -6, b = -
2 2
2 2
æxö æ yö
(c) x2 + y 2 = 13 (d) ç ÷ + ç ÷ = 2
è 2ø è 3ø
www.crackjee.xyz
Application of Derivatives M-123
35. The normal to the curve, x2 + 2xy – 3y2 = 0, at (1, 1) (c) 2x = (p + 4)r (d) (4 – p) x =pr
[2015] 38. The normal to the curve y(x – 2)(x – 3) = x + 6 at
(a) meets the curve again in the third quadrant. the point where the curve intersects the y-axis
(b) meets the curve again in the fourth passes through the point:
quadrant. æ 1 1ö æ 1 1ö
(c) does not meet the curve again. (a) çè , ÷ø (b) ç - , - ÷
(d) meets the curve again in the second 2 3 è 2 2ø
quadrant. æ 1 1ö æ 1 1ö
36. Consider (c) çè , ÷ø (d) ç , - ÷
2 2 è 2 3ø
æ 1 + sin x ö æ ö
f (x) = tan -1 ç ÷, x Î ç0,p÷. 39. Twenty metres of wire is available for fencing off
ç 1 - sin x ÷ ç ÷ a flower-bed in the form of a circular sector. Then
è ø è 2ø
p the maximum area (in sq. m) of the flower-bed, is
A normal to y = f(x) at x = a so passes through : [2017]
6
the point : [2016] (a) 30 (b) 12.5
æp ö æp ö (c) 10 (d) 25
(a) ç ç , 0÷÷ (b) çç , 0÷ ÷ 40. The eccentricity of an ellipse whose centre is
è6 ø è4 ø 1
æ 2pö at the origin is . If one of its directices is x = –
(c) (0, 0) (d) çç0, ÷ ÷ 2
è 3 ø
37. A wire of length 2 units is cut into two parts æ 3ö
4, then the equation of the normal to it at ç1, ÷
which are bent respectively to form a square of è 2ø
side = x units and a circle of radius = r units. If is : [2017]
the sum of the areas of the square and the circle (a) x + 2y = 4 (b) 2y – x = 2
so formed is minimum, then: [2016] (c) 4x – 2y = 1 (d) 4x + 2y = 7
(a) x = 2r (b) 2x = r

Answer Key
1 2 3 4 5 6 7 8 9 10 11 12 13 14 15
(b) (a) (d) (c) (a) (b) (d) (d) (a) (d) (b) (b) (c) (c) (d)
16 17 18 19 20 21 22 23 24 25 26 27 28 29 30
(a) (c) (c) (d) (c) (a) (b) (c) (b) (a) (c) (c) (d) (b) (b)
31 32 33 34 35 36 37 38 39 40
(c) (b) (d) (a) (b) (d) (a) (c) (d) (c)

=a+b
1. (b) Distance of origin from (x, y) = x2 + y 2 \ Maximum distance from origin = a + b
æ at ö ax3 bx 2
= a 2 + b2 - 2ab cos ç t - ÷ ; 2. (a) Let f (x) = + + cx
è bø 3 2
Þ f (0) = 0 and
£ a 2 + b 2 + 2ab
a b 2a + 3b + 6c
f (1) = + +c = =0
éì æ at ö ü ù 3 2 6
ê í cos ç t - ÷ ý = -1ú
Also f (x) is continuous and differentiable
è b ø þ min
ëê î ûú
EBD_7764
www.crackjee.xyz
M-124 Mathematics
in [0, 1] and [0, 1[. So by Rolle’s theorem, Þ 1 = (2 - 1)3 + D Þ D = 0
f ¢ (x) = 0. \ f (x) = ( x – 1)3
i.e ax2 + bx + c = 0 has at least one root in
dx dy
[0, 1]. 7. (d) = - a sin q and = a cos q
dq dq
3. (d) f ( x) = 2 x 3 - 9ax 2 + 12a 2 x + 1
dy
f '( x ) = 6 x 2 - 18ax + 12a 2 ; f ''( x ) = 12 x - 18a \ = - cot q.
dx
For max. or min. \ The slope of the normal at q = tan q
6 x 2 - 18ax + 12 a 2 = 0 Þ x 2 - 3ax + 2a 2 = 0 \ The equation of the normal at q is
Þ x = a or x = 2a.At x = a max . y - a sin q = tan q( x - a - a cos q)

and at x = 2a min Þ y cos q - a sin q cos q = x sin q - a sin q


\ p = a and q = 2a - a sin q cos q
As per question p 2 = q Þ x sin q - y cos q = a sin q
2 Þ y = ( x - a ) tan q
\ a = 2a Þ a = 2or a = 0
but a > 0, therefore, a = 2. which always passes through (a, 0)
8. (d) Let us define a function
1 dy 1
4. (c) y = x + or = 1- ax3 bx 2
x dx x2 f ( x) = + + cx
1 3 2
For max. or min., 1 - = 0 Þ x = ±1 Being polynomial, it is continuous and
x2 differentiable, also,
d2y 2 æ d 2y ö a b
= Þç ÷ =2 f (0) = 0 and f (1) = + + c
dx 2
x 3 ç dx 2 ÷ 3 2
è ø x =2
(+ve minima) 2a + 3b + 6c
\x = 1 Þ f (1) = = 0 (given)
6
dy dy 9 \ f (0) = f (1)
5. (a) y 2 = 18 x Þ 2 y = 18 Þ =
dx dx y \ f (x) satisfies all conditions of Rolle’s
theorem therefore f ¢(x) = 0 has a root in (0, 1)
dy 9 9
Given = 2Þ =2Þ y = i.e. ax 2 + bx + c = 0 has at lease one root
dx y 2
in (0, 1)
9 (a) Area of rectangle ABCD = 2a cos q
Putting in y 2 = 18 x Þ x = 9.
8 (2b sin q) = 2ab sin 2q
æ9 9ö
\ Required point is ç , ÷ Y
è8 2ø
6. (b) f ¢¢ ( x) = 6( x - 1). Inegrating, we get (-a cos q, b sin q) (a cos q, b sin q)
B A
f ¢ ( x) = 3 x 2 - 6 x + c
X
Slope at (2, 1) = f ¢(2) = c = 3
[Q slope of tangent at (2,1) is 3] C D
2 2 ( - a cos q, - b sin q) (a cos q, - b sin q)
\ f ¢ ( x ) = 3x - 6 x + 3 = 3( x - 1)
Inegrating again, we get
Þ Area of greatest rectangle is equal to
f ( x ) = ( x - 1)3 + D 2ab
The curve passes through (2, 1) When sin 2q = 1 .
www.crackjee.xyz
Application of Derivatives M-125

10. (d) x = a ( cos q + q sin q) 13. (c) Clearly function f (x) = 3 x 2 - 2 x + 1 is


dx increasing when f '( x ) = 6x – 2 ³ 0
Þ = a ( - sin q + sin q + q cos q )
dq Þ x Î[1/ 3, ¥)
dx \ f (x) is incorrectly matched with
Þ = aq cos q .....(1)
dq
æ 1ù
y = a ( sin q - q cos q) ç -¥, 3 ú
è û
dy 14. (c) Let the lizard catches the insect after time
= a [ cos q - cos q + q sin q ]
dq t then distance covered by lizard = 21cm
dy + distance covered by insect
Þ = aq sin q .....(2)
dq 1 2
Þ ft = 4 ´ t + 21
From equations (1) and (2) we get 2
dy 1
= tan q Þ Slope of normal = – cot q Þ ´ 2 ´ t 2 = 20 ´ t + 21
dx 2
Equation of normal at 'q ' is y – a (sin q
– q cos q ) Þ t 2 - 20t - 21 = 0
= – cot q (x – a (cos q + q sin q ) Þ t = 21sec
15. (d)
Þ y sin q – a sin 2 q + a q cos q sin q
= – x cos q + a cos 2 q + a q sin q cos q u=0 f s+n
A v
Þ x cos q + y sin q = a t+m
Clearly this is an equation of straight line
which is at a constant distance ‘a’ from
origin. u=0 s
B f¢ v
11. (b) Given that t
dv d æ 4 3ö As per question if point B moves s
= 50 cm3/min Þ ç pr ÷ø = 50
dt dt è 3 distance in t time then point A moves (s +
n) distance in time (t + m) after which
2 dr
Þ 4 pr = 50 both have same velocity v.
dt Then using equation v = u + at we get
dr 50 1 f m
Þ dt = 4 p(15) 2 = cm/min v = f (t + m) = f ' t Þ t = ....(1)
18p f '- f
(here r = 10+5)
12. (b) Let f (x) = an x n + an -1 x n -1 + ...... + a1 x = 0 Using equation v 2 = u 2 + 2 , as we get
The other given equation, f n
v2 = 2 f ( s + n ) = 2 f ' s Þs=
nan xn -1
+ (n – 1) an -1 x n -2 + ....+ a1 = 0 = f '- f
f ¢(x) ....(2)
Given a1 ¹ 0 Þ f (0) = 0 Also for point B using the eqn
Again f (x) has root a, Þ f (a ) = 0 1
s = ut + at 2 , we get
\ f (0) = f (a) 2
\ By Rolle’s theorem f ¢(x) = 0 has root 1
between ( 0, a ) s= f 't 2
2
Hence f ¢ ( x ) has a positive root smaller Substituting values of t and s from
equations (1) and (2) in the above
than a. relation, we get
EBD_7764
www.crackjee.xyz
M-126 Mathematics
2 2 2
f n 1 f m c=
= f' Þ
f '- f 2 ( f '- f ) 2 loge 3 Þ c = 2 log3e
19. (d) Given f (x) = tan–1 (sin x + cos x)
1
Þ ( f '- f ) n = ff ' m2 1
2 f '(x) = .(cos x - sin x )
1 1 + (sin x + cos x) 2
x 2
16. (a) + is of the form y + where
2 x y æ 1 1 ö
2. ç cos x - sin x÷
1 è 2 2 ø
y + ³ 2 and equality holds for y = 1 =
y 1 + (sin x + cos x )2
x æ p p ö
\ Min value of function occurs at = 1 çè cos .cos x - sin .sin x÷ø
2 4 4
i.e., =
at x = 2 1 + (sin x + cos x) 2
æ pö
2 cos ç x + ÷
x 2 1 2 è 4ø
f (x) = + Þ f '( x ) = - 2 = 0 \ f '(x) =
2 x 2 x 1 + (sin x + cos x ) 2
if f ' (x) > 0 then f (x) is increasing function.
4
Þ x2 = 4 or x = 2, – 2; f ''( x) = Hen ce f (x) is increasing, if
x3 p p p
- < x+ <
f ''( x )] x = 2 = + ve Þ f ( x ) has local min 2 4 2
at x = 2. 3p p
Þ - <x<
4 4
1 2
17. (c) Area = x sin q Hence, f (x) is increasing when n
2
æ p pö
Îç - , ÷
x q x è 2 4ø
20. (c) Given that p2 + q2 = 1
\ p = cos q and q = sin q
p
Maximum value of sinq is 1 at q =
2 Then p + q = cos q + sin q
We know that
1 2
Amax = x
2 - a 2 + b2 £ a cos q + b sin q £ a 2 + b 2
18. (c) Using Lagrange's Mean Value Theorem \ – 2 £ cos q + sin q £ 2
Let f (x) be a function defined on [a, b]
Hence max. value of p + q is 2
f (b ) - f ( a )
then, f '(c) = ....(i)
b-a
c Î [a, b] Since, p and q are positive real numbers
1 p2 + q2 = 1 (Given)
\ Given f (x) = logex \ f ' (x) = Using AM ³ GM
x
\ equation (i) become 2
æ p + qö 2
1 f (3) - f (1) \ çè ÷ ³ ( pq)
= 2 ø
c 3-1
1 loge 3 - loge 1 log e3 p 2 + q 2 + 2 pq
Þ = = = ³ pq
c 2 2 4
www.crackjee.xyz
Application of Derivatives M-127
1 + 2 pq æ dy ö
2
³ pq or 1 + 2pq ³ 4pq ( y – 2)2 ç ÷ + ( y – 2)2 = 25
4 è dx ø
1 ³ 2pq or, 2pq £ 1 Þ (y – 2)2 (y')2 = 25 – (y –2)2
1 24. (b) Given that f (x) = x | x | and g (x) = sin x
pq £ or, pq £ 1 So that
2 2
Now, (p + q)2 = p2 + q2 + 2pq go f (x) = g ( f (x)) = g (x | x | ) = sin x | x |
1 ìïsin (– x 2 ), if x < 0
Þ (p + q)2 £ 1 + 2 × Þ p+q £ 2 =í
2 2
ïîsin ( x ), if x ³ 0
21. (a) Let y = x3 – px + q Þ dy = 3 x 2 – p
ïì – sin x , if x < 0
dx 2
dy 2 = í 2
For = 0 Þ 3x – p = 0 ïî sin x , if x ³ 0
dx
p ïì – 2 x cos x , if x < 0
2
Þx=± \ (go f )¢ (x) = í
3 2
ïî 2 x cos x , if x ³ 0
d2y Here we observe
= 6x
dx 2 L (go f )¢ (0) = 0 = R (go f )¢ (0)
d2y d2y Þ go f is differentiable at x = 0
= + ve and = – ve and (go f )¢ is continuous at x = 0
dx 2 x=
p dx 2 x= –
p Now (go f )'' (x)
3 3
ïì – 2 cos x + 4 x sin x , x < 0
2 2 2
p
\ y has minima at x = and maxima at = í
2 2 2
3 ïî 2 cos x - 4 x sin x , x ³ 0
p Here
x=–
3 L(go f )'' (0) = – 2 and R (go f )'' (0) = 2
22. (b) Let f (x) = x7 + 14x5 + 16x3 + 30x –560 Q L(go f)'' (0) ¹ R (go f )'' (0)
Þ f '(x) = 7x6 + 70x4 + 48x2 + 30 > 0, " x Þ go f (x) is not twice differentiable at x = 0.
ÎR \ Statement - 1 is true but statement -2 is
Þ f is an increasing function on R false.
25. (a) We have P (x) = x4 + ax3 + bx2 + cx + d
Also lim f ( x) = ¥ and
x ®¥ Þ P' (x) = 4 x3 + 3ax2 + 2bx + c
But P' (0) = 0 Þ c = 0
lim f ( x ) = – ¥ \ P(x) = x4 + ax3 + bx2 + d
x® – ¥
Þ The curve y = f (x) crosses x-axis only As given that P ( – 1) < P (a)
once. Þ 1–a+b +d < 1+a+b+d
\ f (x) = 0 has exactly one real root. Þ a>0
23. (c) Let the centre of the circle be ( h, 2) Now P ' (x) = 4x3 + 3ax2 +2bx = x (4x2 + 3ax
\ Equation of circle is + 2b)
As P' (x) = 0, there is only one solution
( x – h) 2 + ( y – 2) 2 = 25 …(1) x = 0, therefore 4x2 + 3ax + 2b = 0 should
Differentiating with respect to x, we get not have any real roots i.e. D < 0
dy
2( x – h) + 2( y – 2) = 0 9a 2
dx Þ 9a2 – 32 b < 0 Þ b> >0
dy 32
Þ x – h = –( y – 2) Hence a, b > 0 Þ P' (x) = 4 x 3 + 3ax2 +
dx
Substituting in equation (1) we get 2bx > 0
"x > 0
EBD_7764
w w w . c r a c k j e e . x y
M-128 Mathematics
\ P (x) is an increasing function on (0,1) Y
\ P (0) < P (a)
y = tan x
Similarly we can prove P (x) is decreasing
on (– 1, 0)
\ P (– 1) > P (0) y=x
X´ X
So we can conclude that O
Max P (x) = P (1) and Min P (x) = P (0)
Þ P(–1) is not minimum but P (1) is the
maximum of P.
26. (c) Since tangent is parallel to x-axis,
8 Y´
dy In left neighbourhood of ‘0’
\ = 0 Þ 1- 3 = 0 Þ x = 2 Þ y = 3
dx x tan x < x
Equation of tangent is y – 3 = 0 (x – 2)
> 1 as ( tan x < 0)
tan x
Þy=3
27. (c) x
ì k - 2 x, if x £ -1 at x = 0, f ( x) = 1
f ( x) = í Þ x = 0 is the point of minima
î 2 x + 3, if x > -1 So Statement 1 is true.
Statement 2 obvious.
2x + 3 dy
k-2x 30. (b) Y – y = ( X - x)
1 dx
Y
–1
B (0, y-xdy/dx)

This is true where k = – 1


1 ex
28. (d) f ( x) = =
e x + 2e - x e2 x + 2 (x, y)

(e 2 x + 2) e x - 2e 2 x .e x
f '( x ) =
(e 2 x + 2) 2
f '( x) = 0 Þ e 2 x + 2 = 2e 2 x
X´ X
O
e2 x = 2 Þ e x = 2
A (x – y)/(dy/ dx), 0)

2 1 y
= X-intercept = x -
maximum f (x) = dy / dx
4 2 2
1 xdy
0 < f ( x) £ Y-intercept = y –
"x Î R dx
2 2 According to given statement
1 1 y xdy
Since 0 < < Þ for some c Î R x - = 2 x and y - = 2y
3 2 2
dy dx
1 -y - xdy
f ( c) = =x =y
3 and
dy dx
ì tan x dx
ï , x¹0
29. (b) f ( x) = í x dx dy
Þ + =0
ï 1, x = 0 x y
î
lny = -lnc + lnc
In right neighbourhood of ‘0’ c
tan x > x y=
x
tan x
>1
x
www.crackjee.xyz
Application of Derivatives M-129
Since the above line passes through the
point (2, 3). + +
\ c=6 0
–¥ –1 – 2 – ¥
6
Hence y = is the required equation.
x
31. (c) Volume of spherical balloon So maxima at x = –1, 2
4
= V = pr 3 33. (d) f (x) = 2x3 + 3x + k
3
f'(x) = 6x2 + 3 > 0 " x Î R (Q x2 > 0)
4pr 3
Þ 4500 p = Þ f(x) is strictly increasing function
3 Þ f(x) = 0 has only one real root, so two
(Q Given, volume = 4500pm3) roots are not possible.
Differentiate both the side, w.r.t 't' we get, 34. (a) Let f (x) = a log | x | + bx2 + x
dV æ dr ö Differentiate both side,
= 4pr 2 ç ÷
dt è dt ø a
f ¢ ( x ) = + 2b x + 1
dV x
Now, it is given that = 72 p Since x = –1 and x = 2 are extreme points
dt
\ After 49 min, therefore f ¢ ( x ) = 0 at these points.
Volume = (4500 – 49 ´ 72)p Put x = –1 and x = 2 in f ¢( x ) , we get
= (4500 – 3528)p = 972 p m3
– a –2b + 1 = 0 Þ a +2b = 1 ...(i)
Þ V = 972 p m3
4 a
\ 972 p = pr 3 + 4b + 1 = 0 Þ a +8b = –2 ...(ii)
3 2
On solving (i) and (ii), we get
Þ r3 = 3 ´ 243 = 3 ´ 35 = 36 = (32)3
Þ r=9 1
6b = -3 Þ b = -
dV 2
Also, we have = 72 p \ a=2
dt
35. (b) Given curve is
\ æ dr ö x2 + 2xy – 3y2 = 0 ...(1)
72 p = 4 p´ 9 ´ 9 ç ÷
è dt ø Differentiatew.r.t. x
dr æ 2 ö dy dy
Þ =ç ÷ , 2x + 2x + 2y - 6y =0
dt è 9 ø dx dx
32. (b) Given, f ( x ) = ln x + bx 2 + ax æ dy ö
=1
ç ÷
1 è dx ø(1, 1)
\ f ' ( x ) = + 2bx + a
x Equation of normal at (1, 1) is
At x = –1, f '(x) = –1– 2b + a =0 y=2–x ...(2)
Þ a – 2b = 1 ...(i) Solving eq. (1) and (2), we get
1 x=1,3
At x = 2, f '(x) = + 4b + a = 0 Point of intersection (1, 1), (3, –1)
2 Normal cuts the curve again in 4th quadrant.
1
Þ a + 4b = - ...(ii) æ 1 + sin x ö
2 f ( x) = tan –1 ç
è 1– sin x ÷ø
1 1 36. (d)
On solving (i) and (ii) we get a = , b = -
2 4
1 x 1 2 - x2 + x æ 2ö
æ x xö æ xö
Thus, f ' ( x ) = - + = ç çè sin + cos ÷ø ÷ 1 + tan
x 2 2 2x 2 2 ÷ ç 2÷
= tan –1 ç –1

=
2
-x + x + 2
=
- (x 2
- x - 2) ç
ç
æ x xö
2 ÷ = tan ç
çè sin – cos ÷ø ÷

ç 1 – tan ÷
è è 2ø
2x 2x x 2 ø
- ( x + 1)( x - 2 )
= æ æ p xö ö
2x = tan –1 ç tan ç + ÷ ÷
è è 4 2ø ø
EBD_7764
www.crackjee.xyz
M-130 Mathematics
p x æ1 1ö
Þ y= + \ ç , ÷ satisfy it.
4 2 è2 2ø
dy 1 39. (d) We have
Þ =
dx 2 Total length = r + r + rq = 20
–1 Þ 2r + r q = 20
Slope of normal = = –2
æ dy ö 20 - 2r
çè ÷ø Þq= ...(1)
dx r
æp p pö q 1 1 æ 20 - 2r ö
At çè , + ÷ A = Area = ´ pr 2 = r 2q = r 2 ç ÷
6 4 12 ø 2p 2 2 è r ø
A = 10r – r 2
æp pö æ pö
y – ç + ÷ = –2 ç x – ÷ For A to be maximum
è 4 12 ø è 6ø q
4p 2p r r
y– = –2 x + dA
12 6 = 0 Þ 10 – 2r = 0
p p dr
y – = –2 x + Þr =5
3 3 d 2A
2p = –2 < 0
y = –2 x + dr 2
3 \ For r = 5 A is maximum q r
This equation is satisfied only by the point
From (1)
æ 2p ö 20 - 2(5) 10
çè 0, ÷ø =
3 q = 5 5
=2
37. (a) 4x + 2pr = 2 Þ 2x + pr = 1
2
S = x2 + pr2 A= ´ p(5) 2 = 25 sq. m
2 2p
æ 1 - pr ö 2
S=ç ÷ + pr 1
è 2 ø 40. (c) Eccentricity of ellipse =
2
dS æ 1 - pr öæ -p ö a 1
= 2ç ÷ç ÷ + 2pr Now, – = – 4 Þ a = 4 × = 2 Þ a = 2
dr è 2 øè 2 ø e 2
-p p2 r 1 æ 1ö
Þ + + 2pr = 0 Þ r = We have b = a (1 – e ) = a2 ç 1 - ÷ = 4
2 2 2
2 2 p+ 4 è 4ø
2 3
Þx= Þ x = 2r × =3
p+ 4 4
x+6 \ Equation of ellipse is
38. (c) We have y = (x - 2)(x - 3) x 2 y2
+ =1
At y-axis, x = 0 Þ y = 1 4 3
On differentiating, we get Now differentiating, we get
dy (x 2 - 5x + 6) (1) - (x + 6) (2x - 5) x 2y
= Þ + ´ y¢ = 0 Þ y¢ = – 3x
dx (x 2 - 5x + 6)2 2 3 4y
dy 3 2 1
= 1 at point (0, 1) y¢ (1,3/ 2) = - ´ = -
dx 4 3 2
Slope of normal = 2
\ Slope of normal = – 1
\ Equation of normal at æç1, ö÷ is
Now equation of normal is y – 1 = –1 (x – 0) 3
Þy–1 =–x è 2ø
x+y= 1 3
y – = 2 (x – 1) Þ 2y – 3 = 4x – 4
2
\ 4x – 2y = 1
www.crackjee.xyz
Integrals M-131

Integrals 22
10 p 6. Let f(x) be a function satisfying f '(x) = f(x) with
1. ò0 | sin x | dx is [2002] f(0)=1 and g(x) be a function that satisfies
(a) 20 (b) 8
(c) 10 (d) 18 f(x) + g(x) = x 2 . Then the value of the integral
p/4 1
ò f ( x ) g ( x ) dx, is [2003]
ò
n
2. In = tan x dx then lim n[ I n + I n + 2 ] equals
n®¥ 0
0
[2002] e2 5 e2 5
(a) e + + (b) e - -
(a) 1 (b) 1 2 2 2 2
2 e2 3 e2 3
(c) ¥ (d) zero (c) e + - (d) e - - .
2 2 2 2
2 1
2
3. ò [x ]dx is [2002] 7. The value of the integral I = ò x(1 - x ) n dx is
0 0
[2003]
(a) 2 – 2 (b) 2 + 2
1 1 1
(c) 2 –1 (d) - 2 - 3 + 5 (a) + (b)
n +1 n + 2 n +1
p 2 x (1 + sin x )
4. ò 2
dx is [2002] (c)
1
(d)
1
-
1
.
- p 1 + cos x
n+2 n +1 n + 2
r
p2 n
1
(a) (b) p 2 8.
Lim
n®¥ å n e n is [2004]
4 r =1
p (a) e + 1 (b) e – 1
(c) zero (d) (c) 1 – e (d) e
2
b 3
5. If f (a + b - x) = f ( x) then ò xf ( x )dx is equal to
ò |1- x
9. The value of 2
|dx is [2004]
a
[2003] -2
b 1 14
a+b (a) (b)
2 ò
(a) f (a + b + x )dx 3 3
a
7 28
a+b b (c) (d)
(b) ò f (b - x)dx 3 3
2 a
p/2
(sin x + cos x ) 2
(c)
a +b b
ò f ( x)dx
10. The value of I = ò 1 + sin 2 x
dx is
2 a 0
[2004]
b-a b
(d) ò f ( x )dx . (a) 3 (b) 1
2 a (c) 2 (d) 0
EBD_7764
www.crackjee.xyz
M-132 Mathematics
p p/2 1 1 2
2 3
x2
11. If ò xf (sin x)dx = A ò f (sin x ) dx, then A is 16.
x
If I1 = ò 2 dx , I 2 =
x
ò 2 dx , I3 = ò2 dx
0 0 0 0 1
[2004]
2
(a) 2p (b) p 3
and I 4 = ò 2 dx then
x
[2005]
p 1
(c) (d) 0
4
(a) I 2 > I1 (b) I1 > I 2
f ( a)
ex
12. If f ( x) =
1+ ex
, I1 = ò xg{x (1 - x )}dx (c) I3 = I 4 (d) I3 > I 4
f (-a )
p
cos 2 x
f ( a) 17. The value of ò x
dx , a > 0, is [2005]
and I 2 = ò g{x (1 - x )}dx, then the value - p 1+ a
f (-a )
p
I (a) a p (b)
of 2 is [2004] 2
I1 p
(c) (d) 2p
a
(a) 1 (b) –3
6
(c) –1 (d) 2 x
I =ò
18. The value of integral, dx is
sin x 9- x + x
If ò
3
13. dx = Ax + B log sin( x - a ), +C ,
sin( x - a ) [2006]
then value of (A, B) is [2004]
1 3
(a) (- cos a, sin a) (b) (cos a, sin a) (a) (b)
2 2
(c) (- sin a, cos a) (d) (sin a, cos a) (c) 2 (d) 1
p
dx
14. ò cos x - sin x is equal to [2004] 19. ò xf (sin x)dx is equal to [2006]
0
1 æ x 3p ö
(a) log tan ç + ÷ + C p p
2 è2 8 ø
(a) p ò f (cos x )dx (b) p ò f (sin x )dx
1 æ xö 0 0
(b) log cot ç ÷ + C
2 è 2ø p /2
p /2
p
1 æ x 3p ö
log tan ç - ÷ + C
(c)
2 ò f (sin x)dx (d) p ò f (cos x )dx
(c) è2 8 ø 0 0
2
p
1 æ x pö -
(d) log tan ç - ÷ + C 2
è 2 8ø
2 20. ò [( x + p )3 + cos 2 ( x + 3p )]dx is equal to
2 3p
ïì (log x - 1) ïü -
15. ò íï1 + (log x)2 ýï dx is equal to [2005] 2
[2006]
î þ
4 4
p p p
log x x (a) (b) +
(a) +C (b) +C 32 32 2
(log x)2 + 1 x2 + 1
p p
(c) (d) -1
xe x x
+C 2 4
(c) +C (d)
1 + x2 (log x)2 + 1
www.crackjee.xyz
Integrals M-133
a sin xdx
26. The value of 2ò is [2008]
21. The value of ò [ x] f '( x)dx , a > 1 where [x]
æ pö
sin ç x – ÷
1
denotes the greatest integer not exceeding x is è 4ø
[2006] æ pö
(a) x + log | cos ç x – ÷ | + c
(a) af (a) - { f (1) + f (2) + .............. f ([ a])} è 4ø
(b) [a ] f (a) - { f (1) + f (2) + .............. f ([a ])} æ pö
(b) x – log | sin ç x – ÷ | + c
è 4ø
(c) [a ] f ([a ]) - { f (1) + f (2) + .............. f ( a)}
æ pö
(c) x + log | sin ç x – ÷ | +c
(d) af ([ a]) - { f (1) + f (2) + .............. f ( a )} è 4ø
dx æ pö
22. ò cos x + 3 sin x
equals [2007] (d) x – log | cos ç x - ÷ | + c
è 4ø
p
æx pö
(a) log tan ç + ÷ + C
è 2 12 ø
27. ò [cot x] dx , where [ . ] denotes the greatest
0
integer function, is equal to : [2009]
(b) log tan çæ x - p ÷ö + C
è 2 12 ø (a) 1 (b) –1
p p
1 x p (c) - (d)
(c) log tan çæ + ÷ö + C 2 2
2 è 2 12 ø 28. Let p(x) be a function defined on R such that
1 æx pö p¢(x) = p¢(1 – x), for all x Î [0, 1], p (0) = 1 and p
(d) log tan ç - ÷ + C
2 è 2 12 ø 1

log t
x
(1) = 41. Then ò p( x) dx equals [2010]
Let F(x) = f (x) + f æç ö÷ ,where f ( x) = ò
1 dt , 0
23.
è xø 1+ t (a) 21 (b) 41
l
Then F(e) equals [2007] (c) 42 (d) 41
(a) 1 (b) 2
1
(c) 1/2 (d) 0, 8log(1 + x)
24. The solution for x of the equation 29. The value of ò 1+ x2
dx is [2011]
x 0
dt p p p
ò 2
=
2
is [2007] (a)
8
log 2 (b)
2
log 2
2 t t -1
(c) log 2 (d) p log 2
3 30. Let [.] denote the greatest integer function then
(a) (b) 2 2
2 1.5

ò x éë x

(c) 2 (d) None of these the value of û dx is :. [2011 RS]
1 1 0
sin x cos x 3
25. Let I = ò dx and J = ò dx. Then (a) 0 (b)
0
x 0
x 2
which one of the following is true? [2008] 3 5
(c) (d)
4 4
2 2 31. If the
(a) I > and J > 2 (b) I < and J < 2
5 tan x
3 3
òtan x - 2
dx = x + a ln sin x - 2 cos x + k ,
2 2 then a is equal to : [2012]
(c) I < and J > 2 (d) I > and J < 2 (a) – 1 (b) – 2
3 3
(c) 1 (d) 2
EBD_7764
w w w . c r a c k j e e . x y z
M-134 Mathematics
x
1
32. ò
If g (x) = cos 4t dt , then g (x + p) equals x+
36. The integral ò æç1 + x - 1 ö÷ e x dx is equal to
0 è xø
[2012]
[2014]
g ( x) 1 1
(a) (b) g (x) + g (p) x+ x+
g ( p) (a) ( x + 1) e x +c (b) - xe x +c
(c) g (x) – g (p) (d) g (x) . g (p) 1 1
x+ x+
(c) ( x - 1) e x +c (d) xe x +c
33. If ò f ( x)dx = y(x), then òx
5
f ( x 3 )dx is equal p
x x
to [2013] 37. The integral ò 1 + 4sin 2
2
- 4sin dx equals:
2
0

1é 3 [2014]
(a) x y ( x 3 ) - ò x 2 y ( x3 )dx ùû + C p
3ë (a) 4 3 - 4 (b) 4 3 - 4 -
1 3 3
(b) x y ( x3 ) - 3ò x 3 y ( x 3 )dx + C
3 2p
(c) p - 4 (d) - 4- 4 3
1 3 3
(c) x y ( x3 ) - ò x 2 y ( x3 )dx + C
3 dx
1é 3 38. The integral ò x 2 (x 4 + 1)3/4 equals : [2015]
(d) x y ( x 3 ) - ò x3 y ( x3 ) dx ùû + C
3ë 1
34. Statement-1 : The value of the integral 1 æ x 4 + 1ö 4
p /3
dx (a) - (x 4 + 1) 4 +c (b) - ç 4 ÷ + c
è x ø
ò is equal to p/6
p/61+ tan x
1
b b æ x 4 + 1ö 4 1
(c) ç 4 ÷ + c (d) (x 4 + 1) 4 + c
Statement-2 : ò f ( x )dx = ò f (a + b - x )dx.
è x ø
a a
[2013] 4
log x 2
(a) Statement-1 is true; Statement-2 is true; 39. The integral ò log x 2 + log(36 - 12 x + x 2 ) dx
Statement-2 is a correct explanation for 2
Statement-1. is equal to : [2015]
(b) Statement-1 is true; Statement-2 is true; (a) 1 (b) 6
(c) 2 (d) 4
Statement-2 is not a correct explanation for
Statement-1. 2x12 + 5x 9
40. The integral ò dx is equal to :
(c) Statement-1 is true; Statement-2 is false. 5 3 3
(d) Statement-1 is false; Statement-2 is true. (x + x +1
)
35. The intercepts on x-axis made by tangents to [2016]
x x5
(a) +C
thecurve, y = ò t dt , x Î R, which are parallel
5 3 2
0 (
2 x + x +1
)
to the line y = 2x, are equal to : [2013]
- x10
(a) ± 1 (b) ± 2 (b) +C
2
2 x5 + x3 + 1
(c) ± 3 (d) ± 4
( )
www.crackjee.xyz
Integrals M-135

-x 5 (a) –1 (b) –2
(c) 2
+C (c) 2 (d) 4
x5 + x3 +1
( ) n
42. Let In = ò tan x dx,(n > 1) . I4 + I6 = a tan5x + bx5
x10 + C, where C is constant of integration, then the
(d) 2
+C ordered pair (a, b) is equal to : [2017]
2 x5 + x3 +1
( )
where C is an arbitrary constant.
æ 1 ö
(a) çè - , 0÷ø
æ 1 ö
(b) çè - ,1÷ø
5 5
3p
æ1 ö æ1 ö
4
dx (c) çè , 0÷ø (d) çè , -1÷ø
5 5
41. The integral ò 1 + cos x
is equal to : [2017]
p
4
Answer Key
1 2 3 4 5 6 7 8 9 10 11 12 13 14 15
(a) (b) (d) (b) (c) (d) (d) (b) (d) (c) (b) (d) (b) (a) (d)
16 17 18 19 20 21 22 23 24 25 26 27 28 29 30
(b) (b) (b) (d) (c) (b) (c) (c) (d) (b) (c) (c) (a) (d) (c)
31 32 33 34 35 36 37 38 39 40 41 42
(d) (b, c) (c) (d) (a) (d) (b) (b) (a) (d) (c) (c)

1 lim n [I + I ]
\ In + In+2 = Þ n®¥ n n+2
10 p p n +1
1. (a) I= ò | sin x | dx = 10 ò | sin x | dx
= lim n.
1
= lim
n
0p 0
n ®¥ n + 1 n®¥ n + 1
= 10ò sin x dx
n
0 = lim =1
n®¥ æ 1ö
n ç1 + ÷
[Q | sin x| is periodic with period p and sin x > 0 è nø
if 0 < x < p]
2 1 2
p/2
ò éë x ùû dx = ò éë x ùû dx + ò é x 2 ù dx +
2 2
sin x dx = 20 [ - cos x ]0
p/2 3. (d) ë û
I = 20 ò = 20
0 0 1
0
3 2
p/4
éx2 ù + é x 2 ù dx
2. (b) I n + I n+ 2 = ò
n 2
tan x(1 + tan x )dx ò ë û ò ë û
2 3
0

p/ 4 p/4 1 2 3 2
é tan n +1 x ù
= ò
n
tan x sec x dx = ê 2
ú = ò 0dx + ò 1dx + ò 2dx + ò 3dx
0 êë n + 1 úû 0 0 1 2 3

1- 0 1 = [ x ]1 + [ 2 x ]
2 3
+ [ 3x] 3
2
= = 2
n +1 n + 1
EBD_7764
www.crackjee.xyz
M-136 Mathematics
= 2 -1 + 2 3 - 2 2 + 6 - 3 3 [Q given that f(a + b – x) = f(x)]
b
= 5- 3 - 2 2I = (a + b)ò f ( x)dx
p 2 x (1 + sin x) a
4. (b) ò-p 1 + cos 2 x
dx
( a + b)
b

2 ò
Þ I= f ( x )dx
p 2 x dx p x sin x a
= ò-p + 2ò dx
1 + cos x 2 -p 1 + cos 2 x f ¢( x )
6. (d) Given f ¢ ( x ) = f ( x) Þ =1
é a ù
f ( x)
p x sin x dx
= 0+4 ò0 ; êQ ò f ( x ) dx = 0ú Integrating
1 + cos2 x êë - a úû log f ( x) = x + c Þ f ( x ) = e x + c
if f(x) is odd
f (0) = 1 Þ f ( x ) = e x
a
= 2ò f ( x ) dx if f(x) is even. 1 1
\ ò f ( x ) g ( x) dx = ò e x ( x 2 - e x ) dx
0 0 0
p (p - x) sin (p - x ) 1 1
I = 4 ò0 dx = ò x 2 e x dx - ò e 2 x dx
1 + cos2 (p - x ) 0 0

I=4 ò0
p (p - x) sin x
dx = [ ] - 2[xe
1
x 2e x 0 x
- ex ]1
0 -
2
[ ]
1 2x
e
1
0
1 + cos 2 x
é e2 1 ù 2
p sin x dx x sin x dx = e - ê - ú - 2[e - e + 1] = e - e - 3
Þ I = 4pò - 4ò
0 1 + cos 2 x 1 + cos 2 x ëê 2 2 úû 2 2
1 1
p sin x I = ò x(1 - x )n dx = ò (1 - x )(1 - 1 + x) n dx
Þ 2I = 4p ò dx 7. (d)
1 + cos 2 x 0
0 0
put cos x = t Þ - sin x dx = dt 1
é x n +1 x n + 2 ù
1
n
-1 1
= ò (1 - x) x dx = ê - ú
1 1 êë n + 1 n + 2 úû 0
ò 1+ t2 dt = 2p ò 0
\ I = -2 p 2
dt
1 -1 1 + t =
1
-
1
1 n +1 n + 2
-1
= 2p éë tan t ùû n r
-1 8. (b) 1
Lim
n®¥ å ne n [Using definite integrals as
= 2p éë tan 1 - tan ( -1) ùû
-1 -1 r =1
limit of sum]
1
é p æ -p ö ù p
= 2p ê 4 - ç 4 ÷ ú = 2p. 2 = p2 = ò e x dx = e - 1
ë è ø û 0
b b 3 3
ò |1 - x ò|x
2 2
5. (c) I = ò xf ( x )dx = ò (a + b - x ) f (a + b - x )dx 9. (d) | dx = - 1| dx
a a -2 -2
b b x - 1 if ì 2 x £ -1
= (a + b )ò f (a + b - x)dx - ò xf (a + b - x )dx ïï
2 2
a a Now | x - 1|= í1 - x if -1 £ x £ 1
ï 2
ïî x - 1 if x ³1
b b
= (a + b )ò f ( x) dx - ò xf ( x)dx
a a
\ Integral is
www.crackjee.xyz
Integrals M-137
-1 1 3 x -x
e e
ò (x - 1)dx + ò (1 - x2 )dx +ò ( x 2 - 1)dx
2 12. (d) f ( x) = Þ f (- x ) =
-2 -1 1
1+ e x
1 + e- x
-1 1 3 1
é x3 ù é x3 ù é x3 ù =
x
= ê 3 - xú + ê x - 3 ú + ê 3 - xú e +1
ëê úû -2 ëê ûú -1 ëê ûú1 \ f ( x) + f ( - x) = 1 " x
f (a )

æ 1 ö æ 8 ö æ27 2 ö 1 Now I1 = ò xg{x(1 - x )}dx


= ç - + 1÷ - ç - + 2 ÷++æçç 2 -- 3÷ - æç - 1ö÷ f ( - a)
è 3 ø è 3 ø èè 3 3 ø è 3 ø f (a )
2 2 4 2 28
= + + +6+ = = ò (1 - x) g{x (1 - x )}dx
3 3 3 3 3 f ( - a)
p
2
(sin x + cos x )2 é b b ù
10. (c) I= ò dx ê using ò f ( x ) dx a = ò f ( a + b - x ) dx ú
0
1 + sin 2 x êë a a úû

We know [(sin x + cos x)2 = 1 + sin 2 x] , so = I 2 - I1 Þ 2 I1 = I 2


sin x sin( x - a + a)
p
2
13. (b) ò sin( x - a) dx = ò sin( x - a)
dx
(sin x + cos x )2
I= ò dx sin( x - a ) cos a + cos( x - a)sin a
(sin x + cos x ) =ò dx
0 sin( x - a )
p
= ò {cos a + sin a cot( x - a )}dx
2
= (cos a) x + (sin a) logsin( x - a) + C
ò
= (sin x + cosx)dx
\ A = cosa, B = sin a
0
dx dx
14. (a) ò
cos x - sin x ò
é pù =
êQ sin x + cos x > 0 if 0 < x < 2 ú æ pö
2 cos ç x + ÷
ë û è 4ø
p
1 æ pö
or I = [ - cos x + sin x ] = 2 2
0
=
2
ò sec çè x + 4 ÷ø dx
p 1 æ p x pö
= log tan ç + + ÷ + C
11. (b) Let I = ò xf (sin x )dx 2 è 4 2 8ø
0
é æ p xö ù
êQ ò sec x dx = log tan çè + ÷ø ú
p
= ò (p - x) f (sin x)dx ë 4 2 û
0 1 æ x 3p ö
p = log tan ç + ÷ + C
2 è2 8 ø
p 2
\ 2 I = pò f (sin x)dx = p.2 ò f (sin x )dx (log x - 1) 2
2 0
15. (d) ò (1 + (log x)2 )2 dx
p
1 + ( log x ) - 2 log x
2
2 =ò dx
\ I = p ò f (sin x )dx Þ A = p é1 + ( log x )2 ù
2

0 ë û
é 1 2 log x ù
= òê 2
- 2 2ú
dx
êë (1 + (log x) ) (1 + (log x) ) úû
EBD_7764
www.crackjee.xyz
M-138 Mathematics
é et p p
2t et ù
= òê - ú dt put log x = t 2 2
2
ëê1 + t (1 + t 2 )2 ûú Þ I = 2 ò dx - 2 ò cos x dx
2

0 0
Þ dx = et dt
æ pö p
t é 1 2t ù Þ I + I = 2ç ÷ = p Þ I =
= òe ê 2
- ú dt è 2ø 2
ëê1 + t (1 + t 2 )2 ûú 6
x
é Which is of the form e ( f ( x) + f '( x ) ) dxù 18. (b) I =ò dx … (1)
ò
x
ë û 3
9- x + x
t
e x
= +c = +c 6
9- x
1+ t2 1 + (log x ) 2 I =ò dx … (2)
3
9-x+ x
1 1
2 3
16. (b) I1 = ò 2 x dx, I 2 = ò 2 x dx, b b
0 0
[ using ò f ( x)dx = ò f (a + b - x )dx ]
1 1 a a
x2 x3 Adding equation (1) and (2)
I3 = ò 2 dx, I 4 = ò 2 dx " 0 < x < 1, x > x 2 3
6
0 0 3
2 I = ò dx = 3 Þ I =
1 1 2
2 3 3
x x
Þ ò 2 dx > ò 2 dx Þ I1 > I 2 p p
0 0
19. (d) I = ò xf (sin x)dx = ò (p - x ) f (sin x)dx
p
cos 2 x 0 0
17. (b) Let I = ò 1 + a x dx ....(1) p
-p = p ò f (sin x) dx - I
p
cos 2
( -x) 0
p
= ò -x
dx
Þ 2 I = p ò f (sin x )dx
-p 1 + a
0
é b b ù pp p /2
I = ò f (sin x )dx = p ò f (sin x )dx
ê Using ò f ( x ) dx = ò f ( a + b - x ) dx ú 20 0
êë a a úû p /2

p
= p ò f (cos x ) dx
cos 2 x 0
= ò x
dx ....(2) -
p
-p 1 + a 2
Adding equations (1) and (2) we get
20. (c) I= ò [( x + p )3 + cos 2 ( x + 3p )]dx
3p
p æ 1+ ax ö p -
2 2
2 I = ò cos x ç x÷
dx = ò cos 2 x dx Put x + p = t
-p è 1 + a ø -p p p
p 2 2
= 2ò cos 2 x dx 3
+ cos 2 t ]dt = 2 2
I= ò [t ò cos tdt
0 p p
- -
p p 2 2
2 2 [using the property of even and odd
= 2 ´ 2 ò cos 2 x dx = 4 ò sin 2 x dx function]
0 0 p
2
p p p
= ò (1 + cos 2t )dt = 2 + 0
ò (1 - cos )
2 2
Þ I = 2 ò sin x dx = 2
2 2 0
x dx
0 0
www.crackjee.xyz
Integrals M-139
21. (b) Let a = k + h where k is an integer such that 1 1 dz
[a] = k and 0 £ h < 1 \ Put = z Þ - 2 dt = dz Þ dt = – 2
t t z
a 2 3 and limit for t = 1 Þ z = 1 and for t = 1/e
\ò [ x] f '( x )dx = ò 1 f '( x )dx + ò 2 f '( x )dx+ Þ z=e
1 1 2
æ 1ö
k k +h log ç ÷
e è zø æ dz ö
... ò (k - 1)dx + ò kf '( x )dx \ I= ò1 1 çè - 2 ÷ø
z
k -1 k 1+
z
= {f (2) – f (1)} + 2{f (3) – f (2)} + 3{f (4) –f
(3)}+ ........ + (k – 1) {f(k) – f(k – 1)} + k{f(k e (log1 - log z ).z æ dz ö
+ h) – f(k)} =ò çè - 2 ÷ø
1 z +1 z
= – f (1) – f (2) – f (3) ......... – f (k) + kf (k + h)
= [a] f (a) - { f (1) + f (2) + f (3) + ¼ f ([a])} e log z æ dz ö
=ò - ç- ÷ [Q log1 = 0]
dx 1 ( z + 1) è z ø
22. (c) I = ò
cos x + 3 sin x e log z
dx =ò dz
Þ I =ò 1 z ( z + 1)
é1 3 ù
2 ê cos x + sin x ú e log t
ë2 2 û \ I= ò dt
1 t (t + 1)

b b
1
= ò
dx [By property òa f (t )dt = òa f ( x)dx ]
2 é p p ù Equation (A) becomes
êësin 6 cos x + cos 6 sin x úû e log t e log t
1
= .ò
dx F(e) = ò1 1 + t dt + ò1 t (1 + t ) dt
2 æ pö
sin ç x + ÷ e t.log t + log t e (log t )(t + 1)
è 6ø =ò dt = ò dt

1 t (1 + t ) 1 t (1 + t )
1 æ

Þ I= . cosec ç x + ÷ dx
è 6ø e log t
But we know that
Þ F(e) = ò1 t
dt

1
ò cosec x dx = log | (tan x / 2) | + C Let log t = x \
t
dt = dx
1 x p
\ . log tan æç + ö÷ + C
I=
2 è 2 12 ø [for limit t = 1, x = 0 and t = e, x = log e = 1]
æ 1ö é x2 ù
1
23. (c) Given F (x) = f (x) + f ç ÷ ,where 1
è xø \ F(e) =
ò0 x dx F(e) = ê ú
ëê 2 ûú 0
x logt
f (x) = ò1 1+ t
dt
1
Þ F(e) =
2
\ F(e) = f (e) + f æç 1 ö÷
è eø x dt p
24. (d) ò 2t 2
t -1
=
2
e log t 1/ e log t
Þ F(e) =
ò1 1+ t
dt + ò
1 1+ t
dt ....(A)
\ ésec-1 t ù
x
=
p
ë û 2 2
1/ e logt
Now for solving, I = ò1 1+ t
dt
EBD_7764
www.crackjee.xyz
M-140 Mathematics
é ù p
ò0 [cot x] dx
dx 1
êQ
ëê
ò
x x2 - 1
= sec -1 x ú
ûú
27. (c) Let I = ....(1)
p p

Þ sec–1 x – sec–1 2 =
p = ò0 [ cot (p - x) ] dx = ò0 [- cot x] dx ....(2)
2 Adding two values of I in eqn s (1) & (2),
p p p p We get
Þ sec–1x – = Þ sec–1x = +
4 2 2 4 p

3p 3p
2I = ò0 ([ cot x] + [ - cot x]) dx
Þ sec–1x = Þ x = sec 4 p
4 = ò ( -1)dx
Þ x= – 0
2
[Q[ x] + [- x] = -1, if x Ïz and [x] + [–x] =
sin x
25. (b) We know that < 1 , for x Î (0, 1) 0, if x Î z]
x p
sin x = [ - x ]p0 = -p Þ I = –
< x on x Î (0, 1) 2
Þ
x 28. (a) p '( x) = p '(1 - x)
1 1 1 Þ p ( x ) = - p (1 - x ) + c
sin x é 2 x3 / 2 ù
at x = 0
Þ ò dx < ò xdx = ê ú
0
x 0 ëê 3 ûú 0 p(0) = – p(1) + c Þ 42 = c
Now, p( x ) = - p(1 - x) + 42
1
sin x 2 2
Þ ò x
dx <
3
ÞI<
3
Þ p ( x ) + p (1 - x) = 42
0 1

cos x 1 Þ I = ò p ( x)dx ...(i)


Also < for xÎ(0, 1) 0
x x 1
1 1 Þ I = ò p(1 - x)dx ...(ii)
cos x 1
Þ ò x
dx < ò x –1/ 2 dx = éë 2 x ùû = 2
0
0
on adding (i) and (ii),
0 0
1
1
cos x 2 I = ò (42) dx Þ I = 21
Þ ò x
dx < 2 Þ J < 2
0
0
sin xdx p 1
26. (c) Let I = 2ò put x – = t 29. (d) 8 log(1 + x )
æ p ö 4 I =ò dx
sin ç x – ÷ 1+ x2
è 4ø 0
Put x = tan q,
æ pö \
dx
= sec2 q Þ dx = sec 2 q dq
sin ç t + ÷
è 4ø dq
Þ dx = dt Þ I = 2 ò dt p /4
sin t \ I =8 log(1 + tan q )
2 æ sin t + cos t ö
ò 2
1 + tan q
.sec 2 q dq
= ò ç sin t ÷ø dt
2 è p /4
0

Þ I = ò (1 + cot t )dt = t + log |sin t| + c1


I =8 ò log(1 + tan q )dq ...(i)
0
p æ pö p /4
= x – + log sin ç x – ÷ + c1 é æp öù
4 è 4ø =8 ò log ê1 + tan ç - q ÷ ú dq
ë è4 øû
0
æ pö æ pö
= x + log sin çè x – ÷ø + c çè where c = c1 – ÷ø p /4
é 1 - tan q ù
4 4
=8 ò log ê1 + ú dq
ë 1 + tan q û
0
www.crackjee.xyz
Integrals M-141
p /4 = I1 + I2
é 2 ù
=8 ò log ê ú dq
ë1 + tan q û
where
cos x + 2 sin x
ò sin x - 2 cos x dx
0
p /4
ò
I1 = dx and I 2 = 2
=8
ò [log 2 - log(1 + tan q )]dq put sin x – 2cos x = t
0
Þ (cos x + 2sin x) dx = dt
p /4
I = 8.(log 2)[ x]p0 / 4 - 8 dt
ò log(1 + tan q ) dq \ I2 = 2
t ò
= 2 ln t + C
0
p = 2 ln (sin x–2cos x) + C
I = 8. .log 2 - I [From equation (i)] Hence,
4
Þ 2 I = 2p log 2
\ I = p log 2
ò
I1 + I 2 = dx + 2 ln ( sin x - 2 cos x ) + c
= x + 2ln |(sin x – 2 cos x)| + k Þ a =2
30. (c)
1.5 1 2 1.5 x +p
ò x éë x ùû dx = ò x[ x ]dx + ò x éë x ùû dx + ò x éë x ùû dx
2 2 2 2
0 0 1 2
32. (b, c) g ( x + p ) = ò cos 4t dt
0
1 2 1.5 x p+ x
= ò x.0 dx + ò xdx + ò
0 1 2
2xdx
ò
= cos 4t dt + ò cos 4t dt
0 x
2 p
é x2 ù 2 1.5
= 0 + ê ú + éë x ùû 2
ë 2 û1
ò
= g ( x ) + cos 4t dt
0
1 1 (from graph of cos 4t, it is clear that
= ( 2 - 1) + ( 2.25 - 2) = + 0.25 p+ x p
2 2
1 1 3 ò ò
cos 4t dt = cos 4t dt
= + = x 0
2 4 4 = g (x) + g (p) = g (x) – g (p)
sin x (Qfrom graph of cos 4t, g (p) = 0)
5
5 tan x cos x dx
31. (d) òtan x - 2
dx =
sin x
-2
ò 33. (c) Let ò f ( x)dx = y ( x)
cos x Let I = ò x 5 f ( x 3 )dx
æ 5 sin x cos x ö
= ç ò ´ ÷ dx
è cos x sin x - 2 cos x ø
put x3 = t
Þ 3x2dx = dt
5 sin x dx 1
= ò
sin x - 2 cos x
I=

3 · x 2 · x 3 · f ( x3 ) · dx
æ 4 sin x + sin x + 2 cos x - 2 cos x ö
= ç
è ò sin x - 2 cos x
÷ dx
ø =
1
ò
1
tf (t )dt = ét ò f (t )dt - ò f (t )dt ù
3 3ë û
( sin x - 2 cos x ) + ( 4 sin x + 2cos x )
= ò sin x - 2cos x
dx 1
= éët y (t ) - ò y (t )dt ùû
3
( sin x - 2 cos x ) + 2 ( cos x + 2 sin x )
= ò ( sin x - 2 cos x )
dx
=
1é 3
x y ( x3 ) - 3ò x 2 y ( x3 )dx ù + c
3ë û
sin x - 2cos x æ cos x + 2sin x ö
= ò sin x - 2cos x dx + 2òçè sin x - 2cos x ÷ø dx =
1 3
x y ( x 3 ) - ò x 2 y ( x3 )dx + c
3
cos x + 2 sin x
ò
= dx + 2 ò sin x - 2cos x dx
EBD_7764
w w w . c
M-142 Mathematics
p /3 æ 1 ö x+ 1
dx x+ 1 x - ò x ç 1 - ÷ e x dx
34. (d) Let I = ò = x.e
è x2 ø
p/61+ tan x
æ 1 ö x+ 1
p /3
dx + ò ç x - ÷ e x dx
è xø
= ò æp ö
p/6
1 + tan ç - x÷ x+
1
è2 ø æ 1 ö x+ 1
= x.e x - ò ç x - ÷ e x dx
è xø
p /3
tan x dx
= ò …(1) æ 1 ö x+ 1
+ ò ç x - ÷ e x dx
p/6 1+ tan x
è xø
Also, given
1
p /3 x+
tan x dx = xe x +C
I= ò …(2)
p / 6 1 + tan x p
x x
By adding (1) and (2), we get 37. (b) Let I = ò 1 + 4 sin 2 - 4sin dx
2 2
p/3 0
2I= ò dx p
x
p/6
= ò 2 sin 2 - 1 dx
1 ép pù p 0
Þ I= - = ,
2 êë 3 6 úû 12 p /3
æ xö æ x ö
p
statement-1 is false = ò ç 1 - 2 sin 2 ÷ dx + ò ç 2 sin 2 - 1÷ dx
0
è ø p/3
è ø
b b

ò f ( x )dx = ò f (a + b - x ) dx é x 1 x p
a a êQ sin 2 = 2 Þ 2 = 6
ë
It is fundamental property.
p x 5p 5p ù
x Þx= , = Þx= ú
35. (a) Since, y = ò t dt , x ÎR 3 2 6 3û
0 p /3 p
é xù é x ù
dy = ê x + 4cos ú + ê -4 cos - x ú
therefore = x ë 2 û0 ë 2 ûp / 3
dx
p 3 æ 3 pö
dy = +4 - 4 +ç0 - p + 4 + ÷
But from y = 2x, =2 3 2 ç 2 3 ÷ø
dx è
Þ |x|=2 Þ x=±2 p
±2 = 4 3 -4-
3
Points y = ò t dt = ± 2
dx
dx
\
0
equation of tangent is
38. (b) I= ò = ò x 3 (1 + x -4 )3/ 4
x 2 (x 4 + 1)3/ 4
y – 2 = 2(x – 2) or y + 2 = 2(x + 2)
Let x–4 = y
Þ x-intercept = ± 1. Þ –4x–3 dx = dy
-1 3
æ 1 ö x+ 1 Þ dx = x dy
36. (d) Let I = ò ç1 + x - ÷ e x dx 4
è xø
-1 x 3dy -1 dy
4 ò x 3 (1 + y)3 / 4 4 ò (1 + y)3/ 4
x+ 1 æ 1 ö x+ 1 \I= =
= òe x dx
+ ò çè x - ÷ø e x dx
x
www.crackjee.xyz
Integrals M-143
-1 10
= ´ 4(1 + y)1/ 4 = -(1 + x -4 )1/ 4 + C =
x
+C
4
2( x +x3 +1) 2
5
1/ 4
æ x 4 + 1ö
= –ç ÷ +C 3p
è x4 ø 4
dx
4
log x 2
41. (c) I= ò 1 + cos x ...(i)
p
39. (a) I=ò dx
2
2 log x + log(36 -12x + x 2 ) 4
3p
4
log x2 4
dx
I=ò dx .
2
2 log x + log(6 - x)
2 ...(i) I= ò 1 - cos x ...(ii)
p
4
log(6 - x)2 4
I=ò dx ...(ii)
b b
2
2 log(6 - x) + log x
2 Using ò f (x)dx = ò f (a + b - x) dx
Adding (i) and (ii) a a
4
Adding (i) and (ii)
2I = ò dx = [ x ]2 = 2
4
3p
4
2 2
2I = ò sin 2 x
dx
I=1 p
4
2x12 + 5x9 3p
40. (d) ò (x 5 + x3 + 1)3 dx 4
ò cosec
2
Dividing by x15 in numerator and denominator I= x dx
2 5 p
+ 6 dx
3 4
ò æ x 1 x 1 ö3 é 3p pù
ç1 + 2 + 5 ÷ I = – (cot x)3pp/ 4/ 4 = – ê cot - cot ú = 2
è x x ø ë 4 4û
1 1
Substitute 1+ + 5 =t n
x 2
x 42. (c) In = ò tan x dx, n > 1
æ –2 5 ö Let I = I4 + I6
Þ ç – dx = dt
è x 3 x 6 ÷ø = ò (tan 4 x + tan 6 x)dx = ò tan 4 x sec2 x dx
æ2 5 ö Let tan x = t
Þ çç + ÷÷ dx = - dt
è x3 x 6 ø Þ sec2 x dx = dt
This gives,
\ I = ò t 4 dt
2 5
+ 6 dx
x3 x – dt 1 t5
ò 3
= ò t3
=
2t 2
+C =
5
+C
æ 1 1ö
çè1 + 2 + 5 ÷ø
x x 1 5
= tan x + C Þ On comparing, we have
1 5
= +C
æ 1 1 ö2 1

ç1+ 2 + 5 ÷
÷ a= ,b=0
è x x ø 5
EBD_7764
www.crackjee.xyz
M-144 Mathematics

Applications of Integrals 23
1. If y = f(x) makes +ve intercept of 2 and 0 unit on 7. Let f (x) be a non – negative continuous function
x and y axes and encloses an area of 3/4 square such that the area bounded by the curve y = f (x),
2 p p
x - axis and the ordinates x = and x = b > is
unit with the axes then ò xf ¢ ( x)dx is [2002] 4 4
0 æ p ö æpö
ç b sin b + cos b + 2b ÷. Then f ç ÷ [2005]
(a) 3/2 (b) 1 è 4 ø è2ø
(c) 5/4 (d) –3/4
æp ö æp ö
2. The area bounded by the curves y = lnx, y = ln (a) ç + 2 - 1÷ (b) ç - 2 + 1÷
|x|,y = | ln x | and y = | ln |x| | is [2002] è4 ø è4 ø
(a) 4sq. units (b) 6 sq. units æ p ö æ p ö
(c) ç1 - - 2 ÷ (d) ç1 - + 2 ÷
(c) 10 sq. units (d) none of these è 4 ø è 4 ø
3. The area of the region bounded by the curves 8. The area enclosed between the curves y2 = x
and y = | x | is [2007]
y = x - 1 and y = 3 - x is [2003]
(a) 1/6 (b) 1/3
(a) 6 sq. units (b) 2 sq. units (c) 2/3 (d) 1
(c) 3 sq. units (d) 4 sq. units. 9. The area of the plane region bounded by the
4. The area of the region bounded by the curves curves x + 2y2 = 0 and x + 3y2 = 1is equal to
y =| x - 2 |, x = 1, x = 3 and the x-axis is [2004] [2008]
(a) 4 (b) 2 5 1
(a) (b)
(c) 3 (d) 1 3 3
5. The area enclosed between the curve y = 4
(c) 2 (d)
log e ( x + e) and the coordinate axes is [2005] 3 3
10. The area of the region bounded by the parabola
(a) 1 (b) 2 (y – 2)2 = x –1, the tangent of the parabola at
(c) 3 (d) 4 the point (2, 3) and the x-axis is: [2009]
6.
2 2
The parabolas y = 4 x and x = 4 y divide the (a) 6 (b) 9
square region bounded by the lines x = 4, y = 4 (c) 12 (d) 3
11. The area bounded by the curves y = cos x and
and the coordinate axes. If S1 , S2 , S3 are y = sin x between the ordinates x = 0 and x =
respectively the areas of these parts numbered 3p
from top to bottom; then S1 : S2 : S3 is [2005] is [2010]
2
(a) 1 : 2 : 1 (b) 1 : 2 : 3 (a) 4 2 + 2 (b) 4 2 - 1
(c) 2 : 1 : 2 (d) 1 : 1 : 1
(c) 4 2 + 1 (d) 4 2 - 2
www.crackjee.xyz
Applications of Integrals M-145

12. The area of the region enclosed by the curves 16. The area of the region described by

1
A= {( x, y ) : x 2
}
+ y 2 £ 1 and y 2 £ 1 - x is:
y = x, x = e, y = and the positive x-axis is [2011]
x [2014]
3 p 2 p 2
(a) 1 square unit (b) square units (a) - (b) +
2 2 3 2 3
5 1
(c) square units (d) square unit p 4 p 4
2 2 (c) + (d) -
2 3 2 3
13. The area bounded by the curves [2011 RS]
17. The area (in sq. units) of the region described by
y 2 = 4 x and x2 = 4y is: {(x, y) : y2 £ 2x and y ³ 4x – 1} is [2015]
32 16 15 9
(a) sq units (b) sq units (a) (b)
3 3 64 32
8 7 5
(c) sq. units (d) 0 sq. units (c) (d)
3 32 64
y
14. The area between the parabolas x 2 = and 18. The area (in sq. units) of the region {(x, y) :
4 y2 ³ 2x and x2 + y2 £ 4x, x ³ 0, y ³ 0} is :
x2 = 9y and the straight line y = 2 is : [2012] [2016]
10 2 4 2 p 2 2
(a) 20 2 (b) (a) p- (b) -
3 3 2 3
20 2
(c) (d) 10 2 4 8
3 (c) p- (d) p-
15. The area (in square units) bounded by the curves 3 3
19. The area (in sq. units) of the region
y = x , 2y – x + 3 = 0, x-axis, and lying in the
first quadrant is : [2013] {(x, y) : x ³ 0, x + y £ 3, x2 £ 4y and y £ 1 + x }
(a) 9 (b) 36 is : [2017]
27 5 59
(c) 18 (d) (a) (b)
4 2 12
3 7
(c) (d)
2 3

Answer Key
1 2 3 4 5 6 7 8 9 10 11 12 13 14 15
(d) (a) (d) (d) (a) (d) (d) (a) (d) (b) (d) (b) (b) (c) (a)
16 17 18 19
(c) (b) (d) (a)
EBD_7764
www.crackjee.xyz
M-146 Mathematics

2
3. (d)
3
1. (d) We have ò f ( x)dx = ; Now,,
4 y=–x+1 (0,3)
0
(–1,2) y=x–1
2 2 2 (2,1)
ò xf ¢( x)dx = xò f ¢( x)dx - ò f ( x)dx
0 0 0 y=3+x (1,0) y=3–x

2 3 3
= [ x f ( x)]0 - = 2 f (2) - 0
4 4 A= ò { (3 + x) - (- x + 1)} dx +
3 3 -1
= 0 - (Q f (2) = 0) = - .
4 4
1 2
2. (a) First we draw each curve as separate graph
y=ln|x|
ò { (3 - x) - (- x + 1)} dx + ò { (3 - x) - ( x - 1)} dx
y=lnx 0 1
0 1 2

x x
= ò (2 + 2 x)dx + ò 2dx + ò (4 - 2 x)dx
-1 0 1
(1,0) (–1,0) (1,0)
0 2
+ [ 2 x] + é4 x - x2 ù
é2 x - x 2 ù 1
=
ë û -1 0ë û1
= 0 - ( -2 + 1) + (2 - 0) + (8 - 4) - (4 - 1)
y=|l n x| y=|l n |x|| = 1 + 2 + 4 - 3 = 4 sq. units

4. (d) The required area is shown by shaded


x x region
(1,0) (–1,0) (1,0)

Y
Note: Graph of y = | f(x) | can be obtained from the
graph of the curve y = f(x) by drawing the mirror (y = 2 – x) (y = x – 2)
image of the portion of the graph below x-axis,
with respect to x-axis. Clearly the bounded area
is as shown in the following figure.
y

y = –ln (–x) y = –ln x


X
3
x
–1 O 1 Required Area
y = ln (–x) y = ln x
3 3
1
A = ò | x - 2 |dx = 2ò ( x - 2)dx
Required area = 4 ò (- lnx )dx
1 2
0
= - 4 [ x l n x - x ]0 = 4 sq. units 3
1
é x2 ù
= 2 ê - 2xú = 1
ëê 2 ûú 2
www.crackjee.xyz
Applications of Integrals M-147

5. (a) The graph of the curve y = log e ( x + e ) is é 3 ù


4
4æ 2ö
as shown in the fig. x ê 2 x 2 x3 ú
y Also S2 = ò ç 2 x - ÷ dx = ê - ú

4ø ê 3 12 ú
êë 2 úû 0
y=loge(x+e)
0) 4 16 16
= ´8 - = sq. units
(1–e,
(0,1)
3 3 3
x
0 \ S1 : S2 : S3 = 1:1:1
7. (d) Given that
b
p
x+e=0
ò f ( x )dx = b sin b +
4
cos b + 2b
p/4
Required area
0 0 Differentiating w. r . t b
A= ò ydx = ò log e ( x + e)dx p
1-e 1- e f ( b ) = b cos b + sin b – sin b + 2
4
put x + e = t Þ dx = dt also At x = 1 – e,
t=1 æ pö æ p ö p p
f ç ÷ = ç1 - ÷ sin + 2 = 1 - + 2
At x = 0, t = e è 2ø è 4ø 2 4
e 8. (a) The area enclosed between the curves
\ A = ò loge tdt = [ t log e t - t ]1
e
y2 = x and y = | x |
1 From the figure, area lies between y2 = x
e - e - 0 +1 = 1 and y = x
Hence the required area is 1 square unit. y=x
Y
6. (d) Inter section points of x = 4 y and 2 (1, 1)
y = -x A y2 = x
y 2 = 4 x are (0, 0) and (4, 4). The graph is y2
as shown in the figure. y1
y X' X
2 (0, 0) O (1, 0)
x =4y
y2=4x
y=4 S (4, 4)
1
S2 Y'
S3
x \ Required area = 1
(0, 0) 0
x=4 ò0 ( y2 - y1 )dx
1
1 é x3 / 2 x 2 ù
= ò ( x - x)dx = ê - ú
0 ëê 3 / 2 2 úû
0
By symmetry, we observe
\ Required area = é x3 / 2 ù - 1 é x 2 ù
2 1 1
4 ë û ë û
3 0 2 0
S1 = S3 = ò ydx 2 1 1
0 = - =
4 3 2 6
4 2 é x3 ù x
x 16 2 2
=ò dx = ê ú = sq. units 9. (d) x + 2 y = 0 Þ y = –
4 2
0 ëê 12 ûú0 3
[Left handed parabola with vertex at (0, 0)]
1
x + 3 y 2 = 1 Þ y 2 = – ( x – 1)
3
[Left handed parabola with vertex at (1, 0)]
EBD_7764
www.crackjee.xyz
M-148 Mathematics
Solving the two equations we get the 3
= 3 + ò ( y - 2) + 1 dy
2
points of intersection as (–2, 1), (–2, –1) 0

Y 3
é ( y - 2)3 ù
A (–2, 1) = 3+ ê + yú
ëê 3 úû 0

D é1 8ù
X = 3 + ê + 3 + ú = 3 + 6 = 9 Sq. units
X´ C (1, 0) ë3 3û
B 11. (d)
(–2, –1)
cos x sin x

5p 3p
The required area is ACBDA, given by
p 4 2
1 é 3 ù1 0 p p 2p
5y
= ò (1 – 3 y – 2 y )dy = ê y – 3 ú
2 2
4 2
–1 ëê ûú –1
Area above x-axis = Area below x-axis
æ 5ö æ 5ö
= çè1 – 3 ÷ø – ç –1 + ÷ = 2 ´ 2 = 4 sq. units. \ Required area
è 3ø 3 3
é p ù
10. (b) The given parabola is (y – 2)2 = x – 1 ê 2 ú
Vertex (1, 2) and it meets x –axis at (5, 0) ê p (cos x - sin x)dx + ò (sin x - cos x)dx + ú
Also it gives y2 – 4y – x + 5 = 0 ê4 p ú
So, that equation of tangent to the parabola ê
= 2êò 4
ú
ú
at (2, 3) is ê0 p ú
1
y.3 – 2 (y + 3) – (x + 2) + 5 = 0
ê
ê
ò sin x dx ú
ú
2 p
êë 2 úû
or x – 2y + 4 = 0
which meets x-axis at (– 4, 0). = 4 2 -2
In the figure shaded area is the required 12. (b) Area of required region AOBC
area. 1 e
1
Let us draw PD perpendicular to y – axis. = ò xdx + ò dx
0 1
x
Y
1 3
(2, 3) = + 1 = sq. units
D 2 2
P
(0,2)A y x=e

B y=x
X
(–4, 0) O C(5,0)

(1, 1)
Then required area = Ar DBOA + Ar (OCPD) A
– Ar (DAPD) 1
B e, e
1 3 1 O
= ×4×2+
2 ò 0
xdy - ´ 2 ´ 1
2 C
x
www.crackjee.xyz
Applications of Integrals M-149

13. (b) 3 3 2
é2 1 2 2 ùú
ê
= 2 ´ 3.y - ´ .y
2
ê3 2 3 ú
ë û0

é 3 1 3ù 3 2
5
= 2 ê2y 2 - y 2 ú = 2 ´ y 2
ê 3 ú 3
ë û 0
5 20 2
= 2. 2 2 =
3 3
15. (a) Given curves are
y= x …(1)
and 2y – x + 3 = 0 …(2)
4
æ x2 ö On solving both we get y = –1, 3
Area = ò ç 2 x - ÷ dx Y


4
é æ x3/ 2 ö x3 ù 4 64
= ê2 ç ÷ - 12 ú = ´ 8 -
ëê è 3 / 2 ø ûú 0 3 12
32 16 X
= - 16 = sq. units
3 3
2 y
14. (c) Given curves x = and x2 = 9y are the
4
parabolas whose equations can be written 3

ò { (2 y + 3) - y } dy
2
1 Required area =
as y = 4x2 and y = x2 .
9 0
3
Y 1 2 y3
y = 4x2 y = 9 x = y2 + 3y – = 9.
3 0
y=2
16. (c) Given curves are x2 + y2 = 1 and y2 = 1 –x.
Intersecting points are x = 0, 1
X Area of shaded portion is the required area.
So, Required Area = Area of semi-circle
Also, given y = 2. + Area bounded by parabola
Now, shaded portion shows the required 1
area which is symmetric. pr 2
= + 2 ò 1 - xdx
2 2
æ yö 0
\
è
ò
Area = 2 çç 9y - ÷ dy
4 ÷ø p
1
0 = + 2 ò 1 - x dx (Q radius of circle = 1)
2 2
æ yö 0
ò
Area = 2 ç 3 y -
ç
è
÷ dy
2 ÷ø é (1 - x ) 3 2 ù
1
0 p
= + 2ê ú
é 1
+1
1 ù
+1
2 2 ê -3 ú
ë 2 û0
ê 3.y 2 1 y2 ú
= 2ê - . ú p 4 p 4
ê 1 +1 2 1 +1ú = - (-1) = + Sq. unit
ëê 2 2 ûú 0 2 3 2 3
EBD_7764
www.crackjee.xyz
M-150 Mathematics
17. (b) Required area Points of intersection of the two curves are (0,
0), (2, 2) and (2, –2)
Area = Area (OAB) – area under parabola (0 to 2)

B æ 1 ,1ö p ´ (2) 2
2
çè ÷ = - ò 2 x dx = p - 8
2 ø
4 3
0
O C æ 1 - 1ö
çè , ÷ø 19. (a)
8 2
x+y=3

y=1+ x
1 1 (1, 2)
y +1 y2 4y = x 2
= ò 4
dy - ò 2
dy
(0, 1)
-1/ 2 -1/ 2

1 1
1 é y2 ù 1 é y3 ù (2, 1)
= ê + yú - ê ú
4 ëê 2 úû -1/ 2 2 ëê 3 ûú -1/ 2
(0, 0) (1, 0) (2, 0)

1 é 3 3 ù 9 15 9 27 9
+ - = - = =
4 êë 2 8 úû 48 32 48 96 32
=
Area of shaded region
A (2, 2) 1 2
x2
2
=ò (1 + x )dx + ò (3 - x)dx - ò 4 dx
0 1 0

18. (d) 3 ù1
O B 2 2
x2 ú x2 ù x3 ù 5
= x] + + 3x ]
1 2
- ú - ú = sq.units
3 úú
0 1 Sq.unit
2 úû 12 úû 2
1 0
2 úû 0
(2, –2)
www.crackjee.xyz
Differential Equations M-151

Differential Equations 24
1. The order and degree of the differential 5. The differential equation for the family of circle
2/3 x 2 + y 2 - 2ay = 0, where a is an arbitrary
æ dy ö d3y
equation ç1 + 3 ÷ =4 are [2002] constant is [2004]
è dx ø dx3
(a) ( x 2 + y 2 ) y ¢ = 2 xy
2
(a) (1, ) (b) (3, 1)
3 (b) 2( x 2 + y 2 ) y ¢ = xy
(c) (3, 3) (d) (1, 2)
(c) ( x 2 - y 2 ) y ¢ = 2 xy
2
d y
2. The solution of the equation = e -2 x (d) 2( x 2 - y 2 ) y ¢ = xy
dx 2
6. Solution of the differential equation
[2002]
ydx + ( x + x 2 y )dy = 0 is [2004]
e -2 x e -2 x
(a) (b) + cx + d 1
4 4 (a) log y = Cx (b) - + log y = C
xy
1 -2 x 1 -4 x
(c) e + cx 2 + d (d) e + cx + d
4 4 1 1
(c) + log y = C (d) - = C
3. The degree and order of the differential xy xy
equation of the family of all parabolas whose 7. The differential equation representing the family
axis is x - axis, are respectively.
(a) 2, 3 (b) 2, 1 [2003] of curves y 2 = 2c ( x + c ) , where c > 0, is a
(c) 1, 2 (d) 3, 2. parameter, is of order and degree as follows :
4. The solution of the differential equation [2005]
-1 dy (a) order 1, degree 2 (b) order 1, degree 1
(1 + y 2 ) + ( x - e tan y
) = 0 , is [2003] (c) order 1, degree 3 (d) order 2, degree 2
dx
-1 -1 dy
(a) xe 2 tan y
= e tan y
+k
8. If x = y (log y – log x + 1), then the solution
dx
-1 of the equation is [2005]
(b) ( x - 2) = ke 2 tan y
æ xö æ yö
-1 -1 (a) y log ç ÷ = cx (b) x log ç ÷ = cy
(c) 2 xe tan y
= e 2 tan y
+k è yø è xø
-1
(d) xe tan y
= tan -1 y + k æ yö æ xö
(c) log ç ÷ = cx (d) log ç ÷ = cy
è xø è yø
EBD_7764
www.crackjee.xyz
M-152 Mathematics
9. The differential equation whose solution is dV (t )
by differential equation = - k (T - t ),
2 2 dt
Ax + By = 1 where A and B are arbitrary
where k > 0 is a constant and T is the total life in
constants is of [2006]
years of the equipment. Then the scrap value
(a) second order and second degree V(T) of the equipment is [2011]
(b) first order and second degree
(c) first order and first degree kT 2 k (T - t )2
(a) I - (b) I -
(d) second order and first degree 2 2
10. The differential equation of all circles passing 21
through the origin and having their centres on (c) e– kT (d) T -
k
the x-axis is [2007] 16. Consider the differential equation [2011RS]
dy (b) 2 dy
(a) y 2 = x 2 + 2 xy y = x 2 - 2 xy æ 1ö
dx dx y 2 dx + ç x - ÷ dy = 0. If y (1) = 1, then x is
è yø
dy dy
(c) x 2 = y 2 + xy (d) x 2 = y 2 + 3 xy . given by :
dx dx
1 1
11. The solution of the differential equation y y
2 e 1 e
dy x + y (a) 4 - - (b) 3 - +
= satisfying the condition y(1) =1 is y e y e
dy x 1 1
y y
[2008]
(c) 1 + 1 - e (d) 1 - 1 + e
(a) y = ln x + x (b) y = x ln x + x2 y e y e
(c) y = xe(x – 1) (d) y = x ln x + x 17. The population p (t) at time t of a certain mouse
12. The differential equation which represents the
dp ( t )
c2 x , species satisfies the differential equation =
family of curves y = c1e where c1, and c2 dt
are arbitrary constants, is [2009] 0.5 p(t) – 450. If p (0) = 850, then the time at which
(a) y" = y'y (b) yy" = y' the population becomes zero is : [2012]
(c) yy" = (y')2 (d) y' = y2 (a) 2ln 18 (b) ln 9
13. Solution of the differential equation 1
p (c) ln 18 (d) ln 18
cos x dy = y (sin x - y ) dx, 0 < x < is[2010] 2
2 18. At present, a firm is manufacturing 2000 items.
(a) y sec x = tan x + c (b) y tan x = sec x + c It is estimated that the rate of change of
(c) tan x = (sec x + c) y (d) sec x = (tan x + c) y production P w.r.t. additional number of workers
dy
14. If = y + 3 > 0 and y (0) = 2, then y (ln 2) is dP
dx x is given by = 100 – 12 x . If the firm
dx
equal to : [2011]
employs 25 more workers, then the new level
(a) 5 (b) 13
of production of items is [2013]
(c) – 2 (d) 7
(a) 2500 (b) 3000
15. Let I be the purchase value of an equipment and
V (t) be the value after it has been used for t (c) 3500 (d) 4500
years. The value V(t) depreciates at a rate given 19. Let the population of rabbits surviving at time
t be gover ned by the differen tial
www.crackjee.xyz
Differential Equations M-153
(a) 2 (b) 2e
dp ( t ) 1
equation = p ( t ) –200.If p(0) = 100, (c) e (d) 0
dt 2 21. If a curve y = f(x) passes through the point (1, –1)
then p(t) equals: [2014] and satisfies the differential equation, y(1 + xy)
(a) 600 - 500 e t 2 (b) 400 - 300 e -t 2 æ 1ö
dx = x dy, then f ç ÷ is equal to :
ç- ÷ [2016]
(c) 400 - 300 et 2 (d) 300 - 200 e -t 2 è 2ø
20. Let y(x) be the solution of the differential 2 4
(a) (b)
dy 5 5
equation (x log x) + y = 2x log x, (x ³ 1).
dx 2 4
(c) - (d) -
Then y (e) is equal to: [2015] 5 5

Answer Key
1 2 3 4 5 6 7 8 9 10 11 12 13 14 15
(c) (b) (c) (c) (c) (b) (c) (c) (d) (a) (d) (c) (d) (d) (a)
16 17 18 19 20 21
(c) (a) (c) (c) (a) (b)

2 3 1
æ d yö æ 4d3yö ò dy
tan -1 y
1. (c) 1 + 3 = ç ÷ (1+ y 2 )
çè d x ÷ø I .F = e =e
è d x3 ø
-1
tan -1 y e tan y -1
æ d yö
2
æ d yö 3 3 x( e )=ò e tan y
dy
2
Þ ç1 + 3 = 16 ç ÷ 1+ y
è ÷
d xø è d x3 ø -1
tan -1 y e 2 tan y
2
d y -2 x x( e )= +C
2. (b) 2
= e -2 x ; dy = e +c; 2
dx dx -2 -1 -1
\ 2 xe tan y
= e2 tan y
+k
-2 x
e
y= + cx + d 5. (c) x 2 + y 2 - 2ay = 0 ...........(1)
4
Differentiate,
3. (c) y 2 = 4a( x - h), 2 yy1 = 4a Þ yy1 = 2a
dy dy x + yy ¢
Differentiating, Þ y12 + yy2 = 0 2x + 2 y - 2a =0 Þa=
dx dx y¢
Degree = 1, order = 2.
æ x + yy ¢ ö
-1 dy Put in (1), x 2 + y 2 - 2 ç y=0
4. (c) (1 + y 2 ) + ( x - e tan y ) =0 è y ¢ ÷ø
dx
-1 Þ ( x 2 + y 2 ) y '- 2 xy - 2 y 2 y ' = 0
dx x e tan y
Þ + =
dy (1 + y 2 ) (1 + y 2 ) Þ ( x 2 - y 2 ) y ' = 2 xy
EBD_7764
www.crackjee.xyz
M-154 Mathematics
ln z = ln x + ln c
6. (b) ydx + ( x + x 2 y )dy = 0
æ yö
dx x dx x x = cx or log v = cx or log ç ÷ = cx.
Þ = - - x2 Þ + = - x2 , è xø
dy y dy y
It is Bernoulli form. Divide by x2 9. (d) Ax 2 + By 2 = 1 ...(1)
dx æ 1ö dy
x -2 + x -1 ç ÷ = -1. Ax + By =0 ....(2)
dy è yø dx
2
dx dt d2y æ dy ö
put x -1 = t , - x -2 = A + By + Bç ÷ = 0 ....(3)
dy dy dx 2 è dx ø
We get, From (2) and (3)
dt æ 1ö dt æ 1 ö ìï d2y æ dy ö üï
2
dy
- + t ç ÷ = -1 Þ - t =1 x í - By 2 - B ç ÷ ý + By =0
dy è y ø dy çè y ÷ø dx è ø
dx ïþ dx
ïî
It is linear in t. Dividing both sides by –B, we get
Integrating factor
2
1 d2yæ dy ö dy
ò - dy xy + xç ÷ - y =0
=e y
= e - log y = y -1 dx 2 è dx ø dx
Which is a DE of order 2 and degree 1.
\ Solution is t ( y -1 ) = ò ( y -1 )dy + C 10. (a) General equation of circles passing through
origin and having their centres on the x-axis
1 1 1 is
Þ . = log y + C Þ log y - =C
x y xy x2 + y2 + 2gx = 0...(i)
On differentiating w.r.t x, we get
7. (c) y 2 = 2c ( x + c ) ........ (i)
dy æ dy ö
2 yy ' = 2c.1 or yy ' = c ........ (ii) 2x + 2y .
dx
+ 2g = 0 Þ g = – çè x + y ÷ø
dx
Þ y 2 = 2 yy ' ( x + yy ') \ equation (i) be
[On putting value of c from (ii) in (i)] ì æ dy ö ü
On simplifying, we get x2 + y2 + 2 í - ç x + y ÷ ý . x = 0
î è dx ø þ
( y - 2 xy ')2 = 4 yy '3 ........ (iii)
dy
Hence equation (iii) is of order 1 and Þ x2 + y2 – 2x2 – 2x .y = 0
degree 3. dx
xdy dy
8. (c) = y (log y – log x + 1) Þ y2 = x2 + 2xy
dx
dx
dy x + y y
dy y æ æ yö ö 11. (d) = = 1+
= ç log ç ÷ + 1÷ dx x x
dx x è è xø ø
dy dv
Put y = vx Putting y = vx and =v+x
dx dx
dy xdv xdv
=v+ Þ v+ = v (log v + 1) we get
dx dx dx
dv dx
= 1+ v Þ ò
x ò
xdv dv dx v+x = dv
= v log v Þ = dx
dx v log v x
Þ v = ln x + c Þ y = xlnx + cx
Put log v = z As y(1) = 1
1 dz dx \ c = 1 So solution is y = x lnx + x
dv = dz Þ =
v z x
www.crackjee.xyz
Differential Equations M-155
c2 x k k
12. (c) We have y = c1e V (T ) = I + (T 2 - 2T 2 ) = I - T 2
2 2
Þ y ¢ = c1c2 e c2 x = c2 y
dx x 1
y ¢¢ 2 16. (c) + =
Þ = c2 Þ y ¢¢ y - ( y ¢ ) = 0 dy y 2 y 3
y y2 1
ò dy –
1
y2 y
Þ y ¢¢ y = ( y ¢ ) 2 I.F. = e =e
1 1
-
13. (d) cos x dy = y (sin x - y ) dx 1 –
So x.e y
=ò e y
dy
dy y3
= y tan x - y 2 sec x -1
dx Let =t
y
1 dy 1
- tan x = - sec x ...(i) 1
y 2 dx y Þ dy = dt
y2
1 1 dy dt
Let =tÞ- = Þ I = - ò tet dt = et - tet
y y 2 dx dx
1 1
From equation (i) -
y 1 -y
=e + e +c
dt y
- - t tan x = - sec x 1 1 1
dx - - 1 -
y y
Þ xe =e + e y +c
dt y
Þ + (tan x )t = sec x
dx 1
Þ x = 1+ + c.e1/ y
I.F. = e ò tan x dx = (e ) log|sec x|
sec x y
Since y (1) = 1
Solution : t(I.F) = ò (I.F) sec x dx
1
1 \ c=-
e
Þ y sec x = tan x + c
1 1
Þ x = 1 + - .e1/ y
dy y e
dy
14. (d)
dx
= y+3 Þ ò y + 3 = ò dx 17. (a) Given differential equation is
dp ( t )
= 0.5 p ( t ) - 450
Þ ln y +3 = x+ c dt
Since y (0) = 2, \ l n 5 = c dp ( t ) 1
Þ = p ( t ) - 450
Þ ln y + 3 = x + ln5 dt 2
When x = ln 2 , then ln |y + 3| = ln 2 + ln5 dp ( t ) p ( t ) - 900
Þ ln | y + 3 | = ln 10 Þ =
\ y + 3 = ±10 Þ y = 7, – 13 dt 2
dV (t ) dp ( t )
Þ 2 = - éë900 - p ( t ) ùû
15. (a) = - k (T - t ) dt
dt
dp ( t )
Þ ò dVt = - k ò (T - t )dt Þ 2
900 - p ( t )
= -dt
k (T - t )2
V (t ) = +c Integrate both the side, we get
2
dp ( t )
k
at t = 0, V (t) = I Þ V (t ) = I + (t 2 - 2tT )
-2 ò 900 - p ( t ) = ò dt
2
EBD_7764
w w w . c r a c k j e e . x y z
M-156 Mathematics
Let 900 – p (t) = u
d ( p (t ))
Þ
\
– dp (t) = du
We have,
ò1 = ò dt
p (t ) - 200
du 2
2 ò u ò
= dt Þ 2 ln u = t + c
Let
1
p (t ) - 200 = s Þ
dp(t )
= ds
Þ 2ln [900 – p(t)] = t + c 2 2
when t = 0, p (0) = 850 d p (t )
2 ln (50) = c So, òæ1 ö
= ò dt
é æ 900 - p ( t ) ö ù ç p (t ) - 200 ÷
Þ 2 êln ç ÷ú = t è2 ø
ë è 50 øû
2ds
t Þ ò s
= ò dt
Þ 900 - p ( t ) = 50 e 2 Þ 2 log s = t + c
t
æ p (t ) ö
Þ p (t ) = 900 - 50e 2 Þ 2log ç - 200÷ = t + c
è 2 ø
let p (t1) = 0
t1 1
\ t1 = 2ln 18 p(t )
0 = 900 - 50 e 2 Þ - 200 = e 2 k
2
dP Using given condition p(t) = 400 – 300 et/2
18. (c) Given, Rate of change is = 100 - 12 x
dx dy æ 1 ö
+ y=2
dx çè x log x ÷ø
20. (a) Given,
Þ dP = (100 – 12 x ) dx
By integrating 1
I.F. = ò x log x dx
e
ò dP = ò (100 - 12 x ) dx
P = 100x – 8x3/2 + C = elog(log x) = log x
Given when x = 0 then P = 2000
y. logx = ò 2 log xdx + c
Þ C = 2000
Now when x = 25 then y logx = 2[x log x – x] + c
P = 100 × 25 – 8 × (25)3/2 + 2000 Put x = 1, y.0 = –2 + c
= 4500 – 1000 c=2
Þ P = 3500 Put x = e
19. (c) Given differential equation is y loge = 2e(log e – 1) + c
y(e) = c = 2
dp (t ) 1
= p (t ) - 200
dt 2 21. (b) y (1 + xy)dx = xdy
By separating the variable, we get xdy - ydx
= xdx
é1 ù y2
æx ö
dp(t) = ê p(t ) - 200ú dt Þò- d çç ÷÷ =òxdx
ë2 û
èyø
x x2
dp(t ) - = + C as y(1) = –1 Þ C = 1
Þ = dt y 2 2
1 -2x æ -1 ö 4
p (t ) - 200 Hence, y = 2 Þfç
2 ç ÷ ÷=
x +1 è2ø 5
Integrate on both the sides,
www.crackjee.xyz
Vector Algebra M-157

Vector Algebra 25
r r r ® Ù Ù ® Ù Ù
1. If | a |= 4,| b |= 2 and the angle between a and 6. a = 3 i-5 j an d b = 6 i + 3 j are two
r r r
b is p /6 then (a ´ b )2 is equal to [2002] ®
vectors and c is a vector such that
(a) 48 (b) 16 ® ® ® ® ® ®
® c = a ´ b then | a |:| b |:| c |
(c) a (d) none of these
[2002]
® ® ® ®®®
2. If a , b , c are vectors such that [ a b c ] = 4 (a) 34 : 45 : 39 (b) 34 : 45 : 39
® ®® ® ® ®
then [ a ´ b b ´ c c ´ a ] = [2002] (c) 34 : 39 : 45 (d) 39 : 35 : 34
® ® ® ® ® ® ® ® ®
(a) 16 (b) 64
7. If a ´ b = b ´ c = c ´ a then a + b + c =
(c) 4 (d) 8 [2002]
® ® ® ® ® ® (a) abc (b) –1
3. If a , b , c are vectors such that a + b + c = 0 (c) 0 (d) 2
® ® ® 8. The sum of two forces is 18 N and resultant whose
and | a |= 7,| b |= 5,| c |= 3 then angle direction is at right angles to the smaller force is
12 N. The magnitude of the two forces are [2002]
® ® (a) 13, 5 (b) 12, 6
between vector b and c is [2002] (c) 14, 4 (d) 11, 7
(a) 60° (b) 30° 9. A bead of weight w can slide on smooth circular
(c) 45° (d) 90° wire in a vertical plane. The bead is attached
r r r
4. If | a |= 5, | b |= 4, | c |= 3 thus what will be the by a light thread to the highest point of the
wire and in equilibrium, the thread is taut and
rr rr rr
value of | a.b + b .c + c.a | , given that make an angle q with the vertical then tension
of the thread and reaction of the wire on the
® ® ®
a+ b+ c =0 [2002] bead are
(a) 25 (b) 50 (a) T = w cos q R = w tan q [2002]
(c) –25 (d) –50 (b) T = 2w cos q R=w
r r (c) T = w R = w sin q
5. If the vectors c , a = xiˆ + yjˆ + zkˆ and bˆ = ˆj
r r (d) T = w sin q R = wcot q
are such that a, c and form a right handed r r r
r 10. L e t u = iˆ + ˆj, v = iˆ - ˆj and w = iˆ + 2 ˆj + 3kˆ .
system then c is: [2002] r
r If n̂ is a unit vector such that u.nˆ = 0 and
ˆ
(a) zi - xkˆ (b) 0 r r
v .nˆ = 0 , then w. nˆ is equal to [2003]
(c) yj ˆ (d) - ziˆ + xkˆ
(a) 3 (b) 0
(c) 1 (d) 2.
EBD_7764
www.crackjee.xyz
M-158 Mathematics
11. A particle acted on by constant forces r r r
17. If u , v and w are three non- coplanar vectors,
4iˆ + ˆj - 3kˆ and 3iˆ + ˆj - kˆ is displaced from r r r r r r r
then (u + v - w).(u - v ) ´ (v - w) equals [2003]
rr r
the point ˆi + 2jˆ - 3kˆ to the point 5iˆ + 4jˆ + kˆ . (a) 3u .v ´ w (b) 0
The total work done by the forces is [2003] r r r rr r
(c) u .(v ´ w) (d) u.w ´ v .
(a) 50 units (b) 20 units r
(c) 30 units (d) 40 units. 18. A couple is of moment G and the force forming
r r
12. The vectors AB = 3iˆ + 4kˆ & AC = 5iˆ - 2 ˆj + 4kˆ the couple is P . If P is turned through a
are the sides of a triangle ABC. The length of right angle the moment of the couple thus
r r
the median through A is [2003] formed is H . If instead , the force P are turned
(a) (b) through an angle a , then the moment of couple
288 18
becomes [2003]
(c) 72 (d) 33 r r
(a) H sin a - G cos a
r r r r r r r r
13. a , b , c are 3 vectors, such that a + b + c = 0 , (b) G sin a - H cos a
r r r rr rr rr r r
a = 1, b = 2, c = 3, then a.b + b .c + c .a is (c) H sin a + G cos a
r r
equal to [2003] (d) G sin a + H cos a .
(a) 1 (b) 0 r r r r
19. The resultant of forces P and Q is R . If Q
(c) –7 (d) 7
r
14. A tetrahedron has vertices at O(0, 0, 0), A(1, 2, 1) is doubled then R is doubled. If the direction
B(2, 1, 3) and C(-1, 1, 2). Then the angle between r r
the faces OAB and ABC will be [2003] of Q is reversed,then R is again doubled.

-1 æ 19 ö Then P 2 : Q 2 : R 2 is [2003]
(a) 90 o (b) cos ç ÷
è 35 ø (a) 2 : 3 : 1 (b) 3 : 1 : 1
(c) 2 : 3 : 2 (d) 1 : 2 : 3.
(c) cos -1æç
17 ö
÷ (d) 30 o 20. A body travels a distance s in t seconds. It starts
è 31 ø
from rest and ends at rest. In the first part of the
a a2 1 + a3 journey, it moves with constant acceleration f
and in the second part with constant retardation
15. If b b2 1 + b 3 = 0 and vectors (1, a, a 2 ),
r. The value of t is given by [2003]
c c2 1 + c3
æ 1 1ö æ1 1ö
(a) 2sçç + ÷÷ (b) 2 sçç + ÷÷
(1, b, b2 ) and (1, c, c 2 ) are non- coplanar, then è f rø è f rø
the product abc equals [2003]
(a) 0 (b) 2 2s
(c) –1 (d) 1 (c) (d) 2s( f + r )
1 1
+
16. Consider points A, B, C and D with position vectors f r
7iˆ - 4 ˆj + 7 kˆ, iˆ - 6 ˆj + 10kˆ , - iˆ - 3 ˆj + 4 kˆ and 21. Two stones are projected from the top of a cliff h
metres high , with the same speed u, so as to hit the
5iˆ - ˆj + 5kˆ respectively. Then ABCD is a [2003] ground at the same spot. If one of the stones is
(a) parallelogram but not a rhombus projected horizontally and the other is projected
(b) square horizontally and the other is projected at an angle
(c) rhombus (d) rectangle. q to the horizontal then tan q equals [2003]
www.crackjee.xyz
Vector Algebra M-159
26. A particle is acted upon by constant forces
2 2u
(a) u gh (b) gh 4iˆ + ˆj - 3kˆ and 3iˆ + ˆj - kˆ which displace it
from a point iˆ + 2 ˆj + 3kˆ to the point
u u
(c) 2 g (d) 2 h g 5iˆ + 4 ˆj + kˆ . The work done in standard units
h
22. Two particles start simultaneously from the by the forces is given by [2004]
same point and move along two straight lines, (a) 15 (b) 30
r (c) 25 (d) 40
one with uniform velocity u and the other from r r r
r 27. If a, b , c are non-coplanar vectors and l is a real
rest with uniform acceleration f . Let a be the
angle between their directions of motion. The number, then the vectors a + 2b + 3c , lb + 4c
relative velocity of the second particle w.r.t. the and (2l - 1)c are non coplanar for [2004]
first is least after a time [2003]
(a) no value of l
u sin a
(a) u cos a
(b) all except one value of l
(b)
f f (c) all except two values of l
(d) all values of l
f cos a
(c) (d) u sin a 28. Let u , v , w be such that | u |= 1,| v |= 2,
u

3
| w |= 3. If the projection v along u is equal
23. The upper th portion of a vertical pole to that of w along u and v , w are
4
perpendicular to each other then
3
subtends an angle tan -1 at a point in the | u - v + w | equals [2004]
5
horizontal plane through its foot and at a (a) 14 (b) 7
distance 40 m from the foot. A possible height
of the vertical pole is [2003] (c) 14 (d) 2
(a) 80 m (b) 20 m
29. Let a , b and c be non-zero vectors such that
(c) 40 m (d) 60 m.
24. Let R1 and R2 respectively be the maximum 1
(a ´ b ) ´ c = | b || c | a . If q is the acute
ranges up and down an inclined plane and R 3
be the maximum range on the horizontal plane. angle between the vectors b and c , then
Then R1, R , R2 are in [ 2003] sinq equals [2004]
(a) H.P (b) A.G..P
2 2 2
(c) A.P (d) G..P. (a) (b)
r r r 3 3
25. Let a, b and c be three non-zero vectors such
2 1
that no two of these are collinear. If the vector (c) (d)
3 3
r r r r
a + 2b is collinear with c and b + 3cr is 30. With two forces acting at point, the maximum
collinear with ar (l being some non-zero scalar)
affect is obtained when their resultant is 4N. If
they act at right angles, then their resultant is
r
then ar + 2b + 6cr equals [2004] 3N. Then the forces are [2004]
r
(a) 0 (b) lb æ 1 ö æ 1 ö
r (a) çè 2 + 3 ÷ N and ç 2 - 3÷ N
(c) lc (d) lar 2 ø è 2 ø
EBD_7764
www.crackjee.xyz
M-160 Mathematics

(b) ( 2 + 3 ) N and ( 2 - 3 ) N 34. A velocity


1
4
m / s is resolved into two

æ 1 ö æ 1 ö components along OA and OB making angles


(c) çè 2 + 2 ÷ N and ç 2 - 2÷ N
2 ø è 2 ø 30° and 45° respectively with the given velocity.
Then the component along OB is [2004]
(d) ( 2 + 2 ) N and ( 2 - 2 ) N 1
( 6 - 2 )m / s
1
(a) (b) ( 3 - 1) m / s
31. In a right angle DABC , ÐA = 90° and sides a, 8 4
b, c are respectively, 5 cm, 4 cm and 3 cm. If a 1 1
r (c) m/s (d) m/s
force F has moments 0, 9 and 16 in N cm. units 4 8
respectively about vertices A, B and C, then 35. If t1 and t2 are the times of flight of two
r
magnitude of F is [2004] particles having the same initial velocity u
(a) 9 (b) 4
and range R on the horizontal , then t12 + t22 is
(c) 5 (d) 3
r r equal to [2004]
32. Three forces P, Q and R acting along IA, IB
(a) 1 (b) 4u 2 / g 2
and IC, where I is the incentre of a DABC are
r r r (c) u 2 / 2g (d) u 2 / g
in equilibrium. Then P : Q : R is [2004]
36. If C is the mid point of AB and P is any point
A B C outside AB, then [2005]
(a) cos ec : cos ec : cos ec
2 2 2 uuur uuur uuur
(a) PA + PB = 2 PC
A B C uuur uuur uuur
(b) sin : sin : sin (b) PA + PB = PC
2 2 2 uuur uuur uuur
(c) PA + PB + 2 PC = 0
A B C
(c) sec : sec : sec uuur uuur uuur
2 2 2 (d) PA + PB + PC = 0
A B C 37. For any vector a , the value of
(d) cos : cos : cos
2 2 2 ur ur ur
33. A paticle moves towards east from a point A to a (a ´ iˆ)2 + (a ´ ˆj ) 2 + (a ´ kˆ)2 is equal to
point B at the rate of 4 km/h and then towards ur 2 ur 2
north from B to C at the rate of 5km/hr. If AB = 12 (a) 3a (b) a [2005]
km and BC = 5 km, then its average speed for its ur 2 ur 2
journey from A to C and resultant average velocity (c) 2a (d) 4a
direct from A to C are respectively [2004] uur uur uur
38. If a , b , c are non coplanar vectors and l is
13 17
(a) km / h and km / h a real number then [2005]
9 9 uur uur 2 uur uur uur uur uur uur
[l (a + b ) l b l c ] = [a b + c b ]
13 17
(b) km / h and km / h for
4 4
(a) exactly one value of l
17 13
(c) km / h and km / h (b) no value of l
9 9
(c) exactly three values of l
17 13
(d) km / h and km / h (d) exactly two values of l
4 4
www.crackjee.xyz
Vector Algebra M-161
uur uur 44. ABC is a triangle, right angled at A. The
39. Let a = iˆ - kˆ , b = x iˆ + ˆj + (1 – x) k̂ and
uur uur uur uur resultant of the forces acting along AB, BC
c = y iˆ + x ĵ + (1 + x – y) k̂ . Then [a , b , c ]
depends on [2005] 1 1
with magnitudes and respectively
(a) only y (b) only x AB AC
(c) both x and y (d) neither x nor y is the force along AD , where D is the foot of
ur ur ur
40. ABC is a triangle. Forces P , Q , R acting the perpendicular from A onto BC. The
along IA, IB, and IC respectively are in magnitude of the resultant is [2006]
equilibrium, where I is the incentre of D ABC. AB 2 + AC 2
(a) (b) ( AB )( AC )
Then P : Q : R is [2005] 2 2 AB + AC
( AB) ( AC )
(a) sin A : sin B : sin C
A B C 1 1 1
(c) + (d)
(b) sin : sin : sin AB AC AD
2 2 2
B C
45. If ( a ´ b ) ´ c = a ´ (b ´ c ) where a, b and c are
A
(c) cos : cos : cos
2 2 2 any three vectors such that a .b ¹ 0 , b . c ¹ 0
(d) cos A : cos B : cos C then a and c are [2006]
41. A particle is projected from a point O with velocity
u at an angle of 60° with the horizontal. When it is p
(a) inclined at an angle of between them
moving in a direction at right angles to its direction 3
at O, its velocity then is given by [2005]
p
u u (b) inclined at an angle of between them
(a) (b) 6
3 2
(c) perpendicular
2u u (d) parallel
(c) (d)
3 3 46. The values of a, for which points A, B, C with
42. A and B are two like parallel forces. A couple of position vectors 2iˆ - ˆj + kˆ , iˆ - 3 ˆj - 5kˆ and
moment Hlies in the plane ofAand Band is contained aiˆ - 3 ˆj + kˆ respectively are the vertices of a right
with them. The resultant ofA and B after combining
is displaced through a distance [2005] p
angled triangle with C = are [2006]
2
2H H
(a) (b) (a) 2 and 1 (b) – 2 and – 1
A-B A+ B

H H (c) – 2 and 1 (d) 2 and – 1


(c) (d) 47. A particle has two velocities of equal magnitude
2( A + B ) A-B
inclined to each other at an angle q . If one of
43. The resultant R of two forces acting on a
them is halved, the angle between the other
particle is at right angles to one of them and its
and the original resultant velocity is bisected
magnitude is one third of the other force. The
ratio of larger force to smaller one is: [2005] by the new resultant. Then q is [2006]
(a) 90° (b) 120°
(a) 2 : 1 (b) 3 : 2 (c) 45° (d) 60°
(c) 3 : 2 (d) 3 : 2 2
EBD_7764
www.crackjee.xyz
M-162 Mathematics
48. A body falling from rest under gravity passes a r r r
54. The non-zero vectors are a , b and c are
certain point P. It was at a distance of 400 m r r r r
from P, 4s prior to passing through P. If related by a = 8b and c = –7b . Then the angle
between ar and cr is [2008]
g = 10m / s 2 , then the height above the point
p
P from where the body began to fall is [2006] (a) 0 (b)
(a) 720 m (b) 900 m 4
(c) 320 m (d) 680 m p
(c) (d) p
49. If û and v̂ are unit vectors and q is the acute 2
55. The projections of a vector on the three
angle between them, then 2 û ×3 v̂ is a unit coordinate axis are 6, –3, 2 respectively. The
vector for [2007] direction cosines of the vector are : [2009]
(a) no value of q
6 -3 2 6 -3 2
(b) exactly one value of q (a) , , (b) , ,
5 5 5 7 7 7
(c) exactly two values of q
-6 -3 2
(d) more than two values of q (c) , , (d) 6, – 3, 2
r 7 7 7
r r r r
50. Let a = iˆ + ˆj + kˆ, b = iˆ - ˆj + 2kˆ and 56. If u, v, w are non-coplanar vectors and p, q
r r are real numbers, then the equality
=c xiˆ + ( x - 2) ˆj - kˆ . If the vector c lies in r r r rr r r r r
r [3u pv pw] - [ pv w qu ] - [2w qv qu ] = 0
r
the plane of a and b , then x equals [2007] holds for : [2009]
(a) – 4 (b) – 2 (a) exactly two values of (p, q)
(c) 0 (d) 1. (b) more than two but not all values of (p, q)
51. The resultant of two forces Pn and 3n is a force of (c) all values of (p, q)
7n. If the direction of 3n force were reversed, the (d) exactly one value of (p, q)
r r
resultant would be 19 n. The value of P is [2007] 57. Let a = ˆj - kˆ and c = iˆ - ˆj - kˆ. Then the vector
(a) 3 n (b) 4 n
r r r r r r r
b satisfying a ´ b + c = 0 and a. b = 3 is[2010]
(c) 5 n (d) 6 n.
52. A particle just clears a wall of height b at a (a) 2iˆ - ˆj + 2kˆ (b) iˆ - ˆj - 2kˆ
distance a and strikes the ground at a distance (c) iˆ + ˆj - 2kˆ (d) -iˆ + ˆj - 2kˆ
c from the point of projection. The angle of
r r
projection is [2007] 58. If the vectors a = iˆ - ˆj + 2kˆ , b = 2iˆ + 4 ˆj + k%ˆ
bc bc r
(a) tan -1 (b) tan -1 and c = liˆ + ˆj + m kˆ are mutually orthogonal,
a(c - a) a
then (l, m) = [2010]
b (a) (2, –3) (b) (–2, 3)
(c) tan -1 (d) 45°.
ac (c) (3, –2) (d) (–3, 2)
r r 1
53. A body weighing 13 kg is suspended by two
strings 5m and 12m long, their other ends being
59. If a =
1
10
( ) (
3iˆ + kˆ and b = 2iˆ + 3 ˆj - 6kˆ ,
7
)
fastened to the extremities of a rod 13m long. If
r r é r r r r
the rod be so held that the body hangs ( )( ) (
then the value of 2a - b ë a ´ b ´ a + 2b ùû )
immediately below the middle point, then is [2011]
tensions in the strings are [2007] (a) –3 (b) 5
(a) 5 kg and 12 kg (b) 5 kg and 13 kg (c) 3 (d) –5
(c) 12 kg and 13 kg (d) 5 kg and 5 kg
www.crackjee.xyz
Vector Algebra M-163
r r uuur uuur
60. The vectors a and b are not perpendicular 65. If the vectors AB = 3iˆ + 4kˆ and AC =
r r
and c and d are two vectors satisfying 5iˆ – 2 ˆj + 4kˆ are the sides of a triangle ABC,
r r r r rr
b ´ c = b ´ d and a.d = 0 . Then the vector then the length of the median through A is
r [2013]
d is equal to [2011]
r (a) 18 (b) 72
rr
r æ a.c ö r r æ b .cr ö r
(a) c + çr ÷r b (b) b + ç r r ÷c (c) 33 (d) 45
è a.b ø è a.b ø r r r r r r rrr 2
r If éë a ´ b b ´ c c ´ a ùû = l éë a b c ùû then l is
rr r æ b.cr ö r
66.
r æ a.c ö r
(c) c - ç r r ÷ b (d) b - ç r r ÷c equal to [2014]
è a.b ø è a.b ø (a) 0 (b) 1
61. If the pi$ + $j + k$ , $i + q $j + k$ and (c) 2 (d) 3
® ® ®

iˆ + ˆj + rkˆ ( p ¹ q ¹ r ¹ 1) vector are coplanar,, 67. Let a , b and c be three non-zero vectors such
that no two of them are collinear and
then the value of pqr - ( p + q + r ) is [2011RS] ® ® ® 1® ®®
(a) 2 (b) 0 (a ´ b) ´ c = b c a . If q is the angle
3
(c) – 1 (d) – 2
r r r ® ®
62. Let a, b, c be three non-zero vectors which between vectors b and c , then a value of sin q
r r is : [2015]
are pairwise non-collinear. If a + 3b is collinear
r r r r 2 -2 3
with c and b + 2c is collinear with a , then (a) (b)
r r r 3 3
a + 3b + 6c is : [2011RS] 2 2 - 2
r r (c) (d)
(a) a (b) c 3 3
r r r ® ® ®
(c) 0 (d) a + c 68. Let a , b and c be three unit vectors such that
r r
63. Let a and b be two unit vectors. If the vectors ® æ® ® ö
r 3 æç® ® ö ®
r a ´ çç b´ c ÷÷ = b+ c ÷ . If b is not parallel
c = aˆ + 2 bˆ and d = 5 aˆ - 4bˆ are perpendicular è ø 2 è
ç ÷
ø
to each other, then the angle between â and ® ® ®
to c , then the angle between a and b is:
b̂ is : [2012] [2016]
p p
(a) (b) 2p 5p
6 2 (a) (b)
3 6
p p
(c) (d) 3p p
3 4 (c) (d)
4 2
64. Let ABCD be a parallelogram such that r r r
uuur r uuuur r 69. Let a = 2i + j - 2k and b = i + ˆj . Let c be a
ˆ ˆ ˆ ˆ
AB = q , AD = p and ÐBAD be an acute angle.
r r r r r
r
If r is the vector that coincide with the altitude ( )
vector such that | c - a | = 3, a ´ b ´ c = 3
r r
and the angle between rc and a ´ b be 30°.
directed from the vertex B to the side AD, then
r
r is given by : [2012] rr
rr r r Then a.c is equal to : [2017]
r r 3 ( p.q ) r ( p .q )
(a) r = 3q - r r p (b) rr = -qr + r r pr 1 25
( p.p ) ( p . p) (a) (b)
r r r r 8 8
r r (p.q) r r r 3 ( p.q ) r (c) 2 (d) 5
(c) r = q - r r p (d) r = -3q - r r p
(p.p) (p .p)
EBD_7764
www.crackjee.xyz
M-164 Mathematics
Answer Key
1 2 3 4 5 6 7 8 9 10 11 12 13 14 15
(b) (a) (a) (a) (a) (b) (c) (a) (b) (a) (d) (d) (c) (b) (c)
16 17 18 19 20 21 22 23 24 25 26 27 28 29 30
(none ) (c) (c) (c) (a) (a) (a) (c) (a) (c) (d) (c) (c) (a) (c)
31 32 33 34 35 36 37 38 39 40 41 42 43 44 45
(c) (d) (d) (a) (b) (a) (c) (b) (d) (c) (d) (b) (d) (d) (d)
46 47 48 49 50 51 52 53 54 55 56 57 58 59 60
(a) (b) (a) (b) (b) (d) (c) (a) (a) (b) (d) (d) (d) (d) (c)
61 62 63 64 65 66 67 68 69
(d) (c) (c) (b) (c) (b) (c) (b) (c)

® ® ® p Þ 2 × 5 × 3cos q = 15;
1. (b) We have, a . b = | a | | ®
b | cos 6 p
Þ cos q = 1/2; Þ q = = 60°
3 3
= 4 ´ 2´ =4 3.
2 4. (a) We have, ® ® ® ®
a+ b+ c = 0
® ® ® ®
Now, ( a ´ b )2 + ( a × b )2 = a 2b 2 ; ® ® ®
Þ ( a + b + c )2 = 0
® ®
Þ ( a ´ b ) 2 + 48 = 16 ´ 4 ® ® ®
Þ | a |2 + | b |2 | + | c |2
® ®
Þ ( a ´ b )2 = 16 ® ® ® ® ® ®
+ 2( a × b + b × c + c × a ) = 0
® ® ® ® ® ®
2. (a) We have, [ a ´ b b x c c ´ a] ® ® ® ® ® ®
Þ 25 + 16 + 9 + 2( a × b + b × c + c × a ) = 0
® ® ì ® ® ® ®ü ® ® ® ® ® ®
= ( a ´ b ). í( b x c ) ´ ( c ´ a )ý Þ ( a × b + b × c + c × a ) = -25 .
î þ
® ® ®® ®®
® ® ì ® ® ® ® ® ®ü \ | a × b + b × c + c × a |= 25 .
= ( a ´ b ). í( m × a ) c - ( m × c ) a )ý
î þ r r r
5. (a) Since a , c , b form a right handed system,
® ® ® iˆ ˆj kˆ
(where m = b ´ c ) r r r
\ c = b ´ a = 0 1 0 = ziˆ - xkˆ
® ® ® ® ® ®
= {( a ´ b ). c }.{( a × ( b ´ c )} x y z
®®®
= [ a b c ]2 = 42 = 16 . ® ® ® ®
® ® ® ® ® ® 6. (b) We have a ´ b = 39 k = c
3. (a) a+ b+ c = 0 Þ b+ c = - a
® ® ®
Also | a |= 34,| b |= 45, | c |= 39 ;
® ® ® ®®
Þ ( b + c ) 2 = ( a ) 2 = 52 + 32 + 2 b × c = 7 2 ® ® ®
\ | a |:| b |:| c |= 34 : 45 : 39 .
® ®
Þ 2 | b | | c | cos q = 49 - 34 = 15 ;
www.crackjee.xyz
Vector Algebra M-165
r r r r
7. (c) Let a + b + c = r . Then
i j k
r r r r r r
a ´ (a + b + c ) = a ´ r 1 1 0
r r r r r r 1 -1 0 -2 kˆ
Þ0+a´b +a´c =a ´r nˆ = = = - kˆ
r r r r r r r r r 2´ 2 2
Þ a ´b -c ´a = a ´r Þ a ´r =0 r
r r r r r r w.nˆ = (i + 2 j + 3k ).( - k ) = -3 = 3
Similarly b ´ r = 0 & c ´ r = 0
Above three conditions will be satisfied r r r
11. (d) F + F1 + F2 = 7i + 2 j - 4k
r r
for non-zero vectors if and only if r = 0 r r r
8. (a) Given P + Q = 18 .......(1) d = PV
. of B - PV . of A = 4i + 2 j - 2k
r r
P2 + Q2 + 2PQ cos a = 144 ......(2) W = F .d = 28 + 4 + 8 = 40 unit
Q sin a 12. (d) A
tan 90º =
P + Q cos a

R=12 3i + 4 k 5i – 2j + 4k
Q

a
B D C
P
Þ P + Q cos a = 0 ......(3) uuur (3 + 5)i + (0 - 2) j + (4 + 4) k
From (2) and (3), P.V of AD =
2
Q2 – P2 = 144 Þ (Q – P) (Q + P) = 144 uuur
= 4i - j + 4k or AD = 16 + 16 + 1 = 33
144
\ Q-P=
18
=8 r r r r r r r r r
13. (c) a + b + c = 0 Þ (a + b + c ).( a + b + c ) = 0
From (1), On solving, we get Q = 13, P = 5 r2 r2 r2 rr rr rr
9. (b) Ð TQW = 180 – q ; Ð RQW = 2q ; a + b + c + 2(a.b + b .c + c .a ) = 0
Ð RQT = 180 – q r r r r r r -1 - 4 - 9
P a.b + b .c + c .a = = -7
2
14. (b) Vector perpendicular to the face OAB
q T
O q iˆ ˆj kˆ
q uuur uuur
Q = OA ´ OB = 1 2 1 = 5iˆ - ˆj - 3kˆ
90–2q
2 1 3
R
Vector perpendicular to the face ABC
w
Applying Lami's theorem at Q. iˆ ˆj kˆ
uuur uuur
T R W = AB ´ AC = 1 -1 2 = iˆ - 5 ˆj - 3kˆ
= =
sin 2q sin(180 - q) sin(180 - q) -2 -1 1
Þ R = W and T = 2W cos q
r r r Angle between the faces = angle between
10. (a) sin ce n is perpendicular u and v , their normals
r r
r u´n 5+5+9 19 æ 19 ö
n= r r cos q = = or q = cos -1 ç ÷
| u || n | 35 35 35 è 35 ø
EBD_7764
www.crackjee.xyz
M-166 Mathematics
r r r r
18. (c) G=r ´ p; G = rp sin q
a a 2 1 + a3
r
15. (c) b b 2 1 + b3 = 0 H = rp cos q éQ sin(90o + q) = cos qù
ë û
c c2 1 + c3
G = rp sin q.......(1) H = rp cos q........(2)
x = rp sin( q + a)...... ( 3 )
a a2 1 a a2 a3
From (1), (2) & (3),
Þb b 2
1+ b b 2 3
b =0 r r
x = G cos a + H sin a .
2 2
c c 1 c c c3 19. (c)

R 2 = P 2 + Q 2 + 2 PQ cos q .......(1)
2 2 2
1 a a2 4 R = P + 4Q + 4 PQ cos q .......(2)
Þ (1 + abc) 1 b b 2 = 0 4 R 2 = P 2 + Q 2 - 2 PQ cos q ........(3)
2 2 2 2
1 c c On (1) + (3), 5R = 2 P + 2Q ........(4)

1 a a2 On (3) ´ 2 + (2), 12 R 2 = 3 P 2 + 6Q 2 .....(5)

As 1 b b 2 ¹ 0 ( given condition ) 2 P 2 + 2Q 2 - 5 R 2 = 0 .....(6)

1 c c2 3P 2 + 6Q 2 - 12R 2 = 0 .....(7)
2 2 2 .
\ abc = -1 P Q R
= =
-24 + 30 24 - 15 12 - 6
16. (none)
P 2 Q2 R 2
= = or P 2 : Q 2 : R 2 = 2 : 3 : 2
6 9 6
A = (7,-4,7), B = (1,-6,10), C = (-1, - 3, 4)
20. (a) Let the body travels from A to B with
and D = (5, –1, 5) constant acceleration t and from B to C
with constant retardation r.
AB = (7 - 1)2 + ( -4 + 6)2 + (7 - 10)2 x y
= 36 + 4 + 9 = 7 A t1 B t2 C
If AB = x, BC = y, time taken from A to B = t1
Similarly BC = 7, CD = 41, DA = 17 and time taken from B to C = t2, then s = x +
\ None of the options is satisfied y and t = t1 + t2
r r r r r r r r r r r For the motion from A to B
17. (c) (u + v - w).(u ´ v - u ´ w - v ´ v + v ´ w)
v 2 = u 2 + 2 fs Þ v 2 = 2 fx (Q u = 0 )
r r r r r r r r r
= (u + v - w).(u ´ v - u ´ w + v ´ w)
r r r v2
= u.(u ´ v ) Þ x= ....(1)
r r r r r r r r r r r r 2f
-u.(u ´ w) + u.(v ´ w) + v .(u ´ v ) - v .(u ´ w) and v = u + ft Þ v= ft1
r r r r r r r r r r r r
+v .(v ´ w) - w.(u ´ v ) + w.(u ´ w) - w.(v ´ w) v
r r r r r r r r r Þ t1 = ...(2)
= u.(v ´ w) - v .(u ´ w) - w.(u ´ v ) f
rrr rr r r rr r r r For the motion from B to C
= [uvw)] + [vwu )] - [ wuv ] = u.(v ´ w)
v 2 = u 2 + 2 fs
v2
Þ 0 =v2 – 2ry Þ y = ...(3)
2r
www.crackjee.xyz
Vector Algebra M-167
and v = u + ft Þ 0 = v – rt2 1
and h = -u sin q ´ t + gt 2 ....(3)
v 2
Þ t2 = From (1) and (2) we get
r
Adding equations (1) and (3), we get
2h
v2 é 1 1 ù u = u cos q ´ t
x+ y = ê + ú = s g
2 ë f rû
Adding equations (2) and (4), we get 1 2h
é 1 1ù Þ t=
t1 + t 2 = v ê + ú = t cos q g
ë f rû Substituting this value of t in eq (3) we get
2
é 1 1ù
v2 ê + ú
t 2
ë f rû = 1 + 1 u sin q 2h 1 é 2h ù
= h=- + gê ú
\ 2s v2 æ 1 1 ö f r cos q g 2 êë g cos 2 q ûú
2´ ç + ÷
2 è f rø 2h
h = -u tan q + h sec 2 q
æ 1 1ö g
Þ t = 2s ç + ÷
è f rø 2h
h = -u tan q + h tan 2 q + h
21. (a) For the stone projected horizontally, for g
horizontal motion, using distance
= speed × time Þ R = ut
2 2
and for vertical motion tan 2 q - u tan q = 0; \ tan q = u
hg hg
u
1
h = 0 ´ t + gt 2 22. (a) We can consider the two velocities as
2 r
r $ $
v1 = ui$ and v2 = ( ft cos a ) i + ( ft sin a ) j
2h
Þ t =h
g ft

2h
\ We get RR= u ....(1)
g
a
For the stone projected at an angle q, for
horizontal and vertical motions, we have u
\ Relative velocity of second with
u respect to first
q r r r
v = v2 - v1 = ( ft cos a - u ) $i + ft sin a $j
r2
v = ( ft cos a - u ) + ( ft sin a )
2 2
Þ
h 2 2 2
= f t + u - 2uft cos a
r
For v to be min we should have
2
dv
R = 0 Þ 2 f 2t - 2uf cos a = 0
dt
R = u cos q ´ t ....(2) u cos a
Þ t=
f
EBD_7764
w w w . c r a c k j
M-168 Mathematics
r r r r r r r
2 2 Þ a + 2b + 6c = tc + sa - b + 3c
d v
Also = 2 f 2 = + ve r r r r r r
dt 2 = tc + (b + 3c ) - b + 3c = (t + 6)c
r r r
2
and hence v is least at the time é using s a = b + 3c ù
\ v ë û
r
= lc , where l = t + 6
u cos a
r
f 26. (d) Resultant of forces F = 7iˆ + 2 ˆj - 4kˆ
r
­ Displacement d = 4iˆ + 2 ˆj - 2kˆ
23. (c) 3
h r r
4 \ Work done = F .d = 28 + 4 + 8 = 40
¯ r r r r r
­ 27. (c) Vectors a + 2b + 3c , lb + 4c , and
h r
b 4 (2l - 1)c are coplanar if
a q ¯
40 m 1 2 3
0 l 4 =0
æ3ö
q = a + b , b = tan -1 ç ÷ 0 0 2l - 1
è5ø
or b = q - a 1
Þ l(2l - 1) = 0 Þ l = 0 or
tan q - tan a 2
Þ tan b = \ Forces are noncoplanar for all l, except
1 + tan q.tan a
1
h h l = 0,
- 2
3 40 160 rr rr
or = r r v . u v .u
5 h h 28. (c) Projection of v along u = r =
1+ .
40 160 |u | 2
r r rr
h2 - 200h + 6400 =
0 r r w. u w.u
projection of w along u = r =
|u | 2
Þ h= 40 or 160 metre
rr rr
\ possible height =40 metre v .u w.u
Given = ....(1)
2 2
24. (a) Let b be the inclination of the plane to the
Also , vr.wr = 0 ....(2)
horizontal and u be the velocity of
r r r
projection of the projectile Now | u - v + w |2
u2 u2 r 2 r2 r 2 rr r r rr
R1 = and R2 = = | u | + | v | + | w | -2u.v - 2v .w + 2u.w
g (1 + sin b) g (1 - sin b)
= 1 + 4 + 9 + 0 [ From (1) and (2)] = 14
1 1 2g 1 1 2é u2 ù r r r
+ = 2 or + = êQ R = ú \| u - v + w |= 14
R1 R2 u R1 R2 R êë g úû
\ R1 , R, R2 arein H .P. r r r 1 r r r
r r r r r 29. (a) Given (a ´ b ) ´ c = | b || c | a
r 3
25. (c) Let a + 2b = tc and b + 3c = s a, where t
r r
and s are scalars. Adding, we get Clearly a and b are noncollinear
r r r r r
a + 3b + 3c = tc + sa
rr r rr r 1 r r r
Þ (a.c )b - (b .c )a = | b | | c | a
3
www.crackjee.xyz
Vector Algebra M-169
rr rr 1 r r A
\ a.c = 0 and - b.c = | b || c |
3
-1
Þ cos q = P
3
1 2 2 I
\ sin q = 1 - =
9 3 A R
r r 90+
[q is acute angle between b and c ] Q 2
30. (c) Let forces be P and Q. then P + Q = 4 ....(1) B C
P Q R
and P 2 + Q 2 = 32 ....(2) = =
A æ Bö æ Cö
Solving we get the forces sin(90° + ) sin ç 90° + ÷ sin ç 90° + ÷
2 è 2ø è 2ø
æ 2ö æ 2ö
ç 2 + 2 ÷ N and ç 2 - 2 ÷ N A
P : Q : R = cos
B C
è ø è ø Þ : cos : cos
2 2 2
31. (c) Since, the moment about A is zero, hence
r 33. (d) Time taken by the particle in complete
F passes through A. Taking A as origin. journey
r
Let the line of action of force F be 12 5
y = mx . (see figure) T= + = 4 hr.
4 5
3m r
Moment about B = | F |= 9 ....(1)
2
1+ m
Y

C(0,4) 5 km
F

(y = mx)
A 12km B

12 + 5 17
\ Average speed = =
A B(3,0) 4 4
4 r
Moment about C= | F |= 16....(2) 12 2 + 5 2 13
Average velocity = =
1 + m2 4 4
Dividing (1) by (2), we get [using vector addition]
3 v 1
m= Þ| F |= 5 N . 34. (a) If v = , component along OB
4 4
32. (d) IA, IB, IC are bisectors of the angles A, B 1 1
´
and C as I is incentre of DABC. v sin 30° 6- 2
= = 4 2 =
B C A sin(45° + 30°) 3 +1 8
Now ÐBIC = 180 - - = 90° + etc.
2 2
2 2 2
Applying Lami’s theorem at I 35. (b) For same horizontal range the angles of
p
projection must be a and -a
2
2u sin a
\ t1 = and
g
EBD_7764
www.crackjee.xyz
M-170 Mathematics
æp ö
2u sin ç - a ÷ l ( a1 + b1 ) l ( a2 + b2 ) l ( a3 + b3 )
è2 ø 2u cos a
t2 = = Þ l 2b1 l 2b2 lb3
g g
lc1 lc2 lc3
2
4u
\ t12 + t22 =
g2 a1 a2 a3
uuur uuur uuur uuur = b1 + c1 b 2 + c2 b3 + c3
36. (a) PA + AP = 0 and PC + CP = 0
b1 b2 b3
uuuur uuuur uuuur
Þ PA + AC + CP = 0
uuuur uuuur uuuur a1 + b1 a2b2 a3 + b3
and PB + BC + CP = 0 4
Þl b1 b2 b3
Adding, we get
uuuur uuuur uuuur uuuur uuur c1 c2 c3
PA + PB + AC + BC + 2CP = 0.
uuuur uuuur uuuur uuuur a1 a2 a3
Since AC = - BC & CP = - PC
uuuur uuuur uuuur = b1 + c1 b2 + c2 b3 + c3
Þ PA + PB - 2PC = 0. b1 b2 b3
P R1 - R2 R2 - R3

a1 a2 a3 a1 a2 a3
Þ l 4 b1 b2 b3 = c1 c2 c3
c1 c2 c3 b1 b2 b3

A C B Þ l 4 = -1
r r r r Hence l has no real values.
37. (c) Let a = xi + yj + zk r r
39. (d) a = iˆ - kˆ, b = xiˆ + ˆj + (1 - x)kˆ and
r r r r r r r
a ´ i = zj - yk Þ (a ´ i )2 = y 2 + z 2 c = yiˆ + xjˆ + (1 + x - y )kˆ
Similarly, 1 0 -1
r r r r r rr rr r
(a ´ j )2 = x 2 + z 2 and (a ´ k )2 = x 2 + y 2 [a b c ] = a.b ´ c = x 1 1- x
r r r r r r y x 1+ x - y
Þ (a ´ i )2 + (a ´ j )2 + (a ´ k )2
r = 1 é1 + x - y - x + x 2 ù - é - x 2 - y ù
= 2( x 2 + y 2 + z 2 ) = 2a 2 ë û ë û
38. (b) Let us consider = 1 - y + x 2 - x2 + y
r =1
a = a1$i + a2 $j + a3kˆ rrr
Hence éë a b c ùû is independent of x and y
r
b = b1$i + b2 $j + b3kˆ both.
r 40. (c) IA, IB, IC are bisectors of the angles A, B
c = c1i$ + c2 $j + c3kˆ and C as I is incentre of DABC.
then as per question B C A
r r r r Now ÐBIC = 180 - - = 90° + etc.
é l ar + b l 2 b lcr ù = é ar b + cr b ù
( ) 2 2
Applying Lami’s theorem at I
2
ë û ë û
www.crackjee.xyz
Vector Algebra M-171
and AC respectively are
A
æ 1 öˆ æ 1 ö
çè ÷ i and çè ÷ ĵ
AB ø AC ø
P \ Their resultant along AD
æ 1 ö æ 1 ö
=ç i+ j
è AB ÷ø çè AC ÷ø
I
A R
90+ \ Magnitude of resultant is
Q 2
B C
P
=
Q
=
R æ 1 ö æ 1 ö
2 2 AC 2 + AB 2
= ç +ç =
Sin(90° +
A
) æ Bö
sin ç 90° + ÷
æ Cö
sin ç 90° + ÷ è AB ÷ø è AC ÷ø AB 2 + AC 2
2 è 2ø è 2ø
A B C BC
Þ P : Q : R = cos : cos : cos =
2 2 2 AB. AC
41. (d) u cos 60° = v cos 30° C
(as horizontal component of velocity
remains the same)
1 3 1
Þ u× = v× or v = u ^j
2 2 3 1 D
Y AC
q
q
o 30
o A B
60
u
30°
o
vcos30 1 ^
v
i
60°
o
30 AB
o X
O ucos60
But from figure DABC ~ DDBA
42. (b) Let A and B be displaced by a distance x
BC AC BC 1
then Change in moment of (A + B) Þ = Þ =
=applied moments AB AD AB ´ AC AD
\ The required magnitude of resultant
H
Þ ( A + B) ´ x = H Þ x = 1
A+ B becomes .
43. (d) F ' = 3F cos q and F = 3F sin q AD

F 45. (d) ( a ´ b ) ´ c = a ´ ( b ´ c ) , a .b ¹ 0 ,

3F b .c ¹ 0
Þ (a . c ).b - ( b . c )a = (a . c ).b - (a . b ).c

F' Þ (a . b ).c = ( b . c ) a Þ a || c .
uuur
46. (a) CA = (2 - a )iˆ + 2 ˆj ;
Þ F ' = 2 2 F Þ F : F ' :: 3 : 2 2 . uuur
CB = (1 - a)iˆ - 6 kˆ
44. (d) If we consider unit vectors iˆ and ĵ in uuur uuur
CA.CB = 0Þ (2 - a)(1 - a) = 0
the direction AB and AC respectively,
then as per quesiton, forces along AB Þ a = 2, 1
EBD_7764
www.crackjee.xyz
M-172 Mathematics
47. (b) For two velocities u and u at an angle q 1
1 2
to each other the resultant is given by 48. (a) Using h = gt 2 and h + 400 = g (t + 4)
2 2
D C

u R h
E
q/2 q/4 R¢ Q(t)
u/2
q/2 q/4 400m
A B
u
P(t+4)
R2 = u2 + u2 + 2u2 cosq = 2u2
(1 + cos q)
Subtracting, we get 400 = 8g + 4gt
q Þ t = 8 sec
Þ R 2 = 4u 2 cos 2 q / 2 or R = 2u cos
2 1
Now in second case, the new resultant \ h = ´ 10 ´ 64 = 320m
2
AE (i.e., R¢) bisects ÐCAB , therefore \ Desired height = 320 + 400 = 720 m
using angle bisector theorem in DABC , 49. (b) Given | 2uˆ ´ 3vˆ | = 1 and q is acute angle
we get
between û and v̂ , | uˆ | = 1, | vˆ | = 1
AB BE u u/2
= Þ = ÞR=u Þ 6 | uˆ | | vˆ | | sin q | = 1
AC EC R u/2
q 1
Þ 2u cos = u Þ 6 | sin q | = 1 Þ sin q =
2 6
q 1 q Hence, there is exactly one value of q for
Þ cos = = cos 60° Þ = 60° which 2 û × 3 v̂ is a unit vector..
2 2 2
r r
or q = 120° 50. (b) Given a = iˆ + ˆj + kˆ , b = $i - $j + 2k$ and
r
c = xi$ + ( x - 2) $j - k$
u r r
q
sin q r
tan = 2 If c lies in the plane of a and b , then
4 u + u cos q rrr
[a b c ] = 0
2
q 1 q 1 q 1 1 1
Þ sin + sin cos q = sin q cos
4 2 4 2 4 i.e. 1 -1 2 =0
q 3q q q x ( x - 2) -1
\ 2 sin = sin = 3sin - 4sin 3
4 4 4 4 Þ 1[1 – 2(x – 2)] – 1[– 1 – 2x] + 1[x – 2 + x] = 0
q 1 q Þ 1 – 2x + 4 + 1 + 2x + 2x – 2 = 0
\ sin 2 = Þ = 30° or q = 120° Þ 2x = –4 Þ x = – 2
4 4 4
R2 R1 r 8r
51. (d) Clearly a = – c
7
r r
Þ a || c and are opposite in direction
r r
\ Angle between a and c is p.
u qq
44
q/ 2 rr r r
a.c 8b × (–7b )
u/2 u cos q = r r = r r = –1 Þ q = p
ac 8b 7b
www.crackjee.xyz
Vector Algebra M-173
52. (c) Given : Force P = Pn, Q = 3n, resultant R = é aù
= a tan a ê1 - ú
7n & P' = Pn, Q' = (–3)n, R' = 19 n ë Rû
æ u 2 sin 2 a ö
ç Q R = ÷
è g ø
Ö19 P
7
Þ é aù
b = a tan a ê1 - ú
a ë cû
æ c - aö
Þ b = a tan a . ç
è c ÷ø
–3 3

We know that R2 = P2 + Q2 + 2PQ cos a bc


Þ (7)2 = P2 + (3)2 + 2 × P × 3 cos a Þ tan a =
a(c - a)
Þ 49 = P2 + 9 + 6P cos a The angle of projection,
Þ 40 = P2 + 6P cos a .....(i) bc
( ) a = tan–1
2
and 2 2
19 = P + (–3) + 2P × –3 cos a a(c - a)
Þ 19 = P2 + 9 – 6P cos a
Þ 10 = P2 – 6P cos a .....(ii) 54. (a) A
Adding (i) and (ii) 50 = 2P2
q 13
Þ P2 = 25 Þ P = 5n. m
53. (a) Let B be the top of the wall whose M
coordinates will be (a, b). Range (R) = c 12m
B
B (a,b) q
u q 9 0–
T1
C 5m
b
a 13 kg
A C
a D
c Q 132 = 5 2 + 122 Þ AB2 = AC2 + BC2
B lies on the trajectory Þ ÐACB = 90°
1 x2 Q m is mid point of the hypotenuse AB,
y = x tan a – g 2
2 u cos 2 a therefore MA = MB = MC
1 a2 Þ ÐA = ÐACM = q
Þ b = a tan a – g 2 Applying Lami’s theorem at C, we get
2 u cos 2 a
é ga ù T1 T2 13kg
Þ b = a tan a ê1 - ú = =
2 2
ë 2u cos a tan a û sin(180 - q) sin(90 + q) sin 90°
é a ù Þ T1 = 13 sin q and T2 = 13 cos q
= a tan a ê1 - 2 ú
ê 2u cos 2 a. sin a ú Þ
5
T1 = 13 ´ and T2 = 13 ´
12
êë g cos a úû 13 13
é a ù Þ T1 = 5 kg and T2 = 12 kg
ê 1- 2 ú 55. (b) Let P (x1, y1, z1) and Q (x2, y2, z2) be the
= a tan a u .2sin a cos a
ê ú initial and final points of the vector whose
ëê g ûú projections on the three coordinate axes
é a ù are 6, – 3, 2 then
1-
= a tan a ê u 2 sin 2a ú x2 – x1, = 6 ; y2 – y1 = – 3 ; z2 – z1 = 2
ê ú uuur
êë g úû So that direction ratios of PQ are 6, – 3, 2
EBD_7764
w w w . c r a c k j e e . x y z
M-174 Mathematics
uuur r r
\ Direction cosines of PQ are 58. (d) Since, a, b and cr are mutually orthogonal
6 -3 r r r r r r
, , \ a . b = 0, b . c = 0 , c . a = 0
2 2 2
6 + ( - 3) + 2 6 + ( - 3)2 + 22
2
Þ 2l + 4 + m = 0 ...(i)
2 6 –3 2 l - 1 + 2m = 0 ...(ii)
2 2 2 = , ,
6 + ( - 3) + 2 7 7 7 On solving (i) and (ii), we get l = -3, m = 2
r r r r r r r r
r
56. (d) Q u , v , w are non coplanar vectors
r r r
(
59. (d) (2a - b). (a ´ b ) ´ (a + 2b ) )
\ [ u , v , w] ¹ 0 r r r r r r r r
Now,
(
= (2 a - b ). ( a ´ b ) ´ a + 2( a + b ) ´ b )
r r r r r r r r r r r r r r r r r r
[ 3u , pv , p w]–[ pv , p w, qu ] [ 2 w, qv , qu ] = 0 r r æ (a . a )b - (a . b )a + 2(a . b )b ö
= (2a - b ) ç r r r÷
r r r r r r è -2(b . b ) a ø
Þ 3 p2 [ u , v , w] – pq [ v , w, u ] – 2q2
r r r r r r r
[ w, v , u ] = 0 = (2 a - b )(b - 0 + 0 - 2 a )
r r r r r r r r r r r r r
Þ 3 p2 [ u , v , w] - pq [ u , v , w] + 2q 2 [ u , v , w] [Q a . b = 0 and b . b = 1]
r r r rr rr
Þ (3 p2 – pq + 2q2) [ u , v , w ] = 0 = -4 a.a - b .b = -5
rr r
Þ 3p2 – pq + 2q2 = 0 60. (c) a.b ¹ 0 , ar.d = 0
q 2 7 q2 r r r r
Þ 2p2 + p2 – pq + + =0 Now, b ´ c = b ´ d
4 4 r r r r r r
Þ a ´ (b ´ c ) = a ´ (b ´ d )
r r r r r r r r r r r r
2
æ qö 7 2
Þ 2p2 + çè p - ÷ø + q = 0 Þ ( a . c )b - ( a . b ) c = ( a . d ) b - ( a . b ) d
2 4 rr r rr r rr r
Þ (a.b )d = -(a.c )b + (a.b )c
Þ p = 0, q = 0, p = q / 2
This is possible only when p = 0, q = 0 r r æ ar.cr ö r
\ There is exactly one value of (p, q). d = c - ç r r ÷b
rr r r r rr è a.b ø
r
57. (d) cr = b ´ ar Þ b. c = b.(b ´ a ) Þ b . c = 0 61. (d) The given vectors are collinear if

( )(
Þ b1iˆ + b2 ˆj + b3 kˆ . iˆ - ˆj - kˆ = 0,) p 1 1
r 1 q 1 =0
where b = b1iˆ + b2 ˆj + b3 kˆ
1 1 r
b1 - b2 - b3 = 0 ...(i)
r r Þ p ( qr - 1) + 1(1 - r ) + 1(1 - q) = 0
and a. b = 3 Þ ( ˆj - kˆ).(b1iˆ + b2 ˆj + b3kˆ) = 3
Þ pqr - p + 1 - r + 1 – q = 0
Þ b2 - b3 = 3 Þ pqr - ( p + q + r ) = -2
From equation (i) r r r
62. (c) a + 3b = l c .............(i)
b1 = b2 + b3 = (3 + b3 ) + b3 = 3 + 2b3 r r r
r b + 2c = μa ............(ii)
b = (3 + 2b3 )iˆ + (3 + b3 ) ˆj + b3 kˆ
On solving equation (i) and (ii)
From the option given, it is clear that b3 equal to r r
either 2 or –2.
r
(1 + 3μ ) a - (λ + 6) c = 0
If b3 = 2 then b = 7iˆ + 5 ˆj + 2kˆ which is not r r
possible As a and c are non collinear,,
r
If b = -2, then b = -iˆ + ˆj - 2kˆ \ 1 + 3μ = 0 and l + 6 = 0
3 r r r r
From (i), a + 3b + 6c = 0
www.crackjee.xyz
Vector Algebra M-175
r r r r rrr r r rr
63. (c) Let c = aˆ + 2 bˆ and d = 5 aˆ - 4bˆ = (a ´ b).[[b c a ] c] [Q b ´ c. c = 0]
r r r rr r r r rrr 2
Since c and d are perpendicular to each other
r r = [a b c].(a ´ b. c ) = [a b c ]
\ c .d = 0 r r r r r r rrr
= [a ´ b b ´ c c ´ a ] = [a b c ]2
Þ ( aˆ + 2bˆ ) .( 5aˆ - 4bˆ ) = 0
r r
So l = 1
r r r 1 r rr
Þ 5 + 6 aˆ.bˆ - 8 = 0
1 p
( Q a .a = 1) 67. (c) ( )
a´b ´c = b c a
3
Þ aˆ.bˆ = Þ q = r r r 1 r rr
64. (b) Let ABCD
2
be a
3
parallelogram such that
Þ ( )
–c ´ a ´ b = b c a
3
uuur r uuuur r rr r rr r 1 r r r
AB = q , AD = p and ÐBADbe an acute angle. Þ ( ) ( )
– c.b a + c.a b = b c a
3
We have r r r rr r 1 r rr
B C Þ ( )
– b c cos qa + c.a b = b c a
3
r r r
Q a, b, c are non collinear, the above
q equation is possible only when
r rr
1
– cosq = and c.a = 0
3
A X D 1 2 2
p Þ cosq = – Þ sinq = ; qÎ II quad
3 3
uuur æ pr . qr öæ pr ö pr.qr r r r r 3 r r
AX = ç r ÷ç r ÷ = r 2 p 68. (b) a ´ (b ´ c) = (b + c)
ç p ÷ç p ÷ 2
è øè ø p rr r r r r r r 3r 3r
r uuur uuur uuur r p.q r Þ (a × c)b - (a × b)c = b+ c
Let r = BX = BA + AX = -q + r p 2 2
2
p
65. (c) We have, On comparing both sides
uuur uuur uuur uuur uuur uuur r r
AB + BC + CA = 0 Þ BC = AC - AB 3 3
uuur uuur uuur a ×b = - Þ cosq= -
uuuur AC - AB æ uuuur BC ö 2 2
r r
Now, BM = çQ BM = 2 ÷ [Q a and b are unit vectors]
2 è ø
r r
where q is the angle between a and b
5p
q=
6
69. (c) Given :
r r
a = 2iˆ + ˆj - 2k,
ˆ b = ˆi + ˆj
r
Þ | a |= 3
r r
Also, we have \ a ´ b = 2iˆ - 2jˆ + kˆ
uuur uuuur uuuur
AB + BM + MA = 0 r r
uuur uuur | a ´ b |= 22 + 22 + 12 = 3
uuur AC - AB uuuur r r r r r r
Þ AB + = AM We have (a ´ b) ´ c = | a ´ b || c | sin 30 n
2
uuur uuur r r r r 1 r 1
uuuur AB + AC Þ | (a ´ b) ´ c | = 3| c |. Þ 3 = 3 | c | .
Þ 1 AM = = 4iˆ - ˆj + 4kˆ r
2 2
2 \ |c| =2
uuuur r r
Þ AM = 33 Now | c - a | = 3
On squaring, we r rget rr
r r r r r r Þ c2 + a2 – 2 – c.a = 9 Þ 4 + 9 – 2 – a.c = 9
66. (b) L.H.S = (a ´ b).[(b ´ c) ´ (c ´ a)] rr rr rr
r r r r r r r rr r Þ a.c = 2 [Q c.a = a.c ]
= (a ´ b).[(b ´ c. a )c - (b ´ c.c )a ]
EBD_7764
www.crackjee.xyz
M-176 Mathematics

Three Dimensional Geometry


26
1. A plane which passes through the point (3, 2, 6. The radius of the circle in which the sphere
x-4 y -7 z -4 x 2 + y 2 + z 2 + 2 x - 2 y - 4 z - 19 = 0 is cut by the
0) and the line = = is[2002]
1 5 4 plane x + 2 y + 2 z + 7 = 0 is [2003]
(a) x – y + z =1 (b) x + y + z = 5 (a) 4 (b) 1
(c) 2 (d) 3
(c) x + 2y – z = 1 (d) 2x – y + z = 5 7. Two system of rectangular axes have the same
2. The d.r. of normal to the plane through (1, 0, 0), origin. If a plane cuts them at distances a, b, c
(0, 1, 0) which makes an angle p /4 with plane and a' , b' , c ' from the origin then [2003]
x + y = 3 are [2002]
1 1 1 1 1 1
(a) 1, 2 ,1 (b) 1, 1, (a) + + - - - =0
2 a 2
b 2
c 2
a' 2
b' 2
c' 2
(c) 1, 1, 2 (d) 2 , 1, 1 1 1 1 1 1 1
(b) + + + + + =0
3. The shortest distance from the plane a2 b2 c2 a'2 b' 2 c '2
12 x + 4 y + 3z = 327 to the sphere 1 1 1 1 1 1
(c) + - + + - =0
2 2 2 2 2
a b c a' b' c'2
x 2 + y 2 + z 2 + 4 x - 2 y - 6 z= 155 is [2003]
1 1 1 1 1 1
(a) 39 (b) 26 (d) - - + - - =0.
a2 b2 c2 a'2 b'2 c'2
4 8. Distance between two parallel planes 2x + y + 2z
(c) 11 (d) 13.
13 = 8 and 4x + 2y +4z + 5 = 0 is [2004]
4. The two lines x = ay + b , z = cy + d and x = a¢y 9 5
+ b¢, z = c¢y + d¢ will be perpendicular, if and (a) (b)
2 2
only if [2003]
7 3
(a) aa¢ + cc¢ + 1 = 0 (c) (d)
(b) aa¢ + bb¢ + cc¢ + 1 = 0 2 2
(c) aa¢ + bb¢ +cc¢ = 0 9. A line with direction cosines proportional to 2,
(d) (a + a¢) (b + b¢) +(c + c¢) = 0. 1, 2 meets each of the lines x = y + a = z and
x + a = 2 y = 2 z . The co-ordinates of each of
5. The lines x - 2 = y - 3 = z - 4 [2003] the points of intersection are given by [2004]
1 1 -k
(a) (2a,3a, 3a), (2a, a, a )
x -1 y - 4 z - 5
and = = are coplanar if (b) (3a, 2a,3a), (a, a, a)
k 1 1
(a) k = 3 or –2 (b) k = 0 or –1 (c) (3a, 2a, 3a), (a, a, 2a )
(c) k = 1 or –1 (d) k = 0 or –3.
(d) (3a,3a,3a), (a, a, a)
www.crackjee.xyz
Three Dimensional Geometry M-177
10. If the straight lines [2004] 15. If the plane 2ax – 3ay + 4az + 6 = 0 passes
x = 1 + s , y = -3 - l s , z = 1 + l s through the midpoint of the line joining the
centres of the spheres
t
and x = , y = 1 + t , z = 2 - t , with parameters x 2 + y 2 + z 2 + 6 x - 8 y - 2 z = 13 and
2
s and t respectively, are co-planar, then l equals. x 2 + y 2 + z 2 - 10 x + 4 y - 2 z = 8 then a equals
(a) 0 (b) –1 [2005]
1 (a) – 1 (b) 1
(c) - (d) –2 (c) – 2 (d) 2
2
11. The intersection of the spheres 16. The distance between the line
ur
x 2 + y 2 + z 2 + 7 x - 2 y - z = 13 and r = 2iˆ - 2 ˆj + 3kˆ + l(i - j + 4k ) and the plane
ur
r .(iˆ + 5 ˆj + kˆ) = 5 is [2005]
x 2 + y 2 + z 2 - 3x + 3 y + 4 z = 8 is the same as
the intersection of one of the sphere and the 10 10
(a) (b)
plane [2004] 9 3 3
(a) 2 x - y - z = 1 (b) x - 2 y - z = 1
3 10
(c) x - y - 2 z = 1 (d) x - y - z = 1 (c) (d)
10 3
12. A line makes the same angle q, with each of the
17. If non zero numbers a, b, c are in H.P., then the
x and z axis. If the angle b, which it makes with
x y 1
y-axis, is such that sin 2 b = 3 sin 2 q, then straight line + + = 0 always passes
a b c
cos2q equals [2004] through a fixed point. That point is [2005]
2 1 (a) (– 1, 2) (b) (– 1, – 2)
(a) (b)
5 5 æ 1ö
(c) (1, – 2) (d) ç1,- ÷
3 2 è 2ø
(c) (d)
5 3 18. Let a, b and c be distinct non- negative
x +1 y -1 numbers. If the vectors aiˆ + ajˆ + ckˆ , iˆ + kˆ and
13. If the angle q between the line =
1 2
ciˆ + cjˆ + bkˆ lie in a plane, then c is [2005]
z -2
= and the plane 2x – y + l z + 4 = 0 is such (a) the Geometric Mean of a and b
2
(b) the Arithmetic Mean of a and b
1 (c) equal to zero
that sin q = then the value of l is [2005]
3 (d) the Harmonic Mean of a and b
5 -3 19. The plane x + 2y – z = 4 cuts the sphere
(a) (b)
3 5 x 2 + y 2 + z 2 – x + z – 2 = 0 in a circle of radius
3 -4 [2005]
(c) (d)
4 3 (a) 3 (b) 1
14. The angle between the lines 2x = 3y = – z and
6x = – y = – 4z is [2005] (c) 2 (d) 2
(a) 0° (b) 90°
(c) 45° (d) 30°
EBD_7764
www.crackjee.xyz
M-178 Mathematics
20. The two lines x = ay + b , z = cy + d ; and which one of the following gives possible
values of a and b? [2008]
x = a ' y + b ' , z = c ' y + d ' are perpendicular (a) a = 2, b = 2 (b) a = 1, b = 2
to each other if [2006] (c) a = 2, b = 1 (d) a = 1, b = 1
(a) aa '+ cc ' = -1 (b) aa '+ cc ' = 1 26. The line passing through the points (5, 1, a)
and (3, b, 1) crosses the yz-plane at the point
(c) a c (d) a + c = 1
+ = -1 æ 17 –13ö
a' c' a' c'
çè 0, , ÷ . Then [2008]
21. The image of the point (–1, 3, 4) in the plane 2 2 ø
x - 2 y = 0 is [2006] (a) a = 2, b = 8 (b) a = 4, b = 6
(c) a = 6, b = 4 (d) a = 8, b = 2
æ 17 19 ö
(a) ç - , - , 4 ÷ (b) (15,11, 4) x –1 y – 2 z – 3
è 3 3 ø 27. If the straight lines = = and
k 2 3

(c) æç - , - ,1ö÷
17 19 x – 2 y – 3 z –1
(d) None of these = = intersect at a point, then
è 3 3 ø 3 k 2
the integer k is equal to [2008]
22. If a line makes an angle of p / 4 with the positive
(a) –5 (b) 5
directions of each of x- axis and y- axis, then
(c) 2 (d) –2
the angle that the line makes with the positive
direction of the z-axis is [2007] x – 2 y –1 z + 2
28. Let the line = = lie in the
p p 3 –5 2
(a) (b) plane x + 3y – az + b = 0. Then (a, b) equals
4 2
[2009]
p p (a) (–6, 7) (b) (5, –15)
(c) (d)
6 3 (c) (–5, 5) (d) (6, –17)
23. If (2, 3, 5) is one end of a diameter of the sphere 29. Statement -1 : The point A(3, 1, 6) is the mirror
x2 + y2 + z2 – 6x – 12y – 2z + 20 = 0, then the image of the point B(1, 3, 4) in the plane x – y +
cooordinates of the other end of the diameter z = 5.
are [2007] Statement -2: The plane x – y + z = 5 bisects
(a) (4, 3, 5) (b) (4, 3, – 3) the line segment joining A(3, 1, 6) and B(1, 3, 4).
(c) (4, 9, – 3) (d) (4, –3, 3). [2010]
24. Let L be the line of intersection of the planes 2x + (a) Statement -1 is true, Statement -2 is true ;
3y + z = 1 and x + 3y + 2z = 2. If L makes an angle Statement -2 is not a correct explanation for
a with the positive x-axis, then cos a equals Statement -1.
[2007] (b) Statement -1 is true, Statement -2 is false.
(c) Statement -1 is false, Statement -2 is true .
1
(a) 1 (b) (d) Statement - 1 is true, Statement 2 is true ;
2 Statement -2 is a correct explanation for
1 1 Statement -1.
(c) (d) .
3 2 30. A line AB in three-dimensional space makes angles
r 45° and 120° with the positive x-axis and the positive
25. The vector a = a iˆ + 2 ˆj + bkˆ lies in the plane y-axis respectively. If AB makes an acute angle q
r r with the positive z-axis, then q equals [2010]
of the vectors b = iˆ + ˆj and c = ˆj + kˆ and
(a) 45° (b) 60°
r
bisects the angle between b and cr . Then (c) 75° (d) 30°
www.crackjee.xyz
Three Dimensional Geometry M-179

x y (a) 10 3 (b) 5 3
31. The line L given by + = 1 passes through
5 b (c) 3 10 (d) 3 5
the point (13, 32). The line K is parallel to L and 35. The length of the perpendicular drawn from

has the equation


x y
+ = 1 . Then the distance the point (3, -1, 11) to the line
c 3
between L and K is [2010] x y - 2 z -3
= = is : [2011RS]
2 3 4
17
(a) 17 (b) (a) 29 (b) 33
15
(c) 53 (d) 66
23 23
(c) (d) 36. A equation of a plane parallel to the plane
17 15
x – 2y + 2z –5 = 0 and at a unit distance from the
y -1 z - 3 origin is : [2012]
32. If the angle between the line x = = (a) x – 2y + 2z – 3 = 0 (b) x – 2y + 2z + 1 = 0
2 l
(c) x – 2y + 2z – 1 = 0 (d) x – 2y + 2z + 5 = 0
æ 5 ö x -1 y +1 z-1
and the plane x + 2y + 3z = 4 is cos–1 ç ÷, 37. If the line = = and
è 14 ø 2 3 4
then l equals [2011] x-3 y-k z
= = intersect, then k is equal to:
1 2 1
3 2 [2012]
(a) (b)
2 5
2
(a) –1 (b)
5 2 9
(c) (d)
3 3 9
33. Statement-1: The point A(1, 0, 7)) is the mirror (c) (d) 0
2
image of the point B(1, 6, 3) in the line : 38. Distance between two parallel planes 2x + y +
x y -1 z - 2 2z = 8 and 4x + 2y + 4z + 5 = 0 is [2013]
= =
1 2 3 3 5
(a) (b)
x y -1 z - 2 2 2
Statement-2: The line = =
1 2 3 7 9
bisects the line segment joining A(1, 0, 7) and (c) (d)
2 2
B(1, 6, 3) . [2011]
(a) Statement-1 is true, Statement-2 is true; x - 2 y -3 z - 4
39. If the lines = = and
Statement-2 is not a correct explanation for 1 1 -k
Statement-1.
x -1 y -4 z -5
(b) Statement-1 is true, Statement-2 is false. = = are coplanar, then k
k 2 1
(c) Statement-1 is false, Statement-2 is true.
(d) Statement-1 is true, Statement-2 is true; can have [2013]
Statement-2 is a correct explanation for (a) any value
Statement-1. (b) exactly one value
34. The distance of the point (1, – 5, 9) from the (c) exactly two values
plane x – y + z = 5 measured along a straight x (d) exactly three values
= y = z is [2011RS]
EBD_7764
www.crackjee.xyz
M-180 Mathematics
44. The distance of the point (1, –5, 9) from the
x -1 y - 3 z - 4
40. The image of the line = = in plane x – y + z = 5 measured along the line x = y
3 1 -5 = z is : [2016]
the plane 2 x - y + z + 3 = 0 is the line: [2014] 10 20
(a) (b)
x-3 y +5 z - 2 3 3
(a) = =
3 1 -5 (c) 3 10 (d) 10 3
x-3 y +5 z - 2 x -3 y+2 z+4
(b) = =
-3 -1 5 45. If the line, = = lies in the
2 -1 3
2 2
x+3 y -5 z - 2 plane, lx + my – z = 9, then l + m is equal to :
(c) = = [2016]
3 1 -5
(a) 5 (b) 2
x+3 y -5 z + 2 (c) 26 (d) 18
(d) = =
-3 -1 5 46. If the image of the point P(1, –2, 3) in the plane,
41. The angle between the lines whose direction 2x + 3y – 4z + 22 = 0 measured parallel to
cosines satisfy the equations l + m + n = 0 and x y z
line, = = is Q, then PQ is equal to :
1 4 5
l 2 + m 2 + n 2 is [2014]
(a) 6 5 (b) 3 5
p p
(a) (b) (c) 2 42 (d) 42
6 2 47. The distance of the point (1, 3, –7) from the
p p plane passing through the point (1, –1, –1),
(c) (d) having normal perpendicular to both the
3 4
42. The equation of the plane containing the line x -1 y + 2 z - 4
2x – 5y + z = 3; x + y + 4z = 5, and parallel to the lines = = and
1 -2 3
plane, x + 3y + 6z = 1, is : [2015]
(a) x + 3y + 6z = 7 (b) 2x + 6y + 12z = – x - 2 y +1 z + 7
= = , is : [2017]
13 2 -1 -1
(c) 2x + 6y + 12z = 13 (d) x + 3y + 6z = –7
10 20
43. The distance of the point (1, 0, 2) from the point (a) (b)
74 74
x - 2 y +1 z - 2
of intersection of the line = = 10 5
3 4 12
(c) (d)
and the plane x – y + z = 16, is [2015] 83 83
(a) 3 21 (b) 13
(c) 2 14 (d) 8

Answer Key
1 2 3 4 5 6 7 8 9 10 11 12 13 14 15
(a) (b) (d ) (a) (d) (d) (a) (c) (b) (d) (a) (c) (a) (b) (c)
16 17 18 19 20 21 22 23 24 25 26 27 28 29 30
(b) (c) (a) (b) (a) (d) (b) (c) (c) (d) (c) (a) (a) (a) (b)
31 32 33 34 35 36 37 38 39 40 41 42 43 44 45
(c) (d) (a) (a) (c) (a) (c) (c) (c) (c) (c) (a) (b) (d) (b)
46 47
(c) (c)
www.crackjee.xyz
Three Dimensional Geometry M-181

1. (a) As the point (3, 2, 0) lies on the given line


6. (d)
x-4 y -7 z -4
= =
1 5 4 O
\ There can be infinite many planes
passing through this line. But here out of A C
the four options only first option is
satisfied by the coordinates of both the
points (3, 2, 0) and (4, 7, 4)
\ x – y + z = 1 is the required plane. centre of sphere = (-1, 1, 2)
2. (b) Equation of plane through (1, 0, 0) is Radius of sphere 1 + 1 + 4 + 19 = 5
a (x – 1) + by + cz = 0 ...(i) Perpendicular distance from centre to the
(i) passes through (0, 1, 0). plane
–a + b = 0 Þ b = a; Also, -1 + 2 + 4 + 7 12
OC = d = = = 4.
a+a 1+ 4 + 4 3
cos 45° =
2 2
Þ 2a = 2a2 + c2
2(2a + c ) AC 2 = AO 2 - OC 2 = 52 - 4 2 = 9
Þ AC = 3
Þ 2a2 = c2 Þ c= 2a . x y z
7. (a) Eq. of planes be + + =1 &
So d.r of normal are a, a 2a i.e. 1, 1, 2 . a b c
3. (d) Shortest distance = perpendicular distance x y z
between the plane and sphere = distance + + =1
a' b' c '
of plane from centre of sphere – radius
( ^ r distance on plane from origin is
-2 ´ 12 + 4 ´ 1 + 3 ´ 3 - 327 same.)
= - 4 + 1 + 9 + 155
144 + 9 + 16 -1 -1
=
1 1 1 1 1 1
= 26 – 13 = 13 + + + +
a 2 b 2 c2 2
a' 2
b' c '2
4. (a) x - b = y = z - d ; x - b ' = y = z - d ' . 1 1 1 1 1 1
a 1 c a' 1 c'
2
+ 2
+ 2
- 2
- = 0 2
-
For perpendicularity of lines a b c a' b' c' 2
aa'+1 + cc' = 0 8. (c) The planes are 2 x + y + 2 z - 8 = 0 . ...(1)
x2 - x1 y2 - y1 z2 - z1 and 4 x + 2 y + 4 z + 5 = 0
5. (d) l1 m1 n1 =0 5
or 2 x + y + 2 z +=0 ...(2)
l2 m2 n2 2
\ Distance between (1) and (2)
1 - 1 -1 0 0 -1 5
+8
1 1 -k = 0 Þ 2 1+ k -k = 0 2 21 7
= 2 2 2
= =
k 2 1 k+2 1 1 2 +1 + 2 2 9 2

k 2 + 3k = 0 Þ k (k + 3) = 0 or k = 0 or - 3 9. (b) Let a point on the line x = y + a = z is


(l, l - a, l ) and a point on the line
æ m mö
x + a = 2 y = 2 z is ç m - a, , ÷ , then
è 2 2ø
EBD_7764
w w w . c r a c k j e e . x y z
M-182 Mathematics
direction ratio of the line joining these 2 2
Þ 2 cos q = 3 - 3 cos q
m m
points are l - m + a, l - a - , l - 3
2 2 \ cos 2 q =
If it respresents the required line, then 5
13. (a) If q is the angle between line and plane
m m
l-a- l- æp ö
l-m+a
= 2 = 2 then çè - q÷ø is the angle between line
2 1 2 2
and normal to plane given by
on solving we get l = 3a, m = 2a
\ The required points of intersection are æp
cos ç - q÷ =
ö ( )(
$i + 2 $j + 2kˆ . 2$i - $j + l kˆ
)
è2 ø 3 4 +1+ l
æ 2a 2a ö
(3a, 3a-a,3a) and çè 2a - a, , ÷ø
2 2 æp ö 2-2+2 l
cos ç - q ÷ =
or (3a, 2a,3a) and (a,a,a) è 2 ø 3´ 5 + l
10. (d) The given lines are 2 l 1
Þ sin q = = Þ 4l = 5 + l
y + 3 z -1 3 5 +l 3
x -1 = = =s ..........(1)
-l l 5
Þl= .
z -2 3
and 2 x = y - 1 = =t
-1 14. (b) The given lines are 2 x = 3 y = - z
.........(2)
The lines are coplanar, if x y z
or = = [Dividing by 6]
3 2 -6
0 - ( -1) -1 - 3 -2 - ( -1) and 6 x = - y = -4 z
1 -l l =0 x y z
or = = [Dividing by 12]
1 2 -12 -3
1 -1
2 \ Angle between two lines is
3.2 + 2.( -12) + ( -6) . ( -3)
1 -5 -1 cos q =
32 + 22 + ( -6) 2 2 + ( -12 ) + ( -3)
2 2 2
c2 ® c2 + c3 ; 1 0 l =0
1 6 - 24 + 18
0 -1 = = 0 Þ q = 90o
2 49 157
l 15. (c) Plane 2ax - 3ay + 4az + 6 = 0 passes
Þ 5( -1 - ) = 0 Þ l = -2
2 through the mid point of the centre of
11. (a) The equations of spheres are spheres
S1 : x 2 + y 2 + z 2 + 7 x - 2 y - z - 13 = 0 and x 2 + y 2 + z 2 + 6 x - 8 y - 2 z = 13 and
S2 : x 2 + y 2 + z 2 - 3x + 3 y + 4 z - 8 = 0 x 2 + y 2 + z 2 - 10 x + 4 y - 2 z = 8
Their plane of intersection is respectively centre of spheres are (– 3, 4,
S1 - S2 = 0 Þ 10 x - 5 y - 5 z - 5 = 0 1) and (5, – 2, 1). Mid point of centres is
(1, 1, 1).
Þ 2x - y - z =1 Satisfying this in the equation of plane,
12. (c) The direction cosines of the line are we get
cos q, cos b, cos q 2a - 3a + 4a + 6 = 0
\ cos 2 q + cos 2 b + cos 2 q = 1 Þ a = -2.

Þ 2 cos 2 q = sin 2 b = 3 sin 2 q (given)


www.crackjee.xyz
Three Dimensional Geometry M-183
16. (b) The given line is
r r r r r r r 20. (a) Equation of lines x - b = y = z - d
r = 2i - 2 j + 3k + l (i - j + 4k ) a 1 c
r r r r
and the plane is r × (i + 5 j + k ) = 5 x - b' y z - d '
= =
or x + 5y + z = 5 a' 1 c'
Required distance Line are perpendicular
2 - 10 - 2 + 3 - 5 10 Þ aa '+ 1 + cc ' = 0
= = 21. (d) If ( a, b, g ) be the image, then mid point of
1 + 25 + 1 3 3
(a, b, g) and (–1, 3, 4) must lie on x – 2y = 0
1 1 1
17. (c) a, b, c are in H.P. Þ , , are in A.P.. a -1 æ b + 3 ö
a b c \ - 2ç ÷=0
2 è 2 ø
2 1 1 1 2 1
Þ = + Þ - + =0 \ a - 1 - 2b - 6 = 0 Þ a - 2b = 7 … (1)
b a c a b c
x y 1 Also line joining (a , b, g ) and ( –1, 3, 4)
\ + + = 0 passes through (1, –2) should be parallel to the normal of the
a a c
r r r r r plane x –2y = 0
18. (a) Vector ai + aj + ck , i + k and a +1 b - 3 g - 4
r r r \ = = =l
ci + cj + bk are coplanar 1 -2 0
Þ a = l - 1, b = -2l + 3, g = 4 … (2)
a a c From (1) and (2)
1 0 1 = 0 Þ c 2 = ab Þ c = ab 9 13
a= , b=- , g =4
c c b 5 5
None of the option matches.
\ c is G.M. of a and b. 22. (b) Let the angle of line makes with the positive
19. (b) direction of z-axis is a direction cosines of
line with the +ve directions of x-axis, y-axis,
and z-axis is l, m, n respectively.
3 5
2 2 p p
\ l = cos , m = cos , n = cos a
1 , 0, – 1 4 4
as we know that, l2 + m2 + n2 = 1
2 2
p p
\ cos2 + cos2 + cos2 a = 1
4 4
Perpendicular distance of centre 1 1
Þ + + cos2 a = 1
æ1 1ö 2 2
ç ,0,- ÷ p
è 2 2ø Þ cos2 a = 0 Þ a =
from x + 2y – 2 = 4 is given by 2
Hence, angle with positive direction of the
1 1 3 p
+ -4 = z-axis is.
2 2 2 2
6 23. (c) We know that equation of sphere is
x2 + y2 + z2 + 2ux + 2vy + 2wz + d = 0
1 1 5 where centre is (–u, –v, –w)
radius of sphere = + +2 =
4 4 2 given x2 + y2 + z2 – 6x – 12y – 2z + 20 = 0
\ centre º (3, 6, 1)
5 3 Coordinates of one end of diameter of the
\ radius of circle = - =1.
sphere are (2, 3, 5). Let the coordinates of
2 2
EBD_7764
www.crackjee.xyz
M-184 Mathematics
the other end of diameter are (a, b, g ) It crosses yz plane where –2l + 5 = 0
5
\ a + 2 = 3, b+3 = 6 , g +5 =1 l=
2 2 2 2
Þ a = 4, b = 9 and g = –3 æ 5 5 ö æ 17 –13ö
\ çè 0,(b –1) + 1, (1– a) + a÷ø = çè 0, , ÷ø
\ Coordinate of other end of diameter are 2 2 2 2
(4, 9, –3) 5 17 5 13
24. (c) Let the direction cosines of line L be l, m, n, Þ (b – 1) + 1 = and (1 – a ) + a = -
2 2 2 2
then
2l + 3m + n = 0 ....(i) Þ b = 4 and a = 6
and l + 3m + 2n = 0 ....(ii) 27. (a) The two lines intersect if shortest
on solving equation (i) and (ii), we get distance between them is zero i.e.
r r r r
l m n l m n (a2 – a1 ) × b1 ´ b2
= = Þ = = r r =0
6 - 3 1- 4 6 - 3 3 -3 3 b1 ´ b2
r r r r
l m n l 2 + m 2 + n2 Þ (a2 – a1) × b1 ´ b2 = 0
Now = = =
3 -3 3 32 + ( -3) 2 + 32 r
where a1 = iˆ + 2 ˆj + 3kˆ ,
Q l2 + m2 + n2 = 1 r
b1 = kiˆ + 2 ˆj + 3kˆ
l m n 1
\ = = = r
3 -3 3 27 a2 = 2iˆ + 3 ˆj + kˆ , bˆ2 = 3iˆ + kjˆ + 2kˆ

Þ l=
3 1 1 1 1 1 –2
= ,m = - ,n =
27 3 3 3 Þ k 2 3 =0
Line L, makes an angle a with +ve x-axis
3 k 2
1
\ l = cos a Þ cos a = Þ 1(4 –3k) –1(2k – 9) – 2(k2– 6) = 0
3
r r r 5
25. (d) Q a lies in the plane of b and c Þ –2k2 – 5k + 25 = 0 Þ k = –5 or
r r r 2
\ a = b + lc Q k is an integer, therefore k = –5
Þ aiˆ + 2 ˆj + bkˆ = iˆ + ˆj + l ( ˆj + kˆ) x - 2 y -1 z + 2
28. (a) Q The line = = lie in the
Þ a = 1, 2 = 1+ l, b = l Þ a = 1, b = 1 3 -5 2
plane
r r r x + 3y – a z + b = 0
Q a bisects the angle between b and c . \ Pt (2, 1, – 2) lies on the plane
r i.e. 2 + 3 + 2a + b = 0
\ a = l(bˆ + cˆ)
or 2a + b + 5 = 0 ....(i)
l (iˆ + 2ˆj + k)
ˆ Also normal to plane will be perpendicular
Þ aˆi + 2ˆj + bkˆ = to line,
2
\ 3 × 1 – 5 × 3 + 2 × (– a ) = 0
l l Þ a=–6
Þa= , l = 2 , b= From equation (i) then, b = 7
2 2
\ (a, b) = (– 6, 7)
Þ a=b=1
29. (a) A(3, 1, 6); B = (1, 3, 4)
26. (c) Equation of line through (5, 1, a) and
Mid-point of AB = (2, 2, 5) lies on the plane.
x – 5 y –1 z – a and d.r’s of AB = (2, –2, 2)
(3, b, 1) is = = =l
–2 b –1 1– a d.r’s of normal to plane = (1, –1, 1).
\ Any point on this line is a Direction ratio of AB and normal to the
[–2l + 5, (b – 1) l + 1, (1– a) l + a] plane are proportional therefore,
AB is perpendicular to the plane
www.crackjee.xyz
Three Dimensional Geometry M-185
\ A is image of B 5
Statement-2 is correct but it is not correct But it is given that q = cos -1
explanation. 14
30. (b) Direction cosines of the line : (5 + 3l ) 2 5
1 , -1 , \ 1- =
l = cos 45° = m = cos120° = 14(5 + l 2 14
2 2
2
n = cos q Þ l=
where q is the angle, which line makes with 3
33. (a) The direction ratio of the line segment
positive z-axis.
joining points A(1, 0, 7) and B(1, 6, 3) is 0, 6,
Now l 2 + m2 + n2 = 1 –4.
1 1 2 The direction ratio of the given line is 1, 2, 3.
Þ + + cos q = 1
2 4 Clearly 1 × 0 + 2 × 6 + 3 × (–4) = 0
1 So, the given line is perpendicular to line AB.
cos 2 q =
4 Also , the mid point of A and B is (1, 3, 5)
1 which lies on the given line.
Þ cos q = ( q being acute)
2 So, the image of B in the given line is A, because
p the given line is the perpendicular bisector of
Þ q=
3 line segment joining points A and B.
b 34. (a) Equation of line through P (1, -5,9) and
31. (c) Slope of line L = -
5 parallel to the plane x = y = z is
3 x -1 y + 5 z - 9
Slope of line K = - = = = l ( say )
c 1 1 1
Line L is parallel to line k.
Q = ( x = 1 + l, y = -5 + l, z = 9 + l )
b 3
Þ = Þ bc = 15 Given plane x – y + z =5
5 c \ 1+ l + 5 - l + 9 + l = 5
(13, 32) is a point on L.
Þ l = -10
13 32 32 8 \ Q = (– 9 , – 15, – 1)
\ + =1 Þ =-
5 b b 5
3 \ PQ = (1 + 9) 2 + (15 - 5) 2 + ( 9 + 1) 2
Þ b = -20 Þ c = -
4 = 300 = 10 3
Equation of K : 35. (c) Let feet of perpendicular is
y - 4 x = 3 Þ 4x - y + 3 = 0
Distance between L and K ( 2a, 3a + 2, 4a + 3)
52 - 32 + 3 23 Þ Direction ratio of the ^ line is
= =
17 17 2a - 3,3a + 3, 4a - 8. and
32. (d) If q be the angle between the given line Direction ratio of the line 2, 3, 4 are
and plane, then Þ 2 ( 2α –3) +3 (3α + 3) +4 (4α – 8) =0
1´1+ 2 ´ 2 + l ´ 3
sin q = Þ 29a - 29 = 0
12 + 22 + l 2 . 12 + 22 + 32 Þ a =1
5 + 3l Þ Feet of ^ is (2, 5, 7)
=
14. 5 + l 2 Þ Length ^ is 12 + 6 2 + 42 = 53
(5 + 3l ) 2 36. (a) Given equation of a plane is x – 2y + 2z – 5 = 0
Þ cos q = 1 - So, Equation of parallel plane is given by
14(5 + l 2 ) x – 2y + 2z + d = 0
2 Now, it is given that distance from origin to
\ q = cos -1 1 - (5 + 3l ) the parallel plane is 1.
14(5 + l 2 )
d
\ =1 Þ d = ±3
1 + 2 2 + 22
2
EBD_7764
www.crackjee.xyz
M-186 Mathematics
So equation of required plane
A (1, 3, 4)
x – 2y + 2z ± 3 = 0
x -1 y +1 z -1
37. (c) Given lines are = = 3i + j - 5k
2 3 4
x-3 y-k z P
and = =
1 2 1 3i + j - 5k
r r r r A
Thus, a , b , c and d are given as
r r r (a, b, c)
a ( 1, -1,1 ) , b ( 2, 3, 4 ) , c ( 3, k , 0 ) ; and
r
d ( 1, 2,1) æ a +1 b + 3 c + 4 ö
P =ç , , ÷
These lines will intersect if lines are è 2 2 2 ø
coplanar
r r r r æ 6-l l +8ö
i.e., a - c , b & d are coplanar = ç l + 1, , ÷
è 2 2 ø
r r r r
\ ëé a - c , b , d ûù = 0 6-l l +8
r r \ 2(l + 1) - + +3= 0
Now, a - c = (3 – 1, k + 1, 0 –1) 2 2
= (2, k + 1, –1) 3l + 6 = 0 Þ l = –2
2 k + 1 -1 a = –3, b = 5, c = 2
Þ 2 3 4 =0 x+3 y -5 z - 2
Required line is = =
1 2 1 3 1 -5
41. (c) Given, l + m + n = 0 and l 2 = m2 + n2
Þ 2 (3 – 8) – k + 1 (2 – 4) – 1 (4 – 3) = 0
Now, (–m –n)2 = m2 + n2
Þ 2 (–5) – ( k +1) (–2) – 1 (1) = 0
Þ mn = 0 Þ m = 0 or n = 0
Þ – 10 + 2k + 2 – 1 = 0 Þ k = 9 If m = 0 then l = –n
2 1
38. (c) 2x + y + 2z – 8 = 0 …(Plane 1) We know l 2 + m2 + n2 = 1 Þ n = ±
5 2
2x + y + 2z + =0 …(Plane 2)
2 æ 1 1 ö
i.e. (l1, m1, n1) = ç - , 0, ÷
Distance between Plane 1 and 2 è 2 2ø
5 If n = 0 then l = –m
-8 - l 2 + m2 + n2 = 1 Þ 2m2 = 1
2 -21 7
= = =
22 + 12 + 22 6 2 1
Þ m=±
2
39. (c) Given lines will be coplanar
1 1
Let m = Þl=- and n = 0
-1 1 1 2 2
If 1 1 - k = 0
æ 1 1 ö
k 2 (l2, m2, n2 ) = ç - , , 0÷
è 2 2 ø
Þ –1(1 + 2k) – (1 + k2) + 1(2 – k) = 0
Þ k = 0, –3 1 p
\ cos q = Þ q=
a -1 b - 3 c - 4 2 3
40. (c) = = = l (let) 42. (a) Equation of the plane containing the lines
2 -1 1 2x – 5y + z = 3 and x + y + 4z = 5 is
Þ a = 2l + 1 2x – 5y + z – 3 + l (x + y + 4z – 5) = 0
b=3–l Þ (2 + l) x + (–5 + l) y+ (1 + 4l)z + (–3 – 5l) = 0
c=4+l ...(i)
Since the plane (i) parallel to the given plane x
www.crackjee.xyz
Three Dimensional Geometry M-187
+ 3y + 6z = 1
Þ distance OP = 10 3
2 + l -5 + l 1 + 4l
\ = = 45. (b) Line lies in the plane Þ (3, –2, –4) lie in the
1 3 6 plane
11 Þ 3l – 2m + 4 = 9 or 3l – 2m = 5 ..... (1)
Þ l=- Also, l, m,–1 are dr's of line perpendicular
2
Hence equation of the required plane is to plane and 2, –1, 3 are dr's of line lying in
the plane
æ 11ö æ 11ö æ 44 ö æ 55 ö Þ 2l – m – 3 = 0 or 2l – m = 3 .....(2)
çè 2 - ÷ø x + çè -5 - ÷ø y + çè1 - ÷ø z + çè -3 + ÷ø Solving (1) and (2) we get l = 1 and m = –1
2 2 2 2
=0 Þ l2 + m2 = 2.
46. (c) Equation of line PQ is
Þ (4 - 11)x + (-10 - 11)y + (2 - 44)z + ( -6 + 55) = 0
x -1 y + 2 z - 3
Þ -7x - 21y - 42z + 49 = 0 = =
1 4 5
Þ x + 3y + 6z - 7 = 0
Let F be (l + 1, 4l - 2, 5l + 3)
Þ x + 3y + 6z = 7
43. (b) General point on given line º P(3r + 2, 4r – P(1, –2, 3)
1, 12r + 2)
Point P must satisfy equation of plane
(3r + 2) – (4r – 1) + (12r + 2) = 16
11r + 5 = 16
r=1
P(3 × 1 + 2, 4 × 1 – 1, 12 × 1 + 2) = P(5, 3, 14)
distance between P and (1, 0, 2) F

D = (5 - 1)2 + 32 + (14 - 2)2 = 13


44. (d) Q
P(1, –5, 9) Since F lies on the plane
\ 2 (l + 1) + 3 (4l – 2) – 4 (5l + 3) + 22 = 0
2l + 2 + 12l – 6 – 20l – 12 + 22 = 0
x=y=z Þ –6l + 6 = 0 Þ l = 1
\ F is (2, 2, 8)
PQ = 2 PF = 2 12 + 42 + 52 = 2 42
47. (c) Let the plane be
a (x – 1) + b (y + 1) + c (z + 1) = 0
Q Normal vector
ˆi ˆj kˆ
O 1 -2 3 = 5iˆ + 7ˆj + 3kˆ
2 -1 -1
So plane is 5 (x – 1) + 7 (y + 1) + 3 (z + 1) =
x -1 y + 5 z - 9 0
eqn of PO : = = =l
1 1 1 Þ 5x + 7y + 3z + 5 = 0
Þ x =l+ 1; y =l- 5; z =l+ 9. Distance of point (1, 3, –7) from the plane
Putting these in eqn of plane :- is
l+ 1 -l+ 5 +l+ 9 = 5 5 + 21 - 21 + 5 10
=
Þl= -10 25 + 49 + 9 83
Þ O is (–9, –15, –1)
EBD_7764
www.crackjee.xyz
M-188 Mathematics

Probability 27
1. A problem in mathematics is given to three For the events E = {X is a prime number } and
students A, B, C and their respective probability F = { X < 4}, the P( E È F ) is [2004]
1 1 1 (a) 0.50 (b) 0.77
of solving the problem is , and .
2 3 4 (c) 0.35 (d) 0.87
Probability that the problem is solved is [2002] 6. The mean and the variance of a binomial
distribution are 4 and 2 respectively. Then the
3 1
(a) (b) probability of 2 successes is [2004]
4 2
28 219
2 1 (a) (b)
(c) (d) 256 256
3 3
2. A dice is tossed 5 times. Getting an odd number 128 37
(c) (d)
is considered a success. Then the variance of 256 256
distribution of success is [2002] 7. Three houses are available in a locality. Three
(a) 8/3 (b) 3/8 persons apply for the houses. Each applies for
(c) 4/5 (d) 5/4 one house without consulting others. The
3. The mean and variance of a random variable X probability that all the three apply for the same
having binomial distribution are 4 and 2 house is [2005]
respectively, then P (X = 1) is [2003] 2 1
(a) (b)
1 1 9 9
(a) (b)
4 32 8 7
(c) (d)
1 1 9 9
(c) (d) 8. A random variable X has Poisson distribution
16 8
with mean 2. Then P (X > 1.5) equals [2005]
4
4. The probability that A speaks truth is , while 2
5 (a) 2 (b) 0
3 e
the probability for B is . The probability that 3 3
4 (c) 1 - (d) 2
they contradict each other when asked to speak e2 e
on a fact is [2004] 9. At a telephone enquiry system the number of
4 1 phone calls regarding relevant enquiry follow
(a) (b) Poisson distribution with an average of 5 phone
5 5
calls during 10 minute time intervals. The
7 3 probability that there is at the most one phone
(c) (d)
20 20 call during a 10-minute time period is [2006]
5. A random variable X has the probability
6 5
distribution: (a) (b)
e 6
X: 1 2 3 4 5 6 7 8
5
p(X): 0.2 0.2 0.1 0.1 0.2 0.1 0.1 0.1 6 6
(c) (d)
55 e5
www.crackjee.xyz
Probability M-189
10. Two aeroplanes I and II bomb a target in 2 1
succession. The probabilities of I and II scoring (a) (b)
7 21
a hit correctly are 0.3 and 0.2, respectively. The
second plane will bomb only if the first misses 2 1
(c) (d)
the target. The probability that the target is hit 23 3
by the second plane is [2007] 16. Consider 5 independent Bernoulli’s trials each
(a) 0.2 (b) 0.7 with probability of success p. If the probability
(c) 0.06 (d) 0.14. of at least one failure is greater than or equal to
11. A pair of fair dice is thrown independently three 31
times. The probability of getting a score of , then p lies in the interval [2011]
32
exactly 9 twice is [2007]
(a) 8/729 (b) 8/243 æ 3 11 ù é 1ù
(c) 1/729 (d) 8/9. (a) çè , ú (b) ê0, ú
4 12 û ë 2û
12. It is given that the events A and B are such that
1 1 2
æ 11 ù æ 1 3ù
P ( A) = , P ( A | B ) = an d P ( B | A) = . (c) çè ,1ú (d) çè , ú
12 û 2 4û
4 2 3
Then P(B) is [2008] 17. If C and D are two events such that C Ì D and
P(D) ¹ 0, then the correct statement among
1 1 the following is [2011]
(a) (b)
6 3 (a) P (C | D ) ³ P (C )
2 1 (b) P (C | D ) < P (C )
(c) (d)
3 2
P( D )
13.
æ 1ö
In a binomial distribution B ç n, p = ÷ , if the (c) P (C | D) =
P (C )
è 4ø
probability of at least one success is greater than (d) P (C | D ) = P (C )
18. Let A, B, C, be pairwise independent events
9
or equal to
10
, then n is greater than: [2009] with P (C) > 0 and P ( A Ç B Ç C ) =0. Then

(a)
1 9
(b) log 4 – log 3
(
P Ac Ç B c / C .) [2011RS]
log10 4 + log10 3
( ) ( ) ( )
10 10
c c
(a) P B – P ( B ) (b) P A + P B
c
4 1
P( A ) - P(B ) ( )
(c) (d)
log10 4 – log10 3 log10 4 – log10 3 (d) P A - P ( B )
c c c
(c)
14. One ticket is selected at random from 50 tickets 19. Three numbers are chosen at random without
numbered 00,01,02,...,49. Then the probability replacement from {1,2,3,..8}. The probability
that the sum of the digits on the selected ticket that their minimum is 3, given that their maximum
is 8, given that the product of these digits is is 6, is : [2012]
zero, equals: [2009]
3 1
1 5 (a) (b)
(a) (b) 8 5
7 14
1 1 1 2
(c) (d) (c) (d)
50 14 4 5
15. An urn contains nine balls of which three are 20. A multiple choice examination has 5 questions.
red, four are blue and two are green. Three balls Each question has three alternative answers of
are drawn at random without replacement from which exactly one is correct. The probability
the urn. The probability that the three balls have that a student will get 4 or more correct answers
different colours is [2010] just by guessing is: [2013]
EBD_7764
www.crackjee.xyz
M-190 Mathematics
17 13 22. A box contains 15 green and 10 yellow
(a) 5 (b) balls. If 10 balls are randomly drawn, one-
3 35 by-one, with replacement, then the variance
11 10 of the number of green balls drawn is :
(c) 5 (d) [2014]
3 35
21. Let A and B be two events such that 6 12
(a) (b)
25 5
(
P AÈ B = ) 1
6
( 1
, P A Ç B = and
4
) (c) 6 (d) 4
23. If two different numbers are taken from the
( ) 1
P A = , where A stands for the
4
set (0, 1, 2, 3, ......., 10), then the probability
that their sum as well as absolute difference
complement of the event A. Then the events A are both multiple of 4, is : [2014]
and B are [2014] 7 6
(a) independent but not equally likely. (a) (b)
55 55
(b) independent and equally likely.
(c) mutually exclusive and independent. 12 14
(c) (d)
(d) equally likely but not independent. 55 55

Answer Key
1 2 3 4 5 6 7 8 9 10 11 12 13 14 15
(a) (d) (b) (c) (b) (a) (b) (c) (d) (d) (b) (b) (d) (d) (a)
16 17 18 19 20 21 22 23
(b) (a) (d) (b) (c) (a) (b) (b)

1 1 1 4. (c) A and B will contradict each other if one


1. (a) P(E1) = , P (E2) = and P (E3) = ; speaks truth and other false . So , the
2 3 4 required
_ _ _
4æ 3ö æ 4ö3
P ( E1UE2UE3 ) = 1 - P ( E1 ) P ( E 2 ) P ( E 3 ) Probability = ç1 - ÷ + ç1 - ÷
5è 4ø è 5ø4
æ 1ö æ 1ö æ 1ö
= 1 - ç1 - ÷ ç 1 - ÷ ç1 - ÷ 4 1 1 3 7
è 2 ø è 3ø è 4ø = ´ + ´ =
5 4 5 4 20
1 2 3 3 5. (b) P(E) = P ( 2 or 3 or 5 or 7)
=1 - ´ ´ =
2 3 4 4 = 0.23 + 0.12 + 0.20 + 0.07 = 0.62
2. (d) The event follows binomial distribution with P ( F ) = P (1 or 2 or 3)
n = 5, p = 3/6 = 1/2.
q = 1 – p = 1/2.; \ Variance = npq = 5/4. = 0.15 + 0.23 + 0.12 = 0.50
np = 4 ü 1 1 P ( E Ç F ) = P(2 or 3)
3. (b) npq = 2ý Þ q = 2 , p = 2 , n = 8 = 0.23 + 0.12 = 0.35
þ
7
\ P( EUF ) = P( E ) + P ( F ) - P ( E Ç F )
æ 1 öæ 1 ö = 0.62 + 0.50 - 0.35 = 0.77
P ( X = 1) = 8C1 ç ÷ç ÷
è 2 øè 2 ø 6. (a) mean = np = 4 and variance = npq = 2
1 1 1 1
= 8. 8 = 5 = \p=q= and n = 8
2 2 32 2
www.crackjee.xyz
Probability M-191
6 2 1
æ1ö æ1ö
\ P (2 success) = 8 C2 ç ÷ ç ÷ 12. (b) P(A) = 1/4, P(A/B) = , P(B/A) = 2/3
è2ø è2ø 2
By conditional probability,
28 28 P(A Ç B) = P(A) P(B/A) = P(B)P(A/B)
= =
28 256 1 2 1 1
7. (b) For a particular house being selected Þ ´ = P ( B ) ´ Þ P( B ) =
4 3 2 3
1 13. (d) We have
Probability =
3 9
P (all the persons apply for the same P (x ³ 1) ³
10
house)
9
æ1 1 1ö 1 Þ 1 – P (x = 0) ³
= ç ´ ´ ÷3 = . 10
è 3 3 3ø 9 0 n
8. (c) According to Poission distribution, æ1ö æ3ö 9
Þ 1 - nC0 ç ÷ ç ÷ ³
prob. of getting k successes is è4ø è4ø 10
lk 9 æ 3ö
n
P ( x = k ) = e -l Þ 1- ³ç ÷
k! 10 è 4 ø
P ( x ³ 2) = 1 - P ( x = 0) - P ( x = 1) n
æ 3ö æ 1ö
æ lö Þ çè ÷ø £ çè ÷ø
= 1 - e-l - e-l ç ÷ = 1 - 3 . 4 10
è 1!ø e2 Taking log to the base 3/4, on both sides,
we get
e- m mr
9. (d) P( X = r) = æ3ö æ1ö
r! n log3/4 ç ÷ ³ log3/4 ç ÷
P (at most 1 phone call) 4
è ø è 10 ø
= P ( X £ 1) = P ( X = 0) + P( X = 1) - log10 10
6 Þ n ³ - log 3/4 10 =
= e -5 + 5 ´ e -5 = æ 3ö
log10 ç ÷
e5 è 4ø
10. (d) Given : Probability of aeroplane I, scoring
a target correctly i.e., P(I) = 0.3 probability -1
=
of scoring a target correctly by aeroplane log10 3 - log10 4
II, i.e. P(II) = 0.2 1
Þ n³
\ P( I ) = 1 – 0.3 = 0.7 log10 4 - log10 3
\ The required probability 14. (d) Let A º Sum of the digits is 8
= P ( I Ç II ) = P( I ).P( II ) =0.7× 0.2 = 0.14 B º Product of the digits is 0
11. (b) A pair of fair dice is thrown, the sample Then A = {08, 17, 26, 35, 44}
space S = (1, 1), (1, 2) (1, 3) .... = 36 B = {00, 01, 02, 03, 04, 05, 06, 07, 08, 09, 10,
Possibility of getting 9 are (5, 4), (4, 5), (6, 3), 20, 30, 40,}
(3, 6) A Ç B = { 08 }
\ Possibilityof getting score 9 in a single throw P ( A Ç B ) 150 1
\ P (A/B) = = =
P( B) 14
=
4 1
= 50 14
36 9
\ Probability of getting score 9 exactly twice 15. (a) n( S ) = 9 C3
æ 1ö
2
æ 1ö n( E ) = 3C1 ´ 4 C1 ´ 2 C1
= 3C2 × ç ÷ . ç 1 - ÷ = 3! ´ 1 ´ 1 ´ 8
è 9ø è 9ø 2! 9 9 9 Probability =
3.2! 1 1 8 8 3 ´ 4 ´ 2 24 ´ 3! 24 ´ 6 2
= ´ ´ ´ = = ´ 6! = =
2! 9 9 9 243
9
C3 9! 9 ´8´ 7 7
EBD_7764
www.crackjee.xyz
M-192 Mathematics
31 20. (c) p = p (correct answer), q = p (wrong
16. (b) p (at least one failure) ³ answer)
32
1 2
31 Þ p= , q = , n = 5
Þ 1 – p (no failure) ³ 3 3
32 By using Binomial distribution
5 31 Required probability
Þ 1- p ³ 4 5
32 æ 1ö 2 æ 1ö
1 = 5 C4 ç ÷ · + 5C5 ç ÷
5 è 3ø 3 è 3ø
Þ p £
32 2 1 11
1 = 5· 5 + 5 = 5
Þ p£ 3 3 3
2 21. (a) Given,
But p ³ 0
1 1 5
é 1ù P( A È B ) = Þ P( A È B ) = 1 - =
Hence p lies in the interval ê0, ú . 6 6 6
ë 2û 1 1 3
17. (a) In this case P ( A) = Þ P ( A) = 1 - =
4 4 4
æ C ö P(C Ç D) P (C ) We know,
Pç ÷ = =
èDø P( D ) P ( D) P ( A È B ) = P( A) + P ( B ) - P ( A Ç B)
Where, 0 £ P ( D) £ 1 , hence 5 3 1
Þ = + P( B ) -
æC ö 6 4 4
P ç ÷ ³ P (C ) æ 1ö
èDø çèQ P( A Ç B) = ÷ø
18. (d) P(Ac Ç Bc/C) = 4

((
P Ac Ç B c Ç C ) ) = P (( A È B) Ç C )
c
Þ P( B) =
1
3
P (C ) P (C ) Q P ( A) ¹ P ( B ) so they are not equally likely.
P ((1 - A È B) Ç C) 3 1 1
Also P ( A) ´ P( B ) =
´ =
=
P (C) 4 3 4
= P( A Ç B)
P ((1 - A - B + A Ç B ) Ç C ) So A & B are independent.
= 22. (b) We can apply binomial probability
P (C )
distribution
P(C) – P(A Ç C) – P(B Ç C) + P(A Ç B Ç C) We have n = 10
=
P(C) p = Probability of drawing a green ball =
P(C) – P(A).P(C) – P(B)P(C) + 0 15 3
= =
P(C) 25 5
= 1 - P ( A) - P ( B) 3 2
Also q = 1 - =
( )
= P Ac - P ( B ) 5 5
Variance = npq
19. (b) Given sample space = {1,2,3,.....,8}
Let Event 3 2 12
= 10 × ´ =
A : Maximum of three numbers is 6. 5 5 5
B : Minimum of three numbers is 3.
This is the case of conditional probability 23. (b) Let A º {0, 1, 2, 3, 4, ......., 10}
We have to find P (minimum) is 3 when it n (S) = 11C2 = 55 where 'S' denotes sample
is given that P (maximum) is 6. space
Let E be the given event
æ B ö P ( B Ç A)
2
C \ E º {(0, 4), (0, 8), (2, 6), (2, 10), (4, 8), (6, 10)}
\ Pç ÷ = =5 1
èAø P ( A) C2 Þ n (E) = 6
2 1 n(E) 6
= = \ P(E) = =
10 5 n(S) 55
www.crackjee.xyz
Properties of Triangles M-193

Properties of Triangles 28
1. The sides of a triangle are 3x+4y, 4x+3y and (a) 150° (b) 90°
5x+5y where x, y > 0 then the triangle is [2002] (c) 120° (d) 60°
(a) right angled (b) obtuse angled 7. A person standing on the bank of a river
(c) equilateral (d) none of these observes that the angle of elevation of the top
of a tree on the opposite bank of the river is 60°
2. In a triangle with sides a, b, c, r1 > r2 > r3 and when he retires 40 meters away from the
(which are the ex-radii) then [2002] tree the angle of elevation becomes 30°. The
(a) a >b > c (b) a < b < c breadth of the river is [2004]
(c) a > b and b < c (d) a < b and b > c (a) 60 m (b) 30 m
3. The sum of the radii of inscribed and (c) 40 m (d) 20 m
circumscribed circles for an n sided regular p
polygon of side a , is [2003] 8. In a triangle ABC, let ÐC = . If r is the inradius
2
a æp ö æp ö and R is the circumradius of the triangle ABC,
(a) cot ç ÷ (b) a cot ç ÷ then 2 (r + R) equals [2005]
4 è 2n ø ènø
(a) b + c (b) a + b
a æp ö æp ö (c) a + b + c (d) c + a
(c) cot ç ÷ (d) a cot ç ÷
2 è 2n ø è 2n ø 9. If in a DABC , the altitudes from the vertices A,
4. In a triangle ABC, medians AD and BE are drawn. B, C on opposite sides are in H.P, then sin A, sin
p p B, sin C are in [2005]
If AD = 4, ÐDAB = and ÐABE = , then (a) G. P. (b) A. P.
6 3
the area of the D ABC is [2003] (c) A.P -G.P. (d) H. P
10. A tower stands at the centre of a circular park. A
64 8 and B are two points on the boundary of the
(a) (b)
3 3 park such that AB (= a) subtends an angle of 60°
16 32 at the foot of the tower, and the angle of elevation
(c) (d) of the top of the tower from A and B is 30°. The
3 3 3
height of the tower is [2007]
2æ C ö 2 æ A ö 3b
5. If in a DABC a cos ç ÷ + c cos ç ÷ = , then (a) a/ 3 (b) a 3
è2ø è2ø 2
the sides a, b and c [2003] (c) 2a / 3 (d) 2a 3
(a) satisfy a + b = c (b) are in A.P 11. AB is a vertical pole with B at the ground level
and A at the top. A man finds that the angle of
(c) are in G..P (d) are in H.P. elevation of the point A from a certain point C
6. The sides of a triangle are sin a, cos a and on the ground is 60°. He moves away from the
pole along the line BC to a point D such that
p CD =7 m. From D the angle of elevation of the
1 + sin a cos a for some 0 < a < . Then the
2 point A is 45°. Then the height of the pole is
greatest angle of the triangle is [2004]
EBD_7764
www.crackjee.xyz
M-194 Mathematics
[2008] b+c c+a a+b
14. If in a triangle ABC, = = , then
7 3 1 7 3 11 12 13
(a) m (b) ( 3 + 1)m cosA is equal to [2012]
2 3 –1 2
(a) 5/7 (b) 1/5
7 3 7 3 1 (c) 35/19 (d) 19/35
(c) ( 3 –1)m (d) m
2 2 3 +1 15. A bird is sitting on the top of a vertical pole 20 m
12. For a regular polygon, let r and R be the radii of high and its elevation from a point O on the
the inscribed and the circumscribed circles. A ground is 45° . It flies off horizontally straight
false statement among the following is [2010] away from the point O. After one second, the
r 1 elevation of the bird from O is reduced to 30° .
(a) There is a regular polygon with = Then the speed (in m/s) of the bird is [2014]
R 2

(b) There is a regular polygon with


r 2
=
(a) 20 2 (b) 20 ( 3 -1 )
R 3

(c) There is a regular polygon with


r
=
3 (c) 40 ( 2 -1 ) (d) 40 ( 3- 2 )
R 2 16. Let a vertical tower AB have its end A on the
r 1 level ground. Let C be the mid-point of AB and P
(d) There is a regular polygon with =
R 2 be a point on the ground such that AP = 2AB. If
13. In a DPQR, If 3 sin P + 4 cos Q = 6 and ÐBPC = b, then tan b is equal to : [2017]
4 sin Q + 3 cos P = 1, then the angle R is equal 4 6
to : [2012] (1) (2)
9 7
5p p
(a) (b) 1 2
6 6 (3) (4)
4 9
p 3p
(c) (d)
4 4
Answer Key
1 2 3 4 5 6 7 8 9 10 11 12 13 14 15
(b) (a) (c) (d) (b) (c) (d) (b) (b) (a) (b) (b) (b) (b) (b)
16
(d)

1. (b) Let a = 3x + 4y, b = 4x + 3y and c = 5x + 5y D D D


as x, y > 0, c = 5x + 5y is the largest side 2. (a) r1 > r2 > r3 Þ > > ;
s-a s-b s-c
\ C is the largest angle . Now Þ s – a < s – b < s – c Þ –a < –b < –c Þ
(3 x + 4 y )2 + (4 x + 3 y )3 - (5 x + 5 y ) 2
a>b>c
cos C =
2(3x + 4 y)(4 x + 3 y) æ pö a æ pö a
3. (c) tan ç ÷ = ; sin ç ÷ =
-2 xy è n ø 2r è n ø 2R
= <0
2(3 x + 4 y )(4 x + 3 y ) aé p pù
r+R = cot + cos ec ú
\ C is obtuse angle Þ DABC is obtuse 2 êë n nû
angled
www.crackjee.xyz
Properties of Triangles M-195

\ c is the greatest side and greatest angle is C


a2 + b 2 - c 2
\ cos C =
2ab
2 2
O p sin a + cos a - 1 - sin a cos a 1
= =-
n 2 sin a cos a 2
r \ C = 120°
R

7. (d) T
A N B
a
é p ù é p ù y
cos + 1ú 2cos 2
aê n a ê 2n ú
= ê ú= ê
2ê p ú 2 ê 2sin p p ú
sin cos ú 30° 60°
R
ë n û ë 2n 2n û Q
P 40m x
a p From the figure
= cot
2 2p y
A tan 60° = Þ y = 3 x ....................(1)
4. (d) x
y x + 40
tan 30° = Þy= ...............(2)
30º x + 40 3
From (1) and (2),
E
x + 40
90º 3x = Þ x = 20m
P 3
60º
8. (b) We know that for the circle circumscrib-
B D C ing a right triangle, hypotenutse is the
2 8 4 diameter As ÐC = 90o
AP = AD = ; PD = ; Let PB = x
3 3 3
o 8/3 8 c
tan 60 = or x = \ 2R = c Þ R =
x 3 3 2
1 8 16 1
Area of DABD = ´ 4 ´ = ´ a ´b
2 3 3 3 3 D 2
also r = =
16 32 s a+b+c
\ Area of DABC = 2 ´ =
3 3 3 3 2
[Q Median of a D divides it into two D¢s of ab
equal area.] Þr=
a+b+c
2æ C ö 2 æ A ö 3b
5. (b) If a cos ç ÷ + c cos ç ÷ = 2 ab
è 2ø è2ø 2 \ 2r + 2 R = +c
a[cos C + 1] + c[cos A + 1] = 3b a+b+c
(a + c ) + (a cos C + c cos B) = 3b 2ab + ac + bc + c2 2ab + ac + bc + a 2b 2
= =
a + c + b = 3b or a + c = 2b a+b+c a+b+c
or a, b, c are in A.P. (Q c 2
= a 2 + b2 )
6. (c) Let a = sin a, b = cos a and
( a + b) 2 + ( a + b ) c
c = 1 + sin a cos a = = ( a + b)
a+b+c
Clearly a and b < 1 but c > 1 as sina > 0 and
cosa > 0
EBD_7764
www.crackjee.xyz
M-196 Mathematics

1 1 1 h
9. (b) D= p1a = p2 b = p3b Þx=
2 2 2 3
p1, p2 , p3 , are in H.P.. In D ABD
2D 2D 2D are in H.P.. h
Þ , , = tan45° =1
a b c x+7
1 1 1 h
Þ , , are in H.P.. Þ h=x+7Þh– =7
a b c 3
Þ a, b, c are in A.P. 7 3 3 +1
Þ h= ´
Þ KsinA, K sin B, K sin C are in AP 3 –1 3 +1
Þ sinA, sinB, sinC are in A.P.
10. (a) In the D AOB, Ð AOB = 60°, and Ð OBA = Þ h=
7 3
2
(3 +1 m )
Ð OAB (since OA = OB = AB radius of same 12. (b) If O is centre of polygon and
circle). \ D AOB is a equilateral triangle.
Let the height of tower is h AB is one of the side, then by figure
O
C

p/n
R
p r r
h cos =
n R

A B
30° a
A r 1 1 3
60° O = , ,
a a Þ for n = 3, 4, 6 respectively..
30° R 2 2 2
13. (b) Given 3 sin P + 4cos Q = 6 ...(i)
B 4 sin Q + 3cos P = 1 ...(ii)
m. Given distance between two points A & Squaring and adding (i) & (ii) we get
B lie on boundary of circular park, subtends 9 sin2 P + 16cos2Q + 24 sin P cos Q
an angle of 60° at the foot of the tower is + 16 sin2Q + 9cos2 P + 24 sin Q cos P
AB i.e. AB = a. A tower OC stands at the = 36 + 1 = 37
centre of a circular park. Angle of elevation Þ 9 (sin2P + cos2P) + 16 (sin2 Q + cos2 Q)
of the top of the tower from A and B is 30°. + 24 (sinP cosQ + cosP sinQ) = 37
h 1 h Þ 9 + 16 + 24 sin ( P + Q) = 37
In DOAC, tan 30° = Þ = [Q sin2q + cos2q = 1 and sin A cos B +
a 3 a cos A sin B = sin (A + B)]
a 1
Þ h= Þ sin ( P + Q ) =
3 2
h p 5p
11. (b) In D ABC = tan 60° = 3 Þ P + Q = or
x 6 6
A
Þ R = 5 p or 5 p (Q P + Q + R = p)
6 6
5p p
If R = then 0 < P , Q <
h 6 6
1
Þ cos Q < 1 and sin P <
2
11
Þ 3 sin P + 4 cos Q <
60° 2
45° p
B x So, R =
C 7m D 6
www.crackjee.xyz
Properties of Triangles M-197

14. (b) Letb + c = c + a = a + b = K in a triangleABC. 16. (d) Since AP = 2AB Þ


AB 1
= ...(1)
11 12 13 AP 2
Þ b + c = 11 K, c + a = 12 K, a + b = 13 K Let ÐAPC = a
On solving these, we get
AC 1 AB 1
a = 7K, b = 6K, c = 5 K tan a = = =
Now we know, AP 2 AP 4
1
b2 + c2 - a 2 (Q C is the mid point) (\ AC = AB )
cos A = 2
2bc
1
36 K 2 + 25K 2 - 49 K 2 1 Þ tan a =
= = 4
2 ( 6 K )( 5K ) 5 B
15. (b) Let the speed be y m/sec.
Let AC be the vertical pole of height 20 m.
A B
C

20 b
45° 20 a
A P
30°
O x C y D
tan a + tan b
Let O be the point on the ground such that As tan (a + b) =
1 - tan a tan b
ÐAOC = 45°
Let OC = x, Time t = 1 s tan a + tan b
Þ
20 1 - tan a tan b
From DAOC, tan 45° = ...(i)
x é AB ù
ê Qtan(a + b) = ú
20 1 AP
and from DBOD, tan 30° = ...(ii) = ê ú
x+ y 2ê 1
tan(a + b) = [ From(1) ]ú
From (i) and (ii), we have x = 20 and êë 2 úû
1 20 1
= + tan b
3 x+ y 1 2
Þ 4 = \ tan b =
1 20 1 2 9
= 1 - tan b
Þ Þ 20 + y = 20 3 4
3 20 + y
So, y = 20( 3 - 1)
i.e., speed = 20( 3 - 1) m/s
EBD_7764

S-ar putea să vă placă și